You are on page 1of 850

Histology exam questions

SECTION-1: GENERAL PROPERTIES:


Question: 1 CLASS QUIZ
Which of the following ions has a higher concentration inside the cell than in the extracellular fluid?

Answer Choices:
A Sodium
B Chloride
C Calcium
D Potassium
E Magnesium

D Potassium

A typical mammalian cell maintains a concentration gradient of many substances, including ions
between the extracellular and intracellular fluid. The largest difference is for calcium which
shows a 25,000-fold difference (2.5 mM extracellular compared to 0.1 μM). The difference for
sodium is 14-fold (140 mM extracellular, 10 mM intracellular); potassium is 35-fold (4 mM
extracellular, 140 mM intracellular); magnesium is 4-fold (2 mM extracellular, 0.5 mM
intracellular); chloride is 25-fold (100 mM extracellular, 4 mM intracellular). These gradients are
maintained across the cell membrane by preventing ion flux and by the active transport of ions
from one side of the plasma membrane to the other. The specific transport of molecules across
membranes is mediated by carriers. A carrier is not an enzyme and does not catalyze any
chemical reactions. However, carriers have some properties in common with enzymes in that
they are specific, have dissociation constants, exhibit saturation, and can be blocked by specific
inhibitors.
There are 2 types of transport, passive or facilitated diffusion and active transport. Active
transport involves movement of molecules against a concentration gradient and therefore,
requires energy usually as ATP. Examples include calcium transport, the sodium-potassium
ATPase, and sodium-linked transport of glucose or amino acids. Passive transport involves
movement of molecules down a concentration gradient and includes the movement of glucose
in many cells.

Question: 2 BLOCK 1
The cell is highly organized with numerous vesicles and compartments. The organization of subcellular
vesicles allows for increased specialization of function. The site within the cell responsible for lipid
synthesis is the

Answer Choices:
A Nucleus
B Lysosome
C Rough endoplasmic reticulum
D Golgi apparatus
E Smooth endoplasmic
E Smooth endoplasmic

The smooth endoplasmic reticulum is free of ribosomes and is the site of lipid synthesis.
The nucleus is the site of DNA compartmentalization. The lysosome functions to break down
glycoproteins and glycoaminoglycans under acidic conditions, and the rough endoplasmic
reticulum is the site of protein synthesis with the presence of ribosomes interacting with the
endoplasmic reticulum membrane.
The Golgi apparatus is the membranous region within the cell that fatty acids, sugars,
phosphate, and sulfate are added to newly synthesized proteins. These posttranslational
processes are often markers for subsequent targeting of proteins to different locations within
the cell.

Question: 3 CLASS QUIZ


You examine the micro-cellular structure of a cell type isolated from a tumor from an 83-year-old
man. You note that the cells have an extensive Golgi apparatus, but virtually no rough endoplasmic
reticulum. Which of the following substances is reasonable to be produced by these cells?

Answer Choices:
A Hyaluronic acid
B Immunoglobulin
C Lysozyme
D Amylase
E Angiotensinogen
A Hyaluronic acid

The Golgi apparatus synthesizes carbohydrate polymers such as chondroitin, hyaluronic acid,
sialic acid, and galactose.
Proteins synthesized for extracellular release, such as immunoglobulins, require an extensive
rough endoplasmic reticulum.
Lysozyme is a protein enzyme that is synthesized from extracellular release. This requires the
rough endoplasmic reticulum.
The digestive enzyme, amylase is a protein enzyme that is synthesized from extracellular
release. This requires the rough endoplasmic reticulum.
Angiotensinogen is a protein that is synthesized by the liver for release in general circulation.
Proteins synthesized for extracellular release require an extensive rough endoplasmic
reticulum.
Question: 4
The largest difference in ion concentrations between the intracellular and extracellular fluid is found
for

Answer Choices:
A Calcium
B Potassium
C Chloride
D Sodium
E Magnesium
A Calcium

A typical mammalian cell maintains a concentration gradient of many substances, including ions
between the extracellular and intracellular fluid. The largest difference is for calcium which
shows a 25,000-fold difference (2.5 mM extracellular compared to 0.1 μM). The difference for
sodium is 14-fold (140 mM extracellular, 10 mM intracellular); potassium is 35-fold (4 mM
extracellular, 140 mM intracellular); magnesium is 4-fold (2 mM extracellular, 0.5 mM
intracellular); chloride is 25-fold (100 mM extracellular, 4 mM intracellular). These gradients are
maintained across the cell membrane by preventing ion flux and by the active transport of ions
from one side of the plasma membrane to the other. The specific transport of molecules across
membranes is mediated by carriers. A carrier is not an enzyme and does not catalyze any
chemical reactions. However, carriers have some properties in common with enzymes in that
they are specific, have dissociation constants, exhibit saturation, and can be blocked by specific
inhibitors.
There are 2 types of transport, passive or facilitated diffusion and active transport. Active
transport involves movement of molecules against a concentration gradient and therefore,
requires energy usually as ATP. Examples include calcium transport, the sodium-potassium
ATPase, and sodium-linked transport of glucose or amino acids. Passive transport involves
movement of molecules down a concentration gradient and includes the movement of glucose
in many cells.

Question: 5 CLASS QUIZ


A 23-year-old metal worker presents to your emergency room in a comatose state. Pulse oximetry
indicates that his hemoglobin is 100%. When you draw blood for labs, you notice that the venous
sample is bright red. You suspect cyanide poisoning. As cyanide blocks the cytochrome oxidase system,
you understand that shortly after oxidative phosphorylation stops, glycolysis will stop too. Which of the
following organelles is the major site for anaerobic metabolism?

Answer Choices:
A Mitochondria
B Cytoplasm
C Golgi apparatus
D Nucleolus
E Centrioles
B Cytoplasm

Oxidative phosphorylation occurs in the mitochondria. The TCA cycle occurs in the
mitochondrial matrix. The electron transport chain occurs on the inner mitochondrial
membrane.
Anaerobic metabolism occurs in the cytoplasm of most cells. The mitochondria are the site of
aerobic metabolism.
The Golgi apparatus is responsible for packaging of material for intra- and extracellular use.
The nucleolus is the site with in the nucleus of active transcription a gene to mRNA.
Centrioles are cytoskeletal elements responsible for nuclear division during cell division.

Question: 6

One of the ways that cells communicate with each other is by secretion of various molecules. The
molecules which are secreted for this purpose are known as

Answer Choices:
A Receptor molecules
B Signaling molecules
C Spectrin molecules
D Integrin tetramers
E Anticodons
B Signaling molecules

Cells can communicate with each other by releasing signaling molecules that attach to receptor
molecules on target cells.
Cells communicate with each other via chemical messengers. Within a given tissue, some
chemicals move from cell to cell via gap junctions without entering the ECF. There are three
general types of intercellular communication mediated in this fashion: (i) neural
communication, in which neurotransmitters are released at synaptic junctions from nerve cells
and act across a narrow synaptic cleft on a postsynaptic cell; (ii) endocrine communication, in
which hormones reach cells via the circulating blood; and (iii) paracrine communication, in
which the products of cells diffuse in the ECF to affect neighboring cells that may be some
distance away. It should be noted that in various parts of the body, the same chemical
messenger can function as neurotransmitter, a paracrine mediator, a hormone secreted by
neurons into the blood (neural hormone), and a hormone secreted by gland cells into the
blood.

SECTION 2: BIOLOGICAL MEMBRANES

Question: 1 BLOCK-1
The phospholipid molecules of the cell membranes are arranged

Answer Choices:
A In a single layer with the polar heads toward the extracellular space and the
hydrophobic heads toward the cellular cytoplasm
B In a bilayer with the polar heads outward and the hydrophobic tails facing each
other
C In a single layer with random orientation
D In a single layer with the polar heads toward the cellular cytoplasm and the
hydrophobic tails toward the extracellular space
E In a bilayer with the polar heads facing each other and the hydrophobic tails facing
outward
B In a bilayer with the polar heads outward and the hydrophobic tails facing each other
Image(s) / Chart(s):
Click image to view full size. Click open image to close. Click and hold open image to move.
The cell membrane consists of two layers of phospholipid molecules which are oriented with
their hydrophobic (lipid) tails facing each other and with their polar (phosphate) heads facing
outward, either toward the cytoplasm of the cell or toward the extracellular matrix. (See attached
image)
This provides a relatively fluid membrane in which some of the proteins of the membrane are
able to migrate around the cell surface.
The phospholipid component of the membrane provides the barrier that keeps certain
concentrations of ions, chemicals and water on one side of the membrane and different
concentrations on the other. Bacteria that produce phospholipases, e.g. Clostridium perfringens
(gas gangrene), cause tissue destruction by destroying the phospholipids of cell membranes.

Question: 2 BLOCK-1
The formation of secretory granules containing secretory proteins occurs

Answer Choices:
A In the trans Golgi network
B At the plasma membrane
C In the cytoplasm
D In the lysosomes
E In the absence of clathrin
A In the trans Golgi network

Cells that specialize in secreting their products contain large numbers of secretory granules.
These secretory granules form from clathrin-coated budding from the trans Golgi network.
These secretory granules release their contents to the exterior of the cell by exocytosis in
response to a specific signal. In the process of forming the secretory granules the secretory
products aggregate by an unknown process. These are then taken up by the trans Golgi
network by a process similar to phagocytosis. The immature secretory granules are coated with
the protein clathrin that is rapidly shed as the granule matures. Clathrin is not part of the mature
secretory granule.

Question: 3 CLAS QUIZ


Integrins make up a large family of transmembrane proteins involved in:

Answer Choices:
A Transport of ions across membranes.
B Binding extracellular matrix proteins.
C Binding growth factors.
D Binding immunoglobulins.
E Controlling the fluidity of a biological membrane.
B Binding extracellular matrix proteins.

The integrins make up a large family of homologous transmembrane proteins involved in the
binding of extracellular matrix proteins such as collagen, fibronectin, and laminin. These are
heterophilic interactions since they occur between different types of molecules. The integrins
differ from other hormone receptor molecules in that they have a relatively low affinity for the
ligands and there are many more of them per cell.
Integrins are heterodimers composed of an α and a β chain held together noncovalently. There
are many different α and β chains that combine in various combinations to give binding
specificity. Integrins interact with the cytoskeleton as well and allow communication between the
cytoskeleton and the extracellular matrix across the plasma membrane. The intracellular
domains interact with actin filaments via talin, vinculin, and other proteins. Many factors regulate
the expression of different integrins. Intracellular signaling cascades can be activated by
integrins. The cadherins are proteins that mediate cell-cell interactions by interacting with similar
molecules on adjacent cells (homophilic interactions).

Question: 4 BLOCK-1
Cholesterol levels in serum are often determined as part of blood tests given to patients during routine
physical examinations. While many patients are aware of the fact that they should watch their intake of
cholesterol, many do not realize that cholesterol is a required component of animal cell membranes
and influences membrane fluidity. Cholesterol modulates membrane fluidity in animal cell membranes
by

Answer Choices:
A Preventing the insertion of lipids containing unsaturated fatty acyl chains
B Disrupting the packing of the acyl chains of lipids
C Acting as a hormone receptor
D Binding to membrane proteins
E Restricting the size of membrane proteins in the membrane

Image(s) / Chart(s):
Click image to view full size. Click open image to close. Click and hold open image to move.

B Disrupting the packing of the acyl chains of lipids

Cholesterol (structure shown in image) is the major sterol found in the membranes of animal
cells. It is an amphipathic molecule with the hydroxyl at C-3 acting as a polar head group and
the steroid nucleus and the aliphatic side chain at C-17 serving as the nonpolar hydrocarbon
moiety. The amount of cholesterol found in the membranes of different tissues can vary widely.
Cholesterol serves as a key regulator of membrane fluidity. It prevents the "crystallization" of
the fatty acyl side chains of the phospholipids thereby disrupting their packing.
Cholesterol can also sterically block the large motions of the fatty acyl chains making the
membrane less fluid.

Question: 5 CLASS QUIZ


Which of the following diseases involves a defect in the transport of chloride ions?

Answer Choices:
A Sickle cell anemia
B Cystic fibrosis
C Muscular dystrophy
D Phenylketonuria
E Gaucher's Disease
B Cystic fibrosis

Cystic fibrosis is a genetic disease caused by a mutation in a gene coding for a chloride ion
channel protein called the cystic fibrosis transmembrane regulator (CFTR). This protein is a
member of a large family of carrier proteins referred to as ABC transporters. The ABC
transporters are very important clinically because they contain an ATP-binding cassette as part
of their structure. This family also includes the multidrug resistance (MDR) ATPase found in
mammalian cells and the chloroquine-resistance ATPase in P. falciparum, which causes
malaria. The CFTR requires both ATP hydrolysis and cAMP-dependent phosphorylation in order
to open. Defective Cl- transport occurs in the epithelia of cystic fibrosis patients, including the
sweat ducts and cells lining the respiratory tract. The disease is characterized by abnormally
thick mucus production by cells lining the respiratory and gastrointestinal tracts, leading to
obstruction of pulmonary airways and the promotion of bacterial infections. The multidrug
resistance (MDR) ATPase is responsible for increasing drug efflux from the cell, thereby making
less drug available within the cancer cell to kill it. MDR affects large classes of unrelated drugs,
and the cells become resistant to drugs even if not treated with those drugs. The protist P.
falciparum, which causes malaria, can become resistant to the drug chloroquine because the
ABC transporter gene is amplified and pumps chloroquine out of the cells.

Question: 6 BLOCK-1
Which of the following mechanisms of transport of small molecules is the least selective?

Answer Choices:
A Passive diffusion
B Facilitated diffusion
C Ion channels
D Active transport
E Small ion transport must be highly selective
A Passive diffusion

Passive diffusion occurs when a molecule dissolves in the lipid bilayer and then re-dissolves in
the aqueous solution on the other side. This is a non-selective process and any molecule that
can dissolve in the bilayer can cross the membrane. However, only small, relatively hydrophobic
molecules can dissolve in the membrane and diffuse at a significant rate. These molecules
include carbon dioxide, oxygen, benzene, water, and ethanol. Large charged molecules such as
amino acids and glucose cannot move across membranes by passive diffusion. Neither can
ions such as sodium, protons, chloride, or calcium freely cross phospholipid membranes.

Question: 7
Which of the following molecules would freely cross a phospholipid bilayer membrane?

Answer Choices:
A Glucose
B Glucose 6-phosphate
C Glycine
D Sodium ions
E Oxygen
E Oxygen

Passive diffusion occurs when a molecule dissolves in the lipid bilayer and then re-dissolves in
the aqueous solution on the other side. This is a non-selective process and any molecule that
can dissolve in the bilayer can cross the membrane. However, only small, relatively hydrophobic
molecules can dissolve in the membrane and diffuse at a significant rate. These molecules
include carbon dioxide, oxygen, benzene, water, and ethanol. Large charged molecules such as
amino acids and glucose cannot move across membranes by passive diffusion. Neither can
ions such as sodium, protons, chloride, or calcium freely cross phospholipid membranes.

Question: 8
Cholesterol modulates membrane fluidity in animal cell membranes by

Answer Choices:
A Preventing the insertion of lipids containing unsaturated fatty acyl chains
B Disrupting the packing of the acyl chains of lipids
C Acting as a hormone receptor
D Binding to membrane proteins
E Restricting the size of membrane proteins in the membrane

Image(s) / Chart(s):
Click image to view full size. Click open image to close. Click and hold open image to move.
B Disrupting the packing of the acyl chains of lipids

Cholesterol (structure below) is the major sterol found in the membranes of animal cells. It is an
amphipathic molecule with the hydroxyl at C-3 acting as a polar head group and the steroid
nucleus and the aliphatic side chain at C-17 serving as the nonpolar hydrocarbon moiety. The
amount of cholesterol found in the membranes of different tissues can vary widely. Cholesterol
serves as a key regulator of membrane fluidity. It prevents the "crystallization" of the fatty acyl
side chains of the phospholipids. Cholesterol can also sterically block the large motions of the
fatty acyl chains making the membrane less fluid.

Question: 9 BLOCK-1
Examination of the blood smear of a young patient displays misshapen erythrocytes, and the pathology
report indicates hereditary spherocytosis. Defects in which one of the following proteins causes this
condition?

Answer Choices:
A Signaling molecules
B G proteins
C Spectrin
D Hemoglobin
E Ankyrin
C Spectrin

Hereditary spherocytosis is caused by a defect in spectrin that renders the protein incapable of
binding to band 4.1 protein, thus destabilizing the spectrin-actin complex of the cytoskeleton.
Although defects in hemoglobin (the respiratory protein of erythrocytes)also cause erythrocyte
(red blood cell) anomalies, hereditary spherocytosis is not one of them.
Defects in signaling molecules, and G proteins will affect intercellular communication, while
those in Ankyrin will affect the affixing of the spectrin-actin complex to the transmembrane
proteins of the plasma membrane.
Question: 10
The hormone ACTH (adrenocorticotrophic hormone) travels through the bloodstream, enters
connective tissue spaces, and attaches to specific sites on target-cell membranes. These sites are

Answer Choices:
A Peripheral proteins
B Signaling molecules
C G proteins
D G-protein-linked receptors
D G-protein-linked receptors

G-protein-linked receptors are sites where ACTH and some other signaling molecules attach.
Binding of ACTH to its receptor causes Gs protein to activate adenylate cyclase, setting in
motion the specific response elicited by the hormone.
Peripheral proteins usually function as part of the cytoskeleton or of an intracellular secondary
messenger system, while the signaling molecules (comprising neurotransmitters and hormones)
are used in intercellular communication. G proteins act as signal transducers mediating the
interaction between the receptor and a membrane bound enzyme or ion channel.

Question: 11
On transmission electron microscopy, the cell membrane is seen to be composed
of

Answer Choices:
A A single electron dense layer
B An inner electron dense layer and an outer electron lucent layer
C An inner electron lucent layer and an outer electron dense layer
D Two electron dense layers with an intervening electron lucent layer
E A single electron lucent layer
D Two electron dense layers with an intervening electron lucent layer
Image(s) / Chart(s):
Click image to view full size. Click open image to close. Click and hold open image to move.
The cell membrane consists of three layers (or is trilaminar ) on electron microscopy.
Biochemically, the membrane consists of two layers of phospholipid molecules which are
oriented with their hydrophobic tails facing each other and with their polar heads facing outward,
either toward the cytoplasm of the cell or toward the extracellular matrix. (See attached image)
Proteins and cholesterol molecules are also scattered throughout the cell membrane.

Question: 12 CLASS QUIZ


Coated vesicle endocytosis differs from ordinary endocytosis because coated vesicle endocytosis

Answer Choices:
A Is receptor mediated
B Requires cell membranes with an unusually thick layer of glycocalyx
C Is seen only in the neural system
D Is seen only in the intestinal endothelium
E Is used to pass an electrical impulse along a muscle cell
A Is receptor mediated

Coated vesicle endocytosis is initiated when a receptor in the cell membrane binds to a specific
molecule (or ligand). After binding occurs, small spikes appear, jutting out of the inner
(cytoplasmic) surface of the cell membrane. The portion of the membrane lined by these spikes
invaginates, eventually pinching off from the plasma membrane and forming a spiked vesicle
within the cell cytoplasm.

Question: 13
Glycocalyx (cell coat) consists of

Answer Choices:
A Desmin
B Collagen
C Carbohydrate
D Mucin
E Tubulin
C Carbohydrate

Glycocalyx consists of carbohydrate linked either to the proteins or the phospholipids in the cell
membrane. All cells have a glycocalyx, but it is better developed in some cells than others.
(Intestinal cells have abundant glycocalyx.) Glycocalyx helps regulate the microenvironment
around the cell, facilitating binding and/or absorption of molecules and aiding in interaction
between cells. Some molecules of the glycocalyx are thought to be the antigens that organisms
use to recognize cells which are part of itself and to reject cells of foreign tissue (ABO, HLA
antigens). There is speculation that alterations in the cell coat may play a part in allowing
cancerous cells to metastasize.

Question: 14
The plasma cell membrane consists primarily of
Answer Choices:
A Phospholipid, protein and cholesterol
B Proteoglycan and triglycerides
C Phospholipid only
D Desmin and GFAP
E Cholesterol and protein
A Phospholipid, protein and cholesterol
Image(s) / Chart(s):
Click image to view full size. Click open image to close. Click and hold open image to move.

The cell membrane consists of two layers of phospholipid molecules which are oriented with
their hydrophobic tails facing each other and with their polar heads facing outward, either toward
the cytoplasm of the cell or toward the extracellular matrix. (See attached image) Proteins and
cholesterol molecules are also scattered throughout the cell membrane. Cholesterol makes the
membrane more stable. The proteins have many roles, including acting as ion pumps, as
transducers, as receptors and as channels connecting the extracellular space with the cell's
cytoplasm. Proteins also provide structure.

Question: 15 CLASS QUIZ


An example of a membrane-bound protein that allows for the specific recognition and localized binding
of substances to the outer surface of a plasma membrane is referred to as a

Answer Choices:
A Transducer
B Pump
C Receptor
D Channel
E Coated vesicle
C Receptor
Plasma membranes contain integral proteins which participate in the intrinsic function related to
that particular cell. The primary, secondary and tertiary structures of proteins allow for very
structurally specific "recognition sites" , or receptors. Being extremely specific to structure,
"ligands" such as the protein bind to these sites on the outside of the plasma membrane through
this recognition process. Receptors differ from other integral proteins such as pumps which
serve to "actively" transport specific ions across membranes, or channels, which allow for the
passage of small ions through a membrane by passive diffusion. A transducer may be an
integral protein that is involved in the coupling reaction of a stimulated membrane receptor
protein to activate a cytoplasmic enzyme.

SECTION 3:- LIFE CYCLE OF CELLS

Question:

Cyclins
are

Answer Choices:
A Primarily extracellular proteins that bind to cell membrane receptors to initiate a
cascade of events leading to DNA synthesis
B Proteins found only in malignant cells that appear to be responsible for unrestrained
cell division
C Proteins that bind to intracellular kinases to initiate cell division
D Found only in the G1 phase
E Found only in the G0 phase and appear to suppress DNA replication

C Proteins that bind to intracellular kinases to initiate cell division

Cyclins are a heterogenous group of proteins that are produced late in the G1 phase and in the
G2 phase. They regulate cell division by binding to protein kinases and regulating their activity.
These cyclin-kinase complexes phosphorylate a number of different proteins that initiate DNA
synthesis, initiate mitosis and enable other processes required for cell division.
Cyclins have added clinical importance because a number of oncogenes are believed to act
through interactions with cyclins.

Question: 2
All cells progress through a cell cycle which can vary between 30 minutes for bacteria to 24 hours for
the typical mammalian cell. As part of a research project you want to investigate the molecular
mechanisms governing progression through the mammalian cell cycle in tumor cells. You compare the
cell cycle of a cell line derived from a colon carcinoma with that of a normal epithelial cell. You first
investigate the covalent modifications of cell cycle control proteins. Classes of proteins called cyclins
are required for progression through the cell cycle. The mechanism by which cyclins control the cell
cycle is by
Answer Choices:
A dephosphorylation of downstream target proteins
B direct phosphorylation of downstream target proteins
C modulation of the activity of protein kinases
D initiation of protein synthesis
E direct interaction with DNA polymerase
C modulation of the activity of protein kinases
Image(s) / Chart(s):
Click image to view full size. Click open image to close. Click and hold open image to move.

There are 4 phases in the standard eukaryotic cell cycle. The M (mitosis) phase where cells
divide is followed by the G1 (gap) phase, which is the interval between the completion of mitosis
and the beginning of DNA synthesis (S phase). Cell growth occurs during the G1 phase.
Following S phase is the G2 phase, which is the interval between the end of DNA synthesis and
the beginning of mitosis. Mitotic cyclins bind to cyclin-dependent protein kinase (Cdk) molecules
during G2 and are required for entry into mitosis. The G1 cyclins bind to Cdk molecules during
G1 and are required for entry into the S phase. The Cdk proteins induce downstream
phosphorylation of selected proteins on serine or threonine residues. Cyclins bind to the
Cdk molecules and control their ability to phosphorylate the appropriate target proteins.
Cyclins are regulatory proteins and do not possess catalytic activity by themselves. Cdk activity
is terminated by the degradation of the cyclins.

Question: 3
Most students taking a cell biology course initially do not think about the process of cell death as a
normal part of the development of an organism. However, programmed cell death, apoptosis, does
occur as part of the normal developmental process. The initial step in programmed cell death
(apoptosis) involves

Answer Choices:
A Disruption of the plasma membrane
B Inhibition of DNA replication
C Breakdown of the nuclear membrane
D DNA fragmentation
E Activation of serine proteases
D DNA fragmentation

Apoptosis, or programmed cell death, plays a role in the maintenance of adult tissues and in
embryonic development. Apoptosis is very different from cell death due to injury. There are
specific genes responsible for the regulation and execution of apoptosis and these genes are
conserved from nematodes to humans. The process involves DNA fragmentation and
chromatin condensation. This is followed by fragmentation of the nucleus and then
fragmentation of the cell. The regulators of this process include the bcl-2 family of proteins
believed to be involved in regulating the ICE protease family. Bcl-2 is an integral membrane
protein mostly found in the outer membrane of the mitochondria. Over-expression of bcl-2
prevents apoptosis induced by a wide variety of agents.
The ICE proteases are a family of cysteine proteases that play a key role in apoptosis,
indicating that induction of apoptosis involves proteolytic cleavage of one or more target
proteins. The bcl-2 protein can act as an oncogene if it is overexpressed and appears to
contribute to lymphomas by protecting against apoptosis rather than stimulating cell division.
Abnormal expression of bcl-2 blocks apoptosis and maintains cell survival under conditions that
would normally induce cell death.

Question: 4
A wide variety of signaling pathways exist in different mammalian cell types. Many types of compounds
and toxins exert their effects on the target cell by altering a signal transduction pathway within that cell.
Which of the following compounds catalyzes the ADP-ribosylation of a signaling protein thereby
inhibiting this specific signaling pathway?

Answer Choices:
A tetrodotoxin
B pertussis toxin
C cobrotoxin
D tubocurarine
E epinephrine
B pertussis toxin

Signal transduction pathways are mechanisms by which external signals (chemical, mechanical,
or electrical) are amplified and converted to a cellular response. For example, the binding of a
hormone to its cell surface receptor, generates various signals inside the cell that can result in
changes in many aspects of cell metabolism. Compounds other than hormones, including
neurotransmitters and ions, can generate signals. One major signaling pathway in the cell, that
converts an external signal to an intracellular message, involves the activation of G proteins.
After a ligand binds to its membrane receptor, a G-protein transducer is activated by the
exchange of bound GDP for GTP. This activated G protein then interacts with other proteins to
generate a second messenger within the cell. One pathway is the binding of a G protein to, and
subsequent activation of, phospholipase C. Another pathway is the G-protein activation of
membrane-bound adenylate cyclase, which produces the second messenger cAMP. Pertussis
toxin, from the bacteria Bordetella pertussis, catalyzes the ADP-ribosylation of a Gi protein
preventing GDP displacement by GTP. This blocks the inhibition of adenylate cyclase by Gi,
which produces the symptoms of whooping cough, including hypersensitivity to histamines and
a decrease in blood glucose. Tetrodotoxin, from the puffer fish, blocks neurotransmission by
preventing the opening of Na+ channels. Cobrotoxin, found in snake venom, and tubocurarine,
used as a poison on arrowheads in the Amazon, block the acetylcholine receptor or the opening
of the ion channel. This blocks the signals from the nerves to the muscles leading to paralysis
and death. Signal transduction also occurs in prokaryotes. For example, the methyl accepting
chemotaxis protein in bacteria is part of a system that lets bacteria sense their environment. The
bacteria respond by swimming towards an attractant, such as a sugar, or away from a chemical
repellent.

Question: 5
All cells progress through a cell cycle which can vary between 30 minutes for bacteria to 24 hours for
the typical mammalian cell. As part of a research project you want to investigate the molecular
mechanisms governing progression through the mammalian cell cycle in tumor cells. You compare the
cell cycle of a cell line derived from a colon carcinoma with that of a normal epithelial cell. You first
investigate the covalent modifications of cell cycle control proteins. The image shows a 2-dimentional-
gel immunostained indicating the presence of a specific protein from the colon carcinoma cell (labeled
'cc') and the normal epithelial cell (labeled 'epi'). Suppose both proteins (previously found to contain
phosphoserine) were treated with a serine phosphatase before being run under the same gel system
resulting in the pattern in gel #2 of the image. The most likely conclusion is that the untreated proteins
have

Answer Choices:
A The same molecular weights but different charges
B Different molecular weights but the same charge
C Different molecular weights and different charges
D The same molecular weights and the same charge
E Different disulfide bond patterns

Image(s) / Chart(s):
Click image to view full size. Click open image to close. Click and hold open image to move.
A The same molecular weights but different charges
Image(s) / Chart(s):
Click image to view full size. Click open image to close. Click and hold open image to move.

The gels indicate that the 2 proteins have the same molecular weight but differ in charge in the
normal vs carcinoma cells. In gel #1 in the image, the 2 proteins migrate at the same molecular
weight. However, the 'cc' protein is more acidic (pI closer to 3). This increased negative charge
could be due to an increase in the phosphorylation of the 'cc' protein compared to the 'epi'
protein. This increased phosphorylation would introduce additional negative charges on the 'cc'
protein resulting in a lower pI. 2-dimensional gel electrophoresis separates proteins first based
on their isoelectric point and then based on their molecular weight using SDS-polyacrylamide
gel electrophoresis in the second dimension. The spots migrate to the same molecular weight
so the ‘epi’ protein is not larger or smaller in molecular weight. The addition of several
glucose molecules or a lipid molecule would alter the molecular weight of the proteins.

There are 4 phases in the standard eukaryotic cell cycle as shown in the image. The M (mitosis)
phase where cells divide is followed by the G1 (gap) phase, which is the interval between the
completion of mitosis and the beginning of DNA synthesis (S phase). Cell growth occurs during
the G1 phase. Following S phase is the G2 phase, which is the interval between the end of DNA
synthesis and the beginning of mitosis. Mitotic cyclins bind to cyclin-dependent protein kinase
(Cdk) molecules during G2 and are required for entry into mitosis. The G1 cyclins bind to Cdk
molecules during G1 and are required for entry into the S phase. The Cdk proteins induce
downstream phosphorylation of selected proteins on serine or threonine residues. Cyclins bind
to the Cdk molecules and control their ability to phosphorylate the appropriate target proteins.
Cyclins are regulatory proteins and do not possess catalytic activity by themselves. Cdk activity
is terminated by the degradation of the cyclins.

Question: 6
All cells progress through a cell cycle which can vary between 30 minutes for bacteria to 24 hours for
the typical mammalian cell. As part of a research project you want to investigate the molecular
mechanisms governing progression through the mammalian cell cycle in tumor cells. You compare the
cell cycle of a cell line derived from a colon carcinoma with that of a normal epithelial cell. You first
investigate the covalent modifications of cell cycle control proteins. The image shows a 2-dimentional-
gel immunostained (gel #1 in the image) indicating the presence of a specific protein from the colon
carcinoma cell (labeled 'cc') and the normal epithelial cell (labeled 'epi'). Based on the positions of the
proteins in gel #1, what is the most likely description of the mutant "cc" protein?

Answer Choices:
A A larger molecular weight that the 'epi' protein
B More phosphate groups than the 'epi' protein
C A lower molecular weight than the 'epi' protein
D Several glucose molecules attached
E A covalently bound lipid molecule

Image(s) / Chart(s):
Click image to view full size. Click open image to close. Click and hold open image to move.
B More phosphate groups than the 'epi' protein
Image(s) / Chart(s):
Click image to view full size. Click open image to close. Click and hold open image to move.

The gels indicate that the 2 proteins have the same molecular weight but differ in charge in the
normal vs carcinoma cells. In gel #1 in the image, the 2 proteins migrate at the same molecular
weight. However, the 'cc' protein is more acidic (pI closer to 3). This increased negative charge
could be due to an increase in the phosphorylation of the 'cc' protein compared to the 'epi'
protein. This increased phosphorylation would introduce additional negative charges on the 'cc'
protein resulting in a lower pI. 2-dimensional gel electrophoresis separates proteins first based
on their isoelectric point and then based on their molecular weight using SDS-polyacrylamide
gel electrophoresis in the second dimension. The spots migrate to the same molecular weight
so the ‘epi’ protein is not larger or smaller in molecular weight. The addition of several
glucose molecules or a lipid molecule would alter the molecular weight of the proteins.
There are 4 phases in the standard eukaryotic cell cycle as shown in the image. The M (mitosis)
phase where cells divide is followed by the G1 (gap) phase, which is the interval between the
completion of mitosis and the beginning of DNA synthesis (S phase). Cell growth occurs during
the G1 phase. Following S phase is the G2 phase, which is the interval between the end of DNA
synthesis and the beginning of mitosis. Mitotic cyclins bind to cyclin-dependent protein kinase
(Cdk) molecules during G2 and are required for entry into mitosis. The G1 cyclins bind to Cdk
molecules during G1 and are required for entry into the S phase. The Cdk proteins induce
downstream phosphorylation of selected proteins on serine or threonine residues. Cyclins bind
to the Cdk molecules and control their ability to phosphorylate the appropriate target proteins.
Cyclins are regulatory proteins and do not possess catalytic activity by themselves. Cdk activity
is terminated by the degradation of the cyclins.

Question: 7
All cells progress through a cell cycle which can vary between 30 minutes for bacteria to 24 hours for
the typical mammalian cell. As part of a research project you want to investigate the molecular
mechanisms governing progression through the mammalian cell cycle in tumor cells. You compare the
cell cycle of a cell line derived from a colon carcinoma with that of a normal epithelial cell. You know
that protein kinases are involved in cell cycle regulation. The function of the cyclin-dependent protein
kinases is to

Answer Choices:
A target cyclins for degradation
B phosphorylate all nuclear proteins
C induce downstream phosphorylation of target proteins
D allow the cells to leave mitosis
E dephosphorylate downstream target proteins
C induce downstream phosphorylation of target proteins

There are 4 phases in the standard eukaryotic cell cycle. The M (mitosis) phase where cells
divide is followed by the G1 (gap) phase, which is the interval between the completion of mitosis
and the beginning of DNA synthesis (S phase). Cell growth occurs during the G1 phase.
Following S phase is the G2 phase, which is the interval between the end of DNA synthesis and
the beginning of mitosis. Mitotic cyclins bind to cyclin-dependent protein kinase (Cdk) molecules
during G2 and are required for entry into mitosis. The G1 cyclins bind to Cdk molecules during
G1 and are required for entry into the S phase. The Cdk proteins induce downstream
phosphorylation of selected proteins on serine or threonine residues. Cyclins bind to the
Cdk molecules and control their ability to phosphorylate the appropriate target proteins.
Cyclins are regulatory proteins and do not possess catalytic activity by themselves. Cdk activity
is terminated by the degradation of the cyclins.

Question: 8
All cells progress through a cell cycle which can vary between 30 minutes for bacteria to 24 hours for
the typical mammalian cell. As part of a research project you want to investigate the molecular
mechanisms governing progression through the mammalian cell cycle in tumor cells. You compare the
cell cycle of a cell line derived from a colon carcinoma with that of a normal epithelial cell. One of the
key covalent modifications which occurs in cells to regulate the cell cycle is
Answer Choices:
A glycosylation of membrane proteins
B cleavage of localization signals allowing replication inhibitors to leave the nucleus
C methylation of DNA
D phosphorylation on tyrosine residues on cytoplasmic proteins
E phosphorylation of serine residues on downstream target proteins
E phosphorylation of serine residues on downstream target proteins

There are 4 phases in the standard eukaryotic cell cycle. The M (mitosis) phase where cells
divide is followed by the G1 (gap) phase, which is the interval between the completion of mitosis
and the beginning of DNA synthesis (S phase). Cell growth occurs during the G1 phase.
Following S phase is the G2 phase, which is the interval between the end of DNA synthesis and
the beginning of mitosis. Mitotic cyclins bind to cyclin-dependent protein kinase (Cdk) molecules
during G2 and are required for entry into mitosis. The G1 cyclins bind to Cdk molecules during
G1 and are required for entry into the S phase. The Cdk proteins induce downstream
phosphorylation of selected proteins on serine or threonine residues. Cyclins bind to the
Cdk molecules and control their ability to phosphorylate the appropriate target proteins.
Cyclins are regulatory proteins and do not possess catalytic activity by themselves. Cdk activity
is terminated by the degradation of the cyclins.

Question: 9
Most students taking a cell biology course initially do not think about the process of cell death as a
normal part of the development of an organism. However, programmed cell death, apoptosis, does
occur as part of the normal developmental process. A key protein (bcl-2) involved in apoptosis is found
predominantly in which subcellular compartment?

Answer Choices:
A Mitochondria
B Nucleus
C Golgi apparatus
D Lysosomes
E Plasma membrane
A Mitochondria

Apoptosis, or programmed cell death, plays a role in the maintenance of adult tissues and in
embryonic development. Apoptosis is very different from cell death due to injury. There are
specific genes responsible for the regulation and execution of apoptosis and these genes are
conserved from nematodes to humans. The process involves DNA fragmentation and
chromatin condensation. This is followed by fragmentation of the nucleus and then
fragmentation of the cell. The regulators of this process include the bcl-2 family of proteins
believed to be involved in regulating the ICE protease family. Bcl-2 is an integral membrane
protein mostly found in the outer membrane of the mitochondria. Over-expression of bcl-2
prevents apoptosis induced by a wide variety of agents.
The ICE proteases are a family of cysteine proteases that play a key role in apoptosis,
indicating that induction of apoptosis involves proteolytic cleavage of one or more target
proteins. The bcl-2 protein can act as an oncogene if it is overexpressed and appears to
contribute to lymphomas by protecting against apoptosis rather than stimulating cell division.
Abnormal expression of bcl-2 blocks apoptosis and maintains cell survival under conditions that
would normally induce cell death.

Question: 10
Most students taking a cell biology course initially do not think about the process of cell death as a
normal part of the development of an organism. However, programmed cell death, apoptosis, does
occur as part of the normal developmental process. Errors in the apoptosis pathway can lead to tumor
development. The over-expression of a protein that normally inhibits programmed cell death can lead
to tumor development by

Answer Choices:
A Acting as a potent growth factor
B Preventing normal cell death
C Inhibiting the proteolytic activity required for cell death
D Activating receptors for growth factors
E Stimulating DNA fragmentation
B Preventing normal cell death

Apoptosis, or programmed cell death, plays a role in the maintenance of adult tissues and in
embryonic development. Apoptosis is very different from cell death due to injury. There are
specific genes responsible for the regulation and execution of apoptosis and these genes are
conserved from nematodes to humans. The process involves DNA fragmentation and
chromatin condensation. This is followed by fragmentation of the nucleus and then
fragmentation of the cell. The regulators of this process include the bcl-2 family of proteins
believed to be involved in regulating the ICE protease family. Bcl-2 is an integral membrane
protein mostly found in the outer membrane of the mitochondria. Over-expression of bcl-2
prevents apoptosis induced by a wide variety of agents.
The ICE proteases are a family of cysteine proteases that play a key role in apoptosis,
indicating that induction of apoptosis involves proteolytic cleavage of one or more target
proteins. The bcl-2 protein can act as an oncogene if it is overexpressed and appears to
contribute to lymphomas by protecting against apoptosis rather than stimulating cell division.
Abnormal expression of bcl-2 blocks apoptosis and maintains cell survival under conditions that
would normally induce cell death.

Question: 11
The initial step in programmed cell death (apoptosis) involves

Answer Choices:
A Disruption of the plasma membrane
B Inhibition of DNA replication
C Breakdown of the nuclear membrane
D DNA fragmentation
E Activation of serine proteases
D DNA fragmentation

Apoptosis, or programmed cell death, plays a role in the maintenance of adult tissues and in
embryonic development. Apoptosis is very different from cell death due to injury. There are
specific genes responsible for the regulation and execution of this process, and these genes are
conserved from nematodes to humans. The process involves DNA fragmentation and chromatin
condensation. This is followed by fragmentation of the nucleus and then fragmentation of the
cell. The regulators of this process include the bcl-2 family of proteins believed to be involved in
regulating the ICE protease family. Bcl-2 is an integral membrane protein mostly found in
the outer membrane of the mitochondria. Overexpression of bcl-2 prevents apoptosis
induced by a wide variety of agents.
The ICE proteases are a family of cysteine proteases that play a key role in apoptosis,
indicating that induction of apoptosis involves proteolytic cleavage of one or more target
proteins. The bcl-2 protein can act as an oncogene if it is overexpressed and appears to
contribute to lymphomas by protecting against apoptosis rather than stimulating cell division.
Abnormal expression of bcl-2 blocks apoptosis and maintains cell survival under conditions that
would normally induce cell death.

Question: 12
A key protein (bcl-2), involved in apoptosis, is found predominantly in which subcellular compartment?

Answer Choices:
A Mitochondria
B Nucleus
C Golgi apparatus
D Lysosomes
E Plasma membrane

A Mitochondria

Apoptosis, or programmed cell death, plays a role in the maintenance of adult tissues and in
embryonic development. Apoptosis is very different from cell death due to injury. There are
specific genes responsible for the regulation and execution of this process, and these genes are
conserved from nematodes to humans. The process involves DNA fragmentation and chromatin
condensation. This is followed by fragmentation of the nucleus and then fragmentation of the
cell. The regulators of this process include the bcl-2 family of proteins believed to be involved in
regulating the ICE protease family. Bcl-2 is an integral membrane protein mostly found in
the outer membrane of the mitochondria. Overexpression of bcl-2 prevents apoptosis
induced by a wide variety of agents.
The ICE proteases are a family of cysteine proteases that play a key role in apoptosis,
indicating that induction of apoptosis involves proteolytic cleavage of one or more target
proteins. The bcl-2 protein can act as an oncogene if it is overexpressed and appears to
contribute to lymphomas by protecting against apoptosis rather than stimulating cell division.
Abnormal expression of bcl-2 blocks apoptosis and maintains cell survival under conditions that
would normally induce cell death.

Question: 13
Overexpression of a protein which normally induces programmed cell death can lead to tumor
development by

Answer Choices:
A Acting as a potent growth factor
B Preventing normal cell death
C Inhibiting the proteolytic activity required for cell death
D Activating receptors for growth factors
E Stimulating DNA fragmentation
B Preventing normal cell death

Apoptosis, or programmed cell death, plays a role in the maintenance of adult tissues and in
embryonic development. Apoptosis is very different from cell death due to injury. There are
specific genes responsible for the regulation and execution of this process, and these genes are
conserved from nematodes to humans. The process involves DNA fragmentation and chromatin
condensation. This is followed by fragmentation of the nucleus and then fragmentation of the
cell. The regulators of this process include the bcl-2 family of proteins believed to be involved in
regulating the ICE protease family. Bcl-2 is an integral membrane protein mostly found in
the outer membrane of the mitochondria. Overexpression of bcl-2 prevents apoptosis
induced by a wide variety of agents.
The ICE proteases are a family of cysteine proteases that play a key role in apoptosis,
indicating that induction of apoptosis involves proteolytic cleavage of one or more target
proteins. The bcl-2 protein can act as an oncogene if it is overexpressed and appears to
contribute to lymphomas by protecting against apoptosis rather than stimulating cell division.
Abnormal expression of bcl-2 blocks apoptosis and maintains cell survival under conditions that
would normally induce cell death.

Question: 14 CLASS QUIZ


A 30-year-old man with a very high blood cholesterol level (290) has been diagnosed with premature
atherosclerosis. His father and grandfather both died of heart attack at age 43 and 45, respectively,
and his mother, 50 years of age, has coronary artery disease. Which of the following is the most likely
explanation of his condition?

Answer Choices:
A He has a lysosomal storage disease and cannot digest cholesterol
B He suffers from a peroxisomal disorder and produces low levels of hydrogen peroxide
C The smooth endoplasmic reticulum (SER) in his hepatocytes has proliferated and
produced excessive amounts of cholesterol
D He has a genetic disorder and synthesizes defective LDL receptors
E He is unable to manufacture endosomes

D He has a genetic disorder and synthesizes defective LDL receptors

Cells import cholesterol by the receptor-mediated uptake of low-density lipoproteins (LDLs)in


coated vesicles. Certain individuals inherit defective genes and cannot make LDL receptors, or
they make defective ones that cannot bind to clathrin-coated pits. The result is an inability to
internalize LDLs, which build to high levels in the bloodstream, predisposing the individual to
premature atherosclerosis and the risk of heart attack.
Lysosomal storage diseases, exemplified by Tay-Sachs disease, glycogen storage disease and
Hurler syndrome) are hereditary conditions in which the synthesis of specific lysosomal acid
hydrolases is impaired. They are all characterized by the accumulation of glycogen (and
glycogen compounds) in many tissues, including the nervous system, liver and muscles.
Peroxisomes (= microbodies) are small (0.5 μm diameter), membrane-limited spherical bodies
that contain oxidative enzymes, particularly catalase and other peroxidases that break down
hydrogen peroxide (H2O2). The smooth endoplasmic reticulum (SER) functions in steroid
hormone synthesis, drug detoxification, as well as in muscle contraction and relaxation (release
and recapture of Ca2+ ions by the sarcoplasmic reticulum in the skeletal muscles), while
endosomes (both early and late) form multivesicular bodies, phagolysosomes and
autophagolysosomes (types of lysosomes). The inability to manufacture endosomes will have
an adverse effect on the synthesis of specific lysosomal acid hydrolases. None of these
conditions will have effect of the synthesis of LDLs.

Question: 15
The enzymes involved in the chain of reactions that trigger DNA synthesis in a cell in the G 0 or G1 state
are

Answer Choices:
A Aryl sulfatases
B Phenylalanine hydroxylases
C Adenosine deaminases
D Tyrosine kinases
E Antitrypsins
D Tyrosine kinases

Cells are induced to begin DNA synthesis and cell division by polypeptide growth factors which
attach to receptors on the cell membrane. This attachment activates tyrosine kinases which in
turn start a cascade of phosphorylation reactions culminating in DNA synthesis.

Question: 16
During which phase of the cell cycle can the chromosomes be labeled with radioactive markers such
as tritiated thymidine?

Answer Choices:
A G1
B S
C G2
D M
E Prophase
B S

During the S phase, when DNA is being synthesized, radioactively labeled bases will be
incorporated into the newly forming DNA.

Question: 17
Which phase of the cell cycle is characterized by a tetraploid DNA content throughout the entire
phase?

Answer Choices:
A G1
B S
C G2
D M
E DNA synthesis phase

C G2

During the S phase, the diploid DNA complement is gradually doubled as DNA synthesis
proceeds. (The S phase is the same as the DNA synthesis phase.) When the S phase is over
and the tetraploid complement of DNA is established, the cell moves into the G2 phase for a
relatively short time prior to mitosis. During the G2 phase the cellular DNA content is tetraploid.
Following G2 the cell goes into mitosis, when the DNA content is again halved.

Question: 18
In which phase of the cell cycle (G1, S, G2, M) do the non-dividing cells of mature organs rest?

Answer Choices:
A G1
B S
C G2
D M
E DNA synthesis phase
A G1

Cells of mature organs which no longer have the need to divide come to rest at the end of the
G1 phase, before DNA synthesis is to begin. Some authors refer to a restriction point at which
normal cells are prevented from growing indefinitely. Other authors refer to a G 0 stage, in which
cells no longer go through the cell cycle.

Question: 19
This attached photomicroscopic image demonstrates an epithelium containing dividing cells throughout
the field. Based upon their ability to regenerate, the somatic cells in this epithelium may be classified as
what type of cell population?

Answer Choices:
A Static cell population
B Stable cell population
C Rapidly renewing cell population
D Apoptotic cell population

Image(s) / Chart(s):
Click image to view full size. Click open image to close. Click and hold open image to move.

C Rapidly renewing cell population

The ability of somatic cells in various tissues to regenerate may be classified according to their
mitotic activity. Static cell populations are cells that do not regularly divide in the adult organism,
such as neurons. Stable cell populations are cell populations that divide episodically to maintain
tissue or organ structure, such as cells associated with bone or cartilage. Rapidly renewing cell
populations are those cell groups that readily divide, and mitotic figures are easily observed.
Examples of this would include blood cells in the marrow cavities or epithelial cells of the
gastrointestinal tract, as shown in this image. An apoptotic cell population refers to a cell or
group of cells that are dying through programmed cell death in which the nuclear chromatin
condenses and then fragments along with the cytoplasm, allowing for phagocytosis by adjacent
cells.

Case #301499:
A 55-year-old man complains of a severe burning pain at the base of his big toe that awoke
him in the middle of the night. The evening before the attack, he drank too much. He had
almost the same symptoms a few years ago and remembers that his father had similar
symptoms as well. On physical examination, you find his metatarsal-phalangeal joint at the
base of his big toe warm, swollen, red, and extremely tender; the patient cannot tolerate even
the slightest touch. You think that he has gout and advise him to take colchicine.

Question: 2- BLOCK 2
What process is impaired by this treatment option?

Answer Choices:
A Oxidative phosphorylation
B Glycolysis
C Endocytosis
D Mitosis
E Apoptosis
D Mitosis

Your patient has the classical symptom of gout: recurrent attacks of acute inflammatory arthritis
after alcohol consumption. However, gout may also present as tophi, kidney stones, or urate
nephropathy. Gout is caused by elevated uric acid in the blood that crystallizes and is deposited
in joints, tendons, and/or the surrounding tissues. Anti-inflammatory drugs, colchicin, and
lifestyle changes can improve symptoms. Colchicine inhibits microtubule polymerization by
binding to tubulin that is one of the main constituents of microtubules. If microtubules cannot be
formed, chromosomes cannot be separated after they are duplicated during S phase of
interphase. Availability of tubulin is essential to mitosis; therefore, colchicine effectively functions
as a "mitotic poison" or spindle poison. Colchicine is used in laboratory to arrest cells in
metaphase so that the chromosomes can be studied when they are at their most visible state.

Oxidative phosphorylation is a metabolic pathway that uses energy released by the oxidation of
nutrients to produce adenosine triphosphate. It is not influenced by colchicine.

Glycolysis is a metabolic pathway that converts glucose into piruvate to release free energy for
producing adenosine triphosphate and reduced nicotinamide adenine nucleotide. Colchicine
does not impaire glycolysis.

Endocytosis is the process by which cells absorb molecules by engulfing them. It is used by all
cells of the body for the substances that cannot pass through the cell membrane. Colchicine
does not influence endocytosis.

Apoptosis is the process of programmed cell death that occurs in multicellular organisms by
specific biochemical reactions and with characteristic morphological changes. It is not
influenced by colchicin.

References:
Nuki G. Colchicine: its mechanism of action and efficacy in crystal-induced inflammation. Curr
Rheumatol Rep. 2008 Jul;10(3):218-27.
King MF: The medical biochemistry page: http://themedicalbiochemistrypage.org/

Keywords: Cell cycle

SECTION 4: THE CYTOPLASM:

Question: 1
When the space between the two mitochondrial membranes increases to the point that it occupies
more than 1/2 the volume of the organelle, the cristae are no longer distinct and the mitochondrial
matrix is greatly reduced in volume, what can be said about the organelle?

Answer Choices:
A It has been damaged, but may recover
B It has been damaged irreparably
C It is engaging in high level oxidative phosphorylation
D The patient has muscular dystrophy
E It is engaging in low level oxidative phosphorylation
C It is engaging in high level oxidative phosphorylation

Changes in the morphology of mitochondria indicate that changes in their functional state are
taking place. In high level oxidative phosphorylation the space between the inner and outer
mitochondrial membranes increases to at least 50% of the organelle's volume, the cristae
become indistinct and the mitochondrial matrix is reduced in volume.
In low level oxidative phosphorylation the mitochondria have a more usual appearance - they
have prominent cristae with an abundance of mitochondrial matrix.

Question: 2
While actin and myosin are microfilaments that participate in muscle cell contraction, the
intermediate filament found in muscle cells is:

Answer Choices:
A Laminin
B Neurofilament
C Cytokeratin
D Desmin
E Spectrin
D Desmin

Intermediate filaments are involved in cellular support and structure. There are several
different classes of intermediate filaments based upon their amino acid sequence, molecular
weight and cellular location. Desmin is an intermediate filament limited to muscle cells. Laminin
is a protein found as a component of the basement membrane. Spectrin is found in red blood
cells. Cytokeratin is an intermediate filament found in epithelial cells. Neurofilaments are
intermediate filaments located within neuronal cells.

Question: 3
The Golgi apparatus is very well developed in

Answer Choices:
A Poorly differentiated, malignant cells
B Cells which produce steroid hormones e.g. adrenal cortical cells
C Cells which produce abundant lipid
D Cells that need to produce large amounts of plasma membrane, e.g. nerve cells
D Cells that need to produce large amounts of plasma membrane, e.g. nerve cells

The Golgi apparatus is primarily involved in producing complex proteins to be secreted outside
the cell and proteins that become part of the cell membrane. Poorly differentiated tumor cells
are usually characterized by a poorly developed Golgi apparatus.
The smooth endoplasmic reticulum is responsible for the synthesis of steroids and lipids.

Question: 4
Alkaloid chemotherapeutic agents like vincristine and colchicine cause cell death by

Answer Choices:
A Destroying the integrity of the cell membrane
B Uncoupling the electron transport chain from the production of ATP
C Interfering with the polymerization of tubulin
D Preventing the transcription of messenger RNA
E Leaking lysosomal enzymes into the cell's cytoplasm
C Interfering with the polymerization of tubulin

Alkaloid chemotherapeutic agents like vincristine, vinblastine, and colchicine prevent the
polymerization of tubulin to form microtubules. Without microtubules, the mitotic spindle
cannot form and the cell cannot divide.
Calcium can also inhibit the polymerization of tubulin.

Question: 5
Intermediate filaments differ from microfilaments and microtubules because

Answer Choices:
A Intermediate filaments form the terminal web
B Intermediate filaments are permanent structures
C Intermediate filaments form basal bodies
D Intermediate filaments are the primary means of cellular locomotion
E Intermediate filaments are found only in the extracellular matrix
B Intermediate filaments are permanent structures

Unlike microfilaments and microtubules, which can dissolve and re-form over a matter of hours
in response to changing cytoplasmic conditions, intermediate filaments appear to be
permanent structures. For this reason the primary purpose of intermediate filaments is thought
to be to provide structural support for the cell.
The ends of certain intermediate filaments (keratin, vimentin, and desmin) are specially
designed to attach to various structures within the cell and to transmembrane proteins, which
in turn have attachments to other cells and to molecules in the extracellular matrix. This
arrangement gives cells a mechanism for interacting with other cells and with the extracellular
environment.
Microfilaments form the terminal web and provide a means of locomotion for the cell.
Microtubules form basal bodies and are instrumental in cell migration.

Question: 6
Which one of the following type of cytoplasmic intermediate filaments provide structural support to
astrocytes, oligodendrocytes and Schwann cells?

Answer Choices:
A Keratin
B Tonofilaments
C Glial filaments
D Neurofilaments
E Lamins (A, B and C)
C Glial filaments

Glial filaments are a type of intermediate filaments, composed of glial fibrillary acidic protein
(GFAP), present in fibrous astrocytes. These filaments are supportive, but they may also play
additional role in both normal and pathologic processes in the central nervous system.
Keratin (19 distinct forms; acidic, neutral and basic) (A) provides structural support or have
tension-bearing role in epithelial cells. As such, they are used as markers for tumors of
epithelial origin. Tonofilaments (B) are associated with desmosome-hemidesmosomes.
Neurofilaments (C) provide support for axons and dendrites, as well as facilitate gel state of the
cytosol in neurons. Lamins (A, B, and C)(E) are present in the nuclear lamina of all cells, and they
are responsible for the organization of the nuclear envelope and perinuclear chromatin.

Question: 7
Which one of the following cytoplasmic organelles divides by fission?

Answer Choices:
A Centriole
B Golgi complex
C Peroxisome
D Rough endoplasmic reticulum
E Smooth endoplasmic reticulum
C Peroxisome

A Peroxisome originates from preexisting peroxisomes. It imports specific cytosolic proteins and
then undergoes fission. The other organelle that divides by fission is the mitochondrion.
A centriole self-duplicates itself in the S phase of the cell cycle, as each parent centriole forms a
procentriole at right angle to itself. The Golgi complex (= apparatus), as well as smooth and
rough endoplasmic reticula are the structural components of the cell.

Question:8
The clathrin-coated vesicles are involved in

Answer Choices:
A Movement of protein from the rough endoplasmic reticulum to the cis face of the
Golgi apparatus
B Retrieval of secretion-granule membrane after exocytosis
C Movement of glycoprotein from cis to medial Golgi cisternae
D Uncoupling of endocytosed ligands from receptors
E Transport of proteins from the Golgi complex to the plasma membrane
B Retrieval of secretion-granule membrane after exocytosis

Membrane recycling after exocytosis (= receptor-mediated uptake ) of the contents of a


secretion granule occurs via clathrin-coated vesicles . These vesicles consist of three large and
three small polypeptide chains that form a triskelion (= three-legged structure). Thirty-six
clathrin triskelions associate to form a polyhedral cage-like network around the vesicle. These
vesicles are also associated with signal-directed transport of proteins from the trans Golgi
network ultimately to lysosomes or to secretory granules.
Transport of proteins, including glycoproteins, from the Golgi complex to the plasma
membrane, rough endoplasmic reticulum to the cis face of the Golgi apparatus, and from the
cis to medial Golgi cisternae, as well as Uncoupling of endocytosed ligands from receptors is
accomplished by non-clathrin-coated vesicles . These vesicles have coats which as yet are not
fully characterized, but primarily consist of β-COP protein, which does not form a cage-like
network around vesicles and has a different appearance than clathrin.

Question: 9
The primary importance of intermediate filaments in pathology and cytology lies in the fact that
Answer Choices:
A Malignant cells lose the ability to make intermediate filaments
B Intermediate filaments can pinpoint the origin of tumors
C Only malignant cells make intermediate filaments
D Intermediate filaments are seen only in cells of neural origin
E Intermediate filaments are seen only in embryonic tissue
B Intermediate filaments can pinpoint the origin of tumors

Like microfilaments, intermediate filaments are heterogenous. Intermediate filaments are


divided into five categories: cytokeratin, vimentin, desmin, neurofilament and GFAP (glial
fibrillary acidic protein). The type or types of intermediate filaments found in a cell are
characteristic of the type of cell it is.
For instance, epithelial cells characteristically contain cytokeratin. Desmin is found in muscle
cells; mesenchymal cells contain vimentin. Neurons have neurofilament and GFAP is present in
glial cells. Many cells produce combinations of intermediate filaments; again particular
combinations are considered characteristic of certain cell types.
If a tumor is sufficiently differentiated, its cells will continue to make earmark intermediate
filaments even after they become neoplastic. If immunoperoxidase techniques are used to stain
for specific types of intermediate filaments, a pathologist or cytologist may be able to use the
results to identify the tumor type.

Question:10
Microfilaments

Answer Choices:
A Are composed of actin
B Are composed of myosin
C Are composed of both actin and myosin
D Make up the mitotic spindle
E Can be divided into five different groups based on chemical composition
C Are composed of both actin and myosin

Most microfilaments are composed of actin; however some are composed of myosin. The actin
filaments are arranged directly under the cell membrane (terminal web) and are thought to give
the membrane its elasticity. These filaments are also in contact with the inner layer of the cell
membrane and are believed to be responsible for the movement of proteins through the
membrane.
Microfilaments give a cell its motility, allow the cell membrane to undergo exo- and endo-
cytosis and form the inner core of microvilli.
Question: 11
Microtubules

Answer Choices:
A Are composed of actin
B Are composed of myosin
C Are composed of both actin and myosin
D Make up the mitotic spindle
E Can be divided into five different groups based on chemical composition

D Make up the mitotic spindle

Microtubules are a very uniform group of cytoplasmic organelles. They are formed by the
polymerization of tubulin dimers and make up the mitotic spindle, basal bodies, cilia, and
flagella. They are responsible for maintaining cell shape and for cell migration. They cause cilia
and flagella to move and they move cell organelles, secretory granules, and cellular inclusions
through the cytoplasm. Microtubules also form bridges between cells.

Question:12
The defect in Tay-Sachs disease is

Answer Choices:
A A hereditary lack of lysosomal acid phosphatase
B A hereditary lack of lysosomal phospholipase
C A hereditary lack of lysosomal galactosidase
D A hereditary lack of a form of lysosomal nuclease
E A hereditary lack of lysosomal aryl sulfatase
C A hereditary lack of lysosomal galactosidase

Tay-Sachs disease is the result of a hereditary lack of lysosomal galactosidase. This leads to the
build up of lipid in the cell, seen as concentric lamellated structures in tertiary lysosomes of
nerve cells on electron microscopy. The result is blindness and mental retardation.

Question: 13
The purpose of the proton pump in the lysosome's membrane is

Answer Choices:
A To create an acid environment for its hydrolytic enzymes
B To create an alkaline environment for its hydrolytic enzymes
C To create a positively charged membrane to attract the material to be digested
D To create a negatively charged membrane to attract the material to be digested
E To rid the cell of its excess hydrogen ions

A To create an acid environment for its hydrolytic enzymes

Lysosomes are membrane bound organelles which contain strong hydrolytic enzymes that
function most efficiently at a low pH. They are therefore equipped with proton pumps to
generate the acid environment its enzymes need. Lysosomes then fuse with vesicles containing
material that needs to be digested (phagosomes) and the hydrolytic enzymes break down the
contents of the vesicle.
Frequently the lysosomal enzymes cannot entirely break down the contents of the phagosome
and some material is left in the lysosome. This material, primarily phospholipid and unsaturated
fat, is gradually oxidized and becomes lipofuscin pigment.
Lysosomes are divided into primary, secondary and tertiary types. Primary lysosomes have not
yet fused with a phagosome. Secondary lysosomes have fused with a phagosome and tertiary
lysosomes are the residual bodies left when the contents of the phagosome can not be digested
any further.

Question:14
The azurophilic granules of neutrophils are

Answer Choices:
A Mitochondria
B Calcium phosphate crystals
C Staining artifacts
D Swollen Golgi apparatuses
E Lysosomes
E Lysosomes

The azurophilic granules of neutrophils are lysosomes. They contain lysosomal enzymes and
peroxidases. Azurophilic granules are larger than the neutrophil's specific granules and usually
have a dense core.
Neutrophils are remarkable for the fact that they can release their lysosomal contents outside
the cell when they participate in an acute inflammatory reaction. (Osteoclasts can also do this
when resorbing bone.)
Question: 15
The enzymes of the electron transport system are located
Answer Choices:
A In the cell's cytoplasm
B In the smooth endoplasmic reticulum
C On the mitochondrial cristae
D In the mitochondrial matrix
E On the outer mitochondrial membrane
C On the mitochondrial cristae

The enzymes of the electron transport system and an ATPase are located in elementary
particles, which are racquet shaped particles attached to the mitochondrial cristae, or inner
mitochondrial membrane. These particles protrude into the mitochondrial matrix, where the
enzymes for the Krebs cycle are. This allows substrates of these cycles to pass easily down the
enzyme chains.

Question: 16
Mitochondrial cristae

Answer Choices:
A Support elementary particles
B Contain matrix granules
C Are formed from invaginations of the outer mitochondrial membrane
D Have a composition equivalent to that of the cell membrane
A Support elementary particles

Mitochondrial cristae are formed by invaginations of the inner mitochondrial membrane, not
the outer one. The inner mitochondrial membrane is noteworthy in that its chemical
composition differs from the membranes of most eukaryotic organisms (it has more acidic
phospholipid and no cholesterol), but is very similar to that of most bacteria.
Mitochondrial cristae support racquet shaped particles, called elementary particles, which jut
into the inner mitochondrial compartment or mitochondrial matrix (the area bounded by the
inner mitochondrial membrane) and which contain the enzymes of the electron transport
system and an ATPase. These enzymes are then free to interact with the Krebs cycle enzymes,
which are present in the mitochondrial matrix itself.
Matrix granules are present in the mitochondrial matrix and not in the cristae.

Question: 17
Under the light microscope, the Golgi apparatus appears as
Answer Choices:
A A pale-staining area next to the nucleus
B A diffuse granularity of the cytoplasm
C A poorly defined hyaline drop in liver cells
D A well defined ground glass inclusion
E A basophilic crescent next to the nucleus

A A pale-staining area next to the nucleus

When the Golgi apparatus is very well defined, it appears as a clear zone adjacent to one side of
the nucleus. The Golgi apparatus is particularly prominent in plasma cells, where it contributes
to the cell's classic appearance.
In the plasma cell, the Golgi apparatus modifies immunoglobulins and prepares them for
secretion.

Question: 18
Cells with large amounts of smooth endoplasmic reticulum (sER)

Answer Choices:
A Appear basophilic
B Have copious amounts of dense cytoplasm
C Frequently manufacture steroids or lipids
D Are only rarely identified in muscle
E Modify proteins for export out of the cell
C Frequently manufacture steroids or lipids

Cells with large amounts of sER have abundant clear or eosinophilic cytoplasm. These cells are
usually involved in steroid synthesis, lipid synthesis, or drug detoxification. Adrenal cortical cells
and testicular Leydig cells are good examples of steroid secreting cells. Liver cells are usually the
site of drug detoxification.
Modified sER is an important component of muscle cells, where it is called sarcoplasmic
reticulum. The sarcoplasmic reticulum stores calcium that is required for muscle contraction.
The Golgi apparatus is responsible for enzymatic modification of proteins.

Question: 19
Where do proteins to be secreted move to after going from the lumen of the rough endoplasmic
reticulum?
Answer Choices:
A Extracellular space
B Mitochondria
C Golgi apparatus
D Smooth endoplasmic reticulum
E Cytoplasm
C Golgi apparatus

The Golgi apparatus is continuous with the rough endoplasmic reticulum (rER) and, like the rER,
is most prominent in cells that secrete large amounts of protein. Plasma cells have such large
Golgi apparatuses that can be seen in the light microscope as a clear zone next to the nucleus -
the perinuclear hof.
The Golgi apparatus makes oligosaccharides, which can then be added to the newly synthesized
protein to form glycoproteins. Sulfate groups are added to form proteoglycans. The products of
these reactions are sorted, concentrated, and packaged into vesicles, and the vesicles are
pinched off. These vesicles containing newly made complex proteins then migrate to their
appropriate individual destinations.

Question: 20
With regard to protein transcription along the rough endoplasmic reticulum, what is a signal peptide?

Answer Choices:
A An amino acid segment that directs the ribosome to begin synthesizing a protein
B An amino acid sequence that directs the ribosome to stop synthesizing a protein
C An amino acid sequence that directs the newly forming protein into the endoplasmic
reticulum
D An RNA sequence that directs a ribosome to start synthesizing a protein
E An amino acid sequence that directs the ribosome to insert a disulfide bond at a
particular site

C An amino acid sequence that directs the newly forming protein into the endoplasmic
reticulum

The signal peptide is the first set of amino acids in a newly formed protein that needs to be
directed into the endoplasmic reticulum (ER). These are a hydrophobic group of peptides which
bind to a receptor on the rough endoplasmic reticulum and which cause the newly forming
protein to pass into the ER as it is being made. If only part of the protein strand is to enter the
ER, a second hydrophobic sequence will signal a stop to the process.
As the protein passes through the ER and Golgi apparatus, chemical modifications are made
and the protein is packaged in vesicles. These proteins will eventually either be exported out of
the cell, become part of the cytoplasmic membrane, or be contained within a lysosome.
This process is also referred to as contratranslational insertion of the protein into the rER.

Question: 21
The particles seen lining the rough endoplasmic reticulum on transmission electron microscopy are

Answer Choices:
A Clusters of glycogen molecules
B Strands of tubulin cut in cross-section
C Exocytotic vesicles
D Ribosomes
E Centrioles

D Ribosomes

Ribosomes lining endoplasmic reticulum create the cytoplasmic element called rough
endoplasmic reticulum (rER). rER is a major site of protein synthesis in the cell. The ribosomes
along the ER make strands of protein by transcribing messenger RNA; the strands of protein are
either released into the cytoplasm or, if they are to be modified or exported out of the cell,
directed into the interior of the endoplasmic reticulum.

Question: 22
Microfilaments and microtubules

Answer Choices:
A Dissolve and re-form in response to fluctuations in the pH and calcium concentrations
in the cytoplasm
B Replicate at time of cell division
C Are fixed and permanent cellular structures
D Are produced by the rough endoplasmic reticulum
E Are found only in cells with asymmetric shapes, e.g. nerve cells
A Dissolve and re-form in response to fluctuations in the pH and calcium concentrations in the
cytoplasm

An important characteristic of microfilaments and microtubules is that they can dissolve and re-
form in response to local conditions within the cell. Calcium ion concentration and pH are two
factors that are known to regulate the formation of these cytoskeletal elements.
For example: The presence of a wound is known to induce the formation of actin
microfilaments in surrounding fibroblasts. The actin microfilaments allow the cells surrounding
the wound to contract, producing the well known phenomenon wound contraction.

Question:23
The primary function of a cell demonstrating an abundance of smooth endoplasmic reticulum (sER)
within the cytoplasm would be:

Answer Choices:
A Glycogen metabolism
B Muscle contraction
C Depolarization and conduction
D Antibody synthesis
E Oxygen transport

A Glycogen metabolism

Smooth ER (sER), when viewed by an electron microscope, appears as a complex system of


anastomosing or dilated tubules, but without association with ribosomes. Cells which contain
substantial amounts of SER in the cytoplasm are associated with functions relating to glycogen
metabolism, steroid synthesis, or drug metabolism. Cells associated with muscle contraction
may contain small amounts of SER in the cytoplasm, but the predominant intracellular feature
of these cells are actin and myosin filaments. Similarly, nerve cells, or their processes, may
contain SER, but the predominant form of ER present in cells associated with depolarization and
conduction would be rough endoplasmic reticulum (RER). Antibody synthesis, such as
performed by a plasma cell, shows extensive RER in the cytoplasm. A cell that is associated with
oxygen transport, such as the erythrocyte, would have few cytoplasmic organelles.

Question: 24
Which of the following cell type would have substantial amounts of smooth endoplasmic reticulum
(sER) throughout its cytoplasm?

Answer Choices:
A Plasma cell
B Red Blood cell
C Fibroblast
D Testicular interstitial (Leydig) cell
E Macrophage

D Testicular interstitial (Leydig) cell


Smooth ER (sER), when viewed by an electron microscope, appears as a complex system of
anastomosing or sometimes dilated tubules, but without association with ribosomes. Cells that
are involved with the formation of steroids show abundant cytoplasmic sER, such as the
testicular interstitial (Leydig) cell. A plasma cell produces antibodies which are proteinaceous
and consequently show abundant rough ER in their cytoplasm. A red blood cell has little or no
membranous organelles within the cytoplasm, reserving most of the cell volume for
hemoglobin. The fibroblasts, when active are involved in the production of collagen, and as
such show abundant rough ER. Macrophages show little sER in the cytoplasm.
Question:25
Mitochondrial matrix granules are

Answer Choices:
A Clusters of respiratory enzymes
B Storage sites for calcium ions
C Ribosomes
D Clumps of degenerated membrane material
E Lipofuchsin

B Storage sites for calcium ions

Mitochondrial matrix granules are storage sites for divalent calcium ions. These granules
increase in size and number as the amount of calcium in the cytoplasm increases.
Mitochondria do contain ribosomes, but ribosomes are not matrix granules.

Question: 26
A Russell body is

Answer Choices:
A A clump of immunoglobulins
B An acidophilic body whose presence in liver cells usually indicates alcohol abuse
C A cluster of degenerating mitochondria seen in some myopathies
D A viral inclusion
E An inactive X chromosome seen in some of the nuclei of female patients
A A clump of immunoglobulins

In patients with chronic inflammation, plasma cells may produce immunoglobulin faster than it
can be extruded. The immunoglobulin then backs up through the Golgi apparatus into the
endoplasmic reticulum, which in turn becomes dilated. In the light microscope, these clumps of
backed up immunoglobulin are seen as eosinophilic hyaline inclusions. Classic examples of
Russell bodies resemble mulberries.

SECTION 5_ MICROCHEMISTRY AND HISTOCHEMISTRY

Question: 1
What is the classic interpretation of cytoplasmic basophilia?

Answer Choices:
A Cellular degeneration
B Increased protein synthesis
C Increased numbers of mitochondria
D A well developed Golgi apparatus
E Malignancy
B Increased protein synthesis

Cytoplasmic basophilia is a well recognized sign of increased protein synthesis. When a cell is
synthesizing lots of protein, the basic dye hematoxylin (as in hematoxylin and eosin, or H & E)
binds to the phosphate residues of the abundant RNA molecules in the cell's cytoplasm. This
gives the cell's cytoplasm its classic blue color.
Cells that usually exhibit high levels of protein synthesis include red blood cell precursors
(hemoglobin), plasma cells (immunoglobulins), and certain developing cells (muscle and nerve
cells).
Another stain useful in demonstrating cells with large accumulations of RNA, especially plasma
cells, is methyl green pyronin. Some pathologists use this stain to assess the stage of transplant
rejection.

Question: 2
Oncocytes are

Answer Choices:
A Malignant cells
B Cells with large accumulations of glycogen
C Cells which stain prominently with hematoxylin
D Cells with large accumulations of mitochondria
E Cells with large lipid vacuoles
D Cells with large accumulations of mitochondria
Oncocytes are metaplastic or neoplastic cells which have accumulated large numbers of
mitochondria. On light microscopy these cells have large amounts of granular, eosinophilic
cytoplasm.
Studies have shown that the mitochondria in oncocytes do not function properly; there is
speculation that their large numbers may represent a compensatory hyperplasia.

Question: 3
The stain most commonly used in cytology preparations is

Answer Choices:
A PAS
B Methylene blue
C Papanicolaou stain
D Acid fuchsin
E Trichrome

C Papanicolaou stain

The Papanicolaou stain is the stain used most commonly in cytology preparations. This
procedure uses hematoxylin, eosin-alcohol and orange G as stains. Its advantages include the
fact that it stains nuclear chromatin well and that it is a very sensitive stain for the presence of
keratin in the cytoplasm (squamous cells). The advantage most frequently cited, however, is
that it leaves the cell's cytoplasm relatively transparent, letting the cytologist focus up and
down through a cluster of cells.

Question: 4
The mucopolysaccharidoses are a group of genetically independent disorders caused by a deficiency
of enzymes catalyzing the stepwise degradation of glycoaminoglycans. The clinical features, while
variable, are progressive and involve multiple organs. The cellular manifestations of these different
genetic disorders include many changes that are secondary to the primary cellular change, which is
characterized by

Answer Choices:
A Decreased protein synthesis
B Abnormal lipid synthesis
C Loss of membrane integrity
D Lysosomal storage
E Increased carbohydrate synthesis
D Lysosomal storage

Lysosomal storage of undegraded substrates, primarily glycoaminoglycans, is the most


characteristic feature of the mucopolysaccharidoses. The accumulated substrates are due to a
deficiency in an enzyme required in the stepwise catabolism of the glycoaminoglycans.
Decreased protein synthesis is not the correct choice because it is variable and is a secondary
manifestation of the mucopolysaccharidosis.
Lipid synthesis is not impaired in the mucopolysaccharidoses.
The cellular and subcellular membranes in the mucopolysaccharidoses remain intact. There is
not an increase in carbohydrate synthesis in the mucopolysaccharidoses.

Question: 5
In freeze-fracture

Answer Choices:
A Live fixed tissue is quick frozen to minus 160° F. The frozen tissue (after proper
dehydration and staining) may be examined under light or electron microscope
B A cryoprotectant (such as glycerol) is allowed to infiltrate the tissue, and the tissue is
rapidly frozen to minus 160° F. The frozen tissue is fractured (either in a freeze fracture
apparatus, or with a sharp knife or razor blade) and the frozen water is evaporated in
vacuum. The tissue is then coated , typically with platinum, to create a shadow cast. This
tissue is removed and the surface replica, not the tissue itself, is picked up on grids to be
examined with transmission electron microscope (TEM)
C A cryoprotectant (such as glycerol) is allowed to infiltrate the tissue, and the tissue is
rapidly frozen to minus 160° F. The frozen tissue is fractured (either in a freeze fracture
apparatus, or with a sharp knife or razor blade) and the frozen water is evaporated in
vacuum. The tissue is then coated, typically with platinum, placed on copper grids to be
examined with transmission electron microscope (TEM)
D A cryoprotectant (such as glycerol) is allowed to infiltrate the tissue, and the tissue is
rapidly frozen to minus 160° F. The frozen tissue is mounted in a cryostat and thin
sections are cut. The water in the sections is evaporated in vacuum, the tissue is coated,
typically with platinum, placed on copper grids and examined in a transmission electron
microscope (TEM)
E The live tissue containing the cryoprotectant (such as glycerol) is rapidly frozen, and
thin sections are cut in a cryostat. The sections are processed through dehydration and
clearing agents, stained with proper dyes, mounted in appropriate medium and are
examined under either the light or electron microscope (transmission or scanning)
B A cryoprotectant (such as glycerol) is allowed to infiltrate the tissue, and the tissue is rapidly
frozen to minus 160° F. The frozen tissue is fractured (either in a freeze fracture apparatus, or
with a sharp knife or razor blade) and the frozen water is evaporated in vacuum. The tissue is
then coated , typically with platinum, to create a shadow cast. This tissue is removed and the
surface replica, not the tissue itself, is picked up on grids to be examined with transmission
electron microscope (TEM)

A cryoprotectant (such as glycerol) is allowed to infiltrate the tissue, and the tissue is rapidly
frozen to minus 160° F. The frozen tissue is fractured (either in a freeze fracture apparatus, or
with a sharp knife or razor blade) and the frozen water is evaporated in vacuum. The tissue is
then coated , typically with platinum, to create a shadow cast. This tissue is removed and the
surface replica, not the tissue itself, is picked up on grids to be examined with transmission
electron microscope (TEM). The method is especially important in the ultrastructural study of
cell membranes.
The other methods described do not apply to the procedure of Freeze fracture .

Question: 6
For a histochemical reaction to be recognized as valid and meaningful, which one of the following
requirements must it fulfill?

Answer Choices:
A The product being analyzed must not diffuse out of its original site
B The substance to be visualized may have to be fixed with an appropriate fixing agent
before the histochemical test can be applied
C The method employed must denature or block reactive groups
D The product of the reaction should be soluble and non-electron scattering

A The product being analyzed must not diffuse out of its original site

It is imperative that the product (e.g., an inorganic salt such as calcium, iron, copper, etc., or
organic substances, such as saccharides, lipids, proteins, etc.) not diffuse out of its original site,
otherwise it will be difficult to localize.
The use of fixed tissue for enzyme demonstration is debatable, and the staining (or other
procedures employed) must not denature or block the reactive groups. The product of
histochemical reaction must at least be insoluble, so that it can be visualized at its original site.

Question: 7
Lipofuscin is

Answer Choices:
A A form of iron pigment
B A cluster of tertiary lysosomes
C A form of bile pigment
D A form of melanin pigment
E A cluster of degenerating mitochondria

B A cluster of tertiary lysosomes

Lipofuscin is a brown pigment which accumulates in cells over a period of time. It is made up of
tertiary lysosomes - lysosomes which contain phagocytized material that cannot be hydrolyzed
any further. These tertiary lysosomes (or residual bodies) are slowly oxidized to form a brown
pigment.
Because lipofuscin forms slowly over a period of time, it is called "age pigment". In cytology it is
regarded as a sign of a benign process because most malignancies grow too quickly for it to
form.

SECTION 6_ ISOLATION OF WHOLE CELLS AND PART OF CELLS

Question: 1
A laboratory technician has been asked to prepare a suspension of nuclei from isolated epithelial cells
acquired from the small intestines of an adult male. Which one of the following procedures you will
recommend that he should follows?

Answer Choices:
A Cell disruption; washing in Tris-NaCl buffer; centrifugation at 650 g for 20 minutes
B Cell disruption; washing in Tris-NaCl buffer; centrifugation at 650 g for 20 minutes;
resuspension in Tris-NaCl buffer; centrifugation at 480 g for 20 minutes
C Cell disruption; washing in Tris-NaCl buffer; centrifugation at 650 g for 20 minutes;
Resuspension in Tris-NaCl buffer and recentrifugation at 650 g for 20 minutes;
resuspension and homogenization to disperse clumped material; further centrifugation at
480 g for 20 minutes
D Cell disruption; washing in Tris-NaCl buffer; centrifugation at 650 g for 20 minutes;
Resuspension in Tris-NaCl buffer and recentrifugation at 650 g for 20 minutes;
resuspension and homogenization to disperse clumped material; further centrifugation at
480 g for 20 minutes. Resuspension of pellet in Tris-NaCl buffer and centrifugation at 480
g for 20 minutes
E Cell disruption; washing in Tris-NaCl buffer; centrifugation at 650 g for 20 minutes;
Resuspension in Tris-NaCl buffer and recentrifugation at 650 g for 20 minutes;
resuspension and homogenization to disperse clumped material; further centrifugation at
480 g for 20 minutes; centrifugation of supernatant pool at 4000 g for 20 minutes
D Cell disruption; washing in Tris-NaCl buffer; centrifugation at 650 g for 20 minutes;
Resuspension in Tris-NaCl buffer and recentrifugation at 650 g for 20 minutes; resuspension
and homogenization to disperse clumped material; further centrifugation at 480 g for 20
minutes. Resuspension of pellet in Tris-NaCl buffer and centrifugation at 480 g for 20 minutes

Cell disruption; washing in Tris-NaCl buffer; centrifugation at 650 g for 20 minutes;


Resuspension in Tris-NaCl buffer and recentrifugation at 650 g for 20 minutes; resuspension
and homogenization to disperse clumped material; further centrifugation at 480 g for 20
minutes. Resuspension of pellet in Tris-NaCl buffer and centrifugation at 480 g for 20 minutes.
Steps detailed out under answers (A), (B), (C) and (E) will provide cell slurry, cells, mixture of
nuclei and nondisrupted cells, and mitochondrial pellets, respectively.

Question: 2
Reagents such as silicone, albumin, Hypaque, and Ficoll are used to isolate cancer cells and leukocytes
in bloody fluids on the basis of their:

Answer Choices:
A Specific gravity
B Staining characteristics
C DNA concentration
D Cell membrane stability
E Nuclear characteristics

A Specific gravity

Silicone, albumin, Hypaque, and Ficoll can all be mixed with a bloody fluid to form a layer with
the white cells and cancer cells on one side of the layer and red cells on the opposite side. The
white cell/cancer cell layer is then removed with a pipette and smeared on a microscopic slide.
The layering is possible because red cells have a specific gravity of 1.092 to 1.097, white cells
have a specific gravity of 1.065, and cancer cells have an average specific gravity of 1.056.
These methods all have a major drawback - the cells are not very well preserved by the time
they are spread on the slide.
Question: 3
A tissue is homogenized in buffered sucrose (0.25M, pH 7.5) and the homogenate subjected to
centrifugation at various speeds. Which of the following subcellular components would require the
highest centrifugal force to sediment as a pellet?

Answer Choices:
A Mitochondria
B Golgi complex
C Endoplasmic reticulum
D Nuclei
E Lysosomes

C Endoplasmic reticulum

Subcellular components of cells can be separated by differential centrifugation. In this


procedure, a homogenate is prepared and then centrifuged at low speed. The resulting
supernatant is then centrifuged at a higher speed. This process continues for several rounds,
with the supernatant subjected to higher and higher centrifugal force. The homogenate is first
centrifuged at 960 g for 10 min. This will pellet the nuclei and leave all other components in the
supernatant. This supernatant is then centrifuged at 25, 000 g for 10 min to pellet the
mitochondria, lysosomes, and peroxisomes. A centrifugal force of 35,000 g for 30 min will pellet
the Golgi complex. A final centrifugation at 105,000 g for 100 min will pellet the smooth and
rough endoplasmic reticulum. The supernatant resulting from this 105,000 g for 100 min
centrifugation contains the cytosol of the cells.

Question: 4
Which one of the following purposes would be a suitable use for a flow cytometer?

Answer Choices:
A Measurement of cellular DNA
B Measurement of cellular hemoglobin
C Measurement of plasma flow in a capillary
D Measurement of cytoplasmic flow
E Measurement of cell organelle content
A Measurement of cellular DNA

One type of instrument called a fluorescence-activated cell sorter can be used in flow
cytometry to either sort cells or to measure either a cell's DNA or RNA content. Measurement
of cellular DNA and RNA is determined by using a DNA-binding dye and measuring the amount
of fluorescence from the cells passing through a detector. Similarly, this instrument and
technique can also be used to sort cells using antibodies specific to a cell surface molecule and
chemically conjugated with a fluorescent dye. The cells, having the fluorescent dye attached to
an antibody specific to a cell surface molecule, are passed in a flowing stream in front of a beam
of light with appropriate wavelength to cause the cells with the dye attached to them to
fluoresce. The fluorescence can be measured by a detector and when desired, the stream
containing the cells may be directed between 2 electrically charged plates appropriate to the
charge of the dye, and the cells onto which the dye is attached will separate from the rest of
the cells that do not contain the specific membrane marker.
Question: 5

You desire an analysis of the hepatocyte lysosomal β-galactosidase levels in a patient suspected of a
deficiency in this enzyme. To isolate the hepatic lysosomes from biopsied tissue, you would
recommend which of the following techniques to get a purified lysosomal preparation necessary for
the biochemical analysis?

Answer Choices:
A Differential centrifugation
B Density-gradient centrifugation
C Metalo-precipitation
D Electron microscopy
E Homogenization

B Density-gradient centrifugation

The organelles found in a homogenized tissue may be purified through a variety of processes
and the technique used depends upon the outcome to be achieved. In this case, it is desirable
to have a fairly pure fraction of lysosomes that can be utilized in a biochemical (enzymatic)
analysis. Isolation of a purified fraction of lysosomes in which biochemical activity is retained is
best achieved by density gradient centrifugation. Various organelles differ in both size and
density and may be centrifuged at various velocities (centrifugal forces) to remove specific
organelles based on their density, and hence, sedimentation rate. This process of differential
centrifugation, while useful, does not always provide a pure fraction of the desired organelles,
such as lysosomes. Subjecting a tissue homogenate to centrifugation in a medium of increasing
density, such as sucrose, allows each cellular particle to migrate to a position at which its
density is equal to the density of the surrounding liquid. Purity of the fraction isolated by
density-gradient centrifugation can be assessed by the use of electron microscopy to visualize
the content of the fraction collected.

Question: 6
Freeze fracture is a technique for examining

Answer Choices:
A Cell membranes
B Mitochondria
C Cell nuclei
D Endoplasmic reticulum
E Extracellular secretions
A Cell membranes

In freeze fracture a very small piece of tissue is infused with a substance like glycerol, which will
protect the tissue while it is being frozen. The tissue is quickly frozen to about -1600 C and hit
with a knife blade. The blow causes a fracture which usually occurs through the hydrophobic,
central lipid core of the cell membrane. The membrane is divided into the half that lined the
extracellular surface and the half that lined the intracellular surface. This process produces a
specimen that provides macromolecular details of the cell membrane when it is examined by
electron microscopy.

SECTION 7_ MICROSCOPY

Question: 1
Which statement is true of birefringence?

Answer Choices:
A Birefringence is created by the presence of two optical axes in a given material
B All crystals are birefringent
C Only a Congo red stain can make a protein birefringent
D A substance must be crystalline to be birefringent
A Birefringence is created by the presence of two optical axes in a given material

Birefringence is seen in substances that have two optical axes. An optical axis is the plane
through which light can move without being refracted. A substance with two optical axes will
refract light in two different directions, creating two different beams, each of which is out of
phase with the other. The degree to which the two beams are differently refracted is what
determines the degree of birefringence.
All crystals are not birefringent and all specimens that are birefringent are not crystals. Striated
muscle is birefringent and amyloid can be made to exhibit "apple-green" birefringence by
staining it with Congo red. Congo red is not necessary to demonstrate birefringence in every
substance, protein or not.

Question: 2
The primary advantage of darkfield microscopy is that it:

Answer Choices:
A Provides a 3 dimensional image of a specimen
B Is faster than regular brightfield microscopy
C Is less expensive than brightfield microscopy
D Demonstrates small particles better than brightfield microscopy
E Shows only the external surface of a specimen

D Demonstrates small particles better than brightfield microscopy

Dark field microscopy provides indirect illumination of a specimen so that the background of
the field is dark. The only light seen by the microscopist is light that has been scattered by the
specimen. This provides a very high level of contrast between the background and the
specimen, which makes it easier to identify small particles than it is on brightfield microscopy.
Darkfield microscopy is used to evaluate autoradiographs, to identify the presence of crystals,
and to identify spirochetes.
Question: 3
The purpose of staining a histologic or cytologic slide is to:

Answer Choices:
A Increase the illumination of the specimen.
B Increase the contrast between the elements of the specimen.
C Produce a uniform light absorbance throughout the specimen.
D Give histologists and pathologists a uniform frame of reference.
E Make studying histology and cytology easier for those starting out.

B Increase the contrast between the elements of the specimen.

There are three properties of a microscopic system that are of primary importance in
determining the quality of that particular system. These are its resolution, its magnification and
its level of contrast. However, even at the optimal microscopic settings, the contrast between
the different elements of a specimen are inadequate for most purposes. Staining is therefore
done to enhance the contrast between different cells and cellular components. Although
hematoxylin and eosin (H & E) is most commonly performed, other stains can be chosen to
enhance the contrast between cellular elements that are not well defined on H & E.
Question: 4
The numerical aperture (N.A.) of a lens

Answer Choices:
A Measures the focal length of the lens
B Measures the focal point of the lens
C Measures the ability of the lens to collect the light illuminating the specimen
D Is reported in degrees of an angle
E Is totally independent of the refractive ability of the lens
C Measures the ability of the lens to collect the light illuminating the specimen
Image(s) / Chart(s):
Click image to view full size. Click open image to close. Click and hold open image to move.

The numerical aperture of a lens is a measure of both the amount of light a lens takes in thus of
its ability to resolve the finer elements of a image. It is calculated by multiplying the sine of the
aperture angle by the refractive index of the material making up the lens. (This result is
expressed as a sine value rather than the degrees of an angle and is therefore called
"numerical" aperture.)
To understand the aperture angle, one must think of the light beams illuminating each point of
the specimen as radiating away from the specimen in all directions. A given lens can only gather
the part of the light that radiates toward it at an appropriate angle. A drawing of this portion of
the radiated light forms a cone, with the tip of the cone at the specimen and the base of the
cone at the lens. The aperture angle is the angle between the central axis of the cone and the
rays forming the outer perimeter of the cone.
Numerical aperture can also be seen as a measure of the depth of field produced by a given
lens. A lens with a small N.A. will have a shallow depth of field, but will allow the microscopist
to see a very large portion of the specimen. A lens with a large N.A. will have a deep depth of
field, but will focus on only a small portion of the specimen.

Question: 5
If the lens of a microscope has a short focal length
Answer Choices:
A The lens is weak
B The lens is strong
C The lens is convex
D The lens is concave
E The lens is not useful for histologic or cytologic work

B The lens is strong


Image(s) / Chart(s):
Click image to view full size. Click open image to close. Click and hold open image to move.

The focal length is the distance between the center of a lens and its focal point. A lens with a
short focal length is strong and a lens with a longer focal length is weaker.
Any strength lens may either be concave or convex. A convex lens has a surface that curves
away from the light beam. This kind of lens is used when one wants the elements of a light
beam to converge.
A concave lens has a surface that curves toward the light beam. It is used to make the elements
of a light beam diverge.

Question: 6
The primary factor that allows an electron microscope to resolve subcellular structures is

Answer Choices:
A The wavelength of the electron beam
B The numerical aperture (N.A.) of the lens
C The focal length of the lens
D The magnification of the lens
E The voltage of the microscope

A The wavelength of the electron beam

The resolving power of a microscope is ultimately limited by the wavelength of the light used to
illuminate the specimen. Because the wavelengths of electron beams are very short when
compared to those of visible light, microscopes that use electron beams for illumination have
greater resolving power than those that use visible light.
Theoretically, an electron microscope can resolve structures about 100,000 times smaller than
a light microscope can. In practice, however, the electron microscopes that have been
developed have a resolution only about 1000 times greater than a light microscope.

Question: 7
The resolution of a microscope is:

Answer Choices:
A The same as the magnification of the microscope
B Determined by an inverse relationship with the magnification of the microscope
C Determined by a direct relationship with the magnification of the microscope
D Not determined by magnification
E Only a function of the numerical aperture of the microscope

D Not determined by magnification


Image(s) / Chart(s):
Click image to view full size. Click open image to close. Click and hold open image to move.
Resolution and magnification are separate factors, although resolution does determine how
much magnification is useful. In other words, it is possible to magnify an object without making
it any easier to distinguish between the different elements of the object, producing empty
magnification . On the other hand, it is also possible for a microscope to be capable of resolving
two different elements but not being capable of magnifying the resolved image enough to be
seen by the microscopist.
The resolution of a particular microscope is determined by a number of different factors, but
the ultimate factor that determines resolution is the diffraction pattern of the light illuminating
the specimen. This is in turn related to the wavelength of the light.
The diffraction pattern of a beam of light is the pattern formed by the light waves as they pass a
through an opening (like the aperture of a microscope) or around a barrier. The pattern consists
of a series of concentric rings or circles and the resolution of a particular wavelength of light is
limited by the diameter of the first circle, or diffraction ring.
If two points are spaced too closely, their diffraction patterns interfere with each other and it
will not be possible to distinguish the objects. They will be seen only as a blurred spot.

Question: 8
What is meant by a "compound" microscope?

Answer Choices:
A A microscope that produces both resolution and magnification
B A microscope that produces its resolution in two steps
C A microscope that magnifies in two steps
D A microscope that can be adjusted to either polarize light or not polarize it
E A microscope with two specimen stages

C A microscope that magnifies in two steps

A compound microscope is one that magnifies the image of the specimen in two stages. (This is
the most common type of microscope available today.) The first magnification is accomplished
by the objective - thus, the objective both resolves and magnifies the details of the specimen.
The second magnification is done by the ocular, but it does not resolve the details of the
specimen in any way, as the objective does.

Question: 9
An investigator has been provided funds to study the dynamic interaction between cell surfaces and
deeper structure. Which one of the following microscopes will you recommend that the investigator
buy?
Answer Choices:
A Interference microscope
B Phase contrast microscope
C Scanning electron microscope
D Confocal microscope
E Atomic force microscope
E Atomic force microscope

Recommend the atomic force microscope. The atomic force microscope (AFM), invented by G.
Binning and associates, in 1986, is capable of imaging hard surfaces at a very high, even atomic
resolution. The feat is accomplished by mechanically scanning with a small sharp probe (tip),
which is brought into very close proximity to the hard surface of a tissue. By guiding the probe
over the surface, the AFM can collect three-dimensional data that reflect the mechanical
interaction between the probe and the dynamics of the surface. The data collected by the
microscope are at much higher resolution (at least one full order of magnitude) than those
imaged by any conventional optical microscope.
The interference microscope uses two separate beams of light, and, as such, is used for
measuring the density (thickness) (and the calculation of the mass) of the tissue. Phase contrast
and confocal microscopes are ideal for studying unstained, particularly living cells. A Confocal
microscope is particularly valuable in observing the cell cytoskeletal components and
extracellular matrix (fluorescent microscopy). The scanning electron microscope provides a
three-dimensional scan of specimen which must be fixed, dried and coated with a thin layer of
gold or platinum (a technique known as shadowing) prior to examination.

Question: 10
The primary advantage of phase contrast microscopy is that it

Answer Choices:
A Can see through many layers of cells
B Can provide a three dimensional view of a specimen
C Can enhance the staining properties of a specimen
D Can generate enough contrast in a specimen that staining is not necessary
E Is much less expensive than regular brightfield microscopy

D Can generate enough contrast in a specimen that staining is not necessary

The primary advantage of phase contrast microscopy is that it generates enough contrast in
non-stained material that staining is not necessary. The process uses the fact that denser parts
of the specimen refract the illuminating light beam more than other parts. The denser parts of
the specimen are seen as dark spots and the less dense areas are seen as lighter areas.
The fact that phase contrast microscopy does not require staining means that living cells and
tissues, especially tissue cultures, can be examined with it. (The first step in staining tissue is to
fix, or kill, it.)

Question: 11
The "bright-field" microscope is routinely used in histology and pathology. The resolution (ability to
discriminate the distance between 2 separate objects) of this microscope is dependent upon:

Answer Choices:
A The level of magnification by the ocular lens
B The wavelength of light
C The numerical aperture
D The wavelength of the light and the numerical aperture
E The wavelength of the light and the magnification by the ocular lens
D The wavelength of the light and the numerical aperture

There are many factors that affect resolution in a bright-field microscope such as specimen
thickness, fixation, and staining qualities. The basic characteristic is that structures within the
specimen diffract light and the angle that light is diffracted is directly proportional to the
wavelength of the incident light. Additionally, the diffraction angle is inversely proportional to
the spacing of the structures within the specimen that diffracts the light. As the aperture of the
objective is increased, more diffracted light can participate in the image formation. This
ultimately results in resolution of objects that are more closely positioned.

SECTION 8_ BIOLOGICAL MICROSCOPIC PREPARATION

Question: 1

Which one of the following tissue staining techniques is routinely applied in the detection of
adenocarcinomas of mammary glands, kidneys, colon, and other gastrointestinal cancers, as well as,
endometrial and ovarian cancer?

Answer Choices:
A Periodic acid Schiff (PAS)
B Feulgen staining technique
C Metachromasia
D Mallory staining technique
E Hematoxylin and eosin
B Feulgen staining technique
Feulgen microspectrophotometry, or Feulgen technique, is a method that was developed to
study DNA increases in developing cells and to analyze ploidy (the number of times the normal
DNA content of a cell is multiplied). Recently, it has become a valuable tool for surgical
pathologists in evaluating the metastatic potential of the malignant tumor and in making
prognostic and treatment decisions. The technique of static cytometry of Feulgen-stained
sections of tumors (contrasted with flow cytometry, which can only be used on isolated
individual cells) uses microspectrophotometry, coupled with a digitizing imaging system to
measure the absorption of light at 560 nm by cells and cell clusters in Feulgen-stained sections.
This allows the pathologist to describe ploidy pattern in epithelial cancer (adenocarcinomas).
This method is particularly advantageous in the study of breast cancer, kidney cancer, colon
and other gastrointestinal cancers, endometrial (uterine epithelium) cancer and ovarian cancer.
Periodic acid-Schiff (PAS) reaction, as it stains carbohydrates and carbohydrate-rich
macromolecules, is used to demonstrate glycogen in cells, mucus in various cells and tissues,
the basement membrane that underlies epithelia and reticular fibers in connective tissue.
Metachromasia by toluidine blue are used to highlight cells and tissue with high concentrations
of ionized sulfate and phosphate groups, such as the ground substance of cartilage, the
heparin-containing granules of the mast cell, and the rough endoplasmic reticulum of plasma
cells. Mallory staining technique (consisting of a mixture of aniline blue, acid fuchsin and orange
G) selectively stains collagen, ordinary cytoplasm and red blood cells. Hematoxylin (a water
soluble stain) and eosin (an alcohol soluble counterstain) are used in routine examination of
tissues. They display structural features but provide no information on chemical characteristics
of the specimen.

Question: 2
After fixation, tissue to be examined microscopically is mounted in

Answer Choices:
A Plaster
B Gelatin
C Plastic
D Paraffin
E Resin

D Paraffin

After fixation, tissue needs to be embedded in a substance that will give the tissue enough
strength and flexibility to be cut smoothly in very thin sections (about 10 microns in diameter).
To do this, the fixed tissue is washed repeatedly in increasing concentrations of alcohol and
organic solvents (a process called dehydration). The specimen is infiltrated with paraffin and
the paraffin is allowed to harden into a solid block. The block is then placed in a microtome ,
which cuts the sections for microscopic examination.

Question: 3
In cytology, "air-dryin" is

Answer Choices:
A The preferred method of fixation
B The preferred method of fixation only for non-gynecological specimens
C Always to be avoided
D Permissible when nuclear features must be preserved
E Permissible when cytoplasmic textures are important
E Permissible when cytoplasmic textures are important

"Air-drying" is a type of fixation that occurs when a Pap smear is not immediately placed in
fixative after it is obtained. The nuclei of air-dried cells swell and become distorted and the
chromatin pattern, which is vital to diagnosing dysplasia and malignancy, is lost. The cytoplasm
of air-dried cells does not reliably take up the Papanicolaou stain and air-dried smears should
not be stained with Pap stain. In routine gyn cytology air-drying is always to be avoided.
In some cases, particularly those involving non-gynecological specimens, some cytologists feel
that an air-dried smear stained in an appropriate manner (a Romanowsky stain) provides
important information about a cell's cytoplasm. In these cases both alcohol fixed and air dried
smears will be made.

Question: 4
If demonstrating the presence of lipid (fat) is crucial to a diagnosis, how must the routine tissue
handling be altered?

Answer Choices:
A The tissue is frozen and cut while frozen
B The concentration of formaldehyde in the fixative is doubled
C The dehydration steps in alcohol are skipped
D One to two more alcohol dehydration steps are added
E The amount of time spent in the eosin stain is increased
A The tissue is frozen and cut while frozen

Because formalin does not react with and preserve lipid, lipid is washed out during the routine
tissue processing steps that involve washes with organic solvents. If the importance of staining
for lipid is known at the time the tissue specimen is taken, a portion of the tissue will be frozen
and cut into microscopic sections while frozen. This avoids the steps that allow the lipid to be
lost and the section can then be stained with a stain specific for lipid.

Question: 5
When solutions of heavy metals are used as fixatives

Answer Choices:
A The mitochondria are preferentially stained
B The cell membranes are highlighted
C The nuclear DNA stands out from the associated proteins
D Intestinal microvilli are more obvious
E Collagen becomes birefringent

B The cell membranes are highlighted

Heavy metals such as osmium and permanganate bind to the phospholipids of the cellular
membranes, preserving them as formalin does not. Osmium is a common electron microscopy
fixative, accounting for the excellent preservation of membranes in electron microscopy.

Question: 6
Tissue excisions and biopsies are preserved by fixation in

Answer Choices:
A 95% ethanol
B Formalin
C 100% methanol
D Glutaraldehyde
E 100% ethanol

B Formalin

Formalin, usually a solution of 37% formaldehyde, is used to preserve, or fix, tissue biopsies and
larger tissue specimens. It works by crosslinking the amine groups of proteins. Formalin does
not preserve lipid-based structures, such as cell membranes, very well.
Ethanol and methanol are used to fix cytology specimens, which are at most only a few cell
layers thick. Glutaraldehyde is used to preserve tissue for electron microscopy.

Question: 7
Fracturing frozen membranes along a plane separating the bilipid leaflets of the plasma membrane
results in 2 new surfaces. These 2 surfaces or "faces" are referred to as:

Answer Choices:
A The E-face (extracellular) and P-face (cytoplasmic)
B The I-face (intracellular) and E-face (extracellular)
C The P-face (cytoplasmic) and I-face (intracellular)
D The C-face (cytoplasmic) and E-face (extracellular)
E The H-face (hydrophobic) and P-face (hydrophilic)

A The E-face (extracellular) and P-face (cytoplasmic)

A cryoprotectant is used to infiltrate a tissue specimen, which is then rapidly frozen. Due to the
weakness of the hydrophobic portion of the plasma membrane in the frozen state, the plane of
fracture will pass preferentially through the 2 leaflets of the hydrophobic lipid bilayer, resulting
in the formation of 2 surfaces. One of the surfaces is called the E-face since it represents the
exposed extracellular surface. The other surface is called the P-face since it represents the
cytoplasmic (protoplasmic) surface remaining intact on the cell.

SECTION 9_ NUCLEUS

Question: 1
In cytology the nucleus is studied to determine

Answer Choices:
A The function of the cell
B The health of the cell
C The origin of the cell
D How differentiated an abnormal cell is
B The health of the cell

Classically, the cell's nucleus is thought to show the health of the cell. The earliest sign of cell
death is considered to be smudging of the nuclear chromatin around the cell membrane and
with increasing deterioration the entire nucleus develops a homogeneously smudged dark
purple color.
As a cell becomes more premalignant or malignant, its nucleus accumulates more and more
abnormal copies of its DNA. As the amount of DNA increases, the nucleus becomes larger, more
basophilic and more irregular.
Question:2
The nuclear membrane is

Answer Choices:
A A single unit membrane
B Formed from the rough endoplasmic reticulum after cell division
C Visible on light microscopy
D Continuous with the outer cell membrane
E Without interruptions along the nuclear surface
B Formed from the rough endoplasmic reticulum after cell division

The nuclear membrane is a double unit membrane interrupted along its surface by nuclear
pores, which allow molecules to move between the nucleus and cytoplasm. The area between
the 2 layers of membrane is called the perinuclear cisternal space or nuclear cisterna and is
continuous with the cisternal space of the rough endoplasmic reticulum (rER). The outer
nuclear membrane is itself continuous with the membrane of the rough endoplasmic reticulum
and the cytoplasmic surface of the outer nuclear membrane can be lined by ribosomes, as is the
rER. When cell division is completed, it is the rough endoplasmic reticulum that provides the
material for the new nuclear membrane.
The nuclear membrane is not visible on light microscopy. What many microscopists refer to as
the nuclear membrane is in reality nuclear chromatin marginated along the nuclear membrane.
The nuclear cisterna is usually not visible on light microscopy either, but certain insults, like
radiation, may cause the space between the membrane layers to enlarge and form a vacuole.
These vacuoles may be visible on light microscopy.

Question: 3
The purpose of the nucleolus is to

Answer Choices:
A Synthesize ribosomal subunits
B Synthesize protein
C Synthesize DNA
D Synthesize messenger RNA
E Replicate viral DNA
A Synthesize ribosomal subunits

The nucleolus is the site of synthesis of ribosomal subunits. The subunits are then transported
through the nuclear pores and assembled in the cytoplasm. Because the purpose of ribosomes
is to assemble proteins guided by a strand of messenger RNA, the nucleolus is very large in cells
that are producing large amounts of protein and in many malignant cells.

Question: 4
The major function of the nucleolus is to

Answer Choices:
A Synthesize DNA
B Synthesize ribosomal RNA
C Repair DNA
D Transport RNA into the nucleus
E Synthesize mRNA
B Synthesize ribosomal RNA

The nucleolus is a large distinct structure within the nucleus of the cell. Its main function is in
ribosome synthesis and it is the site of ribosomal RNA synthesis. Ribosomal proteins,
synthesized in the cytoplasm, enter the nucleolus and are assembled with the rRNA to form the
immature ribosomal subunits. The transport and activation of the ribosomal subunits to the
cytoplasm creates functional ribosomes. The nucleolus is a highly organized structure that is not
bounded by a membrane. Its size depends on the level of activity within it, the larger the size,
the more ribosomal RNA gene activity. The area of DNA not actively being transcribed shows in
electron micrographs as a pale staining region called the fibrillar center. The dense fibrillar
component contains RNA molecules being synthesized and the granular component contains
the maturing ribosomal particles. The synthesis of mRNA and the replication and repair of DNA
occur in the nucleus not the nucleolus.

Question:5
The dissolution of the nuclear envelope at the end of prophase is the result of the

Answer Choices:
A Action of proteolytic enzymes
B Phosphorylation of nuclear lamins
C Dephosphorylation of specific protein kinases
D Condensation of the chromosomes
E Action of glycosidases

B Phosphorylation of nuclear lamins

Prophase marks the beginning of mitosis and is characterized by the appearance of condensed
chromosomes and the development of mitotic spindle. The nuclear envelope breaks down at
the end of prophase in most cells. The activation of the protein kinase cdc2 signals the entry
into mitosis. The nuclear lamina consists of a meshwork of lamin filaments. The
depolymerization of the nuclear lamina occurs as a result of the phosphorylation of lamins by
cdc2 and other protein kinases allowing the lamin dimers to dissociate from each other. The
nuclear membrane then fragments into vesicles. Inactivation of cdc2 at the end of mitosis
allows the nuclear envelope to reform.

Question: 6
After being produced, melanin pigments are taken up by the

Answer Choices:
A Langerhans cells
B Melanocytes
C Merkel cells
D Keratinocytes
E Dendritic cells

D Keratinocytes

The keratinocytes deposit the melanin granules on the superficial side of their nuclei. This is
thought to protect the nuclei from UV radiation by absorbing it before it can reach the nuclei.

Question: 7
Large nuclear proteins, such as the DNA and RNA polymerases, end up in the nucleus because they

Answer Choices:
A Are synthesized on ribosomes found in the nucleus
B Diffuse into the nucleus
C Contain a short peptide sequence which directs them to the nucleus
D Contain a signal sequence which attaches polysomes to the nucleus
E Contain covalently attached lipids

C Contain a short peptide sequence which directs them to the nucleus

All nuclear proteins are synthesized in the cytosol on free ribosomes. Small proteins up to
approximately 15 kDa, such as histones, can readily diffuse into the selective openings in the
nuclear envelope called nuclear pores. Larger proteins (>90 kDa) cannot diffuse into the
nucleus. They must contain a specific sequence of amino acids called the nuclear localization
sequence. If a small protein has a nuclear localization sequence it will be transported more
quickly into the nucleus. The nuclear localization sequence is a stretch of 4-8 amino acids which
can vary between different proteins and can be located anywhere within the protein. The
sequence is rich in arginine and lysine and usually contains a proline. If the sequence is
mutated, the protein will accumulate in the cytoplasm. If a nuclear localization sequence in
added to a cytoplasmic protein, this protein will end up in the nucleus.

Question: 8
An individual who exhibits the characteristics of Down's syndrome is

Answer Choices:
A Aneuploid for chromosome 21
B Aneuploid for the X chromosome
C Monoploid for chromosome 21
D Monoploid for Y chromosome
A Aneuploid for chromosome 21

Aneuploid is the term applied to an abnormal number of chromosomes in the genome. In


Down's syndrome, the person has three of number 21 chromosomes (= trisomy 21), rather than
the normal pair. The number and shape of X and Y chromosomes are normal.

Question: 9
Nuclear pores

Answer Choices:
A Tend to be found in areas where the DNA is particularly active
B Rely on diffusion to move molecules between the nucleus and the cytoplasm
C Apparently are not associated with any specialized structures
D Are associated only with ATP-mediated transport
E Do not have any known ability to regulate the movement of material between the
nucleus and cytoplasm

A Tend to be found in areas where the DNA is particularly active

Nuclear pores have a tendency to be found in areas in which DNA is active. The pores are
characterized by an octagonal protein formation called the nuclear pore complex. The nuclear
pore complexes have receptor sites for specific molecules that allow the complexes to actively
transport certain molecules across the nuclear membrane. This is a process that requires ATP.
Molecules less than 9 nm in diameter can also cross the nuclear pores by simple diffusion.
Question: 10
A Barr body is

Answer Choices:
A A crystalloid lysosomal inclusion
B An acidophilic, degenerated hepatocyte
C An indicator of a patient's genetic make-up
D The hallmark of an always-lethal genetic abnormality
E Found only in the extracellular space

C An indicator of a patient's genetic make-up

A Barr body is the condensed, inactive X chromosome found in genetic females. (The second X
chromosome is active.) It appears as a rectangular or triangular dark staining area along the
nuclear membrane. To determine the presence or absence of Barr bodies, a smear is made of
cells scraped off the inner surface of a patient's cheek (buccal smear). In Turner's syndrome
(XO) and in genetic males, Barr bodies are not present. In XXX disease, too many Barr bodies
are seen. Malignant cells may also have too many Barr bodies.

Question: 11
Human somatic cells have

Answer Choices:
A 46 chromosomes
B 23 chromosomes
C 42 autosomes and 2 sex chromosomes
D 36 chromosomes
E 48 chromosomes

A 46 chromosomes

Human somatic cells have 46 chromosomes (23 homologous pairs), 2 of which are sex
chromosomes and 44 of which are autosomes. Each chromosome is made up of 2 chromatids,
which are joined together by a centromere. The autosomes are designated by a number from 1
to 22, and a normal human has 2 chromosomes of each number. The sex chromosomes are
designated either X or Y. Males have 1 X and 1 Y chromosome; females have 2 X chromosomes.
Egg and sperm cells have only 23 chromosomes, the haploid chromosome number, because
they have undergone meiosis. After the sperm fertilizes the egg, the fertilized egg regains the
diploid (46) chromosome number.

Question: 12
The term chromatin refers to

Answer Choices:
A Nuclear DNA only
B Nuclear DNA and histone proteins
C All nuclear material
D All nuclear material that stains with basic dyes
E Complexes of DNA, histone proteins, and some non-histone proteins

E Complexes of DNA, histone proteins, and some non-histone proteins

Chromatin refers to complexes formed between DNA strands, basic histone proteins, and
certain acidic non-histone proteins. A small amount of RNA may also be included. There are five
types of histone proteins - their function is to protect and package the DNA. There are many
kinds of non-histone proteins with many different functions. Some of their functions include
acting as initiators and controllers of DNA transcription.

Question: 13
The nuclear lamina

Answer Choices:
A Extends throughout the cell's nucleus
B Is primarily formed from microtubules
C Supports the chromatin, nuclear membranes and nuclear pores
D Is identifiable on light microscopy
E Appears only in the region surrounding the nucleolus
C Supports the chromatin, nuclear membranes and nuclear pores

The nuclear lamina is a network of intermediate filaments arranged between the nuclear
membrane and the marginated nuclear chromatin. It consists of nuclear lamins and supports
the surrounding chromatin, the nuclear membranes and the nuclear pores. Like the nuclear
membranes, it is not visible on light microscopy but is outlined by the adjacent marginated
chromatin, which stains well with hematoxylin.
Question: 14
Large nucleoli within a cell's nucleus indicate

Answer Choices:
A That the cell is engaged in active protein synthesis
B That the cell is malignant
C That there is an approximately 50% chance that the cell is malignant
D That the cell is engaged in active DNA synthesis
A That the cell is engaged in active protein synthesis

Nucleoli are the centers in the nucleus where ribosomal RNA is made. Because rRNA is used in
the cytoplasm to transcribe messenger RNA into protein, large nucleoli are indicative of
increased protein synthesis. (rRNA does not produce protein while it is still in the nucleus.)
Many malignant cells are characterized by very large nucleoli and by very abnormal numbers
and shapes of nucleoli. This is partially due to increased protein synthesis, but can also be due
to the removal of normal restraints on the number of nucleoli in one nucleus and to the
increased number of chromosomes in some malignant cells.
The presence of an enlarged nucleolus alone does not usually indicate malignancy.
Question: 15
A nucleosome is

Answer Choices:
A A DNA packaging unit found only in chromosomes during mitosis
B A DNA packaging unit found only in heterochromatin
C A DNA packaging unit found only in euchromatin
D The smallest unit of chromatin
E A complex of DNA and transcribed RNA

D The smallest unit of chromatin

A nucleosome is the smallest unit of chromatin. It consists of a strand of DNA wrapped twice
around a complex made up of eight histone molecules. Nucleosomes are present along DNA
strands at approximately 1.5 nm intervals and give the appearance of "beads on a string".
The strands of DNA containing nucleosomes are then coiled to form fibrils, which are the
packing units for both chromosomes and interphase chromatin. In euchromatin the coiling is
loose and in heterochromatin the coiling is tight.

Question: 16
The DNA in a cell's nucleus contains
Answer Choices:
A All the genetic information needed to generate any cell in any organism
B All the genetic information needed to generate any cell in its particular organism
C Only the genetic information needed to generate another cell in its particular organ
system
D Only the genetic information needed to generate another cell of its particular
histologic type, e.g. endothelial cell, squamous cell, glandular cell, etc
E Only the genetic information necessary to regenerate an exact copy of itself

B All the genetic information needed to generate any cell in its particular organism

The most outstanding feature of the cell's nucleus is that it contains a copy of all the organism's
DNA - all the structural information the organism and its ancestors have accumulated
throughout evolution. In normal cells only the DNA needed to make one type of cell is "read",
e.g. heart muscle cells only read the heart muscle DNA, kidney tubule cells only read kidney
tubule DNA, but in diseased states, parts of the DNA not normally expressed may be read.

Question: 17
Ribosomal proteins and histones synthesized in the cytoplasm have been demonstrated to be
incorporated into ribosomes in the nucleolus, or utilized within the nucleus, but without nuclear
protein synthesis. Transport of large molecular size materials, such as these proteins through the
nuclear pore complex occurs by what process

Answer Choices:
A Simple diffusion
B Selective transport
C Active transport
D Passive diffusion
E Microvesicular transport

C Active transport

The nuclear pores are the result of the union of the inner and outer membranes of the nuclear
envelope. Using transmission electron microscopy, these nuclear pores are seen as openings in
the envelope and possess eight protein subunits arranged in an octagonal array at the
periphery of the pore. This array consists of complex proteins referred to as nucleoporins.
There are "binding sites" associated with the pore complex that appear to be somewhat
specific and "bind" in some manner large molecules such as ribosomal proteins or histones,
which will be transported into the nucleus. Transportation of these large molecules occurs by
an ATP-dependent transport mechanism, and thus are actively transported through the pore.

Case #301454:
A mother brings her 7-year-old son to your office because of limited growth, baldness, and
thick wrinkled skin. He has a history of prenatal growth retardation and the failure to thrive.
On physical examination, you find short stature and low weight for height child with
characteristic “plucked-bird" appearance with small face and jaw, pinched nose, and diffuse
signs of accelerated aging, including hypodontia, hearing loss, hypertension, cardiomyopathy,
lipodistrophy, and atherosclerotic changes on the fundus oculi. His intelligence is normal.

Question: 18
You conclude the patient has a rare disease caused by the disturbance in what?

Answer Choices:
A DNA repair
B Nuclear lamins
C Lysosomal storage
D Mitochondrial function
E DNA damage

B Nuclear lamins

The patient has progeria, a rare disease caused by disturbance in nuclear lamins. Noxious factor
in this child was obviously present before the birth (prenatal growth failure), and it is still active
and affects almost all tissues. That implies the early presence of a genetic defect that affects all
cells. The only such known defect is caused by a mutation in the gene for lamin A, a filament
protein that forms a scaffold underneath nuclear envelope and connects nuclear matrix and
nuclear lamina. Laminin is a part of the building block of the nuclear membrane, necessary for
the nuclear envelope stability and required for nuclear DNA replication and nuclear
organization.

Accelerated aging of varying degrees can be seen in most DNA repair deficiency diseases.
However, they start after the birth and display different aspects of aging - never every aspect.
There are usually other specific symptoms and signs, as well as higher risk of developing cancer
and neurodegeneration. Examples are Ataxia teleangiectasia, Bloom syndrome, Werner
syndrome, Xeroderma pigmentosum, etc.

Defective or missing lysosomal protein will lead to the defective recycling and accumulation of
substrates within the cell. The distribution of the accumulating material will determine which
organs are affected and what will be the symptoms of the disease. Besides selective organ
impairment, lysosomal storage diseases cause metabolic and neurological disturbances this
child obviously does not have. Examples of these disturbances are sphingolipidoses,
glycoproteinoses, mucolipidoses, mucopolysaccharidoses, and others.

According to the role of mitochondria in human organism, mitochondrial dysfunction will


manifest on the organs that have the greatest need for energy (brain, muscle, heart, glands,
etc). Skin is not affected. Mitochondrial dysfunction is implicated in the aging process, but it is
not the main cause of aging nor is aging accelerated.

DNA damage can be due to the environmental factors and normal metabolic processes. It can
be recognized by enzymes, can be correctly repaired or retained and translated into a protein,
or it can cause a cell death. If translated into a protein, it will cause a disease connected with
the function of that specific protein. Neither is the case in this patient.

Keywords: Intracellular sorting, Aging

SECTION _ CELL DIVISION

Question: 1

The cell in the attached photomicrographic image displayed between the two arrows is best
described as being in what stage of cell division?

Answer Choices:
A Interphase
B Prophase
C Metaphase
D Anaphase
E Telophase

Image(s) / Chart(s):
Click image to view full size. Click open image to close. Click and hold open image to move.
B Prophase

Mitosis is a process in which two daughter cells are produced with the same chromosomal
number and DNA content as the original cell. Prior to entering mitosis, DNA is duplicated within
the original cell during the S or synthesis phase. By the end of the synthesis phase, the
chromosomal number is doubled (4n) and the DNA content is doubled (4n). With the onset of
the process of division (mitosis), the chromosomes condense and become visible with a light
microscope. The nuclear envelope also disintegrates during this phase, and the condensed
chromosomes frequently "clump" together in a random arrangement, at times appearing like
an asterisk, as in this example. The arrow is pointing at a cell that is in the prophase stage.
Question: 2
Nuclear events associated with meiosis during the S-phase results in DNA content which is best
described as

Answer Choices:
A 1n
B 2n
C 3n
D 4n
E Two "daughter" cells, each with 2n

D 4n

During the cell cycle, the S phase precedes cell division and is a period of DNA synthesis in the
nucleus. This period, which lasts 6-8 hrs, will result in a doubling of the DNA, forming new
chromatids. Cells entering this "synthesis" phase have a 2n chromosome number, and a DNA
content of 2n. At the end of this phase, both the chromosome number AND the DNA content is
4n. Following the S phase, cells are ready to enter either mitosis to form two daughter cells
each with DNA content of 2n, or enter the first meiotic (reduction) division, ultimately resulting
in the formation of gametes containing 1n DNA.
Question: 3
In the first meiotic division

Answer Choices:
A The maternal and paternal chromosomes are separated from each other
B The chromatids are separated from each other
C The chromosome number is reduced from diploid to haploid
D The chromosome number remains constant
E The centromeres split as the chromosomes are pulled apart

A The maternal and paternal chromosomes are separated from each other

The first meiotic division is preceded by an S phase during which the DNA content is raised from
diploid to tetraploid. During the first meiotic division, the chromosome number is reduced from
tetraploid to diploid. This is accomplished by separating the maternal chromosomes from the
paternal chromosomes. During the second meiotic division, the chromosome number is
reduced further, from diploid to haploid, by separating the chromatids from each other at the
centromere.

Question: 4
During the telophase I of the meiotic stage of the human ovum

Answer Choices:
A Each daughter cell contains 23 chromosomes (= the haploid number ) but a 2CDNA
content (= the diploid amount). Each chromosome at this stage is composed of two sister
chromatids, which are similar but not genetically identical
B The chromosomes continue to condense and chiasmata are observed, indicating sites
where crossing over has taken place
C Homologous maternal and paternal chromosomes pair and make physical contact (=
synapsis ) via the synaptonemal complex, forming a tetrad
D Chiasmata are formed and crossing over (= random exchanging of genes between
segments of homologous chromosomes) occurs; an event that is crucial for increasing
genetic diversity
E Nucleoli disappear, chromosomes are condensed maximally, and the nuclear envelope
disappears
A Each daughter cell contains 23 chromosomes (= the haploid number ) but a 2CDNA content
(= the diploid amount). Each chromosome at this stage is composed of two sister chromatids,
which are similar but not genetically identical
Telophase I in the meiotic cycle is similar to the telophase of the mitotic cycle. The phase
includes reformation of the nucleus and cytokinesis, forming two daughter cells. At this phase,
each daughter cell contains 23 chromosomes (= the haploid number ) but a 2CDNA content (=
the diploid amount ). Each chromosome at this stage is composed of two sister chromatids,
which are similar but not genetically identical.
Answers (B), (C), (D) and (E) describe the stages diplotene, zygotene, pachytene and diakinesis
of the prophase I of meiosis I, respectively.

Question: 5
During prophase of mitosis, the chromosomes are found

Answer Choices:
A Condensed, and the mitotic spindle is forming
B Aligned at the center of the cell
C Scattered through out the nucleus
D Decondensed in the reformed nuclei
E Separated and moving to opposite poles of the spindle

A Condensed, and the mitotic spindle is forming

Mitosis is the division of the nucleus which is followed by cytokinesis; cell division to give two
diploid daughter cells. Meiosis is the division of diploid cells to produce haploid cells. Prophase
is the beginning of mitosis at which time the chromosomes appear condensed and the mitotic
spindle is forming. Metaphase is the stage where the chromosomes are aligned at the
metaphase plate in the center of the cell. The sister chromatids separate and move to opposite
poles of the spindle during anaphase. At telophase, the final stage of mitosis, the nuclei reform
and the chromosomes decondense. This is followed by the division of the cell called cytokinesis.
Leptotene is the first phase of meiosis in which the homologous chromosomes pair before
condensation.

Question: 6
Early cleavage of the fertilized egg in most species is under the control of

Answer Choices:
A The sperm DNA
B Proteins derived from the sperm membrane
C The maternal mitochondrial DNA
D Newly synthesized RNA
E Stored maternal RNA
E Stored maternal RNA
In most species, early cleavage of the fertilized egg is under the control of proteins and mRNA
stored in the egg. The mammalian genome is activated during early cleavages. If the fertilized
egg is experimentally treated with drugs that inhibit transcription, early cell division is not
inhibited. During the early stages of cell division, the amount of cytoplasm decreases relative to
the nuclear volume. New transcription is activated at precise times in the embryo. In the mouse
and goat, this occurs at the 2 cell stage.

Question: 7
During anaphase of mitosis, the chromosomes are found

Answer Choices:
A Condensed, and the mitotic spindle is forming
B Aligned at the center of the cell
C Scattered through out the nucleus
D Decondensed in the reformed nuclei
E Separated and moving to opposite poles of the spindle
E Separated and moving to opposite poles of the spindle

Mitosis is the division of the nucleus which is followed by cytokinesis; cell division to give two
diploid daughter cells. Meiosis is the division of diploid cells to produce haploid cells. Prophase
is the beginning of mitosis at which time the chromosomes appear condensed and the mitotic
spindle is forming. Metaphase is the stage where the chromosomes are aligned at the
metaphase plate in the center of the cell. The sister chromatids separate and move to opposite
poles of the spindle during anaphase. At telophase, the final stage of mitosis, the nuclei reform
and the chromosomes decondense. This is followed by the division of the cell called cytokinesis.
Leptotene is the first phase of meiosis in which the homologous chromosomes pair before
condensation.
Question: 8
The stage of mitosis in which the chromosomes separate is called

Answer Choices:
A Metaphase
B Anaphase
C Prophase
D Telophase
E Leptotene

B Anaphase
Mitosis is the division of the nucleus which is followed by cytokinesis; cell division to give two
diploid daughter cells. Meiosis is the division of diploid cells to produce haploid cells. Prophase
is the beginning of mitosis at which time the chromosomes appear condensed and the mitotic
spindle is forming. Metaphase is the stage where the chromosomes are aligned at the
metaphase plate in the center of the cell. The sister chromatids separate and move to opposite
poles of the spindle during anaphase. At telophase, the final stage of mitosis, the nuclei reform
and the chromosomes decondense. This is followed by the division of the cell called cytokinesis.
Leptotene is the first phase of meiosis in which the homologous chromosomes pair before
condensation.
Question: 9
Metaphase, one of the six phases of cell division, is characterized by

Answer Choices:
A Formation of a cleavage furrow
B Dissolution of the nuclear envelope
C Alignment of the chromosomes halfway between the spindle poles
D Migration of chromosomes toward the spindle poles
E Condensation of chromatin into well defined chromosomes

C Alignment of the chromosomes halfway between the spindle poles


Image(s) / Chart(s):
Click image to view full size. Click open image to close. Click and hold open image to move.

Prophase: During prophase, the chromatin slowly condenses into well-defined chromosomes.
While the chromosomes are condensing, the nucleolus disassembles and disappears. Each
chromosome consists of two sister chromatids joined at a specific point known as the
centromere. During prophase, cytoplasmic tubulin polymerizes to form the microtubules of the
mitotic spindle, with the centriole as the focus for spindle formation. The original pair
replicates, and each pair becomes part of a mitotic center that forms the focus of the radial
array of microtubules, the aster. The two asters initially lie adjacent to each other, and during
prophase the microtubules elongate and push the two centers apart.
Prometaphase: Prometaphase begins with the disruption of the nuclear envelope, allowing the
spindle to enter the nuclear area. Kinetochores develop on either face of the centromeres and
become attached to a special set of microtubules, known as the kinetochore fibers. These fibers
extend in opposite directions from both sides of the chromosome and interact with the fibers
of the bipolar spindle. The chromosomes begin moving due to the interactions of their
kinetochore fibers with components of the spindle.
Metaphase: The chromosomes become arranged so that their centromeres all lie in a single
plane. Each chromosome is held at the metaphase plate by the paired kinetochores and the
associated fibers pointing to opposite poles of the spindle.
Anaphase: Anaphase, which typically lasts only a few minutes, begins abruptly as the paired
kinetochores on each chromosome separate, allowing each chromatid to be pulled slowly
toward a spindle pole. During anaphase, the kinetochore fibers shorten as the chromosomes
approach the poles. At the same time, the spindle fibers elongate and the poles of the spindles
move further apart.
Telophase: As the daughter chromatids arrive at the poles, the kinetochore fibers disappear.
The polar fibers continue to elongate, and the nuclear envelope re-forms around each group of
the daughter chromatids. The chromatin expands and the nucleoli begin to reappear. This is the
end of mitosis.
Cytokinesis: Cleavage, the process of cytoplasmic division, usually starts during late anaphase or
telophase. The membrane around the middle of the cell is drawn inward to form the cleavage
furrow, which deepens until it reaches the remains of the mitotic spindle between the two
nuclei. This narrow bridge, the midbody, finally breaks at each end, leaving completed,
separated daughter cells.

Question: 10
What is a synaptonemal complex?

Answer Choices:
A A structure found along the internal surface of a neuron's synapse
B A structure found along the external surface of a neuron's synapse
C A structure holding two homologous pairs of chromosomes together
D A specialized form of junctional complex
E A structure that allows an electrical impulse to travel between two cells

C A structure holding two homologous pairs of chromosomes together

A synaptonemal complex is found only in meiosis, never in mitosis. It holds two pairs of
homologous chromosomes together so that crossing-over, the exchange of DNA between
different chromosomes, can occur.

Question: 11
Histology can distinguish two phases of the cell cycle, interphase and mitosis. These phases can be
further distinguished into G1, S, and G2 as components of interphase; and prophase, metaphase,
anaphase, and telophase as components of mitosis. A long G1 phase is considered a G0 phase, which
can last for days, weeks, or years. Meiosis is a type of specialized cell division that results in the
formation of haploid cells containing one copy of the genome. How does interphase differs in meiosis
from mitosis?
Answer Choices:
A The G1 phase is absent in meiosis
B Mitosis has a short S phase compared to meiosis
C Meiosis II, the second mitotic division has a long prophase II
D S phase is not present in meiosis
E The G2 phase is prolonged in meiosis
D S phase is not present in meiosis

The S phase represents the DNA synthesis phase. Meiosis has no S phase. G1 and G2 phases as
components of interphase at the end of meiosis I are brief. Prophase II is brief in meiosis.

Question: 12
Cytokinesis is defined as the stage at which the

Answer Choices:
A Two daughter cells are formed
B Two nuclei reform
C Chromosomes decondense
D Homologous chromosomes pair and condense
E Mitotic spindle forms
A Two daughter cells are formed

Mitosis is the division of the nucleus which is followed by cytokinesis; cell division to give two
diploid daughter cells. Meiosis is the division of diploid cells to produce haploid cells. Prophase
is the beginning of mitosis at which time the chromosomes appear condensed and the mitotic
spindle is forming. Metaphase is the stage where the chromosomes are aligned at the
metaphase plate in the center of the cell. The sister chromatids separate and move to opposite
poles of the spindle during anaphase. At telophase, the final stage of mitosis, the nuclei reform
and the chromosomes decondense. This is followed by the division of the cell called cytokinesis.
Leptotene is the first phase of meiosis in which the homologous chromosomes pair before
condensation.

Question: 13
DNA (= deoxyribonucleic acid) is duplicated in the cell cycle during the

Answer Choices:
A G2 phase
B S phase
C Leptotene phase
D G1 phase
E Go state
B S phase

The S (= synthesis) phase of cell cycle is the period when DNA replication and histone synthesis
occur, resulting in duplication of the chromosomes. At the end of the S phase , each
chromosome consists of two identical chromatids attached to one another at the centromere.
The cell is assumed to be in G o state when it is "resting" and "not dividing" (the cycle is
temporarily suspended). G 1 phase (of interphase ) is the gap phase just after mitosis during
which cell growth and protein synthesis occur, restoring the daughter cells to normal volume
and size. G 2 phase (of the interphase ) is the beginning of the mitotic cycle, and involves, in five
major stages, division of the nucleus (= karyokinesis ) and cytoplasm (= cytokinesis ), resulting in
the production of two identical daughter cells.
Leptotene phase (the first part of the Prophase I) occurs in meiosis (= reduction division) and
not in mitosis. During leptotene phase the chromatin condenses into the visible chromosomes,
each of which contains two chromatids joined at the centromere.

Question: 14
At the end of a mitotic division each daughter cell contains

Answer Choices:
A A haploid number of chromosomes
B A unique genetic makeup
C An equal amount of cytoplasm and cytoplasmic organelles
D Two pairs of centrioles
E One copy of the newly replicated DNA and one copy of the cell's original DNA
E One copy of the newly replicated DNA and one copy of the cell's original DNA

The nature of the mitotic division is such that it leaves each daughter cell with one copy of the
newly replicated DNA and one copy of the cell's original DNA (a diploid complement). Both
daughter cells have genetic make-ups which are identical to that of the parent cell, in contrast
with the cells produced by meiotic divisions, which are all genetically unique.
The division of cytoplasm in mitosis is not precise and the cytoplasmic material is not
necessarily evenly divided. Only one centriole is present in each daughter cell.

Question: 15
Which of the following statements is true of meiosis?
Answer Choices:
A In the male, meiosis produces four structurally and genetically identical spermatids
B In the female, meiosis produces four structurally identical but genetically different
oocytes
C In the female, the two meiotic divisions produce only one oocyte
D In the male, meiosis produces two genetically different spermatids
E In both the male and the female, meiosis produces four structurally identical but
genetically different gametes
C In the female, the two meiotic divisions produce only one oocyte

In the male, meiosis produces four structurally identical spermatids, but each spermatid has a
different genetic composition. The different genetic composition is a result of crossing-over and
of the random way the members of each chromosome pair are drawn into their respective
cells.
In females, the nuclear divisions of meiosis are the same as in the male, but the cytoplasmic
divisions differ. In the female, most of the cytoplasm remains with one of the gametes, the
oocyte, and the other three cells formed are referred to as polar bodies. These polar bodies
eventually disintegrate.

Question: 16
The earliest readily appreciable event in cell division is

Answer Choices:
A Dissolution of the nuclear membrane
B Condensation of the chromosomes
C Formation of the mitotic spindle
D Division of the mitochondria
E Replication of the centrioles
E Replication of the centrioles

Replication of the centrioles occurs during the S phase of the cell cycle, when the DNA is also
being replicated. This is the earliest observable sign that the cell is going to divide. After the
centriole has replicated each centriole moves to one pole of the nucleus and the mitotic spindle
begins to form as microtubules become anchored to the centrioles.
Question: 17
Prophase is characterized by which event?

Answer Choices:
A Alignment of the chromosomes along the middle of the cell
B DNA replication
C Separation of the chromatids
D The chromosomes become visible
E Re-establishment of the nuclear membrane

D The chromosomes become visible

Prophase is the first stage of mitosis. During prophase the chromosomes start to condense and
become visible, the nuclear membrane dissolves and the nucleolus disappears. During
metaphase the mitotic spindle becomes visible and the chromosomes form a line across the
middle of the cell. In anaphase the chromatids are pulled away from each other and in
telophase a nuclear membrane reappears around each set of chromosomes and the cytoplasm
divides.

Question: 18
The DNA content of the cell located between the two arrows in the attached photomicrographic
image is

Answer Choices:
A 1n
B 2n
C 3n
D 4n
E 6n

Image(s) / Chart(s):
Click image to view full size. Click open image to close. Click and hold open image to move.
D 4n

Mitosis is a process in which two daughter cells are produced with the same chromosomal
number and DNA content as the original cell. Prior to entering mitosis, DNA is duplicated within
the original cell during the S or synthesis phase. By the end of the synthesis phase, the
chromosomal number is doubled (4n) and the DNA content is doubled (4n). With the onset of
the process of division (mitosis), the chromosomes condense and become visible with a light
microscope. The nuclear envelope also disintegrates during this phase, and the condensed
chromosomes frequently "clump" together in a random arrangement, at times appearing like
an asterisk, as in this example. The arrow is pointing at a cell that is in the prophase stage and
both the chromosomal number and DNA content are doubled (4n).

2 . EPITHELIUM QUESTIONS
Question: 1
The epithelial type as seen in this photomicroscopic image is best classified as

Answer Choices:
A Simple squamous epithelium
B Stratified squamous epithelium, keratinized
C Pseudostratified columnar epithelium
D Transitional epithelium
E Stratified squamous epithelium, non-keratinized

Image(s) / Chart(s):
Click image to view full size. Click open image to close. Click and hold open image to move.
E Stratified squamous epithelium, non-keratinized

Classification of epithelial types is descriptive and relates to the number of cell layers,
and the shape of the surface cell (luminal) cell that can be seen. The surface cell as
seen on this image suggests from the shape of the nuclei and the "apparent" shape of
the cell, that the cell is wider than it is thick, being rather "flat" and therefore, squamous.
Also, there are multiple layers of cells within the epithelium, which indicates this is a
stratified epithelium. Finally, careful examination of the surface cells reveal the presence
of nuclei, indicating this is a non-keratinized epithelium. When all characteristics are
considered, the appropriate conclusion is that this epithelium is stratified squamous,
non-keratinized epithelium.

Question: 2
Which of the following cells produce Type IV collagen?

Answer Choices:
A Chondroblast
B Epithelial cell
C Fibroblast
D Megakaryocyte
E Plasma cell
B Epithelial cell

While many collagen types are produced by the fibroblast, the fibroblast does not
produce Type IV collagen. Type IV collagen is produced by epithelial cells and
podocytes in the kidney glomeruli, to form an integral component of the basement
membrane. Chondroblasts, cells that produce cartilage, produce Type II collagen, and
fibroblasts typically produce Type I, or Type III collagen fibers. Plasma cells principally
produce antibodies, and as such, are not collagen producers. The megakaryocyte,
located in the bone marrow, produces platelets important in the process of blood clotting
mechanisms.

Question: 3
Simple squamous epithelium

Answer Choices:
A Consists of one to two layers of epithelial cells whose lateral sides are shorter
than their apical sides
B Consists of one to two layers of epithelial cells that produce keratin
C Consists of only one layer of epithelial cells whose lateral sides are shorter
than their apical sides
D Consists of only one layer of epithelial cells that produce keratin
E Consists of only one layer of epithelial cells that do not produce extracellular
keratin
C Consists of only one layer of epithelial cells whose lateral sides are shorter than their
apical sides

Simple squamous epithelial consists of one layer of cells whose lateral sides are shorter
than their apical surfaces. It is very important to realize that, in this context, the term
"simple squamous" depends only on the histological appearance of the epithelium.
Functional and immunocytochemical characteristics are irrelevant to this particular
classification.
Confusion may result with the term "squamous" because in this context it refers to any
flat epithelial cell, including mesothelial cells, endothelial cells and the cells lining
Bowman's space in the kidney. In more common histologic/pathologic contexts
"squamous" refers to cells with a particular function (keratinization) or
immunocytochemical profile. Cells like the endothelial cells, mesothelial cells and
Bowman's capsule cells will frequently not be included as "squamous" cells in these
discussions.

Question: 4
Which one of the following tissues is lined with simple cuboidal epithelium?

Answer Choices:
A Small intestine
B Trachea
C Lung alveoli
D The distal convoluted renal tubules
E The renal corpuscle
D The distal convoluted renal tubules
Image(s) / Chart(s):
Click image to view full size. Click open image to close. Click and hold open image to
move.

Small intestines are lined by simple columnar epithelium, trachea by pseudostratified


ciliated, and both lung alveoli and the renal corpuscle (= Bowman's capsule) by simple
squamous epithelium.

Question: 5
The epithelium illustrated in the attached photomicroscopic image is best classified as

Answer Choices:
A Simple cuboidal with microvilli
B Simple columnar with microvilli
C Simple columnar with cilia
D Pseudostratified columnar with cilia
E Stratified columnar with microvilli

Image(s) / Chart(s):
Click image to view full size. Click open image to close. Click and hold open image to move.
B Simple columnar with microvilli

The epithelium depicted in this photomicroscopic image is classified as simple columnar


epithelium with microvilli since the cells are typically tall and narrow, and each cell rests
on a basement membrane. Note that the nuclei are in a fairly even row and define,
somewhat the width of the cell. The apical surface is modified and shows microvilli
(brush border), and can be seen as a hazy covering on the free surface of the
epithelium. The image also illustrates the presence of single cell secretory units, the
goblet cells. These cells secrete a mucin substance which does not take up stain with
H&E stains. This epithelium is typical of that found throughout the G.I. tract, particularly
of the small intestine.

Question: 6
While working at a construction site, a 35-year-old man sustains a full-thickness burn over the dorsum
of his right arm. He is given first aid by his coworkers and immediately rushed to the emergency room.
The treatment, he is told, will involve a skin graft. The attending doctor explains the procedure to him
and tells him that a local flap of normal skin and subcutaneous tissue will be rotated into the defect.
This skin will survive because of small, horizontally oriented vessels in which layer?

Answer Choices:
A Stratum corneum
B Stratum lucidum
C Stratum granulosum
D Stratum spinosum
E Stratum basale
F Deep dermis
G Subcutaneous fat
F Deep dermis
The dermis is composed of dense connective tissue rich in blood vessels and
lymphatics. The blood vessels in the deep dermis are horizontally oriented and
communicate with those of the superficial dermis. The thickness of the dermis varies in
different parts of the body, being generally thicker on the dorsal aspect than the ventral.
It is also thicker in men than in women.
Stratum corneum, stratum lucidum, stratum granulosum, stratum spinosum and stratum
basale are all layers of the epidermis, which does not contain any blood vessels.
The subcutaneous fat does contain blood vessels, but these do not supply the skin.
They supply only the fat.

Question: 7
Epithelium of this type (see attached image) would commonly be found in which of the following
location?

Answer Choices:
A Epidermis
B Lining the trachea
C Lining the ureters
D Lining the small intestine
E Lining the kidney tubules

Image(s) / Chart(s):
Click image to view full size. Click open image to close. Click and hold open image to move.

E Lining the kidney tubules

In the case of the epithelium seen in this image, the cells surrounding the lumen are
classified as cuboidal. This is based on the impression that the cells are roughly as wide
as they are tall. Even though the intercellular boundaries cannot be visualized, the
apical and basal extent of the cells and the spacing of the nuclei give the impression
that these cells are probably "cuboidal" in shape. Furthermore, since there is only one
row (layer) of these cells, the epithelium can be classified as being "simple".
Combination of these characteristics allows the most appropriate classification of this
epithelium as simple cuboidal. This epithelial type is typical of the lining which occurs in
many of the tubules located in the kidney medulla, or lining small ducts of the exocrine
glands.

Question: 8
The cells observed in the attached photomicroscopic image, are best classified as what epithelial type,
based upon shape?

Answer Choices:
A Simple squamous
B Simple cuboidal
C Simple columnar
D Pseudostratified cuboidal
E Transitional

Image(s) / Chart(s):
Click image to view full size. Click open image to close. Click and hold open image to move.

B Simple cuboidal

The classification of epithelial types based on shape is very logical. In the case of the
epithelium seen in this image, the cells surrounding the lumen are classified as simple
cuboidal. This is based on the impression that the cells are, roughly, as wide as they are
tall. Even though the intercellular boundaries cannot be visualized, the apical and basal
extent of the cells, and the spacing of the nuclei, give the impression that these cells are
probably "cuboidal" in shape. Furthermore, since there is only one row (layer) of these
cells, the epithelium can be classified as being "simple". Combination of these
characteristics allows the most appropriate classification of this epithelium as simple
cuboidal.
Question: 9
The epithelium depicted in the attached photomicroscopic image would be found typically in which area
of the body?

Answer Choices:
A Bronchus
B Hard palate
C Jejunum
D Uterus
E Vas deferens

Image(s) / Chart(s):
Click image to view full size. Click open image to close. Click and hold open image to move.

C Jejunum

The epithelium depicted in this photomicroscopic image is classified as simple columnar


epithelium since the cells are typically tall and narrow, and each cell rests on a
basement membrane. Note that the nuclei are in a fairly even row and somewhat define
the width of the cell. The apical surface is modified and shows microvilli (brush border),
and can be seen as a hazy covering on the free surface of the epithelium. The image
also illustrates the presence of single cell secretory units, the goblet cells. These cells
secrete a mucin substance which does not take up stain with H&E stains. This
epithelium is typical of that found throughout areas of the G.I. tract, particularly of the
small intestine such as the jejunum. The bronchus is lined with a pseudostratified
columnar epithelium with goblet cells, as well, but with cilia. The hard palate is covered
by stratified squamous epithelium. The uterus is lined by simple cuboidal epithelium, but
goblet cells and microvilli are lacking. The vas deferens is lined with a pseudostratified
columnar epithelium and also lacks microvilli and goblet cells.
Question: 10
Epithelial tissues are labile structures whose cells are renewed continuously by means of mitotic
activity. This renewal rate is variable. It is slowest (renewal taking about 50 days) in the

Answer Choices:
A Intestinal epithelium
B Pancreas
C Stratum germinativum of skin
D Vagina
E Ureter

B Pancreas

The correct answer is the pancreas, as they are not subjected to much wear and tear,
have the slowest renewal rate of approximately about 50 days, while all other tissues
(intestinal epithelium, stratum germinativum of the skin, vagina, and ureter) in which the
cells are subjected to stress forces have a renewal rate of every 2-5 days.

Question: 11
A medical student suffering with chronic respiratory infection seeks the advice of an ear, nose, and
throat specialist. A biopsy of the student's respiratory epithelium reveals alteration in certain epithelial
structures. What is most likely to be abnormal?

Answer Choices:
A Microvilli
B Desmosomes
C Cilia
D Hemidesmosomes
E Stereocilia

C Cilia

A structural abnormality involving absence of dynein arms in cilia (Kartagener's


syndrome, a hereditary disease) in some individuals is associated with chronic
respiratory difficulty (including bronchitis and sinusitis). Ciliary motility is severely
impaired or absent in these individuals.
The epithelium in the respiratory passages does not possess microvilli. Desmosomes
(cell to cell junctions, attached to lateral sides of two cells) (= macula adherens) is the
only attachment device present in epidermal cells, while in other epithelia (especially
those with cuboidal or columnar cells) desmosomes are found in conjunction with a
zonula adherens. They are thought to play a role in dissipating physical forces
throughout the cell from the attachment site. Hemidesmosomes (present on basal
surface of the cells of the immune system) occur where epithelia require stronger
adhesion to the connective tissue, and are believed to confer a "social identity" onto
cells.
Stereocilia (long microvilli) are limited to the epididymis of the male reproductive
system. Stereocilia of the sensory epithelium of the ear are uniform in diameter and
possess an internal filamentous structure. They serve as a receptor device rather than
an absorptive structure.

Ques 12:
Glands

Answer Choices:
A Are always associated with ducts
B Are composed of specialized epithelial cells that both make and secrete a
particular product
C Have ducts made from epithelial cells whose only function is to release the
substance made by the glandular cells
D Do not produce hormones

B Are composed of specialized epithelial cells that both make and secrete a particular
product

Glands are made up of aggregates of specialized epithelial cells that both synthesize
and secrete a product. In exocrine glands the secreted product moves into ducts lined
by epithelial cells which may further modify the product. In endocrine glands, which
produce hormones and have no ducts, the hormone product is secreted directly into the
surrounding connective tissue. The hormone then diffuses into the vasculature.

Question: 13
The movement of the cilia is a result of

Answer Choices:
A An ATPase in the basal body
B Movement of one doublet microtubule along a microtubule of an adjacent
doublet
C Interaction of actin and myosin arms of the microtubules
D Interaction of actin and myosin within the basal body
E Action of the dynein arms of the two central microtubules
B Movement of one doublet microtubule along a microtubule of an adjacent doublet

The microtubule doublets consist of two columns of microtubule units: one, the A
column, is considered to be completely intact; and the other, the B column, which is not
"intact" in that it shares part of its wall with the A column. The A microtubules have
"arms" made up of an ATPase called dynein .
The dynein arms of the A microtubule form a reversible bridge to the B microtubule of
the neighboring doublet.
In the presence of ATP, the dynein slides the bridge along the B microtubule and
causes the cilium to move.

Question: 14
This organ illustrated by the photomicroscopic image is the

Answer Choices:
A Parotid gland
B Adrenal gland
C Sublingual gland
D Pancreas
E Sebaceous gland

Image(s) / Chart(s):
Click image to view full size. Click open image to close. Click and hold open image to move.

C Sublingual gland

The photomicroscopic image depicts the features of a secretory (exocrine) gland in


which ducts and acini (alveoli) can be observed. In this image, the secretory acini
consist of the lighter staining mucus secreting acini and the darker staining serous
secreting acini. Note that the mucus secreting acini predominate, but the serous
staining cells are interspersed among the acini. In some cases, combinations of the 2
cell types can be found as secretory units forming what are called serous demilunes.
These features are consistent with features found in both the submandibular and
sublingual glands, except the sublingual gland will have a predominance of mucus
secreting acini, as seen in this image.

Question: 15
The epithelium depicted in this photomicroscopic image could be found in which one of the following
locations?

Answer Choices:
A Vagina
B Trachea
C Scalp
D Back
E Hard palate

Image(s) / Chart(s):
Click image to view full size. Click open image to close. Click and hold open image to move.

A Vagina

Examination of the epithelium reveals flattened surface nuclei on a multilayered


(stratified) epithelial layer, characteristic of stratified squamous, non-keratinized
epithelium. The most appropriate location for this epithelial type from the list above
would be the vaginal epithelium. The trachea is lined with pseudostratified epithelium,
the scalp and back are lined with stratified squamous epithelium, which is keratinized,
and would probably possess either hair follicles and/or sweat or sebaceous glands. The
hard palate is a possibility, but because of the abrasion against the palate as a result of
mastication, the epithelium at this location is largely keratinized stratified squamous.
Question: 16
The arrow points to what kind of cell in the attached photomicrograph?

Answer Choices:
A Basal cell
B Goblet cell
C Plasma cell
D Adipose cell
E Lymphocyte

Image(s) / Chart(s):
Click image to view full size. Click open image to close. Click and hold open image to move.

B Goblet cell

The arrow is pointing to the cytoplasmic portion of a goblet cell. The goblet cell, so
named because of its goblet shape, is a unicellular exocrine gland and secretes a
mucous product. These are commonly found within the pseudostratified epithelium as
depicted in this photomicrograph.

Question: 17
The attached photomicrograph depicts an exocrine gland that would be best described as

Answer Choices:
A Simple branched tubular
B Simple coiled tubular
C Compound acinar
D Compound tubuloacinar
E Simple tubular

Image(s) / Chart(s):
Click image to view full size. Click open image to close. Click and hold open image to move.
B Simple coiled tubular

The type of gland illustrated in the image is characteristic of an eccrine sweat gland,
which is a simple coiled tubular gland. These multicellular glands form by tubular
invaginations from the surface and the terminal portions of the tubule form secretory
cells. The portion of the gland connecting the secretory cells to the surface is regarded
as the duct. In this case the duct is unbranched (simple), the secretory portion is tubular
shaped and coiled.

Question: 18
The interaction between 2 adjacent cells is important for cells to communicate with one another.
Modifications of the plasma membrane allow some tissues to become a syncytium, enabling the tissue
to respond to a stimulus in an organized array. Cardiac tissue has specialized junctions to reduce the
resistance of electrical current between adjacent cells. This low resistance of electrical conductivity is
due to the direct cytoplasmic continuity between cells. This specialized type of cell-to-cell
communication is which of the following structures?

Answer Choices:
A Gap junction
B Synapse
C Desmosome
D Node of Ranvier
E Tight junction
A Gap junction

A gap junction (nexus) is a concentration of membrane channels between 2 adjacent


cells engaged in ionic transport and transmission of electrical current, which allows for
direct communication of the cytoplasm of one cell with that of the adjacent cell. This
feature facilitates the direct movement of small molecules or ions between cells so there
is little electrical resistance between the 2 cells. By contrast, a synapse requires that a
molecule (transmitter) pass between 2 cell membranes and that the transmitter
indirectly 'conveys' the electrical current through a chemical intermediary. Once an
electrical impulse is formed from 'excitation' of a cell, the impulse may be rapidly
conveyed through processes such as axons. The node of Ranvier, a space along an
axon between adjacent myelin sheaths, allows sites that are more readily depolarized
and permits rapid conduction of the impulse along the process. Tight junctions or the
zonula occludens are structural features between 2 adjacent cells that limit the ability for
molecules to pass between the cells from the luminal surface. Several specific
membrane modifications may be present that act to tightly bind the plasma membranes
between 2 adjacent cells. One such feature that may be present as part of the terminal
bar is the desmosome or macula adherens.

Question: 19
The epithelium as observed by the attached photomicroscopic image can be classified as which type of
epithelium?

Answer Choices:
A Pseudostratified columnar with cilia
B Stratified columnar with cilia
C Pseudostratified columnar with microvilli
D Stratified columnar with microvilli
E Simple columnar with cilia

Image(s) / Chart(s):
Click image to view full size. Click open image to close. Click and hold open image to move.

A Pseudostratified columnar with cilia

The epithelium demonstrated in this photomicroscopic image illustrates cilia along the
luminal surface, a predominant basement membrane, and what appear to be several
layers of nuclei within the epithelium. The nuclei belong to cells that are arranged in the
epithelium at various levels, giving the appearance of stratification. However, electron
microscopic examination of this epithelial type demonstrates there is no true
stratification of the cells, and in fact, each cell has a process that rests upon the
basement membrane. This cell arrangement typifies pseudostratified columnar
epithelium with cilia (long, motile apical processes), which are present on the luminal
surface. Cells whose nuclei lie along the basement membrane are basal cells, where as
those cells, whose nuclei are elongate and displaced above and among the basal cells,
are columnar cells. Goblet cells may also be seen as part of this epithelium.

Question: 20
The type of epithelium illustrated by the photomicroscopic image attached would be found lining the
lumen of which organ?

Answer Choices:
A Vagina
B Trachea
C Ureter
D Aorta
E Vas Deferens

Image(s) / Chart(s):
Click image to view full size. Click open image to close. Click and hold open image to move.

C Ureter

The epithelium illustrated in this image is transitional epithelium, typically found lining
the ureters and urinary bladder. This is a stratified epithelium, and the surface cells
appear "dome-shaped" in the non-stretched form. Sometimes, the surface cells may be
bi-nucleated and contain granules. Frequently, they will cover two or more cells
underlying them. By comparison, the vagina is lined by non-keratinized stratified
squamous epithelium, and the trachea and vas deferens are lined by a pseudostratified
columnar ciliated epithelium. The aorta is lined by endothelium, a simple squamous
epithelium.

Question: 21
The basement membrane, functions, in part, to provide structural attachment of epithelial cells to the
underlying connective tissue. You were assigned the task of identifying the cell type which secretes a
component of the basement membrane. By using in situ histochemical hybridization, you plan to
identify the cells which express an mRNA for

Answer Choices:
A Laminin
B Type I collagen
C Actin
D Myelin
E Elastin

A Laminin

Using the in situ histochemical hybridization technique, it is possible to identify the


specific cells which express the laminin mRNA. Laminin is produced by epithelial cells
and is secreted as a component of the basement membrane, so given the above
choices, a cDNA for laminin would be the best basement membrane component to use
in the study. Using a cDNA for Type I collagen would show expression of mRNA in
fibroblasts, but Type I collagen is not a component of basement membranes. An mRNA
for actin would be expressed by muscle cells; also not a component of the basement
membrane. Likewise, cells expressing mRNA for myelin would be limited to either
Schwann cells, or oligodendrocytes, which do not contribute to the component of the
basement membrane. Elastin, a product of fibroblasts, would not contribute to basement
membrane.
Question: 22
The reinforcing cellular structure that aids in the resistance of separation between adjacent epithelial
cells is a feature of which of the following electron microscopic structures?

Answer Choices:
A Zonula occludens
B Zonula adherens
C Macula adherens
D Macula densa
E Fascia adherens
B Zonula adherens

Separation of luminal spaces from intercellular spaces is a critical function of epithelial


cells. The zonula occludens (tight junction) acts as a diffusion barrier due to the fusion
of adjoining cell membranes creating functional domains among luminal, cellular and
intercellular compartments. The ability of two adjacent cells to resist separation,
however, involves the reinforcement of the region below the zonula occludens by a
structure referred to as the zonula adherens. The macula adherens, also generally
regarded as a component of the junctional complex between adjacent epithelial cells,
provides for 'spot' adhesion at multiple sites on the lateral (and sometimes, basal)
surfaces of epithelial cells. The fascia adherens is a structure found in adjacent cardiac
muscle cells forming broad plates of sites of adhesion. The macula densa is not
associated with lateral surface specializations of epithelial cells, but rather are
specialized cells associated with the distal convoluted tubule in the kidney.

Question: 23
The epithelial type which best describes the epithelium lining the tubule marked with an asterisk (*) in
the attached photomicroscopic image is

Answer Choices:
A Stratified squamous, non-keratinized
B Simple cuboidal
C Simple squamous
D Simple columnar
E Transitional

Image(s) / Chart(s):
Click image to view full size. Click open image to close. Click and hold open image to move.

C Simple squamous

The asterisk is located within the lumen of a thin portion of the Loop of Henle in the
medulla of the kidney. The "thin segment" of the Loop is simple squamous epithelium,
and can be visualized in various tubules in close proximity. Some of the other tubules
are lined with simple cuboidal (tall, in some cases) epithelium. Capillaries can be seen
in the field, which also are lined with simple squamous epithelium, referred to as
endothelium.
Question: 24
Which one of the following types of epithelium forms the linings of the small intestines in humans?

Answer Choices:
A Simple squamous
B Pseudostratified ciliated
C Simple cuboidal
D Stratified squamous, non-keratinized
E Simple columnar

E Simple columnar

Simple squamous is typically located in endothelium (linings of blood vessels),


mesothelium (linings of peritoneum and pleura), as well as in lung alveoli, Bowman's
capsule and thin segment of the loop of Henle, pseudostratified ciliated in trachea
(including primary bronchi, nasal cavity and excretory duct in parotid glands), and
simple cuboidal in the linings of distal renal tubules, ducts in some glands and the
surface of the ovary. Stratified squamous, non-keratinized epithelium is
characteristically present in linings of esophagus, vagina, mouth and true vocal cords.

Question: 25
An example of the location of stratified squamous epithelium of the non-keratinized (wet) type would be

Answer Choices:
A Lining of the trachea
B Lining of the vagina
C Lining sweat gland ducts
D Lining an artery
E Lining the ureter

B Lining of the vagina

Stratified squamous epithelium is located throughout various locations in the body, but
depending upon whether the surface cells contain nuclei will determine its specific type
and location. Nuclei located in the surface cells typify a non-keratinized epithelium (wet)
and indicate a location with low abrasion. The lining of the vagina is a typical area
where non-keratinized, stratified squamous epithelium is located. The trachea is
typically lined by a ciliated, pseudostratified columnar epithelium. The sweat gland ducts
typically have a stratified cuboidal epithelium, and the ureter has transitional epithelium.
An artery is lined by a simple squamous epithelium, specifically referred to as
endothelium.
Question: 26
Apical modifications of epithelial cell surfaces may possess specialized structures capable of moving
fluid or particles along a luminal surface. These cell modifications are

Answer Choices:
A Stereocilia
B Microvilli
C Cilia
D Microtubules
E Villi

C Cilia

Epithelial surfaces may exhibit modifications capable of performing specialized


functions, such as movement of fluids or particles along the luminal surface. The
specialized structures responsible for this function are cilia. Cilia are long, motile
cytoplasmic processes that extend from the surface of specific epithelial cells located in
specialized areas in which transport of particulate matter or fluids is necessary. Cilia
contain microtubule components arraigned in a specific, radial 9+2 pattern and require
energy for movement of the process. The synchronized pattern of movement by groups
of cilia provide for the ability to move substances along the surface of the cell. Microvilli,
in contrast, are cell surface extensions, but lack microtubules and are non-motile.
Stereocilia are misnomers, since they too lack microtubules and are non-motile. They
are essentially very long microvilli.
Question: 27
The apical surface of most epithelial cells is characterized by

Answer Choices:
A Desmosomes
B Junctional complexes
C Many cellular infoldings
D Microvilli
E Microvilli and cellular infolding

D Microvilli

Microvilli and, more rarely, cilia are found on the apical aspect of epithelial cells.
Desmosomes and junctional complexes are found on the lateral portion of the cell
membrane and undulations of the cell membrane are found on both the basal and
lateral cell surfaces.

Question: 28
Membrane folds along the lateral membranes of some epithelial cells

Answer Choices:
A Are primarily seen in cells that transport large amounts of fluid
B Are primarily a method of forming a tighter junction with neighboring cells
C Are only seen in the epidermis
D Are only seen in transitional epithelium
E Allow intestinal cells to move fatty acids into the circulation quickly
A Are primarily seen in cells that transport large amounts of fluid

Lateral membrane folds are particularly noticeable in the intestinal epithelium and allow
these cells to move large amounts of fluid from the intestinal lumen into circulation
quickly.
The fluid first enters the cell along its apical surface. Below the junctional complexes of
the lateral membrane, Na+ is actively transported out of the cell and into the intercellular
space. Fluid that has been taken into the cell follows the sodium osmotically. The
hydrostatic pressure in the intercellular space builds up and forces the fluid into the
underlying connective tissue and vascular spaces. The many membrane folds of the
lateral surface allow this process to proceed rapidly.

Question: 29
The secretory product of the exocrine gland illustrated in the attached photomicrographic image is:

Answer Choices:
A Serous
B Mucous
C Mixed

Image(s) / Chart(s):
Click image to view full size. Click open image to close. Click and hold open image to move.
A Serous

The apical cytoplasm of serous cells typically stain more intensely with eosin and the
perinuclear cytoplasm stains more basophilic. The basophilia is due to extensive rough
endoplasmic reticulum located in the basal compartment of the cell, characteristic of
cells that are producing a proteinaceous secretory substance. Mucous secreting cells, in
contrast, stain poorly with H&E since the cytoplasmic contents fix poorly and the
glycosylated proteins are lost during the tissue preparation. In this image, the secretory
cells are seen as simple cuboidal cells that stain less intense than the basophilic
staining cells of the duct. Careful inspection reveals that the ducts are lined with
stratified cuboidal epithelium.
Question: 30
Paracrine secretion differs from endocrine secretion in that paracrine secretion

Answer Choices:
A Releases electrolytes instead of hormones
B Does not enter the bloodstream
C Requires glands with ducts
D Regulates salt and water concentration
E Is regulated by the pituitary

B Does not enter the bloodstream

Paracrine secretion is secretion that diffuses into the surrounding epithelial cells and
affects them alone. Paracrine secretions do not reach the bloodstream.
Question: 31
Which of the following statements is true of mucous secreting cells?

Answer Choices:
A They are found in the parotid gland and the pancreas
B They stain with PAS when routinely processed
C The nucleus of mucin secreting cells is characteristically flattened against the
basal surface of the cell
D The mucin produced is poorly glycosylated protein
E On routine H & E staining, the cytoplasm of mucin secreting cells is
eosinophilic
C The nucleus of mucin secreting cells is characteristically flattened against the basal
surface of the cell

All the statements in this question distinguish serous glands from mucous glands.
Serous glands produce a watery material that consists of poorly glycosylated protein.
Because it is poorly glycosylated it does not stain with PAS stain, but on routine H & E
stain the cytoplasm of the serous secretory cells will stain with eosin, especially at the
apical surface.
Mucinous glands produce a thicker secretion that consists of well glycosylated protein
that in theory stains well with PAS stain. However, because this protein is water soluble,
in routine processing it is washed out, leaving secretory cells that appear empty under
the microscope.
The mucin product of mucinous glands classically flattens the nucleus of these cells
against the basal surface of the cell. The nuclei of serous cells are more centrally
located and have a more round to oval shape. The perinuclear area in serous cells is
frequently basophilic due to an increase in the endoplasmic reticulum, a reflection of the
increased protein secretion.
The sublingual gland, goblet cells and the gastric lining cells secrete mucous. The
pancreatic and parotid gland cells are serous. Some glands consist of a mixture of
mucous and serous glands, e.g. the submandibular gland. In these glands the serous
portions are located distal to the mucinous component.

Question: 32
The olfactory basal stem cells do which of the following?

Answer Choices:
A Secrete mucus
B Produce olfactory receptors
C Produce columnar epithelium which lines the nasal mucosa
D Function as first order bipolar neurons
E Are innervated by the olfactory nerve
B Produce olfactory receptors

The basal stem cells of the olfactory epithelium lie sandwiched between and at the base
of the supporting cells. They are continuously dividing, and produce new olfactory
receptors as pre-existing month-old receptors die. These receptors are actually bipolar
first-order neurons. Usually, neurons are unable to reproduce, so these olfactory
receptors are unique in the nervous system.
The olfactory cells which produce mucus are found in Bowman's glands. These glands
lie deep within the supporting connective tissue of the nasal mucosa. Mucus is
produced in order to provide lubrication and to trap foreign airborne particles before they
can reach the deeper respiratory passages.
The columnar epithelium of the nasal mucosa are also known as the olfactory
supporting cells. These cells provide the olfactory receptor cells with nutrients, support,
electrical insulation and detoxification of airborne toxins.
Olfactory receptors are actually first order neurons. They are bipolar, and conduct nerve
impulses from their exposed tip (a dendrite which projects beyond the epithelial layer) to
the olfactory bulb in the brain. On the surface of the dendrites are olfactory hairs which
convert odorants (chemicals with an odor) to a generator potential.
The olfactory structures innervated by the first cranial nerve are the olfactory receptors.
Branches of cranial nerve seven (facial nerve) supply innervation to the nasal olfactory
glands, allowing the glands to secrete mucus onto the lining of the nasal mucosa.

section_ blood
Question: 1

What is the primary function of the nucleated cell seen in this photomicrographic image of a peripheral
blood smear?

Answer Choices:
A Antibody production
B Phagocytosis
C Antigen production
D Oxygen transport
E Heparin sulfate production

Image(s) / Chart(s):
Click image to view full size. Click open image to close. Click and hold open image to move.
B Phagocytosis

Monocytes represent about 3-8% of the circulating leukocytes and are larger than red
blood cells, being 10-15μm in diameter. The nucleus is large and may appear rounded,
kidney-shaped, or horseshoe-shaped, but it is never segmented. At times two or three
nucleoli may be observed in the pale-staining nucleus. The blue-gray cytoplasm is
relatively abundant and contains fine azurophilic granules, which ultrastructurally
represent primary lysosomes. These cells will emigrate into various organs and tissues
throughout the body where they may differentiate into tissue macrophages. Monocytes
may also function to concentrate foreign antigens and present them to lymphocytes
during the immune response. The plasma cell is a cell whose primary function is to
produce antibodies in response to foreign antigens. Oxygen transport is performed by
the red blood cells. Heparin sulfate production occurs by appropriate stimulation of mast
cells.
Question: 2

The cell found in the peripheral blood associated with secretion of the vasoactive agent referred to as
slow-reacting substance (SRS) of anaphylaxis is the

Answer Choices:
A Neutrophil
B Eosinophil
C Lymphocyte
D Basophil
E Monocyte

D Basophil
The basophil, as found in the peripheral blood, contains large, darkly staining,
membrane-bound granules that contain a variety of substances, including SRS, heparin
sulfate and histamine. Upon stimulation, the release of histamine and SRS will induce
the dilation of small blood vessels. The basophil, as found in the peripheral blood, may
be related, at least functionally, to the tissue mast cell.

Question: 3

Platelets are derived from which of the following cell types?

Answer Choices:
A Lymphocytes
B Plasma cells
C T cells
D Neutrophils
E Megakaryocytes
E Megakaryocytes

The plasma cells are derived from resting B lymphocytes and are responsible for
antibody secretion. They occur mainly in secondary lymphoid tissue. The plasma cells
are characterized by an abundance of rough endoplasmic reticulum that is required for
antibody synthesis and secretion.
Megakaryocytes are giant cells found in the bone marrow and are the source of
circulating platelets. The megakaryocyte undergoes many rounds of DNA replication
without undergoing cell division. Once DNA synthesis has stopped, the cytoplasm of the
cell matures.
Platelets bud from the main cell body of the megakaryocyte. Platelets lack nuclei and
function in hemostasis, coagulation, and inflammation.
The release of histamine and/or serotonin occurs to mediate inflammation. The
cytoplasm of the small or "resting" lymphocyte is taken up mostly by the nucleus. When
lymphocytes contact antigen, activation signals trigger development into large
lymphocytes or plasma cells in the case of B lymphocytes.
Neutrophils are phagocytic cells which are recruited to the sites of inflammation.
Histamine is secreted by mast cells, basophils and platelets.
Question: 4

The function of the cell located at the tip of the arrow in this photomicroscopic image is to

Answer Choices:
A Recognize foreign antigens
B Produce antibodies
C Phagocytose foreign particles
D Transport oxygen
E Secrete heparin

Image(s) / Chart(s):
Click image to view full size. Click open image to close. Click and hold open image to move.

A Recognize foreign antigens

Characteristic features of lymphocytes found within a peripheral blood smear reveal that
they are small cells, about the size of a red blood cell. The small, intensely staining
nucleus occupies most of the cell with a thin, pale blue rim of cytoplasm surrounding the
nucleus. Lymphocytes are the most common agranulocyte, accounting for about 30% of
the total number of leukocytes in the peripheral blood. The small lymphocyte, as seen in
this image, ranges in size from about 6 to 18μm. These cells function to recognize and
respond to foreign antigens. Cells that produce antibodies are plasma cells, which are
differentiated B lymphocytes. Macrophages and monocytes are cells that are
phagocytic, and red blood cells (surrounding the lymphocytes in the image), are filled
with hemoglobin which binds oxygen for transport by the cell. Mast cells secrete
heparin, among other things. Mast cells, largely found in the tissue, are filled with large,
intensely staining vacuoles.
Question: 5

Formation of a clot at the site of vessel injury is the primary function of which of the following?

Answer Choices:
A Red blood cells
B Megakaryocytes
C Macrophages
D Monocytes
E Platelets
E Platelets

Platelets, cytoplasmic fragments derived from large polyploid cells in the bone marrow
(megakaryocytes), adhere to the ruptured end of an injured vessel. Upon aggregation
and formation of the clot, they will release serotonin and thromboplastin, to cause
vasoconstriction and initiate formation of a fibrin clot, respectively. These substances
are released from granules
that are contained within the platelet cytoplasm. Macrophages, derived from monocytes,
may also participate in wound healing following injury of a vessel and clot formation.
These cells may phagocytose cell and tissue debris, including expendable fibrin clot.

Question: 6

One of blood cells in a blood smear taken from a female subject showed a drumstick-like appendage
on one of the lobes of its nucleus. To which one of the following types of blood cells does this cell
belong?

Answer Choices:
A Acidophils
B Basophils
C Neutrophils
D Monocytes
E Lymphocytes

C Neutrophils

In females, the inactive X chromosome appears as a drumstick-like appendage on one


of the lobes of the nucleus. This is not obvious in all neutrophils, however.
Acidophils, basophils, monocytes, and lymphocytes do not show such structures.
Question: 7

A 10-year-old boy suffering from a severe toothache is taken to a pedodontist, who prescribes
painkillers and recommends tooth extraction after one week. Following the treatment schedule, his
mother takes the boy for a tooth extraction. The lab investigation just before tooth extraction reveals
the following
RBC count 6X106/cu mm of blood
TWBC count 5500 cells/cu mm of blood
hematocrit 46%
Hb concentration 15.5 gm %
ESR 3 mm at the end of first one hour
Bleeding time 16 minutes (Ivy's method)
Clotting time 6 minutes (Wright's method)
Which lab result shows abnormal value?

Answer Choices:
A Clotting time
B TWBC count
C ESR
D Bleeding time
E Hematocrit
D Bleeding time

All the values are within normal range except the bleeding time. Bleeding time can be
defined as the time taken for the beginning of bleeding to complete cessation of
bleeding. In Ivy's method, the normal bleeding time varies from 1-9 minutes.
TWBC normal range is 4,500-10,000 white blood cells per microliter (mcL).
ESR stands for erythrocyte sedimentation rate. It is a test that indirectly measures how
much inflammation is in the body. However, it rarely leads directly to a specific
diagnosis.
Adults (Westergren method):
Men under 50 years old: less than 15 mm/hr
Men over 50 years old: less than 20 mm/hr
Women under 50 years old: less than 20 mm/hr
Women over 50 years old: less than 30 mm/hr
Children (Westergren method):
Newborn: 0 to 2 mm/hr
Neonatal to puberty: 3 to 13 mm/hr
Hematocrit is a blood test that measures the percentage of red blood cells found in
whole blood. This measurement depends on the number of red blood cells and the size
of red blood cells. The hematocrit is almost always ordered as part of a complete blood
count.
Normal results vary, but in general are as follows:
Male: 40.7 - 50.3%
Female: 36.1 - 44.3%

Keywords: bleeding time, RBC count

Question: 8

A 55-year-old male with liver cancer has a CBC performed for evaluation. The patient is found to be
anemic and has a significant morphological characteristic to the red blood cells (refer to the image).
These red blood cells are often described as spherical with irregular thorny projections. What are these
cells called?

Answer Choices:
A Spherocytes
B Stomatocytes
C Acanthocytes
D Codocytes
E Hypochromic/microcytes
F Elliptocytes
Image(s) / Chart(s):
Click image to view full size. Click open image to close. Click and hold open image to move.

C Acanthocytes

Acanthocytes are red blood cells that are roughly spherical with irregular thorny
projections. They occur in vitamin E deficiency, McLeod syndrome, in some patients
with severe hepatocellular diseases, patients with lipoproteinemia, and in anorexia
nervosa.

Stomatocytes are red blood cells that have a mouth-like band of pallor across the center
of the stained cell. These cells are seen in hereditary hydrocytosis, acute alcoholism,
and various types of liver disease. In hereditary hydrocytosis there is an associated
inherited abnormality in red cell cation permeability that may be associated with
abnormal red blood cell hydration or membrane lipids.
Spherocytes are red blood cells that have a circular shape with no discernible pallor to
the cells. They may appear as macro-spherocytes or micro-spherocytes. The
spherocytes have a decreased life span when compared to normal red blood cells.
These cells occur in hereditary spherocytosis, acquired hemolytic anemia due to
exposure to oxidants, severe burns, and some infections. In hereditary spherocytosis
splenectomy is the recommended therapy.
Codocytes (also called target cells) are red blood cells that have an increase in
membrane surface area or a decrease in cell volume. The resulting flattened cells
appear in blood smears as target cells. Codocytes are found in hemoglobin C,
obstructive jaundice, and thalassemia.
Hypochromic/microcytes are red blood cells that are inadequately filled with hemoglobin
and are microcytic (smaller than normal). These cells many times are in combination
with elliptocytes and ovalocytes. There is a noticeable increase in the size of the central
pallor of the red blood cells. The mean corpuscular hemoglobin concentration (MCHC)
is usually low. Iron deficiency from low iron intake, diminished iron absorption, or excess
iron loss can cause hyphochromic/microcytic anemia.
Elliptocytes are red blood cells that are have a defect to the membrane skeleton. The
cells obtain a predominantly elliptical shape. Elliptocytes can be seen in
hypochromic/microcytic anemia smears, but are more readily noticeable in patients with
mild or hemolytic elliptocytosis. In mild hereditary elliptocytosis there is usually a mild
hemolytic anemia (a small minority will have mild hemolytic anemia). In hemolytic
elliptocytosis the clinical presentation is similar to hereditary spherocytosis (moderate
hemolysis, mild anemia, and splenomegaly). As in hereditary spherocytosis, patients
with hereditary elliptocytosis respond well to splenectomy.
Question: 9

A 40-year-old male comes to the office as a newly referred patient. The patient appears to be a healthy
adult male with normal vital signs. A routine CBC was performed and the patient was found to be
slightly anemic. The patient revealed that he had thalassemia, which was confirmed by his records.
The CBC smear was characteristic for a morphological type of red blood cell seen in thalassemics
(refer to the image). What are these cells called?

Answer Choices:
A Spherocytes
B Stomatocytes
C Acanthocytes
D Codocytes
E Hypochromic/microcytes
F Elliptocytes

Image(s) / Chart(s):
Click image to view full size. Click open image to close. Click and hold open image to move.
D Codocytes

Codocytes (also called target cells) are red blood cells that have an increase in
membrane surface area or a decrease in cell volume. The resulting flattened cells
appear in blood smears as target cells. Codocytes are found in hemoglobin C,
obstructive jaundice, and thalassemia.

Stomatocytes are red blood cells that have a mouth-like band of pallor across the center
of the stained cell. These cells are seen in hereditary hydrocytosis, acute alcoholism,
and various types of liver disease. In hereditary hydrocytosis there is an associated
inherited abnormality in red cell cation permeability that may be associated with
abnormal red blood cell hydration or membrane lipids.
Spherocytes are red blood cells that have a circular shape with no discernible pallor to
the cells. They may appear as macro-spherocytes or micro-spherocytes. The
spherocytes have a decreased life span when compared to normal red blood cells.
These cells occur in hereditary spherocytosis, acquired hemolytic anemia due to
exposure to oxidants, severe burns, and some infections. In hereditary spherocytosis
splenectomy is the recommended therapy.
Acanthocytes are red blood cells that are roughly spherical with irregular thorny
projections. They occur in vitamin E deficiency, McLeod syndrome, in some patients
with severe hepatocellular diseases, patients with lipoproteinemia, and in anorexia
nervosa.
Hypochromic/microcytes are red blood cells that are inadequately filled with hemoglobin
and are microcytic (smaller than normal). These cells many times are in combination
with elliptocytes and ovalocytes. There is a noticeable increase in the size of the central
pallor of the red blood cells. The mean corpuscular hemoglobin concentration (MCHC)
is usually low. Iron deficiency from low iron intake, diminished iron absorption, or excess
iron loss can cause hyphochromic/microcytic anemia.
Elliptocytes are red blood cells that are have a defect to the membrane skeleton. The
cells obtain a predominantly elliptical shape. Elliptocytes can be seen in
hypochromic/microcytic anemia smears, but are more readily noticeable in patients with
mild or hemolytic elliptocytosis. In mild hereditary elliptocytosis there is usually a mild
hemolytic anemia (a small minority will have mild hemolytic anemia). In hemolytic
elliptocytosis the clinical presentation is similar to hereditary spherocytosis (moderate
hemolysis, mild anemia, and splenomegaly). As in hereditary spherocytosis, patients
with hereditary elliptocytosis respond well to splenectomy.
Question: 10

A 40-year-old man and known alcoholic presents to the emergency department unconscious. His
friends brought him after spending the day at a summer picnic, drinking all day. Blood was drawn for
the following laboratory tests: CBC, glucose, BUN, Creatinine, Electrolytes, alcohol, and SGOT. The
results are in the following table.
Tests Results Differential
Alcohol 300mg/100ml Metas 0.3%
Glucose 200mg/dl Bands 3.2%
BUN 24 mg/dl Segs 55.3%
Creatinine 1.2 mg/dl Eos 4.0%
SGOT 100 U/ml Basos 0%
Chloride 100mmol/L Lymphs 34.0%
Sodium 137 mmol/L Monos 3.2%
Urinalysis Acetone 1+, Blood 2+ Platelets 380 x 109 /l
RBC 4.3 x 1012/l
Hgb 13.4 gm/dl
Hct 0.39
MCV 82 fl
MCHC 34%
MCH 28 pg
WBC 8.0 x 109/l
The patient is diagnosed with acute alcohol intoxication. The elevated SGOT is indicative of liver
disease probably from alcohol abuse. The RBC morphology was significant for what kind of
morphological cell type (refer to the image) that is also indicative of acute alcoholism and liver disease?

Answer Choices:
A Spherocytes
B Stomatocytes
C Acanthocytes
D Codocytes
E Hypochromic/microcytes
F Elliptocytes

Image(s) / Chart(s):
Click image to view full size. Click open image to close. Click and hold open image to move.

B Stomatocytes

Stomatocytes are red blood cells that have a mouth-like band of pallor across the center
of the stained cell. These cells are seen in hereditary hydrocytosis, acute alcoholism,
and various types of liver disease. In hereditary hydrocytosis there is an associated
inherited abnormality in red cell cation permeability that may be associated with
abnormal red blood cell hydration or membrane lipids.
Spherocytes are red blood cells that have a circular shape with no discernible pallor to
the cells. They may appear as macro-spherocytes or micro-spherocytes. The
spherocytes have a decreased life span when compared to normal red blood cells.
These cells occur in hereditary spherocytosis, acquired hemolytic anemia due to
exposure to oxidants, severe burns, and some infections. In hereditary spherocytosis
splenectomy is the recommended therapy.
Acanthocytes are red blood cells that are roughly spherical with irregular thorny
projections. They occur in vitamin E deficiency, McLeod syndrome, in some patients
with severe hepatocellular diseases, patients with lipoproteinemia, and in anorexia
nervosa.
Codocytes (also called target cells) are red blood cells that have an increase in
membrane surface area or a decrease in cell volume. The resulting flattened cells
appear in blood smears as target cells. Codocytes are found in hemoglobin C,
obstructive jaundice, and thalassemia.
Hypochromic/microcytes are red blood cells that are inadequately filled with hemoglobin
and are microcytic (smaller than normal). These cells many times are in combination
with elliptocytes and ovalocytes. There is a noticeable increase in the size of the central
pallor of the red blood cells. The mean corpuscular hemoglobin concentration (MCHC)
is usually low. Iron deficiency from low iron intake, diminished iron absorption, or excess
iron loss can cause hyphochromic/microcytic anemia.
Elliptocytes are red blood cells that are have a defect to the membrane skeleton. The
cells obtain a predominantly elliptical shape. Elliptocytes can be seen in
hypochromic/microcytic anemia smears, but are more readily noticeable in patients with
mild or hemolytic elliptocytosis. In mild hereditary elliptocytosis there is usually a mild
hemolytic anemia (a small minority will have mild hemolytic anemia). In hemolytic
elliptocytosis the clinical presentation is similar to hereditary spherocytosis (moderate
hemolysis, mild anemia, and splenomegaly). As in hereditary spherocytosis, patients
with hereditary elliptocytosis respond well to splenectomy.
Question:11

A cellular physiologist performs experiments to monitor the cellular activities and metabolism as
monocytes mature into macrophages. During the maturation, which of the following statements is
accurate?

Answer Choices:
A Cellular activities decrease
B Cellular activities increase
C Cellular activities remain the same
D Basal metabolism remains low
E Basal metabolism diminishes

B Cellular activities increase

Macrophages possess a basal metabolism that does not remain continuously high, but
is rather variable. Specific ligand receptor interactions can kick the basal metabolism
into "high gear", including those that produce a respiratory burst.
Cellular activities increase, as monocytes become macrophages. Specifically, the
mitochondria increase in number, as does the activity of their enzymes, and the
respiration rate. Also indicative of an increase in cellular activity, the numbers of
lysosomes and their associated enzymes also increase at the transition. Even as they
appear to be at rest, the metabolism of macrophages is very high, with the plasma
membrane renewing twice an hour.

Question: 12

Spectrin is the major protein responsible for the cortical skeleton of what cell type?

Answer Choices:
A Myoblasts
B Leukocytes
C Neutrophils
D Erythrocytes
E Almost all mammalian cell types

D Erythrocytes

Spectrin is an actin binding protein that occurs as a tetramer composed of 2α and 2β


chains. Actin binds to the β chain. Spectrin associates with short actin filaments
resulting in a spectrin/actin network. This network forms the cortical skeleton of red
blood cells giving them their characteristic biconcave shape.

Question: 13

Which one of the following proteins associated with the erythrocyte plasma membrane is responsible
for maintaining the cell's biconcave disc shape?

Answer Choices:
A HbA1
B HbA2
C Fidrin
D Spectrin
E α-Actinin

D Spectrin

Spectrin (a long flexible protein, about 110 nm long, which forms tetramers and provides
a scaffolding for structural reinforcement) is associated with the erythrocyte cell
membrane and assists in the maintenance of its biconcave disc shape.
Hemoglobin, the respiratory protein located in erythrocytes, occurs in several normal
forms differing in their chain composition.
The predominant form of adult hemoglobin is HbA1 (α2β2); a minor form is HbA2 (α2δ2).
Neither form plays any part in the cytoskeleton of the erythrocyte.
α-Actinin (which cross links actin filaments to form a mesh work) and fidrin (which
serves as a nonerythroid spectrin) are present in nonerythroid cells.

Question: 14

Which one of the following types of mature blood cells is enclosed in the plasma membrane but does
not contain a nucleus?

Answer Choices:
A Monocytes
B Basophils
C Thrombocytes
D Neutrophils
E Acidophils

C Thrombocytes
Image(s) / Chart(s):
Click image to view full size. Click open image to close. Click and hold open image to
move.

Platelets (= thrombocytes) are non-nucleated, disk-like cell fragments 2-4 μm in


diameter. They originate from the fragmentation of giant polyploid megakaryocytes
residing in the bone marrow.
Acidophils, basophils, neutrophils, and monocytes (all of them leukocytes) contain
nuclei of various shapes.
See the attached image: (A) Megakaryocyte in the bone marrow; (B) Thrombocytes [(a)
LM; (b) EM] in a blood smear.

Question: 15

The granules in the nucleated cell as depicted in this image contain a high content of what enzyme?

Answer Choices:
A Phagocytin
B Peroxidase
C Lactase
D Lipase
E Nuclease

Image(s) / Chart(s):
Click image to view full size. Click open image to close. Click and hold open image to move.

B Peroxidase

This cell depicted in this photomicroscopic image is an eosinophil demonstrating the


characteristic bi-lobed nucleus and the distinctive closely packed brilliant red granules.
These granules are lysosomes, and contain a high content of peroxidase. They differ
from the azurophil granules of the neutrophil in that they do NOT contain phagocytin.
Eosinophil granules also contain histaminase and other hydrolytic enzymes. Lactase,
lipase and nucleases are located within lysosomes of cells, but do not constitute a high
content in eosinophils.

Question: 16

The nucleated cell as seen in this photomicroscopic image is best described as which cell type?
Answer Choices:
A Neutrophil
B Basophil
C Eosinophil
D Lymphocyte
E Monocyte

Image(s) / Chart(s):
Click image to view full size. Click open image to close. Click and hold open image to move.

C Eosinophil

The eosinophil is one of the granulated leukocytes having a lobulated nucleus. The
nucleus of the eosinophil is frequently bi-lobed, as seen in this image, but may have
three or four nuclear lobules. The distinctive feature of this cell is the presence of large
brilliant, red-staining cytoplasmic granules that are so closely packed they tend to
obscure the nucleus.

Question: 17

The nucleated cell as seen in this photomicroscopic image is best described as which cell type?

Answer Choices:
A Neutrophil
B Basophil
C Eosinophil
D Lymphocyte
E Monocyte

Image(s) / Chart(s):
Click image to view full size. Click open image to close. Click and hold open image to move.
E Monocyte

This cell represents a mononuclear leukocyte and is regarded as agranular, even


though small, faintly staining granules may be seen in the cytoplasm. The monocyte is a
large cell with a diameter of 10-15μm. The nucleus is never segmented, but it usually
stains less dense than a lymphocyte and appears rounded, kidney or horseshoe-
shaped. Several nucleoli may be visualized. The "agranular" appearance of the
cytoplasm is due to the presence of fine, abundant azurophil granules. These cells
differentiate in the tissue into macrophages.

Question: 18

The nucleated cell in this photomicroscopic image may be best described as what cell type?

Answer Choices:
A Basophil
B Eosinophil
C Monocyte
D Neutrophil

Image(s) / Chart(s):
Click image to view full size. Click open image to close. Click and hold open image to move.

Neutrophil
The granulated neutrophil in peripheral blood is commonly characterized by the
lobulated shape of the nucleus (four, in this specimen), which are connected by thin
filaments. As in this cell, a nuclear appendage (drumstick) can be visualized,
representing the female sex chromatin. The cytoplasm contains two types of granules,
specific granules which can be seen as the smaller, pinkish granules, and the larger
azurophil granules that stain a reddish-purple.

Question: 19

One of the primary functions of the cells seen in this photomicroscopic image is to

Answer Choices:
A Transport oxygen
B Transport water
C Secrete histamine
D Secrete antibodies
E Phagocytosis of foreign material

Image(s) / Chart(s):
Click image to view full size. Click open image to close. Click and hold open image to move.

A Transport oxygen
The cells as depicted in this photomicrograph are mature erythrocytes. These "non-
nucleated" cells are filled with hemoglobin, which has an affinity for oxygen and carbon
dioxide. Hence, one of the primary functions of these cells is to transport oxygen.
Histamine is secreted by mast cells in the tissues, antibodies are formed and secreted
by plasma cells upon appropriate stimulation, and several cells will phagocytose foreign
material, including monocytes and macrophages.

Question: 20

The cells observed on this photomicroscopic image are best identified as

Answer Choices:
A Erythrocytes
B Eosinophils
C Neutrophils
D Lymphocytes
E Monocytes

Image(s) / Chart(s):
Click image to view full size. Click open image to close. Click and hold open image to move.

A Erythrocytes
This image depicts the appearance of mature red blood cells (erythrocytes) seen when
a peripheral blood smear is performed and a Romanowsky-type stain is used. The
erythrocytes are 6-8μm in diameter, lack a nucleus and are best described as bi-
concave discs. Each cell is filled with hemoglobin and are responsible for transport of
oxygen, carbon dioxide, or other gases capable of binding with the hemoglobin.
Eosinophils also stain reddish and appear in peripheral blood, but are nucleated and
contain large, predominant granules within the cytoplasm. Neutrophils also are
nucleated cells, frequently having 2, 3 or more lobules since the nucleus is segmented.
Their cytoplasm stains with a slight bluish tint, and contain small granules. Lymphocytes
are small cells, about the size of red blood cells, but the cytoplasm is filled by the
nucleus leaving only a small rim of cytoplasm surrounding the darkly staining nucleus.
Monocytes are large nucleated cells with an agranular cytoplasm. The nucleus is
frequently kidney-shaped, indented or appearing as being folded.
Question: 21

Which of the following cells is most likely increased in patients with parasitic diseases?

Answer Choices:
A Neutrophils
B Eosinophils
C Basophils
D Platelets
E Monocytes

B Eosinophils

Neutrophils are the most numerous, about 60%, of the white blood cells. They are
highly mobile, phagocytic, and are attracted by destruction products from damaged
tissues; a process call chemotaxis. Monocytes represent the second defense, as they
are much larger and slower than the neutrophils. They appear in high concentrations
several days later than the neutrophils. They are part of the monocyte-macrophage
system, which is also referred to as the reticuloendothelial system.
Similar to neutrophils are the eosinophils, except that they are less chemotactic and
less phagocytic. Their numbers are increased in allergic reactions and in parasitic
diseases. Basophils can be compared to mast cell. They produce large amounts of
heparin, which they release into the blood stream to prevent blood coagulation in the
normal circulation. Platelets are cell fragments rather than whole cells. They are
necessary for the blood coagulation.
Question: 22

Following their maturation in the thymus gland and release into the vascular system, T-lymphocytes
preferentially migrate to which one of the following tissues?

Answer Choices:
A Paracortex of lymph nodule
B Cortical lymphoid nodules of lymph nodes
C Lymphoid nodules of the spleen
D Hilus of lymph nodes
E Lymphoid nodules of the lingual tonsils

Paracortex of lymph nodule

T-lymphocytes generally re-circulate through the deep cortex or paracortical regions of


lymph nodes and the periarterial sheaths in the white pulp of the spleen. The B-
lymphocytes, on the other hand, are found in lymphoid nodules (a part of the lymph
nodes, tonsils, and the spleen).

Question: 23

Which one of the following clinical conditions will promote a circulating increase in atypical
lymphocytes?

Answer Choices:
A von Willebrand's disease
B Spherocytosis
C Infectious mononucleosis
D Pernicious anemia

C Infectious mononucleosis

Infectious mononucleosis (caused by the Epstein-Barr virus and characterized by


swollen and tender lymph nodes, sore throat, fever, fatigue) will cause an increase in
blood lymphocytes, many of which are atypical.
von Willebrand's disease results from a decrease in von Willebrand's factor which,
along with factor VIII, promotes platelet aggregation. Spherocytosis is caused by a
genetic defect in spectrin that decreases its ability to bind with band 4.1 protein.
Pernicious anemia is caused by a severe deficiency of vitamin B12. None of these
conditions cause an increase in atypical lymphocytes.
Question: 24

The cell located at the tip of the arrow in the photomicroscopic image is a

Answer Choices:
A Plasma cell
B Lymphocyte
C Normoblast
D Neutrophil
E Basophil

Image(s) / Chart(s):
Click image to view full size. Click open image to close. Click and hold open image to move.

B Lymphocyte

Characteristic features of lymphocytes found within a peripheral blood smear reveal that
they are small cells, about the size of a red blood cell, and the small, intensely staining
nucleus occupies most of the cell with a thin, pale blue rim of cytoplasm surrounding the
nucleus. Lymphocytes are the most common agranulocyte, accounting for about 30% of
the total number of leukocytes in the peripheral blood. The small lymphocyte, as seen in
this image, ranges in size from about 6 to 18μm. These cells function to recognize and
respond to foreign antigens.
Question: 25

The nucleated cell depicted in this photomicrographic image of a peripheral blood smear is a

Answer Choices:
A Lymphocyte
B Basophil
C Neutrophil
D Monocyte
E Eosinophil
Image(s) / Chart(s):
Click image to view full size. Click open image to close. Click and hold open image to move.

D Monocyte

Monocytes represent about 3-8% of the circulating leukocytes and are larger than red
blood cells, being 10-15μm in diameter. The nucleus is large and may appear rounded,
kidney-shaped, or horseshoe-shaped, but it is never segmented. At times two or three
nucleoli may be observed in the pale-staining nucleus. The blue-gray cytoplasm is
relatively abundant and contains fine azurophilic granules, which ultrastructurally
represent primary lysosomes. These cells will emigrate into various organs and tissues
throughout the body where they may differentiate into tissue macrophages. Monocytes
may also function to concentrate foreign antigens and present them to lymphocytes
during the immune response. The lymphocyte is another "agranular" leukocyte, like the
monocyte, but is much smaller as seen in a peripheral blood smear, has little cytoplasm,
and contains a small, dense nucleus. The basophil, neutrophil and eosinophil are all
"granular" leukocytes, and as such, contain variable numbers and sizes of granules in
their cytoplasm. Also, each of these cells contain segmented nuclei that appear as
lobes.

Case #301588:
A 6-month-old child has many reddish papules and scaling plaques on its chest, back,
and scalp. In addition, this child has hepatosplenomegaly and lymphadenopathy. An
immunohistochemical analysis of a skin biopsy reveals numerous rounded dermal
cells with the CD1a marker. Electron microscopic analysis of abnormal dermal cells
reveals that some have pentalaminar rods with terminal dilations. Further tests make
the diagnosis of Letterer-Siwe disease.
Question: 26

Which cell in the skin is MOST likely involved in this disease?

Answer Choices:
A dermal fibroblast
B Langerhans cell
C eosinophil
D melanocyte
E plasma cell
B Langerhans cell

Letterer-Siwe disease (acute disseminated Langerhans cell histiocytosis) is one variety


of Langerhans cell histiocytosis. Langerhans cells are part of the mononuclear
phagocyte system (MPS). These dendritic cells are thought to take up foreign antigens
and present them to lymphocytes. Thus, they are important for the immunological
protection of the skin. Dermal fibroblasts are not involved in histiocytosis. Another form
of histiocytosis also involves abnormalities in Langerhans cells in marrow cavities. This
form (unifocal or multifocal Langerhans cells histocytosis) is associated with marked
eosinophilia in the marrow cavities of bones, but does not affect the skin. Melanocytes
and plasma cells are not directly involved in histiocytosis.

Langerhans cells have characteristic Birbeck granules (bodies) in their cytoplasm. In the
electron microscope, these peculiar organelles are pentalaminar, rod-like, and have
terminal dilations. They resemble tennis rackets. Although the function of these
granules is unclear, some theorize that they function in receptor-mediated endocytosis,
much like clathrin-coated pits. Langerhans cells are bone marrow derivatives. Dermal
fibroblasts are not abnormal in histiocytosis. Monocytes in peripheral blood, pulmonary
macrophages, Kupffer cells, osteoclasts, microglia, and histocytes are also included in
the MPS. These cells are dedicated phagocytes. They also have important antigen-
presenting functions.

References:
R. Cotran, et al. Robbins Pathologic Basis of Disease, ed. 6. pp. 685-686
Fawcett, D.W. A Textbook of Histology, ed. 12. pp. 531-533

Keywords: Mononuclear phagocyte system, Langerhans cell, histiocytosis, Birbeck granules, Letterer-Siwe disease

Case #301589:
A 6-month-old child has many reddish papules and scaling plaques on its chest, back,
and scalp. In addition, this child has hepatosplenomegaly and lymphadenopathy. An
immunohistochemical analysis of a skin biopsy reveals numerous rounded dermal
cells with the CD1a marker. Electron microscopic analysis of abnormal dermal cells
reveals that some have pentalaminar rods with terminal dilations. Further tests make
the diagnosis of Letterer-Siwe disease.

Question: 27

The cell that is abnormal in this disease is MOST closely related to which other cell
type in the body?

Answer Choices:
A lymphocyte
B fibrous astrocyte
C chondrocyte
D microglial cell
E osteocyte
D microglial cell

Letterer-Siwe disease (acute disseminated Langerhans cell histiocytosis) is one variety


of Langerhans cell histiocytosis. Langerhans cells are part of the mononuclear
phagocyte system (MPS). These dendritic cells are thought to take up foreign antigens
and present them to lymphocytes. Thus, they are important for the immunological
protection of the skin. Langerhans cells have characteristic Birbeck granules (bodies) in
their cytoplasm. In the electron microscope, these peculiar organelles are pentalaminar,
rod-like, and have terminal dilations. They resemble tennis rackets. Although the
function of these granules is unclear, some theorize that they function in receptor
mediated endocytosis, much like clathrin-coated pits. Langerhans cells are bone
marrow derivatives. Monocytes in peripheral blood, pulmonary macrophages, Kupffer
cells, osteoclasts, microglia, and histiocytes are also included in the MPS. These cells
are dedicated phagocytes. They also have important antigen-presenting functions.
Lymphocytes have a wide variety of immunological functions but are not directly
involved in histiocytosis. Fibrous astrocytes are central nervous system glial cells, but
unlike microglia, they are not part of the MPS. Chondrocytes are the cells of cartilage.
Osteocytes are the cells of bone. Neither is involved directly in histiocytosis.

SECTION- ADIPOSE TISSUE


Question: 1

During extensive periods of stress or heightened excitement, adipose cells will mobilize lipid stores.
This is due, in part, to which of the following mechanisms

Answer Choices:
A Activation of lipase by the release of acetylcholine from the parasympathetic nervous
system
B Activation of lipase by the release of norepinephrine from the parasympathetic
nervous system
C Activation of lipase by the release of norepinephrine from the sympathetic nervous
system
D Activation of lipase by the release of acetylcholine from the sympathetic nervous
system
E Activation of lipase by the release of insulin from the pancreas
C Activation of lipase by the release of norepinephrine from the sympathetic nervous system

The neural mobilization of lipid occurs by the release of norepinephrine from the sympathetic
nervous system. Norepinephrine acts upon receptors present in the adipose tissue to initiate a
cascade of metabolic changes leading to activation of lipase. Lipase mobilizes the lipid by
cleavage of the triglycerides. This action may occur during extensive periods of stress,
excitement, extended fasting or exposure to cold. Secretion of insulin from the pancreas
enhances conversion of glucose into triglycerides forming additional lipid in the adipocyte.

Question: 2
Which of the following statements about sex differences in adipose tissue deposition is true?

Answer Choices:
A Sex differences in body fat distribution begin in utero
B Prepubertal boys have more body fat than girls
C Estrogens and progesterone cause women to develop more fat in the lower body
D Men and women have similar percentages of body fat; only the distribution is
different
E Fat is preferentially deposited in the breast in females but not males
C Estrogens and progesterone cause women to develop more fat in the lower body

The differences in fat deposition between males and females are not noted until early
childhood. Then girls begin to develop more fat than boys. During and after puberty, estrogens
and progesterone cause body fat in women to be deposited lower in the body than in men
(trochanteric distribution). Women also have a higher body fat content than men.
Fat is preferentially deposited in the breast in both men and women.

Question: 3 CLASS QUIZ

The structural difference between white adipose tissue and brown adipose tissue that allows brown
adipose tissue to readily oxidize lipids to produce heat is the presence of cytochrome oxidase
pathway and the absence of what structure?

Answer Choices:
A Golgi
B Mitochondrial elementary particles
C Nuclear pores
D Endoplasmic reticulum with ribosomes
E Plasma membranes
B Mitochondrial elementary particles

Brown adipose tissue, or multilocular adipose, is present in large amounts in the newborn since
there is a high surface-to-mass ratio. The presence of the brown adipose tissue serves as a
readily available source of lipid which can be rapidly oxidized to produce heat that is used to
warm the blood flowing through the brown fat. White adipose tissue, or unilocular adipose, is
present in the newborn, but is not readily metabolized to yield heat. Heat production in brown
fat is due to the presence of cytochrome oxidase allowing the mitochondria to produce energy
for dissipation. The mitochondria of white adipose cells contain enzymes for ATP production
and will preferentially produce ATP rather than heat when the lipid is oxidized. The
mitochondria of brown adipose tissue are unique since they do not contain elementary
particles upon which are many of the enzymes for ATP production.

Question: 4
Fat is a source of

Answer Choices:
A Calories and oxygen
B Calories and antioxidants
C Calories and mucin
D Calories and water
E Calories and complex carbohydrates
D Calories and water

Fat is a very efficient way of storing calories - twice as many calories can be stored in a given
volume of fat as in the same volume of carbohydrate or protein. When necessary, fat can also
be metabolized to produce water (a camel's hump is composed of fat).

Question: 5
Lipoprotein lipase, which catalyzes the transfer of triglycerides from vascular lipoproteins to adipose
tissue, is produced in which cell type?
Answer Choices:
A Vascular endothelial cells
B Vascular smooth muscle cells
C Adipocytes
D Primitive mesenchymal cells of the adventitia
E Fibroblasts
C Adipocytes

Lipoprotein lipase, which catalyzes the transfer of triglycerides from vascular lipoproteins to
adipocytes, is made by adipocytes. It then migrates to the endothelial surface of a closely
associated blood vessel, where it performs its function.
This interaction between adipose tissue and the vascular system explains in part the synergy
between developing blood vessels and adipocytes

Question: 6
Which of the following statements is true about the enzyme histochemistry of developing adipocytes?

Answer Choices:
A Lipoprotein lipase is found only in adipose tissue
B Glucose-6-phosphate dehydrogenase is a marker for late-stage adipocytes
C Malate dehydrogenase is a marker for mid-stage adipocytes
D Preadipocytes are characterized by lipase activity
E The amount of lipoprotein lipase in mature adipocytes is regulated by insulin
E The amount of lipoprotein lipase in mature adipocytes is regulated by insulin

Enzyme histochemistry can be used to identify adipose tissue.


Preadipocytes have only relatively nonspecific enzyme activity, but more mature adipocytes
develop dehydrogenase, reductase and lipase activity. The specific enzymes that have been
identified are glucose-6-phosphate dehydrogenase, malate dehydrogenase, NADH-tetrazolium
reductase and ADPH-tetrazolium reductase. Malate dehydrogenase is found only in very late
stage adipocytes.
Lipoprotein lipase (or esterase) is present in mature adipocytes, but it can also be found in
endothelial cells, especially in cardiac and skeletal muscle. Also, the levels of lipoprotein lipase
vary with insulin levels
Question: 7
What is the primary composition of the lipid in white adipocytes?
Answer Choices:
A Triglycerides
B Cholesterol
C Fatty acids
D Phospholipid
E Lipoprotein
A Triglycerides

99% of the lipid in white adipose tissue is stored as triglycerides. In malignant lesions, the lipid
contains more phospholipid and cholesterol. Nile blue sulfate staining and the hot stage
polarizing light microscope are histologic methods of determining the biochemical make-up of
lipids in tissue

Question: 8
Which of the following is true of postnatal white adipose tissue development?

Answer Choices:
A The number of adipose cells a human has is fixed at birth
B In the first six months of life, adipocytes increase in size, but not in numbers
C Once formed, adipocytes do not disappear
D Until puberty, adipocytes increase in both size and number
E The number of adipose cells a human has is fixed at puberty
B In the first six months of life, adipocytes increase in size, but not in numbers

At birth, the newborn has approximately 5 billion fat cells, about 16% of the number of
adipocytes it will eventually develop. During the first six months of life the baby's adipocytes
increase in size but not number. From then until puberty the cells increase primarily in number.
At puberty the adipose cells increase rapidly in both size and number until the child develops a
number of adipocytes that is very similar to that of the adult.
The number of adipocytes can change after puberty. Overdistension of the fat cells stimulates
the development of new adipocytes and strict dieting can result in the disappearance of
adipocytes.

Question: 9
Explain the presence of large amounts of rough endoplasmic reticulum (rER) in preadipocytes,
something usually associated with cells that make and secrete large amounts of protein:
Answer Choices:
A Lipids are synthesized by rER
B Lipids are metabolized by rER
C Preadipocytes make fibrinogen for the basal lamina
D Preadipocytes make procollagen for the basal lamina
E Preadipocytes make lipoprotein lipase for endothelial cells
E Preadipocytes make lipoprotein lipase for endothelial cells

Preadipocytes have a well developed rough endoplasmic reticulum because they are making
and secreting lipoprotein lipase, a protein that breaks down lipoproteins in the blood stream.
After it leaves the lipoblast, this protein moves to the endothelial surface of the nearby
capillaries. Once there, it begins producing free fatty acids which are taken up by the adipocytes

Question: 10
Which of the following statements about the distribution of white adipose tissue is FALSE?

Answer Choices:
A Large accumulations of adipose tissue are found surrounding the kidneys
B Hormone levels control fat distribution patterns
C The hypodermis, or subcutaneous fat, is particularly well developed over the
abdomen, buttocks, axilla and thighs
D Internally, fat is most prominent in the mesentery, omentum and retroperitoneal
space
E Identical twins may differ markedly in adipose tissue distribution
E Identical twins may differ markedly in adipose tissue distribution

The distribution of adipose tissue is controlled genetically and by hormone levels. Differences in
fat distribution resulting from differing hormone levels are primarily responsible for the
respective body habitus of men and women. Subcutaneous adipose tissue is most prominent
over the abdomen, buttocks, axilla and thighs. Estrogens and progesterone are thought to
predispose women to fat deposits around the hips and upper thighs, while adipose tissue in the
man is centered around the abdomen.
Internally, adipose tissue is found in the mesentery, omentum and retroperitoneal space. Very
large deposits are found surrounding each kidney.
There is a genetic component that influences adipose tissue distribution. Identical twins may
develop different amounts of adipose tissue, but the distribution of the adipose tissue is the
same for each twin
Question: 11
Developing adipose tissue maintains a close association with what other type of tissue?

Answer Choices:
A Blood vessels
B Nerves
C Smooth muscle
D Collagen
E Elastic fibers
A Blood vessels

Adipose tissue develops from immature mesenchymal cells arranged around proliferating
primitive vessels, and later, capillaries. As one might guess, adipose tissue appears first in the
well vascularized parts of the fetus. This close physical association between adipose tissue and
blood vessels is maintained throughout the life of the organism.

Question: 12
The adipose tissue underlying the skin of the palms of the hand, the soles of the feet, the orbit of the
eye and the fat surrounding the heart differs from other adipose tissue because it is

Answer Choices:
A Brown fat
B White fat
C Formed earlier in fetal life than other fat
D Structural fat
E More prone to malignant degeneration than other adipose tissue

D Structural fat

The fat under the palms of the hands, the soles of the feet, the orbit and the fat surrounding
the heart is called structural adipose tissue. This fat softens the force of the impacts these
tissues regularly receive and is only metabolized under extreme conditions.

Question: 13
Which one of the following statements about the multilocular adipose tissue is INCORRECT?

Answer Choices:
A The tissue cells are polygonal. Their cytoplasm contains a great number of lipid
droplets of different sizes, a spherical and central nucleus, and numerous mitochondria
with abundant long crista
B The tissue cells are signet ring-shaped, as each cell, in standard microscopic
preparations, appears as a thin ring of cytoplasm surrounding the vacuole left by the
dissolved lipid. Electron microscope studies reveal that each cell usually possesses minute
lipid droplets in addition to a single large droplet; the droplets are not surrounded by a
membrane. These cells have eccentric and flattened nucleus
C The tissue resembles an endocrine gland in that its cells assume almost epithelial
arrangement of closely packed masses associated with blood capillaries
D The tissue is subdivided by partitions of connective tissue into lobules
E The cells receive direct sympathetic innervation
B The tissue cells are signet ring-shaped, as each cell, in standard microscopic preparations,
appears as a thin ring of cytoplasm surrounding the vacuole left by the dissolved lipid. Electron
microscope studies reveal that each cell usually possesses minute lipid droplets in addition to a
single large droplet; the droplets are not surrounded by a membrane. These cells have eccentric
and flattened nucleus

The description under this answer pertains to the unilocular (white) adipose tissue.
The remaining answers are correct.
Multilocular (Brown) adipose tissue cells are polygonal. Their cytoplasm contains a great
number of lipid droplets of different sizes, a spherical and central nucleus, and numerous
mitochondria with abundant long crista. The tissue resembles an endocrine gland in that its
cells assume almost epithelial arrangement of closely packed masses associated with blood
capillaries. It is subdivided by partitions of connective tissue into lobules. The cells receive
direct sympathetic innervation.
Question: 14
The adipocytes of unilocular adipose tissue contain all of the following components, EXCEPT

Answer Choices:
A A flattened nucleus
B Receptors for insulin
C Many small fat droplets
D A thin rim of cytoplasm
E A basal lamina
C Many small fat droplets

White adipose tissue is composed of unilocular adipocytes with a flattened nucleus. The cells
contain a single large fat droplet, which squeezes the cytoplasm and nucleus to the periphery.
Brown adipose tissue is composed of multilocular adipocytes (but the nucleus is not flattened),
which contain many small fat droplets. Both tissues contain receptors for insulin, a thin rim of
cytoplasm and a basal lamina.

Question: 15
The cell type indicated by the asterisks in the attached image may be identified as

Answer Choices:
A Spongiocytes
B Chromophobes
C Goblet cells
D Adipocytes
E Macrophages

Image(s) / Chart(s):
Click image to view full size. Click open image to close. Click and hold open image to move.

D Adipocytes

Due to the lipid inclusion which fills the cytoplasm of the adipocyte, little more than a shell of
cytoplasm remains at the periphery of the cell, and the nucleus is compressed in an eccentric
position. When processing of the tissue occurs and H&E stains are used, the lipid material is
"washed out" and their is little or no staining of the cytoplasmic area, giving rise to what appear
to be large cells with complete open spaces. Because of the ring appearance with the
eccentrically located nucleus, these cells are sometimes referred to as "Signet-rings". A
spongiocyte is a cell located in the adrenal cortex that produces glucocorticoids. The
chromophobe is a small, poorly staining cell located in the anterior pituitary gland. The goblet
cell is a unicellular mucous-producing gland that is located throughout the respiratory tract and
alimentary system. Macrophages are large cells which have large kidney-shaped nuclei and
extensive cytoplasmic projections.
Question: 16
The breast of the non-lactating adult female is composed mainly of what tissue?

Answer Choices:
A Adipose tissue
B Dense irregular connective tissue
C Secretory acini
D Elastic cartilage
E Dense regular connective tissue
A Adipose tissue

Adipose tissue is distributed in the connective tissue of the dermis (hypodermis) throughout
the body of both sexes. The breast, however, is a preferential site for accumulation of adipose
tissue and the non-lactating breast in the adult female is composed primarily of adipose tissue.
Secretory acini and associated duct systems develop during pregnancy and with preparation for
lactation as a major histological component of the breast. These glandular elements regress
following the cessation of lactation and are replaced by adipose tissue. Elastic cartilage is never
normally found in the breast. Dense irregular connective tissue may be found spread among
the adipose tissue, serving as areas of structural support. Suspensory ligaments associated with
breast tissue may have some characteristics of dense regular connective tissue. Neither dense
irregular connective tissue, nor dense regular connective tissue are major components of breast
tissue.

Question: 17
One cell type that constitutes a specialized form of connective tissue is characterized by cells that are
capable of accumulating extensive amounts of lipid. The singe lipid inclusion occupies the central
portion of the cytoplasm, eventually compressing the nucleus to an eccentric position in the cell. This
cell is termed a

Answer Choices:
A Spongiocyte
B Adipocyte
C Chromophobe
D Melanocyte
E Goblet cell

B Adipocyte

The mature adipocyte is characterized by a singular lipid inclusion in the central portion of the
cytoplasm which compresses the nucleus to an eccentric position in the cell, as well as
displacing all other cytoplasmic organelles to the periphery of the cell. The lipid inclusion is not
membrane bound. Compression of the nucleus to an eccentric location and filling of the
cytoplasm with the lipid inclusion produces an adipocyte that has a "signet-ring" appearance in
the mature state. A spongiocyte is a cell located in the adrenal cortex that produces
glucocorticoids. The chromophobe is a small, poorly staining cell located in the anterior
pituitary gland. The melanocyte is located in the epidermis and produces the pigment, melanin.
The goblet cell is a unicellular mucous-producing gland that is located throughout the
respiratory tract and alimentary system.

Question: 18 CLASS QUIZ


A full-term newborn baby girl found abandoned in a city alley was brought to the Emergency
Department. The ambient temperature was approximately 40F. The baby was found wrapped only in
newspaper. Vital signs and physical exam revealed a normal, healthy infant. One mechanism by which
body temperature may have been maintained was through the presence of what tissue in the
neonate?

Answer Choices:
A Brown adipose tissue
B White adipose tissue
C Dense connective tissue
D Mesenchyme tissue

A Brown adipose tissue

Brown adipose tissue, or multilocular adipose, is present in large amounts in the newborn
because there is a high surface-to-mass ratio. The presence of brown adipose tissue serves as a
readily available source of lipid, which can be rapidly oxidized to produce heat that is used to
warm the blood flowing through the brown fat. White adipose tissue, or unilocular adipose, is
present in the newborn but is not readily metabolized to yield heat. Heat production in brown
fat is due to the presence of cytochrome oxidase, allowing the mitochondria to produce energy
for dissipation. The mitochondria of white adipose cells contain enzymes for ATP production
and will preferentially produce ATP rather than heat when the lipid is oxidized.

Question: 19
The primary purpose of brown fat is

Answer Choices:
A Efficient storage of calories
B Mechanical insulation
C Thermal insulation
D Heat production
E Storage of cholesterol

D Heat production

Brown" fat differs from "white" fat in that brown fat has a singular mitochondria enzyme called
thermogenin that uncouples fatty acid oxidation from ATP production. This uncoupling raises
the heat produced by fatty acid oxidation by more than 20%. Hibernating animals and rodents
use this nonshivering thermogenesis to survive during the winter. It is thought that the
presence of brown fat in neonates reflects the otherwise relatively poor thermoregulatory
ability of newborns

Question: 20
The immature lipocyte, or lipoblast, is morphologically identical to the

Answer Choices:
A Fibroblast
B Osteoblast
C Germinal squamous epithelial cell
D Vascular endothelial cell
E Smooth muscle cell

A Fibroblast

Lipoblasts, or preadipocytes , appear to develop from undifferentiated mesenchymal cells


present in the adventitia of small veins and venules. The fibroblasts and myofibroblasts found in
healing wounds also appear to develop from this cell and may be indistinguishable from
lipoblasts at the early stages of their development.

Case #20290:
6 months after undergoing a Roux-en-Y gastric procedure for the treatment of morbid
obesity, a 37-year-old man presents with hematuria. He notes flank pain radiating to his
groin. He denies fever and chills, recent respiratory symptoms, recent rash, or recent dysuria.
He has had intermittent mild watery stools and flushing for the past 6 months, particularly
after eating sweets. He has no other bleeding sites and no recent trauma. His medications
include multiple vitamin supplements. His family history is negative for hematuria or kidney
disease.
pH 8, specific gravity 1.020, numerous red blood cells, no white
Urinalysis
blood cells, no casts, trace protein, no leukocyte esterase, no
nitrate
Kidney ureter bladder X-ray Calcified material in ureter area
Urine culture No growth

Question: 21
What is the most likely diagnosis?

Answer Choices:
A Calcium oxalate stones
B Struvite triple phosphate stones
C Uric Acid stones
D Citric Acid Stones
E Primary hyperoxaluria
A Calcium oxalate stones

This patient most likely is forming calcium oxalate stones. About 70% of stones formed are
calcium (Johnson). The gastric procedure above results in a restriction of food intake by limiting
the stomach size and causing malabsorption of fats and vitamins. Non-absorbed fats will bind
gastrointestinal calcium, leaving less calcium available to complex with oxalate; excess oxalate
may be excreted in the urine. Other gastrointestinal diseases that cause malabsorption are
known to cause calcium oxalate stones as well.
Calcium oxalate stones typically form at high urine pH and at low urine volumes.
Diets rich in oxalates (chocolates, teas, nuts, leafy green vegetables, etc.) may promote stone
formation. Oxalate stones have been reported following therapy with Orlistat, an inhibitor of
gastric and pancreatic lipase (Aisling). Oxalate stones may be treated by increasing fluid intake,
decreasing dietary oxalate, and using calcium carbonate with meals in order to bind up
intestinal oxalate.
Struvite (triple phosphate) stones form in urine infected with urease-producing bacteria, (e.g.
Proteus). These bacteria split urea into ammonium. The lack of urine growth or recent urinary
infections makes struvite stones unlikely.
Uric acid stones form in acidic urine and are radiolucent, making them unlikely in a patient with
a urine pH of 8 and the notation of calcified material on his kidney ureter bladder X-ray.
Citric acid inhibits stone formation. Urinary citrate levels can be assessed with a 24-hour urine
collection. A 24-hour urine excretion less than 320 mg is considered low, and as such is a risk
factor for stone formation. Citric acid is not a common ingredient in urine crystals; given the
history of gastric surgery, oxalate stones are more likely.
Primary hyperoxaluria is a genetic disorder whereby hepatic enzymes (alanine glyoxylate
aminotransferase) are deficient and oxalate is produced in excess. Oxalate deposits in various
organs including the liver, kidneys, etc. Liver transplantation or liver + renal transplantation in
patients with renal failure has successfully treated this disorder.
Case #20291:
A new patient of yours wants to lose weight. His previous attempts with diet and exercise
have not been long lasting. He was working in construction, but has become more easily
winded over the past year, and so he hasn't been working. His review of systems is otherwise
negative. His family history is notable for obesity (his parents are heavy, and his sister had
gastric bypass surgery); and both parents have diabetic nephropathy. His Body Mass Index
(BMI) is 55 kg/m2. He is 5 feet 10 inches tall. His blood pressure is 120/80 mmHg. His
cardiopulmonary exam is grossly normal, as is his retinal exam.
Electrocardiogram Sinus rhythm, otherwise normal
Serum creatinine 0.8 mg/dl
Hemoglobin 13 g/l
Fasting blood glucose 96 mg/dl
Serum bicarbonate 25 meq/l

Question: 22
What modifiable factor puts him at most risk for developing renal failure?

Answer Choices:
A Sibling with history of gastric bypass
B Family history of nephropathy
C Personal history of obesity
D Personal history of diabetes
E Personal history of retinopathy
C Personal history of obesity

This patient's obesity is both modifiable and puts him at high risk for developing progressive
renal failure.
The patient reports that previous weight loss attempts have not lasted. A variety of treatments
for weight management are available, including caloric reduction, increasing exercise, inhibitors
of gastrointestinal absorption, agents to increase satiety, counseling, etc. Gastric surgeries,
either those that restrict intake by reducing stomach size or those that reduce absorbed
nutrients by causing malabsorption, are indicated for the treatment of severe obesity. In
patients with associated risk factors, such as diabetes and sleep apnea, such procedures may be
indicated for patients if BMI exceeds 35 kg/m2; in patients without associated risk factors,
these procedures may be indicated for BMI greater than 40 kg/m2 (DeMaria).
The risk for developing progressive renal failure increases incrementally with increasing body
mass index (Kramer, Ejerblad). Patients with BMI > 40 kg/m2 have approximately 7-fold
increased risk of developing progressive renal failure compared to the non-obese (<30 kg/m2)
population. Increased BMIs pose a risk for renal failure even in patients without diabetes and
hypertension (Ejerblad).
In obese patients, a fixed number of nephrons are challenged to process the fluids and
nutrients of increased body mass. They respond by hyperfiltering, increasing glomerular
volume, and decreasing podocyte number/density, eventually leading to obesity-related
glomerulopathy and/or obesity-related glomerular scarring (Hui-Mei).
Weight loss of even 5 - 10% can significantly improve patients' risk factors for obesity-
associated disease (Aisling). Given his severely elevated BMI, a gastric surgical procedure is
likely indicated.
His family history of kidney disease is unfortunately not modifiable.
This patient is at risk for diabetes given his obesity. He does not meet criteria for diabetes,
given his fasting blood glucose <126 mg/dl. Diabetes is a strong risk factor for kidney disease
and should be screened for in at-risk individuals.
No retinopathy was detected on examination of his eye grounds. A fully dilated retinal exam by
an experienced retinal ophthalmologist would be the best way to detect retinopathy. Of note,
retinal changes can be seen in the pre-diabetic state in some individuals.

Case #20292:
A 42-year-old morbidly obese woman wants to lose weight. Previous attempts with dietary
modification and exercise have been unsuccessful. She has never taken any stimulants or
weight loss drugs. She has no history of excessive analgesic use, joint disorders, pain,
paresthesias, or hematuria. She has seasonal allergic rhinitis and takes antihistamines in the
springtime as needed. Her body mass index (BMI) is 40.5 kg/m2. Her blood pressure is 135/78
mmHg. Her eye grounds are normal. She has mild lower extremity edema, good peripheral
pulses, and no venous stasis.
Her yearly laboratories show:
Renal ultrasound Normal size, no cortical thinning, no masses
1.020, pH 6, trace protein, no blood, no cells, no casts, no crystals,
Urinalysis
no glucose
Urine albumin/creatinine ratio 40 mg/g
Serum calcium, corrected 9.5 mg/dl
Serum phosphorus 3.5 mg/dl

Serum creatinine 1.2 mg/dl


Serum bicarbonate 24 meq/l
Serum potassium 4 meq/l
Serum chloride 100 meq/l
Hepatitis B Antibody and Antigen, Hepatitis C
All negative
antibody
Antineutrophil antibody Negative
Fasting serum glucose 100 mg/dl
Complement 3,4 Normal

Question: 23
What is the most likely cause of her proteinuria?
Answer Choices:
A Diabetic kidney disease
B Obesity-related glomerular disease
C Membranoproliferative glomerulonephritis
D Hypertensive nephrosclerosis
E Tubular disease
B Obesity-related glomerular disease

Given the lack of findings to support alternate listed diagnoses, obesity-related glomerular
disease is the most likely cause of this patient's proteinuria.
Obesity is an increasingly recognized modifiable risk factor for progressive kidney disease. The
risk for developing progressive renal failure increases incrementally with increasing body mass
index. Patients with BMI >40 kg/m2 have approximately 7-fold increased risk of developing
progressive renal failure compared to the non-obese (<30 kg/m2) population. Increased BMIs
pose a risk for renal failure even in patients without diabetes and hypertension.
In obese patients, a fixed number of nephrons are challenged to process the fluids and
nutrients of an increased body mass. They respond by hyperfiltering, increasing glomerular
volume, decreasing podocyte number/density, and eventually leading to obesity-related
glomerulopathy (proteinuria, elevated serum creatinine) and/or obesity related glomerular
scarring (glomerular sclerosis, proteinuria).
Weight loss of even 5 - 10% can significantly improve patients' risk factors for obesity-
associated disease. Given her severely elevated BMI, a gastric surgical procedure is likely
indicated. BMI greater than 40 kg/m2 is an indication for such procedures.
Diabetic kidney disease is a well-known cause of renal failure. It is one of the leading causes of
end-stage renal disease in the United States. This patient does not meet criteria for diabetes
(fasting glucose >126 mg/dl, random glucose >200 mg/dl, etc.). She also had no glycosuria.
Although microvascular disease may precede the diagnosis of diabetes, no diabetic changes
were noted on her retinal exam.
Hypertensive nephrosclerosis is also a very common cause of progressive kidney failure in the
United States, but is unlikely the cause of this patient's proteinuria. It occurs at 2 - 3 times the
rate in patients with consistently elevated pressures (150 - 160/80 mmHg+) than in patients
with pressures in the 130-140/80 mmHg range. This patient's current pressure is 135/78 mmHg.
If her pressure was consistently at this level, we'd call her pre-hypertensive. Knowing that her
creatinine is elevated and that she has proteinuria, a more desirable pressure would be in the
120 - 130/70-80 mmHg range in order to best prevent progressive renal function loss.
Membranoproliferative renal disease typically presents with proteinuria and hematuria.
In some cases, hypocomplementemia, positive hepatitis titers, and/or positive antineutrophil
antibodies are also noted. The lack of these findings makes this diagnosis unlikely.
Tubular disease may lead to low levels of proteinuria. Compared to glomerular disease, much
lower levels of protein are found in the urine. Also expected in tubular disease would be some
electrolyte or acid-base dysfunction, since the tubules are key in regulating acid base and
mineral reabsorption and excretion. The normal urine pH, serum bicarbonate, serum chloride,
serum potassium, serum calcium, and phosphorus make this a less likely diagnosis.

Case #20293:
A 45-year-old morbidly obese man (BMI 40.5 kg/m2) returns to your office. At the last visit,
trace proteinuria and a serum creatinine of 0.8mg/dl were noted. No other abnormalities
were noted on his urinalysis. Subsequently, a 24-hour urine showed:
100 mg protein
Estimated creatinine clearance 130 ml/min
Additional workup for his proteinuria showed:
Hepatitis B and C antibodies Negative
Serum and urine protein electrophoresis Normal
Antinuclear antibody Negative
Complement 3,4 Normal
Serum sodium, potassium, chloride, bicarbonate,
All normal
calcium, phosphorous, magnesium, glucose
Serum creatinine 0.7 mg/dl
Hemoglobin 13.5 g/l

Question: 24
How should you treat his proteinuria?

Answer Choices:
A Prescribe lisinopril alone
B Prescribe losartan alone
C Prescribe aliskiren alone
D Prescribe weight loss regimen
E Weight loss and lisinopril
E Weight loss and lisinopril

Since this patient's workup for proteinuria revealed 100 mg/day urinary protein excretion and
no potential etiology other than obesity, he should be started on an antiproteinuric agent and
prescribed a weight loss program.
Treating for a presumptive diagnosis of obesity related kidney disease is appropriate.
If they had been detected, antinuclear antibodies and reduced complement levels may have
been suggestive of immunologic-based glomerular diseases as the cause of proteinuria.
Likewise, hepatitis serologies and abnormal protein electrophoresis may have detected
hepatitis-related glomerulopathies or monoclonal gammopathies as the cause of the
proteinuria. Each of these pathologies could be confirmed by renal biopsy.
Obesity is an increasingly recognized modifiable risk factor for progressive kidney disease. The
risk for developing progressive renal failure increases incrementally with increasing body mass
index. Patients with BMIs >40 kg/m2 have approximately 7-fold increased risk of developing
progressive renal failure compared to the non-obese (<30 kg/m2) population. Increased BMIs
pose a risk for renal failure even in patients without diabetes and hypertension.
In obese patients, a fixed number of nephrons are challenged to process the fluids and
nutrients of increased body mass. They respond by hyperfiltering, increasing glomerular
volume, decreasing podocyte number/density, and eventually leading to obesity-related
glomerulopathy (proteinuria, elevated serum creatinine), and/or obesity related glomerular
scarring (glomerular sclerosis, proteinuria).
Weight loss of even 5-10% can significantly improve patients' risk factors for obesity-associated
disease. His BMI of 40.5 kg/m2 is an indication for gastric bypass surgery.
To treat his proteinuria, a variety of antiproteinuric agents are available including lisinopril, an
ACE inhibitor, and losartan, an angiotensin II receptor blocker. Aliskiren is a new agent which
directly inhibits renin and thereby inhibits the conversion of angiotensinogen to angiotensin I.
Aliskiren is indicated for the treatment of hypertension; since it is relatively new, it has not
been as extensively investigated as the other agents in the management of proteinuria.
Prescribing an antiproteinuric agent alone is not advised, given his obesity and its known renal
risk. Addressing his obesity with a weight loss regimen is crucial.
Only one of the above choices combines weight loss with an antiproteinuric agent.

Case #20294:
A 45-year-old morbidly obese woman develops new onset flank pain and hematuria 2
months after initiating Orlistat. A urinalysis shows gross blood, but it is otherwise normal. A
CT scan shows a 2 mm stone in her right ureteropelvic junction. With hydration and
analgesics, her symptoms improve. After 8 weeks, her symptoms resolve, and a 24-hour
urine shows:
Volume 1.1 liters
pH 6.5
Calcium 180 mg (normal)
Citrate 330 mg (normal>320 mg)
Oxalate 75 mg (normal<40 mg)
Creatinine clearance 90 ml/min/1.73 m2
Additional labs serum liver function tests Normal

Question: 25
She wants to continue Orlistat therapy. How should you prevent stone recurrence?

Answer Choices:
A Continue 1.1 liter/day fluid intake
B Prescribe potassium citrate supplements
C Prescribe sodium bicarbonate supplements
D Prescribe calcium supplements with meals
E Prescribe sodium citrate supplements
D Prescribe calcium supplements with meals
This patient had a single episode of flank pain and hematuria, presumably due to oxalate-based
kidney stones. She should be prescribed calcium supplements to be taken with meals.
Orlistat has been reported in association with calcium oxalate stones. Orlistat inhibits gastric
and pancreatic lipase, leading to malabsorption of ingested fats and their elimination in the
stool. These fats may bind calcium, leaving gastrointestinal oxalates unbound and available for
reabsorption and deposition in the renal tubules.
In her workup for stones, a 24-hour urine showed increased levels of oxalate.
Hyperoxaluria can be treated by using calcium supplements with meals. Dietary calcium binds
to oxalate and allows it to be eliminated in the stool. Primary hyperoxaluria is a genetic disorder
whereby hepatic enzymes (alanine glyoxylate aminotransferase) are deficient and oxalate is
produced in excess. Oxalate deposits in various organs including the liver, kidneys, etc. Liver
transplantation or liver + renal transplantation in patients with renal failure has successfully
treated this disorder. Cholestyramine is also used to bind dietary oxalate.
Hypocitraturia can be a risk for renal stones because citrate inhibits stone formation; however,
no hypocitraturia was noted, making the need to prescribe sodium citrate unnecessary. Excess
sodium intake can exacerbate calcium-based stones by allowing for increased renal calcium
excretion. Sodium bicarbonate use is therefore not advised in this patient. Bicarbonate
supplements may further alkalinize the urine. Since calcium stones precipitate at high pH,
bicarbonate supplements may exacerbate their formation.
1.1 liters daily fluid intake is likely insufficient for the prevention of stones. In most patients,
urine is concentrated with this volume of intake; concentrated urine promotes stone formation.

SECTION- SKELETAL MUSCLE

Question: 1

What is the function of the cells whose nuclei can be identified in a row located at the tips of the
arrows in the photomicroscopic image?

Answer Choices:
A Secretion of alkaline phosphatase
B Secretion of Type II collagen fibers
C Secretion of Type IV collagen fibers
D Secretion of calcitonin
E Dissolution of calcium salts

Image(s) / Chart(s):
Click image to view full size. Click open image to close. Click and hold open image to move.
A Secretion of alkaline phosphatase

During bone development and maintenance, small cuboidal osteoblasts located on the surface
of the bone (outer, or inner in the presence of a marrow cavity), retain the ability to secrete
both collagen and ground substance that constitutes the matrix of un-mineralized bone or
osteoid. The osteoblast secretes small vesicles into the matrix of the osteoid, which are rich in
alkaline phosphatase. The membrane-bound matrix vesicle will rupture producing an increase
in local concentration of enzymes (alkaline phosphatase) and mineral to initiate the process of
mineralization in bone formation. Type II collagen fibers are commonly secreted by
chondrocytes during cartilage formation. Secretion of Type IV collagen fibers occurs by
epithelial cells. Secretion of calcitonin, which reduces osteoclastic activity, is secreted by
parafollicular cells of the thyroid gland. Osteoclasts are the cells that are responsible for
dissolving calcium salts and are large multinucleated cells.

Question: 2 EXAMS
The primary secretory product of the cell located at the tip of the pointer in the attached image is

Answer Choices:
A Type I collagen
B Type II collagen
C Type III collagen
D Type IV collagen
E Type VI collagen
Image(s) / Chart(s):
Click image to view full size. Click open image to close. Click and hold open image to move.

A Type I collagen

The cell located at the tip of the arrowhead is best identified as the location of the osteocyte. In
growing an mature bone, the osteocyte was responsible for secreting and maintaining the
matrix of the bone. The primary fiber found in the matrix in compact bone is type I collagen.
The matrix also contains ground substance consisting of proteoglycans and non-collagenous
glycoproteins which become mineralized in the bone.
Question: 3 BLOCK 1
A 2 year old boy is brought to the Emergency Department by a paramedical team who found him
abandoned in an apartment. It was apparent that he had not received adequate nourishment, and he
was 2 standard deviations below height and weight for his age. While he was in no apparent distress,
it was noted that he had several jagged subcutaneous cuts on his fingers and hands which had failed
to form adequate tissue in their repair. X-ray of the long bones revealed hypolucent bones and a
bowed formation. Among the various problems associated with the development of this child, it is
apparent the child has not received adequate Vitamin C for a substantial period of time. Your
conclusion is based upon the fact that Vitamin C is associated with:

Answer Choices:
A Increased calcium deposition in the long bones
B Production of epithelial cells associated with the wound repair
C Hydroxylation of proline and lysine residues in formation of collagen
D Translation of procollagen within the fibroblasts and osteoblasts
E Inhibition of microtubular transport associated with exocytosis of procollagen

C Hydroxylation of proline and lysine residues in formation of collagen

During the synthesis of collagen fibers, several intracellular events occur. In addition to the
formation of polypeptide chains by translational processes, the polypeptide chains are cleaved
from the signal peptide. The proline and lysine residues are hydroxylated, terminal portions of
the molecules may be glycosylated, and there is a triple helix formed by three polypeptide
chains. Hydrogen bonds stabilize the shape of the resultant procollagen molecule. Vitamin C
(ascorbate) is essential to the function of enzymes that hydroxylate proline and lysine. Lack of
vitamin C results in the formation of the helical procollagen that is not stabilized by the
hydrogen bonds, and the final collagen molecule cannot form, or is weakly formed. Therefore,
wounds that extend into the dermis do not readily heal, and weight bearing bones are
deformed since the collagen molecules associated with the bone matrix are compromised.
Question: 4
An example of a bone that forms by intramembranous ossification is the

Answer Choices:
A Clavicle
B Femur
C Metatarsal
D Phalanges
E Vertebral body

A Clavicle

The only bone on this list that forms by intramembranous ossification is the clavicle. All of the
other bones are "weight bearing" and typically form through a cartilage precursor by
endochondral bone formation. Generally, flat bones, such as the bones of the skull and face,
the mandible and clavicle, develop by intramembranous bone formation.

Question: 5
Which one of the following statements regarding hyaline cartilage is true?

Answer Choices:
A It is a vascular structure
B It contains type IV collagen
C It grows by appositional growth only
D It is located at the articular ends of long bones
E It is located where flexible support is required

D It is located at the articular ends of long bones


Hyaline cartilage is located at the articular ends of long bones.
Hyaline cartilage is avascular, contains type II collagen, and grows both interstitially and
appositionally.
Only elastic cartilage is located where flexible support is required (e.g., in the pinna of the ears,
auditory tube and epiglottis).

Question: 6
The first histologic sign of osteoarthritis in articular cartilage is

Answer Choices:
A An infiltration of the cartilage by leukocytes
B The proliferation of chondrocyte in zone I
C The death of chondrocyte and disappearance of lacunae in zone I
D The widening of the deep, mineralized layer
E The wearing of the cartilage on the surface of layer I along the lines of the Bennington
arcades
E The wearing of the cartilage on the surface of layer I along the lines of the Bennington
arcades

The first histologic sign of osteoarthritis in articular cartilage is a characteristic wearing away of
the surface cartilage. This wearing takes place along the lines of the Bennington arcades formed
by the large collagen bundles of the cartilaginous matrix.
The subsequent pathology of arthritis involves the pathological calcification of the cartilage and
its replacement by bone. Obviously, this causes a considerable limitation of movement.

Question: 7
Identify the cell depicted by the small cuboidal nuclei located at the tips of the arrows in this
photomicroscopic image

Answer Choices:
A Chondroblasts
B Osteoblasts
C Fibroblasts
D Chondrocytes
E Osteoclasts

Image(s) / Chart(s):
Click image to view full size. Click open image to close. Click and hold open image to move.
B Osteoblasts

During bone development and maintenance, small cuboidal cells located on the surface of the
bone (outer, or inner in the presence of a marrow cavity), retain the ability to secrete both
collagen and ground substance that constitutes the matrix of un-mineralized bone or osteoid
and are called osteoblasts. Chondroblasts share some of the same characteristics of
osteoblasts, except they are found in developing cartilage. Chondrocytes differ from
chondroblasts in that the chondrocyte is the mature cell found in cartilage and located within a
lacunae surrounded by the cartilage matrix. Osteoclasts, on the other hand, are large
multinucleated cells that function in bone reabsorption.

Question: 8 CLASS QUIZ


The mineral component of bone is:

Answer Choices:
A Hydroxyapatite - [Ca10(HPO4)6(OH)2]
B Monobasic calcium phosphate
C Calcium phosphate - Ca(H2PO4)2
D Calcium phosphate - CaHPO4
E Calcium phosphate - Ca3(PO4)2

A Hydroxyapatite - [Ca10(HPO4)6(OH)2]
The mineral component of bone is hydroxyapatite, [Ca10(HPO4)6(OH)2]. Hydroxyapatite hardens
the bone so that the it can provide mechanical support for the body and protection for the soft
organs. Also, the mineral in hydroxyapatite can be mobilized if there is a systemic depletion of
calcium.

Question: 9
The tip of the arrowhead is pointing to specific structures that are best identified as which of the
following in the attached photomicroscopic image?

Answer Choices:
A Osteocytes
B Lacunae
C Interstitial lamellae
D Canaliculi
E Volkmann's canals

Image(s) / Chart(s):
Click image to view full size. Click open image to close. Click and hold open image to move.

D Canaliculi

Close inspection of the image reveals small, dark lines seemingly radiating outward from the
circumferentially layered osteocytes. These lines represent miniature "canals", hence canaliculi,
that contained cytoplasmic processes of the osteocytes. Upon preparation of the ground bone,
the cytoplasmic material in the canaliculi is charred, leaving the dark lines that radiate from the
osteocyte lacunae. These cytoplasmic processes of the osteocytes served as communications
from the Haversian canal where blood vessels were located, to osteocytes located in
peripherally located lamellae.
Question: 10
Which one of the following bones have two layers of compact bone (called plates, or tables),
separated by a layer of spongy bone (called diplo')?

Answer Choices:
A Diaphysis of femur
B Epiphysis of ulna
C Flat bones of calvaria
D Diaphysis of radius
E Cervical vertebra atlas

C Flat bones of calvaria

The flat bones that form the calvaria have two layers of compact bone called plate (table),
separated by a layer of spongy bone called the diplo'.
Epiphyses of long bones (e.g., ulna) are composed of spongy bone covered by a thin layer of
compact bone, while their diaphyses are almost totally composed of compact bone, with a
small component of spongy bone on its inner surface around the bone marrow cavity. Short
bone (e.g., vertebrae) usually has a core of spongy bone completely surrounded by compact
bone.

Question: 11
The photomicroscopic image depicts a longitudinal cut through the distal end of a fetal finger. The
region depicted in the attached image by the area denoted as "E" is best described as what
developmental zone in endochondral bone formation?

Answer Choices:
A Zone of reserve cartilage
B Zone of calcification & cell death
C Zone of cellular maturation & hypertrophy
D Zone of proliferation
E Zone of ossification & resorption

Image(s) / Chart(s):
Click image to view full size. Click open image to close. Click and hold open image to move.
A Zone of reserve cartilage

Endochondral bone formation is typified by the formation of a cartilage model, which is later
replaced through a gradual and dynamic process with bone. Important to this model is the
removal of cartilage and the deposition of bone matrix. This process occurs in long bones of the
body, as is demonstrated in this photomicroscopic image through the distal end of a fetal
finger. The process of cartilage removal and replacement of bone matrix continues in an orderly
wave of activity that initially extends from the ends of the bone towards the primary
ossification center. Several regions, or zones, can be distinguished in this process. The letter "A"
denotes the zone of ossification and resorption. Vascular connective tissue invades tunnel-like
spaces, providing access to osteogenic cells that differentiate into osteoblasts and form osteoid
on the surface of the calcified cartilage spicules. All of these calcified cartilage spicules covered
by bone will eventually be resorbed as the process continues distally, and the bone marrow
cavity increases in size.

Resorption is primarily the function of osteoclasts. The letter "B" denotes the zone of
calcification and cell death. In this area the matrix among the cartilage cells begins to
accumulate calcium salts. Unable to access adequate nutrients and oxygen, the chondrocytes,
which are lined in rows die, dissolve and leave empty spaces. These spaces "open up" and form
irregular tunnels through the calcified cartilage matrix, which was once cartilage matrix. The
letter "C" denotes the zone of cellular maturation and hypertrophy. In this area the
chondrocytes mature and form additional cartilage matrix, further increasing the length of the
cartilage in this region. With cell maturation, the lacunae enlarge, forming plates of cartilage
between rows of hypertrophied chondrocytes. The letter "D" denotes a region referred to as
the zone of proliferation. This region contains active, proliferating chondrocytes. The cells
become aligned in columns separated by cartilage matrix. The letter "E" denotes the zone of
reserve cartilage. In this area, the cells and their lacunae are randomly arranged throughout the
matrix, and there is slow growth of cartilage in this region in all directions. This area is
composed of primitive hyaline cartilage, which shortens as the endochondral bone formation
process proceeds.
Question: 12
Which one of the following is an example of a diarthrosis type of joint?

Answer Choices:
A Tibiofibular joint
B Radioulnar joint
C Joint between adult skull bones
D Intervertebral joints
E Knee joint
E Knee joint

Diarthroses are joints that generally unite long bones and have great mobility, such as the knee
joint.
The tibiofibular and radioulnar joints are classified as syndesmosis (union of bone by ligament)
type of joints, joints between adult skull bones are known as synostosis (union of bones by
bone) joints, and the intervertebral joints are the examples of amphiarthrodial or
synchondrodial joints.

Question: 13
The type of bone formation depicted by the attached photomicrograph is referred to as

Answer Choices:
A Mesenchymal
B Intramembranous
C Endochondral
D Appositional
E Interstitial

Image(s) / Chart(s):
Click image to view full size. Click open image to close. Click and hold open image to move.
C Endochondral

Bone formation during development of the fetus is characterized as being either


intramembranous or endochondral. Endochondral, as the name implies, is typified as having a
cartilage model which serves as the precursor of the bone. This occurs through a dynamic
process in which the cartilage model is gradually replaced and bone is deposited, as is
exemplified by the appearance of the various zones that occur during this process, which are
seen in this photomicrograph. Alternatively, intramembranous bone formation occurs without
the intervention of a cartilage precursor, where bone is formed by the differentiation of
mesenchymal cells into osteoblasts in regions where bone is destined to form.

Question: 14
Growth of the epiphyseal cartilage during endochondral bone formation is stimulated by a hormone;
if over-secreted, it may result in an individual in which there is an abnormal increase in the length of
bones and their density. What is the hormone?

Answer Choices:
A Calcitonin
B Parathormone
C Corticotrophin
D Testosterone
E Somatotrophin

E Somatotrophin

Somatotropin, or growth hormone (GH), is secreted from the pituitary gland; it stimulates
growth in general, but it especially stimulates growth of the epiphyseal cartilage during
childhood. Over-secretion of GH during childhood will result in an abnormal increase in the
length and density of long bones, causing a clinical condition referred to as gigantism. Calcitonin
and parathormone also have major effects on bones; they do so by either depressing or
stimulating the deposition of calcium in the matrix of bone, respectively.

Question: 15
Osteoblasts stain for

Answer Choices:
A Alkaline phosphatase
B Acid phosphatase
C Lysozyme
D Non-specific esterase
E Diastase

A Alkaline phosphatase

Osteoblasts stain very intensely for alkaline phosphatase(ALP) and, clinically, two of the
explanations of an elevated alkaline phosphatase level are growing bone (ALP levels in children
are always higher than in adults) and bone lesions.
The ALP is particularly concentrated in the matrix vesicles , which are produced by osteoblasts
and whose contents serve as niduses (or nidi) for the mineralization of osteoid.

Question: 16
The term endochondral bone refers to

Answer Choices:
A Tubular bones
B Short bones
C Bones that are both tubular and short
D Bones that develop directly from primitive mesenchyme
E Bones that develop from a cartilaginous precursor

E Bones that develop from a cartilaginous precursor

Endochondral bone develops from a cartilaginous precursor. Endochondral and membranous


bone are histologically identical in adults, but there are neoplasms that develop only from
certain types of bone. For instance, endochondromas and osteochondromas are tumors of
endochondral bones
Question: 17
Which of the following statements is true of the femoral head?

Answer Choices:
A The medial side of the cortex is thicker than the lateral
B The spicules of the cortex run perpendicular to the maximum lines of the stress placed
on the femoral head
C The articular surface is lined by perichondrium
D The head of the femur is smooth and shiny
E Little remodeling can take place in the femoral head, even under pathologic conditions
such as osteoarthritis or fractures

A The medial side of the cortex is thicker than the lateral

The gross anatomy of the femoral head illustrates some basic principles of bone formation and
maintenance. The head of the femur fits into the ilium and is covered by articular cartilage.
Articular cartilage is not covered by perichondrium. The head of the femur has several blunt
projections where tendons and the joint capsule attach to the bone.
The mechanical forces placed on the femur dictate that the cortex on the medial side be thicker
than on the lateral side. They also force the spicules of the cortex to run parallel to the lines of
force generated.
Osteoarthritis, fracture or aseptic necrosis may cause the lines of stress to change, resulting in
relatively rapid bone remodeling. This rapid bone remodeling means that metabolic deficiencies
that affect bone formation quickly affect the femoral head. Femoral neck fractures are
frequently the first sign of these diseases.

Question: 18
Simple trauma to a synovial joint by a single incident or by repeated insult, can damage the articular
hyaline cartilage to a point that it calcifies and begins to be replaced by bone. This can lead to

Answer Choices:
A Gout
B Tuberculosis
C Rheumatoid arthritis
D Ankylosis

D Ankylosis

Simple trauma to a synovial joint by a single incident or by repeated insult, can damage the
articular hyaline cartilage to a point that it calcifies and begins to be replaced by bone. This can
lead to ankylosis, i.e., bony fusion in the joint, and subsequent loss of motion.
Gout, or gouty arthritis is caused by the deposition of crystals of uric acid in the joints,
particularly those of the toes and fingers. Immune responses or infectious processes that
localize in joints, as in rheumatoid arthritis or, tuberculosis, can also damage the articular
cartilages, producing both severe joint pains and gradual ankylosis.

Question: 19 BLOCK-1
The disease osteogenesis imperfecta is associated with mutations in which of the following proteins?

Answer Choices:
A Type I collagen
B Fibronectin
C Collagenase
D Alkaline phosphatase
E Prolyl hydroxylase

A Type I collagen

Collagen is a structural protein composed of three polypeptide chains each containing


approximately 1000 amino acids. The three chains form a triple helix with every third amino
acid being a glycine. Collagen has the general structure (X-Y-Gly)333 where X is Proline in
approximately 100 units and Y is Hydroxyproline in approximately 100 units. This pattern is
necessary for stereochemical reasons for the triple helix to properly form. There are several
different types of collagen that vary in tissue distribution, individual chains making up the helix,
and chemical composition.
Collagen is synthesized as the precursor, preprocollagen, which contains amino- and carboxyl-
terminal propeptides that are globular in structure. After removal of the signal sequence to
form procollagen, hydroxylation of proline and lysine residues occur. This is followed by
glycosylation of hydroxylysine residues, association of three pro-α chains and the formation of
interchain disulfide bonds. The triple helix then forms spontaneously and procollagen is
secreted into the extracellular space. In the extracellular space, the amino- and carboxyl-
terminal regions are removed by specific proteases. Without the globular regions, the collagen
molecules assemble into fibrils. Cross-links are then formed to give the fibrils tensile strength.
There are four types of osteogenesis imperfecta all of which are associated with defects in the
synthesis of Type I collagen. The disease is characterized by fragile bones due to abnormalities
in the collagen within the bone matrix. Several of the molecular defects involve mutations that
convert glycine to another amino acid thereby interfering with the formation of the triple helix.
Question: 20
A common feature of the proteins osteocalcin and prothrombin is that both

Answer Choices:
A Contain γ-carboxyglutamic acid
B Are involved in osteogenesis
C Contain hydroxyproline residues
D Must be activated by cleavage with plasmin
E Are secreted by odontoblasts

A Contain γ-carboxyglutamic acid

Osteocalcin is an abundant protein consisting of 50 amino acids found in dentin, and it contains
3 γ-carboxyglutamic acid (Gla) residues. The two carboxyl groups of Gla are believed to bind
calcium and may function to bind the protein to the calcium on the hydroxyapatite surface. Gla
formation is a post-translational modification that involves the addition of a carboxyl group
from bicarbonate to a glutamic acid of the protein in a Vitamin K dependent process. Gla
residues are also found in several of the blood coagulation factors including prothrombin. The
drugs dicoumarol and warfarin are antagonists of vitamin K and prevent the formation of Gla
residues. Dicoumarol is used as an anticoagulant drug. Osteogenesis is the process of bone
formation and does not involve prothrombin.

Question: 21
Which one of the following is an example of an amphiarthroses type of joint?

Answer Choices:
A Tibiofibular joint
B Radioulnar joint
C Joint between adult skull bones
D Intervertebral joints
E Knee joint
D Intervertebral joints

Amphiarthrodial joint, or Synchondrosis (the union of bones by cartilage), is one of the type of
synarthrodial (immovable) joint, which is exemplified by the joints between the vertebrae.
The tibiofibular and radioulnar joints are classified as syndesmosis (union of bone by ligament)
type of joints, joints between adult skull bones are known as synostosis (union of bones by
bone) joints, and the knee joint is an example of a diarthrodial (synovial, or freely moveable)
joint.
Question: 22
Because of the lack of epiphyseal cartilage, adult bone cannot increase in length when stimulated by
an excess of growth hormone; however, they do increase in width by periosteal growth. In adults,
what does an increase in growth hormone cause?

Answer Choices:
A Gigantism
B Osteomalacia
C Acromegaly
D Rickets
E Osteoporosis

C Acromegaly

In adults, an increase in growth hormone causes acromegaly, a disease in which the bones,
mainly the long ones, become very thick.
An excess of hormone during growing years causes gigantism. Osteomalacia is caused by
calcium deficiency in adults; rickets is caused by calcium deficiency in children. Osteoporosis is
the decrease in bone mass caused by decreased bone formation, increased bone resorption, or
both.

Question: 23 EXAMS
The decrease in bone mass because of decreased bone formation would cause

Answer Choices:
A Rickets
B Osteomalacia
C Osteoporosis
D Acromegaly
E Osteosarcoma

C Osteoporosis

Osteoporosis, frequently found in immobilized patients and postmenopausal women, is a


decrease in bone mass caused by decreased bone formation, increased bone resorption, or
both. In osteoporosis, the ratio of mineral to organic matrix is unchanged in the otherwise
morphologically normal bone.
Rickets is caused by calcium deficiency in children. In this disease the bone matrix does not
calcify normally and the bone spicules formed by the epiphyseal plate become distorted by the
normal strains of body weight and muscular activity. Osteomalacia is the condition of calcium
deficiency in adults, characterized by deficient calcification of recently formed bone and partial
decalcification of already calcified matrix. Acromegaly is caused by increase in growth hormone,
and the long bones become very thick. Osteosarcoma (although rare) is a malignant tumor of
bone cells.

Question: 24
Which of the following definitions is correct?

Answer Choices:
A Cortical bone - the spongy bone in the central region of the bone
B Diaphysis - the shaft of a long bone
C Cancellous bone - the compact bone forming the outer layer of the bone
D Metaphysis - the widened end of a long bone
E Epiphysis - the region between the diaphysis and the metaphysis

B Diaphysis - the shaft of a long bone

The diaphysis is the shaft of a long bone and the epiphysis is the widened region at the end of
the bone; the metaphysis is the region between the diaphysis and the epiphysis.
The epiphyseal growth plate, or physis, is only present in growing bones. It is found between
the epiphysis and metaphysis. It is important to be familiar with the different regions of the
bone because many bone tumors are only found in certain parts of the bone.
Cortical bone is the compact bone lining the outer surface of the bone; cancellous bone is the
spongy bone in the interior of the bone. This is where the bone marrow is.

Question: 25
Woven bone is characterized by

Answer Choices:
A Increased strength
B Increased rigidity
C Slow mineralization
D A high mineral content
E Little mineral deposited outside the collagen fibers

D A high mineral content

In woven bone, there is a high mineral content. The collagen fibers of the matrix in woven bone
are irregularly arranged, and over half of the mineralization occurs outside the collagen fibers.
The fact that so much of the mineralization occurs outside the collagen fibers means that
woven bone can be rapidly mineralized and, if necessary, resorbed. It also means that more
mineral can be deposited in woven bone than in lamellar bone. On the other hand, woven bone
is not as strong, and not as rigid as lamellar bone.

Question: 26
The term membranous bone refers to

Answer Choices:
A Flat bones
B Long bones
C Bones that are both flat and long
D Bones that develop directly from primitive mesenchyme
E Bones that develop from a cartilaginous precursor

D Bones that develop directly from primitive mesenchyme

Membranous bone refers to bone that develops directly from primitive mesenchyme and does
not have a cartilaginous precursor.

Question: 27
With regard to hyaline cartilage, the term capsule refers to

Answer Choices:
A The perichondrium
B The cell membrane of the chondrocyte
C The basophilic rim of matrix surrounding the lacunae
D The outermost layer of articular cartilage
E A structure formed by the union of hyaluronic acid and collagen fibers

C The basophilic rim of matrix surrounding the lacunae

Hyaline cartilage stains metachromatically and with basophilic dyes because it has a high
concentration of sulfated proteoglycans. The highest concentrations of these molecules is in
the matrix immediately surrounding the lacunae, causing this area to stain deep blue on H & E.
When this appears as a thin rim around the lacuna, it is called the capsule. The wider area of
deep blue staining around the lacuna is called the territorial matrix . The lightest staining
matrix, farthest from the neighboring lacunae, is called the intraterratorial matrix .
Question: 28
What is the definitive test for measuring the rate of bone formation and resorption?

Answer Choices:
A Serial calcium levels
B Bone scan
C Bone biopsy with double tetracycline labeling
D Skeletal survey
E MRI scan
C Bone biopsy with double tetracycline labeling

Bone biopsy with double tetracycline labeling is not often performed, but is considered the
definitive method of determining the metabolic state of the bone.
Tetracycline is a fluorescent antibiotic that is bound to bone as it is being formed. The patient is
given two doses of tetracycline separated by an exactly determined interval. The bone is then
biopsied and processed using methods that do not require decalcifying the bone.
A morphometrist evaluates the biopsy, precisely measuring the total amount of bone in the
biopsy, the percentage of the bone involved in resorption or synthesis, and the width of the
osteoid. The rate of bone formation is determined by measuring the distance between the two
fluorescent lines caused by the tetracycline doses given to the patient.
This is an expensive test that requires a great deal of expensive equipment and a
knowledgeable morphometrist.

Question: 29 CLASS QUIZ


Osteoclasts have receptors for

Answer Choices:
A Parathyroid hormone
B Vitamin D
C Calcitonin
D Growth hormone
E Parathyroid hormone and calcitonin

C Calcitonin

Osteoclasts obviously respond to both parathyroid hormone levels and calcitonin, as they
multiply and become more active under the influence of parathyroid hormone and become less
active under the influence of calcitonin. It is surprising, therefore, that osteoclasts only have
receptors for calcitonin and not for parathyroid hormone or vitamin D.
Question: 30
Osteoblasts stain prominently with

Answer Choices:
A Grocott's methenamine silver (GMS)
B Periodic acid Schiff (PAS)
C Trichrome stain
D Iron stain
E Mucicarmine
B Periodic acid Schiff (PAS)

The intense PAS positive staining of osteoblasts is due to intracellular vesicles containing bone
matrix precursors. Contrast these vesicles containing matrix precursors with the matrix vesicles
, which form the niduses of mineralization in the growing bone. Matrix vesicles are not present
intracellularly. Instead the matrix vesicles are formed at the cell membrane, where they pinch
off from the osteoblast.

Question: 31
Which one of the following bones is categorized as an irregular bone?

Answer Choices:
A Ethmoid
B Metacarpals
C Tibia
D Carpals
E Sternum

A Ethmoid

Irregular bones, such as the ethmoid and vertebrae, have a shape that does not fit into either
the category of (i) long bones (tibia and metacarpals; longer in one dimension than the other),
(ii) short bones (carpals of the hand; length and diameter nearly equal), or the (iii) flat bones
(calvarium and sternum; thin and platelike), but have complex shape.

Question: 32
The flared portion of the bone between the shaft and the expanded ends is called

Answer Choices:
A Diaphysis
B Epiphysis
C Metaphysis
D Compact bone
E Spongy bone

C Metaphysis

The flared portion of the bone between the shaft (= diaphysis) and the expanded ends (=
epiphysis) is called metaphysis.
The shaft of the long bone, the diaphysis, contains a large marrow or medullary cavity,
consisting of spongy bone (tissue arranged as trabeculae or spicules), surrounded by a thick-
walled tube of compact bone (without air spaces, or trabeculae).

Question: 33
A 7-year-old boy with fragile long bones may have a diet deficient in

Answer Choices:
A Potassium (K+)
B Calcium (Ca2+)
C Iron (Fe)
D Carbohydrates
E Lead (Pb)

B Calcium (Ca2+)

Although potassium, iron and carbohydrates are needed to some degree for maintenance of
proper health, calcium (metabolism regulated by at least three hormones: 1,25-
dihydroxycholecalciferol formed from vitamin D, parathyroid hormone, PTH, and calcitonin) is
maintained at a constant level in the blood and the tissues. Consequently, a diet deficient in
calcium leads to calcium loss from the bones. As a result, the bone becomes fragile.
Lead and some other toxic elements are also taken up by bone in a manner similar to that in
which calcium is turned over. This process is sometimes called : "detoxifying mechanism"
because it serves to remove them from the body fluids, thus ameliorating the toxic
manifestations.

Question: 34 CLASS QUIZ


At a bone fracture, dense connective tissue and newly formed cartilage is stimulated to form at the
site. This connective tissue and cartilage structure helps to stabilize and bind together the fractured
bone. This structure is referred to as a
Answer Choices:
A Collar
B Callus
C Cartilage splint
D Spicule
E Closing cone
B Callus

At a fracture site, the hemorrhaged and necrotic tissue is initially removed by macrophages.
New loose connective tissue then infiltrates the site, becoming dense and differentiating into
both a dense connective tissue and forming cartilage. The connective tissue and cartilage cover
the bone at the fracture site to form a callus. The callus helps to stabilize and bind together the
bone at the fracture. Eventually, new formation of bone progresses toward the fracture site
until new bone forms a bony sheath over the callus. Deposition of new bone into the callus
gradually replaces the original fibrous and cartilaginous callus with a bony callus. The cartilage
is replace in the bony callus by endochondral ossification. The process usually takes 6-10 weeks
in health individuals.

Question: 35
Growth in width of a long bone, such as the humerus, occurs by what type of bone growth?

Answer Choices:
A Interstitial
B Appositional
C Both interstitial & appositional
D Neither appositional nor interstitial

B Appositional

Long bones characteristically are formed by endochondral bone formation. However, their
continued growth depends on both the histogenesis of endochondral and intramembranous
bone formation. Intramembranous bone formation of long bones is limited to the development
of their width, primarily, through the activity of the periosteal tissue which allows for the
differentiation of the cells into osteoblasts, and the deposition of osteoid on the surface of the
bone, adding to its width by appositional growth.
Question:36
The attached photomicroscopic image shows a tissue that is undergoing endochondral bone
formation. What principle process has occurred in the region indicated by the arrows?
Answer Choices:
A Cartilage formation
B Cartilage calcification
C Osteoid deposition
D Osteoid resorption
E Cartilage resorption

Image(s) / Chart(s):
Click image to view full size. Click open image to close. Click and hold open image to move.

B Cartilage calcification

The area at the tips of the arrows indicate the remaining spicules of cartilage that has become
calcified in the process of endochondral bone formation. Note that in the photomicrograph, the
chondrocytes have aligned themselves into columns, become hypertrophied and are dying. This
is due to the inability of the cells to receive oxygen and other nutrients necessary for their
survival that must diffuse through the cartilage matrix once it is calcified. Eventually, tunnels
will erode through the columns of dead cells bringing osteoblasts to the surfaces of the calcified
cartilage spicules and bone will be deposited on the surface of the calcified cartilage. The entire
bony process, and its calcified cartilage core will eventually be resorbed by osteoclasts, so of
which can be seen in this image.

Question: 37
The attached photomicroscopic image depicts a longitudinal cut through the distal end of a fetal
finger. The region depicted in the image by the area denoted as "A" is best described as what
developmental zone in endochondral bone formation?

Answer Choices:
A Zone of reserve cartilage
B Zone of calcification & cell death
C Zone of cellular maturation & hypertrophy
D Zone of proliferation
E Zone of ossification & resorption

Image(s) / Chart(s):
Click image to view full size. Click open image to close. Click and hold open image to move.

E Zone of ossification & resorption

Endochondral bone formation is typified by the formation of a cartilage model, which is later
replaced through a gradual and dynamic process with bone. Important to this model is the
removal of cartilage and the deposition of bone matrix. This process occurs in long bones of the
body, as is demonstrated in this photomicroscopic image through the distal end of a fetal
finger. The process of cartilage removal and replacement of bone matrix continues in an orderly
wave of activity that initially extends from the ends of the bone towards the primary
ossification center. Several regions, or zones, can be distinguished in this process. The letter "A"
denotes the zone of ossification and resorption. Vascular connective tissue invades tunnel-like
spaces providing access to osteogenic cells, which differentiate into osteoblasts and form
osteoid on the surface of the calcified cartilage spicules. All of these calcified cartilage spicules
covered by bone will eventually be resorbed as the process continues distally, and the bone
marrow cavity increases in size.

Resorption is primarily the function of osteoclasts. The letter "B" denotes the zone of
calcification and cell death. In this area, the matrix among the cartilage cells begins to
accumulate calcium salts. Unable to access adequate nutrients and oxygen, the chondrocytes,
which are lined in rows, die, dissolve, and leave empty spaces. These spaces "open up" and
form irregular tunnels through the calcified cartilage matrix, which was once cartilage matrix.
The letter "C" denotes the zone of cellular maturation and hypertrophy. In this area, the
chondrocytes mature and form additional cartilage matrix, further increasing the length of the
cartilage in this region. With cell maturation, the lacunae enlarge, forming plates of cartilage
between rows of hypertrophied chondrocytes. The letter "D" denotes a region referred to as
the zone of proliferation. This region contains active, proliferating chondrocytes. The cells
become aligned in columns separated by cartilage matrix. The letter "E" denotes the zone of
reserve cartilage. In this area, the cells and their lacunae are randomly arranged throughout the
matrix, and there is slow growth of cartilage in this region in all directions. This area is
composed of primitive hyaline cartilage, which shortens as the endochondral bone formation
process proceeds.

Question: 38
The attached photomicroscopic image depicts a longitudinal cut through the distal end of a fetal
finger. The region depicted in the image by the area denoted as "B" is best described as what
developmental zone in endochondral bone formation?

Answer Choices:
A Zone of reserve cartilage
B Zone of calcification & cell death
C Zone of cellular maturation & hypertrophy
D Zone of proliferation
E Zone of ossification & resorption

Image(s) / Chart(s):
Click image to view full size. Click open image to close. Click and hold open image to move.

B Zone of calcification & cell death

Endochondral bone formation is typified by the formation of a cartilage model, which is later
replaced through a gradual and dynamic process with bone. Important to this model is the
removal of cartilage and the deposition of bone matrix. This process occurs in long bones of the
body, as is demonstrated in this photomicroscopic image through the distal end of a fetal
finger. The process of cartilage removal and replacement of bone matrix continues in an orderly
wave of activity that initially extends from the ends of the bone towards the primary
ossification center. Several regions, or zones, can be distinguished in this process. The letter "A"
denotes the zone of ossification and resorption. Vascular connective tissue invades tunnel-like
spaces providing access to osteogenic cells, which differentiate into osteoblasts and form
osteoid on the surface of the calcified cartilage spicules. All of these calcified cartilage spicules
covered by bone will eventually be resorbed as the process continues distally and the bone
marrow cavity increases in size.

Resorption is primarily the function of osteoclasts. The letter "B" denotes the zone of
calcification and cell death. In this area, the matrix among the cartilage cells begins to
accumulate calcium salts. Unable to access adequate nutrients and oxygen, the chondrocytes,
which are lined in rows die, dissolve and leave empty spaces. These spaces "open up" and form
irregular tunnels through the calcified cartilage matrix, which was once cartilage matrix. The
letter "C" denotes the zone of cellular maturation and hypertrophy. In this area the
chondrocytes mature and form additional cartilage matrix, further increasing the length of the
cartilage in this region. With cell maturation, the lacunae enlarge, forming plates of cartilage
between rows of hypertrophied chondrocytes. The letter "D" denotes a region referred to as
the zone of proliferation. This region contains active, proliferating chondrocytes. The cells
become aligned in columns separated by cartilage matrix. The letter "E" denotes the zone of
reserve cartilage. In this area, the cells and their lacunae are randomly arranged throughout the
matrix and there is slow growth of cartilage in this region in all directions. This area is
composed of primitive hyaline cartilage, which shortens as the endochondral bone formation
process proceeds.

Question: 39 EXAMS
The attached photomicroscopic image depicts a longitudinal cut through the distal end of a fetal
finger. The region depicted in the image by the area denoted as "C" is best described as what
developmental zone in endochondral bone formation?

Answer Choices:
A Zone of reserve cartilage
B Zone of calcification & cell death
C Zone of cellular maturation & hypertrophy
D Zone of proliferation
E Zone of ossification & resorption

Image(s) / Chart(s):
Click image to view full size. Click open image to close. Click and hold open image to move.
C Zone of cellular maturation & hypertrophy

Endochondral bone formation is typified by the formation of a cartilage model, which is later
replaced through a gradual and dynamic process with bone. Important to this model is the
removal of cartilage and the deposition of bone matrix. This process occurs in the long bones of
the body, as is demonstrated in this photomicroscopic image through the distal end of a fetal
finger. The process of cartilage removal and replacement of bone matrix continues in an orderly
wave of activity that initially extends from the ends of the bone towards the primary
ossification center. Several regions, or zones, can be distinguished in this process. The letter "A"
denotes the zone of ossification and resorption. Vascular connective tissue invades tunnel-like
spaces, providing access to osteogenic cells, which differentiate into osteoblasts and form
osteoid on the surface of the calcified cartilage spicules. All of these calcified cartilage spicules
covered by bone will eventually be resorbed as the process continues distally and the bone
marrow cavity increases in size.

Resorption is primarily the function of osteoclasts. The letter "B" denotes the zone of
calcification and cell death. In this area, the matrix among the cartilage cells begins to
accumulate calcium salts. Unable to access adequate nutrients and oxygen, the chondrocytes,
which are lined in rows die, dissolve and leave empty spaces. These spaces "open up" and form
irregular tunnels through the calcified cartilage matrix, which was once cartilage matrix. The
letter "C" denotes the zone of cellular maturation and hypertrophy. In this area, the
chondrocytes mature and form additional cartilage matrix, further increasing the length of the
cartilage in this region. With cell maturation, the lacunae enlarge, forming plates of cartilage
between rows of hypertrophied chondrocytes. The letter "D" denotes a region referred to as
the zone of proliferation. This region contains active, proliferating chondrocytes. The cells
become aligned in columns separated by cartilage matrix. The letter "E" denotes the zone of
reserve cartilage. In this area, the cells and their lacunae are randomly arranged throughout the
matrix and there is slow growth of cartilage in this region in all directions. This area is
composed of primitive hyaline cartilage, which shortens as the endochondral bone formation
process proceeds.
Question: 40
The attached photomicroscopic image depicts a longitudinal cut through the distal end of a fetal
finger. The region depicted in the image by the area denoted as "D" is best described as what
developmental zone in endochondral bone formation?

Answer Choices:
A Zone of reserve cartilage
B Zone of calcification & cell death
C Zone of cellular maturation & hypertrophy
D Zone of proliferation
E Zone of ossification & resorption

Image(s) / Chart(s):
Click image to view full size. Click open image to close. Click and hold open image to move.

D Zone of proliferation

Endochondral bone formation is typified by the formation of a cartilage model that is later
replaced through a gradual and dynamic process with bone. Important to this model is the
removal of cartilage and the deposition of bone matrix. This process occurs in long bones of the
body, as is demonstrated in this photomicroscopic image through the distal end of a fetal
finger. The process of cartilage removal and replacement of bone matrix continues in an orderly
wave of activity that initially extends from the ends of the bone towards the primary
ossification center. Several regions, or zones, can be distinguished in this process.
The letter "A" denotes the zone of ossification and resorption. Vascular connective tissue
invades tunnel-like spaces providing access to osteogenic cells, which differentiate into
osteoblasts and form osteoid on the surface of the calcified cartilage spicules. All of these
calcified cartilage spicules covered by bone will eventually be resorbed as the process continues
distally, and the bone marrow cavity increases in size. Resorption is primarily the function of
osteoclasts. The letter "B" denotes the zone of calcification and cell death. In this area the
matrix among the cartilage cells begins to accumulate calcium salts. Unable to access adequate
nutrients and oxygen, the chondrocytes, which are lined in rows die, dissolve and leave empty
spaces. These spaces "open up" and form irregular tunnels through the calcified cartilage
matrix, which was once cartilage matrix. The letter "C" denotes the zone of cellular maturation
and hypertrophy. In this area, the chondrocytes mature and form additional cartilage matrix,
further increasing the length of the cartilage in this region. With cell maturation, the lacunae
enlarge, forming plates of cartilage between rows of hypertrophied chondrocytes. The letter
"D" denotes a region referred to as the zone of proliferation. This region contains active,
proliferating chondrocytes. The cells become aligned in columns separated by cartilage matrix.
The letter "E" denotes the zone of reserve cartilage. In this area, the cells and their lacunae are
randomly arranged throughout the matrix, and there is slow growth of cartilage in this region in
all directions. This area is composed of primitive hyaline cartilage, which shortens as the
endochondral bone formation process proceeds.

Question: 41 CLASS QUIZ


The type of bone growth as typified by the attached photomicrographic image is best described as

Answer Choices:
A Endochondral
B Interstitial
C Appositional
D Intramembranous
E Callus formation

Image(s) / Chart(s):
Click image to view full size. Click open image to close. Click and hold open image to move.

A Endochondral
This image typifies bone growth that occurs along a metaphysis ossification center of a long
bone cut in longitudinal section. Note the formation of a marrow cavity (diaphyseal marrow
cavity) which enlarges as it expands into the epiphyseal cartilage. During this process, a
cartilaginous model of the bone is transformed through a dynamic process into the "adult"
bone. It is important to note, that the formation of bone by endochondral bone formation, is
NOT the result of calcification of cartilage. Intramembranous bone formation, by contrast, is the
development of bone from condensation of mesenchymal cells in the region of the bone to be
formed. In this situation, the aggregated mesenchymal cells differentiate into osteoblasts,
which then form the bone matrix as non-calcified bone, called osteoid. The osteoid is
subsequently calcified.

Question: 42
The activity of the cell located at the tip of the arrowhead in this photomicroscopic image is increased
by what hormone?

Answer Choices:
A Parathormone
B Calcitonin
C Thyroxin
D Somatotropin
E Melatonin

Image(s) / Chart(s):
Click image to view full size. Click open image to close. Click and hold open image to move.

A Parathormone

The tip of the arrowhead is pointing to a large, multinucleated cell that stains acidophilic with
an H&E stain. This cell, the osteoclast, secretes organic acids to decalcify the calcium salts of
bone, then releases lysosomes containing hydrolytic enzymes to aid in bone reabsorption.
Frequently, the osteoclast may be located in a shallow depression called Howship's lacunae, the
result of active bone resorption. Osteoclasts are responsive to parathyroid hormone,
parathormone (PTH), which will induce osteoclast activity to increase bone resorption, resulting
in increasing blood calcium levels.
Question: 43
The cell located at the tip of the arrowhead in the attached photomicroscopic image can be best
identified as what cell

Answer Choices:
A Osteocyte
B Osteoclast
C Osteoblast
D Megakaryocyte
E Monocyte

Image(s) / Chart(s):
Click image to view full size. Click open image to close. Click and hold open image to move.

B Osteoclast

The tip of the arrowhead is pointing to a large, multinucleated cell that stains acidophilic with
an H&E stain. This cell, the osteoclast, secretes organic acids to decalcify the calcium salts of
bone, then releases lysosomes containing hydrolytic enzymes to aid in bone reabsorption.
Frequently, the osteoclast may be located in a shallow depression called Howship's lacunae, the
result of active bone resorption.

Question: 44 class quiz


The structure located at the tip of the arrowhead in the photomicroscopic image can be best
identified as

Answer Choices:
A Osteoblast
B Osteocyte
C Osteoclast
D Osteon
E Osteoprogenitor cell

Image(s) / Chart(s):
Click image to view full size. Click open image to close. Click and hold open image to move.

B Osteocyte

The photomicroscopic image represents what is typically observed in compact bone. Close
observation of the image reveals the presence of osteons and interstitial lamellae. Central to
the osteon is the Haversian canal. Located circumferentially around the Haversian canal are the
lacuna, filled with the remnants of the osteocytes charred upon preparation of the ground bone
specimen. The tip of the arrowhead is pointing to one of the osteocytes once located in a
lacunae.

Question: 45
The tissue in the photomicroscopic image can be best identified as what tissue?

Answer Choices:
A Compact bone
B Spongy bone
C Cancellous bone
D Hyaline cartilage
E Fibrocartilage

Image(s) / Chart(s):
Click image to view full size. Click open image to close. Click and hold open image to move.
A Compact bone

The photomicroscopic image represents what is typically observed in compact bone. Close
observation of the image reveals the presence of osteons and interstitial lamellae. Central to
the osteon is the Haversian canal. Located circumferentially around the Haversian canal are the
lacuna, filled with the remnants of the osteocytes charred upon preparation of the ground bone
specimen.

Question: 46
A source of new cartilage cells for appositional growth is from the

Answer Choices:
A Periosteum
B Perichondrium
C Pericytes
D Chondrocytes
E Osteoblasts
B Perichondrium

The perichondrium is a dense connective tissue located on the surface of cartilage (there are
exceptions). The cells resemble fibroblasts, and the perichondrium may be stimulated to give
rise to new cartilage cells (chondroblasts). Appositional growth is obtained by the "addition" of
new cartilage on the surface of existing cartilage by differentiation of the fibroblast-like cells of
the perichondrium. A similar situation exists on the outer surface of bone where a periosteum
is located. Pericytes are cells associated with capillaries and are thought to be undifferentiated
mesenchymal cells and may develop into smooth muscle cells associated with vessel walls.
Chondrocytes are mature cells of cartilage, and while they may form new cartilage, and divide
to form new cartilage cells, they do so through interstitial growth. Osteoblasts are cells that are
involved in the formation of new bone.

Question: 47
The type of connective tissue located at the symphysis pubis would be best identified as

Answer Choices:
A Dense regularly arranged connective tissue
B Hyaline cartilage
C Dense irregularly arranged connective tissue
D Elastic cartilage
E Fibrocartilage
E Fibrocartilage

Fibrocartilage is a combination of dense irregular connective tissue which has a matrix typical of
cartilage. The chondrocytes and the associated cartilage matrix occupies a smaller portion of
the tissue mass in fibrocartilage, such that the fibrous nature of the dense connective tissue is
the predominant feature. Nevertheless, because of the matrix, chondrocytes will occupy
lacunae, and the tissue is classified as a cartilage. One location of fibrocartilage is in the
symphysis pubis.

Question: 48
The predominant collagen fiber type found in bone matrix is

Answer Choices:
A Type I
B Type II
C Type III
D Type IV
E Type V

A Type I

Collagen Type I is the predominant collagen fiber found in bone matrix. Type I collagen is also
found in the connective tissue of the dermis, tendon, ligaments and organ capsules. Type II
collagen is found in hyaline and elastic cartilage. Connective tissue of the stroma of many
organs is comprised of collagen Type III fibers. Collagen Type IV fibers are found in basal
laminae of epithelial cells. Collagen Type V fibers are distributed throughout connective tissue
stroma.
Question: 49 block-1
The tissue illustrated in the photomicroscopic image attached, stained with hematoxylin and eosin,
can be best identified as

Answer Choices:
A Hyaline cartilage
B Elastic cartilage
C Fibrocartilage
D Dense irregularly arranged connective tissue
E Spongy bone

Image(s) / Chart(s):
Click image to view full size. Click open image to close. Click and hold open image to move.

A Hyaline cartilage

The photomicrograph illustrates hyaline cartilage as commonly seen with an hematoxylin and
eosin stain. The principal aspects which provide identification of this tissue relate to isogenous
groups of chondrocytes (cell nests), the presence of basophilic appearing territorial and
interterritorial matrix, and the lack of course fibers, which would be found in fibrocartilage.
Elastic cartilage would have the appearance like that of fibrocartilage, unless stained specifically
to demonstrate elastic fibers. The cells found in bone (osteocytes) also are located in lacunae,
like cartilage, but do not form cell nests.

Case #301462: CLASS QUIZ


A 5-year-old boy falls off his bike and fractures his humerus. He is taken to the emergency
room, and the bone is set by one of the emergency room physicians.

Question: 50
What tissue is responsible for producing the majority of the new bone that will reunite the 2
fragments?

Answer Choices:
A Cancellous bone
B Cartilage
C Compact bone
D Marrow
E Periosteum
E Periosteum

When the periosteum is torn during a fracture, it supplies cells that develop into osteoblasts
and are the major producers of the new bone that reunites the 2 ends. Heterotopic ossification
(bone formed outside the regular bone) can occur as a complication of fracture if some of the
osteoblastic cells are misdirected into adjacent tissues.

Preexisting cancellous bone and compact bone are not the major source of osteoblasts that
form the new bone.

Cartilage and marrow do not contribute to new bone formation after fracture.

Keywords: periosteum, cancellous bone, compact bone

SECTION- MUSCLE
Question:CLASS QUIZ
Identify the tissue as illustrated in this photomicroscopic image

Answer Choices:
A Cardiac muscle
B Skeletal muscle
C Smooth muscle
D Tendon
E Nerve fiber bundle

Image(s) / Chart(s):
Click image to view full size. Click open image to close. Click and hold open image to move.
B Skeletal muscle

Visual inspection of this image shows the presence of large, long fibers that appear
unbranched. Also observed are cross-striations as the presence of small, alternating
dark and light bands. There are also several peripherally located nuclei. All of these
structural characteristics indicate the tissue is skeletal muscle. Cardiac muscle contain
central nuclei, with branched fibers that are connected by dense intercellular junctions,
called intercalated discs. Cardiac muscle, like skeletal muscle, is striated. Smooth
muscle fibers are much smaller, are spindle-shaped and have centrally located nuclei.
Myofibrils in smooth muscle are not organized into striations. Tendons are highly
organized dense regularly arranged connective tissue, and as such, have highly
compact, and very dense parallel arranged fibroblast nuclei, alternating with regularly
arranged collagenous fibers. The presence of striations within the cells eliminates nerve
fiber bundle as a possible tissue.

Question: BLOCK EXAMS


"Red" muscle fibers:

Answer Choices:
A Are comparatively large
B Have relatively few cytochromes and mitochondria
C Have abundant myoglobin
D Have relatively little myosin ATPase activity
E Have relatively little NADH-TR activity
C Have abundant myoglobin

"Red," or Type 1, striated muscle fibers are relatively small fibers with a high
concentration of cytochromes, mitochondria, and myoglobin. They stain intensely with
the myosin ATPase and NADH-TR histochemical stains.
"White," or Type 2, fibers are larger and have a smaller concentration of mitochondria
and myoglobin. They contain more glycogen and phosphorylase than the Type 1 fibers.
Some histochemical stains divide the Type 2 group into Type 2A and Type 2B. Type 2A
fibers are considered intermediate between Type 1 and Type 2B.
Question:
The transverse tubular (or T) system of myocytes is an extension of the cell's

Answer Choices:
A Golgi apparatus
B Smooth endoplasmic reticulum
C Nuclear membrane
D Cytoskeletal network
E Plasma membrane
E Plasma membrane

The transverse tubular system is an extension of the cell membrane that forms many
invaginations into the muscle cell. Each of these invaginations wraps around a
sarcomere, where the invaginating tubule is intimately associated with two terminal
cisternae of the sarcoplasmic reticulum. These three elements together are called a
triad .
The number of T tubules wrapped around each sarcomere is characteristic of the
organism from which the muscle is taken and of the site from which the muscle is taken.
In humans a transverse tubule usually surrounds each A-I zone, so there are two
transverse tubules wrapped around each sarcomere. Each of these transverse tubules
is associated with two terminal cisternae, so there are four terminal cisternae around
each sarcomere.
When a nerve impulse reaches a muscle cell, the cell membrane is depolarized and Na +
ions enter into the cell. As the depolarizing impulse travels through the T system, the
electrochemical changes in the T tubule system affect the terminal cisternae and cause
them to release calcium. The calcium in turn causes the muscle to contract.
Question:
The cytoplasmic densities in smooth muscle are aggregates of

Answer Choices:
A Actin
B Desmin
C Tubulin
D Alpha actinin
E Myomesin
D Alpha actinin
The cytoplasmic densities of smooth muscle appear to form a three dimensional
network that extends throughout the muscle cell. They are composed of alpha actinin,
the same protein that binds actin to the Z discs of skeletal muscle sarcomeres. Although
the actin and myosin filaments of smooth muscle are not organized into sarcomeres,
they are bound to a support network composed of alpha actinin and the intermediate
filaments of the cytoskeleton.
Question:
The endomysium is a connective tissue investment that surrounds

Answer Choices:
A Individual muscle fibers
B Muscle fascicles
C Individual myofibrils
D An entire muscle
E Small bundles of muscle cells

A Individual muscle fibers

The endomysium is a thin connective tissue layer, composed of reticular fibers and an
external lamina, that invests individual muscle fibers (= cells). The epimysium surrounds
the entire muscle, and the perimysium surrounds bundles ( = fascicles) of muscle fibers.
Question:
Thick filaments are anchored to Z discs by

Answer Choices:
A C protein
B Nebulin
C Titin
D Myomesin
E α-actinin
C Titin

Titin forms an elastic lattice that anchors thick filaments to Z disks.


C protein, one of several myosin-binding proteins that serve the same function as
myomesin and form several distinct stripes on either side of the M line. Nebulin, an
elongated inelastic protein that is attached to the Z disks and runs parallel to the thin
(actin) myofilaments. Myomesin, a myosin-binding protein that serves to hold thick
myosin filaments in register at the M line. α-Actinin, a short, bipolar, rod-shaped
molecule that bundles actin filaments into parallel arrays at the level of the Z disk and
helps to anchor the thin filaments at the Z discs.

Question:
One hormone that has a stimulatory action on the tissue as seen in this photomicroscopic image is

Answer Choices:
A Relaxin
B Neurophysin
C Pepsin
D Oxytocin
E Luteinizing hormone (LH)

Image(s) / Chart(s):
Click image to view full size. Click open image to close. Click and hold open image to move.

D Oxytocin

This image represents two layers of smooth muscle arranged perpendicular to one-
another such that one layer is observed in longitudinal section, the other layer in cross-
section. Smooth muscle is directly innervated by both sympathetic and parasympathetic
nerves. While smooth muscle contraction may be spontaneous, it is also stimulated by a
variety of hormones such as oxytocin, epinephrine, norepinephrine or serotonin.

Question: FINALS
In humans, the triad of skeletal muscle is located at what position of the sarcomere?

Answer Choices:
A Z line
B A-H band junction
C A-I band junction
D M line
E M line-Z line junction

C A-I band junction

The triad of skeletal muscle is composed of adjoining sarcotubules, the terminal


cisternae, that pass around each myofibril and become continuous with the terminal
cisternae of adjacent myofibrils. The "pairs" of terminal cisternae located at the A - I
junction are separated by a slender invagination of the plasma membrane or
sarcolemma called the T tubule. Thus, the paired terminal cisternae and T tubules are
referred to as the triad, and in human skeletal muscle, are located at the A-I band
junction.

Question:
Atrial granules contain

Answer Choices:
A Lipofuscin
B Myosin
C Natriuretic hormone
D Purkinje cell nuclei
E Actin
C Natriuretic hormone

Atrial granules are perinuclear granules found in the cardiac muscle cells of the atria.
They contain two diuretic hormones, atrial natriuretic factor (ANF) and brain natriuretic
factor (BNF). These hormones regulate sodium excretion by depressing levels of renin
and aldosterone. They also increase the relaxation of vascular smooth muscle.
There is some evidence that ventricular cells can also release BNF.
Question:
Smooth muscle cells can be differentiated from fibroblasts by

Answer Choices:
A A corkscrew shaped nucleus in the smooth muscle cell
B Striations in the smooth muscle cell
C Eosinophilic cytoplasm in the fibroblast
D Elongated nuclei in the smooth muscle cell
E Peripheral nuclei in the smooth muscle cell
A A corkscrew shaped nucleus in the smooth muscle cell

Smooth muscle cells and fibroblasts can be difficult to distinguish on routine H & E
staining. Both are elongate cells with eosinophilic cytoplasm and centrally located
nuclei. Neither of them have cross-striations.
When smooth muscle is fixed, the cell contracts, contorting the nucleus into a
corkscrew. This artifact is not seen in fibroblasts and can be used to distinguish one cell
type from the other. The nucleus of a smooth muscle cell is usually both plumper and
more blunted than a fibroblast nucleus.
Question: EXAMS
Which of the following statements concerning the white fibers (in skeletal muscles) is correct?

Answer Choices:
A They are small fibers with large amounts of myoglobin and cytochromes and
many mitochondria
B They are large fibers with small amounts of myoglobin and cytochromes and
few mitochondria
C They make up slow-twitch motor units
D They have great resistance to fatigue but generate relatively less muscle
tension than any other muscle fiber type
E They are typically found in the limb muscles of mammals, and in the breast
muscles of migrating birds
B They are large fibers with small amounts of myoglobin and cytochromes and few
mitochondria

White fibers are large fibers with small amounts of myoglobin and cytochromes and few
mitochondria. They make up fast-twitch motor units, fatigue rapidly and generate a large
peak muscle tension.
Other statements apply to the red fibers (of skeletal muscles). These fibers are small
with large amounts of myoglobin and cytochromes and many mitochondria. They make
up slow-twitch motor units, have great resistance to fatigue but generate relatively less
muscle tension than any other muscle fiber type, and are typically found in the limb
muscles of mammals, and in the breast muscles of migrating birds.

Question: CLASS QUIZ


A motor unit may be defined as

Answer Choices:
A A single axon innervating one, or more than one muscle fibers
B More than one axon innervating a single muscle fiber
C A single dendrite innervating a single muscle fiber
D More than one dendrite innervating a single muscle fiber
E Skeletal muscle fiber undergoing a graded contraction

A A single axon innervating one, or more than one muscle fibers

A motor unit is a complex of a single axon (= motor fiber) innervating either a single or
more than 160 or more muscle fibers.
Multiple axons never converge on a single muscle fiber. Dendrites are sensory fibers,
and do not make up a motor unit. Individual striated muscle fibers do not show graded
contraction - they contract either all the way or not at all (All-or-none law of muscle
contraction).

Question:
The absent protein in Duchenne muscular dystrophy is called

Answer Choices:
A Troponin I
B Troponin C
C Tropomyosin
D Dystrophin

D Dystrophin

The immediate source of energy for muscle contraction is ATP. Hydrolysis of the bonds
between the phosphate residues of this compound is associated with the release of a
large amount of energy, and the bonds are therefore referred to as high-energy
phosphate bonds.
In muscle, the hydrolysis of ATP (but not GTP or cAMP) to ADP is catalyzed by the
contractile myosin protein. The ATPase activity is found in the heads of the myosin
molecules, where they are in contact with actin. Ca2+ initiates contraction by binding to
troponin C. In resting muscle, troponin I is tightly bound to actin, and tropomyosin
covers the sites where myosin heads bind to actin.
ATP is re-synthesized from ADP by addition of a phosphate group. Under normal
conditions, the energy for this endothermic reaction is supplied by the breakdown of
glucose to CO2 and H2O, but there also exists in the muscle another energy-rich
phosphate compound-phosphorylcreatine that can supply this energy.
Dystrophin, the large protein that is absent on Duchenne muscular dystrophy, is also
associated with sarcotubular system, but its exact function is still unknown.
Question:
The adenosine triphosphatase (ATPase) activity is found in the

Answer Choices:
A Actin
B Dystrophin
C Tropomyosin
D Troponin I
E Heads of the myosin
E Heads of the myosin
The immediate source of energy for muscle contraction is ATP. Hydrolysis of the bonds
between the phosphate residues of this compound is associated with the release of a
large amount of energy, and the bonds are therefore referred to as high-energy
phosphate bonds.
In muscle, the hydrolysis of ATP (but not GTP or cAMP) to ADP is catalyzed by the
contractile myosin protein. The ATPase activity is found in the heads of the myosin
molecules, where they are in contact with actin. Ca2+ initiates contraction by binding to
troponin C. In resting muscle, troponin I is tightly bound to actin, and tropomyosin
covers the sites where myosin heads bind to actin.
ATP is re-synthesized from ADP by addition of a phosphate group. Under normal
conditions, the energy for this endothermic reaction is supplied by the breakdown of
glucose to CO2 and H2O, but there also exists in the muscle another energy-rich
phosphate compound-phosphorylcreatine that can supply this energy.
Dystrophin, the large protein that is absent on Duchenne muscular dystrophy, is also
associated with sarcotubular system, but its exact function is still unknown.
Question:
The immediate source of energy for muscle contraction is

Answer Choices:
A Phosphorylcreatine
B cAMP
C GTP
D ATP
E ADP

D ATP

The immediate source of energy for muscle contraction is ATP. Hydrolysis of the bonds
between the phosphate residues of this compound is associated with the release of a
large amount of energy, and the bonds are therefore referred to as high-energy
phosphate bonds.
In muscle, the hydrolysis of ATP (but not GTP or cAMP) to ADP is catalyzed by the
contractile myosin protein. The ATPase activity is found in the heads of the myosin
molecules, where they are in contact with actin. Ca2+ initiates contraction by binding to
troponin C. In resting muscle, troponin I is tightly bound to actin, and tropomyosin
covers the sites where myosin heads bind to actin.
ATP is re-synthesized from ADP by addition of a phosphate group. Under normal
conditions, the energy for this endothermic reaction is supplied by the breakdown of
glucose to CO2 and H2O, but there also exists in the muscle another energy-rich
phosphate compound-phosphorylcreatine that can supply this energy.
Dystrophin, the large protein that is absent on Duchenne muscular dystrophy, is also
associated with sarcotubular system, but its exact function is still unknown.
stion:
The neuromuscular junction is also known as
Answer Choices:
A Z-disk
B Sarcoplasmic reticulum
C The motor end plate
D Crossbridge regulator
C The motor end plate

The neuromuscular junction is a specialized region of the plasma membrane, known


also as the motor end plate. It provides the connection between the axon of the motor
neuron and the muscle cell. Each muscle cell has a motor end plate, however, since the
neuron branches into several branches, each motor nerve axon controls several muscle
cells. The motor unit is the motor nerve with its associated muscle cells, thus it is the
basic functional contractile element.
The sarcoplasmic reticulum is a membrane system which forms an envelope around
each myofibril.
The Z-disks provide the linkage between sarcomeres on an end-to-end basis.
Crossbridge regulation involves the switching of the muscle cell between on and off
state through calcium ion regulation.

Question: BLOCK EXAM


A 22-year-old woman complains of fatigue of the muscles in her face, eyes, and tongue. Clinical tests
indicate thymoma and the presence of circulating antibodies to acetylcholine receptors. A possible
diagnosis is

Answer Choices:
A Duchenne muscular dystrophy
B Tay-Sachs disease
C Myasthenia gravis
D Amyotrophic lateral sclerosis
E Demyelinating disease

C Myasthenia gravis

Myasthenia gravis is an autoimmune disease in which antibodies to acetylcholine


receptors are produced. As a result, neural stimulation of contraction at the myoneural
junction is impaired.
Duchenne muscular dystrophy is caused by a sex-linked, recessive genetic defect that
results in the inability to synthesize dystrophin. The function of dystrophin is not yet
known. Tay-Sachs disease is caused by the inability of lysosomes to degrade
glycolipids and they accumulate in the nervous system. Amyotrophic lateral sclerosis
(ALS, also known as Lou Gehrig's disease) is marked by degeneration of motor neurons
of the spinal cord, resulting in muscle dystrophy. It results in death usually due to
respiratory muscle failure. Demyelinating disease is due to an insufficiency or lack of
either myelin basic protein or proteolipid protein, or both.
Question:
Which of the following statements concerning the myofibrils is correct?

Answer Choices:
A A section through the I (= isotropic) band region shows only thick filaments,
and these filaments are arranged in a circular pattern
B A section through H (= helle, or light) portion of the A (= anisotropic) bands
show only thin filaments, which are arranged in a hexagonal pattern
C A section through the M (= myosin) band shows a network of fine filaments
connecting the myosin filaments. These are the transversely oriented filaments of
myosin that bind the thick filaments together
D The Z line, actually Z disc (= zwischenscheibe, a between disc) is present in
the middle of the H (= helle, light) band in ideal preparations (longitudinal sections)
of the skeletal muscles
E When seen under a polarized microscope, the dark bands on the skeletal
muscle do not alter the plane of the polarized light (i.e., they are monorefringent)

C A section through the M (= myosin) band shows a network of fine filaments


connecting the myosin filaments. These are the transversely oriented filaments of
myosin that bind the thick filaments together

Cross-striations are evident in H&E (=hematoxylin and eosin) stained preparations of


longitudinal sections of muscle fibers. These alternating dark and light bands are known
as A (= anisotropic, as they alter the plane of polarized light, as seen under polarized
microscope, and hence are birefringent) bands and I (=isotropic, as they do not alter the
plane of polarized light) bands. A section through the M (= myomesin) band shows a
network of fine filaments connecting the myosin filaments. These are the transversely
oriented filaments of myomesin that bind the thick filaments together.
Other statements concerning the myofibrils are INCORRECT. A section through the I (=
isotropic) region will shows only THIN filaments, and these filaments are arranged in a
HEXAGONAL pattern. A section through H (= helle, or light) portion of the A (=
anisotropic) bands show only THICK filaments, which are also arranged in a hexagonal
pattern. The Z line, actually Z disc (= zwischenscheibe, a between disc) is a DENSE
ZONE THAT BISECTS THE I BAND. When seen under a polarized microscope, the
LIGHT bands on the skeletal muscle do not alter the plane of the polarized light (i.e.,
they are monorefringent).

Question:
The functional unit of the myofibril is known as the
Answer Choices:
A Thick filament
B Sarcomere
C Titin
D Nebulin
E Hemoglobin

B Sarcomere

The sarcomere is the basic contractile unit of the striated muscle. It is the segment of
myofibril between two Z discs. The entire muscle exhibits cross-striation because
sarcomeres in adjacent myofibrils are in register.
Thick filaments are composed of protein myosin and are held in register in the center of
the H line by fine, transversely oriented filaments of myosin that constitute the M line.
They together with thin filaments form sarcomere. Titin is a structural protein that
connects the thick filaments to the Z disks, while nebulin is attached to the Z disks and
runs parallel to the thin myofilaments. Hemoglobin is the respiratory protein contained in
the red blood cells.
Question:
Which one of the following statements concerning triads in mammalian skeletal muscle is true?

Answer Choices:
A They are located in the Z disk
B They consist of two terminal cisternae of the sarcoplasmic reticulum separated
by a T tubule
C They can be observed with the light microscope
D They are characterized by a T tubule that sequesters calcium ions
E They consist of two T tubules separated by a central terminal cisterna
B They consist of two terminal cisternae of the sarcoplasmic reticulum separated by a
T tubule

A triad in skeletal muscle is composed of three components, a T tubule and two terminal
cisternae of the sarcoplasmic reticulum that flank it. The sarcoplasmic reticulum
sequesters calcium ions, not T tubule.
They are neither located in the Z disk, nor can be observed with the light microscope.

Question:
In which of the following ways is contraction in smooth muscle similar to contraction of skeletal muscle?

Answer Choices:
A A troponin-tropomyosin complex blocks the actin binding site
B Actin catalyzes the hydrolysis of ATP
C An influx of Ca2+ initiates muscle contraction
D Contraction is rapid, taking well under a second to complete
E Similar amounts of ATP are used to produce a contraction in skeletal muscle
and in smooth muscle
C An influx of Ca2+ initiates muscle contraction

The contraction of smooth muscle is similar to the contraction of skeletal muscle in that
a "sliding filament" model of actin filaments sliding past myosin filaments is believed to
describe the mechanism of contraction and in that an influx of Ca 2+ ions initiates
contraction. Unlike skeletal muscle, in which Ca2+ controls the reaction between actin
and myosin by moving a troponin-tropomyosin complex away from the binding site, in
smooth muscle Ca2+ activates a myosin kinase. Only the phosphorylated myosin can
bind to actin.
The interaction between actin and myosin in smooth muscle is very slow and consumes
only 10% of the ATP that skeletal muscle would require to do the same work.
Actin does not hydrolyze ATP in either skeletal or smooth muscle.

Question: CLASS QUIZ


The T tubule system of cardiac muscle surrounds the sarcomeres at the level of the

Answer Choices:
A M line
B H zone
C A zone
D I zone
E Z line

E Z line

Unlike skeletal muscle, in which the T tubule system encircles the sarcomeres at each
A-I zone interface, the T tubule system of cardiac muscle surrounds the sarcomeres at
the Z line. Obviously then, the T tubule system in skeletal muscle circles the
sarcomeres twice as many times as that of cardiac muscle.
The sarcoplasmic reticulum of cardiac muscle is less developed than that of skeletal
muscle and the terminal cisternae are likewise poorly developed. The terminal cisternae
and the T tubules form a diad , as opposed to the triads of skeletal muscle.
Question:
Examination of the ultrastructure of cardiac muscle cells show that intercalated discs coincide with one
of the myofibril units of the sarcomere. Intercalated discs always coincide with which one of the
following structures?
Answer Choices:
A Z line
B A band
C H band
D M line
E I band

A Z line

The specialized transverse junctions between cardiac muscle cells provide structural
support between apposed ends of the cells. These intercalated discs transmit the forces
of contraction between adjacent cells and structurally always occur coincident with the Z
lines, the terminal aspect of a sarcomere.
Question:
The ends of the terminal sarcomeres of cardiac muscle insert into a specialized structure that helps
stabilize a junctional complex between apposing muscle cells, and to aid in transmitting the contractile
forces from cell to cell. This specialized structure is referred to as the

Answer Choices:
A Zona adherens
B Fascia adherentes
C Macula adherens
D Macula densa
E Nexus

B Fascia adherentes

The fascia adherens is the junctional complex between two adjacent cardiac muscle
fibers. It resembles the zona adherens of epithelial cell junctional complexes, but is
more extensive. The fascia adherentes provide structural support for terminal
sarcomeres, which insert into it to enable transmission of contractile forces.
Desmosome, or desmosome-like structures occur in the fascia adherens to provide
additional intercellular adhesion. Also, gap junctions, or nexus will occur in the
longitudinal portions of the interdigitations of the two cells providing sites of low
electrical resistance between the apposing cells.

Question:
The fascia adherens is a structure that occurs between which cell types?

Answer Choices:
A Cardiac muscle cells
B Smooth muscle cells
C Skeletal muscle cells
D Secretory gland cells
E Absorptive epithelial cells

A Cardiac muscle cells

The fascia adherens is the junctional complex between two adjacent cardiac muscle
fibers. It resembles the zona adherens of epithelial cell junctional complexes, but is
more extensive. The fascia adherens provide structural support for terminal
sarcomeres, which insert into it to enable transmission of contractile forces between
cells. Desmosome, or desmosome-like structures occur in the fascia adherens to
provide additional intercellular adhesion. Also, gap junctions, or nexus will occur in the
longitudinal portions of the interdigitations of the two cells providing sites of low
electrical resistance between the apposing cells.

Question:
The presence of intercalated discs would confirm the identification of which one of the following
tissues?

Answer Choices:
A Skeletal muscle
B Exocrine pancreas
C Smooth muscle
D Cardiac muscle

D Cardiac muscle

Cardiac muscle fibers are large, branching fibers that display one, or sometimes two
centrally located nuclei within the cell. The ends of the fibers branch longitudinally which
appose similar branches on adjacent cells. Between the connecting fibers are
specialized transverse junctions called intercalated discs. The interdigitating intercalated
disc binds the cells, aids in transmitting the contractile forces between abutting cells,
and provides areas of low electrical resistance for the rapid spread of excitation
throughout the myocardium of the heart.

Question: BLOCK-1
2+
The Ca that initiates muscle contraction is stored in the

Answer Choices:
A Sarcoplasmic reticulum
B Transverse tubules
C Mitochondria
D Sarcomeres
E Glycogen granules

A Sarcoplasmic reticulum

The sarcoplasmic reticulum, as the smooth endoplasmic reticulum of myocytes is


known, stores Ca2+- between contractions. The sarcoplasmic reticulum forms an
interlacing system around each myofibril. Each element of the sarcoplasmic reticulum
has a terminal dilation called the terminal cisterna that encircles the sarcomere at the A-
I junction.
Calcium is released from the sarcoplasmic reticulum when the transverse tubular
system transmits a nerve impulse from the nerve to the sarcoplasmic reticulum.
Question:
Which of the following statements is true of the contraction of sarcomeres?

Answer Choices:
A All the sarcomeres in a myofibril contract simultaneously
B The thin (actin) filaments shorten during contraction
C The thick (myosin) filaments shorten during contraction
D The A band narrows during contraction
E The H and I bands narrow during contraction

E The H and I bands narrow during contraction

Myofibrils (and by extension, muscles) contract when their sarcomeres shorten.


Sarcomeres do not all contract at the same time - if they did the opposing forces on the
individual sarcomeres would actually prevent any contraction. Instead the sarcomeres
contract in a wave that spreads down the myofibril.
Sarcomeres shorten when the actin (thin) filaments slide past the myosin (thick)
filaments. Neither filament shortens. As the actin filaments slide by the myosin
filaments, the zones composed of actin filaments alone (the I zones) and the zone
composed of myosin filaments alone (the H zone) narrow. The size of the A zone,
determined by the length of the myosin filaments, does not change.
Question:
White" striated muscle fibers

Answer Choices:
A Are also called "slow-twitch" fibers
B Are more extensively and finely innervated than "red" fibers
C Are very resistant to fatigue
D Don't generate as much tension as "red" fibers
E Are primarily found in the back muscles used to maintain posture
B Are more extensively and finely innervated than "red" fibers

White" striated muscle fibers are also referred to as "fast-twitch" fibers. They can
generate a great deal of tension quickly, but they fatigue rapidly. They are very
extensively and finely innervated, allowing them to make small, precise movements.
"Fast-twitch" fibers are particularly found within the extraocular muscles and the
muscles of the hand.
"Red" striated muscle fibers are also called "slow-twitch" fibers. They cannot produce as
much tension as "white" fibers, but they do not fatigue as quickly. They are found in the
back muscles that are used to maintain posture, among other places.

Question:CLASS QUIZ
Upon contraction, the sarcomere of skeletal muscle shortens. Identify the appropriate histological
sequence of the following events:

Answer Choices:
A I band width shortens, A band remains constant length, H band width shortens
B I band width remains constant, A band width shortens, H band width remains
constant
C I band width shortens, A band width shortens, H band width shortens
D I band width shortens, A band width shortens, H band width remains constant
E I band width remains constant, A band width remains constant, H band width
shortens
A I band width shortens, A band remains constant length, H band width shortens

When visualizing skeletal muscle, the A band consists of the thick filaments (myosin)
and of the overlapping areas of thick and thin (actin) filaments. This band appears
darkly stained. The I band consists primarily of the actin filaments and stains light. The
alternating dark and light staining of the muscle filaments gives rise to the "striations."
Bisecting the A band is a shorter, lighter staining area called the H band, composed of
only the myosin filaments. Upon contraction, the A band width remains constant while
the width of both the I bands and the H bands decrease. This is due to the change in
the relative overlap of the myosin and actin filaments such that while there is no change
in the length of the filaments involved, the apparent width of the I band decreases since
there is more overlapping of the actin filament with the myosin filament. As the ends of
the actin filaments approach one another, the width of the H band also decreases. The
end result is the shortening of the sarcomere.

Question:
Identify the tissue illustrated by the photomicroscopic image
Answer Choices:
A Dense irregularly arranged connective tissue
B Skeletal muscle
C Nerve fiber bundle
D Developing bone
E Cardiac muscle

Image(s) / Chart(s):
Click image to view full size. Click open image to close. Click and hold open image to move.

E Cardiac muscle

Cardiac muscle is identified by its large, branching fibers connected by intercalated


disks (arrowheads). The fibers contain a centrally located nucleus and myofibrils
arranged such that striations may be visible. These differ from skeletal muscle fibers,
which also contain striations, but do not branch. Furthermore, skeletal muscle fibers
contain nuclei that are peripherally located. Dense irregularly arranged connective
tissue may have some similarities to cardiac muscle in this image, except that there are
no striations in dense irregularly arranged connective tissue. Careful inspection of this
image reveals the presence of intercalated discs, which are not found in connective
tissue.
Question:
The tissue primarily depicted in longitudinal and cross-section in this photomicroscopic image is
regulated by

Answer Choices:
A Motor nerve innervation
B Sympathetic nerve innervation
C Nerves from the dorsal root ganglion
D Corticospinal tract nerves
E Peripheral afferent receptors

Image(s) / Chart(s):
Click image to view full size. Click open image to close. Click and hold open image to move.

B Sympathetic nerve innervation

This image represents two layers of smooth muscle arranged perpendicular to one-
another such that one layer is observed in longitudinal section, the other layer in cross-
section. Smooth muscle is directly innervated by both sympathetic and parasympathetic
nerves. While smooth muscle contraction may be spontaneous, it is also stimulated by a
variety of hormones such as oxytocin, epinephrine, norepinephrine or serotonin.

Question:
The tissue depicted in this photomicroscopic image is best identified as

Answer Choices:
A Tendon
B Dense irregular connective tissue
C Smooth muscle
D Skeletal muscle
E Peripheral nerve bundle

Image(s) / Chart(s):
Click image to view full size. Click open image to close. Click and hold open image to move.
C Smooth muscle

The image depicts the appearance of smooth muscle layers arranged perpendicular to
each other, such that one layer is arranged so the cells are longitudinal to the observer,
and the other layer has cells that appear cut in cross-section. With close inspection, the
dense staining, rod-shaped nuclei are located within the muscle fiber; usually in a
central location since the muscle fiber is described as being "spindle shaped". This is
more apparent in the layer cut in cross-section, where some of the dark staining nuclei
can be seen located within the fiber. Nuclei are not found in the cut surface of every cell,
since the cells are spindle-shaped and the nucleus for any one cell is limited to the
widest portion of that cell. In contrast, skeletal muscle fibers are large, multi-nucleated
cells, having peripherally located nuclei. Also, striations may be seen on the longitudinal
sections of skeletal muscle. Tendons resemble smooth muscle when seen in a
preparation as depicted in this image, except that tendons are composed of dense,
regularly arranged collagen bundles, external to the fibroblast. Peripheral nerve bundles
are seen surrounded by a perineurium, and in cross-section, close inspection will reveal
axons surrounded by myelin sheaths, in the case of a myelinated peripheral nerve fiber
bundle.
Question:
An example of muscle comprised predominantly of "fast-twitch" white muscle fibers would be the

Answer Choices:
A Diaphragm
B Supraspinalis
C Gluteus maximus
D Lateral rectus oculi
E Rectus abdominis

D Lateral rectus oculi

Fast-twitch motor units are adapted for rapid contraction in fine motor movements.
White muscle fibers, whose fibers contain less myoglobin and fewer mitochondria,
fatigue rapidly but generate high muscle tension. An example of a muscle comprised of
fast-twitch, white muscle fibers is the extraocular muscle, the lateral rectus. All of the
other muscles listed are muscles having either red fibers or intermediate fibers.
Question:
Refer to the attached image. The function of gamma fibers includes which of the following?

Answer Choices:
A Direct contraction of extrafusal fiber
B Direct contraction of intrafusal fibers
C Signal the stretch in the spindle to the CNS
D Activation of alpha motoneurons
E Signal the tension in the muscle during contraction or stretch to the CNS

Image(s) / Chart(s):
Click image to view full size. Click open image to close. Click and hold open image to move.

B Direct contraction of intrafusal fibers

The intrafusal fibers are very small skeletal muscle fibers inside the fusiform shape of
the muscle spindle. The gamma fibers are small motor fibers that innervate and contract
the intrafusal muscle fibers inside the muscle spindle. They are responsible for
modulating the sensitivity of the stretch receptors. The I alphas are sensory fibers that
mediate information from the stretch receptor to the CNS. As part of the monosynaptic
stretch reflex they directly activate the alpha motoneurons in the spinal ventral horn.
The alpha motoneurons, in turn, stimulate the extrafusal muscle fibers of the same
muscle. Note that the spindle sensory fibers and the extrafusal motor fibers are both
called alphas because they are of the same size category. The Golgi tendon organ is
involved in signaling the tension developed in the muscle during contraction or stretch to
the CNS.
References:
Olkowski Z, Moncha SL: Muscle spindle. In Bourne GH [ed]: The Structure and Function
of Muscle, 2nd ed, Vol. II, Part 2. New York, Academic Press, 1973.
Concepts of Muscle Pathology; Stevens & Lowe, 2nd ed., p. 230.

Case #301532:
You are performing a study on the isometric contractile characteristics and fatigability
in the elbow flexors in 120 pre-meds, freshmen, and second-year medical students
(age from 22-24 years) of both sexes. During the repeated contractions, one group
shows to be generally more fatigue resistant and recovers faster as well.
Question:
Further investigation shows that both fatigue resistance and recovery correlates well
with what factor?

Answer Choices:
A Female gender
B Male gender
C Freshmen status
D Second-year status
E Pre-med status
A Female gender

On average, males are physically stronger than females because of more total muscle
mass (more muscle fibers) in comparison to body weight. Male skeletal muscles are
generally faster and have higher maximum power output than female muscles.
Conversely, during repeated contractions, female muscles are generally more fatigue-
resistant and recover faster. Therefore, both fatigue resistance and recovery correlates
well with gender.

Age difference between groups is too narrow to influence the results of the isometric
contractile characteristics and fatigability. Consequently, the freshmens', second-year
students, and pre-meds' results will not be different than the results of one another.
Case #301538:
The mother of a 3-year-old boy brings her child to the family doctor because the child
was slow to begin walking, has difficulty in walking and standing, and fatigues easily.
She recalls that the youngest brother of her mother had similar problems and died in
his late teens. Tests of the child reveal creatinuria and elevated serum levels of
creatine phosphokinase (CPK). A histological specimen taken from a muscle biopsy
from this patient is shown in the image.
Question:
What is the most likely diagnosis?

Answer Choices:
A Nemaline myopathy
B Becker muscular dystrophy
C Duchenne muscular dystrophy
D Myasthenia gravis
E Mitochondrial myopathy

Image(s) / Chart(s):
Click image to view full size. Click open image to close. Click and hold open image to move.

C Duchenne muscular dystrophy


Image(s) / Chart(s):
Click image to view full size. Click open image to close. Click and hold open image to
move.

This child is suffering from Duchenne muscular dystrophy (DMD). This X-linked
condition is maternally transmitted to male children only. It usually manifests in the first
decade of life as difficulty in standing and walking. Muscle weakness initially appears in
the muscles of the hip girdle and upper legs. Later, the condition involves muscles of
the arms and respiratory system, with death usually occurring before age 20. Patients
fail to produce the protein dystrophin. This is a member of the spectrin superfamily, with
a MW = 427,000. Dystrophin is found underneath the sarcolemma in normal muscle,
especially where membrane folds are formed, e.g., at myotendinous and neuromuscular
junctions. It also interacts with actin filaments, stabilizing their side-side association as
well as binding to the amino-terminus. Muscle from DMD patients shows myocytes of
variable diameter (compared to the diameters of b and c) and unusual infiltration of
leukocytes (at a).

Muscles of DMD patients are more fragile and more easily ruptured. In addition, DMD
patients show reduced levels of enzymes normally found in the sarcoplasm with
concomitant increases in serum levels of creatine phosphokinase (CPK), lactate
dehydrogenase, and glucose phosphate isomerase. Urinary levels of creatine are
usually elevated. One of the normal functions of dystrophin is stabilization of the
sarcolemma against the physical stresses caused by frequent folding during cycles of
contraction and relaxation. Myosin and myoglobin levels would be reduced due to loss
on sarcomeres. Lysosomal hydrolases would be elevated because of increased activity
from phagocytic cells.

Becker muscular dystrophy is similar to DMD in its genetics and clinical course,
although it is less common and less severe. In such instances, dystrophin is formed in
lower quantities and has a slightly lower molecular weight than normal dystrophin.

Nemaline myopathy is usually caused by autosomal recessive mutations that lead to


accumulations of rod-shaped bodies just beneath the membrane of muscle cells. It
probably represents mutations in the protein α-actinin, leading to bundling of Z-line
proteins.

Myasthenia gravis is an autoimmune disease wherein patients develop antibodies to the


acetylcholine receptor of the neuromuscular junction. Routine histological biopsies of
muscle tissue are apparently normal. It does not usually present in young children, but
when seen in patients under 40, it is more common in women than in men.

Mitochondrial myopathy usually presents in young adults initially as proximal muscle


weakness, especially in the oculomotor muscles. The disease is commonly
characterized by peripheral accumulations of mitochondria (demonstrable only by
special stains) in muscle cells.

Keywords: Muscular dystrophy, muscle diseases


Case #301539:
The mother of a 3-year-old boy brings her child to the family doctor because the child
was slow to begin walking, has difficulty in walking and standing, and fatigues easily.
She recalls that the youngest brother of her mother had similar problems and died in
his late teens. Tests of the child reveal creatinuria and elevated serum levels of
creatine phosphokinase (CPK). A histological specimen taken from a muscle biopsy
from this patient is shown in the image.
Question:
What are biochemical tests of a punch biopsy from a skeletal muscle of this patient
most likely to show?

Answer Choices:
A Unusually low levels of CPK
B Elevated levels of myoglobin
C Normal amounts of myosin
D Reduced activity of lactate dehydrogenase
E Reduced activity of lysosomal hydrolases

Image(s) / Chart(s):
Click image to view full size. Click open image to close. Click and hold open image to move.

A Unusually low levels of CPK


Image(s) / Chart(s):
Click image to view full size. Click open image to close. Click and hold open image to
move.

This child is suffering from Duchenne muscular dystrophy (DMD). This X-linked
condition is maternally transmitted to male children only. It usually manifests in the first
decade of life as difficulty in standing and walking. Muscle weakness initially appears in
the muscles of the hip girdle and upper legs. Later, the condition involves muscles of
the arms and respiratory system, with death usually occurring before age 20. Patients
fail to produce the protein dystrophin. This is a member of the spectrin superfamily, with
a MW = 427,000. Dystrophin is found underneath the sarcolemma in normal muscle,
especially where membrane folds are formed, e.g., at myotendinous and neuromuscular
junctions. It also interacts with actin filaments, stabilizing their side-side association as
well as binding to the amino-terminus. Muscle from DMD patients shows myocytes of
variable diameter (compared to the diameters of b and c) and unusual infiltration of
leukocytes (at a).

Muscles of DMD patients are more fragile and more easily ruptured. In addition, DMD
patients show reduced levels of enzymes normally found in the sarcoplasm with
concomitant increases in serum levels of creatine phosphokinase (CPK), lactate
dehydrogenase, and glucose phosphoisomerase. Urinary levels of creatine are usually
elevated. One of the normal functions of dystrophin is stabilization of the sarcolemma
against the physical stresses caused by frequent folding during cycles of contraction
and relaxation. Myosin and myoglobulin levels would be reduced due to loss on
sarcomeres. Lysosomal hydrolases would be elevated because of increased activity
from phagocytic cells.

Becker muscular dystrophy is similar to DMD in its genetics and clinical course,
although it is less common and less severe. In such instances, dystrophin is formed in
lower quantities and has a slightly lower molecular weight than normal dystrophin.

Nemaline myopathy is usually caused by autosomal recessive mutations that lead to


accumulations of rod-shaped bodies just beneath the membrane of muscle cells. It
probably represents mutations in the protein α-actinin, leading to bundling of Z-line
proteins.

Myasthenia gravis is an autoimmune disease wherein patients develop antibodies to the


acetylcholine receptor of the neuromuscular junction. Routine histological biopsies of
muscle tissue are apparently normal. It does not usually present in young children, but
when seen in patients under 40, it is more common in women than in men.

Mitochondrial myopathy usually presents in young adults initially as proximal muscle


weakness, especially in the oculomotor muscles. The disease is commonly
characterized by peripheral accumulations of mitochondria (demonstrable only by
special stains) in muscle cells.

SECTION- NERVOUS TISSUE


Question:
The nerve cell bodies located at the tip of the arrowheads in the attached photomicrographic image are
perikarya which provide what kind of neural innervation?

Answer Choices:
A Preganglionic parasympathetic nerves
B Postganglionic parasympathetic nerves
C Preganglionic sympathetic nerves
D Postganglionic sympathetic nerves
E Sensory nerves for deep pressure

Image(s) / Chart(s):
Click image to view full size. Click open image to close. Click and hold open image to move.

B Postganglionic parasympathetic nerves

The cells depicted in the photomicrographic image at the tip of the arrowheads typify the
appearance of singular autonomic nerve cell bodies as found between the inner circular
and outer longitudinal muscle layers of the muscularis externus found in the intestine.
These cell bodies have a distinct nucleolus located in the center of the nucleus and give
the appearance of an "owl's eye". Nerve cell bodies in this location are part of the
parasympathetic autonomic nervous system and provide local postganglionic
innervation. The nuclei located between the two muscle layers in the intestinal region,
as depicted here, are the myenteric or Auerbach's plexus.

Question: BLOCK-1
Identify the organ in the attached photomicroscopic image based upon the cytoarchitectural
appearance of the cell arrangement and cell shapes.

Answer Choices:
A Cerebral cortex
B Cerebellar cortex
C Adrenal gland
D Dorsal horn of the spinal cord
E Thymus

Image(s) / Chart(s):
Click image to view full size. Click open image to close. Click and hold open image to move.

B Cerebellar cortex

The cerebellar cortex is characterized by 3 layers, which can be seen in this image. The
first is an outer molecular layer, largely a synaptic area with relatively few cell bodies
present. As depicted in this image, these cell bodies are represented with small, darkly
staining nuclei of the stellate cells and basket cells. The second layer is depicted by cell
bodies of the Purkinje cells, hence the Purkinje cell layer. These cells have large pear-
shaped soma aligned in a single row (note arrows). Apical dendrites from the Purkinje
cells extend into the molecular layer and are arranged as large, branching fan-shaped
structures. The Purkinje cell axon, which is small, passes into the deeper cerebellar
areas to synapse on nuclei located there. The third layer is the granule cell layer
consisting of densely packed, small neurons. These small multipolar nerve cells have
an axon that enters the molecular layer to synapse on many Purkinje cell dendrites. The
axon may also synapse on processes of the stellate and basket cells in the molecular
layer. The dendrites are short and branched.

Question:
These large nerve cell bodies are typical of neurons in the dorsal root ganglion, as demonstrated in the
photomicroscopic image, and can be best described as being:

Answer Choices:
A Pseudounipolar
B Bipolar
C Multipolar

Image(s) / Chart(s):
Click image to view full size. Click open image to close. Click and hold open image to move.
A Pseudounipolar

This photomicroscopic image depicts the appearance of a cross section through a


dorsal root ganglion. The neuronal cell bodies present are pseudounipolar neurons
embedded in a connective tissue array surrounding the collection of cell bodies, or
ganglion. Note the large soma with a centrally located nucleus. The faintly stained
nuclei surrounding the perikaryon are the nuclei of satellite cells, supportive cells of the
peripheral nervous system associated with the nerve cell bodies.
Question:
Specialized junctions or synapses between neurons are classified based upon their morphological
junction between 2 structures. A synapse occurring between an axon and the dendrite of an adjacent
neuron would be

Answer Choices:
A Axodendritic
B Axoaxonic
C Axosomatic
D Dendrodendritic
E Axodendroaxonic

A Axodendritic

The synapses between neurons typically occur as specialized junctions, which are
responsible for facilitation of impulse transmission from 1 neuron to the next.
Morphologically, several types of junctions between neurons (or the parts of neurons)
have been described. The synapse that occurs between an axon and a dendrite,
respectively, is regarded as axodendritic. Similarly, the synapse between an axon and a
cell body (soma) is axosomatic. Axoaxonic is a synapse that occurs between 2 axons,
and dendrodendritic is a synapse that occurs between two dendrites.
Question:
Colchicine is useful in experimental studies to aid in the visualization and location of neurohormonal
cells. Enhancement of the location of the cell bodies of these neurons by using this drug is due to the
inhibition of what cellular phenomenon?

Answer Choices:
A Inhibition of mitochondrial ATP production
B Inhibition of axonal transport of proteins
C Inhibition of protein synthesis
D Inhibition of mRNA transport to the endoplasmic reticulum
E Stimulation of protein synthesis

B Inhibition of axonal transport of proteins

While energetics and metabolism of substances within neurons may occur throughout
most parts of the cell, much of the activity is concentrated in the perikaryon. Therefore,
axonal transport is essential to convey material synthesized in the soma, such as
neurohormone to the axon and its terminals. Essential to the transport system are
structural elements such as neurofilaments, which are incorporated as part of the
microtubular system, providing a structural foundation or pathway for the transport
system. Colchicine inhibits synthesis of microtubules, which extend from the perikaryon
to the axon terminal and will result in disrupting the transportation of newly synthesized
proteins, such as neurohormones, to the axon terminals. This results in the
accumulation of greater concentrations of the neurohormone in the soma, allowing
easier detection of cells that synthesize and secrete the respective protein.
Question:
A primary tissue culture of fetal CNS tissue will contain neurons and a variety of glial cells. You wish to
selectively demonstrate the astrocytes in the culture system. Using an immunocytochemical technique,
which one of the following antibodies would you select?

Answer Choices:
A Microtubular protein
B Myelin associated protein
C Neuron-specific growth factor
D Neural cell adhesion molecule
E Glial fibrillary acidic protein
E Glial fibrillary acidic protein

Tissue cultures of the CNS will contain neurons and glial cells. Astrocytes contain
"intermediate" filaments, which are even more prevalent in fibrous astrocytes, than in
the protoplasmic astrocytes. The intermediate filaments are composed of glial fibrillary
acidic protein and antibodies to this protein may be used to selectively demonstrate the
presence of astrocytes in a tissue culture.
Question:
The cell indicated by the arrow in the attached photomicroscopic image is best described as a

Answer Choices:
A Pseudounipolar neuron
B Bipolar neuron
C Macrophage
D Multipolar neuron
E Mast cell

Image(s) / Chart(s):
Click image to view full size. Click open image to close. Click and hold open image to move.

D Multipolar neuron

The cell indicated by the arrow in this photomicroscopic images is typical of a multipolar
neuron. Note there are at least three processes which extend from the cell body, or
perikaryon. It is not possible to clearly visualize a lighter staining region characteristic of
the axon hillock, which would allow differentiation between the axon or dendrites. Note,
however, the predominant nucleus and nucleolus, and the intense granular staining of
the perikaryon cytoplasm. The intense staining is due to the presence of rough
endoplasmic reticulum and polysomes, and is referred to as Nissl substance. Nissl
substance extends into the larger diameter dendrites, but is absent in the axons.
Absence of the Nissl substance in the area from which the axon arises gives rise to the
lighter staining area called the axon hillock.
Question: CLASS QUIZ
Within 2 days following traumatic injury to the spinal cord in which the corticospinal tract was severed
at the T2 level, histological changes in the perikaryon occurs and continues for several weeks in
proportion to the damage to the neuron and amount of axoplasm lost. One alteration of the cell body
that occurs is

Answer Choices:
A Anterograde degeneration
B Chromatolysis
C Neurite induction
D Dendritic sprouting
E Axonal sprouting

B Chromatolysis

After a cell process is cut, immediate changes occur in the cell body. One such change
that is histologically observable within 2 days following severing a process is the loss of
staining of the Nissl substance. This reduction of Nissl substance within the perikaryon
cytoplasm is referred to as chromatolysis, or literally, loss of color. This loss continues in
proportion to the damage to the neuron and may be a harbinger of cell death.
Anterograde degeneration is the degeneration of the axon that occurs distal to the
severed axon. Sprouting and neurite induction relate to regenerative processes if the
neuron survives the trauma.
Question:
Identify the large cell at the arrow as depicted in the attached photomicroscopic image

Answer Choices:
A Purkinje cell
B Betz cell
C Macrophage
D Mast cell
E Spermatozoa

Image(s) / Chart(s):
Click image to view full size. Click open image to close. Click and hold open image to move.
A Purkinje cell

The arrow in this photomicroscopic image depicts a large Purkinje cell. These cells
make up the second layer of a three-layered cerebellar cortex. These cells have very
characteristic appearances being large pear-shaped cells, lying in a single row. They
may also be seen to have a large process extending into the molecular cell layer (as
seen in this image), representing the large, branching dendrite that is reminiscent of a
fan. Underlying the Purkinje cell layer is the granule cell layer, consisting of small,
closely packed nuclei with scant cytoplasm.
Question:
The attached photomicroscopic image best depicts what tissue?

Answer Choices:
A Nerve fiber bundle
B Regularly arranged dense connective tissue
C Epidermis
D Smooth muscle
E Tendon

Image(s) / Chart(s):
Click image to view full size. Click open image to close. Click and hold open image to move.
A Nerve fiber bundle

This image depicts a peripheral nerve fiber bundle cut longitudinally. Upon close
inspection, the axons may be observed traversing the lighter staining myelin sheaths.
Also, a number of nodes of Ranvier are visualized where two adjacent myelinated
segments are apposed. Occasionally, fine strands of connective tissue composing the
endoneurium can be observed among the nerve fibers.
Question:
Upon appropriate stimulation, a neuron may react to impulses that are summated at the axon hillock.
Summation of these impulses may result in a depolarization of the initial segment and the generation of

Answer Choices:
A Neurotransmitter release
B An action potential
C A hyperpolarized state
D Saltatory conduction
E Calcium efflux
B An action potential

The action potential first arises in the initial segment of the axon as a result of the
summation of impulses impinging upon the axon soma and hillock. The action potential
is then conducted along the axon to the axon terminals.

Question:
In this photomicroscopic image, formation of the substance located at the tip of the pointer as found in
the peripheral nervous system is by what cell type
Answer Choices:
A Astrocyte
B Microglia
C Schwann cell
D Satellite cell
E Oligodendrocyte

Image(s) / Chart(s):
Click image to view full size. Click open image to close. Click and hold open image to move.

C Schwann cell

The tip of the pointer is located on the myelin sheath surrounding an axon located in a
peripheral nerve fiber bundle. The cell responsible for forming the myelin sheath in the
peripheral nervous system is the Schwann cell. One Schwann cell is responsible for
forming the myelin sheath on one segment of one axon of a nerve fiber. In contrast, the
oligodendrocyte is responsible for forming the myelin sheath surrounding axons located
in the central nervous system.

Question:
Specialized junctions or synapses between neurons are classified based upon their morphological
junction between 2 structures. A axosomatic synapse describes a junction between

Answer Choices:
A Axon and axon
B Axon and nerve cell body
C Axon and skeletal muscle
D Axon and smooth muscle
E Axon and Schwann cell
B Axon and nerve cell body

The synapses between neurons typically occur as specialized junctions, which are
responsible for facilitation of impulse transmission from one neuron to the next.
Morphologically, several types of junctions between neurons (or the parts of neurons)
have been described. The synapse that occurs between an axon and a dendrite is
regarded as axodendritic. Similarly, the synapse between an axon and a cell body
(soma) is axosomatic. Axoaxonic is a synapse that occurs between 2 axons, and
dendrodendritic is a synapse that occurs between 2 dendrites.
Question:
Type A beta fibers in the mammalian peripheral nervous system carry what type of information?

Answer Choices:
A Proprioception
B Pain and temperature
C Preganglionic autonomic
D Touch and pressure
E Postganglionic sympathetic

D Touch and pressure

A-alpha fibers conduct proprioception information. Pain and temperature are conducted
by A-delta fibers. Preganglionic autonomic information is carried by B-fibers.
Postganglionic sympathetic information is carried primarily by C-fibers.

Question:
Some viruses may enter the CNS after gaining access to a nerve terminal, such as a sensory nerve
ending. The transport system normally active in neurons that aids the transport of viruses which enter a
nerve ending is which one of the following type of transport system?

Answer Choices:
A Anterograde, fast transport
B Anterograde, slow transport
C Retrograde, fast transport
D Retrograde, slow transport
E Passive diffusion

C Retrograde, fast transport

Axonal transport serves to move materials from the perikaryon to the axon terminals by
anterograde transport, and in the opposite direction by retrograde transport. The
transport system has both a slow component and a fast component, that is, different
components based upon the rate at which materials are transported. The fast transport
system requires energy (ATP) and conveys substances up to nearly 500 mm/day while
the slow transport system is much slower, and transports materials at less than 5
mm/day.
Interestingly, anterograde transport utilizes both the fast and slow transport system, but
retrograde transport utilizes the fast transport system, only. Transportation of viruses
which enter a nerve ending occur through a retrograde, fast transport system.
Question:
The basophilic Nissl bodies found in neuron cell bodies are

Answer Choices:
A Clusters of endoplasmic reticulum
B The Golgi apparatus
C Stacks of mitochondria
D Lysosomes
E Aggregates of lipofuscin pigment

Question:
Which one of the following cells in the central nervous system is located in the synaptic region and may
function to help confine the neurotransmitters, when released upon stimulation of the neuron, to the
synaptic cleft?

Answer Choices:
A Astrocyte
B Oligodendrocyte
C Pericyte
D Microglia
E Satellite cell

A Astrocyte

One type of astrocyte, the perisynaptic astrocyte, provides a covering for the axon
terminal at the synaptic junction. It is believed that one function of this astrocyte process
is to help limit the extent the neurotransmitters may diffuse following release into the
synaptic cleft so that the neurotransmitter is limited to the cleft. Also, the astrocyte
process may function in the removal of excess neurotransmitter by a pinocytosis
process.
Question:
The concentric electron dense lamellations ("major dense lines") seen in a myelin sheath are

Answer Choices:
A Fused inner layers of Schwann cell membranes
B Fused outer layers of Schwann cell membranes
C Neurofilament networks
D Tertiary lysosomes
E A shrinkage artifact caused by glutaraldehyde fixation
A Fused inner layers of Schwann cell membranes

The electron dense lamellations of myelin sheaths, called "major dense lines" are
formed from the fusion of the inner layers of the Schwann cell membrane as the cell
wraps around the axon. The fusion of the outer layers of the Schwann cell membrane
produces lighter lines called "intraperiod lines", which alternate with the major dense
lines.

Question:
The blood-brain barrier is either absent or functionally ineffective in which one of the following areas of
the brain?

Answer Choices:
A Hypothalamic median eminence
B CA1 area of the hippocampal formation
C Nucleus accumbens of the striatum
D Subiculum
E Ventral horn of the spinal cord
A Hypothalamic median eminence

The blood-brain barrier, composed of complex tight junctions between the endothelial
cells of the neural capillaries and frequently associated with the end processes of
astrocytes, regulate the microenvironment of the brain. There are some areas of the
central nervous system in which the blood brain barrier is either absent or functionally
ineffective such as in the median eminence. This area, and others associated with
regulation of neurosecretory function for endocrine system control apparently "sample""
the hormones and other circulating macromolecules as part of the endocrine feedback
system of regulation.
Question:
Saltatory conduction is

Answer Choices:
A Continuous conduction of an electrical impulse along an unmyelinated axon
B Discontinuous conduction of an electrical impulse along a myelinated axon
C Impulse conduction that is dependent on sodium salts to propagate
D Impulse conduction that is dependent on potassium salts to propagate
E Conduction of an electrical impulse across synapses

B Discontinuous conduction of an electrical impulse along a myelinated axon


Saltatory conduction is the discontinuous conduction of an electrical impulse along the
nodes of Ranvier of a myelinated axon. Because membrane depolarization occurs only
at the nodes of Ranvier instead of along the entire membrane, saltatory conduction is
very rapid.
Because thicker axons usually have a greater distance between nodes of Ranvier than
thinner ones, thick axons conduct more quickly than thin ones.
In myelinated axons, the sodium channels are concentrated at the nodes of Ranvier and
the potassium channels are found in the internodal segments. If the axons are
demyelinated (as in demyelinating diseases), this placement of channels makes
continuous conduction of an impulse particularly slow. The internodal segments may be
effectively unable to conduct an impulse.
Question:
Identify the structure at the tip of the arrow in the attached photomicrographic image

Answer Choices:
A Schwann cell nucleus
B Axon
C Axon hillock
D Myelin sheath
E Perikaryon

Image(s) / Chart(s):
Click image to view full size. Click open image to close. Click and hold open image to move.

B Axon

The arrow is pointing to the axon of this nerve fiber cut in cross section. The nerve fiber
is myelinated, and the remnants of the myelin sheath can be visualized surrounding the
fiber. Close observation may reveal small connective tissue filaments coursing
throughout the nerve fiber bundle constituting the endoneurium.

Question:
The axon hillock contains
Answer Choices:
A Rough endoplasmic reticulum
B Ribosomes
C Microtubules
D Golgi complex
E Synaptic vesicles

C Microtubules

The axon hillock contains microtubules, which are arranged in bundles. In addition,
neurofilaments, mitochondria and vesicles pass through it into the axon.
It, however, does not contain rough endoplasmic reticulum, ribosomes, Golgi complex
and synaptic vesicles.

Question:
The primary function of the structure at the tip of the pointer in the photomicroscopic image is to

Answer Choices:
A Protect the axon from temperature changes
B Increase the velocity of conduction of the action potential
C Decrease the velocity of conduction of the action potential
D Reduce the occurrence of saltatory conduction
E Provide a blood-brain barrier

Image(s) / Chart(s):
Click image to view full size. Click open image to close. Click and hold open image to move.

B Increase the velocity of conduction of the action potential

The tip of the pointer is located on the myelin sheath of a nerve fiber located in a
peripheral nerve bundle. Furthermore, a "node of Ranvier" can be visualized,
representing the apposition of two segments of myelin along an axon. The myelin
sheath serves to "electrically" insulate the axon, such that conduction of the action
potential along the axon occurs by 'jumping" from node-to-node in a saltatory fashion,
thus substantially increasing the conduction velocity along the axon. The myelin sheath
contributes little to the blood-brain barrier, as found in the central nervous system, which
is formed by the astrocytes.
Question:
Which of the following neural structures present in the deep dermis and hypodermis is regarded as the
deep pressure receptor for mechanical and vibratory pressure?

Answer Choices:
A Merkel's cells
B Meissner's corpuscles
C Pacinean corpuscles
D Ruffini endings
E Arrector pili
C Pacinean corpuscles
Image(s) / Chart(s):
Click image to view full size. Click open image to close. Click and hold open image to
move.

Pacinian corpuscles (composed of myelinated nerve ending surrounded by a capsule


structure) are deep pressure receptor cells for mechanical and vibratory pressure. They
are found in the deeper dermis and hypodermis, especially in the fingertips, in
connective tissue in general, and in association with joints, periosteum and internal
organs.
Merkel's cells (located in stratum basale, and containing small, dense-cored granules)
function as sensory mechanoreceptors. Meissner's corpuscles (located in the lips, and
palmar and volar surfaces) are touch receptors. Ruffini endings are simplest
encapsulated mechanoreceptors. Errector pili are muscles located around the hair
follicle.
Question:
Which of the following is true for passive ion channels?
Answer Choices:
A They are found on cell bodies and dendrites but not on axons or axon
terminals
B They determine whether or not a cell will discharge an action potential
C They are always open
D They are not involved in the integration of incoming information
E They can be selective for chloride but not potassium ions
C They are always open

Passive ion channels are found throughout axons and on axon terminals and are
always open. Chemically and electrically gated (not passively gated) ion channels
initiate and propagate action potentials. However, passively gated ion channels are
clearly involved in the integration of incoming information and can be selective for a
particular type of ion which could be either chloride or potassium among others.
Question:
The cranial nerve motor nuclei in the brainstem correspond to which structure in the spinal cord?

Answer Choices:
A Dorsal horn
B Lateral horn
C Ventral horn
D Intermediolateral cell column
E Internal carotid plexus

C Ventral horn

The cranial nerve motor nuclei in the brainstem correspond to the ventral horn (also
called the anterior horn) of the spinal cord. Both of these structures contain cell bodies
of motor neurons and send axons to innervate muscles, either through spinal nerves or
through cranial nerves.
Question:
The physical basis of the blood-brain barrier is

Answer Choices:
A A specialized basal lamina of CNS neurons
B Tight junctions between capillary endothelial cells in the brain
C The pia-arachnoid membrane
D A unique collagen network in the CNS
E Specialized cell membranes of glial cells
B Tight junctions between capillary endothelial cells in the brain

The physical basis of the blood-brain barrier is the tight junctions between capillary
endothelial cells in the CNS. This barrier is impervious to both body fluids and the solids
dissolved in them. Only the posterior pituitary, the locus ceruleus and the substantia
nigra are not protected by these tight junctions.
There is some experimental evidence that the perineurium forms a similar barrier
around peripheral nerves.
Question:
Which of the receptors respond primarily to movements of collagen fibers in connective tissue?

Answer Choices:
A Bare nerve endings
B Pacinian corpuscles
C Meissner's corpuscles
D Ruffini endings
E End bulb of Krause

D Ruffini endings

Ruffini endings are elongate connective tissue capsules through which collagen nerve
fibers pass. As with the Pacinian and Meissner's corpuscles, an unmyelinated tip of a
myelinated nerve is enclosed within the connective tissue capsule. This unmyelinated
tip branches may times within the capsule, with each tip having a bulbous ending. When
the collagen fibers are disturbed, the nerve endings are stimulated.
Question:
Satellite cells:

Answer Choices:
A Are the cells that make up the perineurium.
B Surround the regenerating tip of an amputated axon.
C Enclose the perikarya (cell bodies) in ganglia.
D Are a form of glial cells in the CNS.
E Are the only cells in the peripheral nervous system that do not originate in the
neural crest.

C Enclose the perikarya (cell bodies) in ganglia.


Satellite cells encircle the cell bodies of neurons in ganglia. Like Schwann cells, they
originate in the neural crest and insulate the neurons from the rest of the organism.

Question:
Schwann cells of the peripheral axons are embryologically derived from

Answer Choices:
A Neural crest cells
B neuroepithelium
C Neural tube
D Neural plate

A Neural crest cells

Neural crest cells migrate throughout the body of the embryo and give rise to cranial
and spinal ganglia, Schwann cells of peripheral axons, and several other structures
(e.g., chromaffin cells of the adrenal medulla, melanocytes, odontoblasts, etc.).
Neuroepithelium thickens and differentiates to form the neural plate. The neural plate
invaginates and thickens to form the neural groove. Neural tube is the cylindrical
structure that results from fusion of the edges of the neural groove. This groove
enlarges at its cranial end to form the brain, while the remaining portion gives rise to the
spinal cord.
Question:
Which of the following cells in the inner ear are involved in detecting the movements of the head?

Answer Choices:
A Hair cells in the maculae
B Outer pillar cells
C Inner pillar cells
D Cells of Hensen
E Hair cells in the organ of Corti

Move
Close
A Hair cells in the maculae

Neuroepithelial hair cells in the maculae of the saccule and the utricle detect linear
movement of the head. These cells are connected to the vestibular portion of the
acoustic nerve.
Inner and outer pillar cells are associated with the spiral organ of Corti, while hair cells
in the organ of Corti function in the reception of sound (and can respond to different
sound frequencies). Cells of Hensen (along with border cells) delineate the inner and
outer borders of the spiral organ of Corti.
Question:
Which of the following detect pressure and vibration in fingers, connective tissue, periosteum, internal
organs and joints?

Answer Choices:
A Bare nerve endings
B Pacinian corpuscles
C Meissner's corpuscles
D Ruffini endings
E End bulb of Krause

B Pacinian corpuscles

Pacinian corpuscles detect pressure and vibration in a wide variety of tissues. They are
one variant of encapsulated nerve ending and consist of multiple concentric layers of
connective tissue capsule. The layers are separated from each other by collections of
fluid that resembles lymph.
One or two nodes after the nerve ending enters the innermost layer, or lamella, it loses
its myelin sheath. From that point on, the nerve ending is surrounded by Schwann cells,
but not by myelin. This layer of Schwann cells is considered the inner capsule.
When the capsule is moved, it displaces the nerve ending. This displacement is
sufficient to depolarize the nerve and send a signal to the brain.
Question: FINALS
In the PNS, after a distal portion of an axon has undergone Wallerian (or anterograde) degeneration,
what part of the distal axon and its sheath remains?

Answer Choices:
A The axon itself
B The myelin sheath
C The Schwann cells
D Nothing; the entire axon and its supporting structure degenerates
E The neurofilament system
C The Schwann cells

After the process of Wallerian degeneration is completed, the Schwann cells remain,
enclosing an empty tube where the axon and its myelin sheath were. If the regenerating
proximal stump of the axon can reach this tube, the tube will guide the regenerating
stump to its appropriate destination and provide an environment that enhances growth
of the axon.
Question:
Which one of the following statements concerning the degenerative changes in the proximal segment
of the axon is true?

Answer Choices:
A Both the axon and its myelin sheath degenerates completely, and their
remnants are removed by the macrophages
B The injury to the axon causes the perikaryon and its processes to degenerate
and die
C After the regressive changes, the proximal segment of the axon grows and
branches, forming several filaments that progress in the direction of the column of
Schwann cells
D When there is an extensive gap between the distal and proximal segments (as
in the case of an amputation of the limb), both the distal and proximal segments
are unable regenerate and fill the gap
C After the regressive changes, the proximal segment of the axon grows and
branches, forming several filaments that progress in the direction of the column of
Schwann cells

Immediately following the injury, the proximal segment of the axon degenerates close to
the wound for a short distance, but growth starts as soon as debris is removed by
macrophages. However, the axonal segment distal to the injury (as it has been
separated from its trophic center) degenerates completely along with its myelin sheath,
and their debris is removed by macrophages. While these regressive changes are
taking place, Schwann cells proliferate within the remaining connective tissue sleeve,
giving rise to solid cellular columns. These rows of Schwann cells serve as guides to the
sprouting axons formed during the reparative phase.
When there is an extensive gap between the distal and proximal segments, or when the
distal segment disappears altogether (as in the case of amputation of a limb), the newly
grown axon may form a neuroma which can be the source of spontaneous pain.
Question:
Lidocaine, procaine and tetracaine are structurally related local anesthetic drugs that act to inhibit the
propagation of the electric current during the neuronal action potential. Each of these drugs blocks the
pore of which type of ion channel?

Answer Choices:
A Inward rectifying potassium channel
B Voltage-gated calcium channel
C Voltage-gated sodium channel
D Voltage-gated potassium channel
E Cyclic nucleotide gated ion channel
C Voltage-gated sodium channel
The local anesthetic drugs mentioned above take advantage of differing ion channel
structures to specifically block the pore of sodium (Na+) channels. The opening of
voltage-gated Na+ channels initiates the neuronal action potential. Therefore, drugs that
block Na+ channels inhibit the initiation of neuronal action potentials to result in
anesthesia.
The opening of voltage-gated potassium (K+) channels follows action potential initiation
and contributes to the repolarization of the neuron to reestablish the resting membrane
potential.
Voltage-gated calcium (Ca+) channels are involved at the terminal site of the neuronal
action potential at the presynaptic membrane where their voltage dependent opening
stimulates the release of neurotransmitter-containing synaptic vesicles.
Inward rectifying potassium (K+) channels are K+ selective channels that establish the
basal permeability of the neuronal membrane and contribute to the resting membrane
potential. These ion channels are often the downstream targets of G-protein coupled
receptors.
Cyclic nucleotide-gated channels are not expressed in all neurons; they are primarily
expressed in sensory neurons such as in retinal rods and cones where they are the
downstream target of the light sensitive G-protein coupled receptor, rhodopsin.
Question:
A second year medical student joins a neurobiology lab during his summer vacations. The lab director
asks him to study the excitability of neurons. The experimental model is the spinal motor neuron
isolated from the body of an experimental animal. Given that he has induced stimulation in the
following five regions, which one would respond to the lowest threshold strength?

Answer Choices:
A Internode
B Initial segment
C Dendrite
D Cell body
E Myelin sheath

B Initial segment

Initial segment is the interval between the axon hillock and the point at which
myelination begins. The axolemma of this segment presents several types of ion
channels, which regulate the conversion of graded potential generated in the cell body
into nerve action potential and transmit it along the axolemma. The high excitability is
determined by the number of sodium channels in a particular area. The number of
sodium channels per square μm of membrane in myelinated mammalian neurons has
been estimated to be 50-75 in the cell body, 350-500 in the initial segment, less than 25
on the surface of the myelin, 2000-12000 at the nodes of Ranvier, and 20-75 at the
axon terminals. Myelin is an effective insulator, and the current flow through this is
negligible. Therefore, impulse conduction from one node of Ranvier to the next is
favored.

Question:
Negative feedback loops serve as control mechanisms for various physiological functions in the body.
Positive feedback loops generally result in pathology except for in a few specific instances. Which of
the following is an example of a positive feedback loop?

Answer Choices:
A Progesterone and luteinizing hormone during normal female reproductive cycle
B Testosterone and follicle stimulating hormone in normal adult male
C Oxytocin and stretching of the cervix in labor
D Progesterone and human chorionic gonadotropin during pregnancy
E Luteinizing hormone and testosterone in the normal adult male
C Oxytocin and stretching of the cervix in labor

As LH levels rise post-ovulation, progesterone from the corpus lutea also increases.
However, when progesterone achieves specific levels, release of LH is inhibited via
negative feedback.
FSH does not directly stimulate testosterone production, yet GnRH release is inhibited
by testosterone.
As the cervix stretches, it releases oxytocin, which stimulates contraction of the muscles
of the wall of the uterus. These contractions put pressure on the cervix, which releases
more oxytocin. This positive feedback loop is broken when the fetus leaves the uterus
and the pressure on the cervix stops.
Human chorionic gonadotropin and progesterone interact in that hCG has LH-like
activity and stimulates the CL to produce progesterone. However, progesterone levels
are maintained at a specific level and do not continue to climb, as they would in positive
feedback.
LH stimulates the Leydig cells to produce testosterone. Testosterone levels are
monitored by the hypothalamus and pituitary. As these levels rise, LH release is
inhibited.
Question:
Axonal injury causes several changes in the perikaryon. What is one of these changes?

Answer Choices:
A Increase in the Nissl substances
B Increase in cytoplasmic basophilia
C A decrease in the volume of the perikaryon
D The migration of the nucleus to the peripheral position in the perikaryon
E Eventual death of the perikaryon

D The migration of the nucleus to the peripheral position in the perikaryon


Axonal injury causes several changes in the perikaryon: chromatolysis (i.e., dissolution
of Nissl substances with a consequent decrease in cytoplasmic basophilia); an increase
in the volume of the perikaryon; and migration of the nucleus to the peripheral position
in the perikaryon. Perikaryon normally persists unless it is involved in the injury.

uestion:
Which one of the following neural components degenerate due to a neural lesion?

Answer Choices:
A Perikaryon and its processes
B Neurons functional connected to the dead neuron
C Neuroglia of central nervous system
D Schwann cells of peripheral nervous system
E Ganglionic satellite cells of the peripheral nervous system

A Perikaryon and its processes

Death of nerve cell is limited to its perikaryon and processes. The neurons functionally
connected to the dead neuron do not die, except for those with only one link. In contrast
to nerve cells, neuroglia of the central nervous system, Schwann cells, and ganglionic
satellite cells of the peripheral nervous system, are able to divide by mitosis. Spaces in
the central nervous system left by nerve cell lost by disease or injury are invaded by
neuroglia.

uestion:
After an axon is crushed or severed, the cytoplasm of the neuron's cell body

Answer Choices:
A Shrinks temporarily
B Takes on a "hard" appearance under the light microscope
C Increases in volume by about 20%
D Becomes deeply eosinophilic
E Becomes deeply basophilic

C Increases in volume by about 20%

After its axon is injured, the cytoplasm of the neuron's cell body swells and the Nissl
substance becomes dispersed. Although under the light microscope the Nissl substance
seems to disappear, the overall amount of Nissl substance increases as the cell body
begins to synthesize more protein.
Both the nucleus of the neuron and its nucleolus move to eccentric locations after an
injury.
Question:
What is the reaction to injury of the segment of the axon proximal to the site of injury?

Answer Choices:
A The proximal axon degenerates entirely and regenerates from the cell body
B The proximal axon degenerates only a short distance
C The proximal axon does not degenerate but accumulates Nissl substance
D The proximal axon secretes nerve growth factor to stimulate Schwann cells
E The myelin sheath of the proximal axon degenerates, leaving the tip of the
axon unmyelinated
B The proximal axon degenerates only a short distance

Although the distal segment of a severed neuron degenerates entirely, the proximal
segment of the axon degenerates only a short distance. Then the tip of the axon swells,
distorting the myelin sheath that covers it. On electron microscopy, the swollen tip is
found to be filled with fibrils and organelles.
About four days after the injury, many regenerating segments of axon, called neurites,
begin growing from the swollen proximal tip of the neuron. These grow at the rate of 1.0
to 2.5 mm/day and are guided by Schwann cells through the scar tissue. Under the
proper conditions, some of the neurites will contact the empty Schwann cell tube of the
degenerated distal axon and begin growing through the tube. The Schwann cell tube
will guide the neurites to the appropriate end organ.

Question: BLOCK-1
Wallerian degeneration is

Answer Choices:
A Degeneration of a nerve's axon distal to an injury site
B Degeneration of a nerve's axon proximal to an injury site
C Degeneration of a dendrite distal to an injury site
D Degeneration of a dendrite proximal to an injury site
E A reaction to an injury to the cell body itself

A Degeneration of a nerve's axon distal to an injury site

Wallerian degeneration is the term used to describe the degeneration of a nerve's axon
distal to the injury site. The distal portion of the axon breaks into pieces. These
individual axon fragments bead up and are surrounded by remnants of the myelin
sheath, which also fragments. In the periphery, this process takes a matter of days. In
the central nervous system it takes weeks.
The remnants of the axon and its myelin sheath are removed by cells of the phagocytic
system - in the PNS, Schwann cells and in the CNS, microglia.
Question:
Which of the following detect low frequency touch in hairless skin

Answer Choices:
A Bare nerve endings
B Pacinian corpuscles
C Meissner's corpuscles
D Ruffini endings
E End bulb of Krause

C Meissner's corpuscles

Meissner's corpuscles are specialized receptors that lie in the papillary dermis,
perpendicular to the surface of the skin. They are primarily found in hairless skin,
particularly the fingers and toes. The capsule of the corpuscle is an elongated cone,
within which an unmyelinated nerve ending spirals to the tip of the cone. As in the
Pacinian corpuscle, the unmyelinated nerve ending is enclosed by Schwann cells.

Question:
Most of the sensory receptors in the skin are

Answer Choices:
A Bare nerve endings
B Pacinian corpuscles
C Meissner's corpuscles
D Ruffini endings
E End bulb of Krause

A Bare nerve endings

Most of the sensory receptors in the skin are bare, or "free", nerve endings. These free
nerve endings are found in the stratum granulosum of the epidermis and looped around
the outer root sheaths of the hair follicles. They are capable of transmitting information
about heat, cold and fine touch to the central nervous system. Those attached to hair
follicles are mechanoreceptors, transmitting information about hair movements.
These bare nerve endings are histologically identical, irrespective of which sensation
they are designed to detect.
Question:
The white matter of the central nervous system contains

Answer Choices:
A Myelinated axons
B Myelinated and unmyelinated axons
C Myelinated and unmyelinated axons and dendrites
D Neuronal cell bodies, axons and dendrites
E Neuroglia, neuronal cell bodies, axons and dendrites

B Myelinated and unmyelinated axons

The white matter of the central nervous system is composed of both myelinated and
unmyelinated axons and their supporting glial cells. When axons are grouped with other
axons that have a similar function, they form tracts. Under normal circumstances, tracts
cannot be distinguished histologically.
Grey matter is composed of neuronal cell bodies, neuroglial cells, dendrites and some
axons. When neuronal cell bodies with a similar purpose are grouped together, the
result is called a nucleus.. Nuclei in the CNS correspond to ganglia in the peripheral
nervous system.

Question:
Which of the following structures contains sympathetic ganglia?

Answer Choices:
A Visceral motor ganglia of cranial nerve III
B Visceral motor ganglia of cranial nerve VII
C Visceral motor ganglia of cranial nerve X
D Ganglia surrounding abdominal and pelvic organs
E Vertebral and paravertebral ganglia

E Vertebral and paravertebral ganglia

The sympathetic ganglia are the vertebral and paravertebral ganglia, also called the
sympathetic chain. Preganglionic neurons of the parasympathetic system lie in the
brainstem and in the sacral level of the spinal cord. Axons from these cells synapse in
the visceral motor ganglia of cranial nerves III, VII, IX and X and in ganglia surrounding
various abdominal and pelvic organs.
Question:
Where do the axons of the autonomic neurons that make up the intermediolateral columns of the
thoracic and sacral spinal cord terminate?

Answer Choices:
A On the effector organ
B In the medulla
C In the pons
D In the peripheral autonomic ganglia
E In the midbrain

D In the peripheral autonomic ganglia

The autonomic neurons of the intermediolateral spinal cord columns are part of an
efferent system, so the axons of these cells extend toward the viscera, not deeper into
the central nervous system. Unlike the efferent somatic system, in which one neuron
extends from the CNS to the effector organ, in the autonomic system two neurons form
a chain that extends from the CNS to the effector organ.
The "presynaptic" or "preganglionic" neuron has its cell body in the spinal cord. Its axon
leaves the spinal cord through the ventral root and synapses with a number of
"postsynaptic" or "postganglionic" neurons. The postsynaptic neurons have their cell
bodies in autonomic ganglia. These cells send axons to the effector organs.

Question:
The importance of the organizational pattern of the neural tube is

Answer Choices:
A A failure of the sulcus limitans to form results in amyotrophic lateral sclerosis
B Its pattern is carried over into the mature spinal cord
C The ventral basal plate develops into the major sensory horn of the spinal cord
D It illustrates the Dilbert principle
E The dorsal alar plate develops into the major motor horn of the spinal cord

B Its pattern is carried over into the mature spinal cord

The importance of the organizational pattern of the neural tube is that it is carried over
into the mature spinal cord. The neural tube contains a ventral motor basal plate which
becomes the ventral motor horn of the spinal cord. The dorsal sensory alar plate
becomes the dorsal sensory horn of the spinal cord. The sulcus limitans, which runs
between the ventral motor basal plate and the dorsal sensory alar plate, is where the
efferent autonomic system develops.

Question: CLASS QUIZ


Wallerian degeneration

Answer Choices:
A Is the degeneration of the axon if it is cut from the cell body, either purposely
or accidentally
B Consists in the fragmentation and loss of the Nissl bodies and in nuclear
changes
C Is the degeneration of one or two segments of the axon proximal to the injury
D Has been widely used for determining the origin of fibers, both of peripheral
nerves and the tracts in the central nervous system

Question:
Which of the following statements concerning the peripheral terminations of autonomic nerve fibers is
true?

Answer Choices:
A These nerve fibers convey information (impulses) from receptors, e.g., in skin,
organs, and special senses, to the central nervous system
B Visceral (involuntary) efferent fibers comprise most of the autonomic nervous
system. They convey impulses from the central nervous system to smooth and
cardiac muscles, and to glands
C They receive sensations near or in the skin, such as pain, heat, cold, pressure
and touch
D They receive stimuli from within the body, arising from the viscera and blood
vessels

Question:
Which of the following statements about the Merkel cells is true?

Answer Choices:
A They are modified epidermal cells that are located in the stratum basale of
fingertips
B These cells closely resemble, both morphologically and functionally, dendritic
cells in the lymph nodes, spleen, and thymus
C They possess surface Fc, Ia, and C3 receptors
D As an antigen-presenting cells, they are involved in the initiation of cutaneous
contact hypersensitivity reactions and in other cell-mediated immune responses in
the skin
E Several laboratories have shown that in skin biopsies from individuals with
AIDS or AIDS-related complex, these cells have the human immunodeficiency
virus (HIV) in their cytoplasm
A They are modified epidermal cells that are located in the stratum basale of fingertips

Merkel's cells are modified epidermal cells that are located in the stratum basale of
fingertips. They are closely associated with the expanded bulb of an afferent myelinated
nerve fibers. The combination of the neuron and epidermal cell, called a Merkel's
corpuscle, is a very sensitive mechanoreceptor.
All other statements are applicable to the Langerhans cells in the skin.
These cells closely resemble, both morphologically and functionally, dendritic cells in
the lymph nodes, spleen, and thymus. They possess surface Fc, Ia, and C3 receptors.
As an antigen-presenting cells, they are involved in the initiation of cutaneous contact
hypersensitivity reactions and in other cell-mediated immune responses in the skin.
Several laboratories have shown that in skin biopsies from individuals with AIDS or
AIDS-related complex, these cells have the human immunodeficiency virus (HIV) in
their cytoplasm.
Question:
Which one of the following functions is attributable to nociceptors?

Answer Choices:
A Pressure reception
B Touch reception
C Fine-touch perception
D Cold perception
E Pain stimuli

E Pain stimuli

Nociceptors (delicate myelinated fibers that lose their myelin sheath before entering the
epidermis) are sensitive to pain stimuli.
Pressure and touch is perceived by Pacinian corpuscles, free nerve endings and
Ruffini's endings, while fine-touch is perceived by Meissner's corpuscles. Cold receptors
respond to temperature below 25° - 30° C.
Question:
Which of the following acts as a supporting cell in the Peripheral Nervous System?

Answer Choices:
A Schwann cell
B Oligodendrocyte
C Ependymal cell
D Microglia
E Astrocyte
A Schwann cell

Most axons in adult nerve tissue are covered by single or multiple folds of a sheath cell.
In Peripheral nerve fibers, the sheath cell is the Schwann cell, and in central nerve
fibers it is the oligodendrocyte.
Ependymal cells are low columnar ciliated epithelial cells that line the cavities of the
central nervous system. Astrocytes are star-shaped cells, and they bind neurons to
capillaries and to the pia mater, while microglia are small elongated cells with short
irregular processes. They are involved with inflammation and repair in the adult central
nervous system, and they produce and release neutral proteases and oxidative radicals.

estion:
Cell bodies of preganglionic sympathetic neurons are located

Answer Choices:
A In the brain
B In the sacral spinal cord
C In the thoracic and lumbar spinal cord
D Throughout the central nervous system
E In autonomic ganglia only

C In the thoracic and lumbar spinal cord

Cell bodies of preganglionic sympathetic neurons are located in the thoracic spinal cord
and first two segments of the lumbar spinal cord. Postganglionic sympathetic cell bodies
are situated in autonomic ganglia located on both sides of the spinal cord; they are not
found in the brain or anywhere else in the central nervous system.

Question:
With regard to axonal injury, chromatolysis is

Answer Choices:
A Loss of nuclear basophilia
B Loss of cytoplasmic eosinophilia
C Loss of Nissl substance
D Loss of the myelin sheath
E Degeneration of the proximal mitochondria

C Loss of Nissl substance


In the first three weeks after an axon is injured, the cell body swells and the nucleus and
nucleolus lose their central locations. On H & E stain, the Nissl substance becomes less
prominent. This loss of Nissl substance is referred to a chromatolysis.

Question:
The implication of the presence of corpora amylacea in astrocytic processes is

Answer Choices:
A Diabetes
B Stroke
C Cryptococcus
D Normal aging
E Hypertension

D Normal aging

Corpora amylacea are basophilic, concentrically laminated bodies found in the foot
processes of astrocytes. They are most commonly found in the brain around blood
vessels, under the pia and under the ependyma. They are thought to be the result of the
normal aging process.
Corpora amylacea are histologically very similar to cryptococcus and have identical
staining reactions with a number of special stains. It is very important not to mistake
them for cryptococcus.

Question:
Which of the following cells has a phagocytic function?

Answer Choices:
A Astrocytes
B Oligodendrocytes
C Microglia
D Schwann cells
E Ependymal cells

Question:
The cells that regulate the metabolic environment of neurons in the central nervous system (CNS) are

Answer Choices:
A Astrocytes
B Oligodendrocytes
C Microglia
D Schwann cells
E Ependymal cells

A Astrocytes

Astrocytes are large glial cells that may have any of a number of shapes depending on
where in the CNS they are located. They may be protoplasmic (usually in the gray
matter), fibrillary (usually in the white matter) or pilocytic (periventricular area, spinal
cord and cerebellum). Bergmann astrocytes are associated with the Purkinje neurons of
the cerebellar cortex.
Some processes of astrocytes line the cell membrane of neurons and others extend to
the surface of CNS capillaries. It is thought that astrocytes mediate the exchange of
metabolites and waste products between the neurons and the capillaries.
Astrocytes also surround CNS axons at the nodes of Ranvier.

Question:
The cells responsible for myelination of the central nervous system (CNS) are

Answer Choices:
A Astrocytes
B Oligodendrocytes
C Microglia
D Schwann cells
E Ependymal cells

B Oligodendrocytes

In the CNS, the oligodendrocytes appear to have the function of both the Schwann cell
and the satellite cells of the peripheral nervous system (PNS). In the white matter,
oligodendrocytes send out many processes, each of which winds around a part of an
axon, forming a myelin sheath. If a histological section is cut along the right angle, the
oligodendrocytes can be seen forming rows of nuclei between fascicles of axons.
In gray matter, oligodendroglial cells can be seen as nuclei arranged around the
neuron's cell body. In some preparations, the cytoplasm of the oligodendroglial cells
swells and becomes vacuolated, giving the cells a characteristic "fried egg" appearance.
The tendency of oligodendrocytes to form satellites around neuron cell bodies and to
appear as "fried eggs" is frequently retained when the cells becomes neoplastic and so
can help identify oligodendrogliomas.

Question:
Schmidt-Lanterman clefts are
Answer Choices:
A The Schwann cell grooves in which axons lie
B Spirals of cytoplasm running through the layers of myelin sheaths
C Characteristic notches in Schwann cell nuclei
D The gaps between Schwann cells where the nodes of Ranvier are formed
E Indentations of the intraperiod lines of the myelin sheaths

B Spirals of cytoplasm running through the layers of myelin sheaths

Schmidt-Lanterman clefts are lines of Schwann cell cytoplasm which spiral through the
layers of the myelin sheath. They appear as "blebs" between the major dense lines.
Schmidt-Lanterman clefts allow the cytoplasm between the innermost layers of myelin
to maintain metabolic contact with the cytoplasm of the outer part of the Schwann cell.
Question:
The presynaptic release of neurotransmitter is dependent on

Answer Choices:
A Calcium
B Potassium
C Sodium
D Chloride
E Magnesium
A Calcium

When a nerve impulse reaches the presynaptic membrane, the membrane is


depolarized, allowing extracellular calcium to enter the cell. This calcium causes the
presynaptic vesicles to move to the cell membrane, where they release their contents
through exocytosis.
uestion:
In addition to synaptic vesicles, the presynaptic portion of the axon contains an abundance of

Answer Choices:
A Rough endoplasmic reticulum
B Smooth endoplasmic reticulum
C Mitochondria
D Lysosomes
E Golgi vesicles

C Mitochondria
On electron microscopy, the presynaptic portion of axons contains many presynaptic
vesicles and mitochondria. The mitochondria generate the energy necessary for the
continuing production of the vesicles.
Both the presynaptic and postsynaptic cells have a layer of electron dense material on
the cytoplasmic surface of their cell membranes in the region of the synapse. These
layers are referred to either as the presynaptic density or the post synaptic density.
Question: BLOCK-1
Which of the following is true of neuropeptide neurotransmitters?

Answer Choices:
A They are found in micromolar concentrations in the brain
B They are usually the only transmitter released by a given nerve terminal
C There are usually reuptake mechanisms present presynaptically
D They are synthesized by rough endoplasmic reticulum (rER)
E They typically have short reaction time
D They are synthesized by rough endoplasmic reticulum (rER)

Neuropeptides are usually found in picomolar concentrations in the brain. They are
usually co-released with at least one other neurotransmitter. There are no reuptake
mechanisms present and they typically have long reaction times and serve as
enhancers to amplify other signals.

Question:
A Pacinian Corpuscle responds to which type of stimulus?

Answer Choices:
A Pain
B Light
C Sound
D Smell
E Pressure
E Pressure

Pacinian Corpuscles are sensory receptors in connective tissue that responds to touch,
pressure, and vibration.

Question:
In the mammalian nerve, what is the conduction velocity of a postganglionic sympathetic nerve fiber?
Answer Choices:
A 0.1 to 0.5 ms
B 0.7 to 2.3 ms
C 3 to 15 ms
D 12 to 30 ms
E 30 to 70 ms

B 0.7 to 2.3 ms

Postganglionic sympathetic nerve fibers are of the C type and have relatively slow
conduction velocities as compared with other peripheral nerves. The conduction velocity
of type B preganglionic autonomic fibers is 3 to 15 ms. The range of conduction velocity
for type A fibers is described as 12 to 30 ms and 30 to 70 ms.

Question:
What type of synaptic contact has the greatest effect on terminal release of neurotransmitter?

Answer Choices:
A Axo-dendritic
B Axo-somatic
C Axo-axonic
D Dendro-dendritic
E Dendro-somatic
C Axo-axonic

Axo-axonic synapses have the greatest effect due to their close proximity to the point of
release (the nerve terminal). Axo-somatic has less of an effect, and axo-dendritic has
even less of an effect due to increasing distances from the initial segment. Dendro-
dendritic synapses typically serve to coordinate the activity of a population of neurons.
Question:
When a nerve terminal is depolarized, the resulting influx of Ca++ causes

Answer Choices:
A Dephosphorylation of synapsin
B Phosphorylation of synapsin
C Depolarization of the nerve terminal
D Hyperpolarization of the nerve terminal
E The binding of vesicles containing neurotransmitter to neurofilaments

B Phosphorylation of synapsin
Calcium influx induces phosphorylation of synapsin which decreases the binding affinity
of vesicles to neurofilaments allowing them to diffuse to the cell membrane where they
are exuded into the terminal cleft by exocytosis.

Question: CLASS QUIZ


As an action potential passes any point along an axon, the first event that occurs at that point is

Answer Choices:
A Calcium ions move into the neuron
B Calcium ions move out of the neuron
C Potassium ions move out of the neuron
D Sodium ions move out of the neuron
E Sodium ions move into the neuron
E Sodium ions move into the neuron

As an action potential is propagated down an axon the cell membrane of the neuron
becomes more permeable to sodium which rushes into the neuron down its
concentration gradient depolarizing the cell. Calcium is involved in the release of
neurotransmitter at the terminal. Potassium is involved in the repolarization of the
neuron.

Question:
The myelin sheath of neurons in the central nervous system is formed from what type of cell?

Answer Choices:
A Schwann cells
B Astrocytes
C Oligodendrocytes
D Microglia
E Purkinje cells

C Oligodendrocytes

The best answer is Oligodendrocytes. Schwann cells form the myelin sheath in the
peripheral not the central nervous system. Astrocytes are star shaped glia that function
as support cells for neurons in the CNS. Microglia look like macrophages and are often
found near blood vessels and migrate to areas of injury. Purkinje cells are neurons, not
glial cells.
Question: CLASS QUIZ
In the attached photomicroscopic image, formation of the substance located at the tip of the pointer as
found in the central nervous system is by what cell type?

Answer Choices:
A Astrocyte
B Microglia
C Schwann cell
D Satellite cell
E Oligodendrocyte

Image(s) / Chart(s):
Click image to view full size. Click open image to close. Click and hold open image to move.

E Oligodendrocyte

The tip of the pointer is located on the myelin sheath surrounding an axon as may be
located in the spinal cord. The cell responsible for forming the myelin sheath in the
central nervous system is the oligodendrocyte. The oligodendrocyte also is capable of
forming segments of myelin sheaths surrounding one, or many different axons by
multiple processes from the glial cell. In contrast, myelination in the peripheral nervous
system is from the Schwann cell, and one Schwann cell is responsible for forming the
myelin sheath on one segment of one axon of a nerve fiber.

Question: BLOCK-1
The predominant chemical constituent of the substance located in the spaces at the tip of the arrow
would be best described as

Answer Choices:
A Lipoproteins
B Carbohydrates
C Hyaluronic acid
D Glycogen
E Nissl substance
Image(s) / Chart(s):
Click image to view full size. Click open image to close. Click and hold open image to move.

A Lipoproteins

The arrow is pointing to the myelin sheath of this nerve fiber cut in cross section. The
axon of the nerve fiber can be visualized within the center of the sheath. Since the
myelin sheath is formed by the wrapping of the Schwann cell plasma membrane
repeatedly around the axon cylinder, the major chemical constituent of the myelin
sheath is lipoprotein. The Schwann cell nucleus cannot be identified; however, close
observation may reveal small connective tissue filaments coursing throughout the nerve
fiber bundle constituting the endoneurium.

Question:
Identify the structure at the tip of the arrows in the attached photomicrographic image

Answer Choices:
A Schwann cell nucleus
B Axon
C Axon hillock
D Myelin sheath
E Perikaryon

Image(s) / Chart(s):
Click image to view full size. Click open image to close. Click and hold open image to move.

D Myelin sheath
The arrow is pointing to the myelin sheath of this nerve fiber cut in cross section. The
axon of the nerve fiber can be visualized within the center of the sheath. Schwann cell
nuclei cannot be identified. However, close observation may reveal small connective
tissue filaments coursing throughout the nerve fiber bundle constituting the
endoneurium.

Question: CLASS QUIZ


The arrows depict what structure on the attached photomicrographic image?

Answer Choices:
A Perimysium
B Perineurium
C Epimysium
D Epineurium
E Endomysium

Image(s) / Chart(s):
Click image to view full size. Click open image to close. Click and hold open image to move.

B Perineurium

This image represents a cross-section through a peripheral nerve fiber bundle. The
arrows indicate a connective tissue sheath, or perineurium, surrounding each group or
bundle of nerve fibers. Close examination of the image will reveal a central axon fiber,
surrounded by a myelin sheath. The entire nerve fiber bundle is surrounded by a
connective tissue layer, the epineurium. In comparison, the perimysium is a similar
connective tissue sheath, except that it surrounds a bundle of skeletal muscle fibers.

Question: BLOCK EXAMS


The structure observed in the attached photomicroscopic image is best described as
Answer Choices:
A Sensory ganglion
B Skeletal muscle
C Thyroid gland
D Nerve fiber bundle
E Mammary gland

Image(s) / Chart(s):
Click image to view full size. Click open image to close. Click and hold open image to move.

D Nerve fiber bundle

This image represents a cross-section through a peripheral nerve fiber bundle. Note
that each group or bundle of nerve fibers is surrounded by perineurium, a connective
tissue sheath, which gives rise to fine connective tissue fibers that invest each nerve
fiber as the endoneurium. Close examination of the image will reveal a central axon
fiber, surrounded by a myelin sheath. The entire nerve fiber bundle is surrounded by a
connective tissue layer, the epineurium.

Question:
A sensory receptor which will react to stimuli from the external environment, such as temperature,
would be classified as which one of the following afferent receptor types?

Answer Choices:
A Interoceptor
B Exteroceptor
C Proprioceptor
B Exteroceptor

Afferent (sensory) receptors have a variety of different structural designs, each


somewhat specific for the function it subserves. These specialized structures are
located at the distal ends of peripheral sensory nerves and may be classified according
to their anatomical position. Exteroceptors are located in the dermis of the skin, and
among special sense organs allowing them to react to stimuli from the external
environment, such as touch, temperature alterations, olfaction, visual stimuli and sound
waves.

Question:
An example of an neuron that is characterized as bipolar is which one of the following neurons?

Answer Choices:
A Betz cell of the cerebral cortex
B Dorsal root ganglion neuron
C Purkinje cell of the cerebellar cortex
D Spiral ganglion cell
E Granule cell in the cerebellar cortex

D Spiral ganglion cell

Neurons may be classified according to the number of processes observed for that
neuron type. The cell body of sensory neurons, such as the neurons of the dorsal root
ganglion, have one process that extends to the periphery, and one branch that extends
into the spinal cord. These cells probably arise as bipolar cells whose processes
migrate from where they arise on the cell body to a common (or closely associated)
origin, wherein the cell processes diverge following their origin from the cell. In this way,
they are classified as pseudounipolar, but function in as a unipolar unit. All of the other
cells from the selection are multipolar, except for the spiral ganglion cells of cranial
nerve VIII, which are bipolar.

Question:
An example of an neuron that is characterized as unipolar (or pseudounipolar) is which one of the
following neurons

Answer Choices:
A Betz cell of the cerebral cortex
B Dorsal root ganglion neuron
C Purkinje cell of the cerebellar cortex
D Amacrine cell of the retina
E Granule cell in the cerebellar cortex
B Dorsal root ganglion neuron

Neurons may be classified according to the number of processes observed for that
neuron type. The cell body of sensory neurons, such as the neurons of the dorsal root
ganglion, have one process that extends to the periphery, and one branch that extends
into the spinal cord. These cells probably arise as bipolar cells whose processes
migrate from where they arise on the cell body to a common (or closely associated)
origin, wherein the cell processes diverge following their origin from the cell. In this way,
they are classified as pseudounipolar, but function in as a unipolar unit. All of the other
cells from the selection are multipolar.

Question:
Neurons located in the dorsal root ganglia are derived from which source?

Answer Choices:
A Neural crest cells
B Neural ectoderm
C Mesoderm
D Ependymal cells
E Somatic ectoderm

A Neural crest cells

Development of ganglion cells in the peripheral nervous system occurs from precursor
cells arising from the neural crest cells. These cells migrate from the neural crest to their
location along the dorsal horn of the developing spinal cord, their future ganglionic site.
The processes of the neurons develop both centrally, to make connections within the
spinal cord, and peripherally, to respective sensory areas.

Question:
Which one of the following cells in the central nervous system forms expanded cytoplasmic processes
that may cover large areas of the surface of a blood vessel or the neurolemma surface, and may aid in
the regulation of the microenvironment of the neurons and provide a structural basis for the blood-brain
barrier?

Answer Choices:
A Oligodendrocyte
B Protoplasmic astrocyte
C Microglia
D Ependymal cell
E Satellite cell

B Protoplasmic astrocyte

The largest of the glial cells, the astrocytes, have elaborate processes that extend and
interdigitate among blood vessels and neurons. The ends of these cellular processes
have expanded feet that may cover either the outer surface of blood capillaries or
provide a covering for the neurolemma. In doing so, the protoplasmic astrocyte, being
more common in gray matter, and the fibrous astrocyte being more common in white
matter, may function in regulation of the microenvironment of the neurons, or form part
of the basis of the blood-brain barrier.
Question:
Which of the following cells would show a rapid increase in number in the cerebral cortex following a
localized ischemic insult?

Answer Choices:
A Fibrous astrocytes
B Microglia
C Protoplasmic astrocytes
D Oligodendroglia
E Ependymal cells

B Microglia

Ischemia ultimately results in cell death of the neurons in the location of the insult. In
response to the tissue damage, microglia proliferate and become phagocytic in the
damaged area. These cells, as part of the mononuclear phagocytic system may also
enter the CNS from the blood vascular system.

Question:
The presumed origin of microglia found in the central nervous system is from what source?

Answer Choices:
A Bone marrow
B Neural crest cells
C Thymus
D Neuroectoderm
E Lymph nodes

A Bone marrow
Microglia are present in the adult CNS in small numbers, except in the presence of
disease, or injury. These cells function as phagocytotic cells and are considered as part
of the mononuclear phagocytic system. As such, they originate in the bone marrow and
will enter the CNS from the peripheral vasculature. Neural crest cells putatively give rise
to the other members of the glia, the astrocytes and oligodendroglia. The nerve cells of
the CNS are formed from the neural ectoderm.
Question:
The major dense line, which is characteristic of the appearance of myelinated axons when seen with
an electron micrograph represents what cellular structure?

Answer Choices:
A Fused inner leaflets of the Schwann cell membrane
B Fused outer leaflets of the Schwann cell membrane
C Fused edges of a cytoplasmic groove that encloses the axon
D Fused inner and outer leaflets of the Schwann cell membrane
E Fused edges of the cytoplasmic groove that forms the outer surface of the
myelin sheath

A Fused inner leaflets of the Schwann cell membrane

When the Schwann cell or oligodendrocyte process begins to wrap around a segment of
an axon to produce the myelin sheath, the cytoplasm of the cellular process is extruded
from the concentric layers allowing the inner leaflets of apposing inner surfaces of the
cell process to fuse. This fusion of the inner leaflets become electron dense in prepared
material and appear as the major dense lines of the myelin sheath. The fused outer
leaflets of the Schwann cell membrane, or membrane of the oligodendrocyte process,
are less electron opaque, and become seen as the intraperiod line.

Question:
When viewing a transmission electron photomicrograph of the cut cross-sectional surface of a
myelinated peripheral nerve, an evagination of the plasma membrane extending from the inner surface
of the Schwann cell to the myelin layers is referred to as the

Answer Choices:
A Major dense line
B Outer mesaxon
C Inner mesaxon
D Intraperiod line
E Node of Ranvier
C Inner mesaxon
The inner mesaxon is represented by the apposition of the internal opposing Schwann
cell processes, and is seen as an evagination of the Schwann cell plasma membrane
extending from the inner surface of the Schwann cell to the myelin layers. In contrast,
the outer mesaxon represents the outer folds of the Schwann cell following myelination
were they meet each other such that the plasma membrane of the Schwann cell
invaginates and extends from the outer surface to the myelin layers. The major dense
line is formed when the cytoplasm in the wrapping processes of the Schwann cell is
extruded from between the two apposing plasma membranes allowing the inner leaflet
to fuse with the inner leaflet of the opposing side of the cell process to form an electron
dense line. The fusion of the apposing outer leaflets form the intraperiod line, which is
much less electron dense.
Question:
An example of a Golgi Type I neuron would be a

Answer Choices:
A Granule cell of the cerebellar cortex
B Molecular cell of the cerebellar cortex
C Interneuron of the dorsal horn of the spinal cord
D Astrocyte of the cerebral cortex
E Motor neuron of the ventral horn of the spinal cord

E Motor neuron of the ventral horn of the spinal cord

Classical description of neurons dictate there is only one axon for each neuron, e.g. the
cellular process that transmits an impulse away from the cell body. The axon from any
one type of neuron may be classified as either long or short. In the event of a "long"
axon, it would be regarded as a Golgi Type I neuron, and in the case of a "short" axon,
the neuron would be regarded as a Golgi Type II. Typically, axons that originate from
neurons located in motor nuclei of the central nervous system are regarded as having
long axons that may extend several feet (as in the case of a corticospinal tract axon),
and hence are Golgi Type I neurons. This would also be true in the case of the motor
neuron whose cell body is located in the ventral horn of the spinal cord and has an axon
extending as much as several feet to its effector organ, skeletal muscle.

Question:
The cell that provides a "covering" between myelinated segments at the node of Ranvier in the central
nervous system is the

Answer Choices:
A Oligodendrocyte
B Microglia
C Schwann cell
D Astrocyte
E Satellite cell

D Astrocyte

While the myelin sheath segments are formed by the oligodendrocyte processes in the
CNS, the astrocytes will provide the covering at the nodes of Ranvier, or at the
synapses in the CNS. Astrocytes have elaborate processes that extend among
neurons, and will form "end feet" capable of covering the bare areas not protected by
myelin. These glial cell processes may help in the manipulation of the microenvironment
of the neuron processes, such as movement of metabolites or regulation of local ionic
concentrations.

Question:
The cells located in the central nervous system (CNS) responsible for myelin formation are the

Answer Choices:
A Astrocytes
B Schwann cells
C Microglia
D Satellite cells
E Oligodendrocytes

E Oligodendrocytes

Oligodendrocytes are the glial cells present in the CNS that are the myelin-forming cells.
The myelin sheath is formed by closely-packed, concentric layers of the oligodendrocyte
plasma membrane. Each oligodendrocyte will provide several flattened cytoplasmic
processes, and each process will wraps itself around a single portion of an axon to form
a myelin sheath segment. Oligodendrocytes aligned in rows will provide adjacent myelin
segments along an axon. One oligodendrocyte, therefore, may form multiple myelin
segments on one axon, or on several nearby axons.

Question:
The faintly staining nuclei identified by arrows in the attached photomicroscopic image are the nuclei of
satellite cells surrounding the perikaryon of the neuron located in the dorsal root ganglion. The satellite
cells are derived from what embryological source?
Answer Choices:
A Endoderm
B Mesoderm
C Neuroectoderm
D Neural crest cells
E Ectoderm

Image(s) / Chart(s):
Click image to view full size. Click open image to close. Click and hold open image to move.

D Neural crest cells

This photomicroscopic image depicts the appearance of a cross section through a


dorsal root ganglion. The neuronal cell bodies present are pseudounipolar neurons
embedded in a connective tissue array surrounding the collection of cell bodies, or
ganglion. Note the large soma with a centrally located nucleus. The faintly stained
nuclei surrounding the perikaryon are the nuclei of satellite cells, supportive cells of the
peripheral nervous system associated with the nerve cell bodies. Satellite cells, like
Schwann cells, arise from neural crest cells during development. The satellite cells are
thought to help establish and maintain the local microenvironment around the neuronal
cell body in the ganglion. Unlike the Schwann cell, the satellite cell does not synthesize
myelin.
Question: CLASS QUIZ
The neurotransmitter found at a neuromuscular junction is

Answer Choices:
A Norepinephrine
B Epinephrine
C Glutamate
D γ-aminobutyric acid (GABA)
E Acetylcholine

E Acetylcholine
Acetylcholine is the neurotransmitter released from the terminal bouton of the axon at
the neuromuscular junction. When released into the synaptic cleft, acetylcholine diffuses
across the cleft to bind to receptors found on the surface of the muscle cell located at
this junction. The presence of the acetylcholine at the receptor site will elicit
depolarization of the muscle membrane and a concomitant muscle contraction.

Question: CLASS QUIZ


The cytoplasm of nerve cell bodies typically stains intensely basophilic with aniline dyes. However, a
conical projection of the perikaryon that leads into one process typically does not readily stain and is
referred to as

Answer Choices:
A Axon hillock
B Synaptic bouton
C Synaptic cleft
D Dendritic spine
E Initial axonal segment

A Axon hillock

The axon originates from a conical projection of the perikaryon, which is referred to as
the axon hillock. This area of the perikaryon is typically devoid of Nissl substance
(stacks of granular endoplasmic reticulum) and does not stain as intensely as the
remaining soma. The dendritic processes are typically larger in diameter and generally
contain Nissl substance, at least at their origin. The initial axonal segment is a region of
the axon between the apex of the axon hillock and the initial portion of the myelin
sheath. While it too lacks the intense staining of the cell body, it is regarded as part of
the axon and extends from the axon hillock; therefore, the axon hillock is the more
correct answer.

Question:
The faintly staining nuclei identified by arrows surrounding the larger cell in this photomicroscopic
image are best described as

Answer Choices:
A Satellite cells
B Schwann cells
C Primordial oocytes
D Chondroblasts
E Oligodendrocytes
Image(s) / Chart(s):
Click image to view full size. Click open image to close. Click and hold open image to move.

A Satellite cells

This photomicroscopic image depicts the appearance of a cross section through a


dorsal root ganglion. The neuronal cell bodies present are pseudounipolar neurons
embedded in a connective tissue array surrounding the collection of cell bodies, called a
ganglion. Note the large soma with a centrally located nucleus. The faintly stained
nuclei surrounding the perikaryon are the nuclei of satellite cells, supportive cells of the
peripheral nervous system associated with the nerve cell bodies. Satellite cells, like
Schwann cells, arise from neural crest cells during development. The satellite cells are
thought to help establish and maintain the local microenvironment around the neuronal
cell body in the ganglion. Unlike the Schwann cell, the satellite cell does not synthesize
myelin.
Question:
The histological structure depicted in the attached photomicroscopic image is best described as:

Answer Choices:
A Ovary
B Dorsal root ganglion
C Autonomic ganglion
D Cerebellar Purkinje cell
E Fibrocartilage

Image(s) / Chart(s):
Click image to view full size. Click open image to close. Click and hold open image to move.
B Dorsal root ganglion

This photomicroscopic image depicts the appearance of a cross section through a


dorsal root ganglion. The neuronal cell bodies present are pseudounipolar neurons
embedded in a connective tissue array surrounding the collection of cell bodies, or
ganglion. Note the large soma with a centrally located nucleus. The faintly stained
nuclei surrounding the perikaryon are the nuclei of satellite cells, supportive cells of the
peripheral nervous system associated with nerve cell bodies.
Question: BLOCK EXAMS
The dense, granular basophilic staining typical in the cell body depicted in this photomicroscopic image
is best described as

Answer Choices:
A Granular endoplasmic reticulum
B Mitochondria
C Golgi bodies
D Nuclei
E Neurofibrils

Image(s) / Chart(s):
Click image to view full size. Click open image to close. Click and hold open image to move.
A Granular endoplasmic reticulum

The cytoplasm of the perikaryon of neurons typically are rich in organelles. Staining with
basic aniline dyes demonstrates basophilic masses within the cytoplasm and dendrites,
but absent in the axon and axon hillock. This basophilic staining substance is called
Nissl substance and consists of densely packed cisternae of granular endoplasmic
reticulum and polysomes. The nucleus of a neuron soma is present in this image, as is
the dominant nucleolus. Neurofibrils form an anastomosing network around the nucleus
and extend into the cell processes, but must be selectively stained to visualize with light
microscopy.

Question:
A location where a nerve cell body having this appearance as seen in the attached photomicroscopic
image may be found in the

Answer Choices:
A Dorsal root ganglion
B Spiral ganglion of the cochlea
C Cerebellar Purkinje layer
D Cerebellar granule layer
E Cerebral cortex, layer V

Image(s) / Chart(s):
Click image to view full size. Click open image to close. Click and hold open image to move.
E Cerebral cortex, layer V

The image depicts a large multipolar nerve cell body. This pyramidal-shaped cell body
is typical of the so-named pyramidal cells located in the cerebral cortex, predominantly
in layers II, III, and V. These cells usually possess a large apical dendrite generally
oriented toward the surface of the cerebral cortex. The singular axon (not identified in
the image) generally are long and leave the cerebral cortex to extend to other areas of
the brain or spinal cord. The dorsal root ganglion possess neurons of the
pseudounipolar type, whereas the spiral ganglion of the cochlea possess bipolar
neurons. Purkinje cells found in the cerebellar cortex are multipolar, but have a typically
flask-shape body and are arranged in discrete rows with dominant fan-shaped dendrites
extending into the cerebellar molecular layer. Cerebellar granule cells are small,
densely packed neurons with short stubby dendrites and an axon which enters the
molecular layer of the cerebellum.

Question:
The cells whose nuclei are depicted at the tip of the arrowheads in the attached photomicrograph are
what kind of cells?

Answer Choices:
A Smooth muscle cells
B Primordial ova
C Large lymphocytes
D Autonomic nerve cell bodies
E Multipolar nerve cell bodies

Image(s) / Chart(s):
Click image to view full size. Click open image to close. Click and hold open image to move.
D Autonomic nerve cell bodies

The cells depicted in the photomicrographic image at the tip of the arrowheads typify the
appearance of small clusters of autonomic nerve cell bodies as found between the inner
circular and outer longitudinal muscle layers of the muscularis externus of the intestine.
These cell bodies have a distinct nucleolus located in the center of the nucleus and give
the appearance of an "owl's eye". Nerve cell bodies in this location are part of the
parasympathetic autonomic nervous system and provide local postganglionic
innervation. The nuclei located between the two muscle layers in the intestinal region,
as depicted here, are called the myenteric or Auerbach's plexus.

SECTION- CARDIOVASCULAR SYSTEM

Question:
The smooth muscle cells present in muscular arteries are found in which one of the layers of the
vessel?

Answer Choices:
A Tunica intima
B Tunica propria
C Tunica media
D Tunica adventitia
E Tunica lamina
C Tunica media

The primary component of the tunica media of arteries are circumferentially arranged
layers of smooth muscle cells. Depending on the vessel size, the number of layers of
smooth muscle is varied. In arteries, this layer is relatively thick and extends from a
layer of elastin called the internal elastic membrane that separates the tunica intima
from the tunica media, to a second continuous layer of elastin called the external elastic
membrane. The smooth muscle cells located within the tunica media secrete all of the
extracellular components of that layer, which may include elastin, reticular fibers, or
proteoglycans.
Question:
The tunica adventitia is composed primarily of what structures?

Answer Choices:
A Smooth muscle
B Endothelial cells
C Elastic fibers
D Collagenous fibers
E Cardiac muscle

D Collagenous fibers

The tunica adventitia resides as the "outermost" wall of the vein or artery, and as such
is composed mostly of collagenous fibers. This connective tissue eventually becomes
continuous with the surrounding loose connective tissue surrounding the vessel.
Because of its remote location to the vessel lumen, the tunica adventitia of larger
arteries or veins may also have vessels of their own, the vasa vasorum.
Question: CLASS QUIZ
Conservation of body temperature when digits are exposed to cold is accomplished by what structural
mechanism?

Answer Choices:
A Activation of arterio-venous shunts bypassing capillary beds in the fingers or
toes
B Dilation of capillary networks in the fingers or toes
C Increasing blood flow through the capillary networks in the fingers or toes
D Shunting of blood flow towards capillaries via metarterioles
E Activation of arterio-venous shunts providing increased capillary blood flow in
the fingers or toes

A Activation of arterio-venous shunts bypassing capillary beds in the fingers or toes

Blood flow into a capillary bed is controlled by precapillary sphincters. Frequently


located in the peripheral vessels is a shunt in which the arterioles are connected directly
to a venule. Constriction of the precapillary sphincter will result in the shunting of blood
flow to the venous system, bypassing the capillary network and, hence, conserving
temperature since the blood will tend to not pass through capillary networks near the
body surfaces, especially in the digits of the hands or feet.
Question:
Molecules travel across vascular endothelium by

Answer Choices:
A Passive diffusion between endothelial cells
B Pinocytosis
C Passive diffusion across endothelial cells
D Active transport through channels created by membrane proteins
E Through fenestrations in the endothelium
B Pinocytosis

Vascular endothelial cells are linked by tight junctions and gap junctions which prevent
passive diffusion of nutrients from the intimal surface into the vessel wall. Nutrients are
carried across endothelial cells by pinocytosis. Hormonal factors that regulate the tone
of blood vessels also travel this way.
Question:
The tunica adventitia of large, elastic vessels

Answer Choices:
A Is at least as thick as the tunica media
B Has a significant smooth muscle component
C Lacks fibroblasts
D Primarily prevents the vessel from becoming over-expanded
E Does not have any physical connection to the surrounding non-vascular
connective tissue

D Primarily prevents the vessel from becoming over-expanded

The tunica adventitia of elastic vessels is thin, less than half the thickness of the tunica
media. It is composed of collagen and elastic fibers (the elastic fibers do not form
lamellae), fibroblasts and macrophages. The vessels of the vasa vasorum and the
nerves of the nervi vascularis run through the adventitia.
The primary function of the tunica adventitia is to prevent the vessel from becoming
over-expanded. The connective tissue of the adventitia blends gradually with the
surrounding non-vascular connective tissue.

Question:
Cerebral arteries are different from other arteries in that

Answer Choices:
A With a diameter of about 0.04-0.4mm, they are the smallest muscular arteries
B They resemble veins in having a thin wall but a prominent internal elastic
lamina
C They are essentially endothelial lined tubules, with a basal lamina, usually
about 8 μm in diameter, although some may be much larger (12 μm or smaller (5
μm)
D They have no smooth muscle in their walls, and as a consequence, cannot
contract
E Their endothelial cells are deficient in basal lamina, and as a consequence,
may rest directly on connective tissue

B They resemble veins in having a thin wall but a prominent internal elastic lamina

Cerebral arteries resemble veins in having a thin wall but a prominent internal elastic
lamina. Arterioles are the smallest muscular arteries. Their diameters are about 0.04-0.4
mm. The capillaries are essentially endothelial lined tubules, with a basal lamina,
usually about 8 μm in diameter, although some may be much larger (12 μm or smaller
(5 μm). They have no smooth muscle in their walls, and as a consequence, cannot
contract. Sinusoids resemble capillaries but possess certain distinctive features. Since
the basal lamina may be deficient or absent, the endothelial cells may rest directly on
connective tissue
Question:
Elastin, which may be found as individual fiber bundles or lamellae within the tunica media of an artery,
are secreted by what cell?

Answer Choices:
A Fibroblast
B Endothelial cell
C Muscle cell
D Erythrocyte
E Macrophage

Move
Close
C Muscle cell

Within the tunica media of the muscular arteries and elastic arteries are smooth muscle cells
arranged in concentric layers. Besides having a contractile function responsible for maintaining
the tonus of the vessel, the muscle cells produce the extracellular components of the tunica
media, such as collagen fibers, elastin (in elastic fibers), and proteoglycans. Fibroblasts are not
found in the tunica media, of either muscular or elastic arteries.

Question:
The veins are classified as capacitance vessels, because they may contain how much of the following
percentage of the total blood volume of the body?

Answer Choices:
A 40
B 50
C 60
D 70
E 90

D 70

When considered as a functional unit, all the veins can be classified as capacitance
vessels because more than 70% of the total blood volume is in this portion of the
cardiovascular system at any one time.

Question: CLASS QUIZ


Regulation of blood flow to capillary beds occurs through which of the following structures?

Answer Choices:
A Elastic artery
B Arteriole
C Vein
D Venule
E Muscular artery

Move
Close
B Arteriole

As the size of the artery diminishes, the number of layers of smooth muscle in the tunica media
of the artery also decreases. Prior to its ramification into a capillary bed, the number of layers of
smooth muscle observed may be limited to only 1 or 2 layers. This defines the arteriole. There
may be a slight increase in the thickness of the muscle layer of the arteriole just prior to the
origin of the capillary bed, which is the precapillary sphincter. It is the arteriole that serves to
regulate flow of blood to the respective capillary bed.

Question:
Inhibition of angiogenesis is an important potential avenue of cancer chemotherapy because

Answer Choices:
A Tumors require a sufficient blood supply to grow
B Angiogenesis inhibitors directly block tumor cell proliferation
C Angiogenesis is required for metastasis
D Angiogenesis inhibitors block pericyte migration
E Stimulation of the immune system results from angiogenesis inhibition

A Tumors require a sufficient blood supply to grow

The blood vessel is lined by a single layer of endothelial cells, which regulates the
exchange of molecules between the bloodstream and the surrounding tissues. The
vessel wall of mature capillaries is composed of an endothelial cell lining, a basement
membrane, and a layer of cells (called pericytes) that partially surround the
endothelium.
Angiogenesis is stimulated by the release of angiogenic growth factors that bind to
receptors on the endothelial cells and initiate a series of events involving stimulation of
the cells to grow, secretion of proteases, and formation of new capillaries. The secreted
proteases result in digestion of the basement membrane surrounding the vessel. Cell
projections from the endothelial cell pass through the space created, and the newly
formed sprout grows towards the source of the stimulus. Among the growth factors that
have been reported to act as angiogenic factors are fibroblast growth factors,
angiogenin, TNF-alpha, TGF-beta, and platelet derived growth factor. Angiogenesis is
also important in tumor growth. In order to grow larger, the tumor must have a blood
supply. A new approach to chemotherapy is to block angiogenesis and thereby "starve"
the tumor of nutrients derived from the bloodstream. Hematopoiesis is the generation of
blood cells that occurs mostly in the bone marrow.

Question:
Early changes of the structure of arterial walls leading to a condition known as atherosclerosis involves
accumulation of lipids and progressive thickening. The cells thought to be involved in this early process
are the

Answer Choices:
A Endothelial cells
B Myointimal cells
C Lymphocytes
D Macrophages
E Pericytes

B Myointimal cells

In elastic arteries and large muscular arteries, the tunica intima consists of a single layer
of flattened endothelial cells. Deep to the endothelium is a thin layer of collagenous
connective tissue, containing a few elastic fibers, scattered fibroblasts, and myointimal
cells, which have ultrastructural characteristics of smooth muscle cells. This layer is the
subendothelium and the myointimal cells will accumulate lipid and thicken the tunica
intima. This thickening may be one of the early structural changes which lead to
atherosclerosis.

Question:
Weibel-Palade bodies are

Answer Choices:
A Electron dense organelles in arterial endothelial cells
B Frayed arterial elastic fibers characteristic of aging
C Cholesterol deposits in the arterial intima
D Calcifications in the tunica media
E Ruptured cholesterol clefts

A Electron dense organelles in arterial endothelial cells

Weibel-Palade bodies are elongated, electron dense organelles found in vascular


endothelial cells. These bodies are storage sites for coagulation Factor VIII, which is
made by the endothelial cells and released into the bloodstream.

Question:
What are the two hallmark features of atherosclerosis?

Answer Choices:
A Intimal fibrosis and calcification
B Intimal fibrosis and smooth muscle proliferation
C Medial fibrosis and smooth muscle proliferation
D Lipid deposition and intimal fibrosis
E Lipid deposition and smooth muscle proliferation
E Lipid deposition and smooth muscle proliferation

The two hallmark features of atherosclerosis are lipid deposition and smooth muscle
proliferation. Other features include eccentric intimal fibrosis and the presence of lipid
laden macrophages.
Early in the process, the lipid deposited in the tunica intima becomes surrounded by a
fibrous capsule which, in later stages, breaks through the endothelial layer. The capsule
may calcify or ulcerate. The sluggish blood flow around the atherosclerotic plaque may
cause blood to clot, causing clinical cardiovascular disease.

Question:
External elastic lamina, found separating the tunica media from the outer tunica adventitia, is present in
which one of the following types of blood vessels?

Answer Choices:
A Arterioles
B Venules
C Elastic arteries
D Small muscular arteries
E Fenestrated capillaries

C Elastic arteries

In both large muscular arteries (e.g., femoral or brachial arteries) and the elastic arteries
(e.g., aorta and its large branches) an external elastic lamina (relatively thinner than the
internal elastic lamina) is often found separating the tunica media from the outer tunica
adventitia. Smaller muscular arteries, arterioles and fenestrated capillaries, however,
lack external elastic lamina.

Question:
Which one of the following statements is applicable to the epicardium?

Answer Choices:
A It is homologous and in structure comparable to the intima of the arteries,
especially the great vessels like aorta and pulmonary vessels
B It forms the fibrous skeleton of the heart
C Its principal components are the septum membranaceum, the trigona fibrosa,
and the annuli fibrosi
D The adipose tissue that generally surrounds the heart accumulates in this layer
E Certain regions of this layer contain nodules of fibrous cartilage

D The adipose tissue that generally surrounds the heart accumulates in this layer

The epicardium is the serous covering of the heart, forming the visceral layer of the
pericardium. It is covered externally by simple squamous epithelium (= mesothelium)
supported by a thin layer of connective tissue. A subepicardial layer of loose connective
tissue contains veins, nerves, and nerve ganglia. The adipose tissue that generally
surrounds the heart accumulates in this layer.
The fibrous skeleton of the heart is formed by dense connective tissue, and its principal
components are the septum membranaceum, the trigona fibrosa, and the annuli fibrosi.
Certain regions of this skeleton contain nodules of fibrous cartilage.
A 500 mL/min

The Fick equation for calculating cardiac output uses the relationship of oxygen
consumption by an individual divided by the difference in oxygen concentration of
arterial blood versus mixed venous blood. The best sample of mixed venous blood is
from the pulmonary artery.
Fick's principle has frequently been used to measure cardiac output using oxygen and
the pulmonary circulation. vO2 (maximal oxygen uptake) is the maximum capacity of an
individual's body to transport and use oxygen during incremental exercise, which
reflects the physical fitness of the individual.
The rate of entry of oxygen into pulmonary circulation by the blood-borne route is equal
to cardiac output (CO) multiplied by the (systemic) mixed venous oxygen content
([Venous O2]).
Fick's equation becomes CO = O2 Uptake / ([Arterial O2] - [Venous O2])
In this case:
CO = v02 / (arterial PO2 - pulmonary artery PO2)
= 300 / (20-14)
= 300/6 mL/min
= 500 mL/min or 5 L/min

Question:
A 57-year-old male suffering from chronic alcoholic cirrhosis of the liver has recently observed pain and
difficulty while passing stool. Lately he has also had some bleeding during defecation. On a recent
follow up visit, the physician detects hemorrhoids in the rectum during a rectal examination. He opines
that this is due to the involvement of the portal vein within the liver. Which of the following pairs of veins
unite to form the portal vein?

Answer Choices:
A Hepatic and inferior mesenteric
B Superior and inferior mesenteric
C Splenic and inferior mesenteric
D Superior mesenteric and splenic
E Hepatic and superior mesenteric

D Superior mesenteric and splenic

The portal vein is formed between the head and neck of the pancreas by the union of
the splenic (which drains organs supplied by the celiac artery) and the superior
mesenteric veins. It ascends through the hepatoduodenal ligament to the porta hepatis
and divides into the right and left portal veins. The right portal vein enters the right lobe
of the liver, and the left passes transversely through the porta hepatis to supply the
caudate, quadrate, and left lobes of the liver. There are no functioning valves in the
portal system.
When the portal vein is slowly obstructed because of the liver or from other causes, the
portal blood may enter the inferior vena cava by way of certain anastomotic veins, which
then become dilated and varicose. They may also burst. The portacaval venous
anastomoses are as follows:
At the upper end of the gastrointestinal tract: the esophageal branches of the left gastric
vein anastomose with esophageal branches of the azygos veins in the thorax (when
varicose they are called esophageal varices).
At the lower end of the gastrointestinal tract: the superior rectal vein anastomoses with
the middle and inferior rectal veins and, and most importantly, with the pelvic venous
plexuses (when varicose, the rectal or hemorrhoidal veins are called hemorrhoids).
Fine paraumbilical veins run with the round ligament of the liver from the left portal vein
to the umbilicus, where they anastomose with the superficial and deep epigastric veins.
Twigs of the colic and splenic veins anastomose in the extraperitoneal fat with twigs of
the renal vein and with veins of the body wall. Included may be twigs from the bare area
of the liver.
Since the anastomotic veins seldom possess valves, they can conduct blood equally
readily in either direction, depending on whether the obstruction is in the portal vein or in
the inferior vena cava.
The hepatic veins carry blood from the gastrointestinal canal to the inferior vena cava
after circulating in the liver. The inferior mesenteric vein is the continuation of the
superior rectal vein (superior hemorrhoidal vein). It ends behind the neck of the
pancreas by joining the angle of union between the superior mesenteric and splenic
veins as they form the portal vein.

Question:
A 75-year-old woman with a history of TIA's presents to your office. Her last episode included vision
changes where there appeared to be a curtain covering the top half of her eye. The ophthalmologist
has ruled out eye problems so a carotid sonogram with Doppler is performed. Increased resolution of
sonographic equipment is allowing visualization of anatomy that has not been seen before. This can
lead to confusion between normal and abnormal processes. Refer to the image. What are the arrows
indicating in the view labeled RT ECA in the image?

Answer Choices:
A Normal intimal reflection
B Intimal thickening
C Arterial adventitia
D Arterial media
E Fibrofatty plaque
F Thrombus
G Atherosclerosis
H Arteriosclerosis deposit

Image(s) / Chart(s):
Click image to view full size. Click open image to close. Click and hold open image to move.
A Normal intimal reflection

The intima and media of the large vessels can be visualized when optimal sonographic
conditions exist. This should not be confused with intimal thickening since there is an
hypoechoic layer approximately 1 mm in thickness below the echogenic line.
The arterial adventitia and media are the other layers of the arteries.
Atherosclerosis is seen as a bright, echogenic material in the intima of the vessel. This
"hard" plaque is the deposit of calcified material that can cause flow disturbances within
the vessel.
Fibrofatty plaque is seen as a low-level irregular patch in the intima. This type of plaque
may not be seen if the overall gain adjustment is too low. Color flow Doppler aids in
locating this type of plaque.

Question:
The blood vessel is lined by a single layer of which of the following cell types?

Answer Choices:
A Epithelial
B Erythrocyte
C Pericyte
D Fibroblast
E Endothelial

E Endothelial

The blood vessel is lined by a single layer of endothelial cells, which regulates the
exchange of molecules between the bloodstream and the surrounding tissues. The
vessel wall of mature capillaries is composed of an endothelial cell lining, a basement
membrane, and a layer of cells (called pericytes) that partially surround the
endothelium.
Angiogenesis is stimulated by the release of angiogenic growth factors that bind to
receptors on the endothelial cells and initiate a series of events involving stimulation of
the cells to grow, secretion of proteases, and formation of new capillaries. The secreted
proteases result in digestion of the basement membrane surrounding the vessel. Cell
projections from the endothelial cell pass through the space created, and the newly
formed sprout grows towards the source of the stimulus. Among the growth factors that
have been reported to act as angiogenic factors are fibroblast growth factors,
angiogenin, TNF-alpha, TGF-beta, and platelet derived growth factor. Angiogenesis is
also important in tumor growth. In order to grow larger, the tumor must have a blood
supply. A new approach to chemotherapy is to block angiogenesis and thereby "starve"
the tumor of nutrients derived from the bloodstream. Hematopoiesis is the generation of
blood cells that occurs mostly in the bone marrow.
Question:
In the mature capillary, angiogenesis is initially stimulated by the release of

Answer Choices:
A Hemoglobin
B Specific growth factors
C Proteases
D Collagen
E α2-macroglobulin

B Specific growth factors

The blood vessel is lined by a single layer of endothelial cells, which regulates the
exchange of molecules between the bloodstream and the surrounding tissues. The
vessel wall of mature capillaries is composed of an endothelial cell lining, a basement
membrane, and a layer of cells (called pericytes) that partially surround the
endothelium.
Angiogenesis is stimulated by the release of angiogenic growth factors that bind to
receptors on the endothelial cells and initiate a series of events involving stimulation of
the cells to grow, secretion of proteases, and formation of new capillaries. The secreted
proteases result in digestion of the basement membrane surrounding the vessel. Cell
projections from the endothelial cell pass through the space created, and the newly
formed sprout grows towards the source of the stimulus. Among the growth factors that
have been reported to act as angiogenic factors are fibroblast growth factors,
angiogenin, TNF-alpha, TGF-beta, and platelet derived growth factor. Angiogenesis is
also important in tumor growth. In order to grow larger, the tumor must have a blood
supply. A new approach to chemotherapy is to block angiogenesis and thereby "starve"
the tumor of nutrients derived from the bloodstream. Hematopoiesis is the generation of
blood cells that occurs mostly in the bone marrow.
Question:
Which one of the following chambers of the heart has the thickest layer of endocardium?
Answer Choices:
A Right ventricle
B Left ventricle
C Right atrium
D Left atrium
E All four chambers are lined with identical thickness of endocardium

D Left atrium

The endocardium varies in thickness in reverse proportion to the thickness of the


underlying myocardium. It has the thickest lining in the left atrium and thinnest in the left
ventricle.

Question:
The blood-brain barrier is thought to occur because capillaries in the central nervous system have
which one of the following characteristics?

Answer Choices:
A Discontinuous basal lamina
B Fenestrae with diaphragms
C Fenestrae without diaphragms
D A few pinocytic vesicles
E No basement membrane

D A few pinocytic vesicles

Capillaries in the central nervous system are of the continuous type and thus lack
fenestrae but have a continuous basal lamina. In contrast to continuous capillaries in
other parts of the body, they contain only a limited number of pinocytic vesicles; this
characteristic is thought to be partly responsible for the blood-brain barrier.

Question:
Which one of the following kinds of capillaries would one expect to be present most frequently in the
nervous system?

Answer Choices:
A Fenestrated (with diaphragm)
B Somatic (with few pinocytic vesicles)
C Fenestrated (without diaphragm)
D Discontinuous sinusoidal
E Somatic (with many pinocytic vesicles)

B Somatic (with few pinocytic vesicles)

The somatic, or continuous, capillaries are characterized by the absence of fenestrae in


their wall. In addition, very few or no pinocytotic vesicles are encountered in these
capillaries supplying the nervous system. This feature, in part, accounts for the
existence of the blood-brain barrier.
Fenestrated (with diaphragm) capillaries are encountered in the kidney, the intestine,
and the endocrine glands (where rapid exchange of substances occurs between blood
and tissues).
Fenestrated (without diaphragm) capillaries are characteristics of renal glomerulus.
The discontinuous sinusoidal capillaries are found mainly in the liver and in
hematopoietic organs, such as the bone marrow and spleen. The interchange between
blood and tissues is greatly facilitated by the structure of the wall of these capillaries.

Question:
Which of the following characteristics distinguishes somatic capillaries from visceral capillaries?

Answer Choices:
A Presence or absence of fenestrae
B Size of the lumen
C Thickness of the vessel wall
D Presence or absence of pericytes
E Thickness of the basal lamina

A Presence or absence of fenestrae

Somatic capillaries lack fenestrae, whereas visceral capillaries are characterized by the
presence of these fenestrae. Both types of capillaries possess a continuous basal
lamina and are surrounded by occasional pericytes.
Question:
Which one of the following statements concerning the vasa vasorum is correct?

Answer Choices:
A They are rod-like inclusions located in the cytoplasm of arterial endothelial
cells
B They are a system of vessels that supply both oxygen and nutrients to the
vascular wall
C They are pinocytotic vesicles that have been shown to transport experimental
markers, such as peroxidase, through the endothelium
D They are a system of blood vessels which serve to connect arterioles to the
capillaries
E They are a system of blood vessels which allow blood to bypass capillaries by
providing direct routes between arteries and veins

B They are a system of vessels that supply both oxygen and nutrients to the vascular
wall

Vasa vasorum is a system of blood vessels that supply both the oxygen and nutrients to
the thick walls in the elastic and muscular arteries. The rod-like inclusions visible in the
cytoplasm of arterial endothelial cells are called Weibel-Palade bodies. These specific
endothelial organelles are electron-dense structures and contain a coagulating factor
(factor VIII).
Nutrients and regulatory substances pass via the pinocytotic vesicles to the cells of the
tunica intima and tunica media. Arteriovenous shunts allow blood to bypass capillaries
by providing direct routes between arteries and veins.

Question:
Which one of the following blood vessels, as they essentially are endothelial lined tubules, CANNOT
actively contract?

Answer Choices:
A Arterioles
B Venules
C Elastic arteries
D Capillaries
E Metarterioles

D Capillaries

A true capillary is essentially an endothelial lined tubules, with a basal lamina, usually
about 8 μm in diameter, although some may be much larger (12 μm) or smaller (5 μm).
Because capillaries have no smooth muscle in their walls, they cannot actively contract.
However, the basal lamina of capillaries splits to enclose pericytes that may contract
and perhaps constrict the capillaries.
Elastic arteries include aorta, and have thick intima and media, but thin adventitia.
Metarterioles (precapillaries, which act as sphincters for capillaries) contain few
discontinuous smooth muscles, and arterioles possess relatively thick media, and as a
consequence, can contract. Venules, which resemble capillaries but may contain a few
smooth muscle cells, in a similar manner with other vessels of the venous system,
generally do not contract.
Question:
Which one of the following types of blood vessels are rolled up tubes of a single layer of endothelial
cells of mesenchymal origin?

Answer Choices:
A Arterioles
B Venules
C Distributing arteries
D Continuous capillaries
E Metarterioles

D Continuous capillaries

Capillaries, including continuous or somatic, are composed of endothelial cells of


mesenchymal origin rolled up in the form of a tube. The venules also have very thin
walls, but their adventitia is relatively thicker, while media may contain only contractile
pericytes along with few muscle cells. Metarterioles and arterioles, on the other hand,
both contain the usual connective and muscular layers, but they are relatively thinner in
metarterioles.

Question:
The atrial muscle of the heart produces a hormone that

Answer Choices:
A Decreases blood pressure
B Increases blood pressure
C Causes vasoconstriction
D Facilitates the release of rennin
E Facilitates Na+ resorption in the kidneys

A Decreases blood pressure

Atrial natriuretic peptide, which decreases blood pressure, is produced mainly by


cardiac muscle cells of the right atrium. It inhibits the release of rennin and causes the
kidneys to decrease the resorption of Na+ and water.

Question:
The most striking difference between elastic and muscular arteries is
Answer Choices:
A The elastic component of the tunica media is diminished in muscular arteries
B The subendothelial connective tissue is thicker in muscular arteries
C The internal elastic membrane is less prominent in muscular arteries
D The tunica adventitia of muscular arteries is thinner relative to that of elastic
arteries
E The vasa vasorum and nervi vascularis are not present in muscular arteries

A The elastic component of the tunica media is diminished in muscular arteries

The most striking difference between elastic and muscular arteries is the diminished
elastic tissue component in the tunica media. As the elastic tissue component
diminishes, the muscular component becomes more prominent, as do the internal and
external elastic membranes.
Other differences between the two types of arteries include a thinner subendothelial
connective tissue component and a relatively thicker adventitia in muscular arteries. The
vasa vasorum and the nervi vascularis can be identified in muscular arteries.
Question:
In large arteries, the collagen and elastic fibers of the intima are produced by

Answer Choices:
A Endothelial cells
B Smooth muscle cells
C Fibroblasts
D Cells of the vasa vasorum
E Cells of the nervi vascularis

B Smooth muscle cells

The subendothelial layer of the tunica intima of large vessels has a significant smooth
muscle component. The smooth muscle has two functions: it can contract the vessel
lumen and it synthesizes the collagen and elastic fibers of the intima. In addition,
smooth muscle cells produce the intimal ground substance.

Question:
The number of elastic sheets, or lamellae, found in the tunica media of elastic arteries is related to

Answer Choices:
A The presence or absence of the htm. gene on chromosome 16
B A patient's sex
C A patient's age and blood pressure
D A patient's cholesterol level
E Pregnancy

C A patient's age and blood pressure

The number of elastic lamellae in the tunica media is related to a patient's age and
blood pressure. Newborns lack these elastic sheets, but adults have 40-70 of them in
large arteries. If a patient is hypertensive, these sheets become more numerous and
become thicker.

Question: BLOCK EXAM


The vasa vasorum is

Answer Choices:
A The system of autonomic nerves supplying the tunica media
B The second capillary bed in a portal system
C A network of blood vessels that supply the walls of large arteries and veins
D A vascular system in which arterioles anastomose with venules
E The first capillary bed in a portal system

C A network of blood vessels that supply the walls of large arteries and veins

The vasa vasorum is a network of blood vessels that supply the walls of large arteries
and veins. The vasa vasorum travels through the tunica adventitia, sending branches
into the tunica media. This system supplies only the outer wall of the vessel; the inner
portion receives oxygen and other molecules from the intimal surface of the vessel.

Question:
The tunica adventitia is thickest in

Answer Choices:
A Large arteries
B Medium arteries
C Small arteries
D Capillaries, arterioles and postcapillary venules
E Veins and muscular venules

E Veins and muscular venules

The tunica adventitia is the outermost layer of the blood vessel wall. It is composed
primarily of collagen and contains some elastic fibers. In the arterial system, the tunica
adventitia is separated from the tunica media by the external elastic lamina.
Throughout most of the arterial system, the tunica adventitia is relatively thin (thinner
than the tunica intima). Capillaries and postcapillary venules do not have a tunica
adventitia. In contrast, in the veins and muscular venules, the tunica adventitia is thick
and well developed.
Question:
In a portal system

Answer Choices:
A There is a direct anastomosis between an arteriole and a venule
B The capillary bed has an unusually intimate relationship with the digestive tract
C Blood from capillaries flows directly into a vein without passing through a
venule
D A vein or an arteriole is present between two capillary beds
E Blood passes from a large artery immediately into an arteriole

D A vein or an arteriole is present between two capillary beds

In a portal system a vein or an arteriole lies between two capillary beds. The venous
portal systems are the hepatic portal system, which carries blood to the liver, and the
hypothalamic-hypophyseal system, which carries blood to the pituitary. An arteriolar
portal system exists in the kidney, where an arteriole transports blood from the
capillaries of the glomerulus to the capillaries of the peritubular system.
Question: CLASS QUIZ
The attached photomicroscopic image depicts a low powered magnification from the wall of a blood
vessel (the lumen is identified with an 'L'). Based upon the histological structure, this vessel is best
identified as a

Answer Choices:
A Elastic artery
B Muscular artery
C Arteriole
D Muscular vein
E Large vein

Image(s) / Chart(s):
Click image to view full size. Click open image to close. Click and hold open image to move.
A Elastic artery

The photomicroscopic image demonstrates the histological appearance of an elastic


artery, such as the aorta. Note the heavily stained and predominant internal elastic
membrane (difficult to discern) immediately deep to the lumen (L) denoting the
boundary between the tunica intima and the thick tunica media. The tunica media is
comprised of multiple layers of sheets of elastic fibers interspersed by smooth muscle
fibers arranged circumferentially around the lumen of the vessel. Finally, a portion of the
tunica adventitia can be observed. Note that it has few elastic fibers and is comprised
mostly of a predominantly collagenous connective tissue.

Question:
The outermost regions of the tunica adventitia receive blood supply by what structures?

Answer Choices:
A Vasa nervi
B Tunica vasa
C Vasa vasorum
D Vasa recta

C Vasa vasorum

The vasa vasorum are a system of vessels that provide blood supply to the tunica
adventitia of larger arteries or veins (and of the epicardium of the heart). Since the
tunica adventitia is the "outermost" layer of the vessels, it may be somewhat removed in
distance from the vessel lumen, and require a source for its own blood supply.

Question:
Relaxation of peripherally located precapillary sphincters will result in which one of the following
responses?

Answer Choices:
A Cooling of the digits of the hands or feet
B Warming of the digits of the hands or feet
C Conservation of core body temperature
D Increased blood flow to visceral capillaries
E Increased blood flow to erectile tissue

B Warming of the digits of the hands or feet

Blood flow into a capillary bed is controlled by precapillary sphincters. Frequently


located in the peripheral vessels is a shunt in which the arterioles are connected directly
to a venule. Constriction of the precapillary sphincter will result in the shunting of blood
flow to the venous system, bypassing the capillary network and, hence, conserving
temperature since the blood will tend to not pass through capillary networks near the
body surfaces, especially in the digits of the hands or feet. Conversely, relaxation of the
precapillary sphincter will result in increased blood flow to the capillary network, and
when located peripherally, the effect will be to warm the digits of the hands or feet.
Question:
During the initial phases of exercise, the skeletal muscles are presented with a condition of low oxygen
tension. What effect does low oxygen tension have on arteriolar smooth muscle?

Answer Choices:
A Relaxation of the smooth muscle and increased blood flow to the skeletal
muscle
B Relaxation of the smooth muscle and decreased blood flow to the skeletal
muscle
C Stimulation of the smooth muscle and increased blood flow to the skeletal
muscle
D Stimulation of the smooth muscle and decreased blood flow to the skeletal
muscle
E Increased blood flow to the skeletal muscle occurs without arteriolar regulation
when heart rate is increased

A Relaxation of the smooth muscle and increased blood flow to the skeletal muscle

An arteriole is defined as a small artery with only one or two layers of smooth muscle in
the tunica media just prior to its ramification into a capillary bed. There may be a slight
increase in the thickness of the muscle layer of the arteriole just prior to the origin of the
capillary bed which is the precapillary sphincter. It is the arteriole which serves to
regulate flow of blood to the respective capillary bed. At the beginning of physical
activity in which skeletal muscle contraction is increased, there may exist a short period
during which the metabolic requirements of the skeletal muscle exceed the existing
supply of oxygen to the muscle, resulting in low oxygen tension. In response to the low
oxygen tension, the smooth muscle cells of the precapillary sphincter relaxes,
vasodilation occurs and an increase in blood flow to the capillary bed results.

Question:
The first portion of the blood vascular system where an internal elastic membrane becomes absent is
in which one of the following vessels?

Answer Choices:
A Elastic artery
B Muscular artery
C Small artery
D Arteriole
E Capillary

D Arteriole

The internal elastic membrane is composed of a layer of elastic tissue situated as the
innermost layer of the tunica intima. This fenestrated membrane may allow substances
to pass through the "pores" to reach deeper portions of the wall of the vessel. It can be
observed in virtually every artery, and is predominantly seen in large to small muscular
arteries. However, the first area where the internal elastic membrane is absent is in
arterioles.

Question:
The vasa vasorum are located in which layer of larger arteries and/or veins?

Answer Choices:
A Tunica adventitia
B Tunica intima
C Tunica media
D Tunica endothelia

A Tunica adventitia

The vasa vasorum are a system of vessels that provide blood supply to the tunica
adventitia of larger arteries or veins (and of the epicardium of the heart). Since the
tunica adventitia is the "outermost" layer of the vessels, it may be somewhat removed in
distance from the vessel lumen, and require a source for its own blood supply.

Question: CLASS QUIZ


To differentiate between a medium sized vein and its "companion" artery, a difference is found in the
appearance of what component associated with both structures?

Answer Choices:
A Tunica intima
B Tunica media
C Subendothelial layer
D Basal lamina

B Tunica media

Structurally, veins and arteries have the same basic layers, i.e. tunica intima, tunica
media and tunica adventitia. However, these layers tend to be less distinct than in
similar sized arteries, particularly in the smaller veins. These layers are distinguishable
in medium veins and arteries. In these vessels, the tunica intima of both the vein and
artery are similar, whereas the tunica media of medium-sized veins tends to appear
much thinner than that of the companion artery. Furthermore, the tunica adventitia is
usually thicker than the tunica media of the medium-sized vein, and may even be more
extensive than the adventitia of the companion artery. From the list provided, it is the
tunica media which differs most extensively in appearance when comparing similar
sized medium arteries and veins.

Copyright © EXAM MASTER Corporation 2014 Author ID: 23

Question:
The subendothelial lining of an artery is included as which one of the layers of the vessel?

Answer Choices:
A Tunica intima
B Tunica propria
C Tunica media
D Tunica adventitia
E Tunica lamina

A Tunica intima

The innermost (luminal) layer of blood vessels (arteries, veins, or capillaries) are lined
by a simple squamous epithelium, specifically referred to a endothelium. The
endothelium is the luminal surface of the tunica intima. A basal lamina is present
underlying the endothelium, and a subendothelial layer of connective tissue is also
present in the t. intima.

Question:
Many different classes of drugs are used as anti-tumor agents. The rationale for each different drug
takes advantage of a particular property of tumor cells compared to normal cells. Recently, inhibition of
angiogenesis has become an important potential avenue of cancer chemotherapy. The blood vessel is
lined by a single layer of which of the following cell types?

Answer Choices:
A epithelial
B erythrocyte
C pericyte
D fibroblast
E endothelial

E endothelial

Angiogenesis, the process of new blood vessel formation, is stimulated by the release
of angiogenic growth factors that bind to receptors on the endothelial cells and initiate a
series of events involving stimulation cell growth, secretion of proteases, and formation
of new capillaries. The secreted proteases result in digestion of the basement
membrane surrounding the vessel. Cell projections from the endothelial cell pass
through the space created and the newly formed sprout grows towards the source of the
stimulus. Among the growth factors that have been reported to act as angiogenic factors
are fibroblast growth factors, angiogenin, TNF-alpha, TGF-beta, and platelet derived
growth factor. Angiogenesis is important in tumor growth. In order to grow larger, the
tumor must have a blood supply. A new approach to chemotherapy is to block
angiogenesis and thereby "starve" the tumor of nutrients derived from the bloodstream.
A single layer of endothelial cells lines the blood vessel that regulates the exchange of
molecules between the bloodstream and the surrounding tissues. The vessel wall of
mature capillaries is composed of an endothelial cell lining, a basement membrane, and
a layer of cells called pericytes that partially surround the endothelium.

Copyright © EXAM MASTER Corporation 2014 Author ID: 32


Case #7419:
A 45-year-old Caucasian female with a 25-year history of type 1 diabetes complicated
by retinopathy and neuropathy and intermittently followed at a community health clinic
presents to the urgent care center complaining of nausea, vomiting, anorexia, and
lightheadedness for the past several days. The urgent care center physicians draw
some blood and note that her serum creatinine is 6.0 mg/dl, her serum urea nitrogen
is 80 mg/dl, her serum potassium is 6.0 mg/dl, and her serum bicarbonate is 18 meq/l.
She could not void, but a catheterized urine specimen yielded 70 mL of muddy
colored urine with trace protein, trace blood, and muddy-colored casts. She has no
peaked T waves on her electrocardiogram and the urgent care center physician has
given her insulin, intravenous calcium, and sodium polystyrene to treat hyperkalemia.
She has also had 0.5 liter of intravenous normal saline over several hours. She has
not yet had a bowel movement. The urgent care center physician wants to admit her
for treatment of acute renal failure (acute kidney injury) and hyperkalemia.
On physical exam, she has reduced bilateral femoral, popliteal and dorsalis pedis
pulses, but good axillary and radial pulses. Jugular veins are readily visible for
hemodialysis catheter placement.
Question:
Regarding the potential placement of a temporary catheter to treat acute kidney
injury, how will the treating physician best ethically inform this patient about the risks
and benefits of its placement?

Answer Choices:
A Risks of bleeding and bruising in neck and upper extremity
B Risks of bleeding, infection, and bruising in groin and lower extremity
C Risks of bleeding, infection, thrombosis, and stenosis of vessels of upper
extremity
D Risks of bleeding, catheter infection, and decreased urine output
E Risks of bleeding, catheter infection, and urinary tract infection
C Risks of bleeding, infection, thrombosis, and stenosis of vessels of upper extremity

This patient most likely has acute tubular necrosis, a form of acute kidney injury (acute
renal failure) as evidenced by her elevated serum creatinine, blood urea nitrogen, and
the muddy colored casts seen in her urine. Most likely a) decreased oral fluid intake
leads to b) decreased circulating blood volume and hypotension and c) prolonged
hypoperfusion of her kidneys. The kidneys are very vascular and are susceptible to
changes in circulating blood volume. Hypoperfusion causes sloughing of the tubular
endothelium and the kidneys can no longer properly filter the blood of its nitrogenous
wastes. Urine casts,elevated serum creatinine, and urea ensue. If this patient's
potassium, creatinine, and urea nitrogen are persistently elevated and she is
symptomatic (nauseated, shaky, lethargic), she may be a candidate for dialysis.
Temporary catheters for hemodialysis, the most commonly used form of dialysis in the
United States, are preferentially placed in the internal jugular veins. Other sites used
include the subclavian and femoral vessels; however, placement depends on the
viability of the vessels and their proximity to other devices such as cardiac pacemakers.
Placement is associated with a variety of complications and patients should be informed
about them before catheters are placed. Complications include bleeding and bruising
near the site of the catheter insertion, infections of the skin surrounding the
catheter/catheter/bloodstream, pain near the site of the catheter, difficulty moving
secondary to the catheter (moving neck, etc), thrombosis due to venous stasis
surrounding the catheter, and stenosis of the catheterized vessel. All these
complications should be treated. Some can be treated more easily than others.
Bleeding areas can be compressed and line infections can be prevented with
handwashing and dressings and treated with antibiotics. Central stenosis treatment is
more complex, as described below.
Central stenosis is a serious complication of dialysis catheter placement. Stenosed
vessels have reduced caliber, leading to poor perfusion of some tissues and excess
pressure in other tissues. It occurs in the central vessels: the subclavian vein, cephalic
vein, superior vena cava, brachiocephalic vein, and their junctions.
Consequently, patients may develop a variety of signs and symptoms including
headache, neck/face/arm swelling, chest pain, shortness of breath, dilated/engorged
veins, and elevated venous pressures with hemodialysis (>150 mm Hg).
Central stenosis is an upper extremity phenomenon. It does not cause lower extremity
swelling or edema, pain, or decreased pulses and typically does not cause ascites
although it can cause pleural effusions secondary to increased pressure in the
pulmonary vasculature.
Central stenosis can occur with placement of a variety of catheters (hemodialysis,
plasmapheresis, total parenteral nutrition), pacemakers, or simply due to tumor
compression of vessels. Thrombosis secondary to hypercoagulable states can also be a
cause.
The insertion of subclavian vein catheters causes up to 50% of cases of central stenosis
so internal jugular lines are much preferred; pacemaker insertion causes some 30-60%.
Ipsilateral arteriovenous fistula and pacemaker placement are also well known causes
of central stenosis. It may appear months to years after the insertion. Formation of
collaterals may halt its occurrence.
Treatment includes ligating or closing off fistulas and using alternative accesses,
removing grafts, removing unnecessary catheters, debulking tumors that are
compressing vessels, and angioplasty with stenting of stenosed vessels.
Angioplasty alone is not often successful.
Because adequate access is critical for adequate dialysis and catheter use has been
rising, there are initiatives to decrease catheter use and increase fistula use, particularly
in patients who will require long term dialysis access. These initiatives may save
vessels (not use), which will ultimately be used for fistulas. Additionally, phlebography or
venous mapping prior to catheter insertion is helpful to better understand the patency of
patient's vessels.
Simply noting that the patient may have bruising or bleeding after hemodialysis catheter
insertion is inadequate information for the patient described. Most likely, she will not
have a femoral line placed because of her reduced lower extremity pulses. For this
patient, an internal jugular vein hemodialysis catheter is more appropriate.
Hemodialysis catheters function to connect a dialysis machine to the patient so that
wastes can be cleared. The dialysis process is dependent on adequate blood flow.
(usually at least 200 ml/min). If she does not have a femoral line placed, groin bruising
is not a likely complication.
The catheter placement itself is not a likely cause of decreased urine output. A better
explanation is that the acute kidney injury causes decreased glomerular filtration and
decreased urine production. Additionally, dialyzed patients who have ultrafiltration or
fluid removal during dialysis have less excess fluid to be eliminated in urine but the
catheter itself will not decrease urine output directly.
Question:
The emergency room calls regarding your 65-year-old Caucasian American male patient who
presented with weakness and was found to have atrioventricular block. He does not appear to have
had a heart attack and the consulting cardiologist wants to insert a pacemaker. He has chronic kidney
disease secondary to diabetes and his most recent glomerular filtration rate was 35 ml/min/1.73 m2.
What concerns you the most regarding the pacemaker placement and his likely future need for
hemodialysis access?

Answer Choices:
A Pacemakers are a contraindication to dialysis access placement
B Arteriovenous fistulas are a contraindication to pacemaker insertion
C Bleeding is a complication of both pacemaker insertion and dialysis catheter
placement
D Both pacemaker insertion and dialysis catheter placement can cause cellulitis
E Ipsilateral pacemaker and upper extremity dialysis access can cause central
stenosis

E Ipsilateral pacemaker and upper extremity dialysis access can cause central
stenosis

Temporary catheters for hemodialysis, the most commonly used form of dialysis in the
United States, are preferentially placed in the internal jugular veins. Other sites used
include the subclavian and femoral vessels; however, placement depends on the
viability of the vessels and their proximity to other devices such as cardiac pacemakers.
Placement is associated with a variety of complications and patients should be informed
about them before catheters are placed. Complications include bleeding and bruising
near the site of the catheter insertion, infections of the skin surrounding the
catheter/catheter/bloodstream, pain near the site of the catheter, difficulty moving
secondary to the catheter (moving neck, etc), thrombosis due to venous stasis
surrounding the catheter, and stenosis of the catheterized vessel. All these
complications should be treated. Some can be treated more easily than others.
Bleeding areas can be compressed and line infections can be prevented with
handwashing and dressings and treated with antibiotics. Central stenosis treatment is
more complex, as described below.
Central stenosis is a serious complication of dialysis catheter placement. Stenosed
vessels have reduced caliber, leading to poor perfusion of some tissues and excess
pressure in other tissues. It occurs in the central vessels: the subclavian vein, cephalic
vein, superior vena cava, brachiocephalic vein, and their junctions.
Consequently, patients may develop a variety of signs and symptoms including
headache, neck/face/arm swelling, chest pain, shortness of breath, dilated/engorged
veins, and elevated venous pressures with hemodialysis (>150 mm Hg).
Central stenosis is an upper extremity phenomenon. It does not cause lower extremity
swelling or edema, pain, or decreased pulses and typically does not cause ascites
although it can cause pleural effusions secondary to increased pressure in the
pulmonary vasculature.
Central stenosis can occur with placement of a variety of catheters (hemodialysis,
plasmapheresis, total parenteral nutrition), pacemakers, or simply due to tumor
compression of vessels. Thrombosis secondary to hypercoagulable states can also be a
cause.
The insertion of subclavian vein catheters causes up to 50% of cases of central stenosis
so internal jugular lines are much preferred; pacemaker insertion causes some 30-60%.
Ipsilateral arteriovenous fistula and pacemaker placement are also well known causes
of central stenosis. It may appear months to years after the insertion. Formation of
collaterals may halt its occurrence.
Treatment includes ligating or closing off fistulas and using alternative accesses,
removing grafts, removing unnecessary catheters, debulking tumors that are
compressing vessels, and angioplasty with stenting of stenosed vessels.
Angioplasty alone is not often successful.
Because adequate access is critical for adequate dialysis and catheter use has been
rising, there are initiatives to decrease catheter use and increase fistula use, particularly
in patients who will require long term dialysis access. These initiatives may save
vessels (not use), which will ultimately be used for fistulas. Additionally, phlebography or
venous mapping prior to catheter insertion is helpful to better understand the patency of
patient's vessels.
Simply noting that the patient may have bruising or bleeding after hemodialysis catheter
insertion is inadequate information for the patient described. Most likely, he will not have
a femoral line placed because of his reduced lower extremity pulses. For this patient, an
internal jugular vein hemodialysis catheter is more appropriate.
Hemodialysis catheters function to connect a dialysis machine to the patient so that
wastes can be cleared. The dialysis process is dependent on adequate blood flow.
(usually at least 200 ml/min). If he does not have a femoral line placed, groin bruising is
not a likely complication.
The catheter placement itself is not a likely cause of decreased urine output. A better
explanation is that the acute kidney injury causes decreased glomerular filtration and
decreased urine production. Additionally, dialyzed patients who have ultrafiltration or
fluid removal during dialysis have less excess fluid to be eliminated in urine but the
catheter itself will not decrease urine output directly.
Case #7420:
A 50-year-old Caucasian male you have been following in the hospital for several
weeks following a living, related kidney transplant for diabetic nephropathy develops
right neck and upper extremity swelling without erythema, warmth, or adenopathy. His
oral exam is normal and his white blood cell count is unchanged. His blood pressure
is 135/80 mm Hg, his temperature is 98.5 Farhenheit, and he has decreased breath
sounds in the base of his right lung. His heart and lower extremity exam are normal.
He has no cough or phlegm. His chest X-ray shows no infiltrate or effusion. His right
lower abdomen has a non-tender incision site (of renal transplantation) without
exudate or bruit.
He has never been dialyzed but he did have central venous access for hemodynamic
monitoring postoperatively. His kidney is functioning well.
Question:
What diagnosis is most likely, given his recent central venous catheter use?
Answer Choices:
A Abscess in arm
B Cellulitis of arm
C Bacteremic sepsis
D Mediastinitis
E Venous stenosis
E Venous stenosis

This patient most likely has Central Venous Stenosis as a complication of indwelling
central venous catheter used perioperatively after his kidney transplant.
Stenosed vessels have reduced caliber, leading to poor perfusion of some tissues and
excess pressure in other tissues. It occurs in the central vessels: the subclavian vein,
cephalic vein, superior vena cava, brachiocephalic vein, and their junctions.
Consequently, patients may develop a variety of signs and symptoms including
headache, neck/face/arm swelling, chest pain, shortness of breath, pleural effusions
(secondary to increase pulmonary vasculature pressure) dilated/engorged veins, and
elevated venous pressures with hemodialysis (>150 mm Hg).
Central stenosis can occur with placement of a variety of catheters (hemodialysis,
plasmapheresis, total parentral nutrition), pacemakers, or simply due to tumor
compression of vessels. Thrombosis secondary to hypercoagulable states can also be a
cause.
The insertion of subclavian vein catheters causes up to 50% of cases of central stenosis
so internal jugular lines are much preferred; pacemaker insertion causes some 30-60%
of cases.
Ipsilateral arteriovenous fistula and pacemaker placement are also well known causes
of central stenosis. It may appear months to years after the insertion. Formation of
collaterals may halt its occurrence.
Treatments include ligating or closing off fistulas and using alternative accesses,
removing grafts, removing unnecessary catheters, debulking tumors that are
compressing vessels, and angioplasty with stenting of stenosed vessels.
Angioplasty alone is not often successful.
Because adequate access is critical for dialysis and catheter use has been rising, there
are initiatives to decrease catheter use and increase fistula use, particularly in patients
who will require long term dialysis access. These initiatives may save vessels that will
ultimately be used for fistulas. Additionally, phlebography or venous mapping prior to
catheter insertion is helpful to better understand the patency of the patient's vessels.
Other causes of arm and neck swelling can include abscesses (deep walled off
infections) and cellulites (skin infections). Treatment for abscesses includes surgical
drainage and antibiotics that will penetrate the abscess at its particular location.
Cellulitis is typically treated with topical and parenteral antibiotics, depending on the
severity of the infection and comorbid conditions. A patient following a transplant is
immunosuppressed and consequently might receive oral antibiotics to prevent infectious
systemic spread from the cellulitis.
Cellulitis and abscesses could follow catheter placement; perhaps skin puncture and
inadequately cleansed areas left an immunocompromised patient susceptible. Although
such a patient is presumably on some form of immunosuppression for his transplant
and therefore might not have as marked a change in white blood cell count or increased
temperature associated with an abscess or cellulitis, the absence of any signs of
infection (leukocytosis, fever, and erythema) make abscesses and cellulitis less likely.
Fever, leukocytosis, and hypotension might all follow a bacteremic/sepsis infection
secondary to an indwelling venous catheter. Their absence makes this diagnosis less
likely. Most patients are treated with intravenous antibiotics and a search is made for
the source of bactremia. Once found, the source (i.e. infected catheter) is removed.
Some patients require pressor-support for hypotension and intensive care for
hemodynamic monitoring.
Although swelling of the neck is a symptom of mediastinitis, other features of infection
like fever and chills are absent. There is also no history of previous upper respiratory
tract or dental infection, which can predispose to mediastinitis.
Question:
A 3-year-old male was brought into the emergency room by his mother with complaints of becoming
bluish on excessive crying. Examination showed a male in moderate distress having difficulty in
breathing. There was general cyanosis of the extremities. Palpation revealed a thrill at the left lateral
sternal border and parasternal lift. Auscultation was positive for a loud S2 with a grade 3/6 pansystolic
murmur in the left third and fourth interspaces. Imaging studies revealed the chest x-ray was positive
for cardiomegaly with prominence of ventricles and increased pulmonary vasculature. ECG was
positive for biventricular hypertrophy, and echo revealed the presence of a large ventricular septal
defect. A color flow Doppler showed a right-to-left shunt. What is the pathophysiology of cyanosis of
this baby?

Answer Choices:
A Decreased left ventricular compliance due to chronicity
B Pulmonary hypertension leading to elevated right ventricular pressure,
ultimately creating a right-to-left shunt
C Elevation of left ventricular end-diastolic pressure
D Ventricular fibrillation suddenly occurs, leading to lack of cardiac output and
rise for hemoglobin
E Ventricular septal defect with a cleft mitral valve

B Pulmonary hypertension leading to elevated right ventricular pressure, ultimately


creating a right-to-left shunt

The above clinical information points towards the presence of VSD. Since there is right-
to-left shunt on the color flow Doppler, this is a case of Eisenmenger syndrome.
In the presence of a VSD, there is sustained left-to-right shunt, and over a period of time
there is eventual shunt reversal, i.e. flow of blood from right-to-left. This reversal of
shunt mixes the oxygenated and deoxygenated blood, leading to a higher concentration
of deoxygenated blood into the systemic circulation. This blood has higher levels of
reduced hemoglobin and hence leads to more cyanosis on minimal exertion in these
patients.
Left ventricular compliance does not play a major role in the etiology of cyanosis in
these patients, nor does elevation of left ventricular end diastolic pressure. Ventricular
septal defect with cleft mitral valve by itself will not lead to an increase for hemoglobin to
produce cyanosis. Ventricular fibrillation will lead to acute cyanosis with fatal
consequences if not detected and treated immediately.
Case #301463:
A 50-year-old electrician reports pain in his left upper extremity and tingling and
numbness in the 4th and 5th digits of his left hand. There is mild swelling of the left
hand. The man reports that most of his pain and numbness occurs when he is doing
electrical work with his arms overhead. X-ray reveals the presence of a cervical rib.
Question:
What artery structure is most likely being compressed?

Answer Choices:
A Axillary artery
B Brachial artery
C Brachiocephalic artery
D Subclavian artery
E Subscapular artery

D Subclavian artery

The subclavian artery passes laterally over the upper surface of the first rib and lies
posterior to the scalenus anterior. In the case of thoracic outlet syndrome, this artery is
usually compressed between the scalenus anterior and a cervical rib. Thoracic outlet
syndrome is a broad term for a group of disorders in which there is compression of
certain neurovascular bundles. The presence of a cervical rib adds to the compression,
and repetitive motion and poor posture are other predisposing factors. When the
neurovascular bundle is entrapped, the patient presents with neurological and/or
circulatory changes in the upper extremity on the involved side.

The axillary artery is the continuation of the subclavian artery. This artery begins at the
outer border of the first rib and continues through the axilla to the lower border of the
teres major.

The brachial artery is the continuation of the axillary artery in the upper extremity and
cannot be compressed by a cervical rib.

The brachiocephalic artery arises from the arch of the aorta. At the level of the right
sternoclavicular joint, it separates into the right common carotid artery and the right
subclavian artery.

The subscapular artery is the largest branch of the axillary artery. It supplies muscles on
the lateral thoracic wall and scapular muscles.
SECTION- HEART
Question:
The tricuspid valve of the heart is located between:

Answer Choices:
A The right atrium and right ventricle
B The left atrium and the left ventricle
C The left ventricle and the aorta
D The right ventricle and the pulmonary artery
E The pulmonary vein and the left atrium
A The right atrium and right ventricle

4 fibrous valves guard the orifices of the heart. They are (i) the tricuspid located between the
right atrium and right ventricle, (ii) the pulmonary semilunar, (iii) the aortic semilunar, and (iv)
the bicuspid or mitral located between the left atrium and left ventricle.
Question:
Which one of the following statements concerning coronary circulation is correct?

Answer Choices:
A They are functionally equivalent to vasa vasorum of large veins and arteries and
anastomose with the vasa vasorum of the great vessels of the heart
B At puberty, they develop longitudinal muscle fibers in the thickened intima called a
cushion
C They have an inner longitudinal and an outer circular layer of smooth muscle in the
tunica media
D They resemble veins in having a thin wall but a prominent internal elastic lamina

A They are functionally equivalent to vasa vasorum of large veins and arteries and anastomose
with the vasa vasorum of the great vessels of the heart

Coronary arteries have thick walls with considerable elastic tissue. They are functionally
equivalent to vasa vasorum of large veins and arteries and anastomose with the vasa vasorum
of the great vessels of the heart.
Cerebral arteries resemble veins in having a thin wall but a prominent internal elastic lamina. At
puberty, the arteries of the penis develop longitudinal muscle fibers in the thickened intima
called a cushion. Umbilical arteries have an inner longitudinal and an outer circular layer of
smooth muscle in the tunica media.

Question:
Valves of the heart consist of leaflets composed of what tissues?

Answer Choices:
A Dense collagenous connective tissue covered by endothelium
B Elastic connective tissue covered by endothelium
C Dense collagenous connective tissue and elastic fibers covered by endothelium
D Elastic cartilage covered by endothelium
E Fibromuscular tissue and elastic fibers covered by endothelium

C Dense collagenous connective tissue and elastic fibers covered by endothelium

The valves of the heart are composed of dense collagenous connective tissue, which forms a
tough fibrous sheet. This fibrous sheet contains a substantial number of elastic fibers. These
leaflets are then covered by a thin endothelial layer. . The connective tissue core becomes more
condensed at the margins of the valves as it becomes continuous with the anulus, or fibrous
ring forming the skeleton of the heart.
Question:
Which one of the following components of the heart has the thickest walls?

Answer Choices:
A Aorta
B Right atrium
C Right ventricle
D Left atrium
E Left ventricle

E Left ventricle
Image(s) / Chart(s):
Click image to view full size. Click open image to close. Click and hold open image to move.
The thickness of the wall varies depending upon the pressure within the various cavities of the
heart. It is the thinnest in the low-pressure atria and thickest in the high-pressure left ventricle
(incidentally, the thickness of the wall is mainly due to the thickness of the myocardium, the
cardiac muscle layer).
Aorta, an elastic artery, has relatively thin walls when they are compared to its lumen.
See the attached image of the aorta.

Question: CLASS QUIZ


The generation of impulses in the normal heart is the responsibility of which of its following
components?

Answer Choices:
A Atrioventricular (AV) node
B Atrioventricular fiber bundles of His
C Sympathetic nerves
D Sinoatrial (SA) node
E Purkinje fibers

D Sinoatrial (SA) node

Impulses in the heart are generated by the rhythmic beating of the sinoatrial (SA) node (the
pacemaker of the heart) located near the opening of the superior vena cava inside the right
atrium. From there, they are picked up by the atrioventricular (AV), and conveyed to the bundle
of His and Purkinje fibers.
The contraction thus is initiated by the SA nodes, but its rate is modulated by the sympathetic
nerve fibers.

D Sinoatrial (SA) node

Impulses in the heart are generated by the rhythmic beating of the sinoatrial (SA) node (the
pacemaker of the heart) located near the opening of the superior vena cava inside the right
atrium. From there, they are picked up by the atrioventricular (AV), and conveyed to the bundle
of His and Purkinje fibers.
The contraction thus is initiated by the SA nodes, but its rate is modulated by the sympathetic
nerve fibers.

B Dense fibrous ring to which the bases of all cusps are centrally located
The cardiac valves consist of a central core of dense fibrous connective tissue (containing both
collagen and elastic fibers) lined on both sides by endothelial layers. The bases of these valves
are attached by means of chordae tendineae (fibrous cords which extend from the inner
surface of the ventricular wall to connect the ventricular surface of each valve leaflet) to the
annuli fibrosi (dense fibrous rings to which the bases of all cusps are centrally located) of the
fibrous skeleton.

Question:
Which one of the following statements concerning the myocardium is true?

Answer Choices:
A The myocardium of the ventricles is substantially thicker than that of the atria because
of the large amount of cardiac muscle in the walls of the two pumping chambers
B The myocardium consists of a layer of mesothelium and its underlying connective
tissue
C The blood vessels and the nerves supplying the heart lie in this layer, and are
surrounded by the adipose tissue
D It consists of an inner layer of endothelium and subendothelial connective tissue, a
middle layer of connective tissue and smooth muscle cells, and a deeper layer of
connective tissue

A The myocardium of the ventricles is substantially thicker than that of the atria because of
the large amount of cardiac muscle in the walls of the two pumping chambers
Image(s) / Chart(s):
Click image to view full size. Click open image to close. Click and hold open image to move.
Myocardium, the cardiac muscle, is the principal component of the heart. The myocardium of
the ventricles is substantially thicker than that of the atria because of the large amount of
cardiac muscle in the walls of the two pumping chambers.
The epicardium consists of a layer of mesothelium and its underlying connective tissue. This
layer also contains blood vessels and the nerves supplying the heart. The vessels and nerves are
surrounded by the adipose tissue which forms a cushion for the heart in the pericardial cavity.
The endocardium is formed by an inner layer of endothelium and subendothelial connective
tissue, a middle layer of connective tissue and smooth muscle cells, and a deeper layer, also
called subendocardial layer of connective tissue, that is continuous with the connective tissue
of the heart.
See the attached image: Various layers of the heart (Endocardium is located in the top of the
picture, and epicardium at the bottom)

Question:
Which of the following statements concerning the epicardium is true?

Answer Choices:
A It is continuous with the endocardium
B It is the visceral pericardium
C It possesses modified cardiac muscle cells
D It functions to increase intraventricular pressure
E It contains the impulse conducting system of the heart

B It is the visceral pericardium

The epicardium, composed of fibrous connective tissue and fat and lined by mesothelial cells,
forms the outer layer of the heart and the visceral layer of the pericardium.
The epicardium is continuous with myocardium, does not possess modified cardiac cells, does
not function to increase intraventricular pressure, and does not contains the impulse
conducting system of the heart.

Question:
The pale-staining cells as indicated by the arrowheads in the photomicroscopic image can be best
identified as what cells?

Answer Choices:
A Macrophages
B Autonomic ganglia
C Dorsal root ganglia
D Purkinje fibers
Image(s) / Chart(s):
Click image to view full size. Click open image to close. Click and hold open image to move.

D Purkinje fibers

The cells as indicated by the arrowheads in the photomicroscopic image are best identified as
Purkinje fibers. Purkinje fibers are located in the subendocardium of the heart and are
structurally related to the cardiac muscle fibers, but they contain reduced numbers of
myofibers compared to the cardiac muscle fibers. Furthermore, they appear swollen by
contrast to the cardiac muscle cells since they contain lipids. These cells are electrically irritable
and are part of the conducting system of the heart.

Question: CLASS QUIZ


The tissue illustrated in the attached photomicrographic image is best identified as

Answer Choices:
A Skeletal muscle in cross-section
B Smooth muscle in cross-section
C Cardiac muscle in cross-section
D Cardiac muscle in longitudinal section
E Skeletal muscle in longitudinal section

Image(s) / Chart(s):
Click image to view full size. Click open image to close. Click and hold open image to move.
C Cardiac muscle in cross-section

The image depicts cardiac muscle when observed in cross-section. Cardiac muscle fibers are
large branching fibers with one or more centrally located nuclei. Since the fibers are large and
branching, a cross sectional image would reveal fiber branches that do not contain nuclei,
whereas other sections of the fiber contain the centrally located nuclei. Nuclei that can be seen
on the peripheral portions of the fibers may include endothelial nuclei of capillaries or
fibroblast nuclei. When observed in cross section, cardiac muscle differs histologically from
skeletal muscle which are multinucleated and located on the periphery of the fiber, and smooth
muscle fibers have a single, centrally located nucleus.
Question:
Purkinje fibers, which are associated with the impulse-conducting pathways coursing throughout the
heart, are best described as being located in what "layer" of the heart?

Answer Choices:
A Epicardium
B Myocardium
C Endocardium
D Subendocardium

D Subendocardium

The subendocardial layer of the endocardium, somewhat analogous to the subendothelium of


the blood vessels, consists of connective tissue that becomes continuous with the connective
tissue of the myocardium. The Purkinje fibers are located within the subendocardial layer of the
endocardium.

Question:
A 45-year-old male reports to your office stating that he "feels his heart beating". He says that the
sensations only occur at nighttime or after a prolonged physical exhaustion. His father died of
cardiovascular causes at the age of 65. His younger brother has hypertension at the age of 45. His vital
signs are as follows: BP is 135/87 mmHg, HR = 76 bpm. Auscultation revealed rhythmic S1 and S2 and
vesicular lung sounds bilaterally. You proceed with treadmill ECG in order to identify the activity, if
any, that may correspond to what the patient refers to as "feeling his heart beating". Refer to the
image. Vital signs during the treadmill were as follows: BP = 135/75 mmHg, HR= 84 bpm. What would
be the most appropriate rationale for the continuum of treatment of this patient?

Answer Choices:
A Refer to angiography
B Refer to echocardiography, possibly perform stress echocardiography
C Prescribe beta-blockers, re-evaluate in 12 weeks
D Refer to myocardial perfusion assessment
E Prescribe mexiletine, re-evaluate in 4 weeks

Image(s) / Chart(s):
Click image to view full size. Click open image to close. Click and hold open image to move.

D Refer to myocardial perfusion assessment

Patients' complaints such as "feeling of the heart beating" are often very confusing for clinicians
since they do not provide enough subjective characterization of the symptom alone nor of its
intensity. However, the fact that these disturbances occur at night and after a heavy physical
exertion, should alert you as a clinician to establish its risk for the patient.
Lets start by examining the factors. The patient is normotensive with rhythmic S1 and S2.
Patient's father died of cardiovascular causes at the age of 65. Even though it is tempting to
classify that as "family history", the use of this diagnostic criterion is confined to premature
CAD in family members (before the age of 45). In this case, the patient's father was 65 when he
died of a cardiovascular event, therefore we cannot consider this case positive for family history
of CAD. The fact that the patient's brother has hypertension at the age of 45 is most probably a
consequence of the lifestyle rather then of genetic interactions. Therefore, this patient does
not have family history of CAD in its classic formulated meaning.
However, because the patient reports the symptoms to occur after a physical exertion, one
should attempt a stress (treadmill) ECG in order to possibly provoke the same symptoms and to
record the correspondent ECG activity. The ECG of this patient clearly shows that the "feeling of
the heart beating" in this case is PVCs. One of the major reasons of PVCs during exercise is the
inadequate myocardial perfusion, possibly by the artery with a flow-limiting stenosis. To rule
that assumption out, one must refer such a patient for the assessment of myocardial perfusion,
which is routinely done with a 201-thallium or 201-thallium-99-Technecium SPECT. When
prescribing the technique of perfusion assessment, one should keep in mind that as a clinician
you will also receive the global myocardial function parameters in the SPECT report. That is why
such a technique is very useful in this case. Sensitivity and specificity for the identification of
CAD has been established for SPECT.
Referral for coronary angiography without preceding SPECT may result in an unnecessary
interventional procedure, patient's exposure, risk and increased costs for the institution should
the angiographic assessment turn out normal. Stress echocardiography, since it is performed
with administration of dobutamine, is unlikely to provide any more additional information then
the treadmill ECG and SPECT. For the most part, the diagnostic information may be duplicated:
if PVCs are provoked by exercise, they will be provoked by dobutamine. In addition, stress
echocardiography may be a little bit more risky in this patient since dobutamine is likely to
provide a larger adrenergic stimulus then exercise and the patient may have a sustained
arrhythmia. Moreover, perfusion abnormalities seen on SPECT may not be translated into wall
motion abnormalities since a critical reduction of blood flow is necessary for a wall motion
abnormality to occur.
In spite of that, a reduction in blood flow, which does not have the magnitude to cause a wall
motion abnormality, may be potent enough to provoke PVCs. For this reason, myocardial
perfusion imaging is advocated in such cases. Even though PVCs respond well to either beta-
blockers or specific anti-arrhythmics such as mexiletine, prescribing treatment without
diagnosing the abnormality and finding its cause does not provide any risk stratification for this
patient, and such approach without performing tests first is not advocated.

Question: CLASS QUIZ

Examine the labeled photomicrograph of the endocardial region of the normal human cardiac
ventricle. What is the MOST appropriate description of the function of the cells labeled A?

Answer Choices:
A Expulsion of blood from ventricle by contraction
B Prevents clotting of blood
C Binds cardiac myocytes together
D Conducts contraction initiating impulses from atrioventricular bundle (of His)
E Carries blood to heart muscle

Image(s) / Chart(s):
Click image to view full size. Click open image to close. Click and hold open image to move.

D Conducts contraction initiating impulses from atrioventricular bundle (of His)

The cells labeled A are Purkinje cells. They are modified cardiac myocytes. They have a cell
diameter approximately twice the diameter of cardiac myocytes (area B). Contraction of these
more abundant cardiac myocytes causes ventricular ejection. Immediately adjacent to the
blood-filled lumen of the ventricles, there is a smooth simple squamous epithelium, which
prevents blood clotting. Connective tissue fibroblasts bind cardiac myocytes to one another.
There are also strong intercalated discs (intercellular junctions) that attach adjacent cardiac
myocytes to one another. Branches of the coronary arteries ramify all throughout the cardiac
muscle, carrying oxygenated blood to the heart muscle.

Coordination of contraction of different parts of the heart so it can function efficiently as a


rhythmic pump occurs from modified myocytes in the sinoatrial node, the atrioventricular
node, and the atrioventricular bundle (of His). The bundle of His contains Purkinje fibers, which
then ramify into the ventricle. The pale perinuclear region of the Purkinje fibers contains
numerous glycogen granules. Purkinje cells also have peripheral bundles of myofibrils.

Question: BLOCK

Examine the labeled photomicrograph of normal human cardiac myocytes. What is the MOST
appropriate description of the intercellular junctions (white arrow) between the cells?

Answer Choices:
A Contains hemidesmosomes
B Contains only gap junctions
C Is a place of cytoplasmic continuity between adjacent cells, creating a syncytium
D Has desmosomes, gap junctions, and fasciae adherentes
E Blocks conduction of impulses from one cell to another

Image(s) / Chart(s):
Click image to view full size. Click open image to close. Click and hold open image to move.

D Has desmosomes, gap junctions, and fasciae adherentes

The arrow labels an intercalated disc, the robust intercellular junction between individual
cardiac myocytes. Individual cardiac muscle cells have ridges and bulges at the intercalated disc
(D), with the projections on the end of one cell fitting into the depressions on the face of the
adjacent cell. At this interface, there are desmosomes (not hemidesmosomes). There are also
gap junctions at the intercalated discs. These gap junctions allow excitatory (contraction
initiating) impulses to spread rapidly throughout the heart muscle. Intercalated discs do not
block passage of contracile impulses. Finally, there are broad areas of close apposition of cell
membranes of adjacent cells, called fasciae adherentes, which bind cells together along with
the desmosomes. In spite of these robust intercellular junctions, cardiac muscle cells do not
form a morphological syncytium. Each branched cell is bounded by a continuous cell membrane
that separates it from its neighbors.

SECTION- life cycle of blood cells (heamopoesis)


Question:
Identify the cell located at the tip of the pointer in this microphotographic image

Answer Choices:
A Mature erythrocyte
B Reticulocyte
C Acidophilic erythroblast
D Polychromatophilic erythroblast
E Platelet

Image(s) / Chart(s):
Click image to view full size. Click open image to close. Click and hold open image to move.
B Reticulocyte

In the process of erythropoiesis, a mature erythrocyte is formed from relatively large, nucleated
progenitor cells. During the process of maturation, the cell size is reduced and the nucleus is
extruded from the cell. Also, the cytoplasm is filled with newly synthesized hemoglobin. Shortly
following the time at which the nucleus is extruded from the cell, the cytoplasm of the new
erythrocyte, called the reticulocyte, shows a slight pinkish-gray tint upon staining rather than
the characteristic reddish stain of the mature red blood cell. The grayish tint of the new red
blood cell is due to remaining ribosomal protein within the cytoplasm, which will diminish with
increasing maturity of the cell. Inspection of the cell at the tip of the pointer in this image
reveals that the cytoplasm of this cell is not as "eosinophilic" as the surrounding erythrocytes,
which is indicative of the reticulocyte. Several small acidophilic erythroblasts (normoblast) can
also be seen in this image. These are cells differentiating into erythrocytes just prior to the loss
of their nucleus. A polychromatophilic erythroblast, which is not seen in the field of this image,
is a more immature stage in the development of the erythrocyte and characteristically has a
larger nucleus with extensive heterochromatin and variable staining cytoplasm due to the
changing proportions of cytoplasmic concentrations of ribosomal nucleoprotein and newly
synthesized hemoglobin.

Question:
In this photomicroscopic image of a bone marrow smear, the "paired" cells are best described as
which one of the following cells?

Answer Choices:
A Normoblasts
B Polychromatophilic erythroblasts
C Basophilic erythroblasts
D Reticulocytes
E Plasma cells

Image(s) / Chart(s):
Click image to view full size. Click open image to close. Click and hold open image to move.
B Polychromatophilic erythroblasts

During the process of erythropoiesis, the cell becomes progressively smaller, the nucleus more
dense and the cytoplasm becomes increasingly acidophilic as it accumulates hemoglobin. The
"paired cells" in this image represent polychromatophilic erythroblasts. The nucleus of this cell
shows a dense chromatin network with scattered coarse clumps of chromatin. Since this cell is
also involved in synthesis of hemoglobin, the staining of islands of cytoplasm varies from a deep
bluish-gray to a light-slate gray, or even a reddish tint. This polychromatophilia represents the
decreasing concentration of ribosomes in the cytoplasm (staining basophilic), and the
increasing accumulation of hemoglobin (staining more acidophilic). These cells are capable of
mitosis, as seen in this image and would be indicative of "late" polychromatophilic
erythroblasts as based upon their size, cytoplasmic staining characteristics, and nuclear profile.

Question:
The cell identified by the letter "A" in the photomicroscopic image of a bone marrow smear is best
identified as what cell type?

Answer Choices:
A Neutrophilic metamyelocyte
B Myeloblast
C Eosinophilic myelocyte
D Neutrophilic myelocyte
E Promyelocyte

Image(s) / Chart(s):
Click image to view full size. Click open image to close. Click and hold open image to move.
E Promyelocyte

The cell depicted by the letter "A" in the image is best identified as a promyelocyte. In the
process of granulocytopoiesis, granulocytes accumulate granules and the nucleus becomes
progressively flattened, indented, and eventually lobulated. The promyelocyte is characterized
by a larger cell, between 15-20μm diameter, in which the basophilic cytoplasm contains a
variable number of very definitive, purplish azurophil granules. The nucleus is somewhat oval
and occupies much of the cell. The nucleus stains lightly and frequently one or more nucleoli
may be identified. This is the last "non-committed" developmental stage: granulocytic
development diverges into three distinct cell lines following the promyelocyte stage.

Question:
Lymphocytes are being continuously exposed to situations that could induce apoptosis, and to protect
against inadvertent cell death they possess a separate set of proteins that inhibit programmed cell
death. Some of these proteins were discovered as oncogenes. Using cloning techniques, the genes for
these proteins can be isolated and the proteins produced and purified. What is one example of these
proteins?

Answer Choices:
A CXCR4
B Mac-1
C SDF-1
D Bcl-3
E IL-8

D Bcl-3

BcL-3, originally identified as a putative oncogene, is a member of the IκB multigene family,
and Bcl-3-deficient mice are defective in antigen-specific antibody production, germinal center
formation, and fail to resist infection. Bcl-3 also contributes to B cell survivaland is related to a
number of genes involved in cell cycle control and cell lineage determination. Bcl-3 expression
is induced by mitogenic stimuli in T and B cells, as well as by GM-CSF, EPO, and IL-9.
CXCR4 is a major co-receptor for HIV. Lymphotropic HIV uses CXCR4 found on T cells and
requires a high density of CD4 on the cell surface. CXCR4 also binds the CXC-chemokine
stromal-derived factor-1 (SDF-1) as a co-receptor. Both co-receptors are G-coupled proteins
with seven transmembrane spanning domains.
Mac-1, also called CD11b/CD18, is stored in specific granules that are shuttled to the
granulocyte surface. It exists as a chemoattractant activation-dependent molecule that
undergoes a conformational change upon stimulation. Until stimulation occurs, it remains in a
resting, non-adhesive state. Mac-1 is a β2 integrin.
SDF-1 is a small chemokine of the CXC subfamily that is produced constitutively by bone
marrow stromal cells and has an important role in early stages of B cell development. It is a
growth factor for B cell progenitors and a chemotactic factor for T cells, monocytes, CD34+
hematopoietic progenitor cells, mature megakaryocytes, and dendritic cells.
Interleukin-8 or IL-8, a CXC chemokine, is an 11 kDa homodimer of 99 amino acids that is also
known as monocyte-derived neutrophil chemotactic factor (MDNCF). It is an important
mediator of neutrophil, lymphocyte, and basophil chemotaxis and activation. It is released from
several cell types in response to an inflammatory response.

Keywords: cell death, apoptosis


Question:
Identify the cells located by the arrows in the photomicroscopic image.

Answer Choices:
A Lymphocytes
B Normoblasts
C Plasma cells
D Platelets
E Reticulocytes

Image(s) / Chart(s):
Click image to view full size. Click open image to close. Click and hold open image to move.

B Normoblasts
These cells appear similar to red blood cells, but contain eccentrically located, small, dark,
pyknotic nuclei. The cell is slightly larger than the surrounding red blood cells, and the
cytoplasm exhibit may appear more "basophilia" than the surrounding corpuscles. These cells
represent immature red blood cells immediately prior to the period when the nucleus is
extruded from the cell, leaving the erythrocyte as a non-nucleated cell with cytoplasm filled
with hemoglobin. This stage, as depicted by the cells in the photomicroscopic image are
regarded as normoblasts, or acidophilic erythroblasts.

Question:
The cells identified by the letter "A" in the photomicroscopic image of a bone marrow smear is best
identified as what cell type?

Answer Choices:
A Neutrophilic metamyelocyte
B Eosinophilic band
C Eosinophilic myelocyte
D Neutrophilic band
E Normoblast

Image(s) / Chart(s):
Click image to view full size. Click open image to close. Click and hold open image to move.

D Neutrophilic band

The cells depicted by the letter "A" in the image are best identified as neutrophilic band cells. In
the process of granulocytopoiesis, granulocytes accumulate granules and the nucleus becomes
progressively flattened, indented, and eventually lobulated. In the case of the neutrophil, the
cytoplasm accumulates a predominance of specific granules, and the nucleus progresses
through the indentation process. At the stage seen in the image, the nucleus of the neutrophilic
band presents with the same general morphology as the mature neutrophil, except that the
nucleus is curved or twisted, but has not become lobulated.

Question:
The differentiation and maturation of erythrocytes involve various developmental stages. In one of
the following stages, the basophilic ribosomes decrease and areas of cytoplasm begin to be filled with
hemoglobin. Staining at this stage causes several colors to appear in the cell. This stage is known as

Answer Choices:
A Proerythroblast
B Basophilic erythroblast
C Orthochromatophilic
D Polychromatophilic erythroblast
E Myeloblast

D Polychromatophilic erythroblast

Polychromatophilic erythroblast, as the name implies, is the stage during which the cell
acquires hemoglobin, and if stained at this stage, causes several colors to appear.
The first recognizable cell in the erythroid series is the proerythroblast. The next stage is
represented by the basophilic erythroblast, with a strongly basophilic cytoplasm and a
condensed nucleus. In the next stage, it develops into a polychromatophilic erythrocytes. In the
next step, the nucleus continues to condense and no cytoplasmic basophilia is evident,
resulting in a uniformly acidophilic cytoplasm -- the orthochromatophilic erythroblast.
The myeloblast is the most immature recognizable cell in the myeloid series.

Copyright © EXAM MASTER Corporation 2014 Author ID: 38


Question:
Expression of various cell-surface molecules is critical to cell-cell interaction. Fas or CD95 is a
membrane glycoprotein belonging to the tumor necrosis/nerve growth factor receptor family. In
some lymphoid cell lines, what can Fas trigger?

Answer Choices:
A Clonal deletion
B Gene duplication
C Apoptosis
D Molecular mimicry
E Gene conversion

C Apoptosis
The understanding that a programmed pattern may control cell death, as is known to be the
case for cell differentiation and proliferation, is a relatively recent development in biology.
Apoptosis, or programmed cell death, proceeds in four phases: the initiation phase, the
detection phase, the effector phase, and the postmortem or degradation phase, when the vital
structures and functions are destroyed giving rise to alterations used to define and identify this
type of cell death. Cell death is controlled by the elements of a core genetic program that
seems to be constitutively expressed in virtually every cell. Apoptotic cells have specific
biochemistry and morphology, and the most rigorous standards for definition of apoptosis
remain chromatin condensation, cytoplasmic vacuolization, and plasma membrane blebbing.
Fas or CD 95 is constitutively expressed by T cells and other cell types and is an important
regulator of apoptosis.
Antigen binding to self-molecules in the bone marrow (B cells) and the thymus (T cells) can lead
to the death or inactivation of immature cells. When developing lymphocytes express receptors
that recognize ubiquitous self cell-surface molecules, such as those of the MHC, they are
deleted from the immunological repertoire. This process is known as clonal deletion.
Gene duplication is the creation of an extra copy of a gene. This is a key mechanism in
evolution. Once a gene is duplicated, the identical genes can undergo changes and diverge to
create two different genes. Duplications typically arise from an event termed unequal crossing-
over that occurs between misaligned homologous chromosomes during meiosis.
If a microbial antigen is very similar to normal host antigens, the immune response to this
antigen may be weak or absent, giving a degree of tolerance. The mimicking of host antigens by
microbial antigens is referred to as molecular mimicry. There is evidence that antibodies
formed against microorganisms sometimes cross-react with host tissues and cause disease.
In the generation of polymorphism, several genetic mechanisms contribute to the generation of
new alleles. Some new alleles are the result of point mutations but many arise by gene
conversion, in which one sequence is replaced in part by another from a homologous gene.

Keywords: programmed cell death, Fas, apoptosis


Question:
Lymphocytes are being continuously exposed to situations that could induce apoptosis, and to protect
against inadvertent cell death they possess a separate set of proteins that control programmed cell
death. Some of these genes inhibit cell death and others promote it. What is an example of the death-
inhibiting genes?

Answer Choices:
A Bcl-2
B CXCR4
C MAC-1
D SDF-1
E TLR-4
A Bcl-2

The balance between death-promoting and death-inhibiting gene expression is critically


important in lymphocytes, because lymphocyte populations are regulated so that an individual
will, in the absence of infection, maintain a constant level of T and B cells despite the
production and death of many lymphocytes each day. The Bcl-2 family of proto-oncogenes is a
critical regulator of apoptosis, whose expression frequently becomes altered in human cancers,
including lymphomas. Bcl-2 was the first member to be identified, by virtue of its involvement
in chromosomal translocation commonly found in B cell non-Hodgkin's lymphoma. The Bcl-2
family members are regulated on the transcriptional level and by post-translational
mechanisms.
CXCR4 is a major co-receptor for HIV. Lymphotropic HIV uses CXCR4 found on T cells and
requires a high density of CD4 on the cell surface. CXCR4 also binds the CXC-chemokine
stromal-derived factor-1 (SDF-1) as a co-receptor. Both co-receptors are G-coupled proteins
with seven transmembrane spanning domains.
Mac-1, also called CD11b/CD18, is stored in specific granules that are shuttled to the
granulocyte surface. It exists as a chemoattractant activation-dependent molecule that
undergoes a conformational change upon stimulation. Until stimulation occurs, it remains in a
resting, non-adhesive state. Mac-1 is a β2 integrin.
SDF-1 is a small chemokine of the CXC subfamily that is produced constitutively by bone
marrow stromal cells and has an important role in early stages of B cell development. It is a
growth factor for B cell progenitors and a chemotactic factor for T cells, monocytes, CD34+
hematopoietic progenitor cells, mature megakaryocytes, and dendritic cells.
Toll-like receptors or TLRs are involved in innate immunity by recognizing invading microbes
through pathogen-associated molecular patterns. TLRs are expressed on macrophages and
dendritic cells and mediate the signal for cytokine release or up-regulation of co-stimulatory
molecules, which together contribute to the generation of host immune responses. One of the
TLRs, TLR-4, is an important mediator of cellular responses to lipopolysaccharide (LPS).

Keywords: apoptosis, cell death, lymphocyte cell cycle


Question:
The differentiation of all types of blood cells occurs in the

Answer Choices:
A Liver
B Erythrocyte
C Bone marrow
D Granulocyte
E Lining of the intestine

C Bone marrow
A stem cell is a cell which can continuously divide in vivo. It produces daughter cells which
either remain as stem cells or differentiate into specialized cells. Stem cells serve as the source
for the production of differentiated cells throughout the organisms lifespan. Examples of
differentiated cells originating from stem cells are blood cells, epithelial cells of the skin, and
epithelial cells lining the intestine. The pluripotent stem cell in the bone marrow gives rise to
precursor cells which eventually differentiate into mature blood cells including lymphocytes,
granulocytes, and erythrocytes.

Question:
A stem cell is best characterized by which of the following statements?

Answer Choices:
A It is a fully differentiated cell
B It can continue to divide
C It contains half the normal complement of chromosomes
D It differentiates into lymphocytes only
E It has a limited life span

B It can continue to divide

A stem cell is a cell which can continuously divide in vivo. It produces daughter cells which
either remain as stem cells or differentiate into specialized cells. Stem cells serve as the source
for the production of differentiated cells throughout the organisms lifespan. Examples of
differentiated cells originating from stem cells are blood cells, epithelial cells of the skin, and
epithelial cells lining the intestine. The pluripotent stem cell in the bone marrow gives rise to
precursor cells which eventually differentiate into mature blood cells including lymphocytes,
granulocytes, and erythrocytes.
Question:
The pluripotential stem cells that give rise to monocytes are known as

Answer Choices:
A CFC-E
B CFC-Meg
C CFC-GM
D CFC-Bas
E CFC-Eosin

C CFC-GM
The myeloid stem cell, a CFU (colony-forming units), will produce progeny that may
differentiate into CFC (colony forming cells) that give rise to several types of blood cells or to a
single type of blood cells. Eventually, CFCs for each mature blood cell type are produced. These
are designated CFC-E for erythrocytes, CFC-Meg for megakaryocytes, CFC-GM for monocyte
and/or neutrophil-producing cells, CFC-Bas for basophils, and CFC-Eosin for acidophils.

Question:
The life-span of a neutrophil inside the connective tissue may last for

Answer Choices:
A 8-12 hours
B 1-2 days
C 120 days
D 7-10 days

B 1-2 days

Neutrophils live for 1-2 days in the connective tissue, after which they are destroyed by
macrophages.
The mature granulocytes circulate in the peripheral blood for about 8-12 hours, after which
they leave the blood to enter the perivascular connective tissue. Red blood cells have a life span
of about 120 days, after which they become fragile and subject to breakage. Platelets have a
life-span of 7-10 days.
Question:
Which of the following cell types is capable of undergoing many rounds of DNA replication without
cell division occurring?

Answer Choices:
A Lymphocytes
B Plasma cells
C Platelets
D Neutrophils
E Megakaryocytes

E Megakaryocytes

The plasma cells are derived from resting B lymphocytes and are responsible for antibody
secretion. They occur mainly in secondary lymphoid tissue. The plasma cells are characterized
by an abundance of rough endoplasmic reticulum that is required for antibody synthesis and
secretion.
Megakaryocytes are giant cells found in the bone marrow and are the source of circulating
platelets. The megakaryocyte undergoes many rounds of DNA replication without undergoing
cell division. Once DNA synthesis has stopped, the cytoplasm of the cell matures.
Platelets bud from the main cell body of the megakaryocyte. Platelets lack nuclei and function
in hemostasis, coagulation, and inflammation.
The release of histamine and/or serotonin occurs to mediate inflammation. The cytoplasm of
the small or "resting" lymphocyte is taken up mostly by the nucleus. When lymphocytes
contact antigen, activation signals trigger development into large lymphocytes or plasma cells
in the case of B lymphocytes.
Neutrophils are phagocytic cells which are recruited to the sites of inflammation.
Histamine is secreted by mast cells, basophils and platelets.
Question:
Which one of the following blood cells arises from the monocytes?

Answer Choices:
A Plasma cells
B Lymphocytes
C Acidophils
D Neutrophils
E Macrophages

E Macrophages

Macrophages derive mainly from precursor cells from the bone marrow that divide, producing
monocytes that circulate in the blood. In a second step, these cells migrate into the connective
tissue where they mature and are called macrophages. Tissue macrophages can proliferate
locally, producing more macrophages.
Plasma cells originate from B-lymphocytes, while eosinophils (= acidophils) and neutrophils
arise from myeloblasts in the myeloid tissue.
estion:
The cell depicted in this photomicroscopic image of a bone marrow smear is best identified as which
one of the following cells?

Answer Choices:
A Proerythroblast
B Basophilic erythroblast
C Early polychromatophilic erythroblast
D Late polychromatophilic erythroblast
E Acidophilic erythroblast

Image(s) / Chart(s):
Click image to view full size. Click open image to close. Click and hold open image to move.

C Early polychromatophilic erythroblast

During the process of erythropoiesis, the cell becomes progressively smaller, the nucleus more
dense and the cytoplasm becomes increasingly acidophilic as it accumulates hemoglobin. The
cell in this image represents an early polychromatophilic erythroblast. The nucleus of this cell
shows a progressively dense chromatic network with scattered coarse clumps of chromatic.
Since this cell is also involved in synthesis of hemoglobin, the staining of islands of cytoplasm
varies from a deep bluish-gray to a light-slate gray, or even a reddish tint. This
polychromatophilia represents the decreasing concentration of ribosomes in the cytoplasm
(staining basophilic), and the increasing accumulation of hemoglobin (staining more
acidophilic). In contrast, the basophilic erythroblast would have deeply basophilic cytoplasm
without the interspersed islands of reddish tint.

Question:
The cell identified by the letter "B" in the photomicroscopic image of a bone marrow smear is best
identified as what cell type?

Answer Choices:
A Neutrophilic metamyelocyte
B Myeloblast
C Eosinophilic myelocyte
D Neutrophilic myelocyte
E Promyelocyte

Image(s) / Chart(s):
Click image to view full size. Click open image to close. Click and hold open image to move.
D Neutrophilic myelocyte

The cell depicted by the letter "B" in the image is best identified as a neutrophilic myelocyte. In
the process of granulocytopoiesis, granulocytes accumulate granules and the nucleus becomes
progressively flattened, indented, and eventually lobulated. The neutrophilic myelocyte is the
first stage of the "committed" granulocyte series; that is, the cell has now diverges into one of
the three granulocytic lines (neutrophil, eosinophil, basophil). The classification of the cell
lineage is dependent upon the appearance of specific granules in the cytoplasm, as can be seen
in this image. Note that the nucleus is progressing from a flattened shape to an indented shape
and the chromatin is becoming more condensed. Some myelocytes, as in this cell, may continue
to demonstrate nucleoli. The myelocyte is the last cell of the series that is normally capable of
mitosis.
Question: BLOCK EXAMS
The cell identified by the letter "b" in the photomicroscopic image of a bone marrow smear is best
identified as what cell type?

Answer Choices:
A Neutrophilic metamyelocyte
B Eosinophilic metamyelocyte
C Neutrophilic band
D Eosinophilic band
E Neutrophilic myelocyte

Image(s) / Chart(s):
Click image to view full size. Click open image to close. Click and hold open image to move.
A Neutrophilic metamyelocyte

The cell depicted as cell "b" in the image is best identified as a neutrophilic metamyelocyte. In
the process of granulocytopoiesis, granulocytes accumulate granules and the nucleus becomes
progressively flattened, indented, and eventually lobulated. At this stage, the nucleus is deeply
indented, nearly forming a horseshoe-shape. However, the nucleus has not become ribbon-like
or twisted, characteristic of the "band" cell stage. Granules can be observed in the cytoplasm
and both specific and azurophil granules are present, with a predominance of specific granules.

Question: CLASS QUIZ


In this photomicroscopic image of a bone marrow smear, the cell at the arrowhead is best described
as which one of the following cells?

Answer Choices:
A Acidophilic erythroblast
B Polychromatophilic erythroblast
C Reticulocyte
D Erythrocyte
E Lymphocyte

Image(s) / Chart(s):
Click image to view full size. Click open image to close. Click and hold open image to move.
A Acidophilic erythroblast

This cell appears similar to a red blood cell (erythrocyte), but contains an eccentrically located,
small, dark, pyknotic nucleus. The cell is slightly larger than the surrounding red blood cells, and
the cytoplasm exhibits slightly more "basophilia" than the surrounding corpuscles. These cells
represent immature red blood cells immediately prior to the period when the nucleus is
extruded from the cell (karyorrhexis), leaving the erythrocyte as a non-nucleated cell with
cytoplasm filled with hemoglobin. This stage, as depicted by the cell in the photomicroscopic
image is regarded as a normoblast, or acidophilic erythroblast. Another normoblast may also be
seen at a slightly earlier stage in the image.
Question:
The light pinkish-gray tint apparent in the cytoplasm of this cell is related to what characteristic?

Answer Choices:
A Loss of hemoglobin in the aged cell
B Loss of the nucleus in the young cell
C Secretory enzymes in the aged cell
D Production of microvesicles in the cell
E Production of hemoglobin in the young cell

Image(s) / Chart(s):
Click image to view full size. Click open image to close. Click and hold open image to move.

E Production of hemoglobin in the young cell


In the process of erythropoiesis, a mature erythrocyte is formed from relatively large, nucleated
progenitor cells. During the process of maturation, the cell size is reduced and the nucleus is
extruded from the cell. Also, the cytoplasm is increasingly filled with newly synthesized
hemoglobin. Shortly following the time at which the nucleus is extruded from the cell, the
cytoplasm of the new erythrocyte, called the reticulocyte, shows a slight pinkish-gray tint
following staining rather than the characteristic reddish-pink stain of the mature cell. The
grayish tint of the new red blood cell is due to remaining ribosomal protein within the
cytoplasm, which will diminish with increasing maturity of the cell. During this period of time,
the cell is filled with a full complement of hemoglobin, which will impart the reddish tint
characteristically seen in mature erythrocytes. Inspection of the cell at the tip of the pointer in
this image reveals that the cytoplasm of this cell is not as "eosinophilic" as the surrounding
erythrocytes, which is indicative of a reticulocyte. Several small acidophilic erythroblasts
(normoblast) can also be seen in this image. These are cells differentiating into erythrocytes just
prior to the loss of their nucleus.
Question:
Identify the cells located by the arrows in the photomicroscopic image.

Answer Choices:
A Lymphocytes
B Normoblasts
C Plasma cells
D Platelets
E Reticulocytes

Image(s) / Chart(s):
Click image to view full size. Click open image to close. Click and hold open image to move.

B Normoblasts

These cells appear similar to red blood cells, but contain eccentrically located, small, dark,
pyknotic nuclei. The cell is slightly larger than the surrounding red blood cells, and the
cytoplasm exhibit may appear more "basophilia" than the surrounding corpuscles. These cells
represent immature red blood cells immediately prior to the period when the cytoplasm is
extruded from the cell, leaving the erythrocyte as a non-nucleated cell with cytoplasm filled
with hemoglobin. This stage, as depicted by the cells in the photomicroscopic image are
regarded as normoblasts, or acidophilic erythroblasts.

Question:
Interleukin-2 (IL-2) stimulates the proliferation of

Answer Choices:
A Fibroblasts
B Activated T cells
C Macrophages
D Eosinophils
E Specific classes of neurons

B Activated T cells

Cyclosporin A (cyclosporine) is a cyclic 11 residue polypeptide derived from fungi. It is a widely


used drug for patients receiving organ transplants because it is a powerful immune-suppressant
preventing allograft rejection. It is also useful in the area of autoimmune diseases. It acts by
two mechanisms to block the activation and proliferation of T lymphocytes, the immune cells
mediating rejection. Cyclosporin A's target is the calcium-sensitive protein phosphatase,
calcineurin, also known as protein phosphatase IIB. This blocks the synthesis of interleukin-2 (IL-
2) by blocking the calcium-activated pathway, leading to its transcription. IL-2 is a 15,000 dalton
protein that stimulates the proliferation of activated T cells and B cells. It binds to a specific
receptor found on T cells. The expression of these IL-2 receptors on helper and cytotoxic T cells
is also inhibited by cyclosporin A, resulting in selective prevention of the differentiation and
proliferation of the cytotoxic cells. Interleukin-3 (IL-3) in a 25,000 dalton protein made by T cells
stimulates the proliferation of various hematopoietic cells.
Question:
Apoptosis represents physiological as opposed to pathological (necrotic) cell death - it is the cellular
equivalent of suicide. Apoptotic cells are recognized by phagocytes and removed. In vitro apoptotic
cells can be detected by a staining procedure called

Answer Choices:
A MAGE
B TLR
C MBL
D RANTES
E TUNEL
E TUNEL

Normal development and health require the precise regulation of cell death. Human cells can
self-destruct via an intrinsic program of cell death. Apoptosis is a form of programmed cell
death that is characterized by specific morphologic and biochemical properties.
Morphologically, apoptosis is a series of structural changes in dying cells: blebbing of the
plasma membrane, condensation of the cytoplasm and nucleus, and cellular fragmentation
into membrane apoptotic bodies.
Apoptotic cells are detected by a procedure known as TUNEL staining - this stands for terminal
deoxyribonucleotidyl transferase (TdT)-mediated dUTP nick end labeling. In this technique, the
3' ends of the DNA fragments generated in apoptotic cells are labeled with biotin-coupled
uridine by using the enzyme terminal deoxynucleotidyl transferase. The biotin label is then
detected with enzyme tagged streptavidin, which binds to biotin. When the colorless substrate
of the enzyme is added to a tissue section or cell culture, it is reacted upon to produce a
colored precipitate only in cells that have undergone apoptosis. The technique has
revolutionized the detection of apoptotic cells.
Some tumor antigens are derived from normal genes that, under normal circumstances, are
programmed to be expressed only during embryogenesis. Examples of these
oncodevelopmental tumor antigens include the melanoma-associated antigen, or MAGE family
of proteins that are not expressed in any normal adult tissues, except for the testes.
Toll-like receptors or TLRs are involved in innate immunity by recognizing invading microbes
through pathogen-associated molecular patterns. TLRs are expressed on macrophages and
dendritic cells and mediate the signal for cytokine release.
Acute inflammation is generally accompanied by a systemic acute-phase response. One of the
proteins produced in this response is mannan-binding lectin (MBL). It is found in normal serum
at low levels but is produced in increased amounts during the acute-phase response. It is a
calcium-dependent sugar-binding protein, or lectin.
Chemokines fall mainly into two related groups, CC and CXC chemokines. CC chemokines such
as RANTES promote the infiltration into tissues of a range of leukocyte cell types including
effector T cells, with individual chemokines acting on different subsets of cells.

Case #301550:
Very worried parents bring their 3-day-old child to your office because of the prolonged
umbilical bleedings, blood in the stool, and easy bruising on the skin. The baby was born at
home, and the mother had no prenatal care. She did not take any medications during
pregnancy. Family history is unremarkable.
Question:
What could hemorrhagic diathesis in this child be a consequence of?

Answer Choices:
A Folate deficiency
B Vitamin K deficiency
C Breast feeding
D Child abuse
E Umbilical cord infection

B Vitamin K deficiency

Newborns are particularly vulnerable to hemorrhagic disorders because: transplacental transfer


of vitamin K is very limited during pregnancy; breast milk is low in vitamin K; the neonatal gut is
sterile during the first few days of life; the neonatal liver is immature with respect to
prothrombin synthesis; and the storage of vitamin K in neonatal liver is also limited. Exogenous
vitamin K is given for prevention of bleeding immediately after birth. This child was born at
home and did not receive vitamin K.

Folate deficiency in mother will more likely result in low birth weight, prematurity, and/or
neural tube defects.

Breast feeding when the mother did not and does not take any drugs cannot be the cause of
hemorrhagic diathesis.

Bleeding from the umbilicus and gut does not indicate child abuse.

Signs of umbilical cord infection are foul-smelling, yellow drainage from the cord, and redness
and tenderness of the skin around the cord. Those signs your patient does not have.
Case #301551:
A 2-month-old, breast-fed female infant presents with low hemoglobin levels. The infant was
born at home, and the mother received no prenatal care. She did not and does not take any
medications. Family history is unremarkable. On examination, baby appears healthy.
Question:
What is the underlying mechanism of low hemoglobin levels in the baby?

Answer Choices:
A Folate deficiency
B Vitamin K deficiency
C Breast feeding
D Infection
E Normal process

E Normal process
Intrauterine hypoxia stimulates erythropoietin. Infants are born with high levels of hemoglobin
and erythrocytes in their blood, which down-regulates erythropoietin. Progressive drop in
hemoglobin during the first 2-3 months is the signal to the infant's body to increase
erythrocytes production. Called physiologic anemia of infancy, this is considered normal and no
treatment is needed.

Folate deficiency in the mother would more likely result in the low birth weight, prematurity,
and/or neural tube defects.

Vitamin K deficiency is the cause of hemorrhagic diathesis. Its manifestations that could
possibly cause anemia would be obvious and reported.

Breast feeding when the mother did not and does not take any drugs cannot be the cause of
anemia in a 2-month-old baby.

Common infections in a 2-month-old baby include infection of the digestive tract, ear, skin, or
upper respiratory tract. All will have relevant symptoms, and this baby appears healthy.
Case #301553:
This photomicrograph is from a hematoxylin and eosin stained section of the normal human
fetal liver. In the middle, there is a large cluster of small cells with eosinophilic cytoplasm,
most of which have nuclei.
Question:
These nucleated cells similar to the cell labeled A are MOST likely what cells?

Answer Choices:
A megakaryocytes
B Kupffer cells
C eosinophilic metamyelocytes
D nucleated fetal erythrocytes
E lymphocytes

Image(s) / Chart(s):
Click image to view full size. Click open image to close. Click and hold open image to move.
D nucleated fetal erythrocytes

During embryonic development, there are 3 major sites of hematopoiesis. First, the yolk sac is
the site of production of erythrocytes. Next, the liver plays an important role in erythropoiesis,
gradually supplanting the yolk sac. Finally, once bones have formed and developed marrow
cavities, these become the most important site of fetal (and postnatal) hematopoiesis,
replacing hepatic blood cell formation. The clear, agranular, highly eosinophilic cytoplasm of
the large cluster of cells in the center of the photomicrograph indicate that these cells are
primitive, nucleated fetal erythrocytes.

Megakaryocytes are much larger cells. When they fragment in bone marrow, they release
platelets into the peripheral circulation.

Kupffer cells are phagocytic cells lining the sinusoids of the adult liver.

Partially differentiated eosinophilic metamyelocytes would have large eosinophilic (specific)


granules in their cytoplasm.

Lymphocytes would have a darkly stained nucleus, but their cytoplasm, although agranular,
would stain purple with hematoxylin.
Case #301554:
This photomicrograph is from a hematoxylin and eosin stained section of the normal human
fetal liver. In the middle, there is a large cluster of small cells with eosinophilic cytoplasm,
most of which have nuclei.
Question:
What is the large cell labeled B most likely called?
Answer Choices:
A a megakaryocyte
B a primitive hematopoietic stem cell
C a basophilic erythroblast
D a hepatic parenchymal cell
E a Kupffer cell

Image(s) / Chart(s):
Click image to view full size. Click open image to close. Click and hold open image to move.

B a primitive hematopoietic stem cell

During embryonic development, there are 3 major sites of hematopoiesis. First, the yolk sac is
the site of production of erythrocytes. Next, the liver plays an important role in erythropoiesis,
gradually supplanting the yolk sac. Finally, once bones have formed and developed marrow
cavities, these become the most important site of fetal (and postnatal) hematopoiesis,
replacing hepatic blood cell formation. The large cell at B has a largely euchromatic nucleus
with a suggestion of a nucleolus and cytoplasm without any granularity. It is most likely a
primitive hematopoietic stem cell. The clear, agranular, highly eosinophilic cytoplasm of the
large cluster of cells in the center of the photomicrograph indicate that these cells are primitive,
nucleated fetal erythrocytes.

Megakaryocytes are much larger cells. When they fragment in bone marrow, they release
platelets into the peripheral circulation.

A basophilic erythroblast would have a bright blue cytoplasm due to the accumulation many
polyribosomes used for hemoglobin synthesis.

The large polygonal, nucleated purple cells arranged in plates and cords in the edges of the
photomicrograph are hepatic parenchymal cells. They have no direct role in fetal
hematopoiesis.

Kupffer cells are phagocytic cells lining the sinusoids of the adult liver. Partially differentiated
eosinophils would have large eosinophilic (specific) granules in their cytoplasm. Lymphocytes
would have a darkly stained nucleus, but their cytoplasm, although agranular, would stain
purple with hematoxylin
Question: CLASS QUIZ
You are required to biopsy the bone marrow of a patient that you suspect may have a
myoproliferative disease. What is the best location to find and sample red marrow that is
hematopoietically active?

Answer Choices:
A Femoral diaphysis
B Iliac crest
C Humeral diaphysis
D Patella
E Femoral metaphysis

B Iliac crest

Bone marrow biopsy requires passing an instrument through the soft tissues surrounding the
bone, followed by penetrating the cortical bone on the peripheral part of the bone of interest,
and finally sampling some bone marrow. The iliac crest is close to the surface of the body and
has little soft tissue over it. In addition, there is plenty of red marrow (hematopoietically active)
in this location. The diaphysis of bones such as the femur and the humerus are surrounded by
more soft tissue, have a much thicker cortex of compact bone, and, in normal patients, contain
yellow marrow, which is hematopoietically inactive. The patella is close to the surface of the
body, but has a very thick cortex. The femoral metaphysic contains red marrow, but is deeper
than the major amount of soft tissue.
Question:
You are required to biopsy the bone marrow of a patient that you suspect may have hemolytic
anemia. In a normal patient, what would be the approximate ratio of myeloid to erythroid cells?

Answer Choices:
A 1:1
B 2:1
C 3:1
D 4:1
E 5:1
C 3:1

Sampling of bone marrow by biopsy followed by histological study of the specimen gives a
different perspective on the architecture and condition of bone marrow when compared to a
smear of bone marrow cells. For example, bone marrow biopsies allow an estimate of the ratio
of red marrow to yellow marrow in the patient. For example, in normal adults, there are
approximately equal amounts of yellow and red marrow in most specimens. However, in a
patient with hemolytic anemia, there will be little or no yellow marrow because of an increase
in the proportion of red marrow due to the need for the production of many new erythrocytes
to replace those lost due to hemolysis of erythrocytes.

Normal marrow contains about 60% of its cells in the granulocyte series but only about 20% of
its cells in the erythrocyte series, yielding a myeloid:erythroid ratio of about 3:1. Any significant
distortion of this ratio from the normal 3:1 is almost certainly an indication of some disease of
the blood and/or bone marrow.
Question:
You are required to biopsy the bone marrow of a patient that you suspect may have hemolytic
anemia. In a normal patient, what is the best way to distinguish a myeloid blast cell from an erythroid
blast?

Answer Choices:
A By determining cell diameter
B By counting number of nucleoli
C By identifying secondary granules in cytoplasm
D By evaluating the degree of nuclear condensation and lobulation
E By evaluating other nearby cells

E By evaluating other nearby cells

Sampling of bone marrow by biopsy followed by histological study of the specimen gives a
different perspective on the architecture and condition of bone marrow when compared to a
smear of bone marrow cells. For example, bone marrow biopsies allow an estimate of the ratio
of red marrow to yellow marrow in the patient. For example, in normal adults, there are
approximately equal amounts of yellow and red marrow in most specimens. However, in a
patient with hemolytic anemia, there will be little or no yellow marrow because of an increase
in the proportion of red marrow due to the need for the production of many new erythrocytes
to replace those lost due to hemolysis of erythrocytes.

The blast cells for both the myeloid series and the erythroid series are quite similar and
especially difficult to distinguish in sections. For example, both have several prominent nucleoli.
Determination of cell diameter is unreliable, especially in bone marrow sections. Blast cells
have no secondary granules and no nuclear condensation or lobulation. However, each type of
undifferentiated cells is typically surrounded by later stages in either the myeloid or the
erythroid series. Therefore, if you look carefully at the surrounding cells in the immediate
vicinity of the blast cell in question, you can be assured that if it is surrounded by myelocytes
and metamyelocytes, then it is in myeloid series; whereas, if it is surrounded by basophilic
erythroblasts and polychromatophilic erythroblasts, then it is in the erythroid series.
Question:
A baby is born with severe bone marrow depression. After careful investigation, you conclude that
the mother’s exposure to a specific fetal bone marrow toxin during her pregnancy is the cause. When
did that exposure most probably happen?

Answer Choices:
A Week 1-2
B Week 5-6
C Month 1-2
D Month 2-3
E Month 5-6

E Month 5-6

Bone marrow hematopoiesis starts between 5-6 months in utero. After birth, all bones are
involved in hematopoiesis. Therefore, bone marrow toxic effect is seen during the first 5-6
months.
Embryonic hematopoiesis starts within the first 10 days in the yolk sac. Bones are not present
yet.
The liver plays the major role in hematopoiesis starting from the first 2-3 months, although
some small amount is also seen in the spleen, thymus and lymph nodes. Bone marrow is not
present yet.
References:
SECTION -BONE MARROW
Question:
The cells depicted by the arrows in the photomicroscopic image MAY be observed in the peripheral
blood. Under what conditions would these cells be found in an adult peripheral blood smear?

Answer Choices:
A Extensive hemorrhage
B Mononucleosis
C Pregnancy
D Atrial fibrillation
E Leukocytosis

Image(s) / Chart(s):
Click image to view full size. Click open image to close. Click and hold open image to move.

A Extensive hemorrhage

The cells in the image are normoblasts (acidophilic erythroblasts) and are normally found in the
bone marrow as a late stage of erythrocyte development. This stage represents the cell
immediately prior to the loss of the nucleus and subsequent movement into the peripheral
circulation. However, under certain conditions, such as extensive hemorrhage, when
erythrocyte loss occurs, erythropoiesis is enhanced in order to produce more red blood cells,
and in response to loss of erythrocytes due to hemorrhage, nucleated red blood cells may be
seen in the peripheral blood smear.
Question:
Which one of the following is the vascular arrangement inside the young red bone marrow?

Answer Choices:
A Nutrition artery-radial arteries-sinusoids-nutrient vein
B Nutrition artery-sinusoids-nutrient vein
C Nutrient artery-central longitudinal arterioles-radial capillaries-sinuses-central
longitudinal veins-nutrient vein
D Nutrient artery-central longitudinal arterioles-radial capillaries-sinuses-nutrient vein
E Radial capillaries-sinuses-nutrient vein

D Nutrient artery-central longitudinal arterioles-radial capillaries-sinuses-nutrient vein

Nutrient artery-central longitudinal arterioles-radial capillaries-sinuses-nutrient vein, is the


correct vascular arrangement inside the young red bone marrow.
A nutrient artery enters the cavity through a large nutrient foramen in the shaft of the bone.
Other arteries typically enter the cavity near the ends of the bone. These are designated as
metaphysical arteries. At least one large nutrient vein leaves the marrow cavity through the
nutrient foramen with the nutrient artery.. Blood vessels also pass from the narrow cavity into
the compact bone of the shaft. The vessels leaving the compact bone of the shaft exit as
periosteal vessels.

Case #301590:
A 32-year-old man has had repeated heavy exposure to benzene while working in a
petroleum refinery. He develops aplastic anemia and requires an allogenic bone marrow
transplant. 10 days after bone marrow transplant, the patient develops a cytomegalovirus
pneumonia, severe skin rash, and jaundice.
Question:
What is the most likely diagnosis?

Answer Choices:
A Acute graft-versus-host (GVH) disease
B Chronic GVH disease
C Transplant rejection
D Goodpasture syndrome
E Hashimoto disease

A Acute graft-versus-host (GVH) disease

Chemical exposure can cause severe damage to the bone marrow, resulting in death of
hematopoietic stem cells. This condition can be treated by allogeneic bone marrow transplants.
This patient is immunodeficient because of his primary disease. Immunocompetent T cells in
the donor bone marrow transplant recognize the host’s HLA tissue antigens as foreign. He soon
develops acute GVH disease, characterized by skin lesions, jaundice due to destruction of small
intrahepatic bile ducts, and an opportunistic cytomegalovirus infection in his lungs. Chronic
GVH disease and transplant rejection is more severe than the acute disease but usually
develops more slowly and insidiously. Goodpasture syndrome (involving renal glomeruli) and
Hashimoto disease (involving the thyroid gland) are both autoimmune diseases with very
different symptoms from acute GVH disease.

Keywords: Bone marrow transplant, aplastic anemia, graft-versus-host disease

SECTION- THYMUS
Question:
The human immunoglobulin that is capable of crossing the placenta to provide passive immunity to
the newborn is which one of the following immunoglobulins?
Answer Choices:
A IgA
B IgM
C IgG
D IgD
E IgE

C IgG

B lymphocytes produce immunoglobulins which possess a variety of actions. One such


immunoglobulin, IgG, will cross the placenta and provide the newborn individual with passive
immunity. This is also the principle immunoglobulin produced during a secondary antibody
response. IgG has the capacity to lyse cells and bacteria.

Question:
The photomicroscopic image demonstrates a thymic or Hassall's corpuscle in the medulla of the
thymus. The inclusions observed in the cell cytoplasm are best identified as

Answer Choices:
A Keratin granules
B Lysosomes
C Ribosomes
D Lipofuscin granules
E Secretory vesicles

Image(s) / Chart(s):
Click image to view full size. Click open image to close. Click and hold open image to move.

A Keratin granules
Thymic corpuscles located in the medulla of the thymus are isolated masses of reticulo-
epithelial cells that are concentrically arranged around one another. Since these cells are
derived from the third branchial pouch and endodermal epithelial cells, they acquire keratin
granules during their life span.

Question:
The purpose of the blood-thymus barrier is to allow

Answer Choices:
A T-lymphocytes to differentiate in an environment free of foreign antigens
B T-lymphocytes to differentiate in an environment free of foreign antibodies
C T-lymphocytes to be exposed to selective antigens in the thymus
D T-lymphocytes to be exposed to selective antibodies in the thymus
E T-lymphocytes to program and differentiate into B-lymphocytes

A T-lymphocytes to differentiate in an environment free of foreign antigens

The blood-thymic barrier consists of structures which form a non-fenestrated, continuous


endothelium having a prominent basal lamina. The layers of this barrier, the perivascular
connective tissue sheath, capillary endothelium, basal lamina and reticulo-epithelial cell sheath,
form an effective barrier around blood vessels of the thymic cortex and separate circulating
blood from the differentiating T-lymphocytes. This barrier effectively allows the T-lymphocytes
do differentiate and undergo the programming process in an environment free from foreign
antigens.

Question:
Which of the following statements concerning T-helper cells is true?

Answer Choices:
A They possess membrane-bound antibodies
B They can recognize and interact with antigens in the blood
C They produce numerous cytokines
D They function only in cell-mediated immunity
E Their activation depends on interferon γ

C They produce numerous cytokines


T-helper cells produce a number of cytokines that affect other cells involved in both the cell-
mediated and humoral immune responses. T-helper cells possess antigen-specific T-cell
receptors (not antibodies) on their membrane. These cells recognize and interact with antigenic
determinants that are associated with class II HLA molecules on the surface of antigen-
presenting cells. IL-1 is necessary for activation of T-helper cells.
Interferon γ activates macrophages, as well as NK cells and T-cytotoxic cells, enhancing their
phagocytic and cytotoxic activity.

Question:
The structures at the tip of the arrows in the photomicroscopic image are best identified as

Answer Choices:
A Macrophages
B Thymic corpuscles
C Thymic lobules
D Thymic follicles
E Large lymphocytes

Image(s) / Chart(s):
Click image to view full size. Click open image to close. Click and hold open image to move.

B Thymic corpuscles

The thymus arises developmentally as an epithelial organ from the third branchial pouch. The
endodermal epithelium is invaded by stem lymphocytes where lymphocytes occupy spaces
among the epithelial cells. The epithelial cells remain in contact and function as a supportive
epithelial framework for the organ. These large cells can be observed throughout the organ, but
are prominently displayed in the medulla as isolated masses of closely packed, and somewhat
concentrically organized groups of reticulo-epithelial cells referred to as thymic corpuscles, or
Hassall's corpuscles, which can be seen at the tips of the arrows in the photomicroscopic image.
Question:
Removal of the organ depicted in the photomicroscopic image in an infant may have which one of the
following consequences to that individual?

Answer Choices:
A Probably no consequence to the individual upon successful remove of this organ.
B Reduction of filtration of lymph fluid in the region the organ was removed.
C Decrease in the formation of red blood cells.
D Decrease in the filtration of blood and removal of "worn out" red blood cells.
E Probable impairment in cell-mediated immune response.

Image(s) / Chart(s):
Click image to view full size. Click open image to close. Click and hold open image to move.

E Probable impairment in cell-mediated immune response.

The organ depicted is the thymus gland. Immature lymphocytes migrate to the thymus from
hemopoietic sites during fetal development and the thymus becomes the site of T-lymphocyte
differentiation and programming. The T-lymphocytes are "long-living" cells that participate in
the cell-mediated response. During childhood, a normal complement of T-lymphocytes are
reached in humans and the organ involutes and is replaced by adipose tissue by the time an
individual reaches the third or fourth decade of life. If the thymus is removed in an infant, the
T-lymphocyte population is reduced or depleted and eventually an impairment of the cell-
mediated immune response would occur.

Question:
Which of the following statements about IgG is true?

Answer Choices:
A It is located in the serum and on the membrane of B cells
B It can cross the placenta
C It is involved in allergic reactions
D It exists as a pentamer
E It binds to antigens on the body surface and in the lumen of the gastrointestinal tract

B It can cross the placenta

IgG, the most abundant immunoglobulin isotype in the serum, can cross the placenta. It is
present only in the serum, exists as a monomer, and functions to activate complement and as
an opsonin.
Question:
Which of the following statements concerning the bursa of Fabricius is true?

Answer Choices:
A In mammals (including humans), it is generally believed that B-lymphocytes (or B-
cells) acquire their differentiated characters in special microenvironment in the bursa of
Fabricius
B When this tissue is destroyed (either surgically, or by the administration of high
levels of testosterone), the process of humoral immunity in the animal is impaired
C The lymph nodes and spleen i n animals that possess bursa of Fabricius are regarded
as bursa-independent areas
D The bursa is a mass of lymphoid tissue present inside the mammalian rectum
E It is the site for the differentiation of mast and eosinophilic cells
B When this tissue is destroyed (either surgically, or by the administration of high levels of
testosterone), the process of humoral immunity in the animal is impaired

In the early 1960s, experiments using chicken embryos revealed one of the anatomic sites of
lymphocyte differentiation. The bursa of Fabricius is a mass of lymphoid tissue near the cloaca
of birds. When this tissue is destroyed in the embryo (either surgically, or by the administration
of high levels of testosterone), chickens lack the ability to produce immunoglobulins (IgM, IgG,
etc.) against specific antigens. That is, humoral immunity, a process that requires the presence
of immunoglobulins in the blood, is impaired. The number of lymphocytes found in specific
areas in the lymph nodes and spleen is profoundly reduced, leading to the designation of these
regions as bursa-dependent areas. The affected lymphocytes are known as B lymphocytes, or B
cells.
In mammals (including humans), it is generally believed that B lymphocytes acquire their
differentiated characters in special microenvironments in the bone marrow.
Question:
In which of the following sites do lymphocytes become immunocompetent?

Answer Choices:
A Germinal center of secondary lymphoid nodules
B White pulp of the spleen
C Thymic cortex
D Red pulp of the spleen
E Paracortex of lymph nodes

C Thymic cortex

T-lymphocytes mature and become immunocompetent in the cortex of the thymus, whereas B-
lymphocytes do so in the bone marrow. Following an antigenic challenge, lymphocytes
proliferate and differentiate in various lymphoid tissues.
Lymphoid nodules are composed of densely packed lymphocytes (mainly B-lymphocytes), and
secondary nodules may contain a germinal center. These centers are a collection of activated
cytoplasm-rich lymphocytes (lymphoblasts) that appear, only after birth, in response to antigen
exposure. White pulp of the spleen consists of lymphoid tissue that ensheathes both central
arteries and lymphoid nodules appended to the sheath. Lymphoid nodules consist of a
preponderance of B-lymphocytes. The red pulp is a reticular tissue with a special characteristic
splenic cords. These cords contain macrophages, lymphocytes, plasma cells, and many blood
elements (erythrocytes, platelets, and granulocytes). Paracortex of lymph nodes contains:
subcapsular sinus communicating with medullary sinuses through intermediate sinuses,
reticular cells and fibers, and T-lymphocytes.

Question:
Which of the following statements concerning Hassall's corpuscles is true?

Answer Choices:
A They are located in the thymic cortex of young individuals
B They are located in the thymic cortex of old individuals
C They are derived from mesoderm
D They are located in the thymic medulla
E They are derived from T-memory cells

D They are located in the thymic medulla


Image(s) / Chart(s):
Click image to view full size. Click open image to close. Click and hold open image to move.

Hassall's corpuscles are concentrically arranged accretions of epithelial reticular cells (derived
from endoderm) found only in the medulla of the thymus. Thymic cortex of young individuals is
composed of an extensive population of T-lymphocytes, dispersed epithelial reticular cells, and
few macrophages. This region, however, does not contain Hassall's corpuscles.
See Figure: Hassall's corpuscle inside thymus medulla.

Question:
Which one of the following statements concerning the thymus is true?

Answer Choices:
A It is derived from ectoderm, and begins to involute at the time of birth
B Thymic cortex contains large number of epithelial reticular cells and mature T-
lymphocytes, which are loosely packed, causing cortex to stain lighter than the medulla
C Thymocytes are immature B-lymphocytes present within the thymic medulla in
various stages of differentiation
D Thymic medulla typically contains Hassall's corpuscles, a whorl-like accretions of
epithelial reticular cell
E Blood-thymus barrier exists in medulla only, making it an immunologically protected
region

D Thymic medulla typically contains Hassall's corpuscles, a whorl-like accretions of epithelial


reticular cell

The medulla in the thymus contains Hassall's corpuscles, which are characteristic of this region.
These structures are concentrically arranged, flattened epithelial reticular cells that degenerate
and become filled with keratohyalin granules and cytokeratin filaments. Hassall's corpuscles
vary in size, according to the stage of development; their function is unknown.
Thymus is derived both from endoderm (epithelial reticular cells) and mesoderm
(lymphocytes). Thymic cortex is supplied by arterioles in the septa; these provide capillary
loops that enter the substance of the cortex. It is a region in which T-cells mature. Thymocytes
are immature T-lymphocytes present in large numbers within the thymic cortex in different
stages of differentiation. Blood-thymus barrier exists in cortex only, making it an
immunologically protected region.

Question:
The functional surface molecule on T-lymphocytes that facilitates antigen recognition via major
histocompatibility complex (MHC) molecules is which one of the following subset of T-cells?

Answer Choices:
A T3
B T4
C T8
D T11
E C3

C T8

Surface molecule receptors located on T-lymphocytes allow them individual cellular functions
mediated by activation of the presence of specific surface molecules. The subset T cells,
referred to as T8, are T cells which possess a molecule that facilitates recognition of antigens via
the MHC molecules.

Question:
Which one of the following cell types are believed to produce thymosin?

Answer Choices:
A T-lymphocytes
B B-lymphocytes
C Primitive lymphocytes
D Reticulo-epithelial cells
E Plasma cells

D Reticulo-epithelial cells
Some evidence exists that indicates thymosin is produced by the reticulo-epithelial cells of the
thymus. Thymosin is a humoral factor that promotes the development and differentiation of T-
lymphocytes in the cortex of the thymus.

Question:
A humoral factor promotes the transformation of immature lymphocytes into T-lymphocytes in the
thymus. This humoral factor is called

Answer Choices:
A Major histocompatibility complex (MHC)
B Endorphin
C Thymosin
D Interleukin
E IgG

C Thymosin

Immature lymphocytes in the thymus are responsive to a humoral factor from the thymus
called thymosin. Thymosin acts to stimulate the development and differentiation of the T-
lymphocytes.

Question:
T-lymphocytes were isolated and their relative numbers quantified in each of five different sites in the
body. The site where mature T-lymphocytes would be in the LOWEST numbers would be the

Answer Choices:
A Thymus
B Lymph nodes
C Bone marrow
D Spleen
E Thoracic duct lymph fluid

C Bone marrow

T-lymphocytes, which are differentiated and programmed, migrate and "seed" other lymphoid
organs. As such, T-lymphocytes are abundantly found in the thymus (where differentiation and
programming occurs), the lymph nodes (particularly the thymus dependent cortical areas), the
spleen (white pulp) and thoracic duct lymph fluid. Relatively few mature T-lymphocytes are
found in the bone marrow.
Question: BLOCK EXAM
The primary function of the organ displayed in the photomicroscopic image is best described as

Answer Choices:
A Filtering and removal of erythrocytes
B Development and differentiation of B lymphocytes
C Development and differentiation of T lymphocytes
D Development and differentiation of erythrocytes
E Filtering and removal of lymphocytes

Image(s) / Chart(s):
Click image to view full size. Click open image to close. Click and hold open image to move.

C Development and differentiation of T lymphocytes

The photomicroscopic image depicts the appearance of the cortex of the thymus, with a small
area of medulla centrally located in a thymic lobule. The function of the thymus is best
described as providing a site for the proliferation, development, and differentiation
(programming) of T lymphocytes.
The T lymphocytes are released by the thymus and colonize organs such as lymph nodes and
the spleen, where they participate in cell-mediated immune responses. There is considerable
proliferation of lymphocytes within the thymus, prior to its involution, and consequently some
removal of newly formed lymphocytes occurs in the organ, but since there are no afferent
lymphatic vessels to the thymus, there is no significant filtering function of the thymus.
Question: CLASS Q
The organ observed in the photomicroscopic image was obtained from a patient of unknown sex and
age. From which one of the following aged individuals was this tissue probably obtained?

Answer Choices:
A Infant male
B Elderly female
C Elderly male
D Adult male

Image(s) / Chart(s):
Click image to view full size. Click open image to close. Click and hold open image to move.

A Infant male

This organ has the histological features of a thymus from an infant. The fully formed and
functioning thymus persists from birth until about the age of puberty in both sexes. At about
the time of puberty, lymphocyte processing and proliferation are dramatically reduced. The
thymic tissue is replaced by connective tissue and adipose tissue resulting in glandular
involution. There are no apparent histological differences between the thymus of male and
females.
Question: CLASS TEST
The human immunoglobulin that is involved in allergic reactions to stimulate mast cells to release
histamine, heparin and leukotrienes is which one of the following immunoglobulins?

Answer Choices:
A IgA
B IgM
C IgG
D IgD
E IgE

E IgE

B lymphocytes produce immunoglobulins which possess a variety of actions. One such


immunoglobulin, IgE, stimulates mast cells to release histamine, heparin and also leukotrienes.
Leukotrienes are the slow-reacting substance of anaphylaxis. IgE also will stimulate the release
of eosinophil chemotactic factor of anaphylaxis which mediates the chemotaxis of eosinophil to
the site characteristic of the allergic reaction.

Question: BLOCK EXAM


The human immunoglobulin that is present in most all body secretions and provides protection
against the proliferation of microorganisms that come into contact (or are found within the fluid) is
which one of the following immunoglobulins?

Answer Choices:
A IgA
B IgM
C IgG
D IgD
E IgE

A IgA

B lymphocytes produce immunoglobulins which possess a variety of actions. One such


immunoglobulin, IgA, is present in most all body secretions, including tears, saliva, vaginal fluid
and the secretions of the respiratory system, the intestine and reproductive system. IgA helps
depress the proliferation of microorganisms that may be found in these fluids, or act against
foreign molecules (viruses) that may penetrate the body by way of body openings to the
respiratory, gastrointestinal, or reproductive systems.

Question:
Polypeptide hormones referred to as interleukins are produced primarily by which one of the
following cell types?

Answer Choices:
A Memory cells
B NK cells
C Killer T cells
D Helper T cells
E Suppressor T cells

D Helper T cells

When B lymphocytes react to a given single antigen the B lymphocyte contacted will transform
into a plasmablast. These cells will proliferate, then synthesize and secrete a specific antibody
to the antigen, and differentiate into mature plasma cells. One type of immunocompetent T
lymphocyte, the helper T lymphocyte assists in the stimulation of B lymphocytes to produce
antibodies. The helper T cells possess surface receptors that bind to surface peptide complexes
on the membranes of B lymphocytes exposed to a specific antigen. The helper T cell is activated
to produce polypeptide hormones called interleukins. It is the interleukin that will stimulate
antibody production by the B cells, macrophages, or plasma cells.

Question:
The cell type responsible for identification and targeting a virus-infected cell that displays viral
glycoproteins on its surface would be a

Answer Choices:
A NK cell
B B lymphocyte
C Killer T cell
D Helper T cell
E Suppressor T cell

C Killer T cell

The function of the immunocompetent T lymphocytes that have been activated by interaction
with an antigen include the ability to monitor other cells for signs of infection, or even other
indications of abnormality, such as development of cell surface membranes identified as "non-
self". As such cytotoxic lymphocytes (killer T cells) will recognize viral or in some cases, cancer
related peptides on the surface of cells. Identification of a virus-infected cell that displays the
viral glycoproteins on the surface membranes would result in the killer T cell to lyse the
infected cell.

Case #301555:
You are a neonatologist on-call on a busy obstetrical service. You examine a newborn male
child with tetany, cyanosis, and unusual craniofacial features, including mandibular
hypoplasia, a reduced philtrum, and low-set ears. Laboratory findings reveal hypocalcemia,
decreased parathormone levels, but a nearly normal immunoglobulin profile.
Question:
What is the most likely diagnosis of this patient’s condition?

Answer Choices:
A Down syndrome
B DiGeorge syndrome
C Bruton agammaoglobulinemia
D mandibulofacial dysostosis
E fetal alcohol syndrome and HIV infection

B DiGeorge syndrome

This newborn male patient is afflicted by DiGeorge syndrome, an immundeficiency syndrome


caused by thymic aplasia. The incidence of Digeorge syndrome is approximately 1 in 2,000 with
a M:F ratio of 1:1. Deletion in the short arm of chromosome 22 has been detected in the vast
majority of cases. The severity of this syndrome is highly variable, depending upon the degree
of organ hypoplasia. The immunological problems in these infants (e.g., recurrent infections by
opportunistic organisms) are due to thymic aplasia or hypoplasia leading to partial or complete
absence of cell-mediated immune response (T cell function). The hypocalcemia is due to
variable parathyroid hypoplasia. Patients with DiGeorge syndrome also commonly have atrial or
ventricular septal defects and craniofacial anomalies.

If a patient had thymic hypoplasia, one would also typically see abnormalities in T cell function.
Recall that undifferentiated bone-marrow precursors of lymphocytes are programmed to
become T cells after transit through and stimulation in the thymus. Absence of T cells would be
most commonly and obviously manifested by decreases in cell-mediated immune response.

Patients with Down syndrome would have more frequent upper respiratory infections (URIs)
and cardiovascular defects, but their T cell function would be normal and they would not have
hypocalcemia. A patient with Bruton agammaglobulinemia would have frequent URIs, recurrent
otitis media, and ocular infections, but their immunoglobulin profile would not be normal
because of a lack of B cell function. A child with mandibulofacial dysostosis would have some of
the same craniofacial anomalies but not the associated cardiovascular problems and would
have normal T cell function. Children with HIV infections might have opportunistic infections
but would not have hypocalcemia. Children with fetal alcohol syndrome would not be any more
susceptible to opportunistic infections and would not have hypocalcemia.
Case #301556:
You are a neonatologist on-call on a busy obstetrical service. You examine a newborn male
child with tetany, cyanosis, and unusual craniofacial features, including mandibular
hypoplasia, a reduced philtrum, and low-set ears. Laboratory findings reveal hypocalcemia
and decreased parathormone levels, but a nearly normal immunoglobulin profile.
Question: CLASS Q
Which organ of the immune system would be MOST altered in this disease?

Answer Choices:
A palatine tonsils
B lymph nodes
C spleen
D thymus
E gut-associated lymphoid tissue (GALT)

D thymus

This newborn male patient is afflicted by DiGeorge syndrome, an immundeficiency syndrome


caused by thymic aplasia. The incidence of Digeorge syndrome is approximately 1 in 2,000 with
a M:F ratio of 1:1. Deletion in the short arm of chromosome 22 has been detected in the vast
majority of cases. The severity of this syndrome is highly variable, depending upon the degree
of organ hypoplasia. The immunological problems in these infants (e.g., recurrent infections by
opportunistic organisms) are due to thymic aplasia or hypoplasia leading to partial or complete
absence of cell-mediated immune response (T cell function). The hypocalcemia is due to
variable parathyroid hypoplasia. Patients with DiGeorge syndrome also commonly have atrial or
ventricular septal defects and craniofacial anomalies.

If a patient had thymic hypoplasia, one would also typically see abnormalities in T cell function.
Recall that undifferentiated bone-marrow precursors of lymphocytes are programmed to
become T cells after transit through and stimulation in the thymus. Absence of T cells would be
most commonly and obviously manifested by decreases in cell-mediated immune response.
Because of abnormalities in T cell differentiation, the palatine tonsils, lymph nodes, spleen, and
GALT would all lack fully functional T-cells, but they would not be grossly abnormal in
morphology.
Case #301557:
You are a neonatologist on-call on a busy obstetrical service. You examine a newborn male
child with tetany, cyanosis, and unusual craniofacial features, including mandibular
hypoplasia, a reduced philtrum, and low-set ears. Laboratory findings reveal hypocalcemia
and decreased parathormone levels, but a nearly normal immunoglobulin profile.

Question: CLASS QUIZ


Which cell of the immune system would be MOST affected by this disease?
Answer Choices:
A T cell
B B cell
C mast cell
D macrophage
E trabecular fibroblasts

A T cell

This newborn male patient is afflicted by DiGeorge syndrome, an immundeficiency syndrome


caused by thymic aplasia. The incidence of Digeorge syndrome is approximately 1 in 2,000 with
a M:F ratio of 1:1. Deletion in the short arm of chromosome 22 has been detected in the vast
majority of cases. The severity of this syndrome is highly variable, depending upon the degree
of organ hypoplasia. The immunological problems in these infants (e.g., recurrent infections by
opportunistic organisms) are due to thymic aplasia or hypoplasia, leading to partial or complete
absence of cell-mediated immune response (T cell function). The hypocalcemia is due to
variable parathyroid hypoplasia. Patients with DiGeorge syndrome also commonly have atrial or
ventricular septal defects and craniofacial anomalies.

If a patient had thymic hypoplasia, one would also typically see abnormalities in T cell function.
Recall that undifferentiated bone-marrow precursors of lymphocytes are programmed to
become T cells after transit through and stimulation in the thymus. Absence of T cells would be
most commonly and obviously manifested by decreases in cell-mediated immune response.
There would be an adequate supply of B cells in this patient. Mast cells, macrophages, and
trabecular fibroblasts in immune organs would also not be affected to any appreciable
degree.
SECTION- LYMPHATIC VESSELS AND LYMPH NODES
Question:
Once they have completed their maturation, B and T lymphocytes enter the bloodstream, from which
they migrate to the peripheral lymphoid organs such as the lymph nodes. Lymph nodes are highly
organized lymphoid structures that are the sites of convergence of an extensive system of vessels that
collect the extracellular fluid from the tissues and return it to the blood. This extracellular fluid is
produced continuously by filtration from the blood, and is called lymph. What is the primary resting
place for B lymphocytes and the area where B cells undergo clonal expansion?

Answer Choices:
A Trabecula
B Germinal center
C Capsule
D Medullary sinus
E Medullary cord
B Germinal center

A lymph node consists of an outermost cortex and an inner medulla. The node is covered
thickly with the fibrous capsule and is subdivided into different compartments by inward
pointing trabeculae. The cortex is composed of an outer cortex of B lymphocytes organized into
lymphoid follicles, and deep, or paracortical, areas made up mainly of T lymphocytes and
specialized cells known as dendritic cells.
Some of the B cell follicles contain central areas of intense B cell proliferation called germinal
centers. These follicles are secondary lymphoid follicles. Lymph draining from the extracellular
spaces of the body carries antigens from the tissues to the lymph node via the afferent
lymphatics. Lymph leaves by the efferent lymphatic in the medulla. The medulla consists of
strings of macrophages and antibody-secreting plasma cells known as the medullary cords,
bounded by the medullary sinuses. Naïve lymphocytes enter the node from the bloodstream
through specialized post-capillary venules and leave with the lymph through the efferent
lymphatic.

Keywords: B cell development, lymph node, clonal expansion

Question: CLASS QUIZ


Following their maturation in the thymus and release into the circulation, T-lymphocytes migrate
preferentially to which of the following sites?

Answer Choices:
A Paracortex of lymph nodes
B Cortical lymphoid nodules of lymph nodes
C Hilus of lymph nodes
D Lymphoid nodules of the tonsils
E Lymphoid nodules of the spleen

A Paracortex of lymph nodes

T-lymphocytes are preferentially located in the paracortex of lymph nodes, whereas B


lymphocytes are found in lymphoid nodules located in lymph nodes, tonsils, and the spleen.

estion:
The level of organization of the lymph tissue demonstrated in the attached photomicrographic image
can best be classified as
Answer Choices:
A Diffuse lymphatic tissue
B Lymph nodule
C Lymph node
D White pulp
E Tonsil

Image(s) / Chart(s):
Click image to view full size. Click open image to close. Click and hold open image to move.

A Diffuse lymphatic tissue

The dense population of small lymphocytes as depicted in this image occurs as part of a process
in which lymphocytes migrate into the infected area as diffuse lymphatic tissue. There are no
lymph nodules present. Higher power investigation of this image would reveal the presence of
macrophages, eosinophils, tissue neutrophils and plasma cells. These cells have migrated into
this soft tissue connective tissue area in response to foreign bodies present in the connective
tissue.

Question: BLOCK EXAM


The level of organization of the lymph tissue demonstrated in the attached photomicrographic image
can best be classified as

Answer Choices:
A Diffuse lymphatic tissue
B Lymph nodule
C Lymph node
D White pulp
E Tonsil

Image(s) / Chart(s):
Click image to view full size. Click open image to close. Click and hold open image to move.
B Lymph nodule

This image depicts a higher powered view of a lymph nodule. Note the well developed germinal
center surrounded by smaller, more densely populated lymphocytes. The germinal centers are
populated by larger, lighter staining null cells (large lymphocytes).

Question: CLASS Q
The organ as depicted by the attached photomicroscopic image is best identified as

Answer Choices:
A Spleen
B Pharyngeal tonsil
C Appendix
D Lymph node
E Palatine tonsil

Image(s) / Chart(s):
Click image to view full size. Click open image to close. Click and hold open image to move.
D Lymph node

The image depicts the appearance of a lymph node. Note that a thin connective tissue capsule
is present, which provides trabeculae that course into the parenchyma of the organ. Both a
cortex and medulla is present, and a lymph nodule containing a germinal center is present in
the cortex. The lymph nodules, as present in the node, is an area rich in B cells, whereas the
cortical area between lymph nodules are T cell rich areas. The medulla is comprised of cords of
lymphocytes. This image also reveals the presence of a subcapsular (cortical) sinus immediately
deep to the capsule.

Question:
A concentration of B lymphocytes would be more prevalent in which area of the lymph node?

Answer Choices:
A Cortex
B Medulla
C Paracortex
D Subcapsular sinus

B Medulla
Within the lymph node, B lymphocytes are most numerous in the medulla, and may also
dominate the lymphatic nodules of the cortex. The T lymphocytes are more prevalent in the
paracortical areas of the lymph node located around the lymphatic nodules.

Question:
A concentration of T lymphocytes would be more prevalent in which area of the lymph node?

Answer Choices:
A Cortex
B Medulla
C Paracortex
D Sinus
E Germinal center

C Paracortex

Within the lymph node, B lymphocytes are most numerous in the medulla, and may also
dominate the lymphatic nodules of the cortex. The T lymphocytes are more prevalent in the
paracortical areas of the lymph node located around the lymphatic nodules.

Question:
The type of lymphocyte that functions to lyse target cells, but does not depend on identifying the
target cell by surface molecules is which one of the following cell types?

Answer Choices:
A Macrophage
B Killer T cell
C NK cell
D B lymphocyte
E Helper T cell

C NK cell

A small percentage of the lymphocytes found in the blood include "large" lymphocytes, also
referred to as null cells. These cells have more cytoplasm than do the small lymphocytes, which
contain large azurophilic granules. One subtype null cell include the natural killer cell (NK cell),
which will cause lysis of target cells. The NK cells differ from the killer T cells in that the NK cell
does not possess surface molecules associated with the T (or B) lymphocytes to identify the
target cell. Therefore, killing of the target cell is a non-antibody-mediated process. This
population of cells may act to survey and destroy cells that undergo spontaneous
transformation and identified as non-self cells.
Question: CLASS QUIZ
A 23-year old male presented at the Emergency Department in a confused state. His blood pressure
was 95/45, and RR 12. Examination of the oral cavity revealed a dry, thickened tongue. He reported
generalized itching, and cutaneous hives were present on the abdomen, chest and thigh areas. There
were 6-8 raised red areas on his face and hands. A friend accompanying him indicated they had been
in a wooded area and the patient had been stung by bees several times after turning over a dead log.
The patient probably is hypersensitive to the bee venom and may be entering anaphylactic shock.
This hypersensitivity is due to the response of which immunological cell type

Answer Choices:
A Suppressor T cells
B NK cells
C Memory cells
D Helper T cells
E Killer T cells

C Memory cells

Only a given B lymphocyte type will react to a given single antigen to which it has been
genetically programmed to react. If exposed to any other antigen, the given B lymphocyte will
not respond. When activated by a specific antigen, the B lymphocyte contacted will transform
into a plasmablast. The cell will proliferate, then synthesize and secrete a specific antibody to
the antigen, and differentiate into a mature plasma cell. However, these cells may retain their
programmed ability to respond rapidly to the same antigen if it should appear again and as
such, are referred to as memory cells. Memory cells do not participate in the initial response to
an antigen, but are cells that may be rapidly recruited upon subsequent exposure as part of the
secondary response. A subsequent exposure to the antigen may activate the secondary
response by the memory cells, but may result in antibody-induced tissue-damaging reactions
by degranulation of mast cells as occurs in allergic reactions.
Question:
A primary cell type generally found within the germinal center of a lymph nodule which gives it the
characteristic lighter staining appearance include

Answer Choices:
A Large lymphocytes
B B lymphocytes
C Killer T lymphocytes
D Neutrophils
E Helper T lymphocytes

A Large lymphocytes

Not all lymph nodules contain germinal centers. The presence of a germinal center is
morphological evidence of lymphatic tissue response to the presence of an antigen. The
characteristic lighter staining of the germinal center is due to the presence of large
lymphocytes, lymphoblasts, and plasmablasts that have large amounts of dispersed
euchromatin in their nuclei rather than the dense heterochromatin characteristic of the small
lymphocytes. Furthermore, these cells are larger, having more cytoplasm than their small
lymphocyte counterparts, which adds to the lighter staining characteristics of the germinal
center.
Question: CLASS QUIZ
The level of organization of the lymph tissue demonstrated in the attached photomicrographic image
can best be classified as

Answer Choices:
A Diffuse lymphatic tissue
B Lymph nodule
C Lymph node
D White pulp
E Tonsil

Image(s) / Chart(s):
Click image to view full size. Click open image to close. Click and hold open image to move.

A Diffuse lymphatic tissue

The least complex organization of lymph tissue is diffuse lymphatic tissue. In the image
depicted, the lymphatic tissue is contained as a dense infiltration of the lamina propria of the
intestinal tract. The lymphatic tissue does not appear to be organized into nodules, at least
within the limits of the section currently observed. T he lymphocyte congregations of diffuse
lymphatic tissue are dynamic in function in that they are strategically located to identify and
react to antigens present at these sites which open to the surface of the body. When these
lymphocytes react to antigens, they will travel to lymph nodes in their proximity to undergo
proliferation and differentiation. Derivatives of these cells will return to the original site as
effector B cells, T cells or plasma cells, or memory cells.

Question:
The simplest level of lymphatic tissue organization is the:

Answer Choices:
A Lymph node.
B Lymph nodule.
C Tonsil.
D Diffuse lymphatic tissue.
E Thymus.

D Diffuse lymphatic tissue.

Throughout the body, but generally located within the lamina propria immediately deep to an
epithelium are large aggregations of lymphocytes, referred to as diffuse lymphatic tissue.
These occur in the subepithelial tissue of the respiratory passages, the gastrointestinal tract and
the genitourinary tracts. The lymphocyte congregations of diffuse lymphatic tissue are dynamic
in function in that they are strategically located to identify and react to antigens present at
these sites open to the surface of the body. When these lymphocytes react to antigens, they
will travel to lymph nodes in their proximity to undergo proliferation and differentiation.
Derivatives of these cells will return to the original site as effector B cells, T cells or plasma cells,
or memory cells.
Question: CLASS QUIZ
Plasma cells may be derived from activation of which one of the following cell types?

Answer Choices:
A Macrophage
B B lymphocyte
C Neutrophil
D T lymphocyte
E NK cell
B B lymphocyte

Only a given B lymphocyte type will react to a given single antigen to which it has been
genetically programmed to react. If exposed to any other antigen, the given B lymphocyte will
not respond. When activated by a specific antigen, the B lymphocyte contacted will transform
into a plasmablast. The cell will proliferate, then synthesize and secrete a specific antibody to
the antigen, and differentiate into a mature plasma cell. This process is part of the humoral
response, leading to antibody-mediated immunity.

Question: CLASS Q
A cell type, which will rapidly synthesize and secrete a specific antibody, in response to an additional
encounter with a specific antigen is a

Answer Choices:
A NK cell
B Memory cell
C T lymphocyte
D Macrophage
E Killer cell

B Memory cell

Only a given B lymphocyte type will react to a given single antigen, to which it has been
genetically programmed to react. If exposed to any other antigen, the given B lymphocyte will
not respond.
When activated by a specific antigen, the B lymphocyte contacted will transform into a
plasmablast.
The cell will proliferate, then synthesize and secrete a specific antibody to the antigen, and
differentiate into a mature plasma cell.
However, these cells may retain their programmed ability to respond rapidly to the same
antigen if it should appear again and as such, they are are referred to as "memory cells".
Memory cells do not participate in the initial response to an antigen, but are cells that may be
rapidly recruited upon subsequent exposure as part of the secondary response.

Question:
B lymphocytes are the cells that synthesize antibody in response to antigen. The interaction of
antigen with the mature B cell generally results in activation. The response of mature B cells to
foreign antigen occurs primarily in the secondary lymphoid organs, the lymph node and spleen, and,
more specifically, in specialized regions of these organs. What are these specialized regions called?
Answer Choices:
A Peyer's patches
B Medullary cords
C Germinal centers
D Medullary sinuses
E Trabeculae

C Germinal centers

Germinal centers are formed in the B cell areas of lymphoid tissue within a week after
stimulation with antigen. The activated B cells proliferate in the environment of follicular
dendritic cells and develop into germinal centers. Within the centers the affinity of the
antibodies increases, a process known as affinity maturation. The germinal center plays an
important role in the differentiation of memory B cells, which control the secondary humoral
immune response. Germinal centers are built up polyclonally; a small number of activated B
cells develop into one germinal center. A positive selection process in the light zone of the
center allows optimization of the affinity to the antigen. The unselected cells die through
apoptosis.
Named after a Swiss anatomist, Peyer's patches are oval elevated areas of lymphoid tissue on
the mucosa of the small intestine, composed of many lymphoid follicles closely packed
together. Antigen is collected by specialized epithelial cells called M cells. The lymphocytes
form a follicle consisting of a large central dome of B lymphocytes surrounded by a smaller
number of T lymphocytes.
The medulla consists of strings of macrophages and antibody-secreting plasma cells known as
the medullary cords, bounded by the medullary sinuses. Naïve lymphocytes enter the node
from the bloodstream through specialized post-capillary venules and leave with the lymph
through the efferent lymphatic.
A lymph node consists of an outermost cortex and an inner medulla. The node is covered
thickly with the fibrous capsule and is subdivided into different compartments by inward
pointing trabeculae.

SECTION- SPLEEN
Question:
Which one of the following statements concerning Billroth's cords is true?

Answer Choices:
A They are cord-like, branched extensions of the inner cortex of the lymph node, and
contain B lymphocytes and some plasma cells
B They form the red pulp that lies between the sinusoids of the spleen
C They form the demarcation line between the red and white pulp, and consist of many
sinuses and loose lymphoid tissue
D They form the germinal center inside the secondary lymphoid nodules
E They constitute 10% of serum immunoglobulins and usually exists as a pentamer with
the molecular weight of 900,000

B They form the red pulp that lies between the sinusoids of the spleen

Red splenic pulp, a dark red tissue, is composed of elongated structures, the splenic cords
(known as Billroth's cords), that lie between the sinusoids.
The medulla of the lymph node is composed of the medullary cords, cord-like, branched
extensions of the inner cortex, that contains B lymphocytes and some plasma cells. The
demarcation line between the white pulp and the red pulp is formed by the marginal zone
consisting of many sinuses and loose lymphoid tissue. The germinal center in secondary
nodules presents a clear zone and has a collection of activated cytoplasm-rich lymphocytes
(lymphoblasts) that appear only after birth in response to antigen exposure. IgM constitute 10%
of serum immunoglobulins and usually exists as a pentamer with the molecular weight of
900,000.
Question:
Blood will leave the splenic sinuses by way of what structures?

Answer Choices:
A Splenic capillaries
B Sheathed capillaries
C Marginal veins
D Splenic veins
E Trabecular veins

E Trabecular veins

As blood courses through the spleen it first enters the white pulp via the central artery.
Branches of the central artery continue into the red pulp as penicillar arterioles that empty into
the splenic sinuses. Blood collected from the sinuses drain into trabecular veins which converge
with other trabecular veins and form the splenic vein. Blood in the splenic vein will join the
hepatic portal vein.
Question:
Removal of aged red blood cells and their destruction occurs in what organs following the removal of
the spleen?
Answer Choices:
A Bone marrow and thymus
B Bone marrow and liver
C Bone marrow and lymph node
D Liver and lymph node
E Liver and thymus

B Bone marrow and liver

Surgical removal of the spleen shifts the removal of aged and damaged red blood cells to the
bone marrow and liver.
Question:
It is believed that two possible pathways may be taken by blood as it courses through the spleen,
both of which may coexist in the organ. To best describe the "closed circulation model" blood cells
would be exposed to splenic macrophages by

Answer Choices:
A Direct contact as the blood percolates through the cords
B First leaving the sinuses then reentering them
C Leaving the sinuses without reentering them
D Indirect contact as blood percolates through the sinuses
E Direct contact as the blood leaves the PALS

B First leaving the sinuses then reentering them

While there is evidence that both an "open" and "closed" model of splenic circulation exists,
evidence for the closed model suggests a continuous vascular channel exists through the red
pulp and blood cells would have to leave the splenic sinuses for exposure to the macrophages,
then reenter the sinuses and continue through the venous circulation. The open circulation
model demonstrates that the Penicillar arterioles empty directly into the reticular meshwork of
the red pulp. Blood cells would then percolate through the splenic cords, which are not
endothelial lined, and be exposed by direct contact to macrophages within the cords. The blood
would then return to the circulation by entering a venous sinus.
Question:
It is believed that two possible pathways may be taken by blood as it courses through the spleen,
both of which may coexist in the organ. To best describe the "open circulation model", blood cells
would be exposed to splenic macrophages by:

Answer Choices:
A Direct contact as the blood percolates through the cords.
B First leaving the sinuses then reentering them.
C Leaving the sinuses without reentering them.
D Indirect contact as blood percolates through the sinuses.
E Direct contact as the blood leaves the PALS.

A Direct contact as the blood percolates through the cords.

While there is evidence that both an "open" and "closed" model of splenic circulation exists,
evidence for the open circulation model demonstrates that the Penicillar arterioles empty
directly into the reticular meshwork of the red pulp. Blood cells would then percolate through
the splenic cords, which are not endothelial lined, and be exposed by direct contact to
macrophages within the cords. The blood would then return to the circulation by entering a
venous sinus. In the closed model, a continuous vascular channel through the red pulp exists
and blood cells would have to leave the splenic sinuses for exposure to the macrophages, then
reenter the sinuses and continue through the venous circulation.

Question: BLOCK EXAMS


Within the white pulp of the spleen, lymphocytes aggregate around a central artery and form a
cylindrical configuration along the axis of the artery. This aggregation of lymphocytes along the artery,
as seen in cross section in this photomicroscopic image, is referred to as

Answer Choices:
A Malpighian corpuscles
B Periarterial lymphatic sheath
C Splenic cords
D Splenic sinus
E Trabecular cord

Image(s) / Chart(s):
Click image to view full size. Click open image to close. Click and hold open image to move.
B Periarterial lymphatic sheath

Lymphocytes aggregate around the central artery as it courses through the spleen and form a
cylindrical pattern around the artery. On cross section, as is visualized by this image, the central
artery occupies an eccentric position as it courses through the lymphatic sheath and the sheath
is observed as an apparent ball of lymphocytes rather than a cylinder. This sheath is referred to
as the periarterial lymphatic sheath (PALS).

Question: CLASS QUIZ


The structure composed of lymphocytes in the photomicroscopic image is best described as

Answer Choices:
A Splenic red pulp
B Splenic white pulp
C Lymph node
D Thymic medulla
E Tonsil

Image(s) / Chart(s):
Click image to view full size. Click open image to close. Click and hold open image to move.
B Splenic white pulp

The arrows outline numerous lymphocytes that accumulate around a branch of the splenic
artery, the central artery, within the white pulp of the spleen. The lymphocytes which
aggregate around the central artery as it courses through the spleen form a cylindrical pattern
around the artery. On cross section, as is visualized by this image, the central artery occupies an
eccentric position as it courses through the lymphatic sheath, and the sheath is observed as an
apparent ball of lymphocytes rather than a cylinder. This sheath may form true lymphatic
nodules around the central artery.

Question: CLASS
The organ depicted in this photomicroscopic image is best identified as the

Answer Choices:
A Spleen
B Thymus
C Bone marrow
D Liver
E Lymph node

Image(s) / Chart(s):
Click image to view full size. Click open image to close. Click and hold open image to move.
A Spleen

This image represents an image of a spleen at low power. Note several characteristics such as a
prominent connective tissue capsule with trabeculae penetrating the parenchyma of the gland,
and the prevalent white pulp and red pulp. The white pulp appears as the circular and/or
elongated areas of gray areas consisting of accumulations of lymphocytes. The white pulp is
surrounded by the red pulp, primarily consisting of red blood cells.

Question:
Discontinuous or sinusoidal capillaries are characteristic of the type of capillary found in which one of
the following locations?

Answer Choices:
A Thyroid gland
B Anterior pituitary gland
C Duodenum
D Spleen
E Lung

D Spleen

Based upon their ultrastructural characteristics, capillary endothelia may be described as being
continuous, fenestrated or discontinuous (sinusoidal). Discontinuous capillaries
characteristically have larger diameter and more irregularly shaped lumen than that of other
capillaries. Furthermore, these large diameter capillaries may have gaps between endothelial
cells and have sparse or absent basal lamina underlying the endothelium. These types of
capillaries are common to the spleen, but are also found in the liver and in bone marrow.
Question:
Splenic vein originate from

Answer Choices:
A trabecular veins
B trabecular arteries
C Penicillar arterioles
D Central arteries
E White pulp arteries

A trabecular veins

From the red pulp sinusoids, blood proceeds to the red pulp veins that join together and enter
the trabeculae, forming the trabecular veins. The splenic vein originates from these vessels
and emerges from the hilum of the spleen. The trabecular veins do not have individual muscle
walls; i.e. their walls are composed of trabecular tissue. They can be considered channels
hollowed out in the trabecular connective tissue and lined by endothelium.

Question:
Which of the following tissue is derived from the endoderm?

Answer Choices:
A Cartilage
B Mammary glands
C Pigment cells of the dermis
D Parenchyma of the tonsils

D Parenchyma of the tonsils

Ectoderm gives rise to the central nervous system (brain and spinal cord), the peripheral
nervous system, the sensory epithelia of the eye, ear, and nose, the epidermis and its
appendages (hair and nails), mammary glands, cerebral hypophysis, subcutaneous glands, and
the enamel of the teeth. Neural crest cells, derived from neuroectoderm, give rise to the cells of
the spinal, cranial, and autonomic ganglia, ensheathing cells of the peripheral nervous system,
pigment cells of the dermis, muscle, connective tissues, and bone of branchial arch origin, the
suprarenal (adrenal) medulla, and the meninges of the brain and spinal cord.
Mesoderm gives rise to connective tissue, cartilage, bone, striated and smooth muscles, the
heart, blood and lymph vessels and cells, the kidneys, ovarian and testes, the genital ducts,
serous membranes lining the body cavities (pericardial, pleural, and peritoneal), the spleen, and
the cortex of the suprarenal gland.
Endoderm gives rise to the epithelial lining of the gastrointestinal and respiratory tracts, the
parenchyma of the tonsils, thyroid and parathyroid glands, thymus, liver, and pancreas, the
epithelial lining of the urinary bladder and most of the urethra, and the epithelial lining of the
tympanic cavity, tympanic antrum, and auditory tube.
Question:
The structure indicated by the arrow in the photomicroscopic image is best identified as

Answer Choices:
A Hassall's corpuscle
B Germinal center
C Central artery
D Penicillar arteriole
E Reticulo-epithelial cell

Image(s) / Chart(s):
Click image to view full size. Click open image to close. Click and hold open image to move.

C Central artery

The arrow marks the location of the central artery within the white pulp of the spleen. The
lymphocytes which aggregate around the central artery as it courses through the spleen form a
cylindrical pattern around the artery. On cross section, as is visualized by this image, the central
artery occupies an eccentric position as it courses through the lymphatic sheath and the sheath
is observed as an apparent ball of lymphocytes rather than a cylinder.

Question: CLASS Q
The arrows in the photomicroscopic image depict what structure found in the spleen?

Answer Choices:
A Periarterial lymphatic sheath
B Central artery
C Splenic trabecula
D Penicillar arteriole
E Splenic nodule

Image(s) / Chart(s):
Click image to view full size. Click open image to close. Click and hold open image to move.

D Penicillar arteriole

As the central artery continues into the red pulp (loosing the periarterial lymphatic sheath), it
may branch into several straight arterioles called Penicillar arterioles. These arterioles, as
depicted by the arrows in the photomicroscopic image, allow for either an "open circulation" or
"closed circulation" model allowing blood to be filtered through the splenic sinuses of the red
pulp.

Question: CLASS QUIZ


Which one of the following statements concerning the blood flow through the spleen is true?

Answer Choices:
A Blood moves through the spleen in an open type of circulation, and the flow through
the pulp is controlled by rhythmic contraction and relaxation of individual or groups of
arterioles
B The movement of blood through the spleen is relatively fast, and does not touch the
reticular network of the sinuses
C The recent majority opinion is that splenic arterioles are connected with venules by
capillaries that have a complete endothelial lining
D Splenic blood arterioles have fenestrated endothelial linings, and as a consequence,
do not penetrate the lymphatic nodules
E The splenic capillaries supply Hassall's corpuscles with the medulla, and within these
bodies they connect with venules

A Blood moves through the spleen in an open type of circulation, and the flow through the
pulp is controlled by rhythmic contraction and relaxation of individual or groups of arterioles

Some authors have claimed that splenic arterioles are connected with venules by capillaries and
that they have a complete endothelial lining. However, it has been recently confirmed that the
endothelial connections are incomplete. In this open type of circulation the blood moves slowly
through the splenic pulp and bathes the reticular network of sinuses. The flow through the pulp
is controlled by rhythmic contraction and relaxation of individual arterioles and groups of them.
The altered coat of the arterioles, consisting of lymphoid tissue, presents here and there
thickening of lymphatic nodules. Hassall's corpuscles are found only in the thymus, and not in
the spleen.

Question:
A 44-year-old Caucasian woman is admitted to the hospital with a severe nosebleed. The patient
states that there was no history of trauma and she never has had nosebleeds before. She also reports
a history of upper respiratory infection (URI) symptoms a week ago. The patient's blood work is
notable for a platelet count of 10,000/cmm. A presumptive diagnosis of idiopathic thrombocytopenic
purpura (ITP) is made, and the patient is treated with steroids. The patient's platelet count is
refractory and a splenectomy is performed. After a few months, which of the following would you
expect to see in this patient's peripheral blood smears?

Answer Choices:
A Burr cells
B Howell-Jolly bodies
C Hypersegmented neutrophils
D Schistocytes
E Spherocytes

B Howell-Jolly bodies

Howell-Jolly bodies are basophilic red cell inclusions commonly found in splenectomy patients.
This patient has had a splenectomy after being diagnosed with ITP, and therefore, the most
likely finding in this patient's peripheral smear would be Howell-Jolly bodies. Howell-Jolly
bodies are round, basophilic staining nuclear fragments of DNA in the red blood cell. This
finding is classic in patients who have had splenectomy, and in the case of sickle cell disease in
which the spleen auto-infarcts.
Burr cells are triangulated helmet shaped cells that are typically seen in diseases of small blood
vessels. They can be seen in a number of different diseases including uremia and pyruvate
kinase deficiency
Hypersegmented neutrophils are found in folate and B12 deficiency.
Schistocytes are commonly found in Thrombotic Thrombocytopenic Purpura and Disseminated
Intravascular Coagulation. A schistocyte is a red blood cell undergoing fragmentation, or a
fragmented part of a red blood cell.
Spherocytes are found in hereditary spherocytosis. These patients have the classic triad of
jaundice, gallstones, and anemia.

SECTION-SKIN
Question:
A nail (fingernail or toenail) is made of three layers on cross-section. Each layer has characteristic
properties. Of the three layers of a nail, which layer makes up the bulk of the nail?

Answer Choices:
A Ventral layer
B Dorsal layer
C Intermediate layer
D Epidermal layer
E Vascular layer
C Intermediate layer

The dorsal layer of the nail is a few cells thick and contains only hard keratin. The ventral layer
consists of a few cornified cells. The intermediate layer makes up the bulk of the nail and
contains softer keratin. The nail does not contain an epidermal layer or vascular layer.
Question:
The dermis is the thickest in the:

Answer Choices:
A Soles of feet
B Palms of hand
C Upper portion of the back
D Eyelids
E Perineum

C Upper portion of the back


Anatomically, the thickest skin on the body is found on the upper portion of the back where the
dermis is exceedingly thick. The palms of the hand and the soles of the feet, which are subject
to most abrasion, are hairless and have a much thicker epidermal layer than the skin in any
other location. Both eyelids and the perineum have thin epidermis.

Question:
The epidermis is connected to the dermis by

Answer Choices:
A Reticular fibers, which constitute most of the dermis
B The smooth muscle fibers
C A mixture of smooth muscle and reticular fibers
D Numerous connective tissue conical projections, called papillae
E Cells of Langerhans

D Numerous connective tissue conical projections, called papillae


Image(s) / Chart(s):
Click image to view full size. Click open image to close. Click and hold open image to move.

The vascular dermis consists of several irregular, felt-like layers of collagenous fibers with a
scattering of elastic fibers. It is anchored to the epidermis by numerous connective tissue
conical projections, the papillae.
Reticular fibers form the thicker and deeper fibrous reticular layer of the dermis. The fibers of
the reticular layer intermingle with the superficial papillary dermal layer. S muscle fibers are not
found in either the epidermis or dermis.
Cells of Langerhans are stellate-shaped cells, found principally in stratum spinosum. Although
their origin and functions are obscure, it is thought that they are special type of macrophages
that entrap exogenous antigens in the skin. Thus, they may be involved in the primary immune
body response.
See Figure: Skin, epidermis (>E<) and, papillary layer.
Question:
Keratinization, or the formation of "soft" keratin in the skin, occurs principally in which layer of the
skin in the photomicroscopic image?

Answer Choices:
A A
B B
C C
D D
E Among the cells in the layer indicated by the arrowheads

Image(s) / Chart(s):
Click image to view full size. Click open image to close. Click and hold open image to move.

C C

Skin is an organ composed of stratified squamous epithelium that have distinctive layers of
cells. In the photomicroscopic image, the letter 'A' indicates the stratum corneum, the letter 'B'
indicates the stratum lucidum, the letter 'C' indicates the stratum granulosum, and the letter 'D'
indicates the stratum spinosum.
The arrowheads point to cells of the stratum basale.
The keratinocytes of the upper layers of the stratum spinosum begin to synthesize keratohyalin
granules and lamellar bodies, but this occurs principally in the cells of the stratum granulosum,
indicated by 'C' in the image. Tonofibrils combine with the keratohyalin granules in the cells
found in the stratum granulosum to form a complex called soft keratin common to skin.
Question:
The cells responsible for producing pigment are located in which layer of the photomicroscopic
image?

Answer Choices:
A A
B B
C C
D D
E Among the cells indicated by the arrowheads

Image(s) / Chart(s):
Click image to view full size. Click open image to close. Click and hold open image to move.

E Among the cells indicated by the arrowheads

Skin is an organ composed of stratified squamous epithelium that have distinctive layers of
cells. In the photomicroscopic image, the letter 'A' indicates the stratum corneum, the letter 'B'
indicates the stratum lucidum, the letter 'C' indicates the stratum granulosum, and the letter 'D'
indicates the stratum spinosum.
The arrowheads point to cells of the stratum basale.
Pigmentation of the epidermis is a function of the melanocyte. Melanocytes are located among
the basal cells of the stratum basale. These cells are of neural crest origin and are "dendritic"
cells having long processes that extend between the keratinocytes of the stratum spinosum.
One melanocyte will maintain an association with a given number of cells in the stratum
spinosum.
Question:
The cells located in the region denoted by the arrowheads in the photomicroscopic image are part of
which layer?

Answer Choices:
A Stratum basale
B Stratum corneum
C Stratum lucidum
D Stratum granulosum
E Stratum spinosum

Image(s) / Chart(s):
Click image to view full size. Click open image to close. Click and hold open image to move.

A Stratum basale

Skin is an organ composed of stratified squamous epithelium that have distinctive layers of
cells. In the photomicroscopic image, the letter 'A' indicates the stratum corneum, the letter 'B'
indicates the stratum lucidum, the letter 'C' indicates the stratum granulosum, and the letter 'D'
indicates the stratum spinosum.
The arrowheads point to cells of the stratum basale. The cells of this layer rest on a basal
lamina, separating the epidermis from the underlying dermis. These cells contain the stem cells
for the epidermis, although mitosis may occur in both the stratum basale and in the cells of the
stratum spinosum. The single layer of cells in the stratum basale are usually cuboidal or low
columnar in shape. Daughter cells from mitotic divisions in this layer begin the process of
migrating upward through the layers of the epidermis, eventually to be sloughed off of the
epithelial surface.
Question:
The cells located in the region denoted by the line right of the letter 'C' in the photomicroscopic image
are part of which layer?

Answer Choices:
A Stratum basale
B Stratum corneum
C Stratum lucidum
D Stratum granulosum
E Stratum spinosum

Image(s) / Chart(s):
Click image to view full size. Click open image to close. Click and hold open image to move.

D Stratum granulosum

Skin is an organ composed of stratified squamous epithelium that have distinctive layers of
cells. In the photomicroscopic image, the letter 'A' indicates the stratum corneum, the letter 'B'
indicates the stratum lucidum, the letter 'C' indicates the stratum granulosum, and the letter
'D' indicates the stratum spinosum.
The arrowheads point to cells of the stratum basale. The cells of the stratum granulosum
contain many prominent keratohyalin granules. These granules are rich in cystine and histidine
and are easily seen due to their propensity to stain intensely.

Question:
The cells located in the region denoted by the line right of the letter 'D' in the photomicroscopic
image are part of which layer?
Answer Choices:
A Stratum basale
B Stratum corneum
C Stratum lucidum
D Stratum granulosum
E Stratum spinosum

Image(s) / Chart(s):
Click image to view full size. Click open image to close. Click and hold open image to move.

E Stratum spinosum

Skin is an organ composed of stratified squamous epithelium that have distinctive layers of
cells. In the photomicroscopic image, the letter 'A' indicates the stratum corneum, the letter 'B'
indicates the stratum lucidum, the letter 'C' indicates the stratum granulosum, and the letter 'D'
indicates the stratum spinosum.
The arrowheads point to cells of the stratum basale. The stratum spinosum layer is variable in
thickness, but consists of large, polygonal cells that appear to have short spines or extensions
between adjacent cells. Because of their "spiny" appearance, these cells also have been
referred to as "prickle" cells. The spiny appearance is due to the presence of desmosomes that
are located between adjacent cells. This appearance also is enhanced by the shrinkage of the
cells that occurs during preparation, except at the point of sites of the desmosomes.
Question:
The organ visualized by the photomicroscopic image is best identified as

Answer Choices:
A Thin skin
B Urinary bladder
C Thick skin
D Vagina
E Gall bladder

Image(s) / Chart(s):
Click image to view full size. Click open image to close. Click and hold open image to move.

C Thick skin

The photomicroscopic image displays the histological characteristics of thick skin. Thin skin is
histologically similar except that thin skin does NOT possess a stratum lucidum.
The skin in the image can be classified as stratified squamous, keratinized. This would eliminate
the skin observed in the vagina, since the vagina is normally lined by stratified squamous, non-
keratinized epithelium. The urinary bladder is lined by transitional epithelium, and the gall
bladder is lined by simple columnar epithelium.

Question:
The nail plate as found on the dorsal aspect of the distal phalanges has a 'free' (distal) edge, and an
'attached' proximal end. The boundary layer associated with the distal or free end of the nail is the

Answer Choices:
A Eponychium
B Hyponychium
C Endonychium
D Perionychium
E Nychium
B Hyponychium

The hyponychium is the boundary layer of skin associated with the distal or free end of the nail.
This layer is continuous with the stratum corneum of the adjacent epidermis. The junction of
the nail with the skin on the proximal end is the eponychium, which also is continuous with the
stratum corneum.

Question:
The germinative cells are located in which region of the growing hair follicle?

Answer Choices:
A Matrix
B Outer root sheath
C Cuticle
D Dermal papilla
E Keratogenous zone

A Matrix

Matrix cells are found in the bulb of the growing hair follicle and those cells immediately
adjacent to the dermal papilla represent the germinative layer of the follicle.

Question:
Formation of keratin in the hair shaft and internal root sheath occurs in which region of the follicle?

Answer Choices:
A Keratogenous zone
B Bulb
C Cuticle formation zone
D Matrix region
E Granulosum zone

A Keratogenous zone

Shortly after cells leave the germinal area of the bulb, or the matrix, the internal root sheath
cells differentiate into keratin-producing cells. The cortical portion of the hair follicle and the
internal root sheath cells begin the process of keratinization in the keratogenous zone near the
matrix. The cuticle, which is a part of the inner root sheath consist of overlapping cells that
become filled with keratin early in the process. The hair is entirely keratinized by the time it
emerges from the follicle.

Question:
Which letter in the photomicroscopic image would best identify the location of the extension of the
epidermis from the skin's surface?

Answer Choices:
A A
B B
C C
D D
E E

Image(s) / Chart(s):
Click image to view full size. Click open image to close. Click and hold open image to move.

C C

In the photomicroscopic image, the letter 'a' indicates the location of the dermal papilla, which
is a tuft of dermal connective tissue within the bulb at the base of the hair follicle. The letter 'b'
indicates the location of the inner root sheath. The letter 'c' indicates the location of the outer
root sheath, which is continuous with the epidermis of the skin. The letter 'd' indicates the
location of the medulla of the developing hair shaft, and the letter 'e' indicates the location of
the cortex of the developing hair shaft.

uestion:
Which letter in the photomicroscopic image would best identify the location of the cortex of the
developing hair shaft?

Answer Choices:
A A
B B
C C
D D
E E

Image(s) / Chart(s):
Click image to view full size. Click open image to close. Click and hold open image to move.

E E

In the photomicroscopic image, the letter 'a' indicates the location of the dermal papilla, which
is a tuft of dermal connective tissue within the bulb at the base of the hair follicle. The letter 'b'
indicates the location of the inner root sheath. The letter 'c' indicates the location of the outer
root sheath, which is continuous with the epidermis of the skin. The letter 'd' indicates the
location of the medulla of the developing hair shaft, and the letter 'e' indicates the location of
the cortex of the developing hair shaft.

Question:
Which letter in the photomicroscopic image would best identify the location of the medulla of the
developing hair shaft?

Answer Choices:
A A
B B
C C
D D
E E

Image(s) / Chart(s):
Click image to view full size. Click open image to close. Click and hold open image to move.
D D

In the photomicroscopic image, the letter 'a' indicates the location of the dermal papilla, which
is a tuft of dermal connective tissue within the bulb at the base of the hair follicle. The letter 'b'
indicates the location of the inner root sheath. The letter 'c' indicates the location of the outer
root sheath, which is continuous with the epidermis of the skin. The letter 'd' indicates the
location of the medulla of the developing hair shaft, and the letter 'e' indicates the location of
the cortex of the developing hair shaft.
Question:
Which letter in the photomicroscopic image would best identify the location of the dermal papillae?

Answer Choices:
A A
B B
C C
D D
E E

Image(s) / Chart(s):
Click image to view full size. Click open image to close. Click and hold open image to move.

A A
In the photomicroscopic image, the letter 'a' indicates the location of the dermal papilla, which
is a tuft of dermal connective tissue within the bulb at the base of the hair follicle. The letter 'b'
indicates the location of the inner root sheath. The letter 'c' indicates the location of the outer
root sheath, which is continuous with the epidermis of the skin. The letter 'd' indicates the
location of the medulla of the developing hair shaft, and the letter 'e' indicates the location of
the cortex of the developing hair shaft.
Question:
The structures as indicated by the arrows in this photomicroscopic image are referred to as

Answer Choices:
A Rete ridges
B Dermal papillae
C Panniculus adiposis
D Prickle cells
E Arrector pili

Image(s) / Chart(s):
Click image to view full size. Click open image to close. Click and hold open image to move.

B Dermal papillae

The numerous finger-like connective tissue protrusions of dermis into the basal surface of the
epidermis are called dermal papillae. These structures serve to provide better attachment of
the epidermis to the dermis. Rete ridges are the epidermal extensions that project into the
dermis (between the arrows). Panniculus adiposis is the adipose tissue layer deep to the dermal
layer of the skin. Prickle cells, are contained within the stratum spinosum layer of the
epidermis, and derive their name from intercellular connections. The arrector pili are smooth
muscles connecting the deep part of hair follicles to the superficial dermis, and when
contracted, produce an erection of the hairs.
Question:
The function of the structures at the location of the arrows depicted in the photomicroscopic image is
to

Answer Choices:
A Provide barrier to liquids on the surface of the epidermis
B Enhance attachment of epidermis to dermis
C Enhance interface between epidermis and dermis in areas of little mechanical stress
D Increase blood vessel surface area to epidermis
E Reduce interface between epidermis and dermis in areas of increased mechanical
stress

Image(s) / Chart(s):
Click image to view full size. Click open image to close. Click and hold open image to move.

B Enhance attachment of epidermis to dermis

The numerous finger-like connective tissue protrusions of dermis into the basal surface of the
epidermis are called dermal papillae. These structures serve to provide better attachment of
the epidermis to the dermis. In doing so, they will serve to enhance the interface between the
epidermis and dermis in areas of increased mechanical stress.

Question:
Which letter in the photomicroscopic image would best identify the location of the outer root sheath?
Answer Choices:
A a
B b
C c
D d
E e

Image(s) / Chart(s):
Click image to view full size. Click open image to close. Click and hold open image to move.

C c

In the photomicroscopic image, the letter 'a' indicates the location of the dermal papilla, which
is a tuft of dermal connective tissue within the bulb at the base of the hair follicle. The letter 'b'
indicates the location of the inner root sheath. The letter 'c' indicates the location of the outer
root sheath, which is continuous with the epidermis of the skin. The letter 'd' indicates the
location of the medulla of the developing hair shaft, and the letter 'e' indicates the location of
the cortex of the developing hair shaft.
Question:
"Baldness" is best described by which classification?

Answer Choices:
A Follicular anagen
B Follicular catagen
C Follicular telogen
D Terminal hair
E Vellus hair

E Vellus hair
Vellus hair is characterized by short, fine, hairs that may not be easily visible, without
magnification. These hairs dominate the "bald" head, such that, baldness is not the absence of
hair, but rather the presence of the vellus hair type. During the balding process, terminal hairs,
which are long, coarse hairs, common to the scalp, become gradually converted to vellus hair
follicles which give rise to vellus hair. Vellus hair follicles characteristically remain in a telogen
state, a period of follicular rest, for relatively long periods of time. Follicular anagen refers to a
growth period of the hair follicle, and follicular catagen refers to a brief period, in which hair
growth normally stops.
Question:
The nail plate as found on the dorsal aspect of the distal phalanges has a 'free' (distal) edge, and an
'attached' proximal end. The boundary layer associated with the proximal end of the nail is the

Answer Choices:
A Eponychium
B Hyponychium
C Endonychium
D Perionychium
E Nychium

A Eponychium

The hyponychium is the boundary layer of skin associated with the distal or free end of the nail.
This layer is continuous with the stratum corneum of the adjacent epidermis. The junction of
the nail with the skin on the proximal end is the eponychium, which also is continuous with the
stratum corneum.

Question:
Identify the layer of skin indicated by the line to the right of the letter 'A' in the photomicroscopic
image

Answer Choices:
A Stratum basale
B Stratum granulosum
C Stratum spinosum
D Stratum lucidum
E Stratum corneum

Image(s) / Chart(s):
Click image to view full size. Click open image to close. Click and hold open image to move.
E Stratum corneum

Skin is an organ composed of stratified squamous epithelium that have distinctive layers of
cells. In the photomicroscopic image, the letter 'A' indicates the stratum corneum, the letter 'B'
indicates the stratum lucidum, the letter 'C' indicates the stratum granulosum, and the letter 'D'
indicates the stratum spinosum.
The arrowheads point to cells of the stratum basale. The cells in the stratum corneum are
anucleate, squamous cells (hence the classification of stratified squamous epithelium), filled
with keratin. This layer varies in thickness, and is thickest in thick skin, such as on the palms of
the hands or soles of the feet.
Question:
The keratinocytes of the epidermis are embryologically derived from what germ layer?

Answer Choices:
A Neural crest cells
B Surface ectoderm
C Endoderm
D Somatic mesoderm
E Splanchnic mesoderm

B Surface ectoderm

Keratinocytes are the epidermal cells that arise from the stratum basale by mitosis. These cells
eventually undergo keratinization and are sloughed from the surface of the skin. The embryonic
origin of these cells is from the surface ectoderm.

Question:
Which layer of the skin contains end-bulbs of Krause?

Answer Choices:
A Dermal reticular layer
B Dermal papillary layer
C Epidermal stratum basale (germinativum)
D Epidermal stratum spinosum
E Epidermal stratum granulosum

A Dermal reticular layer

Dermal reticular layer contains end-bulbs of Krause (cold and pressure receptors). Dermal
papillary layer contains capillary loops and Meissner's corpuscles. Epidermal stratum basale
contains melanocytes and Merkel's cells. Epidermal stratum spinosum contains Langerhans'
cells, while stratum granulosum contains flattened keratinocytes.

Question:
Defects in the formation of normal keratin filaments leads to

Answer Choices:
A Defects in mitosis
B Development of skin abnormalities
C Defects in neuromuscular function
D Defects in protein secretion
E Development of decreased immune function

B Development of skin abnormalities

Intermediate filaments are composed of proteins called keratins, which are a large family of
over 20 distinct proteins found in human epithelia. In addition, there are at least 8 more "hard"
keratins found in hair and nails. Keratins are expressed in epithelial cells and each cell
synthesizes at least one of the Type I, acidic group, and one of the Type II, neutral/basic group.
These 2 types copolymerize to give rise to intermediate filaments. A single epithelial cell may
synthesize several different types of keratin proteins. Intermediate filaments, in contrast to
actin filaments and microtubules, are not involved in cell movement but play a role in providing
mechanical strength to tissues and cells. Experiments using transgenic mice containing
defective keratins showed the direct involvement of keratins in maintaining the mechanical
strength of epithelial cells. These defective mice developed severe skin abnormalities including
blistering and cell lysis even from minor skin trauma. Microtubules are made of polymerized
tubulin. Cilia and flagella are composed of microtubules.

Question:
The expansion of a growing hair follicle that is formed by various cellular components at its base is
called a

Answer Choices:
A Terminal
B Bulb
C Club
D Papillae
E Zonulas

B Bulb

When examining a growing follicle, it is relatively uniform in diameter, except at its base where
it forms an expansion called a bulb. The bulb is formed, in part, by an invagination of dermal
connective tissue called a dermal papilla.
Question:
Which letter in the photomicroscopic image would best identify the location of the inner root sheath?

Answer Choices:
A A
B B
C C
D D
E E

Image(s) / Chart(s):
Click image to view full size. Click open image to close. Click and hold open image to move.
B B

In the photomicroscopic image, the letter 'a' indicates the location of the dermal papilla, which
is a tuft of dermal connective tissue within the bulb at the base of the hair follicle. The letter 'b'
indicates the location of the inner root sheath. The internal root sheath consists of a cuticle,
Huxley's layer and Henle's layer (not identifiable on this image). The letter 'c' indicates the
location of the outer root sheath, which is continuous with the epidermis of the skin. The letter
'd' indicates the location of the medulla of the developing hair shaft, and the letter 'e' indicates
the location of the cortex of the developing hair shaft.

Question:
Hair color is a function of the melanocytes present in which part of the hair follicle?

Answer Choices:
A Matrix cells
B Cuticle
C Outer root sheath
D Cortex
E Inner root sheath

A Matrix cells

Melanocytes are scattered among the matrix cells of the germinative layer of the follicle.
Melanosomes are contributed to the developing hair cells, which imparts the basis of color to
the hair shaft.
Question:
Growth of the nail plate occurs from which part of the nail?
Answer Choices:
A Nail bed
B Eponychium
C Nail root
D Hyponychium
E Dermis

C Nail root

The nail plate is derived as keratinized cells arising from the proximal part of the nail, the nail
root. The nail root is buried in a fold of epidermis where the germinative cells are located. The
germinative cells divide regularly and migrate to the root of the nail where they produce
keratin. There is a constant addition of new cells at the nail root, which accounts for nail plate
growth. The nail bed is a layer of epithelial cells continuous with the stratum basale and the
stratum spinosum on which the nail plate rests. The hyponychium is the boundary layer of skin
associated with the distal or free end of the nail. This layer is continuous with the stratum
corneum of the adjacent epidermis. The junction of the nail with the skin on the proximal end is
the eponychium, which also is continuous with the stratum corneum.

Question:
The example of skin as seen in this photomicrographic image is found at which one of the following
locations?

Answer Choices:
A Lip
B Chest
C Vagina
D Sole of foot
E Penis

Image(s) / Chart(s):
Click image to view full size. Click open image to close. Click and hold open image to move.
D Sole of foot

The type of skin as seen in the image depicts thick skin of the keratinized variety because of the
presence of the layers labeled 'B' (stratum lucidum) and 'A' (stratum corneum). Thick skin is
limited to areas of the body such as the sole of the foot and palms of the hands. Skin on the lip,
chest, and penis are examples of thin skin, which lacks the stratum lucidum of the keratinized
variety. Skin lining the vagina is thin skin of the non-keratinized variety.

Question:
Intercellular bridges are characteristic of which of the following cellular layers of the epidermis?

Answer Choices:
A Stratum corneum
B Stratum lucidum
C Stratum granulosum
D Stratum basale
E Stratum spinosum

E Stratum spinosum

Intracellular bridges, as shown by the electron microscope, are associated with desmosomes
(maculae adherentes) that link the processes of adjacent cells in the stratum spinosum.
Desmosomes may also provide communication between cells of other epidermal layers, but
these do not form characteristic "bridges". The keratinocytes of stratum basale also contain
hemidesmosomes, and these attach the cells of this layer to underlying basal lamina.

Question:
Meissner's corpuscles are present in which of the following regions of the skin?

Answer Choices:
A Dermal reticular layer
B Dermal papillary layer
C Hypodermis
D Stratum basale
E Epidermal ridges

B Dermal papillary layer


Image(s) / Chart(s):
Click image to view full size. Click open image to close. Click and hold open image to move.

Meissner's corpuscles are encapsulated nerve endings present in dermal papillae, which are
part of the papillary layer of the dermis. They function as receptors for fine touch.
Dermal reticular layer may contain pacinian corpuscles (pressure receptors) and end-bulbs of
Krause (cold and pressure receptors) in its deeper aspects. Hypodermis (subcutaneous tissue)
contains fat cells. Epidermal stratum basale contains melanocytes and Merkel's cells, and
epidermal ridges are evaginations of the epidermis that accommodate dermal papillae.
See Figure: Meissner's corpuscle in the dermal papillary layer.

Question:
A nail (fingernail or toenail) is made of three layers on cross-section. Each layer has characteristic
properties. Which layer consists of only hard keratin and is only a few cells thick?
Answer Choices:
A Ventral layer
B Dorsal layer
C Intermediate layer
D Epidermal layer
E Vascular layer
B Dorsal layer

The dorsal layer of the nail is a few cells thick and contains only hard keratin. The ventral layer
consists of a few cornified cells. The intermediate layer makes up the bulk of the nail and
contains softer keratin. The nail does not contain an epidermal layer or vascular layer.
Case #301558:
This photomicrograph is a full-thickness specimen of the epidermis of normal human skin. It
also includes parts of 2 dermal papillae in the bottom of the image. Match the labeled
structure in the photomicrograph with the MOST appropriate structural or functional
description in the item below.
Question:
Dividing cells in what layer have protective caps of melanosomes shielding their nuclei from
carcinogenic ultraviolet radiation?

Answer Choices:
A A
B B
C C
D D
E E

Image(s) / Chart(s):
Click image to view full size. Click open image to close. Click and hold open image to move.
E E

This is a photomicrograph of normal human thin skin. It is an example of stratified squamous


keratinized epithelium. Stratified squamous keratinized epithelium also occurs on the tongue
and the gingivae, but is not properly considered to be the epidermis of skin.

The epidermis is constantly turning over. Stem cells in the stratum basale (E) divide repeatedly
and begin to differentiate into cells of the most superficial part of the epidermis, the stratum
corneum. There is usually a neat umbrella-shaped cap of melanin granules protecting the apical
side of the nuclei of dividing cells of the stratum basale from carcinogenic ultraviolet (UV)
radiation from the sun. Melanin is rich in aromatic amino acids that absorb UV radiation in
sunlight.

Thin skin is abundant in the face and has a relatively thin stratum corneum (A), unlike thick
skin, which is abundant in areas of the body subjected to repeated abrasive forces, such as the
palms of the hands and the soles of the feet.

Mitotic pressure pushes cells away from the stratum basale. Desmosomes, firm intercellular
junctions with many associated keratin-rich intermediate filaments, connect the cells to one
another, but give the cells in the stratum spinosum (D). As these cells continue to differentiate
and migrate apically, they accumulate keratohyaline granules, which are responsible for the
purple granularity of cells in the stratum granulosum (C). The red layer between the stratum
corneum (A) and the stratum granulosum (C) is called the stratum lucidum (B). After complete
differentiation (keratinization), cells are sloughed from the apical surface of the stratum
corneum.
Case #301559:
This photomicrograph is a full-thickness specimen of the epidermis of normal human skin. It
also includes parts of 2 dermal papillae in the bottom of the image. Match the labeled
structure in the photomicrograph with the MOST appropriate structural or functional
description in the item below.
Question:
Cells in what layer have numerous keratohyaline granules?

Answer Choices:
A A
B B
C C
D D
E E

Image(s) / Chart(s):
Click image to view full size. Click open image to close. Click and hold open image to move.

C C

Thin skin is abundant in the face and has a relatively thin stratum corneum (A), unlike thick
skin, which is abundant in areas of the body subjected to repeated abrasive forces, such as the
palms of the hands and the soles of the feet.

The epidermis is constantly turning over. Stem cells in the stratum basale (E) divide repeatedly
and begin to differentiate into cells of the most superficfial part of the epidermis, the stratum
corneum. There is usually a neat umbrella-shaped cap of melanin granules protecting the apical
side of the nuclei of dividing cells of the stratum basale from carcinogenic ultraviolet (UV)
radiation from the sun. Melanin is rich in aromatic amino acids, which absorb UV radiation in
sunlight.

Mitotic pressure pushes cells away from the stratum basale. Desmosomes, firm intercellular
junctions with many associated keratin-rich intermediate filaments, connect the cells to one
another, but give the cells in the stratum spinosum (D). As these cells continue to differentiate
and migrate apically, they accumulate keratohyaline granules, which are responsible for the
purple granularity of cells in the stratum granulosum (C). The red layer between the stratum
corneum (A) and the stratum granulosum (C) is called the stratum lucidum (B). After complete
differentiation (keratinization), cells are sloughed from the apical surface of the stratum
corneum.
Case #301560:
This photomicrograph is a full-thickness specimen of the epidermis of normal human skin. It
also includes parts of 2 dermal papillae in the bottom of the image. Match the labeled
structure in the photomicrograph with the MOST appropriate structural or functional
description in the item below.
Question:
Cells in what layer have prominent spinous processes, representing firm desmosomal
intercellular junctions?

Answer Choices:
A A
B B
C C
D D
E E

Image(s) / Chart(s):
Click image to view full size. Click open image to close. Click and hold open image to move.
D D

Thin skin is abundant in the face and has a relatively thin stratum corneum (A), unlike thick
skin, which is abundant in areas of the body subjected to repeated abrasive forces, such as the
palms of the hands and the soles of the feet.

The epidermis is constantly turning over. Stem cells in the stratum basale (E) divide repeatedly
and begin to differentiate into cells of the most superficial part of the epidermis, the stratum
corneum. There is usually a neat umbrella-shaped cap of melanin granules protecting the apical
side of the nuclei of dividing cells of the stratum basale from carcinogenic ultraviolet (UV)
radiation from the sun. Melanin is rich in aromatic amino acids, which absorb UV radiation in
sunlight.

Mitotic pressure pushes cells away from the stratum basale. Desmosomes, firm intercellular
junctions with many associated keratin-rich intermediate filaments, connect the cells to one
another, and are particularly abdunant in the stratum spinosum (D). As these cells continue to
differentiate and migrate apically, they accumulate keratohyaline granules, which are
responsible for the purple granularity of cells in the stratum granulosum (C). The red layer
between the stratum corneum (A) and the stratum granulosum (C) is called the stratum
lucidum (B). After complete differentiation (keratinization), cells are sloughed from the apical
surface of the stratum corneum.
Case #301561:
This photomicrograph is a full-thickness specimen of the epidermis of normal human skin. It
also includes parts of 2 dermal papillae in the bottom of the image. Match the labeled
structure in the photomicrograph with the MOST appropriate structural or functional
description in the item below.
Question:
Which layer is called the stratum corneum?

Answer Choices:
A A
B B
C C
D D
E E

Image(s) / Chart(s):
Click image to view full size. Click open image to close. Click and hold open image to move.

A A

Thin skin is abundant in the face and has a relatively thin stratum corneum (A), unlike thick
skin, which is abundant in areas of the body subjected to repeated abrasive forces, such as the
palms of the hands and the soles of the feet.
The epidermis is constantly turning over. Stem cells in the stratum basale (E) divide repeatedly
and begin to differentiate into cells of the most superficial part of the epidermis, the stratum
corneum. There is usually a neat umbrella-shaped cap of melanin granules protecting the apical
side of the nuclei of dividing cells of the stratum basale from carcinogenic ultraviolet (UV)
radiation from the sun. Melanin is rich in aromatic amino acids, which absorb UV radiation in
sunlight.

Mitotic pressure pushes cells away from the stratum basale. Desmosomes, firm intercellular
junctions with many associated keratin-rich intermediate filaments, connect the cells to one
another, but give the cells in the stratum spinosum (D). As these cells continue to differentiate
and migrate apically, they accumulate keratohyaline granules, which are responsible for the
purple granularity of cells in the stratum granulosum (C). The red layer between the stratum
corneum (A) and the stratum granulosum (C) is called the stratum lucidum (B). After complete
differentiation (keratinization), cells are sloughed from the apical surface of the stratum
corneum.
Case #301562:
This photomicrograph is a full-thickness specimen of the epidermis of normal human skin. It
also includes parts of 2 dermal papillae in the bottom of the image.
Question:
This variety of skin would be found MOST commonly in which location in the body?

Answer Choices:
A Sole of foot
B Palm of hand
C Anywhere where skin was subjected to regular abrasion or pressure
D Upper surface of tongue and gingival
E Forehead

Image(s) / Chart(s):
Click image to view full size. Click open image to close. Click and hold open image to move.
E Forehead

Thin skin is abundant in the face and has a relatively thin stratum corneum (A), unlike thick
skin, which is abundant in areas of the body subjected to repeated abrasive forces, such as the
palms of the hands and the soles of the feet.

The epidermis is constantly turning over. Stem cells in the stratum basale (E) divide repeatedly
and begin to differentiate into cells of the most superficfial part of the epidermis, the stratum
corneum. There is usually a neat umbrella-shaped cap of melanin granules protecting the apical
side of the nuclei of dividing cells of the stratum basale from carcinogenic ultraviolet (UV)
radiation from the sun. Melanin is rich in aromatic amino acids, which absorb UV radiation in
sunlight.

Mitotic pressure pushes cells away from the stratum basale. Desmosomes, firm intercellular
junctions with many associated keratin-rich intermediate filaments, connect the cells to one
another, but give the cells in the stratum spinosum (D). As these cells continue to differentiate
and migrate apically, they accumulate keratohyaline granules, which are responsible for the
purple granularity of cells in the stratum granulosum (C). The red layer between the stratum
corneum (A) and the stratum granulosum (C) is called the stratum lucidum (B). After complete
differentiation (keratinization), cells are sloughed from the apical surface of the stratum
corneum.

Reference:
Question:
This photomicrograph comes from a specimen taken from the hypodermis of normal adult human
skin. The large, round structure in the middle of the photomicrograph, labeled with an X, MOST likely
has which function?

Answer Choices:
A Formation of hairs
B Secretion of sebum
C Detection of hot and/or painful stimuli
D Detection of light touch
E Detection of deep pressure

Image(s) / Chart(s):
Click image to view full size. Click open image to close. Click and hold open image to move.

E Detection of deep pressure

This is a Pacinian corpuscle. It is a mechanoreceptor in the dermis and hypodermis of the skin.
It is mainly responsible for detection of deep pressure applied to the skin. It consists of multiple
layers of thin, flattened cells with gel-filled gaps between the cells. It also has an axon of a
sensory nerve fiber in its center. The skin also has other mechanoreceptors besides Pacinian
corpuscles, including Meissner’s corpuscles, which are encapsulated but are restricted to the
dermal papillae of thin skin. Meissner’s corpuscles are sensitive to light touch. Unencapsulated
free nerve endings also penetrate the epidermis from the dermis and are sensitive to hot and
painful stimuli.

Hair follicles are responsible for formation of hairs. They have a circular profile in cross section,
but the layers of cells surrounding the centrally placed hair shaft are highly varied, reflecting
histological changes in epidermal cells as they accumulate and secrete keratin, the major
constituent of the hair itself. Hair follicles are also associated with sebaceous glands, which
consist of clusters of foamy epithelial cells that release sebum, an oily conditioner of hair and
skin, as they rupture.
Case #301591:
A 13-year-old young woman spilled boiling water on herself while making tea. She received a
second-degree burn on her forearm, where a large patch of epidermis blistered. Within a few
days, the blistered epidermis sloughed, revealing a 4 cm2 raw area. In a few weeks, new
epidermis healed the burned patch.
Question:
What is the MOST significant source of regeneration of new epidermis in this young woman?

Answer Choices:
A Stratum basale
B Deep appendages
C Dermal fibroblasts
D Edge of stratum spinosum
E Apocrine sweat glands
B Deep appendages

This patient has a partial-thickness burn. First-degree burns involve only the epidermis. Second-
degree burns involve the epidermis and superficial dermis. Full-thickness burns (third-degree
and fourth-degree) involved both epidermis and deep portions of the dermis. The ducts and
secretory parts of eccrine sweat glands, which are abundant in the forearm, along with
epidermal cells in hair follicles are the source of new epidermal cells during healing after a burn.
In a second-degree burn, the deep portions of the epidermal appendages are spared, and so
new epidermis can regenerate from these healthy elements. Since the patient has a second-
degree burn, the cells in all layers of the epidermis proper, as well as the superficial portion of
the dermis, have been damaged. Thus, cells in the stratum basale and stratum spinosum could
not be the source of the new cells. Dermal fibroblasts can not differentiate into epidermal
epithelial cells. Furthermore, the apocrine sweat glands are restricted mostly to the axillae, the
mons pubis, and the circumanal region.
Case #301605:
The child shown has marked cutaneous sagging, especially marked in the face and trunk,
fibromuscular dysplasia of the renal and carotid arteries, and coarctation of the aorta. The
child’s father and paternal uncle both had similar symptoms.
Question:
What is the most likely diagnosis of this child’s disease?

Answer Choices:
A Marfan syndrome
B Ehlers-Danlos syndrome
C cutis laxa
D osteogenesis imperfecta
E Hunter syndrome

Image(s) / Chart(s):
Click image to view full size. Click open image to close. Click and hold open image to move.

C cutis laxa
Image(s) / Chart(s):
Click image to view full size. Click open image to close. Click and hold open image to move.
The child shown in the image suffers from cutis laxa, a rare disease which, as in this case, is
caused by an autosomal dominant mutation in the gene encoding the extracellular matrix
protein elastin, a secretion of the dermal fibroblast. The elastin is abnormal and subject to
more rapid turnover. Some forms of cutis laxa are also caused by autosomal recessive
mutations. An example of a child with the autosomal recessive form of cutis laxa is shown in the
image.

A phenocopy of this condition is also seen in the offspring of pregnant women with Wilson
disease (an abnormality of copper metabolism) as a result of treatment with penicillamine, a
copper-chelating agent. An example of such a patient is shown in the labeled image.

Elastin is an important component of elastic fibers. These connective tissue fibers are abundant
throughout the dermis and in the walls of large blood vessels, such as the aorta and large
muscular arteries. They are completely absent from the epidermal strata.

Children with cutis laxa have droopy skin and vascular abnormalities. The enzyme responsible
for cross-linking elastin into elastic fibers requires copper as a cofactor. The fetuses of pregnant
women with potentially fatal Wilson disease, who have been appropriately treated with the
copper-chelating agent penicillamine, will perhaps be born with a reversible condition
mimicking cutis laxa.
Case #301606:
The child shown has marked cutaneous sagging, especially marked in the face and trunk,
fibromuscular dysplasia of the renal and carotid arteries, and coarctation of the aorta. The
child’s father and paternal uncle both had similar symptoms. Further tests revealed that the
child was suffering from the autosomal dominant form of cutis laxa.
Question:
What cell type is MOST involved in this condition?

Answer Choices:
A stem cell in stratum basale
B squames in the stratum corneum
C fibroblast in dermis
D Langerhans cells in epidermis
E mast cell in dermis

Image(s) / Chart(s):
Click image to view full size. Click open image to close. Click and hold open image to move.
Case #301607:
The child shown has marked cutaneous sagging, especially marked in the face and trunk,
fibromuscular dysplasia of the renal and carotid arteries, and coarctation of the aorta. The
child’s father and paternal uncle both had similar symptoms. Further tests revealed that the
child was suffering from the autosomal dominant form of cutis laxa.
Question:
What part of the child’s skin is most affected in this condition?

Answer Choices:
A sebaceous glands
B hair follicles
C epidermal stratum granulosum
D dermis
E hypodermis

Image(s) / Chart(s):
Click image to view full size. Click open image to close. Click and hold open image to move.
Case #301631:
A 50-year-old woman presents because of well-defined white patches distributed
symmetrically over her face, hands, and neck. The discoloration started about 4 months
ago, and the patches of discoloration easily burn when exposed to sun.
Question:
If you perform a biopsy of the lesion, you would expect to find the absence of melanocytes
primarily in what structure?

Answer Choices:
A Stratum corneum
B Stratum lucidum
C Stratum granulosum
D Stratum spinosum
E Stratum germinativum
E Stratum germinativum

Your patient has vitiligo, the loss of the melanocytes. The stratum germinativum is the
deepest layer of the epidermis, where melanocytes are primarily found. Basal cells in stratum
germinativum can be considered the stem cells of the epidermis.

Melanocytes are not a part of the stratum corneum. Stratum corneum is the outermost layer
of the epidermis and consists of the dead cells – corneocytes.

The stratum lucidum is a layer of dead skin cells in the epidermis found only in areas of thick
skin (the palms of the hands and the soles of the feet). Melanocytes determine the darkness
of the stratum lucidum, but they are not a part of it.

Melanocytes are not a part of the stratum granulosum. Stratum granulosum is a thin layer of
cells in the epidermis that consists of granular cells (the keratinocytes migrated from the
stratum spinosum).

The stratum spinosum is a layer of the epidermis where keratinization begins. Melanocytes
are not a part of the stratum spinosum.
Case #301633:
Your patient is a 45-year-old woman concerned about the accelerated aging of her skin. 5
years ago she underwent a total hysterectomy with oophorectomy. Since then, she gained
about 20 pounds. She also has diabetes mellitus type II. On examination, you find a slightly
overweight female (body mass index 26) with skin that has lost its elasticity and has
reduced water-holding capacity, increased pigmentation, and decreased vascularity.
Question:
What are her symptoms signs of?

Answer Choices:
A Elevated testosterone
B Decreased testosterone
C Elevated estrogen
D Decreased estrogen
E Insulin resistance

D Decreased estrogen

Lower levels of estrogen are associated with skin aging, most probably because of telomeres
shortening. The effects of reduced estrogen levels cause loss of elasticity, reduced water
holding capacity, thickness, increased pigmentation, decreased vascularity in the skin, and
facial hair. Some of those signs are found in your patient.

Your patient has no ovaria, and testosterone is primarily secreted in the ovaries (small
amounts are also secreted by the adrenal glands). Too much testosterone will cause
excessive oiliness and acne rather than accelerated aging of the skin.

Testosterone levels in this woman are decreased after the removal of the uterus and
ovaries. Symptoms related to her skin are not characteristics of decreased testosterone
levels. More often in decreased testosterone there will be a loss of muscle strength and
mass, accumulation of fat, especially around the abdomen, osteoporosis, vaginal dryness,
etc.
High levels of estrogen can be found in women who are pregnant, extremely overweight,
have diabetes, or have liver disease. They are sometimes associated with acne, red flushed
appearance, spider nevi, etc. Skin changes in this patient cannot be attributed to high
estrogen.

Insulin resistance is a condition when insulin becomes less effective at lowering blood sugars.
It can lead to weight gain and estrogen hyperproduction, resulting in skin changes
attributable to high estrogen levels (acne, red flushed appearance, spider nevi, etc). Skin
changes in this patient cannot be attributed to the high estrogen.
Case #301490:
A 24-year-old delivery driver is involved in an accident and sustains a wide abrasion over
his left elbow. The abrasion results in the total loss of epidermis over a large area of his left
arm, but 1 month later the abrasion has healed with regrowth of the epidermis.
Question:
What mechanism accounts for the restoration of the epidermis over the abraded area?

Answer Choices:
A Growth of epidermis from hair follicles and sweat glands in the dermis
B Migration of endothelial cells from newly grown capillaries
C Transformation of dermal fibroblasts into epidermal cells
D Transformation of macrophages into epidermal cells
E Transformation of melanocytes into epidermal cells

A Growth of epidermis from hair follicles and sweat glands in the dermis

The dermis contains skin appendages (e.g. hair follicles) that contain epithelial stem cells. In
the process of healing a large area where the epidermis has been lost, but the dermis is
intact, re-epithelialization occurs by ingrowth of epidermal cells from the underlying skin
appendages, as well as from the intact epidermis along the wound edges.

None of the other cell types are known to directly contribute to the regeneration of
epidermis over abraded skin.

SECTION-TEETH
Question:
A patient is seen in a dentist's office for a routine dental exam. You find that she has developed
gingivitis. She tells you that she is diabetic and that she has her blood glucose levels under good
control. You tell her that her blood glucose levels are probably not under very good control
because excess glucose:
Answer Choices:
A Erodes the tooth enamel
B Allows bacteria to proliferate
C Enhances bacterial adhesion to enamel
D Inhibits synthesis of enamel
E Alters the solubility of bone calcium
B Allows bacteria to proliferate

Bacteria require sugars, including glucose, in order to proliferate. In diabetics who do not have
their blood glucose levels under tight control, the amount of glucose in the serum and in the
mouth can be very high. This high concentration of glucose allows the bacteria in the mouth to
proliferate at a fast rate. This makes it more difficult to adequately brush and floss to remove
plaque. Gums become red and swollen and can bleed. This stage is called gingivitis and it can be
reversed by improved dental hygiene. The next stage of the disease is periodontitis. This is an
infection of the tissues that hold teeth in place. The gums pull away from the teeth, allowing
pockets of infection to occur around the teeth. This infection leads to bone loss and eventually
tooth loss due to the loss of the supporting structures. Tooth enamel is eroded by acid
produced by the bacteria, not directly by glucose.
Keeping blood glucose under tight control in patients with diabetes is essential for preventing
the complications often associated with diabetes including loss of eyesight, neuropathy, and
kidney problems. Good oral health also depends on tight control of blood glucose levels.

Question:
A patient is seen in a dentist's office for a routine dental exam. You find that she has developed
gingivitis. She tells you that she is diabetic and that she has her blood glucose levels under good
control. You tell her that her blood glucose levels are probably not under very good control and that
eventually she may develop periodontal disease. One of the enzymes implicated in producing the
tissue damage associated with this disease is phospholipase C. Phospholipase C catalyzes the
hydrolysis of the lipid phosphatidylinositol 4,5-bisphosphate to yield inositol 1,4,5-triphosphate, and

Answer Choices:
A diacylglycerol
B glycerol
C cAMP
D ceramide
E sphingosine
A diacylglycerol

Bacteria require sugars including glucose in order to proliferate. In diabetics that do not have
their blood glucose levels under tight control, the amount of glucose in the serum and in the
mouth can be very high. This high concentration of glucose allows the bacteria in the mouth to
proliferate at a fast rate. This makes it more difficult to adequately brush and floss to remove
plaque. Gums become red and swollen and can bleed. This stage is called gingivitis and can be
reversed by improved dental hygiene. The next stage of the disease is periodontitis. This is an
infection of the tissues that hold teeth in place. The gums pull away from the teeth, allowing
pockets of infection to occur around the teeth. This infection leads to bone loss and eventually
tooth loss due to the loss of the supporting structures. Phospholipase C can act synergistically
with other factors to produce tissue damage. This may contribute to the pathogenesis of
periodontal disease.
The cleavage of phosphatidylinositol 4,5-bisphosphate by phospholipase C results in the
formation of inositol 1,4,5-triphosphate and diacylglycerol. Mammals contain 4 kinds of
phospholipase C enzymes. They are cytosolic enzymes that act on membrane inserted
phosphoinositide substrates. Each phospholipase has a defined specificity. Inositol 1,4,5-
triphosphate acts as a second messenger that opens calcium channels in the endoplasmic
reticulum (or the sarcoplasmic reticulum in muscle cells). This increases the level of cytoplasmic
calcium that can trigger a variety of processes depending on the cell type. Diacylglycerol acts as
a second messenger to activate the enzyme protein kinase C. Protein kinases C's are a family
of serine/threonine kinases that can phosphorylate many different substrates. Many signaling
pathways are triggered by the activation of protein kinase C.

Question: class quiz


Which one of the following embryonic components is responsible for the formation of the enamel?

Answer Choices:
A Ectodermal component of a tooth bud
B Ectomesenchymal component
C Dental papilla
D Ameloblasts
E Cervical loop

D Ameloblasts

Tomes' processes, the short, conical extensions of ameloblasts, are the sites of the secretion of
enamel matrix. The lateral surfaces of these processes secrete the organic matrix of the
interrod enamel, while the apical surface is responsible for deposition of the matrix of enamel
rods.
Ectodermal component of a tooth bud forms the enamel organ, responsible for the secretion
of the enamel. Cells immediately adjacent to the dental papilla assume a columnar shape and
form the inner (internal) enamel epithelium. Odontoblasts differentiate from the dental
papilla, which in turn is formed from the ectomesenchymal component. The point where the
inner enamel epithelium meets the outer enamel epithelium is termed the cervical loop.
Question:
Which one of the following embryonic components forms odontoblasts during embryonic
development

Answer Choices:
A Ectodermal component of a tooth bud
B Ectomesenchymal component
C Dental papilla
D Ameloblasts
E Cervical loop

C Dental papilla

Odontoblasts differentiate from the dental papilla, which in turn is formed from the
ectomesenchymal component. Ectodermal component of a tooth bud forms the enamel
organ, responsible for the secretion of the enamel. Cells immediately adjacent to the dental
papilla assume a columnar shape and form the inner (internal) enamel epithelium. These cells
differentiate into ameloblasts (cells that will secrete enamel). The point where the inner
enamel epithelium meets the outer enamel epithelium is termed the cervical loop.
Question:
Which one of the following statements concerning the gingivae is true?

Answer Choices:
A They are covered by stratified squamous parakeratinized to keratinized epithelium
with long rete ridges
B It has a calcified matrix containing collagen fibers
C It is richly vascularized and contains afferent nerve fibers. All sensation from it is
interpreted as pain in the central nervous system
D It is continuously elaborated after tooth eruption, compensating for the decrease in
tooth length resulting from abrasion of the enamel
E It consists of an inner cribriform plate and an outer cortical plate of compact bone
with an intervening layer of spongia

A They are covered by stratified squamous parakeratinized to keratinized epithelium with long
rete ridges

The gingivae are mucous membranes covered by stratified squamous parakeratinized to


keratinized epithelium with long rete ridges. They are in immediate contact with the
periodontal ligament.
Cementum has a calcified matrix containing collagen fibers. It is continuously elaborated after
tooth eruption, compensating for the decrease in tooth length resulting from abrasion of the
enamel. Dental pulp is richly vascularized and contains afferent nerve fibers. All sensation from
it is interpreted as pain in the central nervous system. Alveolar bone consists of an inner
cribriform plate and an outer cortical plate of compact bone with an intervening layer of
spongia.

Question:
Which one of the followings methodology was the first one to provide the histological image of the
tooth?

Answer Choices:
A Light microscopy
B An X-ray image
C An autoradiography
D Cryotomy

B An X-ray image

An x-ray was the first to reveal the histological structure of the tooth. However, it was when
better sectioning methods for teeth were developed that all various components of the tooth
were revealed under the light microscope. Autoradiography and cryotomy have been used to
study the incorporation of various inorganic elements into the tooth.
Question:
The periodontal ligament is derived embryologically from mesenchyme called the:

Answer Choices:
A Dental follicle
B Sharpey's mesenchyme
C Cementoblasts
D Dental papilla
E Dental lamina
A Dental follicle

The dental follicle is a condensation of mesenchyme, which forms around the entire developing
tooth bud. This mesenchymal layer will eventually form the periodontal ligament, composed of
connective tissue fibers, attaching the cementum layer of the root to the alveolar bone. The
dental papilla is a mesenchymal mass, surrounded in development by the enamel organ, which
gives rise to the odontoblasts and main portion of the pulp. The dental lamina is seen during
the "cap stage" of development, connecting the enamel organ with the oral mucosa.

Question:
The most prevalent site for pathogenic organisms to form an infection of the gingiva is at the

Answer Choices:
A Crevicular epithelium
B Free gingiva
C Periodontal membrane
D Attached gingiva

A Crevicular epithelium

The oral aspect of the gingiva is a thick, stratified squamous epithelium. However, it undergoes
an abrupt transition at the free gingiva to form a thin layer of epithelial cells only 1 or 2 layers
thick at the base of the gingival crevice. This thin area is referred to as the crevicular epithelium
and is subject to infection by pathogenic organisms. The free gingiva forms a cuff around the
enamel at the neck of the tooth. The attached gingiva provides an epithelial covering to the
upper alveolar bone. The periodontal membrane attaches the cementum, lining the root of the
tooth to the alveolar bone.

Question:
Sharpey's fibers, which compose the periodontal membrane, connect the alveolar bone to the

Answer Choices:
A Dentin
B Enamel
C Predentin
D Pulp
E Cementum

E Cementum

The periodontal membrane is composed of dense, fibrous connective tissue fibers referred to
as Sharpey's fibers. These fibers course obliquely downwards from their attachment on the
alveolar bone to connect to the cementum of the root surface. Because of the oblique
arrangement, the tooth is suspended within its socket.
estion:
Odontoblasts first secrete a zone of uncalcified matrix. This zone is called

Answer Choices:
A Dentin
B Osteoid
C Predentin
D Enamaloid
E Cementum

C Predentin

Predentin is the uncalcified matrix secreted by odontoblasts. The predentin is then calcified to
become mature dentin. Osteoid is the uncalcified matrix secreted by osteoblasts. Cementum is
secreted by cementocytes and covers the root of the tooth.
Question:
Refer to the attached image. In the photomicroscopic image of a cross-section through the root of a
tooth, the arrow to the left of the letter 'd' indicates which part of the tooth

Answer Choices:
A Odontoblasts
B Osteoblasts
C Cementocytes
D Fibroblasts
E Ameloblasts

Image(s) / Chart(s):
Click image to view full size. Click open image to close. Click and hold open image to move.

A Odontoblasts

In the photomicroscopic image of the cross-section through the root of a tooth, the letter 'a'
depicts the area surrounding the alveolar bone adjacent to the tooth.
The letter 'b' indicates the dentin of the tooth.
The arrow indicated by the letter 'c' indicates predentin.
The arrow indicated by the letter 'd' points out the odontoblasts that produce dentin.
The letter 'e' indicates the substance of the pulp cavity.
Osteoblasts are the cells associated with formation of bone, such as the alveolar bone.
Cementocytes produce the mineralized cementum that covers the root of the tooth.
Ameloblasts form the enamel that covers the crown of the tooth. Occasional fibroblast nuclei
may be seen in the pulp cavity, 'e'.

Question:
Which portion of a tooth is covered by cementum?

Answer Choices:
A Clinical crown
B Cusp
C Neck
D Root
E Anatomical crown

D Root

The root of the tooth is covered by cementum. Cementum resembles bone in that it has a
mineral content of 45-50%. Cells within the cementum, cementocytes, are contained within
lacunae and resemble osteocytes. The crown (anatomical and clinical) is covered by enamel, as
is the cusp. The neck is the junction between the crown and root and is regarded as a
cementoenamel junction.

Question:
A human skull was found, and all teeth in the maxilla and mandibular bones were intact. You quickly
examined the dentition and found on each side of the mandible to be 1 central incisor, 1 lateral
incisor, a canine tooth, and 2 molars. You ascertained that the person probably died at about which
age?

Answer Choices:
A 4 years old
B 14 years old
C 28 years old
D 45 years old
E 70 years old
A 4 years old

In a child, the dental formula is 2/1/2, or 2 incisors (1 central incisor and 1 lateral incisor), 1
canine tooth, and 2 molars. These occur on each side of the mandible and maxilla, such that the
number of teeth in a fully developed mouth of a child is 20 teeth.
These, however, are deciduous and are gradually replaced, beginning at about age 6.
Replacement of the deciduous teeth with the permanent teeth results in a dental formula of
2/1/2/3, or 2 incisors, 1 canine, 2 premolars, and 3 molars, usually.
Since the skull was found to have a dental formula of 2/1/2 and all of the teeth were apparently
present in the skull, it can be concluded with reasonable confidence that the skull belonged to
that of a child with all of his/her deciduous teeth present, probably someone younger than the
age of 6 years. Therefore, 4 years would be the most appropriate response for this question.
All of the other ages would exhibit teeth showing a dental formula of 2/1/2/3 or some
combination of this formula, since some teeth may have been removed or missing during the
persons life, leaving a space in the mandible or maxilla.

Question:
The clinical crown of a tooth describes what part(s) of the tooth?

Answer Choices:
A The cusp of the tooth
B The crown exposed above the gum
C The crown above and below the gum
D The junction between the crown and the root
E The crown hidden below the gum

B The crown exposed above the gum

Enamel covers the crown, or exposed surface of the tooth, and part of the tooth that is below
the gum line. The part of the crown that is exposed is called the clinical crown, and the crown
that includes the exposed portion and a part of the tooth covered by enamel below the gum is
referred to as the anatomical crown. The cusp is the biting and grinding surface of the teeth,
also covered by enamel and part of the crown.

Copyright © EXAM MASTER Corporation 2014 Author ID: 23


Question:
Refer to the attached image. In the photomicroscopic image of a cross-section through the root of a
tooth, the letter 'b' indicates which part of the tooth?
Answer Choices:
A Enamel
B Dentin
C Cementum
D Predentin
E Pulp cavity

Image(s) / Chart(s):
Click image to view full size. Click open image to close. Click and hold open image to move.

B Dentin

In the photomicroscopic image of the cross-section through the root of a tooth, the letter 'a'
depicts the area surrounding the alveolar bone adjacent to the tooth.
The letter 'b' indicates the dentin of the tooth.
The arrow indicated by the letter 'c' indicates predentin.
The arrow indicated by the letter 'd' points out the odontoblasts that produce dentin.
The letter 'e' indicates the substance of the pulp cavity.
Question:
Refer to the attached image. In the photomicroscopic image of a cross-section through the root of a
tooth, the letter 'e' indicates which part of the tooth?

Answer Choices:
A Enamel
B Dentin
C Cementum
D Predentin
E Pulp cavity

Image(s) / Chart(s):
Click image to view full size. Click open image to close. Click and hold open image to move.
E Pulp cavity

In the photomicroscopic image of the cross-section through the root of a tooth, the letter 'a'
depicts the area surrounding the alveolar bone adjacent to the tooth.
The letter 'b' indicates the dentin of the tooth.
The arrow indicated by the letter 'c' indicates predentin.
The arrow indicated by the letter 'd' points out the odontoblasts that produce dentin.
The letter 'e' indicates the substance of the pulp cavity.

Question: BLOCK 2
The mineralized organic matrix called predentin is laid down by which of the following cell types?

Answer Choices:
A Odontoblasts
B Epithelial cells
C Ameloblasts
D Osteoblasts
E Fibroblasts

A Odontoblasts

The fully differentiated odontoblasts line the pulp cavity and secrete predentin and dentin. The
organic components of dentin consist of collagen, non-collagenous proteins, proteoglycans, and
lipids, with collagen composing 85-90% of the organic phase.
Comparison of skin and dentin collagen indicates that they are similar but that dentin collagen
contains three times more hydroxylysine. The collagen in both types of tissue has similar
amounts of hydroxyproline. Also dentin and bone contain no Type III collagen but they do
contain 5% Type V collagen, which is presumed to promote formation of fibrillar arrangements
or possibly promote mineralization.
It is believed that the fibrillar structures found in bone and dentin are somehow adapted to
promote or allow mineralization. Ameloblasts are elongated, secretory cells that specifically
form the proteins of enamel. Enamel contains no collagen and the apatite crystals of enamel
are very large and highly oriented.
Question:
Tomes fibers are found in

Answer Choices:
A Tooth enamel
B Tooth dentin
C Tooth cementum
D Alveolar bone
E Gingiva

B Tooth dentin

Unlike the osteoblast, the odontoblast is a polarized, slender cell, producing organic matrix
only at the dentinal surface. The cytoplasm of each of these cells has a slender extension that
penetrates perpendicularly through the entire width of the dentine, forming Tomes fibers.
These fibers gradually become longer as the dentine becomes thicker, running in small canals
(dentinal tubules). These fibers initially have a diameter of ~3-4 µm, gradually become thinner,
and end by branching near the junction between the dentin and the enamel. The
periodontoblastic space between the process of the cell and the tubule contain tissue fluid.
Enamel (97% Calcium and 3% organic material), secreted by ameloblasts, is composed of
elongated hexagonal columns (enamel prisms). Tooth cementum is similar in composition to
bone but lacks Haversian systems and blood vessels. Towards the root of the tooth, it contains
cementocytes. Alveolar bone is immature bone, in which collagen fibers are not arranged in
lamellar pattern, as in adult bone, and gingiva is mucous membrane bound to periosteum. It
contains stratified squamous epithelium and numerous connective tissue papillae.

Question:
A patient is seen in a dentist's office for a routine dental exam. You find that she has developed
gingivitis. She tells you that she is diabetic and that she has her blood glucose levels under good
control. You tell her that her blood glucose levels are probably not under very good control and that
eventually she may develop periodontal disease. One of the enzymes implicated in producing the
tissue damage associated with this disease is phospholipase C. Phospholipase C catalyzes the
hydrolysis of the lipid phosphatidylinositol 4,5-bisphosphate to yield inositol 1,4,5-triphosphate and
diacylglycerol. In mammalian cells, the function of the inositol 1,4,5-triphosphate is to

Answer Choices:
A activate protein kinase C
B increase cytoplasmic calcium
C activate phospholipase A
D activate adenylate cyclase
E form a complex with G proteins

B increase cytoplasmic calcium

Bacteria require sugars including glucose in order to proliferate. In diabetics that do not have
their blood glucose levels under tight control, the amount of glucose in the serum and in the
mouth can be very high. This high concentration of glucose allows the bacteria in the mouth to
proliferate at a fast rate. This makes it more difficult to adequately brush and floss to remove
plaque. Gums become red and swollen and can bleed. This stage is called gingivitis and can be
reversed by improved dental hygiene. The next stage of the disease is periodontitis. This is an
infection of the tissues that hold teeth in place. The gums pull away from the teeth, allowing
pockets of infection to occur around the teeth. This infection leads to bone loss and eventually
tooth loss due to the loss of the supporting structures. Phospholipase C can act synergistically
with other factors to produce tissue damage. This may contribute to the pathogenesis of
periodontal disease.
The cleavage of phosphatidylinositol 4,5-bisphosphate by phospholipase C results in the
formation of inositol 1,4,5-triphosphate and diacylglycerol. Mammals contain 4 kinds of
phospholipase C enzymes. They are cytosolic enzymes that act on membrane inserted
phosphoinositide substrates. Each phospholipase has a defined specificity. Inositol 1,4,5-
triphosphate acts as a second messenger that opens calcium channels in the endoplasmic
reticulum (or the sarcoplasmic reticulum in muscle cells). This increases the level of cytoplasmic
calcium that can trigger a variety of processes depending on the cell type. Diacylglycerol acts as
a second messenger to activate the enzyme protein kinase C. Protein kinases C's are a family
of serine/threonine kinases that can phosphorylate many different substrates. Many signaling
pathways are triggered by the activation of protein kinase C.
Question:
Each tooth has a central, soft core composed of

Answer Choices:
A Enamel
B Dentin
C Cementum
D Pulp
E Alveolar bone

D Pulp

The central core of the tooth is composed of pulp, a loose connective tissue. The pulp core is
richly vascularized and supplied by nerves which enter the pulp cavity at the root foramen.
Enamel, dentin and cementum are all mineralized components of the tooth. Enamel covers the
tooth crown, cementum covers the root, and the dentin is the calcified material that forms
most of the substance of the tooth. The alveolar bone does not participate in the formation or
structure of the tooth. The tooth, however, is anchored to the alveolar bone.

Question:
Refer to the attached image. In the photomicroscopic image of a cross-section through the root of a
tooth, the arrow as indicated to the right of the letter 'c', indicates which part of the tooth?

Answer Choices:
A Enamel
B Dentin
C Cementum
D Predentin
E Pulp cavity

Image(s) / Chart(s):
Click image to view full size. Click open image to close. Click and hold open image to move.

D Predentin

In the photomicroscopic image of the cross-section through the root of a tooth, the letter 'a'
depicts the area surrounding the alveolar bone adjacent to the tooth.
The letter 'b' indicates the dentin of the tooth.
The arrow indicated by the letter 'c' indicates predentin.
The arrow indicated by the letter 'd' points out the odontoblasts that produce dentin.
The letter 'e' indicates the substance of the pulp cavity.

Question:
A patient is seen in a dentist's office for a routine dental exam. You find that she has developed
gingivitis. She tells you that she is diabetic and that she has her blood glucose levels under good
control. You tell her that her blood glucose levels are probably not under very good control and that
eventually she may develop periodontal disease. One of the enzymes implicated in producing the
tissue damage associated with this disease is phospholipase C. Phospholipase C catalyzes the
hydrolysis of the lipid phosphatidylinositol 4,5-bisphosphate to yield inositol 1,4,5-triphosphate and
diacylglycerol. In mammalian cells the function of the diacylglycerol is to
Answer Choices:
A activate protein kinase C
B increase cytoplasmic calcium
C activate phospholipase C
D activate adenylate cyclase
E form a complex with G proteins
A activate protein kinase C

Bacteria require sugars including glucose in order to proliferate. In diabetics that do not have
their blood glucose levels under tight control, the amount of glucose in the serum and in the
mouth can be very high. This high concentration of glucose allows the bacteria in the mouth to
proliferate at a fast rate. This makes it more difficult to adequately brush and floss to remove
plaque. Gums become red and swollen and can bleed. This stage is called gingivitis and can be
reversed by improved dental hygiene. The next stage of the disease is periodontitis. This is an
infection of the tissues that hold teeth in place. The gums pull away from the teeth, allowing
pockets of infection to occur around the teeth. This infection leads to bone loss and eventually
tooth loss due to the loss of the supporting structures. Phospholipase C can act synergistically
with other factors to produce tissue damage. This may contribute to the pathogenesis of
periodontal disease.
The cleavage of phosphatidylinositol 4,5-bisphosphate by phospholipase C results in the
formation of inositol 1,4,5-triphosphate and diacylglycerol. Mammals contain 4 kinds of
phospholipase C enzymes. They are cytosolic enzymes that act on membrane inserted
phosphoinositide substrates. Each phospholipase has a defined specificity. Inositol 1,4,5-
triphosphate acts as a second messenger that opens calcium channels in the endoplasmic
reticulum (or the sarcoplasmic reticulum in muscle cells). This increases the level of cytoplasmic
calcium that can trigger a variety of processes depending on the cell type. Diacylglycerol acts as
a second messenger to activate the enzyme protein kinase C. Protein kinases C's are a family
of serine/threonine kinases that can phosphorylate many different substrates. Many signaling
pathways are triggered by the activation of protein kinase C.
Question:
A human child has the following number of teeth in the maxilla:

Answer Choices:
A 1 central incisor, 1 lateral incisor, 1 canine, and 2 molars
B 2 central incisors, 2 lateral incisors, 2 canines, and 4 molars
C 2 central incisors, 2 lateral incisors, 2 canines, 1 premolar, and 4 molars
D 1 central incisor, 1 lateral incisor, 1 canine, 1 premolar, and 1 molar
E 2 central incisors, 1 lateral incisor, 2 canines, 1 premolar, and 2 molars

B 2 central incisors, 2 lateral incisors, 2 canines, and 4 molars


A child has 10 deciduous teeth in each jaw consisting of 2 central incisors, 2 lateral incisors, 2
canines, and 4 molars. There are no premolars.

Question:
The anatomical crown of a tooth describes what part of the tooth?

Answer Choices:
A The portion of a tooth visible above the gingiva
B The upper, enamel covered part of the tooth
C The portion of a tooth distal to the gingival crevice
D The lower, enamel covered part of the tooth
E The portion of a tooth visible below the gingiva

B The upper, enamel covered part of the tooth

The anatomical crown is the upper portion of the tooth that is covered by enamel.
The portion of the tooth distal to the gingival crevice is the physiological crown of the tooth.
The clinical crown is the portion of the tooth visible above the gingiva.
Reference:
The American Dental Association. “Glossary of Dental Clinical and Administrative Terms.” <
http://www.ada.org/glossaryforprofessionals.aspx> 18 September 2012.
Question:
Which one of the following components of the tooth contains odontoblasts?

Answer Choices:
A Enamel
B Dentin
C Cementum
D Dental pulp
E Periodontal ligament

D Dental pulp

Dental pulp, a gelatinous connective tissue contains odontoblasts in its peripheral layer (closet
to the dentin). Enamel is acellular after tooth eruption. Dentin, which consists of a calcified
matrix containing collagen fibers, is manufactured by odontoblasts, which persist and continue
to elaborate it. Cementum is secreted by cementocytes (comparable to osteocytes).
Periodontal ligament is composed of irregular connective tissue, and contains the usual
collagen fibers and cellular elements.
Question:
The primary cell involved in production of enamel for a tooth is the

Answer Choices:
A Odontoblast
B Ameloblast
C Cementocyte
D Osteoblast

B Ameloblast

The ameloblasts are primarily responsible for production and secretion of enamel covering the
crown of the tooth. Odontoblasts form dentin, a substance resembling bone matrix, forming
the substance of most of the tooth. The cementocyte forms cementum, the bone-like covering
of the root of the tooth. Osteoblasts are cells associated with formation of bone.

Question:
In the embryological development of the tooth, which one of the following parts of the tooth is of
ectodermal origin?

Answer Choices:
A Dentin
B Enamel
C Cementum
D Pulp
E Periodontal ligament

B Enamel

Enamel, which is secreted from epithelial cells called ameloblasts, is of ectodermal origin. All of
the other parts of the tooth are of mesodermal origin.

Question:
During maturation of tooth enamel, one protein present in the developing enamel matrix is removed.
Which one of the following proteins present in the development of enamel is absent in mature
enamel?
Answer Choices:
A Enamelin
B Tuft protein
C Amelogenin
D Hydroxyapatite binding protein
E Osteocalcin

C Amelogenin

Three major proteins are contained in immature enamel matrix. These proteins include
amelogenins, tuft protein, and enamelins. Amelogenins are removed from the matrix during
the maturation process and are not present in mature enamel. Tuft protein is present near the
dentinoenamel junction. Osteocalcin is present in normal bone as a calcium-binding protein in
the bone matrix.

Question:
The addition of fluoride to drinking water and toothpastes has significantly reduced dental caries. The
mechanism that best describes how fluoride reduces dental caries is that

Answer Choices:
A Fluoride combines with hydroxyapatite to form less acid soluble flourhydroxyapatite
providing increased resistance to bacterial action
B Fluoride is bactericidal, cleansing the oral cavity of bacteria causing caries
C Fluoride penetrates the dentin making it impervious to caries
D Fluoride combines with the hydroxyapatite complex, providing bactericidal protection
to the enamel
E The action of fluoride is unknown

A Fluoride combines with hydroxyapatite to form less acid soluble flourhydroxyapatite


providing increased resistance to bacterial action

While mature enamel contains little organic material and the hydroxyapatite complex is
arranged in rods which span the full thickness of the enamel layer, enamel can be damaged by
decalcification by acid-producing bacterial. Fluoride, as contained in toothpastes and drinking
water, will penetrate the enamel and bind to the hydroxyapatite complex, further hardening
the enamel and making it more resistant to bacterial action.

Question:
Which of the following choices best describes the induction of enamel formation by ameloblasts?
Answer Choices:
A Production of predentin by the odontoblast induces ameloblast activity
B Calcification of the organic dentin matrix induces ameloblast activity
C Eruption of the tooth induces ameloblast activity
D Ameloblast activity is not under inductive forces during tooth development and occurs
simultaneously with dentin production

B Calcification of the organic dentin matrix induces ameloblast activity

During development of the tooth, odontoblasts begin deposition of organic dentin matrix
between the layers of odontoblasts and ameloblasts. Calcification occurs within the organic
dentin matrix, which then induces enamel formation by the ameloblasts.

Question:
Which cell produces dentin in the tooth?

Answer Choices:
A Odontoblast
B Maturation ameloblast
C Cementocyte
D Secretory ameloblast
E Osteoblast

A Odontoblast

Odontoblasts form dentin, a substance resembling bone matrix forming the substance of most
of the tooth. The ameloblasts are primarily responsible for production and secretion of enamel
covering the crown of the tooth. The cementocyte forms cementum, the bone-like covering of
the root of the tooth. Osteoblasts are cells associated with formation of bone.

Question:
Sensory reception of the tooth is due to unmyelinated fibers that terminate in which part of the
tooth?

Answer Choices:
A Pulp cavity
B Odontoblast layer
C Ameloblast layer
D Cementocyte layer
E Sharpey's fibers

B Odontoblast layer

The pulp is innervated by myelinated fibers that branch into fine unmyelinated fibers and
extend into the odontoblast layer, lining the surface of the pulp cavity, giving the dentin
exquisite sensory reception.

Question:
Which one of the following embryonic components is responsible for the formation of the dentin?

Answer Choices:
A Ectodermal component of a tooth bud
B Odontoblasts
C Dental papilla
D Ameloblasts
E Cervical loop

B Odontoblasts

Odontoblasts secrete procollagen, which becomes organized into the collagen fibrils of
predentin. These cells also mediate the mineralization of collagen fibrils, leading to the
formation of dentin.
Ectodermal component of a tooth bud forms the enamel organ, responsible for the secretion
of the enamel. Dental papilla give rise to odontoblasts. Ameloblasts form the tooth enamel.
The point where the inner enamel epithelium meets the outer enamel epithelium is termed the
cervical loop.

SECTION- GIT
Question:
Which of the following papillae are located along low parallel ridges along deep mucosal clefts
arranged perpendicular to the long axis of the tongue in which numerous taste buds are found in the
deep clefts?

Answer Choices:
A Filiform papillae
B Fungiform papillae
C Circumvallate papillae
D Foliate papillae
E Foramen papillae
D Foliate papillae

Fungiform papillae are mushroom-shaped projections scattered on the dorsal surface of the
tongue, but tend to be more numerous near the tip of the tongue.
Filiform papillae are elongated projections of connective tissue that are conical shaped on the
dorsal surface of the tongue, but are not associated with taste buds. The filiform papillae also
are oriented such that their conically shaped tips form rows with their tips pointing backwards.
Circumvallate papillae are 8-12, large, dome-shaped epithelial structures, which are located on
the dorsal surface of the tongue just anterior to the sulcus terminalis. Numerous taste buds are
usually found at the base of each circumvallate papillae where lingual salivary glands empty.
Foliate papillae are found more easily in younger individuals on the dorsal lateral and posterior
lateral surface of the tongue. These are identified as parallel low ridges located in deep mucosal
clefts arranged perpendicular to the long axis of the tongue. Numerous taste buds are found in
the deep clefts of epidermis.

Question: CLASS QUIZ


Which of the following papillae are described as mushroom-shaped projections located on the dorsal
surface of the tongue, which are scattered but tend to be more numerous near the tip of the tongue?

Answer Choices:
A Filiform papillae
B Fungiform papillae
C Circumvallate papillae
D Foliate papillae
E Foramen papillae

Case #7219:
A 2-week-old male infant is brought to the ER by his parents with complaint of a 1-week
history of vomiting. The infant's mother informs that vomiting occurs soon after feeding and
sometimes the entire volume of feed is vomited. She also informs that the vomitus is non-
bilious and non-blood stained and that for the past 2 days the infant is having projectile
vomiting. The infant is exclusively breastfed, and after vomiting the infant remains hungry
and eager to feed. There is no history of fever or diarrhea. On exam, the abdomen is slightly
distended with active bowel sounds. A firm, non-tender, mobile mass measuring 1-2 cm is
palpable in the right hypochondriac region. Ultrasound confirms the diagnosis of pyloric
stenosis.
Question: CLASS QUIZ
What is the described orientation of the external muscle layers of the stomach?

Answer Choices:
A Inner and outer circular layer with middle oblique layer
B Inner longitudinal layer with middle and outer circular layer
C Inner circular layer with middle and outer longitudinal layer
D Inner circular, middle longitudinal, and occasionally an outer circular layer
E Inner oblique, middle circular, and outer longitudinal layer
Question:
Which one of the following components of the large intestine contains rectal columns of Morgagni?

Answer Choices:
A Rectal mucosa
B Anal submucosa
C Anal mucosa
D Internal anal sphincter
E External anal sphincter

Question:
Which of the following cell types is present in the gastric glands of the pyloric stomach?

Answer Choices:
A Goblet cells
B Mucous neck cells
C Paneth cells
D Basal cells
E Chief cells
B Mucous neck cells

Only fundic glands contain chief (zymogenic) cells. Mucous neck cells are located in the neck of
gastric glands in all parts of the stomach. Paneth cells are located inside the crypts of
Lieberkühn (lamina propria of small intestine). Goblet cells are characteristics of the mucosal
epithelium of all parts of small intestine, while basal cells (or stem cells) are located inside the
body (fundus) of the stomach.

Question: CLASS QUIZ


Which of the following applies to teniae coli?

Answer Choices:
A They are three flat, longitudinal ribbons of smooth muscles of external musculature
of the colon
B They are fat-filled outpocketings of serosa of the cecum and colon
C They are the longitudinal folds of the mucosa of the anal canal
D They are peritoneal suspensory ligaments of the intestinal wall
E They are the folds of lamina propria of the intestinal lumen
Question: BLOCK 2
The Auerbach's nerve plexus is present in

Answer Choices:
A The intestinal submucosa
B Between the outer longitudinal and inner circular layers of muscularis externa
C The intestinal serosa
D The intestinal adventitia
E The lamina propria which follows intestinal epithelium
Question:
Which one of the following layers (coats) of the gastrointestinal tract contains large blood plexus that
supplies branches to other layers (coats)?

Answer Choices:
A Mucosa
B Submucosa
C Muscularis externa
D Serosa
E Adventitia

Question:
The gastrointestinal tract has histologically the following distinct regions. Which one of these regions
houses Meissner's nerve plexus?

Answer Choices:
A Mucosa
B Submucosa
C Muscularis externa
D Serosa
E Adventitia

Question:
A tissue sample revealed the presence of villi, intestinal glands, goblet cells scattered throughout the
mucosa, and extensively developed aggregations of lymph nodules in the lamina propria. No glands
were observed in the submucosa. From which of the following areas was this tissue sample taken?

Answer Choices:
A Esophagus
B Fundic stomach
C Duodenum
D Ileum
E Colon

Question:
A villus of the small intestine differs from a plica circularis due to

Answer Choices:
A A villus is formed by folds of submucosa, whereas the plica are formed by folds of
mucosa
B A villus is formed by folds of mucosa, whereas the plica are formed by folds of
submucosa
C A villus is formed by folds of submucosa, whereas the plica are formed by folds of
muscularis externa
D A villus is formed by epithelial folds, whereas the plica are formed by folds of mucosa
E A villus is formed by cytoplasmic extensions of absorptive epithelial cells, whereas the
plica are formed by folds of mucosa

B A villus is formed by folds of mucosa, whereas the plica are formed by folds of submucosa

Villi are finger-like extensions of the mucosa having a core of lamina propria, whereas plica are
transverse folds that are formed by the submucosa. These folds are circularly arranged and
become more prominent in the jejunum of the small intestine.

Question:
Simple tubular glands that extend from the muscularis mucosa through the lamina propria and open
at the base of the villi of the small intestine are called

Answer Choices:
A Lacteals
B Peyer's patches
C Gastric crypts
D Crypts of Lieberkühn
E Plica circularis
Question:
What structure is found within the core of the villi within the small intestine?

Answer Choices:
A Lacteal
B Crypt of Lieberkühn
C Peyer's patches
D Muscularis mucosa
E Lymph nodes

Question:
The smooth muscle of the muscularis mucosa of the stomach is innervated predominantly by

Answer Choices:
A Parasympathetic nerve fibers from the myenteric plexus
B Parasympathetic nerve fibers from the submucosal plexus
C Sympathetic nerve fibers from both the myenteric plexus and submucosal plexus
D Sympathetic nerve fibers from the myenteric plexus
E Sympathetic nerve fibers from the submucosal plexus

B Parasympathetic nerve fibers from the submucosal plexus

The muscularis mucosa of the stomach is relatively thin, and generally there are only two layers
present. However, in some regions of the stomach, a third layer may be present.
The orientation of these layers are such that there is an inner circular, a middle longitudinal,
and an outer circular layer. When the third layer is absent, the two layers present are oriented
as an inner circular and outer longitudinal layer.
The ganglion cells that comprise the submucosal plexus give rise to parasympathetic nerve
fibers that innervate the smooth muscle of the muscularis mucosa.
Question:
What is the traditional description of the orientation of the layers of the muscularis externa of the
stomach?

Answer Choices:
A Inner circular, outer longitudinal
B Inner longitudinal, outer circular
C Inner oblique, middle longitudinal, outer circular
D Inner oblique, middle circular, outer longitudinal
E Inner circular, middle oblique, outer longitudinal
Question:
Refer to the attached image. If the cell indicated at the tip of the arrowhead labeled 'a' in the
photomicroscopic image of the fundic stomach was absent, what medical condition would develop?

Answer Choices:
A Pancreatitis
B Anemia
C Duodenal ulceration
D Diabetes mellitus
E Crohn's disease

Image(s) / Chart(s):
Click image to view full size. Click open image to close. Click and hold open image to move.

B Anemia

The tip of the arrowhead labeled 'a' best depicts a parietal cell. The parietal cells are large,
relatively pale-staining cells in the middle or upper portion of the fundic glands. These cells
secrete hydrochloric acid and intrinsic factor. Intrinsic factor complexes with vitamin B12 in the
stomach, which is required for the absorption of the vitamin in the ileum. After the reserve
stores of vitamin B12 in the liver are depleted, the vitamin is not replenished since it cannot be
absorbed, and pernicious anemia results.

Question:
Refer to the attached image. What is the secretory product of the cell indicated at the tip of the
arrowhead labeled 'b' in the photomicroscopic image of the fundic stomach?

Answer Choices:
A Gastrin
B Hydrochloric acid
C Mucin
D Intrinsic factor
E Pepsinogen

Image(s) / Chart(s):
Click image to view full size. Click open image to close. Click and hold open image to move.

Question:
Refer to the attached image. Identify the cell as indicated at the tip of the arrowhead labeled 'b' in the
photomicroscopic image of the fundic stomach. This cell would be best described as

Answer Choices:
A A Chief cell
B A Paneth cell
C A parietal cell
D A mucous neck cell
E An enteroendocrine cell

Image(s) / Chart(s):
Click image to view full size. Click open image to close. Click and hold open image to move.

Question:
Refer to the attached image. Identify the cell as indicated at the tip of the arrowhead labeled 'a' in the
photomicroscopic image of the fundic stomach. This cell would be best described as

Answer Choices:
A A Chief cell
B A Paneth cell
C A parietal cell
D A mucous neck cell
E An enteroendocrine cell

Image(s) / Chart(s):
Click image to view full size. Click open image to close. Click and hold open image to move.
Question:
Refer to the attached image. Identify the cell as indicated at the tip of the arrowhead labeled 'c' in the
photomicroscopic image of the fundic stomach. This cell would be best described as

Answer Choices:
A A Chief cell
B A Paneth cell
C A parietal cell
D A mucous neck cell
E An enteroendocrine cell

Image(s) / Chart(s):
Click image to view full size. Click open image to close. Click and hold open image to move.
Question:
Gastrin stimulates the secretion of which one of the following substances

Answer Choices:
A Pepsin
B Intrinsic factor
C Hydrochloric acid
D Mucinogen
E Bicarbonate ion

Question:
Gastrin is secreted by which one of the following cells in the stomach?

Answer Choices:
A Parietal cells
B Chief cells
C Enteroendocrine cells
D Surface mucous cells
E Mucous neck cells

Question:
Which one of the following cells in the stomach secretes pepsinogen?
Answer Choices:
A Parietal cells
B Chief cells
C Enteroendocrine cells
D Surface mucous cells
E Mucous neck cells

Question:
Apical accumulations of mucinogen granules form surface mucous cells, which create a glandular
sheet of cells that lines the cavity of an organ. This epithelial sheet of surface mucous cells is found in
which one of the following organs?

Answer Choices:
A Stomach
B Esophagus
C Colon
D Duodenum
E Jejunum

Question:
The epithelium that overlies a lymph nodule within the gastrointestinal tract has a specialized cell
possessing distinctive surface microfolds. These folds are able to pinocytose protein from the
intestinal lumen and transport it through the cytoplasm to discharge it within the extracellular space
where the protein may be presented to lymphocytes. What are these specialized cells called?

Answer Choices:
A GALT cells
B Enterocytes
C Paneth cells
D M cells
E Enteroendocrine cells

estion:
The enlargement of what structures form the internal hemorrhoids?

Answer Choices:
A Rectal venous plexus found within the lamina propria of the anal columns
B Superior rectal artery found within the lamina propria of the anal columns
C Rectal venous plexus found within the submucosa of the anal columns
D Superior rectal artery found within the adventitia of the anal columns
E Rectal venous plexus found within the adventitia of the anal columns

Question:
The internal anal sphincter and external anal sphincter are formed by which muscles, respectively?

Answer Choices:
A The internal anal sphincter is formed by the outer longitudinal layer of the muscularis
externa; the external anal sphincter is formed by the inner circular layer of the muscularis
externa
B The internal anal sphincter is formed by the inner circular layer of the muscularis
externa; the external anal sphincter is formed by the muscularis mucosa
C The internal anal sphincter is formed by the muscularis mucosa; the external anal
sphincter is formed by the muscles of the perineum
D The internal anal sphincter is formed by the inner circular layer of the muscularis
externa; the external anal sphincter is formed by the muscles of the perineum
E The muscles of the perineum form both the internal and external anal sphincters
Question:
The most caudal (distal) portion of the
gastrointestinal tract is the:

Answer Choices:
A Colon.
B Appendix.
C Cecum.
D Rectum.
E Anal canal.

Question: LAB
Colonic saccules or haustra are the result of what morphological specializations?

Answer Choices:
A Outpocketing of the mucosa of the colon
B Thickening of the adventitia due to increased formation of dense regularly arranged
connective tissue along the colon
C Thickening of the circular layer of the muscularis externa at irregular intervals along
the colon
D Interdigitation of the taeniae coli with the circular layer of the muscularis externa
along the colon
E Formation of taeniae coli without participation of the circular layer of the muscularis
externa along the colon

D Interdigitation of the taeniae coli with the circular layer of the muscularis externa along the
colon

The bundles of longitudinally arranged smooth muscle of the taeniae coli (from the outer layer
of the muscularis externa) will penetrate and interdigitate with the circular layer of the
muscularis mucosa at irregular intervals along the colon. The "functional segmentation"
formed by the discontinuity of the colon wall allows for portions of the colon to contract
independently of one another leading to formation of saccules along the colon wall.

Question: LAB
The large intestine can best be identified by which one of the following features?

Answer Choices:
A Plicae circulares
B Villi
C A "smooth" mucosal surface
D Lack of lymphatic tissue in the lamina propria
E Lack of an outer longitudinal layer of the muscularis externa

Question: LAB
Epithelial cell renewal in the small intestine occurs from stem cells
located

Answer Choices:
A On the surface of the villi
B Within the intestinal glands
C At the base of the intestinal crypts
D At the opening of the intestinal crypts
E At the base of the villi

Question:
The cells scattered throughout the mucosa of the gastrointestinal tract and endocrine pancreas,
which produce gastrointestinal hormones and paracrine substances within the gastrointestinal tract
and/or pancreas, are collectively grouped together and referred to as
Answer Choices:
A Amine precursor uptake and decarboxylation cells (APUD)
B Argyrophilic and degranulated cells
C Enterochromaffin cells
D Gastric active cells
E Pancreatic active cells

Question:
A histological section was presented in which a junction between two tubular organs of the digestive
tract were identified. The epithelia changed abruptly from a nonkeratinized stratified squamous to
simple columnar. The lamina propria of both areas contained mucous secreting glands, which were
limited to the lamina propria. Diffuse lymphocytic infiltration was present sparsely in both areas. A
thin muscularis mucosa is present in both regions and an underlying submucosa was easily identified.
The muscularis externa, however, became more prevalent in the more distal region and the clearly
defined inner circular and outer longitudinal regions identifiable in the more proximal region, became
lost in the more distal region. Which one of the following junctions are probably represented by this
biopsy?

Answer Choices:
A Recto-anal junction
B Gastroduodenal junction
C Ileo-cecal junction
D Esophageo-gastric junction
E Appendix-cecal junction

Question: LAB
Esophageal cardiac glands are present predominantly in what area of the esophagus?

Answer Choices:
A Lamina propria of the distal esophagus
B Submucosa of the distal esophagus
C Adventitia of the proximal esophagus
D Submucosa of the proximal esophagus
E Submucosa along the entirety of the esophagus

Question: BLOCK 2
A histological section of the digestive tract revealed the presence of skeletal muscle present in the
muscularis externa, and small compound tubuloalveolar mucous secreting glands located in the
submucosa. The epithelium present was non-keratinized stratified squamous. The combination of
these features are limited to which area of the alimentary canal?
Answer Choices:
A Upper esophagus
B Pyloric stomach
C Proximal jejunum
D Descending colon
E Anal canal

Question: CLASS QUIZ


Striated (skeletal) muscle may be found interwoven with smooth muscle in the muscularis externa of
one area of the alimentary canal. Which of the following parts of the digestive tract would have this
type of muscle arrangement?

Answer Choices:
A Upper esophagus
B Fundus of the stomach
C Lower esophagus
D Pyloric stomach
E Middle esophagus

Question: LAB
Microscopic projections of cytoplasm of the mucosal epithelial cells that are present throughout most
the small intestine are modifications of the alimentary tract that result in increasing the surface area
available for absorption of the products of digestion. These structures are referred to as

Answer Choices:
A Rugae
B Plicae circulares
C Villi
D Microvilli
E Taeniae coli

Question:
Modifications of the layers of the alimentary tract result in formation of projections that enhance the
surface area available for absorption of the products of digestion. Mucosal projections that cover
most of the surface of the small intestine are referred to as

Answer Choices:
A Rugae
B Plicae circulares
C Villi
D Microvilli
E Taeniae coli

Question:
The small groups of nerve cell bodies associated with the autonomic nervous system throughout the
digestive tract known as Meissner's plexus are best described as being located within which one of
the following layers of the digestive tract?

Answer Choices:
A Immediately deep to the epithelium, within the lamina propria
B Within a thin connective tissue layer between the inner and outer muscle layers of the
muscularis externa
C Within a thin connective tissue layer between the inner and outer muscle layers of the
muscularis mucosa
D Scattered throughout the connective tissue of the submucosa
E Scattered throughout the connective tissue of the serosa and adventitia

Question:
The general organization of the alimentary canal is such that four structural layers have specific
functions associated with the digestive tract. Which one of the following layers functions to mix and
propel the luminal contents of the tract by peristalsis?

Answer Choices:
A Serosa
B Adventitia
C Mucosa
D Submucosa
E Muscularis

Question:
The general organization of the alimentary canal is such that four structural layers have specific
functions associated with the digestive tract. Which one of the following layers functions as an
epithelial barrier separating the lumen of the tract from the body of the organism?

Answer Choices:
A Serosa
B Adventitia
C Mucosa
D Submucosa
E Muscularis

Question:
The general structural organization of the alimentary canal consists of four distinctive layers. Which
one of the following layers consists of connective tissue that blends into the connective tissue of
adjoining tissue, such as along the esophagus?

Answer Choices:
A Serosa
B Adventitia
C Mucosa
D Submucosa
E Muscularis

Question:
The general structural organization of the alimentary canal consists of four distinctive layers. Which
one of the following layers consists of loose connective tissue covered by a simple squamous
epithelium (mesothelium)?

Answer Choices:
A Serosa
B Adventitia
C Mucosa
D Submucosa
E Muscularis

Question:
The general structural organization of the alimentary canal consists of four distinctive layers. Which
one of the following layers generally consists of two concentric, relatively thick layers of smooth
muscle?

Answer Choices:
A Serosa
B Adventitia
C Mucosa
D Submucosa
E Muscularis

uestion:
The general structural organization of the alimentary canal consists of four distinctive layers. Which
one of the following layers consists of moderately dense, irregular connective tissue that contains
some of the larger blood vessels, lymphatic vessels, and autonomic nerve plexuses?

Answer Choices:
A Serosa
B Adventitia
C Mucosa
D Submucosa
E Muscularis

Question:
The primary secretory product by the cells indicated at the tip of the arrows in the attached
photomicroscopic image is best described as

Answer Choices:
A Bicarbonate ions
B Cortisol
C Mucous
D Hydrochloric acid
E Pepsinogen

Image(s) / Chart(s):
Click image to view full size. Click open image to close. Click and hold open image to move.

D Hydrochloric acid

Observation of the histological features in this photomicrograph indicate the organ is the
stomach. The epithelium observed in this photomicrograph is simple columnar. The mucosa
shows multiple infoldings ending in straight glands. These infoldings are open to the lumen and
referred to as "pits". The glands are lined by large, pale, eosinophilic staining cells, which are
located at the tip of the arrows in the image. These cells are Parietal cells and secrete
hydrochloric acid (HCl).

Question:
The histological characteristics depicted in the attached photomicrographic image are consistent with
which one of the following organs?

Answer Choices:
A Trachea
B Stomach
C Duodenum
D Colon
E Fallopian tube

Image(s) / Chart(s):
Click image to view full size. Click open image to close. Click and hold open image to move.

Question:
The most common anaerobic bacteria colonizing upper respiratory tract are

Answer Choices:
A Peptostreptococcus, Fusobacterium, and Bacteroides
B Viridans group, streptococci
C Hemophilus and Neisseria
D Coagulase negative staphylococci
E Clostridium spp
Question:
The general organization of the alimentary canal is such that four structural layers have specific
functions associated with the digestive tract. Which one of the following layers functions to
selectively absorb the products of digestion?

Answer Choices:
A Serosa
B Adventitia
C Mucosa
D Submucosa
E Muscularis
Question:
The pyloric sphincter at the gastroduodenal junction is formed from a thickening of which layer of the
digestive tract?

Answer Choices:
A Mucosa
B Muscularis mucosa
C Adventitia
D Muscularis externa
E Submucosa

Question:
Modifications of the layers of the alimentary tract result in formation of projections that enhance the
surface area available for absorption of the products of digestion. Mucosal and submucosal folds that
are oriented parallel with the long axis of the stomach are referred to as

Answer Choices:
A Rugae
B Plicae circulares
C Villi
D Microvilli
E Taeniae coli

Question:
Enteroendocrine cells of the small intestine are derived from which location?

Answer Choices:
A Neural ectoderm cells which migrate into the gastrointestinal mucosa
B Cells from Rathke's pouch which migrate into the gastrointestinal mucosa
C Cells from the cloaca which migrate into the gastrointestinal mucosa
D Cells from the zone of cell replication at the base of the intestinal crypts
E Cells from the zone of proliferation within the endochondral model
Question:
Identify the cells located at the tip of the arrows in the attached
photomicrographic image

Answer Choices:
A Goblet cell
B Paneth cell
C Enteroendocrine cell
D Spongiocyte
E Parietal cell

Image(s) / Chart(s):
Click image to view full size. Click open image to close. Click and hold open image to move.

Question:
Examination of the inner surface of the empty stomach reveals a series of longitudinal submucosal
and mucosal folds called

Answer Choices:
A Villi
B Plicae circulares
C Rugae
D Tenia coli
E Lamina propria

Question:
The general structural organization of the alimentary canal consists of four distinctive layers. Which
one of the following layers consists of a lining epithelium, an underlying lamina propria, and a smooth
muscle called muscularis mucosa?

Answer Choices:
A Serosa
B Adventitia
C Mucosa
D Submucosa
E Muscularis

Question:
What cells in the stomach provide for a thick viscous coating that adheres to the epithelial surface
and protects it against abrasion and acid hydrolysis?

Answer Choices:
A Goblet cells
B Enteroendocrine cells
C Parietal cells
D Chief cells
E Surface mucous cells

Question:
Hydrochloric acid (HCl) is secreted by which one of the following cells in the stomach?

Answer Choices:
A Parietal cells
B Chief cells
C Enteroendocrine cells
D Surface mucous cells
E Mucous neck cells

Question:
The general organization of the alimentary canal is such that four structural layers have specific
functions associated with the digestive tract. Which one of the following layers possesses diffuse
lymphatic tissue and functions in the protection of the body from pathogens and other antigens as
part of the gut-associated lymphatic tissue (GALT) system?

Answer Choices:
A Serosa
B Adventitia
C Mucosa
D Submucosa
E Muscularis

Question:
The myenteric plexus, or Auerbach's plexus, consists of small groups of nerve cell bodies associated
with the autonomic nervous system. These 'ganglia' are best described as within which one of the
following layers of the digestive tract?

Answer Choices:
A Immediately deep to the epithelium, within the lamina propria
B Within a thin connective tissue layer between the inner and outer muscle layers of
the muscularis externa
C Within a thin connective tissue layer between the inner and outer muscle layers of
the muscularis mucosa
D Scattered throughout the connective tissue of the submucosa
E Scattered throughout the connective tissue of the serosa and adventitia
Question:
Refer to the attached image. Identify the cell as indicated at the tip of the arrowhead labeled 'd' in the
photomicroscopic image of the fundic stomach. This cell would be best described as

Answer Choices:
A A Chief cell
B A Paneth cell
C A parietal cell
D A mucous neck cell
E An enteroendocrine cell

Image(s) / Chart(s):
Click image to view full size. Click open image to close. Click and hold open image to move.
A A Chief cell

The tip of the arrowhead labeled 'a' best depicts a parietal cell. The parietal cells are large,
relatively pale-staining cells in the middle or upper portion of the fundic glands. These cells
secrete hydrochloric acid and intrinsic factor.
The tip of the arrowhead labeled 'b' best depicts a mucous neck cell. These cells secrete soluble
mucin and are located throughout middle and upper regions of the fundic gland. They are
intermingled among the parietal cells, have small pyknotic nuclei located at the base of the cell,
and exhibit a pale-staining cytoplasm having a 'frothy' appearance.
The tip of the arrowhead labeled 'c' best depicts an enteroendocrine cell as located in the
fundic gland of the stomach. Note that these are small cells with lightly staining cytoplasm.
These cells are scattered throughout the glands of the stomach, and throughout the epithelium
of the gastrointestinal tract. This cell may secrete gastrin.
The tip of the arrowhead labeled 'd' best depicts a chief cell. These cells are located near the
base of the fundic glands. The basal region of these larger cells stain relatively more basophilic,
and the secretory product, located in the more apical portion of the cell cytoplasm, gives a
granular or more eosinophilic appearance to the cell cytoplasm. This cell produces and secretes
pepsinogen, the inactive precursor to the proteolytic enzyme, pepsin.
Question:
Modifications of the layers of the alimentary tract result in formation of projections that enhance the
surface area available for absorption of the products of digestion. Circumferentially arranged
submucosal folds present throughout most of the small intestine are referred to as
Answer Choices:
A Rugae
B Plicae circulares
C Villi
D Microvilli
E Taeniae coli
Question:
Refer to the attached image. What is the secretory product of the cell indicated at the tip of the
arrowhead labeled 'a' in the photomicroscopic image of the fundic stomach?

Answer Choices:
A Gastrin
B Pepsin
C Mucin
D Intrinsic factor
E Pepsinogen

Image(s) / Chart(s):
Click image to view full size. Click open image to close. Click and hold open image to move.

Question:
See the attached photomicroscopic image. Which layer of the stomach depicted in this image is
forming the fold?

Answer Choices:
A Mucosa
B Submucosa
C Muscularis externa
D Adventitia
E Muscularis mucosa

Image(s) / Chart(s):
Click image to view full size. Click open image to close. Click and hold open image to move.

Question:
Which of the following cells would be the first to become affected in the gastrointestinal mucosa by
the administration of a pharmacologic agent inhibiting cellular division?

Answer Choices:
A Chief cells of the stomach
B Paneth cells
C Goblet cells
D Absorptive cells of the small intestine
E Surface mucous cells of the stomach
Question: CLASS QUIZ
A patient presents with excessive gastric acid secretion without evidence of H. pylori infection. What
kind of receptor antagonist can be used to inhibit hydrochloric acid from gastric parietal cells?

Answer Choices:
A α1-adrenergic antagonist
B α2-adrenergic antagonist
C GABAa antagonist
D Dopaminergic D1 antagonist
E Histaminergic H2 antagonist

Question: LAB
The relative population of goblet cells would be the greatest in which one of the following sections of
the gastrointestinal tract?

Answer Choices:
A Rectum
B Esophagus
C Stomach
D Duodenum
E Ascending colon

Question:
A tissue sample revealed a simple columnar epithelium, simple tubular glands with extensive goblet
cells scattered throughout the mucosa. Furthermore, there were no villi or plica present. From which
of the following areas was this tissue sample taken?

Answer Choices:
A Esophagus
B Fundic stomach
C Duodenum
D Ileum
E Colon
Question:LAB
What epithelial type lines the surface of the stomach and the gastric pits?

Answer Choices:
A Stratified squamous
B Simple cuboidal
C Simple columnar
D Pseudostratified columnar
E Transitional

Question:
What is one function ascribed to the surface mucous cells of the stomach?
Answer Choices:
A Protect the surface from acid digestion
B Promote fat absorption in the stomach
C Inhibit fat absorption in the stomach
D Enhance proteolysis in the stomach
E Complex with vitamin B12

Question:
Intrinsic factor is secreted by which one of the following cells in the stomach?

Answer Choices:
A Parietal cells
B Chief cells
C Enteroendocrine cells
D Surface mucous cells
E Mucous neck cells

Question:
Refer to the attached image. What is the secretory product of the cell indicated at the tip of the
arrowhead labeled 'd' in the photomicroscopic image of the fundic stomach?

Answer Choices:
A Gastrin
B Hydrochloric acid
C Mucin
D Intrinsic factor
E Pepsinogen

Image(s) / Chart(s):
Click image to view full size. Click open image to close. Click and hold open image to move.
Question:
Refer to the attached image. Which cell, as indicated by the letter next to the tip of the arrowhead in
the photomicroscopic image of the fundic stomach, secretes hydrochloric acid?

Answer Choices:
A A
B B
C C
D D

Image(s) / Chart(s):
Click image to view full size. Click open image to close. Click and hold open image to move.
Question:
See the attached photomicroscopic image. What part of the stomach is depicted by this image?

Answer Choices:
A Fundic stomach
B Cardiac stomach
C Gastroduodenal junction
D Gastroesophageal junction

Image(s) / Chart(s):
Click image to view full size. Click open image to close. Click and hold open image to move.

A Fundic stomach

The photomicroscopic image depicts the fundic stomach at low power. The layers of the
extensively developed mucosa can be seen (epithelium, lamina propria, and muscularis
mucosa), as well as the submucosa, muscularis externa, and serosa. The fundic stomach is
characterized by a thick mucosa with long, fairly straight, well-developed tubular glands (fundic
glands) having an extremely cellular appearance.
The pits in the fundic region are shorter than in the cardiac region, extending only about 1/4 of
the distance through the mucosa (the pits extend through about 1/2 of the mucosa in the
cardiac stomach).
The gastroesophageal junction would display features common to the esophagus, such as
stratified squamous epithelium, and the presence of glands in the lamina propria (cardiac
glands).
Likewise, the gastroduodenal junction would exhibit features common to the duodenum,
primarily the presence of duodenal glands that fill the submucosa, a simple columnar
epithelium of the mucosa, goblet cells, and the presence of villi.
Question:
A tissue sample revealed the presence of villi, intestinal glands, and goblet cells scattered throughout
the mucosa. Furthermore, glands were present in the submucosa. From which one of the following
areas was this tissue sample taken?

Answer Choices:
A Esophagus
B Fundic stomach
C Duodenum
D Ileum
E Colon

Question:
The small intestine has three histologically distinct regions. Which one of the following statements
concerning the histologic differences in the three regions is true?

Answer Choices:
A Peyer's patches are present only in the ileum
B Goblet cells are present only in the epithelium of the duodenum
C Brunner's glands are located in the duodenum and jejunum, but not the ileum
D Lacteals are present only in the lamina propria of the ileum
E The muscularis mucosae contains three layers of smooth muscle in the ileum and two
layers in the duodenum and jejunum

Question: CLASS QUIZ


Which of the following papillae are large, dome-shaped epithelial structures located on the dorsal
surface of the tongue just anterior to the sulcus terminalis and possess numerous taste buds deep
within sulci surrounding the structures?

Answer Choices:
A Filiform papillae
B Fungiform papillae
C Circumvallate papillae
D Foliate papillae
E Foramen papillae

Question:
What are the cells that form the striated border of the small intestine and are specialized for the
transport of substances from the lumen of the intestine?
Answer Choices:
A Enterocytes
B Enteroendocrine cells
C Paneth cells
D Goblet cells
E Chief cells

Question: LAB
What cells in the small intestine possess secretory granules that contain an antibacterial enzyme?

Answer Choices:
A Enterocytes
B Enteroendocrine cells
C Paneth cells
D Goblet cells
E Chief cells

Question:
What cells in the small intestine secrete mucus?

Answer Choices:
A Enterocytes
B Enteroendocrine cells
C Paneth cells
D Goblet cells
E Chief cells
Question:
Which of the following cell types is absent in the epithelium of the small intestine?

Answer Choices:
A Stem cells
B Surface absorptive cells
C Goblet cells
D Paneth's cells
E Chief cells

Question:
The Gut-Associated Lymphatic Tissue (GALT) is most extensively developed in which one of the
following areas?
Answer Choices:
A Lower esophagus
B Stomach
C Duodenum
D Ileum
E Colon

Question:
Straight tubular glands lined by simple columnar epithelium that extend through the full thickness of
the mucosa and participate principally in reabsorption of water and electrolytes are common to
which one of the following areas?

Answer Choices:
A The esophagus
B The large intestine
C The duodenum
D The stomach
E The jejunum

Question:
The cell renewal in the small intestine, on the average, takes about

Answer Choices:
A 12-24 hours
B 1-2 days
C 3-6 days
D 7-14 days
E 30-60 days

Question:
You are a medical student in pathology class looking at different specimen. You find the intestinal
lining and the pathologist is discussing Paneth cells. The paneth cells, when exposed to bacteria or
bacterial antigens secrete anitmicrobial molecules (also called as alpha defensis orcryptdins) into the
lining of the intestine, maintaining the intestinal mucosal barrier. Which one of the following reflects
the life span of these cells?

Answer Choices:
A 3-6 days
B 7-14 days
C 15-21 days
D 29-32 days
E 45-60 days

D 29-32 days

Paneth's cells of the large intestinal glands turn over much more slowly, living about 29-32 days
before being replaced.

In addition to defensins, Paneth cells secrete lysozyme and phospholipase A2, both of which
have clear antimicrobial activity. This characteristics of secretory molecules give Paneth cells
power against a broad spectrum of agents, including bacteria, fungi, and even some enveloped
viruses.
SECTION- LIVER AND GALL BLADDER
Question: LAB
Cirrhosis of the liver is characterized by an

Answer Choices:
A Infiltration of macrophages
B Increased proliferation of the hepatocytes
C Overproduction of connective tissue
D Inhibition of fibroblast proliferation
E Increase in the synthesis of liver proteins
C Overproduction of connective tissue

The balanced regeneration of cell types in an organ such as the liver is very important. Under
conditions of poisoning with carbon tetrachloride or in alcoholics, fibroblasts proliferate and
secrete extracellular matrix that clogs the liver. This leaves little space for the hepatocytes to
grow even if the toxin is removed.
Question:
The number of mitochondria in a hepatocyte, on an average, amounts to

Answer Choices:
A 2,000
B 1,000
C 500
D 250
E 50-100

A 2,000

Each liver cell has approximately 2,000 mitochondria.


Question:
The liver is covered by a thin connective tissue capsule called

Answer Choices:
A Disse's capsule
B Glisson's capsule
C Peritoneal capsule
D Gastric capsule
E Renal capsule

B Glisson's capsule

The liver is covered by a thin connective tissue Glisson's capsule. It becomes thicker at the
hilum, the entry portal vein and hepatic artery. Renal capsule covers the kidneys.

Question: CLASS QUIZ


Space of Disse may be defined as

Answer Choices:
A A hexagonal mass of tissue primarily composed of plates of hepatocytes, which
radiates from the central vein to the periphery
B A region of connective tissue between lobules, present at each corner of a classic liver
lobule
C Space lined by Kupffer cells, and containing blood
D A triangular region whose three apices are neighboring central veins and whose
center is located in a portal area
E A subendothelial space located between hepatocytes and the endothelial cells of the
liver sinusoids

E A subendothelial space located between hepatocytes and the endothelial cells of the liver
sinusoids
Image(s) / Chart(s):
Click image to view full size. Click open image to close. Click and hold open image to move.
The space of Disse is the space between endothelial linings of the liver sinusoids and the
underlying hepatocytes. The microvilli of the hepatocytes are projected into these spaces.
The hexagonal mass of tissue composed of plates of hepatocytes is known as the classic liver
lobule. The region of connective tissue between lobules, present at each corner of a classic liver
lobule, is called a portal area (portal canal or portal triad). Sinusoids contain Kupffer cells in
their endothelial linings, while a triangular region whose three apices are neighboring central
veins and whose center is located in a portal area, is a portal lobule.
See Figure: Cross section through liver lobules
Question: LAB
Which hormone stimulates gallbladder contraction?

Answer Choices:
A Gastrin
B GIP (gastric inhibitory peptide)
C Secretin
D CCK (cholecystokinin)
E PP (pancreatic polypeptide)

D CCK (cholecystokinin)

GIP is believed to stimulate insulin release, but will inhibit acid production in the stomach.
Gastrin stimulates acid secretion.
Secretin acts primarily on the pancreas to stimulate bicarbonate ion secretion and enzyme
release.
CCK has similar actions on the pancreas to those of secretin, but also will stimulate gallbladder
contraction. CCK will also reduce gastric emptying.
PP has an opposing action to secretin and CCK, which reduces pancreatic bicarbonate ion and
enzyme secretion.
Question: CLASS QUIZ
Refer to the attached image. Occlusion of the structure at 'c' in the photomicroscopic image would
result in damage of the hepatocytes first in which area?

Answer Choices:
A Zone 1
B Zone 2
C Zone 3
D Damage would not occur to hepatocytes if 'c' were occluded

Image(s) / Chart(s):
Click image to view full size. Click open image to close. Click and hold open image to move.

A Zone 1

Hepatocytes in the liver acinus are described as being in three elliptical zones. These surround
the terminal distributing axis of the hepatic artery lying along the border between two classic
lobules (extending between two adjacent portal triads defining the short axis of the rough
diamond shape of the liver acinus). Zone 1 comprises the hepatocytes lying closest to the short
axis and are the first to receive blood. They would be the first to show morphological
alterations in the case of bile stasis, which would occur if the bile duct ('c') was occluded.
Question: CLASS QUIZ
The presence of which cellular organelle increases in hepatocytes following the use of anabolic
steroids?

Answer Choices:
A Nuclei
B Rough endoplasmic reticulum
C Golgi apparatus
D Smooth endoplasmic reticulum
E Lysosomes

D Smooth endoplasmic reticulum

Hepatocytes contain extensive sER, and the amount varies with metabolic activity. Use of
anabolic steroids, such as testosterone, induces the synthesis of new sER membranes and their
associated enzymes since sER is involved in degradation of the steroid. Stimulation of the
amount of sER by anabolic steroids may be so extensive as to become the predominant
organelle in the hepatocyte.
Question:
Exchange of substances between blood cells and the hepatocytes occurs at which of the following
sites?

Answer Choices:
A Space of Mall
B Space of Disse
C The Kupffer cells
D The lumen of sinusoids

B Space of Disse

The space of Disse (or perisinusoidal space) is a small space lying between the basal surface of
the hepatocytes and the basal surface of the endothelial cells (or Kupffer cells) that line the
sinusoids. The basal surface of the hepatocytes are either highly infolded, or sometimes possess
basal microvilli to aid in the exchange of materials between blood cells and the hepatocytes.

Question:
Following bile duct occlusion, a liver biopsy was performed to determine the extent of liver damage.
Morphological changes of hepatocytes in Zone 3 were observed. Zone 3 involvement suggests
hepatocyte damage due to bile stasis. In this patient, this is best described by which one of the
following responses?

Answer Choices:
A Minor
B Moderate
C Severe
D Insufficient data to determine if bile status resulted in liver damage
E Bile stasis does not produce hepatocyte changes

C Severe

Within the liver acinus, cells closest to the short axis (as defined by the terminal branches of the
hepatic artery) are the first to receive nutrition in the blood, but also the first to show
morphological changes following bile stasis. Cells in Zone 3 (those hepatocytes that are in a
rough elliptical pattern along the short axis of the liver acinus) are the last to respond to bile
stasis since they are also the closest to the central vein. Since the liver biopsy revealed cells in
Zone 3 to be morphologically altered, it can be deduced that bile stasis was severe in this
patient.
Question: CLASS QUIZ
Which of the following structures defines the liver acinus as the points of the diamond on its long
axis?

Answer Choices:
A Central veins
B Hepatic arteries
C Bile ducts
D Portal veins
E Hepatic sinusoids

A Central veins

The liver acinus is a rough diamond shape in which the long axis is defined by a line connecting
two adjacent central veins closest to the short axis. The short axis of the liver acinus is defined
by an imaginary line connecting two adjacent portal triads, perpendicular to the closest long
axis (along the edge of a classic lobule). A branch of the hepatic artery is the central feature of
the short axis and is the distributing vessel for the liver acinus.
Question:
Which of the following structures forms the points of the portal lobule of the liver?

Answer Choices:
A Central veins
B Bile ducts
C Hepatic arteries
D Portal veins
E Hepatic sinusoids
A Central veins

The portal lobule is the morphological basis of the bile secretion function of the liver. It is
morphologically defined by a rough triangular block of tissue whose outer margins are
imaginary lines connecting three central veins forming the points of the lobule. This
conceptualization places the portions of three classic lobules that secrete bile such that they
each drain towards a common axial bile duct in a portal triad.

Question: LAB
Lymphatic fluid putatively arises from which area in the liver?

Answer Choices:
A Space of Disse
B Space of Mall
C Malpighians space
D Hepatic sinusoids
E Bile canaliculi

B Space of Mall

The space of Mall is a small space located between the connective tissue stroma and the
hepatocytes located on the outermost edges of the classic hepatic lobule. It is believed that the
space of Mall is the area where lymph arises in the liver.

Question: CLASS QUIZ


The portal triad consists of three structures which compose the corners of the classic hepatic lobule.
The three structures contained within the portal triad include

Answer Choices:
A Hepatic artery, portal vein, and central vein
B Portal vein, bile duct, and hepatic artery
C Central vein, hepatic artery, and bile duct
D Bile duct, hepatic vein, and hepatic artery
E Portal vein, bile duct, and hepatic vein

B Portal vein, bile duct, and hepatic artery


As oxygenated blood flows into the liver from the branch of the celiac trunk, small branches of
the hepatic artery may be found within the portal triad. Along with the distributing branches of
the hepatic artery are branches of the portal vein. These provide blood supply to the hepatic
sinusoids of the liver lobule. The small draining branches of the bile duct system are also found
in the portal triad and make up the third component. It should be noted, however, that a
lymphatic vessel also may be found in the portal triad area.
Question:
Oxygenated blood reaches the liver by way of which one of the following vessels?

Answer Choices:
A Portal artery
B Hepatic artery
C Central artery
D Superior mesenteric artery
E Celiac artery

B Hepatic artery

The liver derives oxygenated blood from a branch of the celiac trunk, the hepatic artery. This
provides about 1/4th of the blood supply to the liver. The portal vein provides the remaining
amount of blood, which comes from the major abdominal organs and is oxygen depleted. Blood
from both sources mixes as it perfuses through the hepatic parenchyma.

Question:
Refer to the attached image. The photomicroscopic image best represents which organ?

Answer Choices:
A Liver
B Medullary kidney
C Anterior pituitary
D Pancreas
E Seminal vesicle

Image(s) / Chart(s):
Click image to view full size. Click open image to close. Click and hold open image to move.
A Liver

The image as depicted by this photomicrograph represents a liver. Note the connective tissue
capsule at 'a', and a central vein at 'b'. A portal triads with a bile duct is indicated by the letter
'c'. While difficult to visualize in human tissue preparations, the "classic" liver lobule is defined
as the central vein ('b') at its center and sinusoids and hepatic plates radiating outward from
the central vein with the corners of the hexagon composed of portal triads.
Question:
The common bile duct is connected to the gall bladder by the

Answer Choices:
A Bile canaliculi
B Common hepatic duct
C Interlobular bile ducts
D Ampulla of Vater
E Cystic duct

E Cystic duct

The cystic duct connects the common bile duct to the gall bladder. The bile canaliculi are non-
epithelial lined, intrahepatic ducts located between adjacent hepatocytes. The common hepatic
duct is formed by the union of the right and left hepatic ducts. The interlobular bile ducts are
intrahepatic passageways and are located in the portal canals. The ampulla of Vater is the
expansion of the common bile duct located at the junction of the common bile duct and the
pancreatic ducts.

Question:CLASS QUIZ
Secretion of the contents of this organ as illustrated by the photomicroscopic image is best described
as being stimulated by hormones in response to the presence of dietary
Answer Choices:
A Protein
B Carbohydrates
C Lipids
D Amino acids
E Sugars

Image(s) / Chart(s):
Click image to view full size. Click open image to close. Click and hold open image to move.

C Lipids

The organ depicted in the image is the gall bladder. Bile is stored within the gall bladder and will
empty upon stimulation by appropriate hormones that are produced in response to the
presence of fats (lipids) in the proximal portion of the duodenum.

Question:
Identify the organ illustrated in the photomicroscopic image:

Answer Choices:
A Uterine tube
B Stomach
C Vas deferens
D Gall bladder
E Seminal vesicle

Image(s) / Chart(s):
Click image to view full size. Click open image to close. Click and hold open image to move.
D Gall bladder

Inspection of the image reveals the presence of a non-ciliated simple columnar epithelium
lining what appears to be a lumen. This immediately eliminates the vas deferens and seminal
vesicle (both have pseudostratified columnar epithelium) and the uterine tube (which is lined
by simple columnar cells with cilia present on many of the cells).
The epithelial cells of the gall bladder do have microvilli, but these are poorly observed on light
microscopic preparations. The underlying lamina propria shows elaborate folds and there is a
predominant muscularis consisting of what appears as interlacing smooth muscle bundles
(unlike the muscularis of the uterine tube, which possesses layers of smooth muscle arranged in
an inner longitudinal and outer circular layers). Underlying the muscularis is an adventitia.
The stomach is not a possible identification since the stomach possesses simple columnar
epithelia arranged into crypts and glands that penetrate into the lamina propria.
Question:
What two organs are illustrated in the photomicroscopic image?

Answer Choices:
A Liver and gall bladder
B Pars intermedia and pars distalis of the pituitary gland
C Seminal vesicle and prostate gland
D Ovary and uterine tube
E Stomach and liver

Image(s) / Chart(s):
Click image to view full size. Click open image to close. Click and hold open image to move.

A Liver and gall bladder

The photomicroscopic image depicts the histological appearance of two organs in close contact;
the liver and gall bladder. Inspection of the epithelial lining of the gall bladder shows the
presence of simple columnar cells. Underlying the epithelium is a loose connective tissue core,
the lamina propria, thrown into irregular folds. A fairly prominent layer of smooth muscle is
seen underlying the lamina propria, forming the muscularis layer of the gall bladder. Glisson's
capsule is absent at the position where the liver and gall bladder are in contact, however, the
characteristic hepatic plates with intervening sinusoids can be seen.
The intervening area common to the liver and gall bladder is composed of connective tissue
adventitia. Note that one of the folds of epithelium in the lumen of the gall bladder is
pronounced and appears like a gland in the sectioned organ.
Without careful inspection of the image, the liver may appear similar to the cells of the
parenchyma within the pars distalis of the pituitary gland, but the pars intermedia may contain
what appear as colloid-filled cysts or follicles lined by simple squamous or cuboidal epithelium,
not present in this image.
The seminal vesicle may show irregular folds of the lamina propria, similar to the gall bladder,
but the epithelium is predominantly pseudostratified columnar lining the lumen.
The prostate gland would show glands, whose lumen possibly contain concretions, lined by a
variety of epithelia, usually pseudostratified columnar. Underlying the glands are a
fibromuscular connective tissue.
The gall bladder also may be confused with the uterine tube, since the uterine tube is lined with
a simple columnar epithelium, and has folds of lamina propria. However, many of the epithelial
cells of the uterine tube are ciliated.
The ovary does not resemble the liver, since the parenchyma of the ovary contains ova in
various stages of maturation interspersed throughout connective tissue.
Question:
What is the function of the cell whose nucleus is located at the tip of the pointer in the
photomicroscopic image?

Answer Choices:
A Production of collagen
B Phagocytosis
C Antibody production
D Vascular lining
E Lipid storage

Image(s) / Chart(s):
Click image to view full size. Click open image to close. Click and hold open image to move.

B Phagocytosis

The photomicroscopic image reveals the structure of the hepatic parenchyma showing the
presence of plates of hepatocytes with intervening sinusoids. The hepatocytes are large cells,
sometimes binucleated. Close examination of the sinusoids shows a loose lining of the hepatic
plates, occasionally revealing the space of Disse between the sinusoid lining and the underlying
hepatic plate. Unlike the sinusoids located in other areas, the hepatic sinusoids are composed
of two cell types, the endothelial cell and the Kupffer cell, whose nucleus is shown at the tip of
the pointer in the image.
The Kupffer cells are believed to be derived from monocytes and may be involved in
phagocytosis, particularly of damaged red blood cells. Histologically, of the two cells lining the
sinusoids, the Kupffer cell has a larger nucleus than the endothelial cell, and considerably more
cytoplasm. At times, the Kupffer cells may appear to obliterate the lumen of the sinusoid.
Question:
Identify the cell whose nucleus is located at the tip of the pointer in the photomicroscopic image.
Answer Choices:
A Endothelial cell
B Hepatocyte
C Ito cell
D Kupffer cell
E Mall cell

Image(s) / Chart(s):
Click image to view full size. Click open image to close. Click and hold open image to move.

D Kupffer cell

The photomicroscopic image reveals the structure of the hepatic parenchyma showing the
presence of plates of hepatocytes with intervening sinusoids.
The hepatocytes are large cells, sometimes binucleated. Close examination of the sinusoids
shows a loose lining of the hepatic plates, occasionally revealing the space of Disse between the
sinusoid lining and the underlying hepatic plate. Unlike the sinusoids located in other areas, the
hepatic sinusoids are composed of two cell types, the endothelial cell and the Kupffer cell,
whose nucleus is shown at the tip of the pointer in the image.
The Kupffer cells are believed to be derived from monocytes and may be involved in
phagocytosis, particularly of damaged red blood cells. Histologically, of the two cells lining the
sinusoids, the Kupffer cell has a larger nucleus than the endothelial cell, and considerably more
cytoplasm. At times, the Kupffer cells may appear to obliterate the lumen of the sinusoid.
Ito cells are infrequently seen and are typified by the presence of lipid granules in the
cytoplasm.
The endothelial cells lining the sinusoids are small cells with attenuated cytoplasm, whose
nuclei generally appear spindle-shaped and do not significantly protrude into the lumen of the
sinusoid.
Question:
Which one of the following statements concerning the neonatal hyperbilirubinemia is true?
Answer Choices:
A It pertains to defective storage of protein in the cytoplasm of the hepatocytes
B It is concerned with the excretion of water soluble bile salts through the kidneys
C It is caused due to the underdeveloped state of the smooth endoplasmic reticulum
in the hepatocytes of the newborn
D It pertains to the formation of cholelithiasis inside the gallbladder
E It is concerned with the defective storage of lipids and carbohydrates in the liver
C It is caused due to the underdeveloped state of the smooth endoplasmic reticulum in the
hepatocytes of the newborn

One of the main processes occurring in the smooth endoplasmic reticulum is the conjugation of
hydrophobic toxic bilirubin by glucuronosyltransferase to form a water-soluble nontoxic
bilirubin glucuronide. This conjugate is excreted by hepatocytes into the bile. When bilirubin, or
bilirubin glucuronide is not excreted, several diseases characterized by jaundice can occur.
One of the frequent causes of jaundice in the newborn is the often an underdeveloped state of
the smooth endoplasmic reticulum in their hepatocytes (neonatal hyperbilirubinemia).
Contrary to what is observed in other glandular cells, the hepatocytes do not store proteins in
their cytoplasm, but continuously release them into the blood stream. Abnormal proportion of
bile acids generally leads to the formation of cholelithiasis (gallstones) inside the gallbladder.
Abnormal storage of glycogen can occur in several diseases, but not in neonatal
hyperbilirubinemia.

Question:
Neural innervation of the liver is best described as which one of the following?

Answer Choices:
A Sympathetic
B Parasympathetic
C Sympathetic and parasympathetic
D Somatic motor
E Sympathetic and somatic motor

C Sympathetic and parasympathetic

Nerves from both the sympathetic and parasympathetic divisions of the autonomic nervous
system are believed to innervate the liver. Sympathetic and possibly parasympathetic fibers are
believed to innervate the smooth muscles surrounding the blood vessels within the portal
triads. Parasympathetic fibers may innervate the larger bile ducts of the liver. There is no
somatic motor innervation to the liver.

Question:
What epithelium best describes the lining of the common hepatic ducts?

Answer Choices:
A Simple cuboidal
B Simple columnar
C Simple squamous
D Stratified cuboidal
E Transitional

B Simple columnar

The common hepatic duct, formed by the union of the right and left hepatic ducts as part of the
extrahepatic passageway, is lined by a simple columnar epithelium. Simple columnar
epithelium is common to all of the extrahepatic passages, while the intrahepatic ducts are lined
with simple cuboidal epithelium.

Question:
Refer to the attached image. The structure at letter 'b' in the photomicroscopic image is best
identified as

Answer Choices:
A Bile duct
B Portal artery
C Central vein
D Hepatic artery
E Portal vein

Image(s) / Chart(s):
Click image to view full size. Click open image to close. Click and hold open image to move.
C Central vein

The image as depicted by this photomicrograph represents a liver. Note the connective tissue
capsule at 'a', and a central vein at 'b'. A portal triad with a bile duct is indicated by the letter
'c'. The central vein ('b') defines the center of the "classic" lobule and sinusoids and hepatic
plates radiating outward from the central vein. The central vein is a thin-walled vessel which
receives blood flowing centrally through the hepatic sinusoids.

Question:
Liver damage by toxins in the blood will first be observed in which area?

Answer Choices:
A Zone 1
B Zone 2
C Zone 3
D Bile duct
E Central vein

A Zone 1

The liver acinus is a rough diamond shape in which the long axis is defined by a line connecting
two adjacent central veins closest to the short axis. The short axis of the liver acinus is defined
by an imaginary line connecting two adjacent portal triads, perpendicular to the closest long
axis (along the edge of a classic lobule). A branch of the hepatic artery is the central feature of
the short axis and is the distributing vessel for the liver acinus. Zone 1 is comprised of
hepatocytes surrounding the short axis in an elliptical pattern which are the first cells that
receive toxins presented to the liver by the blood.
Question:
The endothelium lining the hepatic sinusoids is best classified as which one of the following?
Answer Choices:
A Continuous
B Fenestrated
C Discontinuous
D Open
E Closed

C Discontinuous

The hepatic sinusoids are lined with a discontinuous endothelium. The endothelium contains
both large fenestrations (without diaphragms) and frequently has large gaps between adjacent
cells. Additionally, the basal lamina is also discontinuous, having large areas in which it is
absent.

Question:
What is the minimum number of barriers between toxins carried in the blood and the plasma
membrane of a hepatocyte?

Answer Choices:
A 0
B 1
C 2
D 3
E 4

A 0

Toxins carried in the blood are easily accessible to hepatocytes because the sinusoidal
endothelium is discontinuous and the underlying basal lamina is often absent in large areas.
Therefore, in these areas there would be no barrier between the toxin in the plasma and the
surface of the hepatocyte plasma membrane.

Question:
Cirrhosis of the liver

Answer Choices:
A Is a condition in which the hepatocyte stem-cells become dehydrated, and as such
they are unable to meet the demand for new cells
B Is a pathological condition caused by impaired supply of blood to the liver. Toxic
substances then tend to accumulate inside the liver
C Is a condition in which Glisson's capsule or its derivatives undergo excessive
proliferation, resulting in fibrosis (scar formation) in the liver
D Is due to the death of cells bordering the sinusoids. The death occurs due to lack of O 2
E Is condition in which bile accumulated inside the substance of the liver. This causes
irreparable damage

C Is a condition in which Glisson's capsule or its derivatives undergo excessive proliferation,


resulting in fibrosis (scar formation) in the liver

When the capsule or its derivatives undergo excessive proliferation, the resulting fibrosis (scar
formation) is called cirrhosis, a life-threatening condition. It is manifested by excessive amounts
of connective tissue forming principally around the central veins, in the portal canals, and
surrounding the hepatocytes. As a result of this extensive scarring, blood and bile flow is
obstructed, and the supply of O2 and nutrients to liver cells is impaired. The hepatocytes die
and are replaced by scar tissue.
Question:
Which one of the following embryonic components, or organs, gives rise to the gallbladder?

Answer Choices:
A Duodenal wall
B Wall of common bile duct
C Ventral surface of duodenal fore-gut
D Right hepatic duct
E Left hepatic duct

B Wall of common bile duct

The gallbladder and cystic duct are formed by a hollow evagination from the wall of the
common bile duct. The ventral surface of the duodenal fore-gut forms the outgrowths that will
eventually develop into bile ducts and the secretory portion of the liver. The duodenal wall
gives rise to the common bile duct. Right and left hepatic ducts themselves are produced from
the bile ducts.

Question:
Which one of the following components will increase in numbers in an aging hepatocyte?

Answer Choices:
A Ribosomes
B Microbodies
C Golgi complex
D Smooth endoplasmic reticulum
E Residual bodies

E Residual bodies

Residual bodies are undigested compounds that are retained within the vacuole of the
secondary lysosomes. In long-lived cells (e.g., neurons, heart muscles, hepatocytes), large
quantities of residual bodies, referred to as lipofuscin or age pigments, accumulate.
Lysosomes digest material taken into the cell by heterophagy. The material is taken into
phagocytic vacuole; primary lysosomes then fuse with the membrane of the phagosome and
empty their enzymes into the vacuole: this composite structure is called secondary lysosome
(phagolysosome). Following digestion, undigested compounds are retained as residual bodies.
Ribosomes (protein synthesizers), microbodies (involved in lipid metabolism), Golgi complex
(involved in lysosome formation and secretion of plasma proteins), and smooth endoplasmic
reticulum (involved in oxidation, methylation and conjugation required for inactivation of
toxins) will decrease in number in an aging cells.
Question:CLASS QUIZ
A hepatologist performs a biopsy on the liver of a 70-year-old alcoholic man. The tissue is dissociated
with collagenase and the individual cell types are separated and identified. Cells are stained with
Sirius red. What macrophage is isolated from the liver?

Answer Choices:
A Kupffer cells
B Histiocytes
C Langerhans cells
D Osteoclasts
E Microglia
F synovial type A cells
G interdigitating cells

A Kupffer cells

Microglia, Kupffer cells, and Langerhans cells are fixed macrophages located in the brain, liver,
and skin, respectively. Macrophages, which are located in the spleen and liver, reduce blood
borne pathogen loads. Macrophages also exist in the bone marrow, lamina propria of the GI
tract, lymph nodes, lungs, kidneys, endocrine organs, and perivascular space. Alveolar
macrophages, those located in the lungs, do not benefit from the assistance of opsonins in
their phagocytic activities. This contrasts to the macrophages located in the liver and spleen,
which do benefit from the assistance of circulating complement and immunoglobulin.
Neutrophils are recruited and activated by cytokines, which are secreted by macrophages in the
lung (e.g., Interleukin-8). Also, this production of IL8 is in addition to the production of IL1 and
IL6. Other cell types recognized as macrophages include histiocytes, osteoclasts, interdigitating
cells, and synovial type A cells. Histocyte is an alternate name for fixed macrophage.
Osteoclasts absorb and thereby remove bone, and do so with increased activity in response to
parathyroid hormone. As a result of this heightened activity, the extracellular fluid accumulates
both calcium and phosphorus. Antigen presenting cells in the thymus dependent (parafollicular)
regions of the deep cortex of lymph nodes and spleen are termed interdigitating cells, due to
their interdigitating with contiguous lymphocytes. Ia antigen is part of the major
histocompatibility complex and is located on the surface of the cells. This antigen results in the
aggregation of T cells. The cells making up the lining of the synovium have long been known as
type A and B synoviocytes, with an intermediate form sometimes also described. Accumulating
evidence shows that the type A cells are macrophages and the type B cells are fibroblasts.
Question:
Which of the following best describes the embryonic origin of the gall bladder?

Answer Choices:
A A diverticulum of the embryonic liver.
B An outpocketing of the common hepatic duct.
C A diverticulum of the embryonic stomach.
D An outpocketing of the common bile duct.
E An outpocketing of the pancreatic duct.

B An outpocketing of the common hepatic duct.

The liver, gall bladder, and the pancreas are all embryonic derivatives of endoderm which bud
from the duodenum. More specifically, the gall bladder appears as an outpocketing of the
developing common hepatic duct leading to the duodenum. The common bile duct is the
extrahepatic passageway between the cystic duct and union with the pancreatic duct. It is
thought not to be the embryonic source of the gall bladder.

Question: LAB
The bile canaliculi are best described as being lined by what epithelium?

Answer Choices:
A Simple squamous
B Simple cuboidal
C Simple columnar
D Transitional
E There is no epithelial lining of the bile canaliculi
E There is no epithelial lining of the bile canaliculi

The bile canaliculi are located between adjacent hepatocytes and are regarded as the first
portion of the intrahepatic passageways for bile secretion. As such, the canaliculi are formed as
a channel between the cellular membranes of the adjacent hepatocytes and are NOT lined by
an epithelium.

Question:
Which one of the following substances, is a major exocrine secretory product of the liver?

Answer Choices:
A Albumen
B Glucose
C Gastrin
D Glucagon
E Bile

E Bile

The liver serves as both an endocrine and an exocrine organ. Its function as an exocrine organ is
to synthesize and secrete bile, which is formed by bile acids. Substances that are synthesized
and endocrine secreted by hepatocytes include albumen and glucose. Gastrin and glucagon are
secreted by the pancreas.

Question:
Accumulation of fat in hepatocytes is first observed in which part of the liver lobule?

Answer Choices:
A Zone 1
B Zone 2
C Zone 3
D Surrounding the hepatic artery
E Surrounding the portal vein

C Zone 3

The liver acinus is a rough diamond shape in which the long axis is defined by a line connecting
two adjacent central veins closest to the short axis. The short axis of the liver acinus is defined
by an imaginary line connecting two adjacent portal triads, perpendicular to the closest long
axis (along the edge of a classic lobule). A branch of the hepatic artery is the central feature of
the short axis and is the distributing vessel for the liver acinus. Hepatocytes lying in Zone 3 (a
rough ellipse located furthest from the short axis and closest to the central vein) are the first to
show fat accumulation.

Question:
In addition to endothelial cells, what kinds of cells line the hepatic sinusoids?

Answer Choices:
A Fibroblasts
B Hepatocytes
C Ito cells
D Kupffer cells
E Mall cells

D Kupffer cells

Unlike the sinusoids located in other areas, the hepatic sinusoids are composed of two cell
types; the endothelial cell and the Kupffer cell. The Kupffer cells are believed to be derived
from monocytes and may be involved in phagocytosis functions, particularly of damaged red
blood cells.

Question:
The liver is derived embryologically from which one of the following structures?

Answer Choices:
A IVth pharyngeal pouch
B Stomodeum
C Foregut
D Midgut
E Hindgut

C Foregut

The primitive gut tube is initially a blind-ended straight tube extending from the cranial
buccopharyngeal membrane to the caudal-most cloacal membrane. The three portions of the
gut tube are defined by their arterial supply of which the abdominal foregut is supplied by the
celiac trunk. Derivatives of the foregut include the pharynx, esophagus, stomach, and superior
half of the duodenum. The liver is derived from the endoderm of the foregut.

Question: CLASS QUIZ


Which of the following statements concerning liver sinusoids is true?

Answer Choices:
A They are continuous with bile canaliculi
B They are surrounded by a well-developed basal lamina
C They are lined by nonfenestrated endothelial cells
D They deliver blood to the central vein
E They deliver blood to the portal vein

D They deliver blood to the central vein


Image(s) / Chart(s):
Click image to view full size. Click open image to close. Click and hold open image to move.

Liver sinusoids are lined by fenestrated endothelial cells, lack a basal lamina, and deliver blood
directly to the central vein.
See Figure: A section through a lobule of the liver, showing hepatocytes (H), endothelial cells
(Ec), space of Disse (Sd), and the liver capsule (Lc).
Question:
Which one of the following statements concerning the biliary space is correct?

Answer Choices:
A It is the narrow intercellular space between adjacent hepatocytes
B It is also known as the canals of Hering
C It is the narrow space between the endothelium of the sinusoid and the hepatocyte of
the hepatic lobule
D It is located inside the portal area
E It is the fissure on the surface of the liver where the hepatic artery and portal vein
enter the liver and the hepatic ducts leave

A It is the narrow intercellular space between adjacent hepatocytes

The biliary space (bile canaliculus) is the narrow intercellular space between the two adjacent
hepatocytes. It receives the liver's exocrine secretion (bile) and carries it to the canals of
Hering (bile ductules), located at the very periphery of classic liver lobules.
Bile ducts are located in the portal areas. The fissure on the surface of the liver where the
hepatic artery and portal vein enter the liver and the hepatic ducts leave, is known as porta
hepatis.

Case #301500:
A 50-year-old man initially presented with new onset jaundice and ascites. He has been a
heavy alcohol user on and off for approximately 15 years, and 2 years ago he started drinking
between a pint and a quart of vodka daily. CT shows decreased density of the liver tissue
because of the increased fat component making the image less bright. It raises the possibility
of the diagnosis of steatosis hepatis. Fatty change (steatosis) is one of the hallmarks of liver
cellular injury.
Question:
Of what is his condition most probably the consequence?

Answer Choices:
A Membrane damage by reactive oxygen species
B DNA damage
C Decreased intracellular glycogen levels
D Decreased synthesis of lipoproteins
E Decreased activity of the Na/K ATPase

D Decreased synthesis of lipoproteins

Steatosis is the process of the abnormal retention of lipids within a cell. It reflects an
impairment of the normal processes of the metabolism of triglycerides. This man is at high risk
for developing alcoholic liver disease: amounts of alcohol intake associated with an increased
risk for developing alcoholic liver disease range from 40-80 g/d for 10-12 years. Alcoholic drinks
produce large amounts of chemical energy signaling to the cell to inhibit the breakdown of fatty
acid and simultaneously increase their synthesis. Excess lipid accumulates in vesicles that
displace the cytoplasm.

Partially reduced oxygen free radicals cause injury to cell membranes and other cell
constituents, but they do not cause the accumulation of fat in liver cells.

DNA carries genetic information, and cells cannot function if DNA damage corrupts the integrity
and accessibility of essential information in the genome. However, accumulation of fats is not
linked to the DNA damage.

Glycogen is the secondary long-term energy storage in the cells. It is made primarily by the liver
and muscles, but can also be made by the brain and stomach. Glycogen levels are greatly
reduced in livers of chronic alcohol consumers because alcohol decreases levels of circulating
glucose, thus affecting glycogen repletion. Therefore, decreased intracellular glycogen is the
consequence and not the cause of hepatic steatosis.

Na+/K+-ATPase is an enzyme found in all mammalian cell membranes. It is necessary for proper
cellular function since it helps to preserve the ionic gradients across the cell membrane, and
thus the membrane potential and osmotic equilibrium of the cell. However, it is not, however,
linked to the hepatic steatosis.
SECTION- EXOCRINE PANCREAS AND SALIVARY GLANDS

Question:
Exocrine secretion of the pancreatic acinar cells may be inhibited by which hormone?

Answer Choices:
A Insulin
B Vasoactive intestinal peptide (VIP)
C CCK
D Pancreatic polypeptide (PP)
E Gastrin
D Pancreatic polypeptide (PP)

Several hormones act upon the pancreatic acinar cells to regulate the secretion of the zymogen
granules. Vasoactive intestinal peptide (VIP), CCK, and insulin will each stimulate the secretion
of the proenzymatic products from the acinar cells. Inhibition of the secretion from the acinar
cells is mediated by somatostatin, glucagon, and pancreatic polypeptide (PP). Gastrin stimulates
the secretion of gastric HCl.
Question:
Exocrine secretion of the pancreatic acinar cells may be stimulated by which hormone?
Answer Choices:
A Vasoactive intestinal peptide (VIP)
B Glucagon
C Pancreatic polypeptide (PP)
D Somatostatin
E Gastrin

A Vasoactive intestinal peptide (VIP)

Several hormones act upon the pancreatic acinar cells to regulate the secretion of the zymogen
granules. Vasoactive intestinal peptide (VIP), CCK, and insulin will each stimulate the secretion
of the proenzymatic products from the acinar cells. Inhibition of the secretion from the acinar
cells is mediated by somatostatin, glucagon, and pancreatic polypeptide (PP). Gastrin stimulates
the secretion of gastric HCl.
Question: CLASS QUIZ
Exocrine secretion of the pancreatic acinar cells may be stimulated by which hormone?

Answer Choices:
A CCK
B Glucagon
C Pancreatic polypeptide (PP)
D Somatostatin
E Gastrin

A CCK

Several hormones act upon the pancreatic acinar cells to regulate the secretion of the zymogen
granules. Vasoactive intestinal peptide (VIP), CCK, and insulin will each stimulate the secretion
of the proenzymatic products from the acinar cells. Inhibition of the secretion from the acinar
cells is mediated by somatostatin, glucagon, and pancreatic polypeptide (PP). Gastrin stimulates
the secretion of gastric HCl.
Question:
Exocrine secretion of the pancreatic acinar cells may be stimulated by which hormone?

Answer Choices:
A Insulin
B Glucagon
C Pancreatic polypeptide (PP)
D Somatostatin
E Gastrin

A Insulin

Several hormones act upon the pancreatic acinar cells to regulate the secretion of the zymogen
granules. Vasoactive intestinal peptide (VIP), CCK, and insulin will each stimulate the secretion
of the proenzymatic products from the acinar cells. Inhibition of the secretion from the acinar
cells is mediated by somatostatin, glucagon, and pancreatic polypeptide (PP). Gastrin stimulates
the secretion of gastric HCl.
Question:
Which one of the following nerve fibers provide innervation to regulate local pancreatic blood flow?

Answer Choices:
A Preganglionic, sympathetic fibers
B Postganglionic, sympathetic fibers
C Preganglionic, parasympathetic fibers
D Postganglionic, parasympathetic fibers
E Somatic motor fibers

B Postganglionic, sympathetic fibers

The pancreas receives innervation from both the sympathetic and parasympathetic fibers of the
autonomic nervous system. The sympathetic nervous system has its postganglionic nerve cells
in either the chain ganglia lateral to the vertebral column or in specific ganglia of the visceral
cavity. The postganglionic sympathetic fibers regulate the flow of blood to abdominal organs,
such as the pancreas.
The parasympathetic nervous system has its postganglionic nerve cells within the parenchyma
of the gland which it innervates, and as such, the soma of these postganglionic parasympathetic
fibers may be observed occasionally among the parenchyma of the pancreas. The
postganglionic, parasympathetic fibers stimulate secretion from the acinar cells of the
pancreas.
Question:
Nerve cell bodies may occasionally be found in the parenchyma of the pancreas. These neurons are
the soma for

Answer Choices:
A Preganglionic, sympathetic fiber inhibiting acinar cell secretion
B Postganglionic, sympathetic fibers stimulating acinar cell secretion
C Preganglionic, parasympathetic fibers stimulating acinar cell secretion
D Postganglionic, parasympathetic fibers stimulating acinar cell secretion
E Postganglionic, parasympathetic fibers inhibiting acinar cell secretion
D Postganglionic, parasympathetic fibers stimulating acinar cell secretion

The pancreas receives innervation from both the sympathetic and parasympathetic fibers of the
autonomic nervous system. The sympathetic nervous system has its postganglionic nerve cells
in either the chain ganglia lateral to the vertebral column or in specific ganglia of the visceral
cavity. The postganglionic sympathetic fibers regulate the flow of blood to abdominal organs,
such as the pancreas.
The parasympathetic nervous system has its postganglionic nerve cells within the parenchyma
of the gland which it innervates, and as such, the soma of these postganglionic parasympathetic
fibers may be observed occasionally among the parenchyma of the pancreas. The
postganglionic, parasympathetic fibers stimulate secretion from the acinar cells of the
pancreas.
Question:LAB
The epithelial cells of the intercalated duct will produce large volumes of fluid containing

Answer Choices:
A Pancreozymin
B Bicarbonate ions
C Trypsinogen
D Lipase
E Hydrochloride ions

B Bicarbonate ions

Epithelial cells of the intercalated duct produce large volumes of fluid (approximately 1
liter/day) rich in bicarbonate ions. The bicarbonate aids in neutralizing the acidic chyme that
enters the duodenum from the stomach to reach a pH optimal for activity of the pancreatic
enzymes.
Pancreozymin is a collective term for at least 2 hormones, secretin and CCK, that are secreted
by the enteroendocrine cells of the duodenum and regulate synthesis and secretion of the
exocrine pancreas.
Trypsinogen, pepsinogen, and procarboxypeptidase are pancreatic proenzymes that, when
activated, are responsible for digesting proteins, amylase for digesting carbohydrates, lipase for
digesting lipids, and nuclease for digesting nucleic acids.

Question: CLASS QUIZ


Which proenzyme, when secreted by the pancreas and converted into its active form, catalyzes the
conversion of the other proenzymes secreted by the exocrine pancreas into their active forms?
Answer Choices:
A Lipase
B Trypsinogen
C Nuclease
D Pepsinogen
E Amylase

B Trypsinogen

The acinar cells in the pancreas are characterized by distinct granules, zymogen granules,
located in the apex of each cell. These granules contain a variety of digestive enzymes in an
inactive form.
The proenzymes include trypsinogen, pepsinogen, procarboxypeptidase, that when activated
are responsible for digesting proteins, amylase, which will digest carbohydrates, lipase, which
will digest lipids, and nuclease for digesting nucleic acids.
Activation of the proenzymes contained within the zymogen granules occurs in the lumen of
the small intestine.
Trypsin, as such, is not secreted by the pancreas since trypsin is a metabolic product of
trypsinogen. Trypsin also catalyzes the conversion of the other proenzymes secreted by the
exocrine pancreas.
Question:
One enzyme secreted by the pancreas that is responsible for digesting carbohydrates is

Answer Choices:
A Lipase
B Trypsinogen
C Trypsin
D Pepsinogen
E Amylase

E Amylase

The acinar cells in the pancreas are characterized by distinct granules, zymogen granules,
located in the apex of each cell. These granules contain a variety of digestive enzymes in an
inactive form.
The proenzymes include trypsinogen, pepsinogen, and procarboxypeptidase that, when
activated, are responsible for digesting proteins, amylase which will digest carbohydrates,
lipase which will digest lipids, and nuclease for digesting nucleic acids.
Activation of the proenzymes contained within the zymogen granules occurs in the lumen of
the small intestine.
Trypsin is a metabolic product of trypsinogen and catalyzes the conversion of the other
proenzymes secreted by the exocrine pancreas.
Question:
Refer to the attached image. What are the intensely staining granules that can be seen in each of the
acinus cells in the pancreas at the tip of the pointer in the photomicroscopic image?

Answer Choices:
A Insulin granules
B Trypsin granules
C Pancreozymin granules
D Zymogen granules
E Somatostatin granules

Image(s) / Chart(s):
Click image to view full size. Click open image to close. Click and hold open image to move.

D Zymogen granules

The acinar cells in the pancreas are characterized by distinct granules located in the apex of
each cell. The intensely staining granules as seen in the acinar cells in the photomicroscopic
image are zymogen granules and contain a variety of digestive enzymes in an inactive form.
The proenzymes include trypsinogen, pepsinogen, procarboxypeptidase, amylase, lipase, and
nuclease for digesting proteins, carbohydrates, lipids, and nucleic acids. Activation of the
proenzymes contained within the zymogen granules occurs in the lumen of the small intestine.
Insulin and somatostatin are located within the cells of the islets, and as such are not secreted
into the exocrine component of the pancreas.
Pancreozymin is a collective term for at least two hormones, secretin and CCK, that are
secreted by the enteroendocrine cells of the duodenum and regulate synthesis and secretion of
the exocrine pancreas.
Trypsin is a metabolic product of trypsinogen (found within the zymogen granules) and
catalyzes the conversion of the other proenzymes secreted by the exocrine pancreas.

Question: CLASS QUIZ


Centroacinar cells are unique to the pancreas because
Answer Choices:
A The cell is the first part of the secretory duct that begins within the acinus
B The cell is the first part of the secretory duct that begins adjacent to the acinus
C The cell forms part of the secretory acinus
D The cells form the secretory duct located in the central part of the pancreas
E They form the central cells of the islets and secrete insulin
A The cell is the first part of the secretory duct that begins within the acinus

The exocrine pancreas is composed of acinar or tubuloacinar serous secretory units. The initial
portion of the secretory duct that leads from the acinus actually begins within the acinus and is
referred to as the intercalated duct. The cells located within the acinus that forms the
intercalated duct are the centroacinar cells.
Question:
See the attached photomicroscopic image. The cell that can be seen at the tip if the arrowhead in this
image is best described as a(n)

Answer Choices:
A Centroacinar cell
B Pancreatic acinar cell
C Intercalated duct cell
D Intralobular duct cell
E Pancreatic islet duct cell

Image(s) / Chart(s):
Click image to view full size. Click open image to close. Click and hold open image to move.

A Centroacinar cell
The arrowhead in the photomicroscopic image is pointing at a centroacinar cell, which is an
epithelial cell centrally positioned within the acinus that stains lightly. These cells are the
beginning of the exocrine ductal system for the pancreas and lead to the intercalated ducts.
The intercalated ducts lead to the intralobular duct, thence to large interlobular ducts.
There is no pancreatic islet duct, since the pancreatic islet cells are endocrine and secrete their
products into blood capillaries associated with the islets.

Question:
See the attached photomicroscopic image. The organ as seen in this image is best identified as

Answer Choices:
A Pancreas
B Liver
C Cortical kidney
D Anterior pituitary gland
E Adrenal cortex

Image(s) / Chart(s):
Click image to view full size. Click open image to close. Click and hold open image to move.

A Pancreas

The organ as seen in the photomicroscopic image is the pancreas with an islet of Langerhans
(pancreatic islet). The endocrine pancreas (islet) is identified as a roughly spherical
accumulation of cells that stain more acidophilic than the surrounding cells belonging to the
exocrine pancreas.
Several cell-types comprise the cells of the islet, but cannot be differentiated by H&E staining.
Typically, small cells located at the periphery of the islet are probably alpha (A) cells and secrete
glucagon. Larger beta (B) cells are frequently found in clusters located more centrally in the
islet. Delta (D) cells are located within the islet but are not differentiated by the H&E stain.
The exocrine pancreas can be visualized in the image as acini of serous cells. The acini are
organized in circular or irregular profiles. The lumen of the acini are typically very small and
each cell has a singular nucleus. The cytoplasm is filled with basophilic granules.
This photomicroscopic image may be confused with the cortical kidney since, without close
inspection, the islet may resemble a glomerulus and the acini may resemble proximal
convoluted tubules.

Question:
One enzyme secreted by the pancreas that is responsible for digesting lipids is

Answer Choices:
A Lipase
B Trypsinogen
C Trypsin
D Pepsinogen
E Amylase

A Lipase

The acinar cells in the pancreas are characterized by distinct granules, zymogen granules,
located in the apex of each cell. These granules contain a variety of digestive enzymes in an
inactive form.
The proenzymes include trypsinogen, pepsinogen, and procarboxypeptidase that, when
activated, are responsible for digesting proteins, amylase which will digest carbohydrates,
lipase which will digest lipids, and nuclease for digesting nucleic acids.
Activation of the proenzymes contained within the zymogen granules occurs in the lumen of
the small intestine.
Trypsin is a metabolic product of trypsinogen and catalyzes the conversion of the other
proenzymes secreted by the exocrine pancreas.
Question:
Refer to the attached image. What term best identifies the cells that are located at the tip of the
pointer in the photomicroscopic image of the pancreas?

Answer Choices:
A Centroacinar cells
B Intercalated duct
C Intralobular ducts
D Interlobular ducts
E Pancreatic duct
Image(s) / Chart(s):
Click image to view full size. Click open image to close. Click and hold open image to move.

C Intralobular ducts

In the pancreas, the exocrine ductal system begins within the acinus. The initial duct within the
acinus, made of centroacinar cells, is the intercalated duct. These ductal cells lack the intensely
staining zymogen granules and stain lightly, first appearing as squamous cells, then becoming
thicker as the duct enlarges.
The intercalated ducts are short and drain into intralobular ducts.
The intralobular ducts are composed of lightly staining cuboidal or low columnar cells and are
located within the parenchyma of the pancreatic lobules. This branching network of ducts then
converges and drains into larger interlobular ducts, which are lined with a simple, low columnar
epithelium.
The interlobular ducts are found between the lobules of the pancreas, surrounded by
connective tissue. The interlobular ducts converge to form the pancreatic duct, which is the
major large duct that runs parallel to the long axis of the pancreas. It, too, will be surrounded
by connective tissue and possesses a columnar epithelial lining.
Question:
Which one of the following hormones is the major hormone in regulation of the secretory activity of
the cells of the intercalated ducts of the pancreas, and what product is secreted from these cells?

Answer Choices:
A Secretin, hydrochloride ions
B Secretin, bicarbonate ions
C CCK, bicarbonate ions
D CCK, hydrochloride ions
E Gastrin, hydrochloride ions

B Secretin, bicarbonate ions


The epithelial cells of the intercalated ducts respond to secretin, for the secretion of fluids with
a high bicarbonate ion concentration. Secretin is secreted primarily by the enteroendocrine
cells of the duodenum, in response to the emptying of chyme from the stomach with an acidic
pH. Secretin acts upon the epithelial cells of the intercalated ducts to induce the secretion of
bicarbonate ions, in order to optimize the duodenal pH for enzymatic activity.
CCK is secreted primarily by the enteroendocrine cells of the duodenum and acts upon the
acinar cells of the pancreas, to secrete the proenzymes contained within the zymogen granules,
such as trypsmogen.
Gastrin acts upon the stomach to increase release of hydrochloride ions.

Question:
Exocrine secretion of the pancreatic acinar cells may be inhibited by which hormone?

Answer Choices:
A Insulin
B Vasoactive intestinal peptide (VIP)
C CCK
D Somatostatin
E Gastrin

D Somatostatin

Several hormones act upon the pancreatic acinar cells to regulate the secretion of the zymogen
granules. Vasoactive intestinal peptide (VIP), CCK, and insulin will each stimulate the secretion
of the proenzymatic products from the acinar cells. Inhibition of the secretion from the acinar
cells is mediated by somatostatin, glucagon, and pancreatic polypeptide (PP). Gastrin stimulates
the secretion of gastric HCl.
Question:
In pancreatitis, it is suspected that enzymes normally secreted by the pancreas in an inactive form
(such as trypsinogen and chymotrypsinogen) become activated in the pancreas. The zymogens
trypsinogen and chymotrypsinogen are normally converted to active enzymes in the

Answer Choices:
A Serum
B Large intestine
C Small intestine
D Liver
E Kidney

C Small intestine
Trypsinogen and chymotrypsinogen are synthesized in the pancreas and stored in zymogen
granules. When properly stimulated, these granules fuse with the plasma membrane and
release their contents in a process called exocytosis. These two inactive enzymes (zymogens)
are converted to active enzymes only when they reach the small intestine. The enzyme
enteropeptidase activates trypsinogen by cleaving off an amino-terminal hexapeptide. This
active trypsin can then activate trypsinogen and the other zymogens such as
chymotrypsinogen. A small peptide trypsin inhibitor is found in the pancreatic juice. This
inhibitor will inactivate any trypsin, which forms prematurely in the pancreas, and prevent
digestion and tissue destruction. In pancreatitis, an inflammation of the pancreas, the
zymogens are prematurely activated in the pancreas. This leads to autodigestion of the
pancreas and causes swelling, hemorrhage, and damage to the blood vessels.

Question:
An example of a compound tubuloacinar gland would be which one of the following?

Answer Choices:
A Intestinal glands of the colon
B Exocrine pancreas
C Cardiac glands of the stomach
D Eccrine sweat gland
E Submandibular gland

E Submandibular gland

A compound tubuloacinar gland is a gland in which the duct leading to the surface is branched
(compound), and the secretory portion of the gland is tubular shaped, with a terminal flask-like
shape, the acinus. Of the exocrine glands listed, only the submandibular gland exhibits the
features of a compound tubuloacinar gland. The intestinal glands of the colon are simple
tubular. The pancreas is a compound acinar gland, and the cardiac glands of the stomach are
simple branched acinar glands. Eccrine sweat glands are simple coiled tubular glands.

Question:
One enzyme secreted by the pancreas that is responsible for digesting nucleic acids is

Answer Choices:
A Lipase
B Trypsinogen
C Nuclease
D Pepsinogen
E Amylase

C Nuclease

The acinar cells in the pancreas are characterized by distinct granules, zymogen granules,
located in the apex of each cell. These granules contain a variety of digestive enzymes in an
inactive form.
The proenzymes include trypsinogen, pepsinogen, and procarboxypeptidase that, when
activated, are responsible for digesting proteins, amylase which will digest carbohydrates,
lipase which will digest lipids, and nuclease for digesting nucleic acids.
Activation of the proenzymes contained within the zymogen granules occurs in the lumen of
the small intestine.
Trypsin is a metabolic product of trypsinogen and catalyzes the conversion of the other
proenzymes secreted by the exocrine pancreas.

Question:
Which duct within the pancreas is lined with simple columnar epithelium and runs the length of the
gland parallel to its long axis?

Answer Choices:
A Centroacinar cells
B Intercalated duct
C Intralobular ducts
D Interlobular ducts
E Pancreatic duct

E Pancreatic duct

In the pancreas, the exocrine ductal system begins within the acinus. The initial duct within the
acinus, made of centroacinar cells, is the intercalated duct. These ductal cells lack the intensely
staining zymogen granules and stain lightly, first appearing as squamous cells, then becoming
thicker as the duct enlarges.
The intercalated ducts are short and drain into intralobular ducts.
The intralobular ducts are composed of lightly staining cuboidal or low columnar cells and are
located within the parenchyma of the pancreatic lobules. This branching network of ducts then
converges and drains into larger interlobular ducts, which are lined with a simple, low columnar
epithelium.
The interlobular ducts are found between the lobules of the pancreas, surrounded by
connective tissue. The interlobular ducts converge to form the pancreatic duct, which is the
major large duct that runs parallel to the long axis of the pancreas. It, too, will be surrounded
by connective tissue and possesses a columnar epithelial lining.
Question:
Which one of the following hormones, is the major hormone in regulation of the secretory activity of
the pancreatic acinar cells, and what product is secreted from the acinar cells?

Answer Choices:
A Secretin, trypsinogen
B Secretin, bicarbonate ions
C CCK, bicarbonate ions
D CCK, trypsinogen
E Gastrin, hydrochloride ions
D CCK, trypsinogen

The epithelial cells of the intercalated ducts respond to secretin, for the secretion of fluids with
a high bicarbonate ion concentration. Secretin is secreted primarily by the enteroendocrine
cells of the duodenum, in response to the emptying of chyme from the stomach with an acidic
pH. Secretin acts upon the epithelial cells of the intercalated ducts to induce the secretion of
bicarbonate ions, in order to optimize the duodenal pH for enzymatic activity.
CCK is secreted primarily by the enteroendocrine cells of the duodenum and acts upon the
acinar cells of the pancreas, to secrete the proenzymes contained within the zymogen granules,
such as trypsmogen.
Gastrin acts upon the stomach to increase release of hydrochloride ions.
Question:
Exocrine secretion of the pancreatic acinar cells may be inhibited by which hormone?

Answer Choices:
A Insulin
B Vasoactive intestinal peptide (VIP)
C CCK
D Glucagon
E Gastrin

D Glucagon

Several hormones act upon the pancreatic acinar cells to regulate the secretion of the zymogen
granules. Vasoactive intestinal peptide (VIP), CCK, and insulin will each stimulate the secretion
of the proenzymatic products from the acinar cells. Inhibition of the secretion from the acinar
cells is mediated by somatostatin, glucagon, and pancreatic polypeptide (PP). Gastrin stimulates
the secretion of gastric HCl.
Question:
In pancreatitis, it is suspected that enzymes normally secreted by the pancreas in an inactive form
(such as trypsinogen and chymotrypsinogen) become activated in the pancreas. The zymogen
trypsinogen is normally converted to the active enzyme trypsin by the enzyme

Answer Choices:
A Thrombin
B Chymotrypsinogen
C Enteropeptidase
D Protein kinase A
E Pepsin

C Enteropeptidase

Trypsinogen and chymotrypsinogen are synthesized in the pancreas and stored in zymogen
granules. When properly stimulated, these granules fuse with the plasma membrane and
release their contents in a process called "exocytosis". These two inactive enzymes (zymogens)
are converted to active enzymes only when they reach the small intestine. The enzyme
enteropeptidase activates trypsinogen by cleaving off an amino-terminal hexapeptide.
Normally, this active trypsin can then activate trypsinogen and the other zymogens such as
chymotrypsinogen. A small peptide trypsin inhibitor is found in the pancreatic juice. This
inhibitor will inactivate any trypsin that forms prematurely in the pancreas and prevent
digestion and tissue destruction. In pancreatitis, an inflammation of the pancreas, the
zymogens are prematurely activated in the pancreas. This leads to autodigestion of the
pancreas and causes swelling, hemorrhage, and damage to the blood vessels.

SECTION- ISLETS OF LANGERHANS


Question:
Exocrine secretion of the pancreatic acinar cells may be inhibited by which hormone?

Answer Choices:
A Insulin
B Vasoactive intestinal peptide (VIP)
C CCK
D Pancreatic polypeptide (PP)
E Gastrin
D Pancreatic polypeptide (PP)

Several hormones act upon the pancreatic acinar cells to regulate the secretion of the
zymogen granules. Vasoactive intestinal peptide (VIP), CCK, and insulin will each
stimulate the secretion of the proenzymatic products from the acinar cells. Inhibition of
the secretion from the acinar cells is mediated by somatostatin, glucagon, and
pancreatic polypeptide (PP). Gastrin stimulates the secretion of gastric HCl.
Question:
Exocrine secretion of the pancreatic acinar cells may be stimulated by which hormone?

Answer Choices:
A Vasoactive intestinal peptide (VIP)
B Glucagon
C Pancreatic polypeptide (PP)
D Somatostatin
E Gastrin
A Vasoactive intestinal peptide (VIP)

Several hormones act upon the pancreatic acinar cells to regulate the secretion of the
zymogen granules. Vasoactive intestinal peptide (VIP), CCK, and insulin will each
stimulate the secretion of the proenzymatic products from the acinar cells. Inhibition of
the secretion from the acinar cells is mediated by somatostatin, glucagon, and
pancreatic polypeptide (PP). Gastrin stimulates the secretion of gastric HCl.
Question:
Exocrine secretion of the pancreatic acinar cells may be stimulated by which hormone?

Answer Choices:
A CCK
B Glucagon
C Pancreatic polypeptide (PP)
D Somatostatin
E Gastrin
A CCK

Several hormones act upon the pancreatic acinar cells to regulate the secretion of the
zymogen granules. Vasoactive intestinal peptide (VIP), CCK, and insulin will each
stimulate the secretion of the proenzymatic products from the acinar cells. Inhibition of
the secretion from the acinar cells is mediated by somatostatin, glucagon, and
pancreatic polypeptide (PP). Gastrin stimulates the secretion of gastric HCl.
Question:
Exocrine secretion of the pancreatic acinar cells may be stimulated by which hormone?

Answer Choices:
A Insulin
B Glucagon
C Pancreatic polypeptide (PP)
D Somatostatin
E Gastrin
A Insulin

Several hormones act upon the pancreatic acinar cells to regulate the secretion of the
zymogen granules. Vasoactive intestinal peptide (VIP), CCK, and insulin will each
stimulate the secretion of the proenzymatic products from the acinar cells. Inhibition of
the secretion from the acinar cells is mediated by somatostatin, glucagon, and
pancreatic polypeptide (PP). Gastrin stimulates the secretion of gastric HCl.
Question:
Which one of the following nerve fibers provide innervation to regulate local pancreatic blood flow?

Answer Choices:
A Preganglionic, sympathetic fibers
B Postganglionic, sympathetic fibers
C Preganglionic, parasympathetic fibers
D Postganglionic, parasympathetic fibers
E Somatic motor fibers

B Postganglionic, sympathetic fibers

The pancreas receives innervation from both the sympathetic and parasympathetic
fibers of the autonomic nervous system. The sympathetic nervous system has its
postganglionic nerve cells in either the chain ganglia lateral to the vertebral column or in
specific ganglia of the visceral cavity. The postganglionic sympathetic fibers regulate the
flow of blood to abdominal organs, such as the pancreas.
The parasympathetic nervous system has its postganglionic nerve cells within the
parenchyma of the gland which it innervates, and as such, the soma of these
postganglionic parasympathetic fibers may be observed occasionally among the
parenchyma of the pancreas. The postganglionic, parasympathetic fibers stimulate
secretion from the acinar cells of the pancreas.
Question:
Nerve cell bodies may occasionally be found in the parenchyma of the pancreas. These neurons are
the soma for

Answer Choices:
A Preganglionic, sympathetic fiber inhibiting acinar cell secretion
B Postganglionic, sympathetic fibers stimulating acinar cell secretion
C Preganglionic, parasympathetic fibers stimulating acinar cell secretion
D Postganglionic, parasympathetic fibers stimulating acinar cell secretion
E Postganglionic, parasympathetic fibers inhibiting acinar cell secretion

D Postganglionic, parasympathetic fibers stimulating acinar cell secretion

The pancreas receives innervation from both the sympathetic and parasympathetic
fibers of the autonomic nervous system. The sympathetic nervous system has its
postganglionic nerve cells in either the chain ganglia lateral to the vertebral column or in
specific ganglia of the visceral cavity. The postganglionic sympathetic fibers regulate the
flow of blood to abdominal organs, such as the pancreas.
The parasympathetic nervous system has its postganglionic nerve cells within the
parenchyma of the gland which it innervates, and as such, the soma of these
postganglionic parasympathetic fibers may be observed occasionally among the
parenchyma of the pancreas. The postganglionic, parasympathetic fibers stimulate
secretion from the acinar cells of the pancreas.
Question:
The epithelial cells of the intercalated duct will produce large volumes of fluid containing

Answer Choices:
A Pancreozymin
B Bicarbonate ions
C Trypsinogen
D Lipase
E Hydrochloride ions

B Bicarbonate ions

Epithelial cells of the intercalated duct produce large volumes of fluid (approximately 1
liter/day) rich in bicarbonate ions. The bicarbonate aids in neutralizing the acidic chyme
that enters the duodenum from the stomach to reach a pH optimal for activity of the
pancreatic enzymes.
Pancreozymin is a collective term for at least 2 hormones, secretin and CCK, that are
secreted by the enteroendocrine cells of the duodenum and regulate synthesis and
secretion of the exocrine pancreas.
Trypsinogen, pepsinogen, and procarboxypeptidase are pancreatic proenzymes that,
when activated, are responsible for digesting proteins, amylase for digesting
carbohydrates, lipase for digesting lipids, and nuclease for digesting nucleic acids.
Question:
Which proenzyme, when secreted by the pancreas and converted into its active form, catalyzes the
conversion of the other proenzymes secreted by the exocrine pancreas into their active forms?

Answer Choices:
A Lipase
B Trypsinogen
C Nuclease
D Pepsinogen
E Amylase

B Trypsinogen
The acinar cells in the pancreas are characterized by distinct granules, zymogen
granules, located in the apex of each cell. These granules contain a variety of digestive
enzymes in an inactive form.
The proenzymes include trypsinogen, pepsinogen, procarboxypeptidase, that when
activated are responsible for digesting proteins, amylase, which will digest
carbohydrates, lipase, which will digest lipids, and nuclease for digesting nucleic acids.
Activation of the proenzymes contained within the zymogen granules occurs in the
lumen of the small intestine.
Trypsin, as such, is not secreted by the pancreas since trypsin is a metabolic product of
trypsinogen. Trypsin also catalyzes the conversion of the other proenzymes secreted by
the exocrine pancreas.
Question:
One enzyme secreted by the pancreas that is responsible for digesting carbohydrates is

Answer Choices:
A Lipase
B Trypsinogen
C Trypsin
D Pepsinogen
E Amylase

E Amylase

The acinar cells in the pancreas are characterized by distinct granules, zymogen
granules, located in the apex of each cell. These granules contain a variety of digestive
enzymes in an inactive form.
The proenzymes include trypsinogen, pepsinogen, and procarboxypeptidase that, when
activated, are responsible for digesting proteins, amylase which will digest
carbohydrates, lipase which will digest lipids, and nuclease for digesting nucleic acids.
Activation of the proenzymes contained within the zymogen granules occurs in the
lumen of the small intestine.
Trypsin is a metabolic product of trypsinogen and catalyzes the conversion of the other
proenzymes secreted by the exocrine pancreas.

Question:
Refer to the attached image. What are the intensely staining granules that can be seen in each of the
acinus cells in the pancreas at the tip of the pointer in the photomicroscopic image?

Answer Choices:
A Insulin granules
B Trypsin granules
C Pancreozymin granules
D Zymogen granules
E Somatostatin granules

Image(s) / Chart(s):
Click image to view full size. Click open image to close. Click and hold open image to move.

D Zymogen granules

The acinar cells in the pancreas are characterized by distinct granules located in the
apex of each cell. The intensely staining granules as seen in the acinar cells in the
photomicroscopic image are zymogen granules and contain a variety of digestive
enzymes in an inactive form.
The proenzymes include trypsinogen, pepsinogen, procarboxypeptidase, amylase,
lipase, and nuclease for digesting proteins, carbohydrates, lipids, and nucleic acids.
Activation of the proenzymes contained within the zymogen granules occurs in the
lumen of the small intestine.
Insulin and somatostatin are located within the cells of the islets, and as such are not
secreted into the exocrine component of the pancreas.
Pancreozymin is a collective term for at least two hormones, secretin and CCK, that are
secreted by the enteroendocrine cells of the duodenum and regulate synthesis and
secretion of the exocrine pancreas.
Trypsin is a metabolic product of trypsinogen (found within the zymogen granules) and
catalyzes the conversion of the other proenzymes secreted by the exocrine pancreas.
Question:
Centroacinar cells are unique to the pancreas because

Answer Choices:
A The cell is the first part of the secretory duct that begins within the acinus
B The cell is the first part of the secretory duct that begins adjacent to the acinus
C The cell forms part of the secretory acinus
D The cells form the secretory duct located in the central part of the pancreas
E They form the central cells of the islets and secrete insulin

A The cell is the first part of the secretory duct that begins within the acinus
The exocrine pancreas is composed of acinar or tubuloacinar serous secretory units.
The initial portion of the secretory duct that leads from the acinus actually begins within
the acinus and is referred to as the intercalated duct. The cells located within the acinus
that forms the intercalated duct are the centroacinar cells.
Question:
See the attached photomicroscopic image. The cell that can be seen at the tip if the arrowhead in this
image is best described as a(n)

Answer Choices:
A Centroacinar cell
B Pancreatic acinar cell
C Intercalated duct cell
D Intralobular duct cell
E Pancreatic islet duct cell

Image(s) / Chart(s):
Click image to view full size. Click open image to close. Click and hold open image to move.

A Centroacinar cell

The arrowhead in the photomicroscopic image is pointing at a centroacinar cell, which is


an epithelial cell centrally positioned within the acinus that stains lightly. These cells are
the beginning of the exocrine ductal system for the pancreas and lead to the
intercalated ducts. The intercalated ducts lead to the intralobular duct, thence to large
interlobular ducts.
There is no pancreatic islet duct, since the pancreatic islet cells are endocrine and
secrete their products into blood capillaries associated with the islets.
Question:
See the attached photomicroscopic image. The organ as seen in this image is best identified as

Answer Choices:
A Pancreas
B Liver
C Cortical kidney
D Anterior pituitary gland
E Adrenal cortex

Image(s) / Chart(s):
Click image to view full size. Click open image to close. Click and hold open image to move.

A Pancreas

The organ as seen in the photomicroscopic image is the pancreas with an islet of
Langerhans (pancreatic islet). The endocrine pancreas (islet) is identified as a roughly
spherical accumulation of cells that stain more acidophilic than the surrounding cells
belonging to the exocrine pancreas.
Several cell-types comprise the cells of the islet, but cannot be differentiated by H&E
staining. Typically, small cells located at the periphery of the islet are probably alpha (A)
cells and secrete glucagon. Larger beta (B) cells are frequently found in clusters located
more centrally in the islet. Delta (D) cells are located within the islet but are not
differentiated by the H&E stain.
The exocrine pancreas can be visualized in the image as acini of serous cells. The acini
are organized in circular or irregular profiles. The lumen of the acini are typically very
small and each cell has a singular nucleus. The cytoplasm is filled with basophilic
granules.
This photomicroscopic image may be confused with the cortical kidney since, without
close inspection, the islet may resemble a glomerulus and the acini may resemble
proximal convoluted tubules.

Question:
One enzyme secreted by the pancreas that is responsible for digesting lipids is

Answer Choices:
A Lipase
B Trypsinogen
C Trypsin
D Pepsinogen
E Amylase
A Lipase

The acinar cells in the pancreas are characterized by distinct granules, zymogen
granules, located in the apex of each cell. These granules contain a variety of digestive
enzymes in an inactive form.
The proenzymes include trypsinogen, pepsinogen, and procarboxypeptidase that, when
activated, are responsible for digesting proteins, amylase which will digest
carbohydrates, lipase which will digest lipids, and nuclease for digesting nucleic acids.
Activation of the proenzymes contained within the zymogen granules occurs in the
lumen of the small intestine.
Trypsin is a metabolic product of trypsinogen and catalyzes the conversion of the other
proenzymes secreted by the exocrine pancreas.
Question:
Refer to the attached image. What term best identifies the cells that are located at the tip of the pointer
in the photomicroscopic image of the pancreas?

Answer Choices:
A Centroacinar cells
B Intercalated duct
C Intralobular ducts
D Interlobular ducts
E Pancreatic duct

Image(s) / Chart(s):
Click image to view full size. Click open image to close. Click and hold open image to move.
C Intralobular ducts

In the pancreas, the exocrine ductal system begins within the acinus. The initial duct
within the acinus, made of centroacinar cells, is the intercalated duct. These ductal cells
lack the intensely staining zymogen granules and stain lightly, first appearing as
squamous cells, then becoming thicker as the duct enlarges.
The intercalated ducts are short and drain into intralobular ducts.
The intralobular ducts are composed of lightly staining cuboidal or low columnar cells
and are located within the parenchyma of the pancreatic lobules. This branching
network of ducts then converges and drains into larger interlobular ducts, which are
lined with a simple, low columnar epithelium.
The interlobular ducts are found between the lobules of the pancreas, surrounded by
connective tissue. The interlobular ducts converge to form the pancreatic duct, which is
the major large duct that runs parallel to the long axis of the pancreas. It, too, will be
surrounded by connective tissue and possesses a columnar epithelial lining.
Question:
Which one of the following hormones is the major hormone in regulation of the secretory activity of the
cells of the intercalated ducts of the pancreas, and what product is secreted from these cells?

Answer Choices:
A Secretin, hydrochloride ions
B Secretin, bicarbonate ions
C CCK, bicarbonate ions
D CCK, hydrochloride ions
E Gastrin, hydrochloride ions

B Secretin, bicarbonate ions

The epithelial cells of the intercalated ducts respond to secretin, for the secretion of
fluids with a high bicarbonate ion concentration. Secretin is secreted primarily by the
enteroendocrine cells of the duodenum, in response to the emptying of chyme from the
stomach with an acidic pH. Secretin acts upon the epithelial cells of the intercalated
ducts to induce the secretion of bicarbonate ions, in order to optimize the duodenal pH
for enzymatic activity.
CCK is secreted primarily by the enteroendocrine cells of the duodenum and acts upon
the acinar cells of the pancreas, to secrete the proenzymes contained within the
zymogen granules, such as trypsmogen.
Gastrin acts upon the stomach to increase release of hydrochloride ions.
Question:
Exocrine secretion of the pancreatic acinar cells may be inhibited by which hormone?

Answer Choices:
A Insulin
B Vasoactive intestinal peptide (VIP)
C CCK
D Somatostatin
E Gastrin

D Somatostatin

Several hormones act upon the pancreatic acinar cells to regulate the secretion of the
zymogen granules. Vasoactive intestinal peptide (VIP), CCK, and insulin will each
stimulate the secretion of the proenzymatic products from the acinar cells. Inhibition of
the secretion from the acinar cells is mediated by somatostatin, glucagon, and
pancreatic polypeptide (PP). Gastrin stimulates the secretion of gastric HCl.
Question:
In pancreatitis, it is suspected that enzymes normally secreted by the pancreas in an inactive form
(such as trypsinogen and chymotrypsinogen) become activated in the pancreas. The zymogens
trypsinogen and chymotrypsinogen are normally converted to active enzymes in the

Answer Choices:
A Serum
B Large intestine
C Small intestine
D Liver
E Kidney

C Small intestine

Trypsinogen and chymotrypsinogen are synthesized in the pancreas and stored in


zymogen granules. When properly stimulated, these granules fuse with the plasma
membrane and release their contents in a process called exocytosis. These two inactive
enzymes (zymogens) are converted to active enzymes only when they reach the small
intestine. The enzyme enteropeptidase activates trypsinogen by cleaving off an amino-
terminal hexapeptide. This active trypsin can then activate trypsinogen and the other
zymogens such as chymotrypsinogen. A small peptide trypsin inhibitor is found in the
pancreatic juice. This inhibitor will inactivate any trypsin, which forms prematurely in the
pancreas, and prevent digestion and tissue destruction. In pancreatitis, an inflammation
of the pancreas, the zymogens are prematurely activated in the pancreas. This leads to
autodigestion of the pancreas and causes swelling, hemorrhage, and damage to the
blood vessels.
Question:
An example of a compound tubuloacinar gland would be which one of the following?

Answer Choices:
A Intestinal glands of the colon
B Exocrine pancreas
C Cardiac glands of the stomach
D Eccrine sweat gland
E Submandibular gland

E Submandibular gland

A compound tubuloacinar gland is a gland in which the duct leading to the surface is
branched (compound), and the secretory portion of the gland is tubular shaped, with a
terminal flask-like shape, the acinus. Of the exocrine glands listed, only the
submandibular gland exhibits the features of a compound tubuloacinar gland. The
intestinal glands of the colon are simple tubular. The pancreas is a compound acinar
gland, and the cardiac glands of the stomach are simple branched acinar glands.
Eccrine sweat glands are simple coiled tubular glands.
Question:
One enzyme secreted by the pancreas that is responsible for digesting nucleic acids is

Answer Choices:
A Lipase
B Trypsinogen
C Nuclease
D Pepsinogen
E Amylase

C Nuclease

The acinar cells in the pancreas are characterized by distinct granules, zymogen
granules, located in the apex of each cell. These granules contain a variety of digestive
enzymes in an inactive form.
The proenzymes include trypsinogen, pepsinogen, and procarboxypeptidase that, when
activated, are responsible for digesting proteins, amylase which will digest
carbohydrates, lipase which will digest lipids, and nuclease for digesting nucleic acids.
Activation of the proenzymes contained within the zymogen granules occurs in the
lumen of the small intestine.
Trypsin is a metabolic product of trypsinogen and catalyzes the conversion of the other
proenzymes secreted by the exocrine pancreas.
Question:
Which duct within the pancreas is lined with simple columnar epithelium and runs the length of the
gland parallel to its long axis?

Answer Choices:
A Centroacinar cells
B Intercalated duct
C Intralobular ducts
D Interlobular ducts
E Pancreatic duct

E Pancreatic duct

In the pancreas, the exocrine ductal system begins within the acinus. The initial duct
within the acinus, made of centroacinar cells, is the intercalated duct. These ductal cells
lack the intensely staining zymogen granules and stain lightly, first appearing as
squamous cells, then becoming thicker as the duct enlarges.
The intercalated ducts are short and drain into intralobular ducts.
The intralobular ducts are composed of lightly staining cuboidal or low columnar cells
and are located within the parenchyma of the pancreatic lobules. This branching
network of ducts then converges and drains into larger interlobular ducts, which are
lined with a simple, low columnar epithelium.
The interlobular ducts are found between the lobules of the pancreas, surrounded by
connective tissue. The interlobular ducts converge to form the pancreatic duct, which is
the major large duct that runs parallel to the long axis of the pancreas. It, too, will be
surrounded by connective tissue and possesses a columnar epithelial lining.
Question:
Which one of the following hormones, is the major hormone in regulation of the secretory activity of the
pancreatic acinar cells, and what product is secreted from the acinar cells?

Answer Choices:
A Secretin, trypsinogen
B Secretin, bicarbonate ions
C CCK, bicarbonate ions
D CCK, trypsinogen
E Gastrin, hydrochloride ions

D CCK, trypsinogen
The epithelial cells of the intercalated ducts respond to secretin, for the secretion of
fluids with a high bicarbonate ion concentration. Secretin is secreted primarily by the
enteroendocrine cells of the duodenum, in response to the emptying of chyme from the
stomach with an acidic pH. Secretin acts upon the epithelial cells of the intercalated
ducts to induce the secretion of bicarbonate ions, in order to optimize the duodenal pH
for enzymatic activity.
CCK is secreted primarily by the enteroendocrine cells of the duodenum and acts upon
the acinar cells of the pancreas, to secrete the proenzymes contained within the
zymogen granules, such as trypsmogen.
Gastrin acts upon the stomach to increase release of hydrochloride ions.
Question:
Exocrine secretion of the pancreatic acinar cells may be inhibited by which hormone?

Answer Choices:
A Insulin
B Vasoactive intestinal peptide (VIP)
C CCK
D Glucagon
E Gastrin

D Glucagon

Several hormones act upon the pancreatic acinar cells to regulate the secretion of the
zymogen granules. Vasoactive intestinal peptide (VIP), CCK, and insulin will each
stimulate the secretion of the proenzymatic products from the acinar cells. Inhibition of
the secretion from the acinar cells is mediated by somatostatin, glucagon, and
pancreatic polypeptide (PP). Gastrin stimulates the secretion of gastric HCl.

Question:
In pancreatitis, it is suspected that enzymes normally secreted by the pancreas in an inactive form
(such as trypsinogen and chymotrypsinogen) become activated in the pancreas. The zymogen
trypsinogen is normally converted to the active enzyme trypsin by the enzyme

Answer Choices:
A Thrombin
B Chymotrypsinogen
C Enteropeptidase
D Protein kinase A
E Pepsin

C Enteropeptidase

Trypsinogen and chymotrypsinogen are synthesized in the pancreas and stored in


zymogen granules. When properly stimulated, these granules fuse with the plasma
membrane and release their contents in a process called "exocytosis". These two
inactive enzymes (zymogens) are converted to active enzymes only when they reach
the small intestine. The enzyme enteropeptidase activates trypsinogen by cleaving off
an amino-terminal hexapeptide. Normally, this active trypsin can then activate
trypsinogen and the other zymogens such as chymotrypsinogen. A small peptide trypsin
inhibitor is found in the pancreatic juice. This inhibitor will inactivate any trypsin that
forms prematurely in the pancreas and prevent digestion and tissue destruction. In
pancreatitis, an inflammation of the pancreas, the zymogens are prematurely activated
in the pancreas. This leads to autodigestion of the pancreas and causes swelling,
hemorrhage, and damage to the blood vessels.
SECTION- ISLETS OF LANGERHANS
Question:
Identify the organ as seen in the attached photomicroscopic image

Answer Choices:
A Kidney cortex
B Intermediate lobe of the pituitary gland
C Pancreas
D Thyroid gland
E Apocrine glands

Image(s) / Chart(s):
Click image to view full size. Click open image to close. Click and hold open image to move.

D Thyroid gland

The photomicroscopic image depicts the appearance of the thyroid gland. The thyroid
gland, as one of the endocrine glands, consists of a gel-like mass of colloid which fills
the follicles, each surrounded by a wall of simple squamous or cuboidal cells, the
follicular cells. The follicular cells secrete thyroxine (T4) and T3 (triiodothyronine). These
hormones regulate cellular metabolism. One other cell type present in the follicular wall
is the parafollicular cells (C-cells), which secrete calcitonin, a hormone that lowers blood
calcium levels. The colloid filled follicles are somewhat spherical in shape and
separated from one another by connective tissue where a rich vascular innervation may
be found.
On occasion, limited numbers of similarly appearing follicles or cysts may be observed
in the intermediate lobe of the pituitary gland. These differ, however, in that various cells
associated with the anterior lobe of the pituitary may be found in approximation to the
cysts located in the intermediate lobe.
Likewise, there may be confusion of the thyroid with the appearance of apocrine glands,
the modified sweat glands commonly associated with the axillary and groin regions.
Apocrine glands, however, do not commonly appear filled with the colloid substance,
and are more limited in their numbers in the connective tissue.

Question:
Parasympathetic stimulation of nerves innervating the islets of the pancreas will

Answer Choices:
A Have no effect
B Increase secretion of insulin from B cells
C Decrease secretion of insulin from B cells
D Increase secretion of glucagon from B cells
E Decrease secretion of glucagon from A cells
B Increase secretion of insulin from B cells

The pancreatic islets receive innervation from both the sympathetic and
parasympathetic nervous system. Stimulation of the parasympathetic system increases
secretion of insulin from the B cells. Stimulation of the sympathetic system increases
glucagon secretion from the A cells. Insulin is inhibited by sympathetic stimulation.

Question:
Insulin release may be stimulated by which one of the following hormones from an enteroendocrine
cell?

Answer Choices:
A Gastrin
B VIP (vasoactive intestinal peptide)
C Secretin
D CCK (cholecystokinin)
E PP (pancreatic polypeptide)

B VIP (vasoactive intestinal peptide)


VIP will stimulate insulin release from the pancreas as well as stimulate pancreatic
bicarbonate ion secretion. Gastrin stimulates acid secretion. Secretin acts primarily on
the pancreas to stimulate bicarbonate ion secretion and enzyme release. CCK has
similar actions on the pancreas to those of secretin, but also will stimulate gallbladder
contraction. CCK will also reduce gastric emptying. PP has an opposing action to
secretin and CCK, which reduces pancreatic bicarbonate ion and enzyme secretion.
Question:
Using immunocytochemical staining, cell types within the pancreatic islets have been distinguished.
Cells which secrete gastric inhibitory peptide (GIP) are identified as:

Answer Choices:
A A cells
B EC cells
C D cells
D D-1 cells
E F cells

A A cells

Specific cells within the pancreatic islet have been differentiated using
immunocytochemical stains specific to various hormones. A cells secrete a variety of
hormones, one of which is GIP. Included in this group of hormones secreted from the A
cell is glucagon and CCK (cholecystokinin). EC cells secrete motilin, substance P, and
secretin. D cells secrete somatostatin and D-1 cells secrete VIP (vasoactive intestinal
peptide). F cells secrete pancreatic polypeptide.
Question:
Using immunocytochemical staining, cell types within the pancreatic islets have been distinguished.
Cells classified as D cells secrete

Answer Choices:
A Pancreatic polypeptide (PP)
B Somatostatin
C Glucagon
D Vasoactive intestinal peptide (VIP)
E Insulin

B Somatostatin

Specific cells within the pancreatic islet have been differentiated using
immunocytochemical stains specific to various hormones. Somatostatin is secreted by
D cells and VIP is secreted by cells classified as D-1 cells. The F cell secretes
pancreatic polypeptide; glucagon is secreted by A cells. Insulin is secreted by B cells.
Question:
Using immunocytochemical staining, cell types within the pancreatic islets have been distinguished.
Cells classified as B cells secrete

Answer Choices:
A Pancreatic polypeptide (PP)
B Somatostatin
C Glucagon
D Vasoactive intestinal peptide (VIP)
E Insulin

E Insulin

Specific cells within the pancreatic islet have been differentiated using
immunocytochemical stains specific to various hormones. Somatostatin is secreted by
D cells, and VIP is secreted by cells classified as D-1 cells. The F cell secretes
pancreatic polypeptide; glucagon is secreted by A cells. Insulin is secreted by B cells.

Question:
Using immunocytochemical staining, cell types within the pancreatic islets have been distinguished.
Cells classified as F cells secrete

Answer Choices:
A Pancreatic polypeptide (PP)
B Somatostatin
C Glucagon
D Vasoactive intestinal peptide (VIP)
E Secretin

A Pancreatic polypeptide (PP)

Specific cells within the pancreatic islet have been differentiated using
immunocytochemical stains specific to various hormones. VIP is secreted by cells
classified as D-1 cells. The F cell secretes pancreatic polypeptide; glucagon is secreted
by A cells, secretin is secreted by EC (enterochromaffin) cells, and the D cells secrete
somatostatin.
Question:
Using immunocytochemical staining, cell types within the pancreatic islets have been distinguished.
Cells classified as D-1 cells secrete

Answer Choices:
A Insulin
B Somatostatin
C Glucagon
D VIP
E Secretin

D VIP

Specific cells within the pancreatic islet have been differentiated using
immunocytochemical stains specific to various hormones. VIP is secreted by cells
classified as D-1 cells. Insulin is secreted by B cells, glucagon is secreted by A cells.
Secretin is secreted by EC (enterochromaffin) cells, and the D cells secrete
somatostatin.
Question:
Which of the following histological (or cytological) techniques is best suited to ascertain various cell-
types inside the islets of Langerhans?

Answer Choices:
A Routine H&E technique
B Immunocytological technique
C Radioautography
D In situ hybridization technique

B Immunocytological technique

Whereas all of the above mentioned methods will reveal some aspect of the
morphological differences of some cell-types, only immunocytochemical technique
(antibody-antigen reaction) will reveal the nature of the secretion (cytochemistry) of all
cell-types. Some morphological differences, such as light versus darkly stained
cytoplasm, type of granules and the nature of granules, between A and B cells can be
shown by staining techniques.
Question:
Which one of the following cell-types in the islets of Langerhans produces somatostatin?

Answer Choices:
A A-cells
B B-cells
C C-cells
D D-cells

D D-cells

D, (or delta) cells, which are the largest and make up about 5% of the islet tissue, are
known to secrete somatostatin. This hormone inhibits the release of human growth
hormone.
A, (or alpha) cells, secrete glucagon, B-cells secrete insulin, and C-cells (found only in
certain species, such as guinea pigs) may secrete the so-called pancreatic factor.
The function of the pancreatic factor is not known.

Question:
Using immunocytochemical staining, cell types within the pancreatic islets have been distinguished.
Cells which secrete somatostatin are

Answer Choices:
A A cells
B EC cells
C D cells
D D-1 cells
E F cells
C D cells

Specific cells within the pancreatic islet have been differentiated using
immunocytochemical stains specific to various hormones. A cells secrete a variety of
hormones, one of which is GIP. Included in this group from the A cell is glucagon and
CCK (cholecystokinin). EC cells secrete motilin, substance P, and secretin. D cells
secrete somatostatin and D-1 cells secrete VIP (vasoactive intestinal peptide). F cells
secrete pancreatic polypeptide.

Question:
Which type of cells are generally most centrally located in the pancreatic islet?

Answer Choices:
A A cells
B B cells
C D cells
D D-1 cells
E F cells
B B cells

The B cells are generally positioned most centrally within any one pancreatic islet. The
A and D cells generally are more peripherally located.
Question:
Sympathetic stimulation of nerves innervating the islets of the pancreas will

Answer Choices:
A Have no effect
B Increase secretion of insulin from B cells
C Decrease secretion of insulin from B cells
D Increase secretion of glucagon from A cells
E Decrease secretion of glucagon from A cells

D Increase secretion of glucagon from A cells

The pancreatic islets receive innervation from both the sympathetic and
parasympathetic nervous system. Stimulation of the parasympathetic system increases
secretion of insulin from the B cells. Stimulation of the sympathetic system increases
glucagon secretion from the A cells. Insulin is inhibited by sympathetic stimulation.
Question:
A 16-year old female tells you that she feels faint. Symptoms include continuous hunger, thirst, and
excessive urination. The probable diagnosis is

Answer Choices:
A Diabetes insipidus
B Icterus
C Type I diabetes mellitus
D Type II diabetes mellitus
E Viral hepatitis B
C Type I diabetes mellitus

The three characteristic signs of type I (insulin-dependent) diabetes mellitus (juvenile-


onset) are polyphagia (excessive eating), polydipsia (frequent drinking), and polyuria
(frequent urination). The condition occurs in young individuals, usually before they reach
the age of 20.
Diabetes insipidus is marked by dehydration, excessive thirst (polydipsia), and excretion
of high volumes of dilute urine. However, there is no polyphagia (excessive eating).
Icterus (Jaundice) is characterized by excessive bilirubin in the blood and deposition of
bile pigment in the skin and sclera of the eye, resulting in yellowish appearance of the
skin and the eye.
Type II (non-insulin-dependent) diabetes mellitus does not result from low levels of
plasma insulin and is insulin resistant, which is a major factor in its pathogenesis. It
commonly occurs in overweight individuals over 40 years of age. It is usually controlled
by diet.
Viral hepatitis B (serum hepatitis) may cause cirrhosis, jaundice, and death.

Question:
Using immunocytochemical staining, cell types within the pancreatic islets have been distinguished.
Cells which secrete insulin are

Answer Choices:
A A cells
B B cells
C D cells
D D-1 cells
E F cells

B B cells

Specific cells within the pancreatic islet have been differentiated using
immunocytochemical stains specific to various hormones. A cells secrete a variety of
hormones, one of which is GIP. Included in this group from the A cell is glucagon and
CCK (cholecystokinin). B cells secrete insulin. D cells secrete somatostatin, and D-1
cells secrete VIP (vasoactive intestinal peptide). F cells secrete pancreatic polypeptide.

Question:
In the human, blood perfuse the pancreatic islets from a peripheral to central direction. Which type of
cells would generally be the first to be perfused by blood?

Answer Choices:
A A and D cells
B A and B cells
C B and D cells
D A and D-1 cells
E There is no topographic localization of cell types in the pancreas, so local
blood flow through the islet would not preferentially perfuse any cell type

A A and D cells

Because of the topographic organization of the cells within the islets, some cell types
are perfused prior to the perfusion of other cell types. Since the human blood perfuses
the pancreatic islets from a peripheral to central direction, the A and D cells, which are
generally more peripherally located, are the first to be perfused. The B cells, being
generally positioned most centrally within any one pancreatic islet, are perfused with
blood which has previously perfused the A and D cells.
Question:
Fasting levels of blood glucose were measured in a medical student 30 minutes prior to taking Step 1
of the USMLE exam. He had no history of diabetes. Glucose levels were found to be 120mg/dL
(normal = 70 mg/dL). A logical explanation for the high blood glucose levels is:

Answer Choices:
A The student is stressed and sympathetic innervation of the pancreatic islets
have increased glucagon secretion and inhibited insulin release
B The student is stressed and parasympathetic innervation of the pancreatic
islets have increased glucagon secretion and inhibited insulin release
C The student is stressed, which has no effect on glucagon or insulin secretion
D The student is stressed and sympathetic innervation of the pancreatic islets
have decreased glucagon secretion and stimulated insulin release
E The student is stressed and parasympathetic innervation of the pancreatic
islets have decreased glucagon secretion and stimulated insulin release

A The student is stressed and sympathetic innervation of the pancreatic islets have
increased glucagon secretion and inhibited insulin release

The pancreatic islets receive innervation from both the sympathetic and
parasympathetic nervous system. Stimulation of the parasympathetic system increases
secretion of insulin from the B cells. Stimulation of the sympathetic system, such as
during periods of stress, increases glucagon secretion from the A cells and inhibits
insulin release.

SECTION- RESPIRATORY SYSTEM


Question: CLASS QUIZ
Openings in the inter-alveolar septa allow circulation of air from one alveolus to another. These
openings are referred to as

Answer Choices:
A Alveolar sacs
B Alveolar pores
C Alveolar septa
D Terminal bronchioles
E Respiratory bronchioles
B Alveolar pores

Alveolar pores are openings in the alveolar septa, essentially connecting adjacent
alveoli. These pores allow circulation of air from one alveolus to another.

Question:CLASS QUIZ
Gasses must diffuse through which of the following layers in the thinnest air-blood barrier?

Answer Choices:
A Surfactant, Type I pneumocyte, fused basal lamina, capillary endothelial cell
B Surfactant, Type II pneumocyte, fused basal lamina, capillary endothelial cell
C Surfactant, Type I pneumocyte its basal lamina, the basal lamina of the
capillary endothelial cell, capillary endothelial cell
D Surfactant, Type I pneumocyte, capillary endothelial cell
E Surfactant, Type II pneumocyte, capillary endothelial cell
A Surfactant, Type I pneumocyte, fused basal lamina, capillary endothelial cell

The thinnest air-blood barrier listed above consists of the monomolecular layer of
surfactant, the cell membranes and attenuated cytoplasm of the type I pneumocyte, the
fused basal laminae of the type I pneumocyte and endothelial capillary.

Question:
Which type of cell lines most of the surface of the alveoli and comprises a portion of the air-blood
barrier?

Answer Choices:
A Type I pneumocytes
B Type II pneumocytes
C Dust cells
D Brush cells
E Clara cells

A Type I pneumocytes

Type II pneumocytes, also called septal cells, are secretory cells with a cuboidal shape
located in the alveolar epithelium. These cells secrete surfactant, a surface active agent
that lowers the surface tension at the air-epithelium interface within the alveoli.
Type I pneumocytes are attenuated squamous cells which make up 95% of the surface
of the alveoli. These cells make up a portion of the air-blood barrier.
Dust cells are alveolar macrophages that scavenge the surface of alveoli.
Clara cells secrete a lipoprote that is a surface-active agent.
Brush cells, also found lining the alveolar wall, may serve as receptors within the lung.
Question:
Surfactant is secreted by which type of cell in the respiratory alveoli?

Answer Choices:
A Type I pneumocytes
B Type II pneumocytes
C Dust cells
D Brush cells
E Clara cells

B Type II pneumocytes

Type II pneumocytes, also called septal cells, are secretory cells with a cuboidal shape
located in the alveolar epithelium. These cells secrete surfactant, a surface active agent
that lowers the surface tension at the air-epithelium interface within the alveoli.
Type I pneumocytes are attenuated squamous cells which make up 95% of the surface
of the alveoli. These cells make up a portion of the air-blood barrier.
Dust cells are alveolar macrophages that scavenge the surface of alveoli.
Clara cells secrete a lipoprote that is a surface-active agent.
Brush cells, also found lining the alveolar wall, may serve as receptors within the lung.
Question:
Which one of the following cells secretes surface active agents in the respiratory bronchioles?

Answer Choices:
A Type I pneumocytes
B Capillary endothelial cells
C Clara cells
D Dust cells
E Fibroblasts

C Clara cells

The terminal bronchioles are lined with a simple cuboidal epithelium. Some of the cells
are ciliated, some are non-ciliated.
The non-ciliated cells become more predominant within the respiratory bronchioles.
Some of them have a dome-shaped appearance and have cytoplasmic characteristics
of secretory cells. These cells are Clara cells, which secrete a lipoprotein that is a
surface-active agent.
Type I pneumocytes are attenuated squamous epithelial cells that line the respiratory
alveoli.
Dust cells are alveolar macrophages that scavenge the surface of alveoli.
Question:
Air flow may be greatly reduced by what action within the smaller bronchioles?
Answer Choices:
A Extensive elongation of cilia on the respiratory epithelium
B Increase of mucus secretion within the bronchioles
C Decrease of luminal diameter of the bronchiole by smooth muscle contraction
D Collapse of bronchiolar lumen due to reduced surfactant secretion by septal
cells
E Enhanced thickness of the mucosa limiting luminal diameter
C Decrease of luminal diameter of the bronchiole by smooth muscle contraction

Bronchi become reduced in size and the cartilaginous plates gradually become smaller
and fewer in number, and ultimately disappear when the bronchi reach about 1mm in
diameter. These passages lacking cartilage are referred to as bronchioles. However,
with the disappearance of cartilage, smooth muscle is added such that within the
bronchioles, smooth muscle forms a conspicuous interlacing, continuous layer. This
layer of muscle becomes spirally arranged in the smaller bronchioles, but represents a
major portion of the wall of the bronchioles. Air flow may be greatly reduced by
contraction of the smooth muscle of the bronchiolar wall. The muscle layer is located
between the mucosa and submucosa, as a muscularis. Mucus secreting glands are not
present in the smaller bronchioles.
Question:
A patient undergoes surgical resection of a bronchopulmonary segment. Which one of the following
best describes a bronchopulmonary segment?

Answer Choices:
A A bronchus and the lung parenchyma which it supplies
B One of the two lobes of the left lung
C A secondary (lobar) bronchus and the lung parenchyma which it supplies
D A tertiary bronchus and the lung parenchyma which it supplies
E One of the three lobes of the right lung

D A tertiary bronchus and the lung parenchyma which it supplies

The trachea divides first into two branches and eventually into secondary and tertiary
branches. The segments supplied by the tertiary bronchi each possess their own blood
supply and are contained within their own connective tissue septa forming a segment of
lung parenchyma that lends itself to surgical resection when needed. These
bronchopulmonary segments are defined by a tertiary (segmental) bronchus and the
lung parenchyma that it supplies.
Question:
The tracheal cartilage rings may be partially replaced by another tissue during the aging process.
Which one of the following tissues may, in part, replace the hyaline cartilage of the tracheal cartilage
rings?

Answer Choices:
A Dense regularly arranged connective tissue
B Loose connective tissue
C Elastic cartilage
D Bone
E Dense, irregularly arranged connective tissue

D Bone

The cartilage rings of the trachea may be partially replaced by bone during the normal
process of aging. This results in the loss of some of the flexibility of the trachea in aged
individuals.

Question:
The epithelium covering the surface of the lung, which also is known as the visceral pleura, is best
described as

Answer Choices:
A Simple columnar
B Simple cuboidal
C Simple squamous
D Stratified squamous
E Transitional

C Simple squamous

The visceral pleura covers the surface of the lung as a simple squamous epithelium.
This membrane is continuous with the parietal pleura.

Question:
The larger blood vessels that supply the layers of the tracheal wall are located in the:

Answer Choices:
A Submucosa
B Cartilaginous layer
C Adventitia
D Mucosa
E Esophageal gland tissue

C Adventitia

The adventitia contains the larger blood vessels that supply the other layers of the
tracheal wall, as well as the larger lymph vessels and the nerve bundle fibers that
innervate the tracheal wall. This layer lies outside or peripheral to the cartilaginous layer
and trachealis muscle. As such, it serves to bind the trachea to adjacent structures in
the neck and mediastinum.

Question:
What is the primary function of the "false vocal cords" or ventricular folds of the larynx and the
laryngeal ventricles?

Answer Choices:
A Prevent food from entering the larynx
B Creation of sound resonance
C Vibrate to produce sound
D Equalize atmospheric pressure between the oropharynx and trachea
E Serve as a site for the pharyngeal tonsils

B Creation of sound resonance

The false vocal cords, also referred to as the ventricular folds, are located above the
"true" vocal cords. The ventricular folds have no intrinsic muscles and do not participate
in the production of sound by vibration. However, because of their proximity to the
ventricles, which are outpocketings of the wall of the larynx between the vocal cords
and the ventricular folds, they serve to help create sound resonance. The pharyngeal
tonsils are not located in the ventricular recess.

Question:
The vocal cords are comprised of

Answer Choices:
A Skeletal muscle core and a ligament covered by ciliated, pseudostratified
columnar epithelium
B Smooth muscle core covered by stratified squamous epithelium
C Smooth muscle core and a ligament covered by ciliated, pseudostratified
columnar epithelium
D Connective tissue core and skeletal muscle covered by stratified squamous
epithelium
E Skeletal muscle core and a ligament covered by stratified squamous
epithelium
E Skeletal muscle core and a ligament covered by stratified squamous epithelium

The vocal cords are paired folds that project into the lumen of the larynx which, when
stretched, control the flow of air through the larynx and vibrate to produce sound and
subserve speech. As such, these mucosal folds consist of a core of skeletal muscle and
a ligament covered by stratified squamous epithelium. Ciliated, pseudostratified
columnar epithelium is commonly found lining the remaining respiratory passages of the
larynx and trachea.

Question:
The structure that serves as a passageway for air and food and provides a resonating chamber for
speech is the

Answer Choices:
A Pharynx
B Larynx
C Nasopharynx
D Oropharynx
E Glottis

A Pharynx

The pharynx serves as an open area that connects both nasal and oral cavities to the
larynx and is located posteriorly to the nasal and oral cavities. As such, the pharynx
serves as a passageway for air and food and provides a resonating chamber for
speech. The pharynx may be subdivided into two regions, one of which is the
passageway for air, the nasopharynx, and the other region is a passageway for food
and air, the oropharynx. The oropharynx opens into the larynx, which then becomes
continuous with the trachea. The glottis is the opening into the larynx
Question:
Which one of the following cell types serves as stem cells for replacement of the tracheal respiratory
epithelium?

Answer Choices:
A Ciliated cells
B Brush cells
C Goblet cells
D Granule cells
E Basal cells

E Basal cells
The respiratory epithelium is classified as a ciliated, pseudostratified columnar
epithelium which is composed of several cell types. The most numerous are the ciliated
cells from which apical cilia project into the lumen. Intermittent throughout the
respiratory epithelium are goblet cells, also referred to as mucous cells. These cells
accumulate mucinogen granules and provide a mucous secretion.
Other columnar-shaped cells bear blunt microvilli on their apical surface and are
referred to as brush cells. The basal surface of the brush cells are in synaptic contact
with a sensory nerve ending that subserve general sensation to the epithelium. Granule
cells are scattered sparsely throughout the epithelium and belong to the general class of
enteroendocrine cells. Basal cells serve as the stem cells for maintaining the cells in the
epithelium. Their nuclei reside close to the basement membrane in histological section.

Question:
General sensation to the respiratory epithelium in the trachea is subserved by which cell?

Answer Choices:
A Ciliated cells
B Brush cells
C Goblet cells
D Granule cells
E Basal cells

B Brush cells

The respiratory epithelium is classified as a ciliated, pseudostratified columnar


epithelium which is composed of several cell types. The most numerous are the ciliated
cells from which apical cilia project into the lumen. Intermittent throughout the
respiratory epithelium are goblet cells, also referred to as mucous cells. These cells
accumulate mucinogen granules and provide a mucous secretion.
Other columnar-shaped cells bear blunt microvilli on their apical surface and are
referred to as brush cells. The basal surface of the brush cells are in synaptic contact
with a sensory nerve ending that subserve general sensation to the epithelium. Granule
cells are scattered sparsely throughout the epithelium and belong to the general class of
enteroendocrine cells. Basal cells serve as the stem cells for maintaining the cells in the
epithelium. Their nuclei reside close to the basement membrane in histological section.
Question:
The epithelium covering the surface of the larynx, except for the vocal cords, is

Answer Choices:
A Pseudostratified columnar epithelium
B Simple columnar epithelium
C Stratified cuboidal epithelium
D Stratified squamous epithelium
E Simple cuboidal epithelium

A Pseudostratified columnar epithelium

Most of the luminal surface of the vocal cords is covered by a stratified squamous
epithelium. Because of the air stream passing over the vocal cords, which induces them
to vibrate, the stratified squamous epithelium serves to protect the mucosa from
abrasion. Most of the remaining larynx is covered with ciliated, pseudostratified
columnar epithelium.

Question:
The epithelium covering the luminal surface of the vocal cords is

Answer Choices:
A Pseudostratified columnar epithelium
B Simple columnar epithelium
C Stratified cuboidal epithelium
D Stratified squamous epithelium
E Simple cuboidal epithelium

D Stratified squamous epithelium

Most of the luminal surface of the vocal cords is covered by a stratified squamous
epithelium. Because of the air stream passing over the vocal cords, which induces them
to vibrate, the stratified squamous epithelium serves to protect the mucosa from
abrasion. Most of the remaining larynx is covered with ciliated, pseudostratified
columnar epithelium.

Question:
General sensation is provided to olfactory epithelium through terminal branches of the trigeminal nerve
(cranial nerve V) that synapses on which cells?

Answer Choices:
A Olfactory cells
B Brush cells
C Granule cells
D Sustentacular cells
E Basal cells

B Brush cells
The olfactory epithelium contains olfactory cells which are bipolar neurons responsible
as the sensory cells of olfaction. These cells also possess an apical, dendritic process
from which a number of cilia arise.
The sustentacular cells are columnar cells that provide structural support to the olfactory
cells.
Brush cells are present in the olfactory epithelium and provide blunt-shaped microvilli on
the surface of the epithelium. The basal surface of the brush cells have synaptic contact
with terminal branches of the trigeminal nerve, which provide general sensation to the
olfactory epithelium.
The basal cells, which are present, are regarded as the stem cells of the olfactory
epithelium.
Question:
A patient presented having had a short-lasting accidental exposure to concentrated ammonia gas a
day earlier. He complained of loss of the sense of smell. He had not inhaled the gas beyond the nasal
cavity, but a biopsy revealed extensive damage to the olfactory cells and nearly complete loss of the
apical cilia. Some of the other cells remained, however, including the basal cells. Approximately how
long will it be before some olfaction returns?

Answer Choices:
A 1 day
B 1 month
C 1 year
D 10 years
E Olfaction will probably never return to the individual

B 1 month

Unlike many other neurons, the olfactory cells are somewhat plastic and capable of
replacing themselves postnatally. Olfaction may be disturbed by many causes, one of
which is insult by noxious gases, such as ammonia gas. In this case, since the biopsy
revealed that the apical cilia were lost but the basal cells remained, there is reason to
believe that olfaction will be restored. The basal cells are the stem cells for olfactory
cells and sustentacular cells, and the normal life span of the olfactory cells is about 1
month. Therefore, it is reasonable to believe that the individual's sense of smell may
begin to return within approximately 1 month.
Question:
The olfactory epithelium resembles the respiratory epithelium but lacks which one of the following cell
types?

Answer Choices:
A Goblet cells
B Basal cells
C Brush cells
D Sustentacular cells
E Olfactory cells
A Goblet cells

The olfactory epithelium resembles the respiratory epithelium lining the respiratory
passages. While the olfactory epithelium is pseudostratified columnar epithelium, the
cells are different from that of the respiratory epithelium. The olfactory epithelium
contains olfactory cells which are bipolar neurons responsible as the sensory cells of
olfaction. The sustentacular cells are columnar cells that provide structural support to
the olfactory cells. Brush cells are present in the olfactory epithelium and provide blunt-
shaped microvilli on the surface of the epithelium. The basal cells, which are present,
are regarded as the stem cells of the epithelium. The olfactory epithelium lacks goblet
cells, which are common to the respiratory epithelium.
Question:
Which one of the following features of the respiratory system is considered to be a part of the
"conducting" portion?

Answer Choices:
A Alveoli
B Alveolar ducts
C Respiratory bronchioles
D Alveolar sacs
E Terminal bronchioles

E Terminal bronchioles

The air passages of the respiratory system can be divided into one of two classifications
- a conducting portion or a respiratory portion.
Those passages associated with transport of gases to sites within the lung where gas
exchange occurs are regarded as conducting portions. The conducting portion of the
respiratory system includes the nasal cavities, nasopharynx and oropharynx, the larynx,
trachea, the paired primary bronchi, secondary (distributing) bronchioles, and terminal
bronchioles.
The portion of the respiratory system associated with gas exchange is the respiratory
portion. The respiratory portion includes the respiratory bronchioles, alveolar ducts,
alveolar sacs, and alveoli.
The terminal bronchiole is the "terminal" part of the conducting portion of the respiratory
system. Bronchioles that possess alveolar sacs are regarded as respiratory bronchioles,
and since gaseous transfer occurs through the intermittently located alveolar sacs along
the bronchiole, the respiratory bronchiole is the first part of the respiratory portion of the
respiratory system.
Question:
Which one of the following features of the respiratory system is considered to be the initial part of the
"respiratory" portion?

Answer Choices:
A Larynx
B Trachea
C Bronchus
D Terminal bronchiole
E Respiratory bronchioles

E Respiratory bronchioles

The air passages of the respiratory system can be divided into one of two classifications
- a conducting portion or a respiratory portion.
Those passages associated with transport of gases to sites within the lung where gas
exchange occurs are regarded as conducting portions. The conducting portion of the
respiratory system includes the nasal cavities, nasopharynx and oropharynx, the larynx,
trachea, the paired primary bronchi, secondary (distributing) bronchioles, and terminal
bronchioles.
The portion of the respiratory system associated with gas exchange is the respiratory
portion. The respiratory portion includes the respiratory bronchioles, alveolar ducts,
alveolar sacs, and alveoli.
The terminal bronchiole is the "terminal" part of the conducting portion of the respiratory
system. Bronchioles that possess alveolar sacs are regarded as respiratory bronchioles,
and since gaseous transfer occurs through the intermittently located alveolar sacs along
the bronchiole, the respiratory bronchiole is the first part of the respiratory portion of the
respiratory system.
Question:
Swell bodies

Answer Choices:
A Are three bony shelf-like projections in each lateral wall of the nasal cavernous
chambers
B Are the specialized tissues that cover the superior conchae
C Are the large venous plexus within the lamina propria of the nasal conchae
D Are blind cavities in the frontal, maxillary, ethmoid, and sphenoid bones
E Project from the rim of the larynx, extend into the pharynx, and have both a
lingual and a laryngeal surface

C Are the large venous plexus within the lamina propria of the nasal conchae

Within the lamina propria of the conchae are large venous plexus known as swell
bodies. Every 20-30 minutes, the swell bodies on each side of the nasal fossae become
engorged with blood, resulting in distention of the conchal mucosa and a concomitant
decrease in the flow of air.
The three bony shelf-like projections in each lateral wall of the nasal cavernous
chambers are conchae. The specialized tissue that covers the superior conchae is the
olfactory epithelium. The paranasal sinuses are blind cavities in the frontal, maxillary,
ethmoid, and sphenoid bones. The epiglottis project from the rim of the larynx, extends
into the pharynx, and has both a lingual and a laryngeal surface.

Question:
What does the asterisk identify, as located in a specific area in the attached photomicroscopic image?

Answer Choices:
A Primary bronchiole
B Terminal bronchiole
C Bronchus
D Alveolar duct
E Alveolar sac

Image(s) / Chart(s):
Click image to view full size. Click open image to close. Click and hold open image to move.

B Terminal bronchiole

The asterisk is located within the terminal bronchiole. Note that in this image, a terminal
bronchiole is depicted as cut in longitudinal section in which the conducting portion of
the respiratory system lined by simple cuboidal epithelium abruptly becomes simple
squamous and forms a passage-way in which respiratory alveoli begin to branch off of
the duct. The "terminal bronchiole" is the final part of the conducting portion of the
respiratory system. The process of "respiration" begins as oxygen transfer occurs in the
"respiratory bronchiole," where alveoli may be seen and the epithelium becomes
attenuated to simple squamous.

Question:
What organ is depicted in the attached photomicroscopic image?

Answer Choices:
A Lung
B Medullary kidney
C Adipose tissue
D Breast
E Seminal vesicle

Image(s) / Chart(s):
Click image to view full size. Click open image to close. Click and hold open image to move.

A Lung

Histologically, the lung appears as closely packed "air sacs" or alveoli lined by a simple
squamous epithelium, or alveolar walls. Note that in this image, a terminal bronchiole is
depicted as cut in longitudinal section in which the conducting portion of the respiratory
system lined by simple cuboidal epithelium abruptly becomes simple squamous and
forms a passage-way in which respiratory alveoli begin to branch off of the passage.
The "terminal bronchiole" is the final part of the conducting portion of the respiratory
system. The process of "respiration" begins as oxygen transfer and occurs in the
"respiratory bronchiole" where alveoli may be seen and the epithelium becomes
attenuated to simple squamous.
Question:
The epithelium as depicted in the photomicrographic image is consistent with the type of epithelium at
which location?
Answer Choices:
A Esophagus
B Ureter
C Aorta
D Trachea
E Stomach

Image(s) / Chart(s):
Click image to view full size. Click open image to close. Click and hold open image to move.

D Trachea

The photomicrograph depicts a pseudostratified columnar epithelium, possessing cilia


and containing goblet cells. While this epithelium may be found in several locations
throughout the body, the only place it may be found from the choices given in the
question is the trachea. The pseudostratified columnar epithelium with cilia and goblet
cells is typical of the epithelium located in conducting portions of the respiratory
passages.
Question:
The trachea possesses which one of the following components?

Answer Choices:
A Irregular cartilage plates in its wall
B Skeletal muscle in its wall
C An epithelium containing only 3 cell types
D A thick basement membrane underlying its epithelium
E A ciliated stratified columnar epithelium

D A thick basement membrane underlying its epithelium


Image(s) / Chart(s):
Click image to view full size. Click open image to close. Click and hold open image to
move.
The pseudostratified ciliated columnar epithelium lining the trachea rests upon a thick
basement membrane. Tracheal epithelium contains 5 cell types (ciliated, mature goblet,
small mucous granule, diffused endocrine, and short basal cells). The trachea
possesses C-shaped cartilages with smooth muscle (the trachealis) extending between
their ends.
Cross-section through trachea (a) ciliated pseudostratified columnar epithelium; (b)
cartilage; (c) trachealis.
Question:
Which one of the following features of the respiratory system is considered to be the final part of the
"conducting" portion?

Answer Choices:
A Larynx
B Trachea
C Bronchus
D Terminal bronchiole
E Respiratory bronchioles

D Terminal bronchiole

The air passages of the respiratory system can be divided into one of two classifications
- a conducting portion or a respiratory portion.
Those passages associated with transport of gases to sites within the lung where gas
exchange occurs are regarded as conducting portions. The conducting portion of the
respiratory system includes the nasal cavities, nasopharynx and oropharynx, the larynx,
trachea, the paired primary bronchi, secondary (distributing) bronchioles, and terminal
bronchioles.
The portion of the respiratory system associated with gas exchange is the respiratory
portion. The respiratory portion includes the respiratory bronchioles, alveolar ducts,
alveolar sacs, and alveoli.
The terminal bronchiole is the "terminal" part of the conducting portion of the respiratory
system. Bronchioles that possess alveolar sacs are regarded as respiratory bronchioles,
and since gaseous transfer occurs through the intermittently located alveolar sacs along
the bronchiole, the respiratory bronchiole is the first part of the respiratory portion of the
respiratory system.
Question:
At what point in the respiratory system does gaseous exchange first occur?

Answer Choices:
A Nasal cavity
B Trachea
C Secondary bronchiole
D Respiratory bronchiole
E Terminal bronchiole
D Respiratory bronchiole

The air passages of the respiratory system can be divided into one of two classifications
- a conducting portion or a respiratory portion.
Those passages associated with transport of gases to sites within the lung where gas
exchange occurs are regarded as conducting portions. The conducting portion of the
respiratory system includes the nasal cavities, nasopharynx and oropharynx, the larynx,
trachea, the paired primary bronchi, secondary (distributing) bronchioles, and terminal
bronchioles.
The portion of the respiratory system associated with gas exchange is the respiratory
portion. The respiratory portion includes the respiratory bronchioles, alveolar ducts,
alveolar sacs, and alveoli.
The terminal bronchiole is the "terminal" part of the conducting portion of the respiratory
system. Bronchioles that possess alveolar sacs are regarded as respiratory bronchioles,
and since gaseous transfer occurs through the intermittently located alveolar sacs along
the bronchiole, the respiratory bronchiole is the first part of the respiratory portion of the
respiratory system.
Question:
A 45-year-old individual with a 20 pack-year history of smoking presents because of a chronic cough.
The cough has been worsening for the past 3 months. The patient does not have a primary care
physician and usually visits the ER for exacerbations of his cough. What type of epithelium would be
anticipated lining the larynx in this individual?

Answer Choices:
A Ciliated, pseudostratified columnar
B Simple columnar
C Simple cuboidal
D Transitional
E Stratified squamous
E Stratified squamous

Normally most of the larynx is covered with ciliated, pseudostratified columnar


epithelium and the luminal surface of the vocal cords is covered by a stratified
squamous epithelium. Because of the air stream passing over the vocal cords, which
induces them to vibrate, the stratified squamous epithelium serves to protect the
mucosa from abrasion. Epithelial alterations also may occur with chronic, forceful
movement of air through the larynx, such as during chronic coughing.
In this case, noxious elements contained within the smoke inhibit the movement of the
cilia of the respiratory epithelium, resulting in loss of the ability to remove mucous
containing inhaled particulate matter. Coughing removes the mucous and debris, which
over time results in the replacement of the affected areas with stratified squamous
epithelium due to the sustained alteration in forceful air flow through the larynx.
Question:
The nerve fiber bundles that innervate the tracheal wall are located in which one of the following
layers?

Answer Choices:
A Submucosa
B Cartilaginous layer
C Adventitia
D Mucosa
E Trachealis muscle
C Adventitia

The adventitia contains the larger blood vessels that supply the other layers of the
tracheal wall, as well as the larger lymph vessels and the nerve bundle fibers that
innervate the tracheal wall. This layer lies outside or peripheral to the cartilaginous layer
and trachealis muscle. As such, it serves to bind the trachea to adjacent structures in
the neck and mediastinum.
Question:
The trachealis muscle found in the trachea is located between which two layers of the trachea?

Answer Choices:
A Submucosa and adventitia
B Mucosa and submucosa
C Submucosa and cartilaginous layer
D Cartilaginous layer and adventitia
E Mucosa and the cartilaginous layer

A Submucosa and adventitia

Four layers of tissue are characteristic of the trachea.


The innermost layer, the mucosa, is characterized by the respiratory epithelium lining
the lumen of the trachea, resting upon a prominent basement membrane. A lamina
propria consisting of relatively loose connective tissue underlies the epithelium, which
also is part of the mucosa. Separating the mucosa from the submucosa is an indistinct
band of elastic fibers.
The submucosa consists of relatively loose connective tissue, similar to that of the
lamina propria, which makes it difficult to discern the submucosa from the lamina
propria. However, in some areas of the trachea, the submucosa may contain glands
composed of mucus-secreting acini.
The next outermost layer of the trachea is the cartilage layer consisting of the C-shaped
hyaline cartilages, open at their most posterior aspect. The trachealis muscle is located
in this posterior gap such that the tracheal cartilages and the trachealis muscle separate
the submucosa from the adventitia.
The outermost layer, the adventitia, lies peripheral to the cartilage rings and the
trachealis muscle. The irregularly arranged connective tissue of the adventitia binds the
trachea to adjacent structures, such as the esophagus.
Question:
Which of the following statements concerning the terminal bronchioles is true?

Answer Choices:
A They are part of the conducting portion of the respiratory system
B They function in gas exchange
C They do not contain ciliated cells
D They have cartilage plates present in their walls
E They are lined by stratified squamous epithelium that contains goblet cells

A They are part of the conducting portion of the respiratory system

Terminal bronchioles are the most distal portion of the conducting portion of the
respiratory system. They lack alveoli and, as a consequence, do not participate in the
exchange of gases. They are lined by a simple cuboidal epithelium that contains mostly
Clara cells and some ciliated cells, but no goblet cells.
Question:
Which one of the following features of the respiratory system is considered to be a part of the
"respiratory" portion?

Answer Choices:
A Alveolar duct
B Trachea
C Larynx
D Terminal bronchiole
E Nasal cavity
A Alveolar duct

The air passages of the respiratory system can be divided into one of two classifications
- a conducting portion or a respiratory portion.
Those passages associated with transport of gases to sites within the lung where gas
exchange occurs are regarded as conducting portions. The conducting portion of the
respiratory system includes the nasal cavities, nasopharynx and oropharynx, the larynx,
trachea, the paired primary bronchi, secondary (distributing) bronchioles, and terminal
bronchioles.
The portion of the respiratory system associated with gas exchange is the respiratory
portion. The respiratory portion includes the respiratory bronchioles, alveolar ducts,
alveolar sacs, and alveoli.
The terminal bronchiole is the "terminal" part of the conducting portion of the respiratory
system. Bronchioles that possess alveolar sacs are regarded as respiratory bronchioles,
and since gaseous transfer occurs through the intermittently located alveolar sacs along
the bronchiole, the respiratory bronchiole is the first part of the respiratory portion of the
respiratory system.
Question:
Removal of inhaled particulate matter is the function of which one of the following cells?

Answer Choices:
A Type I pneumocytes
B Type II pneumocytes
C Dust cells
D Brush cells
E Clara cells

C Dust cells

Type II pneumocytes, also called septal cells, are secretory cells with a cuboidal shape
located in the alveolar epithelium. These cells secrete surfactant, a surface active agent
that lowers the surface tension at the air-epithelium interface within the alveoli.
Type I pneumocytes are attenuated squamous cells which make up 95% of the surface
of the alveoli. These cells make up a portion of the air-blood barrier.
Dust cells are alveolar macrophages that scavenge the surface of alveoli.
Clara cells secrete a lipoprote that is a surface-active agent.
Brush cells, also found lining the alveolar wall, may serve as receptors within the lung.
Question:
Which of the following factors reduces the net flow of fluid out of a pulmonary capillary?
Answer Choices:
A Increased interstitial fluid colloid osmotic pressure
B Increased reflection coefficient
C Decreased plasma colloid osmotic pressure
D Increased capillary hydrostatic pressure
E Decreased interstitial hydrostatic pressure
B Increased reflection coefficient

Capillary blood pressure, although relatively low, is sufficient to force fluid and dissolved
substances through capillary pores into the surrounding tissue spaces. The presence of
plasma proteins such as albumin, however, creates a colloid osmotic pressure, or
oncotic pressure, that causes fluid movement from the interstitial spaces back into the
capillaries. In this way, excessive depletion of plasma volume is prevented. Lymphatic
capillaries recover the small amount of fluid and plasma that is not recovered by the
microcirculation.
Fluid movement through the capillary membrane is determined by the 4 primary Starling
forces and is summarized by the following relationship:
Q = K[(Pc - Pi) - σ(πc - πi)]
where:
Q = filtration rate.
K = filtration coefficient is an expression of the net imbalance of forces at the capillary
membrane.
σ = protein reflection coefficient is an expression of the resistance of the capillary
membrane to protein leakage. When protein molecules are unable to pass through the
pores between endothelial cells of capillaries, they are "reflected" from the pore and
cause osmotic pressure. When all of the protein molecules are reflected, σ = 1.0 and a
maximum osmotic effect is created; when all of the proteins pass through the pores to
be filtered, σ = 0.0 and no osmotic effect is produced.
Pc = capillary (hydrostatic) pressure forces fluid outward through the capillary
membrane. Capillary, or microvascular, pressure is created by systemic blood pressure.
Pi = interstitial fluid (hydrostatic) pressure forces fluid inward when Pi is positive, but
outward when Pi is negative. In loose connective tissue, the pumping action of the
lymphatic vessels scavenges fluid from the tissue spaces to create a negative, or
subatmospheric, interstitial fluid pressure.
πc = plasma colloid osmotic (oncotic) pressure causes inward movement of fluid from
the interstitial spaces to the vascular compartment. Only those substances unable to
pass through membrane pores exert osmotic pressure, and since proteins are normally
"reflected" from the capillary membrane pores, it is the plasma proteins in the vascular
compartment that generate an inward osmotic force at the capillaries.
πi = interstitial fluid colloid osmotic (oncotic) pressure is due to the presence of plasma
proteins in the tissue spaces. Proteins leak into the interstitial spaces through larger
capillary membrane pores and generate an outward osmotic force at the capillaries.
Due to the above 4 factors that influence capillary membrane dynamics, a relatively
large volume of fluid is filtered out of the arterial ends of capillaries and reabsorbed at
the venous ends of capillaries. The net outward force at the arterial ends of the
capillaries is called the net filtration pressure and is the result of the Starling forces
discussed above. (Note: Diffusion refers to the movement of fluid in both directions
across the capillary membrane; whereas, filtration refers to the net flow of fluid out of
the proximal ends of capillaries.) Upon entering the loose connective tissue of interstitial
spaces, the filtered fluid "flows" toward the venous ends of the capillaries. Here, the
summation of Starling forces produces a net inward force called the reabsorption
pressure. This reabsorption process is responsible for the recovery of approximately
90% of the filtered fluid, while lymphatic vessels recover approximately 10% of the
filtered fluid and dissolved substances and return them to general circulation.
Question:
Hyaline cartilage plates are replaced by what tissue in bronchioles?

Answer Choices:
A Regularly arranged dense connective tissue
B Loose connective tissue
C Skeletal muscle
D Smooth muscle
E Elastic cartilage

D Smooth muscle

Bronchi become reduced in size and the cartilaginous plates gradually become smaller
and fewer in number, and ultimately disappear when the bronchi reach about 1mm in
diameter. These passages lacking cartilage are referred to as bronchioles. However,
with the disappearance of cartilage, smooth muscle is added such that within the
bronchioles, smooth muscle forms a conspicuous interlacing, continuous layer. This
layer of muscle becomes spirally arranged in the smaller bronchioles, but represents a
major portion of the wall of the bronchioles. Air flow may be greatly reduced by
contraction of the smooth muscle of the bronchiolar wall. The muscle layer is located
between the mucosa and submucosa, as a muscularis.
Question:
The posterior portion of the vestibule of the nasal cavity reveals a transition from:

Answer Choices:
A Keratinized stratified squamous epithelium to non-keratinized stratified
squamous epithelium
B Non-keratinized stratified squamous epithelium to ciliated, simple columnar
epithelium
C Ciliated, simple columnar epithelium to ciliated, pseudostratified columnar
epithelium
D Non-keratinized stratified squamous epithelium to ciliated, pseudostratified
columnar epithelium
E Non-keratinized stratified squamous epithelium to transitional epithelium

D Non-keratinized stratified squamous epithelium to ciliated, pseudostratified columnar


epithelium

The vestibule of the nasal cavity opens anteriorly with the external environment and is
lined with a continuation of the external skin of the face, the stratified squamous
epithelium. Initially, the stratified squamous of the vestibule is keratinized, but deeper
(posteriorly) within its recesses it becomes non-keratinized. Within this region, the
vestibule contains hairs that filter out inhaled particulate matter and helps to condition
the air before it continues into the rest of the respiratory system. At the most posterior
portion of the vestibule, the non-keratinized stratified squamous epithelium becomes
pseudostratified columnar epithelium, with cilia.
Question:
The surface of the olfactory epithelium is kept moist due to secretions from:

Answer Choices:
A Goblet cells within the epithelium
B Serous secreting glands located within the underlying lamina propria of the
epithelium
C Mucous secreting glands located within the underlying lamina propria of the
epithelium
D Goblet cell secretions located near the olfactory epithelium
E Serous secreting cells located within the olfactory epithelium

B Serous secreting glands located within the underlying lamina propria of the
epithelium

Branched, tubuloalveolar glands that secrete a serous secretion are located within the
lamina propria of the olfactory epithelium. These olfactory glands, or Bowman's glands,
secrete a proteinaceous material which serves as a solvent for odiferous substances as
well as provides a flow to rid the mucosa of previously detected odoriferous substances.
Goblet cells are not found as part of the olfactory epithelium.
Question:
Cells within the olfactory epithelium that provide structural support to the olfactory cells are the:

Answer Choices:
A Brush cells
B Sustentacular cells
C Basal cells
D Goblet cells
E Ciliated cells

B Sustentacular cells

The sustentacular cells are the most numerous cells in the olfactory epithelium and
provide structural support to the olfactory cells. These tall columnar cells may also
function in a manner similar to those of glial cells in the nervous system as well as
provide metabolic support to the olfactory cells. The basal cells are regarded as the
stem cells of the olfactory epithelium. Goblet cells are not found in the olfactory
epithelium. Cilia are extensions of the bipolar olfactory cells.
Question:
Which one of the following cell types contained within the tracheal respiratory epithelium is regarded as
an enteroendocrine cell?

Answer Choices:
A Ciliated cells
B Brush cells
C Goblet cells
D Granule cells
E Basal cells

D Granule cells

The respiratory epithelium is classified as a ciliated, pseudostratified columnar


epithelium which is composed of several cell types. The most numerous are the ciliated
cells from which apical cilia project into the lumen. Intermittent throughout the
respiratory epithelium are goblet cells, also referred to as mucous cells. These cells
accumulate mucinogen granules and provide a mucous secretion.
Other columnar-shaped cells bear blunt microvilli on their apical surface and are
referred to as brush cells. The basal surface of the brush cells are in synaptic contact
with a sensory nerve ending that subserve general sensation to the epithelium. Granule
cells are scattered sparsely throughout the epithelium and belong to the general class of
enteroendocrine cells. Basal cells serve as the stem cells for maintaining the cells in the
epithelium. Their nuclei reside close to the basement membrane in histological section.
Question:
The C-shaped cartilaginous rings found in the wall of the trachea are composed of which type of
cartilage?

Answer Choices:
A Hyaline
B Fibrocartilage
C Elastic cartilage
D Interspersed cartilage
E Intrapulmonary cartilage

A Hyaline

The C-shaped cartilaginous rings found in the trachea and the plates of cartilage that
occur in the extra- and intrapulmonary bronchi are comprised of hyaline cartilage.
Elastic fibers may be present surrounding the cartilage plates and interspersed among
the connective tissue of the submucosa, but elastic cartilage does not normally occur
along conducting passages of the respiratory system.

Question:
The epithelium lining normal bronchi differs from that lining the smaller, terminal bronchioles. Epithelial
lining of larger bronchioles is typically _________, becoming __________ as the duct narrows, and
____________ in terminal bronchioles.

Answer Choices:
A Ciliated, simple columnar epithelium; transitional epithelium; simple squamous
epithelium
B Ciliated, pseudostratified columnar epithelium; ciliated, simple columnar
epithelium; simple cuboidal epithelium
C Ciliated, pseudostratified columnar epithelium; simple columnar epithelium;
simple cuboidal epithelium
D Ciliated, simple columnar epithelium; simple cuboidal epithelium; simple
squamous epithelium
E Pseudostratified columnar epithelium; pseudostratified cuboidal epithelium;
simple cuboidal epithelium
B Ciliated, pseudostratified columnar epithelium; ciliated, simple columnar epithelium;
simple cuboidal epithelium

The mucosa of the bronchi are normally lined with a ciliated, pseudostratified columnar
epithelium. The ciliated, pseudostratified columnar epithelium persists in the larger
diameter bronchioles, but a transition occurs such that the smaller diameter bronchioles
are lined by a ciliated, simple columnar epithelium, which eventually becomes simple
cuboidal epithelium in the terminal bronchioles. Furthermore, the number of goblet cells
decreases; they are present in the larger bronchioles, but become absent in smaller
diameter bronchioles.

Question:
At what level of the respiratory system are discontinuous cartilaginous plates commonly found?
Answer Choices:
A Trachea
B Primary bronchi
C Intrapulmonary bronchi
D Terminal bronchioles
E Respiratory bronchioles

C Intrapulmonary bronchi

Primary bronchi become intrapulmonary bronchi upon entering the lungs, which then
branch into secondary and tertiary bronchi. The structure of the intrapulmonary bronchi
differ from that of the primary bronchi and trachea inasmuch that the tracheal rings
common to the trachea and the primary bronchi become discontinuous plates of
cartilage. These plates of cartilage decrease in size as the diameter of the bronchi
decreases and eventually disappears in bronchioles. Therefore, only the intrapulmonary
bronchi contain plates of cartilage, and bronchioles generally lack cartilage in their
walls.
SECTION- URINARY SYSTEM
Question:
A 36-year-old man presents with bloody diarrhea that started 3 days ago. Upon initial assessment, you
notice that he is lethargic and his mucus membranes are very dry. The following laboratory data is
available:
Hb 11.4 gm/dL
Hct 34.2 %
WBC 8,0000
Na+ 140mEq/L
+
K 4.3mEq/L
-
Cl 115mEq/L
HCO3 22 mEq/L
BUN 28 mg/dL
Creatinine 1.4 mg/dL
Physiologically what is the effector most likely responsible to restore euvolia?

Answer Choices:
A Antidiuretic hormone (ADH)
B Cortisol
C Atrial natriuretic peptide
D Aldosterone
E Epinephrine
D Aldosterone

Sodium is the principal osmol of the extracellular space; it acts to hold water particles in
this compartment. Plasma osmolality plays the primary role in the regulation
of intracellular volume. This is achieved by homeostasis of water and is important for
normal cellular functioning. Plasma sodium plays the primary role in the regulation
of plasma volume. Thus, sodium homeostasis is essential for tissue perfusion. Volume
and water homeostasis respond to different stimuli, act on different receptors and
respond differently to restore either euvolia or water balance.
These mechanisms can work together or independently. For example, A patient who
receives an infusion of isotonic saline responds to volume changes but not to osmolality
changes. This patient compensates for volume increase by releasing atrial natriuretic
peptide, resulting in increased secretion of Na+ by the kidney. However, because
osmolality has not been altered, neither thirst nor ADH secretion is activated. On the
other hand, a patient who receives an infusion of hypotonic saline responds both to
volume and osmolality changes. The increase in volume results in Na+ secretion. The
hypoosmolality induced by hypotonic saline suppresses ADH and thirst resulting in the
excretion of free water into the urine.
The following table illustrates both of these systems:
CHANGE RECEPTOR EFFECTORS RESPONSE
VOLUME Cardiopulmonary Aldosterone GFR Atrial EUVOLEMIA
Carotid Juxtaglomerular natruretic peptide (ANP)
OSMOLALITY NORMAL
Hypothalamus Thirst ADH
OSMOLALITY
+
The primary aim of the effectors is to affect Na re-absorption and secretion by the
kidney.
Increased volume → ANP → ↑ Na+ secretion → Euvolemia
Decreased volume → Aldosterone → ↑ Na+ re-absorption → Euvolemia
The sensors for osmolality are located in the hypothalamus and respond to small
changes in plasma osmolality to keep it at a narrow window between 275-290m Osmol.
Persistent changes are rarely seen due to the ability of the kidney to secrete large
amounts of water or retain it by ADH secretion. Osmolality changes may be seen in
renal impaired or comatose patients.
Question:
A 44-year-old obese Hispanic female presents to the emergency room with increasing intensity and
duration of right upper quadrant pain. On review of her records, you notice that she had presented to
the emergency room several times with the same complaint and that she had a recent ultrasound
documenting cholelithiasis. After discussing the case with the surgeon on call, you decide to admit her
to the hospital to undergo a laparoscopic cholecystectomy the next day.
On the ward, the patient becomes nauseous and vomits several times. A nasogastric tube is placed
and intravenous fluids are started, consisting of half-normal saline with 5% dextrose at a rate of 90
cc/hr. As you prepare the patient to undergo surgery, you order a set of laboratory values. The
following laboratory values are available:
Hb 13.4 gm/dl
Hct 40.2 %
WBC 11,0000
Na+ 138mEq/L
K+ 3.1mEq/L
Cl- 100mEq/L
HCO3 32 mEq/L
BUN 12 mg/dl
Creatinine 0.8 mg/dl
The pH in the urine of this patient is likely to be:

Answer Choices:
A Acidic
B Basic
C Neutral
D Unpredictable
E Non-diagnostic
A Acidic

This scenario illustrates the typical presentation of a patient with contraction alkalosis.
This concept refers to a patient who is volume depleted from fluid loss containing HCl
(fluid losses from the stomach as in nasogastric decompression or persistent vomiting).
With inappropriate fluid replacement, a patient is at risk of developing hypochloremic,
hypokalemic, metabolic alkalosis. Because of volume depletion, the kidney will retain
sodium in an attempt to maintain volume. Although there is metabolic alkalosis, the
kidney will excrete hydrogen ions in exchange for the sodium instead of potassium
because hypokalemia would be more deleterious to the body than the metabolic
alkalosis already in place. This concept is known as paradoxical aciduria.
Question:
A 32-year-old male presents with complaints of fever, burning micturition, and purulent urethral
discharge since 2 days ago. He also admits to being sexually promiscuous. The patient is diagnosed
with gonorrhea after relevant investigations. N. gonorrheae attaches to the mucosal surfaces and
penetrates deeper causing infection. Which of the following structure is predominantly present in the
male urethra but not in the female urethra?

Answer Choices:
A Stratified squamous epithelium
B Columnar epithelium
C Urethral glands
D Sphincter urethrae membranaceae
E Connective tissue layer beneath the epithelium
B Columnar epithelium

The male urethra is divided into 3 portions: the prostatic, membranous, and the
cavernous portion. The male urethra is predominantly lined by the columnar epithelium
in the membranous and the cavernous portion. Stratified squamous epithelium lines
most of the female urethra as well as the distal end of the cavernous urethra in males.
Mucus-secreting urethral glands are always present in the male and the female urethra.
Sphincter urethrae membranaceae is present in both the male and the female urethra.
References:
1. Fritsch H, Pinggera GM, Lienemann A, Mitterberger M, Bartsch G, Strasser H. What
are the supportive structures of the female urethra? Neurourol Urodyn. 2006;25(2):128-
34
2. Klauser A, Strasser H, Helweg G, et al. Age-Related Rhabdosphincter Function in
Female Urinary Stress Incontinence. J Ultrasound Med 2004; 23:631-637.
Question:
A 72-year-old female is recovering from a total hip replacement she had two days ago. A nurse calls
you because she noticed the patient's heart rate keeps increasing. She is NPO, and she is on
intravenous fluid consisting of half normal saline with 5% dextrose and 20meq of KCl at 40cc/hr. She is
on intravenous morphine as needed for pain every four hours. She complains of no pain and is slightly
lethargic. Her vital signs are as follows Temperature= 38.5 ° C, Heart Rate = 130, Blood Pressure
102/60, her urine output for the past eight hour is 60 cc.
On physical examination, her mucous membranes are very dry. Her lungs are clear to auscultation,
and her heart rate is regular by tachycardic. The rest of her physical exam is unremarkable. What test
would most accurately provide the diagnosis to explain her tachycardia?

Answer Choices:
A An electrocardiogram
B A chest-X ray
C Orthostatic blood pressure measurements
D Urinary sodium
E Cardiac enzymes times three every eight hours
D Urinary sodium

Oliguria (decreased urine output) is a reliable sign of volume depletion. However, there
are a few causes of oliguria and they have to be investigated carefully in a patient with
decreased urine output. Oliguria is divided into three categories:
OLIGURIA
Prerenal Azotemia Renal Azotemia Postrenal Azotemia
Oliguria caused by
Oliguria caused by obstruction
decreased renal Oliguria caused by pathology to the
of the urinary system below
perfusion. Common functional units of the kidney, which
the kidneys. Common causes
causes include include some glomerulonephropathy
include: Nephrolithiasis,
hypotension (i.e. shock), and acute tubular necrosis (ATN).
benign prostatic hyperplasia
cardiac causes (i.e. However, renal azotemia is primarily
causing obstruction, or penile
severe CHF), and severe cased by ATN.
strictures.
volume depletion.
Postrenal azotemia can be quickly excluded by insertion of a Foley Catheter, which
would then result rapid and large urine output. Ultrasound may reveal hydronephrosis
(distension of the urinary collecting system by fluid). Once post renal azotemia has been
excluded, an important distinction of azotemia in the surgical patient is to differentiate
between prerenal and postrenal azotemia.
Distinction of prerenal and renal azotemia can be made based on the fractional
excretion of sodium Fe Na, which can be calculated by the following formula:
X 100
Fe Na =
Serum Cr X Urine Na
Serum Na X Urine Cr
A value of less than 1% is indicative of prerenal azotemia, while a value greater than
1% is indicative of renal azotemia. Urinary sodium less than 20mEq/L is pathonomic of
decreased tissue perfusion and an excellent test to assess volume status.
Thus, in this case obtaining urinary electrolytes would most likely provide the diagnosis.
A chest-X ray would not be indicated unless there was suspicion of fluid over load, in
which case this could be used to establish pulmonary edema. An electrocardiogram
would not provide any additional information and her tachycardia is most likely
secondary to volume depletion. Orthostatic blood pressure measurements may be a
good alternative in providing the diagnosis and this would be the right answer if urinary
sodium were not a choice. The "gold-standard" is urine electrolytes and thus this is the
right answer

B Modified smooth muscle cells of the afferent and efferent arteriole walls

The smooth muscle cells of the afferent and efferent arteriole walls serve as a
mechanoreceptor to help regulate blood pressure. They produce and release renin to
increase blood pressure.
The cells of the macula densa are modified tubular cells of the distal tubule. However,
these cells measure flow of filtrate and do not activate the renin/angiotensin/aldosterone
system. These cells will measure total sodium in the filtrate to activate aldosterone
directly.
The vascular endothelial cells are thought to produce vasodilatory or vasoconstrictive
agents in response to chemical signals from the macula densa.
The cuboidal tubular cells of the ascending limb of the loop of Henle actively transport
sodium from the filtrate to the plasma. These cells are the targets of furosemide.
Podocytes and mesangial cells are located in the glomerulus and participate in
determining the filterability of specific substances.
Question:
What epithelium lines the distal portion of the female urethra at, or near, its termination?

Answer Choices:
A Transitional
B Pseudostratified columnar
C Stratified cuboidal
D Non-keratinized stratified squamous
E Simple columnar

D Non-keratinized stratified squamous


The female urethra, while quite short (3-5 cm), may have areas that are lined by
different epithelium. The portion of the urethra leaving the urinary bladder is initially
lined with transitional epithelium. There may be a short portion midway between the
urinary bladder and the urethral ostium lined with pseudostratified columnar epithelium.
Near the distal end of the urethra, it becomes lined with non-keratinized stratified
squamous epithelium which is continuous with that of the external skin.
Question:
What epithelium lines the mid-portion of the female urethra

Answer Choices:
A Transitional
B Pseudostratified columnar
C Keratinized stratified squamous
D Non-keratinized stratified squamous
E Simple columnar

B Pseudostratified columnar

The female urethra, while quite short (3-5 cm), may have areas that are lined by
different epithelium. The portion of the urethra leaving the urinary bladder is initially
lined with transitional epithelium. There may be a short portion midway between the
urinary bladder and the urethral ostium lined with pseudostratified columnar epithelium.
Near the distal end of the urethra, it becomes lined with non-keratinized stratified
squamous epithelium which is continuous with that of the external skin.

Question:
What epithelium lines the proximal portion of the female urethra?

Answer Choices:
A Transitional
B Pseudostratified columnar
C Keratinized stratified squamous
D Non-keratinized stratified squamous
E Simple columnar

A Transitional

The female urethra, while quite short (3-5 cm), may have areas that are lined by
different epithelium. The portion of the urethra leaving the urinary bladder is initially
lined with transitional epithelium. There may be a short portion midway between the
urinary bladder and the urethral ostium lined with pseudostratified columnar epithelium.
Near the distal end of the urethra, it becomes lined with non-keratinized stratified
squamous epithelium which is continuous with that of the external skin.

Question:
The penile urethra is surrounded by specialized tissue referred to as the

Answer Choices:
A Corpus spongiosum
B Corpus albicans
C Corpus luteum
D Corpora arenacea

A Corpus spongiosum

The specialized tissue surrounding the penile portion of the urethra is the corpus
spongiosum (corpus cavernosum urethrae). This tissue contains extensive, large
sinuses which, upon erection, engorge with blood to increase the overall size and
rigidity of the tissue. Numerous bundles of smooth muscle may be observed running
longitudinally within the spongiosum. Also, some circularly arranged muscle fibers may
be observed near the periphery of the spongiosum. This entire area is encircled by a
thick fibroelastic connective tissue sheath, the tunica albuginea.
Corpus albicans is a white scar found as the remnant of the corpora lutea of the ovary
following its regression.
A corpus luteum forms from the mature follicular cells following ovulation.
Corpora arenacea are calcified bodies found within the parenchyma of the pineal gland.

Question:
What type of epithelium lines the initial (proximal) portion of the prostatic urethra?

Answer Choices:
A Transitional
B Pseudostratified columnar
C Keratinized stratified squamous
D Non-keratinized stratified squamous
E Simple columnar

A Transitional
The male urethra has three segments, some portions of each lined with a different
epithelium.
The portion of the urethra leaving the urinary bladder is the prostatic urethra and
extends 3-4 cm through the prostate gland. It is initially lined with transitional epithelium
until the ejaculatory ducts join the prostatic urethra, whereupon the epithelium changes
to pseudostratified columnar epithelium.
As the urethra passes from the prostate gland through the pelvic diaphragm, there is a
short (1cm) portion referred to as the membranous urethra, lined with pseudostratified
columnar epithelium.
The pseudostratified columnar epithelium continues through the next segment of the
urethra, the penile urethra, which is about 15 cm long. Near the distal end of the
urethra, it becomes lined with non-keratinized stratified squamous epithelium which is
continuous with that of the skin of the head of the penis.

Question:
What type of epithelium lines much of the length of the penile urethra?

Answer Choices:
A Transitional
B Pseudostratified columnar
C Keratinized stratified squamous
D Non-keratinized stratified squamous
E Simple columnar

B Pseudostratified columnar

The male urethra has three segments, some portions of each lined with a different
epithelium.
The portion of the urethra leaving the urinary bladder is the prostatic urethra and
extends 3-4 cm through the prostate gland. It is initially lined with transitional epithelium
until the ejaculatory ducts join the prostatic urethra, whereupon the epithelium changes
to pseudostratified columnar epithelium.
As the urethra passes from the prostate gland through the pelvic diaphragm, there is a
short (1cm) portion referred to as the membranous urethra, lined with pseudostratified
columnar epithelium.
The pseudostratified columnar epithelium continues through the next segment of the
urethra, the penile urethra, which is about 15 cm long. Near the distal end of the
urethra, it becomes lined with non-keratinized stratified squamous epithelium which is
continuous with that of the skin of the head of the penis.
Question:
Histologically, the principal features of the urinary bladder include
Answer Choices:
A Transitional epithelium and inner circular and outer longitudinal muscle layers
B Transitional epithelium and a thick muscularis with two indistinct layers
C Pseudostratified columnar epithelium and a thick muscularis with three
indistinct layers
D Transitional epithelium and a thick muscularis with three indistinct layers
E Pseudostratified columnar epithelium with inner longitudinal and outer circular
muscle layers

D Transitional epithelium and a thick muscularis with three indistinct layers

As urine is conveyed to the urinary bladder for temporary storage, it is essential that the
mucosa be of a type capable of volume expansion, at least to a limited extent. The
urinary bladder epithelium is transitional which, as the bladder fills, will allow for the
thinning of the mucosa and permit volume expansion. Underlying the epithelium is a
dense lamina propria, but the wall lacks a muscularis mucosa and a submucosa. The
muscularis of the urinary bladder contains three layers of smooth muscle; an inner
longitudinal layer, a middle circular layer, and an outer longitudinal layer. However, the
layers of the thick muscularis are difficult to distinguish and are rather indistinct.

Question:
The photomicroscopic image represents a tubular organ in cross-section. Which of the following organs
is best depicted by this image

Answer Choices:
A Ureter
B Gall bladder
C Penile urethra
D Uterine tube
E Epididymis

Image(s) / Chart(s):
Click image to view full size. Click open image to close. Click and hold open image to move.
A Ureter

The photomicroscopic image depicts a ureter cut in cross-section. Since the ureters
conduct urine from the renal pelvis to the urinary bladder, it is essential that the mucosa
be of a type capable of volume expansion, at least to a limited extent. The epithelium
lining the ureter is transitional, which will aid in the thinning of the mucosa and allow
nominal expansion in the ureters, if necessary. Underlying the epithelium is a dense
lamina propria. For most of the length of the ureters, there are usually only two layers of
smooth muscle in the muscularis; an inner layer that is arranged longitudinal to the axis
of the lumen, and an outer layer arranged in a tight spiral described as a circular layer.
If a section of the ureter is made near the renal pelvis and is oriented such that a cross-
section of the lumen is made, the tissue will reveal only two layers of muscle in the wall.
These layers will appear arranged in an inner longitudinal and outer circular orientation
to the lumen. By comparison, the epithelium lining the gall bladder is simple columnar
and the lamina propria is extensively folded such that the mucosa possess primary,
secondary and tertiary folds. Based simply on epithelium, the penile urethra, for most of
its length is lined by pseudostratified columnar epithelium, except near the opening to
the exterior where the epithelium changes to stratified squamous. The epithelium of the
uterine tube also is simple columnar, but possesses both ciliated and peg cells. The
lamina propria may be highly folded. The lining of the epididymis is pseudostratified
columnar with long, branched stereocilia.

Question:
Which of the following series best describes the pathway of the flow of urine through the excretory
passages?

Answer Choices:
A Renal calyces, renal pelvis, ureter, urinary bladder, urethra
B Renal calyces, renal pelvis, urethra, urinary bladder, ureter
C Renal pelvis, renal calyces, ureter, urinary bladder, urethra
D Renal pelvis, renal calyces, urethra, urinary bladder, ureter
E Ureter, urinary bladder, urethra

A Renal calyces, renal pelvis, ureter, urinary bladder, urethra

As urine flows from the nephron into the collecting tubules, it leaves the tubular system
through the area cribrosa, a sieve-like area of medulla that projects into the minor
calyces. From the area cribrosa, urine flows through the minor calyx to the major calyx,
into the renal pelvis and thence, through the ureters to the urinary bladder. The urine
may be stored in the urinary bladder where, upon elimination, the urine flows through
the urethra to be voided.

Question:
The renal pelvis delivers urine to the

Answer Choices:
A Major calyx of the kidney
B Urinary bladder
C Ureter
D Urethra
E Area cribrosa

C Ureter

As urine flows from the nephron into the collecting tubules, it leaves the tubular system
through the area cribrosa, a sieve-like area of medulla that projects into the minor
calyces. From the area cribrosa, urine flows through the minor calyx to the major calyx,
into the renal pelvis and thence, through the ureters to the urinary bladder. The urine
may be stored in the urinary bladder where, upon elimination, the urine flows through
the urethra to be voided.
Question:
Histological examination of a section of ureter reveals the innermost layer of the muscularis is oriented
longitudinal to the cut and an outer layer of muscle in which the cells appear to be in cross-section. You
conclude that the ureter is from which general area and has been sectioned in what orientation

Answer Choices:
A The ureter is from near the urinary bladder and was cut in cross section to the
axis of the lumen
B The ureter is from near the kidney pelvis and was cut parallel to the axis of the
lumen
C It is not possible to determine from where the ureter was taken, nor how the
ureter was oriented from the observations provided
D The ureter is from near the urinary bladder and was cut parallel to the axis of
the lumen
E The ureter is from near the kidney pelvis and was cut in cross section to the
axis of the lumen
B The ureter is from near the kidney pelvis and was cut parallel to the axis of the
lumen

Since the ureters conduct urine from the renal pelvis to the urinary bladder, it is
essential that the mucosa be of a type capable of volume expansion, at least to a limited
extent. The epithelium of the ureters is transitional, which will aid in the thinning of the
mucosa and allow nominal expansion in the ureters, if necessary. Underlying the
epithelium is a dense lamina propria, but the wall lacks a muscularis mucosa and a
submucosa.
For most of the length of the ureters, there are usually only two layers of smooth muscle
in the muscularis; an inner layer that is arranged longitudinal to the axis of the lumen,
and an outer layer arranged in a tight spiral described as a circular layer. Near the
urinary bladder, an additional layer of smooth muscle is present in the ureters as an
outer layer arranged in a longitudinal orientation.
If a section of the ureter is made near the renal pelvis and is oriented such that a
longitudinal section of the lumen is made, the tissue will reveal two layers of muscle in
the wall. The inner longitudinal layer will appear cut longitudinally, whereas the outer
circular layer of muscle will appear cut in cross-section to the orientation to the lumen.
Question:
Histological examination of a section of ureter reveals an innermost layer of the muscularis which is
sectioned in cross-section and an outer layer of muscle in which the cells are oriented longitudinal to
the cut. You conclude that the ureter is from which general area and has been sectioned in what
orientation

Answer Choices:
A The ureter is from near the urinary bladder and was cut in cross section to the
axis of the lumen
B The ureter is from near the kidney pelvis and was cut parallel to the axis of the
lumen
C It is not possible to determine from where the ureter was taken, nor how the
ureter was oriented from the observations provided
D The ureter is from near the urinary bladder and was cut parallel to the axis of
the lumen
E The ureter is from near the kidney pelvis and was cut in cross section to the
axis of the lumen
E The ureter is from near the kidney pelvis and was cut in cross section to the axis of
the lumen
Since the ureters conduct urine from the renal pelvis to the urinary bladder, it is
essential that the mucosa be of a type capable of volume expansion, at least to a limited
extent. The epithelium of the ureters is transitional, which will aid in the thinning of the
mucosa and allow nominal expansion in the ureters, if necessary. Underlying the
epithelium is a dense lamina propria, but the wall lacks a muscularis mucosa and a
submucosa.
For most of the length of the ureters, there are usually only two layers of smooth muscle
in the muscularis; an inner layer that is arranged longitudinal to the axis of the lumen,
and an outer layer arranged in a tight spiral described as a circular layer. Near the
urinary bladder, an additional layer of smooth muscle is present in the ureters as an
outer layer arranged in a longitudinal orientation. A section of the ureter made near the
renal pelvis and oriented such that a cross-section of the lumen is made, the tissue will
reveal only two layers of muscle in the wall. These layers will appear arranged in an
inner longitudinal and outer circular orientation to the lumen.

Question:
Histological examination of a section of ureter reveals the innermost layer of the muscularis is
sectioned in cross-section, a middle layer of muscle in which the cells are oriented longitudinal to the
cut, then an outermost layer in which the cells are cut in cross-section. You deduce that the ureter is
from which general area and has been sectioned in what orientation

Answer Choices:
A The ureter is from near the urinary bladder and was cut in cross section to the
axis of the lumen
B The ureter is from near the kidney pelvis and was cut parallel to the axis of the
lumen
C It is not possible to determine from where the ureter was taken, nor how the
ureter was oriented from the observations provided
D The ureter is from near the urinary bladder and was cut parallel to the axis of
the lumen
E The ureter is from near the kidney pelvis and was cut in cross section to the
axis of the lumen
A The ureter is from near the urinary bladder and was cut in cross section to the axis
of the lumen

Since the ureters conduct urine from the renal pelvis to the urinary bladder, it is
essential that the mucosa be of a type capable of volume expansion, at least to a limited
extent. The epithelium of the ureters is transitional, which will aid in the thinning of the
mucosa and allow nominal expansion in the ureters, if necessary. Underlying the
epithelium is a dense lamina propria, but the wall lacks a muscularis mucosa and a
submucosa.
For most of the length of the ureters, there are usually only two layers of smooth muscle
in the muscularis; an inner layer that is arranged longitudinal to the axis of the lumen,
and an outer layer arranged in a tight spiral described as a circular layer. Near the
urinary bladder, an additional layer of smooth muscle is present in the ureters as an
outer layer arranged in a longitudinal orientation. When a section of the ureter is made
near the urinary bladder and oriented such that a cross-section of the lumen is made,
the tissue will reveal three layers of muscle in the wall. They will appear arranged in an
inner longitudinal, middle circular, and outer longitudinal orientation to the lumen.

Question:
Which type of epithelium is common to the major calyx of the kidney, the renal pelvis, and the ureters?

Answer Choices:
A Stratified squamous
B Simple cuboidal
C Simple columnar
D Pseudostratified columnar
E Transitional

E Transitional

All of the excretory passages beginning from the calyx of the kidney to the urinary
bladder are lined with transitional epithelium. Transitional epithelium is a stratified
epithelium that allows distensibility of the lining of the excretory passages. The surface
cells are characterized by large, sometimes binucleated, dome-shaped cells that
accommodate the thinning when the passages are filled with urine. Furthermore, this
epithelium is essentially impermeable to ions, such as Na+ or Cl-.

Question:
Sympathectomy of the kidney results in which of the following responses?

Answer Choices:
A Reduction of glomerular filtration rate and decreased urinary output
B Increased urine output
C Increased glomerular filtration rate and no change in urine output
D Decreased urine output
E No change to the glomerular filtration rate, nor to the urine output
B Increased urine output

The kidney is innervated primarily by the sympathetic nervous system, and as such,
regulates the contraction of the smooth muscle surrounding the afferent and efferent
arterioles of the glomerulus. Sympathetic stimulation to the efferent glomerular
arterioles causes their constriction and increases the filtration rate, leading to an
increase in urinary output. Similarly, with the loss of sympathetic innervation, such as
due to sympathectomy of the kidney, relaxation of tone of the afferent arteriole occurs
and the filtration rate increases, which results in an increased urine output. Decreased
urine output may result from the constriction of the afferent arterioles, leading to a
decrease in glomerular filtration rate.

Question:
Interstitial tissue of the kidney cortex is comprised of:

Answer Choices:
A Mesangial cells and fibroblasts.
B Fibroblasts and macrophages.
C Macrophages and epithelial cells.
D Fibroblasts and endothelial cells.
E Mesangial cells and podocytes.

B Fibroblasts and macrophages.

The interstitial tissue found within the kidney cortex is comprised primarily of cells best
described as being like fibroblasts and occasional tissue macrophages. The fibroblasts
are found between the basement membrane of the tubules and peritubular capillaries,
and they function to synthesize and secrete collagen fibers of the interstitial matrix.
These cells of the interstitium surround the components of the nephron and the
associated blood vessels.
Mesangial cells, which may be located either within the glomerulus or outside of the
glomerulus near the vascular pole, are cells that provide structural support for the
podocytes. They also are phagocytic and function to remove particulate matter from the
glomerular basement membrane.
Epithelial cells of the kidney cortex would comprise the epithelium of the proximal and
distal tubules, in general, as well as the specialized epithelial cells of Bowman's
capsule.
Endothelial cells would constitute the cells of the glomerular capillary, as well as the
endothelial cells of the vessels found within the cortex of the kidney.
Question:
Hypertonic urine may be produced by the secretion of which substance?

Answer Choices:
A Aldosterone
B Angiotensinogen
C Antidiuretic hormone (ADH)
D Angiotensin I
E Androstenedione

C Antidiuretic hormone (ADH)

The distal convoluted tubule (DCT) in the kidney cortex functions to reabsorb Na + and
secrete K+ . This is regulated by the adrenal hormone, aldosterone, as stimulated by
angiotensin II, which will increase reabsorption of Na+ and secrete K+ . The DCT also
functions in the continuation of bicarbonate ion reabsorption with hydrogen ion
secretion. The terminal portion of the DCT is sensitive to antidiuretic hormone (ADH)
from the neurohypophysis, which acts to increase the permeability of the tubule to
water, resulting in concentrating the urine.
The action of ADH is more pronounced on the epithelium of the collecting ducts, such
that in the absence of ADH, a copious, dilute urine is produced. Increased secretion of
ADH has the effect of producing a hypertonic urine. ADH secretion occurs in response
to an increase in plasma osmolality or a decrease in blood volume, which may result
from the loss of water by sweating, vomiting, or diarrhea.
Question:
Which of the following cells secrete renin?

Answer Choices:
A Podocytes
B Capillary endothelial cells
C Bowman's capsule epithelial cells
D Intraglomerular mesangial cells
E Juxtaglomerular cells
E Juxtaglomerular cells

Podocytes are modified simple squamous cells that comprise the visceral layer of
Bowman's capsule. These cells also possess foot processes, or pedicels that extend
around the glomerular capillaries and interdigitate with the pedicels of neighboring
podocytes.
Capillary endothelial cells line the glomerular capillary upon which the podocytes reside.
The outer layer of simple squamous epithelial cells of Bowman's capsule comprise the
parietal layer, which is continuous with the cuboidal epithelium of the proximal
convoluted tubule at the urinary pole.
Mesangial cells also are found in association with the renal corpuscle; most within the
corpuscle as intraglomerular mesangial cells, but some outside of the corpuscle along
the vascular pole. The mesangial cells function as phagocytic cells and provide
structural support for podocytes.
Smooth muscle cells circling the afferent arteriole of the glomerulus are modified and
contain secretory granules of renin. These modified muscle cells are called the
juxtaglomerular cells and are best visualized with histochemical stain for the renin.
Question:
Which of the following cells may provide structural support within the renal corpuscle?

Answer Choices:
A Podocytes
B Capillary endothelial cells
C Bowman's capsule epithelial cells
D Mesangial cells
E Juxtaglomerular cells

D Mesangial cells

Podocytes are modified simple squamous cells that comprise the visceral layer of
Bowman's capsule. These cells also possess foot processes, or pedicels that extend
around the glomerular capillaries and interdigitate with the pedicels of neighboring
podocytes.
Capillary endothelial cells line the glomerular capillary upon which the podocytes reside.
The outer layer of simple squamous epithelial cells of Bowman's capsule comprise the
parietal layer, which is continuous with the cuboidal epithelium of the proximal
convoluted tubule at the urinary pole.
Mesangial cells also are found in association with the renal corpuscle; most within the
corpuscle as intraglomerular mesangial cells, but some outside of the corpuscle along
the vascular pole. The mesangial cells function as phagocytic cells and provide
structural support for podocytes.
Smooth muscle cells circling the afferent arteriole of the glomerulus are modified and
contain secretory granules of renin. These modified muscle cells are called the
juxtaglomerular cells and are best visualized with histochemical stain for the renin.
Question:
Which of the following cells are phagocytes within the renal corpuscle

Answer Choices:
A Podocytes
B Capillary endothelial cells
C Bowman's capsule epithelial cells
D Mesangial cells
E Juxtaglomerular cells
D Mesangial cells
Podocytes are modified simple squamous cells that comprise the visceral layer of
Bowman's capsule. These cells also possess foot processes, or pedicels that extend
around the glomerular capillaries and interdigitate with the pedicles of neighboring
podocytes.
Capillary endothelial cells line the glomerular capillary upon which the podocytes reside.
The outer layer of simple squamous epithelial cells of Bowman's capsule comprise the
parietal layer, which is continuous with the cuboidal epithelium of the proximal
convoluted tubule at the urinary pole.
Mesangial cells also are found in association with the renal corpuscle; most within the
corpuscle as intraglomerular mesangial cells, but some outside of the corpuscle along
the vascular pole. The mesangial cells function as phagocytic cells and provide
structural support for podocytes.
Smooth muscle cells circling the afferent arteriole of the glomerulus are modified and
contain secretory granules of renin. These modified muscle cells are called the
juxtaglomerular cells and are best visualized with histochemical stain for the renin.
Question:
Which of the following cells comprises the parietal layer of the Bowman's capsule?

Answer Choices:
A Podocytes
B Capillary endothelial cells
C Bowman's capsule epithelial cells
D Intraglomerular mesangial cells
E Juxtaglomerular cells

C Bowman's capsule epithelial cells

Podocytes are modified simple squamous cells that comprise the visceral layer of
Bowman's capsule. These cells also possess foot processes, or pedicels that extend
around the glomerular capillaries and interdigitate with the pedicels of neighboring
podocytes.
Capillary endothelial cells line the glomerular capillary upon which the podocytes reside.
The outer layer of simple squamous epithelial cells of Bowman's capsule comprise the
parietal layer, which is continuous with the cuboidal epithelium of the proximal
convoluted tubule at the urinary pole.
Mesangial cells also are found in association with the renal corpuscle; most within the
corpuscle as intraglomerular mesangial cells, but some outside of the corpuscle along
the vascular pole. The mesangial cells function as phagocytic cells and provide
structural support for podocytes.
Smooth muscle cells circling the afferent arteriole of the glomerulus are modified and
contain secretory granules of renin. These modified muscle cells are called the
juxtaglomerular cells and are best visualized with histochemical stain for the renin.
Question:
Which of the following cells comprises the visceral layer of the Bowman's capsule
Answer Choices:
A Podocytes
B Capillary endothelial cells
C Bowman's capsule epithelial cells
D Intraglomerular mesangial cells
E Juxtaglomerular cells

A Podocytes

Podocytes are modified simple squamous cells that comprise the visceral layer of
Bowman's capsule. These cells also possess foot processes, or pedicels that extend
around the glomerular capillaries and interdigitate with the pedicels of neighboring
podocytes.
Capillary endothelial cells line the glomerular capillary upon which the podocytes reside.
The outer layer of simple squamous epithelial cells of Bowman's capsule comprise the
parietal layer, which is continuous with the cuboidal epithelium of the proximal
convoluted tubule at the urinary pole.
Mesangial cells also are found in association with the renal corpuscle; most within the
corpuscle as intraglomerular mesangial cells, but some outside of the corpuscle along
the vascular pole. The mesangial cells function as phagocytic cells and provide
structural support for podocytes.
Smooth muscle cells circling the afferent arteriole of the glomerulus are modified and
contain secretory granules of renin. These modified muscle cells are called the
juxtaglomerular cells and are best visualized with histochemical stain for the renin.
Question:
The organ that is seen in the attached image is best identified as

Answer Choices:
A Medullary kidney
B Cortical kidney
C Sweat glands
D Thyroid gland
E Epididymis

Image(s) / Chart(s):
Click image to view full size. Click open image to close. Click and hold open image to move.
A Medullary kidney

This image illustrates a medium power view of the medulla of the kidney. The asterisk
indicates a tubule lined by simple squamous epithelium (Loop of Henle). Other tubules
are seen in various sections with simple cuboidal epithelium. Capillaries are also
observed. No glomeruli are present, which would be indicative of cortical kidney. Sweat
glands are tubular organs, which have a serous (proteinaceous) secretion and stain
more basophilic. The epithelium in the sweat glands would appear "pie" shaped, with a
narrow lumen. The thyroid gland consists of follicles filled with colloid and lined by a
simple squamous, cuboidal or columnar epithelium. The tubules of the epididymis are
lined by pseudostratified columnar epithelium, some of which have stereocilia.

Question:
Which organ is depicted in the attached digitized photomicrograph image?

Answer Choices:
A Pancreas
B Cortical kidney
C Medullary kidney
D Zona glomerulus of adrenal gland
E Parotid gland

Image(s) / Chart(s):
Click image to view full size. Click open image to close. Click and hold open image to move.
B Cortical kidney

Central to the identification of this image is the glomerulus or Bowman's capsule.


Surrounding the glomerulus are tubules cut in a variety of planes such that cross-
sectional, longitudinal and oblique sections are observed. Furthermore, some of these
tubules stain more acidophilic due to substantial numbers of mitochondria located within
the cells, and the lumen appear somewhat filamentous due to the long microvilli (brush
border). These are the proximal convoluted tubules. Other tubules have cells that are
more clear staining, simple cuboidal epithelium characteristic of the distal convoluted
tubules. The combination of these types of tubules and the presence of the glomerulus
indicate this is the cortex of the kidney. The medulla of the kidney, by definition, does
not contain glomeruli. The glomerulus itself can be easily mistaken for a pancreatic islet,
and the tubules mistaken for acini of the pancreas. Also, the tubules of the kidney can
be mistaken for secretory acini of the parotid gland.

Question:
The countercurrent multiplier system in the kidney involves the exchange of water and ions between
the renal interstitium and

Answer Choices:
A The blood in the vasa recta
B The blood in the peritubular capillary network
C The filtrate in the proximal convoluted tubule
D The filtrate in the loop of Henle
E The filtrate in the medullary collecting tubule

D The filtrate in the loop of Henle


The countercurrent multiplier system in the loop of Henle involves ion and water
exchanges between the filtrate and interstitium. It establishes an osmotic gradient in the
interstitium of the medulla, which is greatest at the papilla.
The passive exchange of ion and water between the renal interstitium and the blood in
the vasa recta (the small straight vessels associated with the loop of Henle) is the
countercurrent exchange system. It acts to maintain the interstitial osmotic gradient
created by changes taking place in the filtrate inside the loop of Henle.
In the distal convoluted tubules, active resorption of Na+ ions from the filtrate may occur
(in response to aldosterone), resulting in some water loss.
The blood in the peritubular capillary network, and the filtrate in the medullary collecting
tubule do not influence the countercurrent multiplier system.
Question:
Aldosterone secretion from the adrenal gland acts primarily upon which area of the nephron?

Answer Choices:
A Glomerular basement membrane
B Proximal convoluted tubule
C Thin segment of the loop of Henle
D Afferent arteriole
E Distal convoluted tubule
E Distal convoluted tubule

The distal convoluted tubule (DCT) in the kidney cortex functions to reabsorb Na+ and
secrete K+ . This is regulated by the adrenal hormone, aldosterone, as stimulated by
angiotensin II, which will increase reabsorption of Na+ and secrete K+ . The DCT also
functions in the continuation of bicarbonate ion reabsorption with hydrogen ion
secretion. The terminal portion of the DCT also is sensitive to antidiuretic hormone
(ADH) from the neurohypophysis, which acts to increase the permeability of the tubule
to water, resulting in concentrating the urine.
Question:
Jim Smith, a first year medical student, celebrated passing his first year by attending a class party.
After drinking several glasses of beer, Jim noticed that he was excreting copious amounts of dilute
urine. He knew this was the result of inhibition of one of the following hormones that has an effect on
water absorption on the kidney collecting ducts. Which hormone was inhibited?

Answer Choices:
A Aldosterone
B Angiotensinogen
C Antidiuretic hormone (ADH)
D Angiotensin I
E Androstenedione
C Antidiuretic hormone (ADH)
The distal convoluted tubule (DCT) in the kidney cortex functions to reabsorb Na + and
secrete K+ . This is regulated by the adrenal hormone, aldosterone, as stimulated by
angiotensin II, which will increase reabsorption of Na+ and secrete K+ . The DCT also
functions in the continuation of bicarbonate ion reabsorption with hydrogen ion
secretion. The terminal portion of the DCT is sensitive to antidiuretic hormone (ADH)
from the neurohypophysis, which acts to increase the permeability of the tubule to
water, resulting in concentrating the urine.
The action of ADH is more pronounced on the epithelium of the collecting ducts, such
that in the absence of ADH, a copious, dilute urine is produced. Increased secretion of
ADH has the effect of producing a hypertonic urine. ADH secretion occurs in response
to an increase in plasma osmolality or a decrease in blood volume, which may result
from the loss of water by sweating, vomiting, or diarrhea.
Question:
Constriction of the afferent arteriole to the kidney glomerulus results in

Answer Choices:
A Reduction of glomerular filtration rate and decreased urinary output
B Increased urine output
C Increased glomerular filtration rate and no change in urine output
D Increased glomerular filtration rate and decreased urine output
E No change to the glomerular filtration rate, nor to the urine output

A Reduction of glomerular filtration rate and decreased urinary output

The kidney is innervated primarily by the sympathetic nervous system, and as such,
regulates the contraction of the smooth muscle surrounding the afferent and efferent
arterioles of the glomerulus. Sympathetic stimulation to the efferent glomerular
arterioles causes their constriction, and increases the filtration rate, leading to an
increase in urinary output. Similarly, with the loss of sympathetic innervation, such as
due to sympathectomy of the kidney, relaxation of tone of the afferent arteriole occurs
and the filtration rate increases. This also results in an increased urine output.
Decreased urine output may result from the constriction of the afferent arterioles,
leading to a decrease in glomerular filtration rate.
Question:
The cell bodies for the efferent fibers responsible for the micturition reflex of the urinary bladder are
typically found

Answer Choices:
A In the transitional epithelial layer
B In the lamina propria
C In the submucosa
D In the muscularis
E In the dorsal root ganglia of the sacral portion of the spinal cord

D In the muscularis

Initiation of the micturition reflex occurs upon distension of the wall of the bladder, which
is detected by sensory fibers that project to the sacral portion of the spinal cord. The cell
bodies of these afferent fibers reside in the dorsal root ganglia at these levels.
Upon stimulation, the autonomic motor portion of the micturition reflex is mediated by
efferent parasympathetic fibers which end in ganglia located in the muscle bundles of
the muscularis in the wall of the urinary bladder. These fibers innervate the smooth
muscle which, when stimulated, causes contraction and the evacuation of the bladder.
Postganglionic sympathetic fibers also are found in the urinary bladder, but probably
serve primarily to innervate blood vessels found within the wall of the bladder.
Question:
What type of epithelium lines the membranous portion of the male urethra

Answer Choices:
A Transitional
B Pseudostratified columnar
C Keratinized stratified squamous
D Non-keratinized stratified squamous
E Simple columnar

B Pseudostratified columnar

The male urethra has three segments, some portions of each lined with a different
epithelium.
The portion of the urethra leaving the urinary bladder is the prostatic urethra and
extends 3-4 cm through the prostate gland. It is initially lined with transitional epithelium
until the ejaculatory ducts join the prostatic urethra, whereupon the epithelium changes
to pseudostratified columnar epithelium.
As the urethra passes from the prostate gland through the pelvic diaphragm, there is a
short (1cm) portion referred to as the membranous urethra, lined with pseudostratified
columnar epithelium.
The pseudostratified columnar epithelium continues through the next segment of the
urethra, the penile urethra, which is about 15 cm long. Near the distal end of the
urethra, it becomes lined with non-keratinized stratified squamous epithelium which is
continuous with that of the skin of the head of the penis.
Question:
Identify the structure between the tips of the arrowheads in the photomicroscopic image
Answer Choices:
A Juxtaglomerular cells
B Glomerulus
C Macula densa
D Macula adherens
E Mesangial cells

Image(s) / Chart(s):
Click image to view full size. Click open image to close. Click and hold open image to move.

C Macula densa

The cells located between the tips of the arrowheads represent the cells of the macula
densa. These cells are located in the terminal portion of the distal thick segment of the
nephron and are situated directly adjacent to the afferent and efferent arterioles of the
renal corpuscle. Histologically, the nuclei of these cells are more narrow and appear
crowded along the vascular pole of the glomerulus.
Smooth muscle cells circling the afferent arteriole of the glomerulus are modified and
contain secretory granules of renin. These modified muscle cells are called the
juxtaglomerular cells and are best visualized with histochemical stain for the renin.
Mesangial cells also are found in association with the renal corpuscle; most within the
corpuscle, but some outside of the corpuscle along the vascular pole. The mesangial
cells function as phagocytic cells and provide structural support for podocytes.
The glomerulus is the tuft of specialized capillaries within the renal corpuscle.
The macula adherens is one part of the tight junction that exists between adjacent
epithelial cells.
Question:
Constriction of the efferent arteriole to the kidney glomerulus results in
Answer Choices:
A Reduction of glomerular filtration rate and decreased urinary output
B No change of the glomerular filtration rate, but decreased urine output
C Increased glomerular filtration rate and increased urine output
D Increased glomerular filtration rate and decreased urine output
E No change to the glomerular filtration rate, nor to the urine output
C Increased glomerular filtration rate and increased urine output

The kidney is innervated primarily by the sympathetic nervous system, and as such,
regulates the contraction of the smooth muscle surrounding the afferent and efferent
arterioles of the glomerulus. Sympathetic stimulation to the efferent glomerular
arterioles causes their constriction and increases the filtration rate, leading to an
increase in urinary output. Similarly, with the loss of sympathetic innervation, such as
due to sympathectomy of the kidney, relaxation of tone of the afferent arteriole occurs
and the filtration rate increases, which results in an increased urine output. Decreased
urine output may result from the constriction of the afferent arterioles, leading to a
decrease in glomerular filtration rate.
Question:
What type of epithelium lines the most distal portion of the penile urethra at, or near, its termination?

Answer Choices:
A Transitional
B Pseudostratified columnar
C Keratinized stratified squamous
D Non-keratinized stratified squamous
E Simple columnar

D Non-keratinized stratified squamous

The male urethra has three segments, some portions of each lined with a different
epithelium.
The portion of the urethra leaving the urinary bladder is the prostatic urethra and
extends 3-4 cm through the prostate gland. It is initially lined with transitional epithelium
until the ejaculatory ducts join the prostatic urethra, whereupon the epithelium changes
to pseudostratified columnar epithelium.
As the urethra passes from the prostate gland through the pelvic diaphragm, there is a
short (1cm) portion referred to as the membranous urethra, lined with pseudostratified
columnar epithelium.
The pseudostratified columnar epithelium continues through the next segment of the
urethra, the penile urethra, which is about 15 cm long. Near the distal end of the
urethra, it becomes lined with non-keratinized stratified squamous epithelium which is
continuous with that of the skin of the head of the penis.
Question:
A 64-year-old male is on postoperative day number one following a right hemicolectomy for the
treatment of a colonic lesion. During rounds, you notice that his intravenous fluids have not been
running for the past six hours because the order was not written appropriately. He has not taken
anything orally. His vital signs are as follows: Temperature= 37.5 ° C, Heart Rate = 110, Blood
Pressure 114/66, urine output for the past eight hour is 140 cc.
On physical examination, his mucus membranes are dry. His lungs are clear to auscultation, and his
heart rate is regular by tachycardic. His abdominal exam reveals mild tenderness to palpation. The rest
of his physical exam is unremarkable. What is the most likely diagnosis?

Answer Choices:
A Postrenal azotemia
B Prerenal azotemia
C Renal azotemia
D Third spacing
E Hepatorenal syndrome
B Prerenal azotemia

Volume depletion is common in patients following surgical intervention. This patient has
common signs of volume depletion: increased heart rate, decreased urine output, and
dry mucus membranes.
Magnitude of fluid
Signs and Symptoms
loss
>Mild 10% ECF
mild decrease in urine output
loss
> Moderate 20% Drowsiness, Apathy Marked decrease in urine output Decreased skin turgor
ECF loss Dry mucous membranes Tachycardia Hypotension
> Severe 30% ECF Stupor Coma Anuria Pale, cyanotic, cool skin Sunken eyes Weak pulse
loss Hypotension
Oliguria (decreased urine output) is a reliable sign of volume depletion. However, there
are a few causes of oliguria and they have to be investigated carefully in a patient with
decreased urine output. Oliguria is divided into three categories:
OLIGURIA
Prerenal Azotemia Renal Azotemia Postrenal Azotemia
Oliguria caused by
Oliguria caused by obstruction
decreased renal Oliguria caused by pathology to the
of the urinary system below
perfusion. Common functional units of the kidney, which
the kidneys. Common causes
causes include include some glomerulonephropathy
include: Nephrolithiasis,
hypotension (i.e. shock), and acute tubular necrosis (ATN).
benign prostatic hyperplasia
cardiac causes (i.e. However, renal azotemia is primarily
causing obstruction, or penile
severe CHF), and severe cased by ATN.
strictures.
volume depletion.
Postrenal azotemia can be quickly excluded by insertion of a Foley Catheter, which
would then result rapid and large urine output. Ultrasound may reveal hydronephrosis
(distension of the urinary collecting system by fluid). Once postrenal azotemia has been
excluded, an important distinction of azotemia in the surgical patient is to differentiate
between prerenal and postrenal azotemia.
Distinction of prerenal and renal azotemia can be made based on the fractional
excretion of sodium Fe Na, which can be calculated by the following formula:
Fe Na = Serum Cr X Urine Na X 100
Serum Na X Urine Cr
A value of less than 1% is indicative of prerenal azotemia, while a value greater than
1% is indicative of renal azotemia. Urinary sodium less than 20mEq/L is pathonomic of
decreased tissue perfusion and an excellent test to assess volume status.
The following table illustrates some common causes of volume depletion:
COMMON CAUSES OF VOLUME DEPLETION
SKIN
GI LOSSES THIRD SPACING RENAL
LOSSES
Tissue Injury Trauma Sodium and Water
Vomiting Diarrhea
Surgery Burns Infections Diuretics Osmotic diuresis
Nasogastric suctioning
(peritonitis) Intestinal (severe hyperglycemia)
External Drainage from Burns
obstruction Pleural High salt-wasting renal
fistulas: (bilary, enteric,
effusion Inflammation disease Water Only
pancreatic)
(acute pancreatitis) Diabetes insipidus
Gastrointestinal losses (GI-losses)
GI-losses occur frequently in the surgical patient. These abnormalities are recognized
because they are often associated with acid-base disturbances.
HCl containing secretions
STOMACH
Vomiting, nasogastric decompression → METABOLIC ALKALOSIS
HCO3 Containing secretions
Intestine
Biliary secretion
Pancreatic secretions
Diarrhea
Laxatives
Fistulas
Ostomies
METABOLIC ACIDOSIS
↓ K+
GI-losses resulting in volume depletion can also be due to bleeding. However, bleeding
does not usually result in electrolyte abnormalities.
THIRD SPACING
Third spacing is an important concept that often appears on tests and during rounds.
Third spacing is the loss of fluid from the intravascular space leading to accumulation in
the interstitium or body cavities. Thus, it is a form of volume depletion because the
effective volume in the intravascular space is depleted.
The rate of fluid loss is important. Only those losses for which sodium retention by the
kidney has been unable to compensate the fluid loss are considered third spacing fluid
losses. For example, a patient with ascites may be accumulating fluid in the peritoneum
due to liver disease. However, this accumulation is chronic such that the kidney has had
an opportunity to compensate by increased sodium retention. Thus, the effective
intravascular volume may not be compromised. In this case, ascites may not indicate
volume depletion.
The mediators of third spacing appear to be interleukins, which cause inflammation and
vascular permeability. They have an average half-life of 1-3 days. As the inflammatory
response decreases a few days after surgery and the initial mediators of stress and
third spacing are no longer mediating vascular permeability, it is important to readjust
the amount of fluid provided as iatrogenic fluid overload may occur.
Most fluid losses can be corrected by half normal saline solutions at a rate of 50-100
cc/hr. The rate of fluid maintenance can be roughly estimated by the 4/2/1 rule: 4 cc
fluid per hour for the first 10 kg of body weight, 2cc for the next, then and 1 cc/hr for
each Kg of body weight thereafter.
Question:
A 44-year-old African-American male is on postoperative day number two following a sigmoidectomy
for the treatment of diverticulitis. He keeps asking for his physician because he would like to drink
some water, but he is sill NPO. He is currently receiving intravenous fluid consisting of normal saline
with 5% dextrose and 20 mEq of KCl at 80cc/hr. He complains of no pain. His vital signs are as follows
Temperature= 38.0 ° C, Heart Rate = 140, Blood Pressure 110/68, urine output for the past eight hour
is 100 cc.
On physical examination, his mucus membranes are dry. His lungs are clear to auscultation, and his
heart rate is of a regular rhythm but tachycardic. His abdomen is mildly tender to palpation. His would
is clean, dry and intact. The rest of his physical exam is unremarkable.
The following laboratory data is available:
Hb 14.4 gm/dl
Hct 43.2 %
WBC 9,0000
BUN 28 mg/dl
Creatinine 1.2 mg/dl
The fractional excretion of sodium (FeNa) in this patient is likely to be

Answer Choices:
A Equal to one percent
B Less than one percent
C Greater than one percent
D Unpredictable if the patient does not have a Foley catheter
E Unyielding as this patient is unlikely to have renal disease
B Less than one percent

Distinction of prerenal and renal azotemia can be made based on the fractional
excretion of sodium FeNa, which can be calculated by the following formula:
FeNa = 100 X Serum Cr X Urine Na
Serum Na X Urine Cr
Based on this patient’s low urine output, elevated BUN, and thrist, he is likely
hypovolemic; therefore, his FeNA is most likely to be less than 1%. A value of less than
1% is indicative of prerenal azotemia, while a value greater than 1% is indicative of
renal azotemia. Urinary sodium less than 20mEq/L, is pathoneumonic of decreased
tissue perfusion and an excellent test to assess volume status.

Question:
A 26-year-old Hispanic female just arrived on the surgical ward after an appendectomy and the nurse
has called you to write orders to admit the patient overnight and review some laboratory data recently
ordered by the anesthesiologist.
The following laboratory data is available:
PREOPERATIVE DATA POSTOPERATIVE DATA
Hb 14.4 gm/dl Hb 13.4 gm/dl
Hct 43.2 % Hct 39.9 %
WBC 13,0000 WBC 14,0000
+ +
Na 135 mEq/L Na 130 mEq/L
+ +
K 4.1 mEq/L K 6.4 mEq/L
- -
Cl 110 mEq/L Cl 108 mEq/L
HCO3 22 mEq/L HCO3 18 mEq/L
BUN 12 mg/dl BUN 16 mg/dl
Creatinine 0.8 mg/dl Creatinine 1.0 mg/dl
Electrocardiogram reveals the presence of peaked T waves. Based on this information what should be
the next immediate course of action?

Answer Choices:
A 10 units of insulin intravenously with 1 amp of D50
B Furosemide 10 mg intravenously
C Sodium Bicarbonate intravenously
D Kayexalate 30g/Sorbitol 20%/ 200 cc D5W per mouth
E Calcium Gluconate intravenously
E Calcium Gluconate intravenously

Hyperkalemia is a common electrolyte abnormality in the surgical patient. A drastic


change in potassium from base line has to be diagnosed immediately as hyperkalemia
may result in arrhythmias and cardiac arrest. Thus, the immediate form of action if there
are electrocardiogram changes is to provide calcium intravenously immediately as this
is cardioprotective.
Surgical causes of hyperkalemia
↑ K+ intake
Iatrogenic as in intravenous fluids
Cell shifts
Insufficient insulin
Metabolic acidosis
β-blockers
Blood transfusion
Tissue injury
Impaired excretion
Renal failure
K+ sparing diuretics
Hypoaldosteronism
Clinical manifestations
Hyperkalemic is commonly asymptomatic. Most potassium changes are found by
laboratory analysis.
Arrhythmias
Paresthesias
Weakness
Decreased reflexes
Respiratory failure
EKG Findings
Peaked T waves → Depressed ST segment → ↓ amplitude of R waves → ↑ PR interval
→ small or absent P waves → Widened QRS → sine wave pattern
Treatment
Treatment is divided into two steps depending on EKG changes. If there are EKG
changes, immediate treatment should be cardioprotection with IV calcium followed by
reduction of potassium in the serum. If there are no EKG changes, treatment should be
targeted at reducing the potassium, which can be accomplished in various ways:
Shift K+ into cells
Insulin and glucose-The onset of action is approximately 30 min
Sodium Bicarbonate-The onset of action is approximately 30 min
Albuterol
Elimination of K+ from the body
Sodium polystyrene sulfonate (Kayexalate)-The onset of action is about 2 hours
Furosemide-The onset of action is about 30 minutes
Dialysis (in extremely acute cases)
Sodium Bicarbonate 1 amp = 45mEq
Note insulin and bicarbonate only cause a temporary decrease of serum potassium;
where as Kayexalate and furosemide are definite forms of treatment.
Question:
A 52-year-old Mexican male presents to the emergency department with complaints of acute onset of
abdominal pain that started six hours ago. During routine laboratory analysis, you notice that his
potassium level is 6.4mEq/L. Which of the following if present would prompt immediate intervention?

Answer Choices:
A A previous potassium level of 6.0mEq/L
B Hyperglycemia
C Peaked T waves during electrocardiogram
D An abnormal chest-X ray
E Serum sodium of 135meE/L
C Peaked T waves during electrocardiogram

Treatment of hyperkalemia is divided into two steps depending on EKG changes. If


there are EKG changes, immediate treatment should be cardioprotection with IV
calcium followed by reduction of potassium in the serum. If there are no EKG changes,
treatment should be targeted at reducing the potassium, which can be accomplished in
various ways:
Shift K+ into cells
Insulin and glucose-The onset of action is approximately 30 min
Sodium Bicarbonate-The onset of action is approximately 30 min
Albuterol
Elimination of K+ from the body
Sodium polystyrene sulfonate (Kayexalate)-The onset of action is about 2 hours
Furosemide-The onset of action is about 30 minutes
Dialysis (in extremely acute cases)
Sodium Bicarbonate 1 amp = 45mEq
Note insulin and bicarbonate only cause a temporary decrease of serum potassium;
where as Kayexalate and furosemide are definite forms of treatment.
Since EKG changes would prompt immediate intervention, an EKG would be the first
test to order. Previous elevated serum potassium indicates chronicity and thus
intervention to lower the potassium would not be an immediate concern, but rather
identifying the cause of the chronic high potassium would be most important. A chest X-
ray would not provide any information for immediate treatment.
Question:
A 38-year-old female is on postoperative day number two from an open cholecystectomy. She has had
a nasogastric tube in place with an output of about 30cc/hr. Her urine output has been about 15cc/hr
for the past three hours. What is the most likely acid base disturbance?

Answer Choices:
A Hyperchloremic, hyperkalemic metabolic acidosis
B Hypochloremic, hypokalemic metabolic alkalosis
C Hyperchloremic, hypokalemic metabolic alkalosis
D Hypochloremic, hypokalemic metabolic acidosis
E Hyperchloremic, hyperkalemic, metabolic alkalosis

B Hypochloremic, hypokalemic metabolic alkalosis

This scenario illustrates the typical presentation of a patient with contraction alkalosis.
This concept refers to a patient who is volume depleted from fluid loss containing HCl
(fluid losses from the stomach as in nasogastric decompression or persistent vomiting).
With inappropriate fluid replacement, a patient is at risk of developing hypochloremic,
hypokalemic, metabolic alkalosis. Because of volume depletion, the kidney will retain
sodium in an attempt to maintain volume. Although there is metabolic alkalosis, the
kidney will excrete hydrogen ions in exchange for the sodium instead of potassium
because hypokalemia would be more deleterious to the body than the metabolic
alkalosis already in place. This concept is known as paradoxical aciduria.
Question:
A 26-year-old Asian male is two days status post perforated appendectomy. He has been doing well
and you are ready to advance his diet. He is delighted to be able to drink some water because he has
been very thirsty for the past few hours. What effector most likely will be responsible for his thirst?

Answer Choices:
A Antidiuretic hormone (ADH)
B Cortisol
C Atrial natruretic peptide
D Aldosterone
E Epinephrine

A Antidiuretic hormone (ADH)

Sodium is the principal osmol of the extracellular space; it acts to hold water particles in
this compartment. Plasma osmolality plays the primary role in the regulation
of intracellular volume. This is achieved by homeostasis of water and is important for
normal cellular functioning. Plasma sodium plays the primary role in the regulation
of plasma volume. Thus, sodium homeostasis is essential for tissue perfusion. Volume
and water homeostasis respond to different stimuli, act on different receptors and
respond differently to restore either euvolia or water balance.
These mechanisms can work together or independently. For example, A patient who
receives an infusion of isotonic saline responds to volume changes but not to osmolality
changes. This patient compensates for volume increase by releasing atrial natriuretic
peptide, resulting in increased secretion of Na+ by the kidney. However, because
osmolality has not been altered, neither thirst nor ADH secretion is activated. On the
other hand, a patient who receives an infusion of hypotonic saline responds both to
volume and osmolality changes. The increase in volume results in Na+ secretion. The
hypoosmolality induced by hypotonic saline suppresses ADH and thirst resulting in the
excretion of free water into the urine.
The following table illustrates both of these systems:
CHANGE RECEPTOR EFFECTORS RESPONSE
VOLUME Cardiopulmonary Aldosterone GFR Atrial EUVOLEMIA
Carotid Juxtaglomerular natruretic peptide (ANP)
OSMOLALITY NORMAL
Hypothalamus Thirst ADH
OSMOLALITY
+
The primary aim of the effectors is to affect Na re-absorption and secretion by the
kidney.
Increased volume → ANP → ↑ Na+ secretion → Euvolemia
Decreased volume → Aldosterone → ↑ Na+ re-absorption → Euvolemia
The sensors for osmolality are located in the hypothalamus and respond to small
changes in plasma osmolality to keep it at a narrow window between 275-290m Osmol.
Persistent changes are rarely seen due to the ability of the kidney to secrete large
amounts of water or retain it by ADH secretion. Osmolality changes may be seen in
renal impaired or comatose patients.
Case #301468:
The patient is a 52-year-old male with polycystic kidney disease. Despite a strict diet,
his symptoms of fatigue, pruritus, and headache recently have gotten worse, and he
has episodes of hematuria and urinary tract infections almost every month. He has
gained several pounds during the last week despite a low appetite. On examination,
patient is edematous, has uremic fetor, sallow colored skin with "uremic frost," and
slightly icteric sclera. His blood pressure is 190/120. Laboratory shows normochromic
normocytic anemia, metabolic acidosis, and the imbalance of potassium, sodium, and
chlorides, with increased levels of calcium, phosphate, and magnesium, and with
reduced bicarbonate. Blood urea nitrogen (BUN) and creatinine levels are elevated.
Urinalysis reveals the presence of proteinuria, hematuria, bacteriuria, and
leucocyturia. His glomerular filtration rate (GFR) is 14 mL/min/1.73 square meters.
This patient should start dialysis.
Question:
In addition to dialysis, what substance should be administered to this patient?

Answer Choices:
A Erythropoietin
B Phosphate
C Salt
D Globulin
E Parathormone
A Erythropoietin

All presented data lead to the conclusion that this patient has end-stage renal disease
(ESRD). He has anemia because he lacks erythropoietin. Erythropoietin is synthesized
by renal peritubular cells in adults, with a small amount being produced in the liver.
Regulation is believed to rely on a feed-back mechanism measuring blood oxygenation.
Erythropoietin's primary effect is on red blood cells: to protect them from apoptosis and
to help various growth factors in the development of precursor red cells. Therefore,
erythropoietin is used most commonly to treat anemia in people with anemia due to
chronic kidney disease, in people on dialysis, and in people living with a kidney
transplant.

Your patient has hyperphosphatemia because of decreased excretion of phosphate.


Once renal insufficiency progresses to the loss of 40-50% of renal function, the
decrease in the amount of functioning renal tissue does not allow excretion of the full
amount of ingested phosphate required to maintain homeostasis, and
hyperphosphatemia develops.
Salt should not be substituted. Your patient has high blood pressure. Regulating blood
pressure is linked to the kidneys' ability to excrete enough sodium chloride to maintain
normal sodium balance, extracellular fluid volume, and blood volume. All patients with
ESRD, even on dialysis, should be on a low sodium diet because high blood pressure
can cause more damage that will further reduce kidney function, resulting in even more
fluid and waste build up in the body. Many patients on dialysis can even control blood
pressure through a low-sodium diet without drugs.

Although your patient most probably has hypoproteinemia secondary to renal loss and
due to a low protein diet, he has hypoalbuminemia and normal or elevated globulin
levels (globulins are too big to be lost through the kidneys). There is no need to
substitute globulins in ESRD.

Kidney failure is a common cause of secondary hyperparathyroidism. Kidney failure can


interfere with the body's ability to remove phosphate, resulting in hyperphosphatemia.
Hypocalcemia is the consequence of hyperphosphatemia (and the deficiency in calcitriol
normally produced in kidneys). Hypocalcemia leads to secondary hyperparathyroidism.
Your patient probably has secondary hyperparathyroidism and does not need
parathormone substitution.

References:
Wysolmerski JJ, Insogna KL. The parathyroid glands, hypercalcemia, and
hypocalcemia. In: Kronenberg HM, Schlomo M, Polansky KS, Larsen PR, eds. Williams
Textbook of Endocrinology. 11th ed. St. Louis, Mo: WB Saunders; 2008:chap. 266.
National Kidney Foundation. Clinical practice guidelines for nutrition in chronic renal
failure. K/DOQI, National Kidney Foundation. Am J Kidney Dis. 2000;35(6 Suppl 2):S1-
S140.
Mehorta R, Kopple JD:. Nutritional management of maintenance dialysis patients: why
aren't we doing better? Annu Rev Nutr. 2001;21:343-379.

Case #301595:
A 43-year-old Caucasian woman presents with a 1-day history of burning sensation
during urination and foul-smelling urine. She says that she has the habit of frequently
voiding her bladder about once every 2-3 hours. She is hypertensive and on
treatment with thiazides. She is sexually active with 1 partner, her husband, and he
uses a condom for contraception. Fluid intake and blood glucose are normal.
Leukocyte esterase dipstick test is positive, and urinalysis reveals 10 WBC/ml and
bacteria.
Question:
What is a risk factor for this patient’s condition?

Answer Choices:
A Length of the urethra
B Repeated voiding of the bladder
C Condom use
D Her hypertension
E Treatment with thiazides

A Length of the urethra

The correct answer is the length of the urethra.

The diagnosis in this patient is a urinary tract infection (UTI). The female urethra is
shorter, about 4 cm, compared to the male urethra, which is 22.3 cm (mean) (SD 2.4
cm) in length. The short length favors easy ascent of uropathogens to the bladder.
Other risk factors for UTI in women include recent sexual activity, urinary obstruction,
instrumentation, uterine or bladder prolapse, etc.

Repeated and complete voiding of the bladder has been shown to reduce the risk of
UTI. Diaphragm and spermicide use, not condom use, is associated with higher risk of
UTI. Hypertension and thiazides are not known to be associated with UTI.
Question:BLOCK -2
The epithelium in the upper part of this photomicrograph is MOST likely found in what structure?

Answer Choices:
A ureter
B penile urethra
C vaginal urethra
D visceral layer of Bowman’s capsule
E parietal layer of Bowman’s capsule
F proximal convoluted tubule
G papillary duct
H fossa navicularis of penis

Image(s) / Chart(s):
Click image to view full size. Click open image to close. Click and hold open image to move.
A ureter

This is an example of transitional epithelium in the ureter, a peculiar stratified epithelium


that is found only in the urinary tract. The renal pelves, the renal calyces, the ureters,
the urinary bladder, and in some cases, the membranous urethra has this kind of
epithelium. Especially in the urinary bladder, this epithelium can accommodate dramatic
changes in surface area with distention and emptying. The transitional epithelium of an
empty bladder is many cell layers thick and has rounded, “pillowy” apical cells. As the
bladder fills with urine, the apical cells flatten considerably and the number of cell layers
is reduced.

The penile urethra has stratified cuboidal or stratified columar epithelium. The vaginal
urethra and the fossa navicularis both have stratified squamous unkeratinized
epithelium. The visceral layer of Bowman’s capsule is a simple squamous epithelium.
The parietal layer of Bowman’s capsule is a layer of podocytes. The proximal
convoluted tubule has a simple cuboidal epithelium with a prominent apical microvillous
brush border. Papillary ducts (of Bellini) have a simple columnar epithelium.
Question:
The scanning electron micrograph below is taken from a fractured specimen of the renal cortex. What
are the most likely cells shown?

Answer Choices:
A cuboidal epithelial cells in proximal convoluted tubule
B juxtaglomerula cells in wall of afferent arteriole
C macula densa cells in distal convoluted tubule
D podocytes in visceral layer of Bowman’s capsule
E epithelial cells in collecting ducts

Image(s) / Chart(s):
Click image to view full size. Click open image to close. Click and hold open image to move.
D podocytes in visceral layer of Bowman’s capsule
Question: BLOCK-2
The scanning electron micrograph below is taken from a fractured specimen of the renal cortex. What
is the most appropriate description of the function of these cells?

Answer Choices:
A forming a portion of the selective barrier for production of glomerular filtrate
B resorption of water under the influence of antidiuretic hormone
C resorption of proteins and nutrients from glomerular filtrate
D protection of underlying tissue from acidic urine
E contraction propels urine toward urinary bladder

Image(s) / Chart(s):
Click image to view full size. Click open image to close. Click and hold open image to move.
A forming a portion of the selective barrier for production of glomerular filtrate

This is a scanning electron micrograph of the visceral layer of Bowman’s capsule,


viewed from the perspective of the urinary space, i.e., the parietal layer of Bowman’s
capsule (a simple squamous epithelium) has been fractured away. Podocytes are
reminiscent of the shape of an octopus, with numerous primary, secondary, and tertiary
branches on its “arms.” The branches between adjacent cells are extensively
interdigitated. The tertiary branches are called pedicles or foot processes. In cross
sections of the barrier between the vascular space and the urinary space, slit
diaphragms bridge the small gaps between adjacent pedicles. Along with the glomerular
basement membrane, these slit diaphragms serve as a selective filtration barrier. In
experimental studies, high molecular weight proteins in blood are trapped at the
glomerular basement membrane. Intermediate molecular weight proteins are trapped at
the slit diaphragms. Low molecular weight proteins pass into the glomerular filtrate
entering the urinary space.
Epithelial cells in the collecting ducts are simple cuboidal. These cells are sensitive to
antidiuretic hormone (ADH), which is secreted by the neurohypophysis. ADH increases
the water permeability of collecting ducts and tubules, resulting in the formation of a
small volume of concentrated urine. Cuboidal epithelial cells in the proximal convoluted
tubule have a prominent apical microvillous brush border. These cells are responsible
for recovery of blood proteins and nutrients that make their way into the glomerular
filtrate. Once urine is formed, it is emptied into passages lined by transitional epithelium,
which protects underlying tissues from damage by acidic urine. Smooth muscle cells in
the walls of these passages produce waves of peristalsis to convey urine from the
kidneys, down the ureters, to the urinary bladder.

Question: BLOCK 2
The light micrograph below is taken from the normal human renal cortex. Study the micrograph, identify
the structure marked with the letter L (for its lumen). What is the best description of the function of this
structure?

Answer Choices:
A forming a portion of the selective barrier for production of glomerular filtrate
B resorption of water under the influence of antidiuretic hormone
C resorption of proteins and nutrients from glomerular filtrate
D protection of underlying tissue from acidic urine
E contraction propels urine toward urinary bladder

Image(s) / Chart(s):
Click image to view full size. Click open image to close. Click and hold open image to move.
C resorption of proteins and nutrients from glomerular filtrate

This is a light micrograph of a portion of the normal human renal cortex. The circular
structure with an L in its lumen is a proximal convoluted tubule (PCT). The glomerular
filtrate passes directly from the urinary space of Bowman’s capsule into the lumen of the
PCT. Here, simple cuboidal epithelial cells have several features that reflect the
important resorptive role of the PCT:
Abundant apical microvilli assembled in a brush border increase the surface area of
epithelial cells, promoting resorption.
Numerous mitochondria give the cytoplasm of these cells an acidophilic (stains red with
eosin dye) character. These organelles produce the ATP used to power active transport
of constituents from the lumen through the epithelial cells and into the capillaries that
surround the PCT, returning protein molecules, nutrient, and ions to the blood from the
glomerular filtrate.
Deep infoldings of the basal surfaces of these cells also increase cell surface area for
transport.
Numerous cytoplasmic vesicles, undoubtedly involved in endocytic bulk, transport of
constituents of the glomerular filtrate.

In the renal corpuscle, the glomerular basement membrane and the slit diaphragms
between podocyte pedicles constitute the selective filtration barrier. Epithelial cells in
the collecting ducts and tubules are simple cuboidal. These cells are sensitive to
antidiuretic hormone (ADH), which is secreted by the neurohypophysis. ADH increases
the water permeability of collecting ducts and tubules, resulting in the formation of a
small volume of concentrated urine. Podocytes form part of the selective barrier (along
with the glomerular basement membrane). Once urine is formed, it is emptied into
passages lined by transitional epithelium, which protects underlying tissues from
damage by acidic urine. Smooth muscle cells in the walls of these passages contract,
producing waves of peristalsis to convey urine from the kidneys, down the ureters, to
the urinary bladder.
Question:
What structure in the light micrograph of the renal cortex below is the site of most of the reabsorption of
proteins from the glomerular filtrate?

Answer Choices:
A A
B B
C C
D D
E E

Image(s) / Chart(s):
Click image to view full size. Click open image to close. Click and hold open image to move.
A A

The proximal convoluted tubule (A) is a dominant feature of the renal cortex. It contains
cuboidal epithelial cells with a prominent apical border of microvilli and numerous
cytoplasmic mitochondria. The glomerular filtrate (which empties into space E) contains
some blood proteins and low molecular weight nutrients. Most of these are reabsorbed
in the proximal convoluted tubule. By the time the glomerular filtrate enters the distal
convoluted tubule (B), most of the protein and nutrients have been reabsorbed. Blood
enters the renal glomerulus, which is covered by a layer of podocytes (D), from the
afferent arteriole (C).
Question: BLOCK-2
What structure in the light micrograph of the renal cortex below has modified cells in its wall that are
simple columnar rather than simple cuboidal at the place where it meets the vascular pole of the renal
corpuscle?

Answer Choices:
A A
B B
C C
D D
E E

Image(s) / Chart(s):
Click image to view full size. Click open image to close. Click and hold open image to move.
B B

The distal convoluted tubule near the vascular pole of the renal corpuscle has columnar
cells in its wall rather than cuboidal cells. This is the macula densa, a component of the
juxtaglomerular apparatus (JGA). The juxtaglomerular apparatus (JGA) in the kidney is
located immediately adjacent to the vascular pole of the renal corpuscle. It contains 3
main components:

Juxtaglomerula cells, which are modified smooth muscle cells in the wall of primarily the
afferent arteriole (C), but secondarily in the wall of the efferent arteriole. These contain
granules of renin, a proteolytic enzyme, which when released into blood in response to
decreases in blood pressure, causes the hydrolysis of angiotensinogen to angiotensin I.
Subsequently, the pulmonary angiotensin converting enzyme converts angiotensin I into
angiotensi II, a potent vasoconstrictor. This results in an increase in blood pressure.
The macula densa is a discoid patch of tall columnar cells in the wall of the distal
convoluted tubule (B). The rest of the distal convoluted tubules is lined by simple
cuboidal epithelium with a modest apical brush border of microvilli. Cells in the macula
densa monitor the chemical composition of the urine in their lumen.
Mesangial cells surround the glomerular capillaries. In the vascular pole, they link the
juxtaglomerular cells to the cells in the macula densa via long cellular processes and
gap junctions. They coordinate the physiological responses of the other 2 kinds of cell in
the JGA.

The proximal convoluted tubule (A) joins Bowman’s capsule at the urinary pole of the
renal corpuscle. It is opposite the vascular pole and is not part of the JGA. However, its
cuboidal epithelial cells have a prominent microvillous brush border. Most of the
reabsorption of proteins, amino acids, and sugars in the glomerular filtrate occurs in the
proximal convoluted tubule.

The podocytes (D) are odd epithelial cells that make up the visceral layer of Bowman’s
capsule. They have numerous interdigitating cellular processes. Podocytes, along with
the glomerular basement membrane, are important in regulation of the passage of blood
constituents into the glomerular filtrate, which enters the urinary space (E).
Question:
Which structure in the light micrograph of the renal cortex below has a peculiar epithelial cell with
numerous branched processes? It is important in the regulation of the passage of proteins from the
vascular space to the urinary space.

Answer Choices:
A A
B B
C C
D D
E E

Image(s) / Chart(s):
Click image to view full size. Click open image to close. Click and hold open image to move.

D D

Podocytes (D) are an odd collection of epithelial cells in the visceral layer of Bowman’s
capsule. They have numerous interdigitating primary, secondary, and tertiary
cytoplasmic processes radiating from the cell body. Tight juxtaposition between tertiary
processes (pedicles, foot processes) are important, along with the glomerular basement
membrane, in regulation of passage of blood constituents into the urinary space (E).
Once the glomerular filtrate enters the proximal convoluted tubule (A), at the urinary
pole of the renal corpuscle, most of the proteins and nutrients in the glomerular filtrate
are reabsorbed here. The distal convoluted tubule (B) has a macula densa at the
vascular pole of the renal corpuscle, where, along with the afferent arteriole (C), it forms
part of the juxtaglomerular apparatus, which is important in regulation of blood pressure
and blood/urine composition.
Question:
What structure in the light micrograph of the renal cortex below contains granules of renin in its mural
cells?

Answer Choices:
A A
B B
C C
D D
E E

Image(s) / Chart(s):
Click image to view full size. Click open image to close. Click and hold open image to move.

C C
The juxtaglomerular apparatus (JGA) in the kidney is located immediately adjacent to
the vascular pole of the renal corpuscle. It contains 3 main components:
Juxtaglomerula cells, which are modified smooth muscle cells in the wall of primarily the
afferent arteriole (C), but secondarily in the wall of the efferent arteriole. These contain
granules of renin, a proteolytic enzyme, which when released into blood in response to
decreases in blood pressure, causes the hydrolysis of angiotensinogen to angiotensin I.
Subsequently, the pulmonary angiotensin converting enzyme converts angiotensin I into
angiotensin II, a potent vasoconstrictor. This results in an increase in blood pressure.
The macula densa is a discoid patch of columnar cells in the wall of the distal
convoluted tubule (B). The rest of the distal convoluted tubules is lined by simple
cuboidal epithelium with a modest apical brush border of microvilli. Cells in the macula
densa monitor the chemical composition of the urine in their lumen.
Mesangial cells surround the glomerular capillaries. In the vascular pole, they link the
juxtaglomerular cells to the cells in the macula densa via long cellular processes and
gap junctions. They coordinate the physiological responses of the other 2 kinds of cell in
the JGA.

The proximal convoluted tubule (A) joins Bowman’s capsule at the urinary pole of the
renal corpuscle. It is opposite the vascular pole and is not part of the JGA. However, its
cuboidal epithelial cells have a prominent microvillous brush border. Most of the
reabsorption of proteins, amino acids, and sugars in the glomerular filtrate occurs in the
proximal convoluted tubule. Podocytes (D) are important in regulation of the
composition of the glomerular filtrate, which enters the urinary space (E) before passing
into the proximal convoluted tubule (A).

SECTION- FEMALE REPRODUCTIVE


SYSTEM
Question:
Two large, longitudinal folds of skin that possess hair on their outer surface, but are devoid of hair on
their inner surface and extend posteriorly from the pubic symphysis in the female, are referred to as the

Answer Choices:
A Mons pubis
B Labia majora
C Labia minora
D Clitoris
E Vestibule
B Labia majora

The mons pubis is the prominence of subcutaneous tissue covered by stratified


squamous epithelium overlying the female pubic symphysis. The labia majora are two
large folds of skin that extend posteriorly from the mons pubis. These form the lateral
boundaries of the vaginal opening and are covered with hair on the exterior surface, but
are devoid of hair on the smooth inner surface. Sebaceous and sweat glands are
associated with each surface and the folds contain various amounts of subcutaneous
adipose tissue.
The labia majora are homologous to the skin of the scrotum in the male. The labia
minora are smaller, paired folds of skin that underlie the labia majora and boarder the
vaginal vestibule. These hairless folds of skin have a core of richly vascularized
connective tissue and possess large sebaceous glands.
The clitoris is homologous to the penis and consists of erectile tissue covered by
stratified squamous epithelium. Similar to the structure of the penis, the body of the
clitoris is composed of paired erectile bodies referred to as corpora cavernosa. The
clitoris ends with a small, round tubercle of tissue called the glans clitoris. The vestibule
represents the opening of the vagina and is lined with stratified squamous epithelium.
Vestibular glands are present in the underlying connective tissue, and provide mucus
secretions near the vaginal opening.
Question:
Subcutaneous tissue forms a rounded prominence over the female pubic symphysis. This structure is
referred to as the

Answer Choices:
A Mons pubis
B Labia majora
C Labia minora
D Clitoris
E Vestibule

A Mons pubis

The mons pubis is the prominence of subcutaneous tissue covered by stratified


squamous epithelium overlying the female pubic symphysis. The labia majora are two
large folds of skin that extend posteriorly from the mons pubis. These form the lateral
boundaries of the vaginal opening and are covered with hair on the exterior surface, but
are devoid of hair on the smooth inner surface. Sebaceous and sweat glands are
associated with each surface and the folds contain various amounts of subcutaneous
adipose tissue.
The labia majora are homologous to the skin of the scrotum in the male. The labia
minora are smaller, paired folds of skin that underlie the labia majora and boarder the
vaginal vestibule. These hairless folds of skin have a core of richly vascularized
connective tissue and possess large sebaceous glands.
The clitoris is homologous to the penis and consists of erectile tissue covered by
stratified squamous epithelium. Similar to the structure of the penis, the body of the
clitoris is composed of paired erectile bodies referred to as corpora cavernosa. The
clitoris ends with a small, round tubercle of tissue called the glans clitoris. The vestibule
represents the opening of the vagina and is lined with stratified squamous epithelium.
Vestibular glands are present in the underlying connective tissue, and provide mucus
secretions near the vaginal opening.
Question:
Where in the female reproductive tract does the transition of epithelium from simple cuboidal to
stratified squamous typically occur?

Answer Choices:
A Internal os of the cervix
B Vestibule of the vaginal canal
C External os of the cervix
D Cervical canal
E Base of the uterus

Question:
Identify the organ as depicted in the attached photomicroscopic image.

Answer Choices:
A Cervix
B Uterus
C Uterine tube
D Efferent ducts
E Gall bladder

Image(s) / Chart(s):
Click image to view full size. Click open image to close. Click and hold open image to move.

B Uterus

This organ is identified as an image of a uterus, demonstrating part of the endometrium.


The stratum basalis (A) and part of the stratum functionalis (B) show well-developed
uterine glands. A small part of the myometrium can also be identified. The cervical
mucosa does not change in thickness significantly during the uterine cycle and
possesses large branched glands, which may become dilated as cysts. The uterine tube
has a much thinner but highly folded mucosa and a prominent muscularis showing an
inner circular and outer longitudinal layer. The efferent ducts of the male are tubular
structures possessing alternating tall cuboidal or columnar epithelium and cuboidal
epithelium. The gall bladder is a hollow organ having an extensively branched mucosa.
There are no glands in the mucosa of the gall bladder.
Image(s) / Chart(s):
Click image to view full size. Click open image to close. Click and hold open image to move.

Question:
Your technician failed to label the source of a tubular organ taken from an adult patient. The attached
photomicroscopic image depicts the resulting histological appearance of a cross-section through that
organ. The organ is best identified as

Answer Choices:
A Uterine tube
B Vagina
C Vas deferens
D Duodenum
E Ureter

Image(s) / Chart(s):
Click image to view full size. Click open image to close. Click and hold open image to move.
A Uterine tube

Inspection of the attached photomicroscopic image reveals the tissue has a highly
folded mucosa with a lamina propria core covered by simple columnar epithelium, some
of the cells appearing ciliated. Underlying the mucosa is a thick inner circular layer of
smooth muscle and a outer longitudinal layer of smooth muscle. The outermost layer is
not present, which does not allow for the determination of the presence of a serosa or
adventitia. Nevertheless, these characteristics should provide sufficient evidence to
identify this organ as a cross-section of the uterine tube. The vagina is covered by
stratified squamous epithelium, eliminating that organ as a possibility. The epithelium of
the vas deferens is pseudostratified columnar epithelium with stereocilia. While the
duodenum does possess simple columnar epithelium, a muscularis mucosa is present
forming a submucosa. Typically, glands may be found in the submucosa of the
duodenum (Brunner's glands), which are not found in this photomicroscopic image. The
epithelium of the ureter is transitional, eliminating that organ as a possibility in this
image.
Question:
Increased levels of progesterone would stimulate the production of which cell type, in the mucosal
lining of the uterine tube?
Answer Choices:
A Simple columnar ciliated cells
B Simple columnar peg cells
C Pseudostratified columnar ciliated cells
D Pseudostratified columnar peg cells
E Goblet cells

B Simple columnar peg cells

The mucosal epithelium of the uterine tube is simple columnar, composed of two cell
types, ciliated cells and peg cells (non-ciliated). During the follicular phase, the estrogen
stimulates ciliogenesis and the number of ciliated cells increases. Therefore, the simple
columnar ciliated cells predominate during this phase. During the luteal phase of the
ovary, progesterone increases the number of secretory cells, and the ratio of ciliated to
non-ciliated peg cells changes towards the peg cells. Under the influence of
progesterone, the number of peg cells increase (with a decrease in ciliated cells) with
an increase in the production of oviductal fluid. Goblet cells are not normally found in
the uterine tube.
Question:
What cell type predominates the mucosal lining of the uterine tube during the luteal phase of the
ovarian cycle?

Answer Choices:
A Simple columnar ciliated cells
B Simple columnar peg cells
C Pseudostratified columnar ciliated cells
D Pseudostratified columnar peg cells
E Goblet cells
B Simple columnar peg cells

The mucosal epithelium of the uterine tube is simple columnar, composed of two cell
types, ciliated cells and peg cells (non-ciliated). During the follicular phase, the estrogen
stimulates ciliogenesis and the number of ciliated cells increases. Therefore, the simple
columnar ciliated cells predominate during this phase. During the luteal phase of the
ovary, progesterone increases the number of secretory cells, and the ratio of ciliated to
non-ciliated peg cells changes towards the peg cells. Under the influence of
progesterone, the number of peg cells increase (with a decrease in ciliated cells) with
an increase in the production of oviductal fluid. Goblet cells are not normally found in
the uterine tube.
Question:
What cell type predominates the mucosal lining of the uterine tube during the follicular phase of the
ovarian cycle?

Answer Choices:
A Simple columnar ciliated cells.
B Simple columnar peg cells.
C Pseudostratified columnar ciliated cells.
D Pseudostratified columnar peg cells.
E Goblet cells.

A Simple columnar ciliated cells.

The mucosal epithelium of the uterine tube is simple columnar, composed of two cell
types, ciliated cells and peg cells (non-ciliated). During the follicular phase, the estrogen
stimulates ciliogenesis and the number of ciliated cells increases. Therefore, the simple
columnar ciliated cells predominate during this phase. During the luteal phase of the
ovary, progesterone increases the number of secretory cells, and the ratio of ciliated to
non-ciliated peg cells changes towards the peg cells. Under the influence of
progesterone, the number of peg cells increase (with a decrease in ciliated cells) with
an increase in the production of oviductal fluid. Goblet cells are not normally found in
the uterine tube.
Question:
The longest segment of the uterine tube is referred to as the

Answer Choices:
A Fimbriae
B Ampulla
C Isthmus
D Infundibulum
E Intramural
B Ampulla

The uterine tube may be divided into four segments. The most distal portion, which is
funnel-shaped and most closely associated with the ovary, is the infundibulum. Finger-
like projections of the most distal portion of the infundibulum extend toward the ovary
and are called fimbriae. The ampulla comprises approximately 2/3 of the length of the
uterine tube and is regarded as the longest segment of the tube. Immediately adjacent
to the uterus, the uterine tube becomes narrowed and is called the isthmus. The portion
of the uterine tube that extends through the uterine wall is the intramural segment. This
portion opens into the uterine lumen.
Question:
The segment of the uterine tube located within the uterine wall is referred to as the
Answer Choices:
A Fimbriae
B Ampulla
C Isthmus
D Infundibulum
E Intramural
E Intramural

The uterine tube may be divided into four segments. The most distal portion, which is
funnel-shaped and most closely associated with the ovary, is the infundibulum. Finger-
like projections of the most distal portion of the infundibulum extend toward the ovary
and are called fimbriae. The ampulla comprises approximately 2/3 of the length of the
uterine tube and is regarded as the longest segment of the tube. Immediately adjacent
to the uterus, the uterine tube becomes narrowed and is called the isthmus. The portion
of the uterine tube that extends through the uterine wall is the intramural segment. This
portion opens into the uterine lumen.
Question:
The funnel-shaped segment of the uterine tube located adjacent to the ovary is referred to as the

Answer Choices:
A Fimbriae
B Ampulla
C Isthmus
D Infundibulum
E Intramural
Question:
Atresia of a developing follicle may form which one of the following structures?

Answer Choices:
A Theca lutein cells
B Granulosa lutein cells
C Corpora lutea atretica
D Corpus albicans
E Ovarian hilar cells

C Corpora lutea atretica

Immediately following ovulation, the cells of the mature follicle are transformed into a
corpus luteum. This ovarian structure persists for only 14 days following ovulation. The
follicular cells become the granulosa lutein cells and are associated with increased
secretion of progesterone. The theca interna cells become the theca lutein cells that are
more peripherally located on the corpus luteum. The corpus albicans is formed after
degeneration of the corpus luteum, in which the granulosa lutein cells undergo autolysis
and a hyaline material accumulates within the cellular remnants of the former corpus
luteum. It will eventually blend into the stromal tissue of the ovary. The ovarian hilar
cells are cells found in the hilum of the ovary and may be androgen secreting cells.
Atretic follicles, particularly those that begin the process of atresia at a later stage of
follicular development, may resemble an old corpus luteum and are referred to as a
corpora lutea atretica.
Question:
Cells found in the interstitium of the ovary that resemble steroid secreting cells and may secrete
androgens in the human ovary are

Answer Choices:
A Theca lutein cells
B Granulosa lutein cells
C Corona radiata cells
D Corpus albicans
E Ovarian hilar cells
E Ovarian hilar cells

Immediately following ovulation, the cells of the mature follicle are transformed into a
corpus luteum. This ovarian structure persists for only 14 days following ovulation. The
follicular cells become the granulosa lutein cells and are associated with increased
secretion of progesterone. The theca interna cells become the theca lutein cells that are
more peripherally located on the corpus luteum. The cells of the corona radiata remain
with the mature oocyte, at least until fertilization occurs. The corpus albicans is formed
after degeneration of the corpus luteum, in which the granulosa lutein cells undergo
autolysis and a hyaline material accumulates within the cellular remnants of the former
corpus luteum. It will eventually blend into the stromal tissue of the ovary. The ovarian
hilar cells are cells found in the hilum of the ovary and may be androgen secreting cells.

Question:
For approximately 10 days following ovulation, the follicular cells become the

Answer Choices:
A Theca lutein cells
B Granulosa lutein cells
C Corona radiata cells
D Corpus albicans
E Ovarian hilar cells

B Granulosa lutein cells

Immediately following ovulation, the cells of the mature follicle are transformed into a
corpus luteum. This ovarian structure persists for only 14 days following ovulation. The
follicular cells become the granulosa lutein cells and are associated with increased
secretion of progesterone. The former theca interna cells become the theca lutein cells
and are more peripherally located on the corpus luteum. The cells of the corona radiata
remain with the mature oocyte, at least until fertilization occurs. The corpus albicans is
formed after degeneration of the corpus luteum, in which the granulosa lutein cells
undergo autolysis and a hyaline material accumulates within the cellular remnants of the
former corpus luteum. It will eventually blend into the stromal tissue of the ovary. The
ovarian hilar cells are cells found in the hilum of the ovary and may be androgen
secreting cells.
Question:
Fertilization normally occurs in what portion of the female reproductive system?

Answer Choices:
A Vagina
B Uterus
C Uterine tube
D Peritoneal cavity
E Ovary
C Uterine tube

Fertilization normally occurs in the ampulla of the uterine tube. Ova released into the
peritoneal cavity may fail to be fertilized, and will generally degenerate. Fertilization may
occur in the peritoneal cavity, which can result in ectopic implantation on the surface of
the ovary or inside the rectouterine pouch. Implantation may also occur on the surface
of the mesenteries surrounding the intestines, or even within the lining of the uterine
tube. Deposition of sperm occur in the vagina, which travel through the cervix into the
uterus and find their way to the uterine tube.
Question:
The site upon the surface of the ovary overlying the bulging follicle where ovulation will occur is termed
the

Answer Choices:
A Zona pellucida
B Corona radiata
C Theca interna
D Cumulus oophorus
E Macula pellucida
E Macula pellucida

The large antrum of the mature follicle fills the central part of the cavity pushing several
layers of granulosa cells to the periphery of the follicle. In one region the cells will form a
pedestal supporting the oocyte. This pedestal of cells is referred to as the cumulus
oophorus, projecting the oocyte into the antrum. Granulosa cells immediately
surrounding the oocyte, external to the zona pellucida, are the cells of the corona
radiata, which remain with the oocyte following ovulation. The theca interna cells are
hormone secreting cells surrounding the follicle underlying the basement membrane of
the follicle. The macula pellucida or stigma, is a small area on the surface of the ovary
overlying the bulge caused by the mature follicle. This small, elevated area of germinal
epithelium will be the site of rupture at ovulation.
Question:
Granulosa cells that immediately surround the oocyte in the mature follicle are referred to as the

Answer Choices:
A Zona pellucida
B Corona radiata
C Theca interna
D Cumulus oophorus
E Macula pellucida
B Corona radiata

The large antrum of the mature follicle fills the central part of the cavity pushing several
layers of granulosa cells to the periphery of the follicle. In one region the cells will form a
pedestal supporting the oocyte. This pedestal of cells is referred to as the cumulus
oophorus, projecting the oocyte into the antrum. Granulosa cells immediately
surrounding the oocyte, external to the zona pellucida, are the cells of the corona
radiata, which remain with the oocyte following ovulation. The theca interna cells are
hormone secreting cells surrounding the follicle underlying the basement membrane of
the follicle. The macula pellucida or stigma, is a small area on the surface of the ovary
overlying the bulge caused by the mature follicle. This small, elevated area of germinal
epithelium will be the site of rupture at ovulation.
Question:
The primary oocyte as found within a primary follicle would most likely be in what meiotic stage

Answer Choices:
A Leptotene of meiosis I
B Diplotene of meiosis I
C Zygotene of meiosis I
D Pachytene of meiosis I
E Metaphase of meiosis I

B Diplotene of meiosis I

The primordial follicles are initially observed at about the third month of fetal
development. At this time in the embryo, the primary oocytes within the primordial
follicles begin the first meiotic division, but the process is arrested in prophase at the
diplotene stage of the first meiotic division. Completion of the first meiotic division does
not occur until immediately prior to ovulation of that respective oocyte. During the
diplotene stage of prophase of meiosis I, the chromosomes are highly condensed and
chiasmata between chromatids may appear. The leptotene stage is when the
chromosomes become visible as thin strands, the zygotene stage is when homologous
chromosomes pair and crossing over occurs, the pachytene stage is when
chromosomes condense and individual chromatids become visible. Following the five
prophase stages of meiosis I, the cell enters metaphase of meiosis I.
Question:
At what point in the development of the follicle may atresia occur?

Answer Choices:
A At the primordial follicle stage
B At the primary follicle stage
C At the secondary (antral) follicle stage
D At the mature (Graafian) follicle
E At any of the above stages
E At any of the above stages

Degeneration of the oocyte and follicular cells may occur at any time prior to ovulation.
This degeneration process is termed atresia, which is a spontaneous death and
reabsorption of the oocyte.
Question:
Histological examination of an ovary revealed the presence of an oocyte within the cortical stroma,
surrounded by multiple layers of granulosa cells with fluid filled cavities, which are beginning to form a
central antrum within the follicle. This would be an example of what follicular stage

Answer Choices:
A Primordial follicle
B Primary follicle
C Secondary follicle
D Mature (Graafian) follicle
E Atretic follicle
C Secondary follicle

In a mature ovary, primordial follicles (which first appear in the ovary during the third
month of fetal development), are found deep to the tunica albuginea. Each of these
follicles is surrounded by a basal lamina and the follicular cells surrounding the oocyte
are flattened (squamous). Upon stimulation by gonadotrophins, the primordial follicle
changes; the oocyte enlarges, and the surrounding follicular cells become cuboidal
shaped. At this point, the follicle is identified as a primary follicle. The zona pellucida
appears between the oocyte and follicular cells while in the primary follicle stage. The
single layer of cuboidal follicular cells, upon further stimulation, proliferates and forms a
stratified epithelium called the follicular or granulosa cells. Connective tissue cells,
located just external to the basal lamina of the granulosa cells, form a sheath of cells
called the theca folliculi. These theca cells further differentiate into two layers, an inner
theca interna, and an outer theca externa, which blend into the surrounding stroma of
the ovarian cortex. Continued stimulation of the primary follicle results in continued
proliferation of the granulosa cells. These cells form a fluid called liquor folliculi, which
begins to coalesce among the cells and form a single extracellular cavity, the antrum.
The formation of the antrum is characteristic of what is termed the secondary or antral
follicle. Continued stimulation of the follicle results in a follicle that may become 10mm
or more in diameter, bulges from the surface of the ovary, and is termed a mature, or
graafian, follicle. Degeneration of the oocyte and follicular cells may occur at any time
prior to ovulation. This degeneration process is termed atresia, which is a spontaneous
death and reabsorption of the oocyte.
Question:
Histological examination of an ovary revealed the presence of an oocyte within the cortical stroma
surrounded by two layers of cuboidal follicular cells. This would be an example of what follicular stage?

Answer Choices:
A Primordial follicle
B Primary follicle
C Secondary (antral) follicle
D Mature (Graafian) follicle
E Atretic follicle

B Primary follicle

In a mature ovary, primordial follicles (which first appear in the ovary during the third
month of fetal development), are found deep to the tunica albuginea. Each of these
follicles is surrounded by a basal lamina and the follicular cells surrounding the oocyte
are flattened (squamous). Upon stimulation by gonadotrophins, the primordial follicle
changes; the oocyte enlarges and the surrounding follicular cells become cuboidal
shaped. At this point the follicle is identified as a primary follicle. The zona pellucida
appears between the oocyte and follicular cells while in the primary follicle stage. The
single layer of cuboidal follicular cells, upon further stimulation, proliferates and forms a
stratified epithelium called the follicular or granulosa cells. Connective tissue cells
located just external to the basal lamina of the granulosa cells form a sheath of cells
called the theca folliculi. These theca cells further differentiate into two layers, an inner
theca interna, and an outer theca externa, which blend into the surrounding stroma of
the ovarian cortex. Continued stimulation of the primary follicle results in continued
proliferation of the granulosa cells. These cells form a fluid called liquor folliculi, which
begins to coalesce among the cells and form a single extracellular cavity, the antrum.
The formation of the antrum is characteristic of what is termed the secondary or antral
follicle. Continued stimulation of the follicle results in a follicle that may become 10mm
or more in diameter, bulges from the surface of the ovary, and is termed a mature or
graafian follicle. Degeneration of the oocyte and follicular cells may occur at any time
prior to ovulation. This degeneration process is termed atresia, which is a spontaneous
death and reabsorption of the oocyte.
Question:
Which one of the following ligaments associated with the female reproductive organs is homologous to
the gubernaculum testis in the male

Answer Choices:
A Suspensory ligament
B Ovarian ligament
C Broad ligament
D Mesovarium
E Mesosalpinx
B Ovarian ligament

The suspensory ligament attaches the ovary to the pelvic wall and carries within its folds
the main portion of the ovarian vessels and nerves. The ovarian ligament attaches the
inferior pole of the ovary to the uterus. It is homologous to the gubernaculum testis in
the male. The broad ligament is a double-fold of parietal peritoneum, which hangs over
the uterus (mesentery of the uterus) and connects lateral margins of the uterus with the
side wall of the pelvis. The mesovarium, the posterior extension of the broad ligament,
supports the ovary and becomes continuous with the serosa (or germinal epithelium) of
the ovary. The mesosalpinx, which is the superior limit of the broad ligament, supports
the uterine tube. Some branches of the ovarian vessels lie within the mesosalpinx.

Question:
The support for the uterine tube is best provided by which one of the following

Answer Choices:
A Suspensory ligament
B Ovarian ligament
C Broad ligament
D Mesovarium
E Mesosalpinx

E Mesosalpinx

The suspensory ligament attaches the ovary to the pelvic wall and carries within its folds
the main portion of the ovarian vessels and nerves. The ovarian ligament attaches the
inferior pole of the ovary to the uterus. The broad ligament is a double-fold of parietal
peritoneum, which hangs over the uterus (mesentery of the uterus) and connects lateral
margins of the uterus with the side wall of the pelvis. The mesovarium, the posterior
extension of the broad ligament, supports the ovary and becomes continuous with the
serosa (or germinal epithelium) of the ovary. The mesosalpinx, which is the superior
limit of the broad ligament, supports the uterine tube. Some branches of the ovarian
vessels lie within the mesosalpinx.

Question:
The primary support to the ovary is best provided by which one of the following ligaments

Answer Choices:
A Suspensory ligament
B Ovarian ligament
C Broad ligament
D Mesovarium
E Mesosalpinx
D Mesovarium

The suspensory ligament attaches the ovary to the pelvic wall and carries within its folds
the main portion of the ovarian vessels and nerves. The ovarian ligament attaches the
inferior pole of the ovary to the uterus. The broad ligament is a double-fold of parietal
peritoneum, which hangs over the uterus (mesentery of the uterus) and connects lateral
margins of the uterus with the side wall of the pelvis. The mesovarium, the posterior
extension of the broad ligament, supports the ovary and becomes continuous with the
serosa (or germinal epithelium) of the ovary. The mesosalpinx, which is the superior
limit of the broad ligament, supports the uterine tube. Some branches of the ovarian
vessels lie within the mesosalpinx.
Question:
The round ligament of the uterus is found as the content within which one of the following?

Answer Choices:
A Suspensory ligament
B Ovarian ligament
C Broad ligament
D Mesovarium
E Mesosalpinx

C Broad ligament

The suspensory ligament attaches the ovary to the pelvic wall and carries within its folds the
main portion of the ovarian vessels and nerves. The ovarian ligament attaches the inferior pole
of the ovary to the uterus. The broad ligament is a double-fold of parietal peritoneum, which
hangs over the uterus (mesentery of the uterus) and connects lateral margins of the uterus with
the side wall of the pelvis. The round ligament of the uterus is contained within the broad
ligament and runs from the lateral uterine border near the point of attachment of the ovarian
ligament to fuse with the dermis of the labium major. The mesovarium, the posterior extension
of the broad ligament, supports the ovary and becomes continuous with the serosa (or germinal
epithelium) of the ovary. The mesosalpinx, which is the superior limit of the broad ligament,
supports the uterine tube. Some branches of the ovarian vessels lie within the mesosalpinx.

Copyright © EXAM MASTER Corporation 2014 Author ID: 23


Question:
The main portion of the ovarian artery, vein, and nerves are carried to and from the ovary by way of the

Answer Choices:
A Suspensory ligament
B Ovarian ligament
C Broad ligament
D Mesovarium
E Mesosalpinx
A Suspensory ligament

The suspensory ligament attaches the ovary to the pelvic wall and carries within its folds
the main portion of the ovarian vessels and nerves. The ovarian ligament attaches the
inferior pole of the ovary to the uterus. The broad ligament is a double-fold of parietal
peritoneum, which hangs over the uterus (mesentery of the uterus) and connects lateral
margins of the uterus with the side wall of the pelvis. The mesovarium, the posterior
extension of the broad ligament, supports the ovary and becomes continuous with the
serosa (or germinal epithelium) of the ovary. The mesosalpinx, which is the superior
limit of the broad ligament, supports the uterine tube. Some branches of the ovarian
vessels lie within the mesosalpinx.
Question:
Identify the structure labeled 'A' on the photomicroscopic image.
Answer Choices:
A Mature follicle
B Atretic follicle
C Secondary follicle
D Primary follicle
E Primordial follicle

Image(s) / Chart(s):
Click image to view full size. Click open image to close. Click and hold open image to move.

E Primordial follicle

The structure labeled with the letter 'A' on the photomicroscopic image illustrates a
primordial follicle. The oocyte of the primordial follicle is surrounded by a single layer of
squamous follicular cells, bounded by a basal lamina. Frequently a large filamentous
nucleus may be identified filling most of the oocyte cytoplasm and containing a
prominent nucleolus. The primordial follicle represents the earliest stage of oocyte
development and appears in the ovaries during fetal development.
The structure labeled with the letter 'B' on the image represents a primary follicle. The
primary follicle represents the first stage of a growing follicle and is characterized by one
or more layers of cuboidal cells surrounding the enlarging oocyte. These follicular cells
become stratified and are identified in later primary follicles as granulosa cells.
The secondary follicles, not observed on this image, possess intercellular fluid (liquor
folliculi), which collects into an antrum within the granulosa cell layers. In the later
stages of development, the antrum enlarges and is lined with several layers of
granulosa cells. The oocyte moves to one side of the antrum and is surrounded by a
layer(s) of granulosa cells, the corona radiata, all of which are held on a thickened
mound of pedestal of cells, the cumulus oophorus, that projects into the antrum. As the
follicle reaches a diameter of approximately 10mm or more, the follicle is known as a
mature or Graafian follicle.
Question:
Which of the following statements concerning the vaginal mucosa is true?

Answer Choices:
A It is lined by stratified columnar epithelium
B It is lined by stratified squamous keratinized epithelium
C It possesses no elastic fibers
D It is lubricated by glands located in the cervix
E Its cells secrete lactic acid

D It is lubricated by glands located in the cervix


Image(s) / Chart(s):
Click image to view full size. Click open image to close. Click and hold open image to
move.

The vagina lacks glands throughout its length and is lubricated by secretions from
cervical glands. It is lined by a stratified squamous nonkeratinized epithelium that is
fortified by elastic fibers. These epithelial cells store and release glycogen, which is
used by the normal bacterial flora of the vagina to manufacture lactic acid.
See the attached Figure: A section through vagina. Epithelium (E); Lamina propria (Lm);
Submucosa (Sb).
Question:
During the proliferative phase of the menstrual cycle, the functional layer of the endometrium
undergoes which of the following changes?

Answer Choices:
A Blood vessels become ischemic
B The entire functional layer is renewed
C The stroma swells due to edema
D Glands become coiled
E The endometrium become thickened

B The entire functional layer is renewed

During the proliferative phase of the menstrual cycle, the entire functional layer of the
endometrium is renewed, including the epithelium lining the surface and glands.
Edema in the stroma and coiled glands, as well as thickening of the endometrium, are
characteristics of the secretory phase of the cycle. Ischemia is responsible for the
menstrual phase.
Question:
Which of the following statements concerning secondary ovarian follicles is true?

Answer Choices:
A They lack liquor folliculi
B They contain a secondary oocyte
C Their continued maturation requires FSH (follicle stimulating hormone)
D They lack a theca externa
E They have a single layer of cuboidal follicular cells surrounding the oocyte
C Their continued maturation requires FSH (follicle stimulating hormone)
Image(s) / Chart(s):
Click image to view full size. Click open image to close. Click and hold open image to
move.

Secondary follicles are dependent on FSH (follicle stimulating hormone) for their
continued development. They are established when liquor folliculi (an ultra-filtrate of
plasma and granulosa-cell secretion) begins to accumulate among the granulosa cells.
Secondary follicles contain a primary oocyte blocked in the prophase of meiosis. They
possess theca externa (is largely collagenous, with a few muscle cells, and contains
many blood vessels). Oocyte (primary) is enveloped by a single layer of squamous
follicular cells.
See the attached Figure: Ovary, showing developing follicles.
Question:
Which of the following statements concerning secondary ovarian follicles is true?

Answer Choices:
A They lack liquor folliculi
B They contain a secondary oocyte
C Their continued maturation requires FSH (follicle stimulating hormone)
D They lack a theca externa
E They have a single layer of cuboidal follicular cells surrounding the oocyte
C Their continued maturation requires FSH (follicle stimulating hormone)

Secondary follicles are dependent on follicle stimulating hormone (FSH) for their
continued development. They are established when liquor folliculi (an ultrafiltrate of
plasma and granulosa-cell secretions) begins to accumulate among the granulosa cells.
Secondary follicles contain a primary oocyte blocked in the prophase of meiosis I.
Question:
Which of the following statements concerning the corpus luteum is true?

Answer Choices:
A It produces LH (luteinizing hormone)
B It produces FSH (follicle stimulating hormone)
C It derives its granulosa luteal cells from the theca externa
D It becomes the corpus albicans
E It provides nourishment to theca interna
D It becomes the corpus albicans

A corpus albicans is formed from a corpus luteum that has ceased to function. LH
(luteinizing hormone) and FSH (follicle stimulating hormone) are both produced in the
adenohypophysis. Granulosa luteal cells are derived from the granulosa cells of an
ovulated graafian follicle. Nourishment to theca interna is provided by the theca externa.
Question:
The basal layer of the uterine endometrium

Answer Choices:
A Becomes sloughed during menstruation
B Has no glands
C Is supplied with coiled arteries
D Is supplied by the straight arteries
E Is lined by simple columnar epithelium containing peg cells
D Is supplied by the straight arteries
Image(s) / Chart(s):
Click image to view full size. Click open image to close. Click and hold open image to
move.

The basal layer of the uterine endometrium is supplied by the straight arteries and
contains the deeper portions of the uterine glands. Cells from these glands regenerate
the epithelium of the endometrial surface after the functional layer (supplied by the
coiled arteries) has been sloughed off. The functional layer (functionalis, the thick
superficial layer) of endometrium is sloughed off during the menstrual cycle. This layer
is supplied with coiled arteries. The oviducts (fallopian tubes) are lined by an epithelium
containing both ciliated and peg cells.
See the attached Figure: Uterine endometrium.
Question:
Identify the structure labeled 'B' on the photomicroscopic image

Answer Choices:
A Mature follicle
B Atretic follicle
C Secondary follicle
D Primary follicle
E Primordial follicle

Image(s) / Chart(s):
Click image to view full size. Click open image to close. Click and hold open image to move.
D Primary follicle

The structure labeled with the letter 'A' on the photomicroscopic image illustrates a
primordial follicle. The oocyte of the primordial follicle is surrounded by a single layer of
squamous follicular cells, bounded by a basal lamina. Frequently a large filamentous
nucleus may be identified filling most of the oocyte cytoplasm and containing a
prominent nucleolus. The primordial follicle represents the earliest stage of oocyte
development and appears in the ovaries during fetal development.
The structure labeled with the letter 'B' on the image represents a primary follicle. The
primary follicle represents the first stage of a growing follicle and is characterized by one
or more layers of cuboidal cells surrounding the enlarging oocyte. These follicular cells
become stratified and are identified in later primary follicles as granulosa cells.
The secondary follicles, not observed on this image, possess intercellular fluid (liquor
folliculi), which collects into an antrum within the granulosa cell layers. In the later
stages of development, the antrum enlarges and is lined with several layers of
granulosa cells. The oocyte moves to one side of the antrum and is surrounded by a
layer(s) of granulosa cells, the corona radiata, all of which are held on a thickened
mound of pedestal of cells, the cumulus oophorus that projects into the antrum. As the
follicle reaches a diameter of approximately 10mm or more, the follicle is known as a
mature or Graafian follicle.
Question:
Histological examination of an ovary revealed the presence of an oocyte within the cortical stroma
surrounded by flattened (squamous) follicular cells. This would be an example of what follicular stage?

Answer Choices:
A Primordial follicle
B Primary follicle
C Secondary (antral) follicle
D Mature (Graafian) follicle
E Atretic follicle
A Primordial follicle

In a mature ovary, primordial follicles (which first appear in the ovary during the third
month of fetal development), are found deep to the tunica albuginea. Each of these
follicles is surrounded by a basal lamina and the follicular cells surrounding the oocyte
are flattened (squamous). Upon stimulation by gonadotrophins, the primordial follicle
changes; the oocyte enlarges and the surrounding follicular cells become cuboidal
shaped. At this point, the follicle is identified as a primary follicle. The zona pellucida
appears between the oocyte and follicular cells while in the primary follicle stage. The
single layer of cuboidal follicular cells, upon further stimulation, proliferates and forms a
stratified epithelium called the follicular or granulosa cells. Connective tissue cells
located just external to the basal lamina of the granulosa cells form a sheath of cells
called the theca folliculi. These theca cells further differentiate into two layers, an inner
theca interna, and an outer theca externa, which blend into the surrounding stroma of
the ovarian cortex. Continued stimulation of the primary follicle results in continued
proliferation of the granulosa cells. These cells form a fluid called liquor folliculi, which
begins to coalesce among the cells and form a single extracellular cavity, the antrum.
The formation of the antrum is characteristic of what is termed the secondary or antral
follicle. Continued stimulation of the follicle results in a follicle that may become 10mm
or more in diameter, bulges from the surface of the ovary, and is termed a mature or
graafian follicle. Degeneration of the oocyte and follicular cells may occur at any time
prior to ovulation. This degeneration process is termed atresia, which is a spontaneous
death and reabsorption of the oocyte.
Question:
Histological examination of an ovary revealed the presence of a large follicle (>10mm diameter),
possessing a large antrum, with a pedestal of cells supporting the oocyte projecting into the antrum.
This would be an example of what follicular stage?

Answer Choices:
A Primordial follicle
B Primary follicle
C Secondary (antral) follicle
D Mature (Graafian) follicle
E Atretic follicle

D Mature (Graafian) follicle

In a mature ovary, primordial follicles (which first appear in the ovary during the third
month of fetal development), are found deep to the tunica albuginea. Each of these
follicles is surrounded by a basal lamina and the follicular cells surrounding the oocyte
are flattened (squamous). Upon stimulation by gonadotrophins, the primordial follicle
changes; the oocyte enlarges and the surrounding follicular cells become cuboidal
shaped. At this point the follicle is identified as a primary follicle. The zona pellucida
appears between the oocyte and follicular cells while in the primary follicle stage. The
single layer of cuboidal follicular cells, upon further stimulation, proliferates and forms a
stratified epithelium called the follicular or granulosa cells. Connective tissue cells
located just external to the basal lamina of the granulosa cells form a sheath of cells
called the theca folliculi. These theca cells further differentiate into two layers, an inner
theca interna, and an outer theca externa, which blend into the surrounding stroma of
the ovarian cortex. Continued stimulation of the primary follicle results in continued
proliferation of the granulosa cells. These cells form a fluid called liquor folliculi, which
begins to coalesce among the cells and form a single extracellular cavity, the antrum.
The formation of the antrum is characteristic of what is termed the secondary or antral
follicle. Continued stimulation of the follicle results in a follicle that may become 10mm
or more in diameter, bulges from the surface of the ovary, and is termed a mature or
graafian follicle. Degeneration of the oocyte and follicular cells may occur at any time
prior to ovulation. This degeneration process is termed atresia, which is a spontaneous
death and reabsorption of the oocyte.
Question:
At the time of ovulation, the secondary oocyte would most likely be found in what meiotic stage?

Answer Choices:
A Pachytene of meiosis II
B Diplotene of meiosis II
C Anaphase of meiosis II
D Telophase of meiosis II
E Metaphase of meiosis II
E Metaphase of meiosis II

Completion of the first meiotic division occurs immediately prior to ovulation for that
respective oocyte and one daughter cell receives the majority of the cytoplasm during
the reduction division to form a secondary oocyte (the other daughter cell becomes the
first polar body). Upon completion of the first meiotic division, the second meiotic
division begins, but is arrested at metaphase of meiosis II. This division continues
ONLY if the secondary oocyte is penetrated by a spermatozoon.
Question:
The autolysis of the corpus luteum forms which one of the following structures

Answer Choices:
A Theca lutein cells
B Granulosa lutein cells
C Corpora lutea atretica
D Corpus albicans
E Ovarian hilar cells
D Corpus albicans

Immediately following ovulation, the cells of the mature follicle are transformed into a
corpus luteum. This ovarian structure persists for only 14 days following ovulation. The
follicular cells become the granulosa lutein cells and are associated with increased
secretion of progesterone. The theca interna cells become the theca lutein cells that are
more peripherally located on the corpus luteum. The corpus albicans is formed with the
degeneration of the corpus luteum, in which the granulosa lutein cells undergo autolysis
and a hyaline material accumulates within the cellular remnants of the former corpus
luteum. It will eventually blend into the stromal tissue of the ovary. The ovarian hilar
cells are cells found in the hilum of the ovary and may be androgen secreting cells.
Atretic follicles, particularly those that begin the process of atresia at a later stage of
follicular development, may resemble an old corpus luteum and are referred to as a
corpora lutea atretica.

Question:
The segment of the uterine tube located adjacent to the uterus is referred to as the

Answer Choices:
A Fimbriae
B Ampulla
C Isthmus
D Infundibulum
E Intramural
C Isthmus

The uterine tube may be divided into four segments. The most distal portion, which is
funnel-shaped and most closely associated with the ovary, is the infundibulum. Finger-
like projections of the most distal portion of the infundibulum extend toward the ovary
and are called fimbriae. The ampulla comprises approximately 2/3 of the length of the
uterine tube and is regarded as the longest segment of the tube. Immediately adjacent
to the uterus, the uterine tube becomes narrowed and is called the isthmus. The portion
of the uterine tube that extends through the uterine wall is the intramural segment. This
portion opens into the uterine lumen.
Question:
Finger-like extensions of the most distal portion of the uterine tube located adjacent to the ovary are
the

Answer Choices:
A Fimbriae
B Ampulla
C Isthmus
D Infundibulum
E Intramural
A Fimbriae

The uterine tube may be divided into four segments. The most distal portion, which is
funnel-shaped and most closely associated with the ovary, is the infundibulum. Finger-
like projections of the most distal portion of the infundibulum extend toward the ovary
and are called fimbriae. The ampulla comprises approximately 2/3 of the length of the
uterine tube and is regarded as the longest segment of the tube. Immediately adjacent
to the uterus, the uterine tube becomes narrowed and is called the isthmus. The portion
of the uterine tube that extends through the uterine wall is the intramural segment. This
portion opens into the uterine lumen.
Question:
The part of the uterus indicated as letter 'A' and letter 'B' in the attached photomicroscopic image is
best described as (respectively)

Answer Choices:
A Myometrium, stratum basalis
B Stratum basalis, stratum functionalis
C Stratum functionalis, stratum basalis
D Myometrium, stratum functionalis
E Stratum basale, stratum granulosum

Image(s) / Chart(s):
Click image to view full size. Click open image to close. Click and hold open image to move.

B Stratum basalis, stratum functionalis

During the uterine, or menstrual cycle, the stratum functionalis (indicated by letter B) or
functional layer of the endometrium changes in thickness, vascularity, and glandularity.
Under the influence of estrogens from the developing follicle, the endometrium thickens,
and the surface epithelium invaginates to form simple tubular glands, known as the
uterine glands. Branches of the spiral artery provide a rich supply of blood to the
developing endometrial layer. These changes are typical of the proliferative stage.
Following ovulation, during the secretory phase, the endometrium is influenced by
progesterone from the corpus luteum of the ovary, and the extensive uterine glands
become enlarged, torturous, and glycogen-rich. The endometrial stroma becomes
highly cellular and the endometrium is prepared to accept and sustain the fertilized ova.
Without fertilization and implantation, the corpus luteum degenerates, and the stratum
functionalis layer of the uterine endometrium responds by becoming ischemic, and
eventually sloughing off, as the menses occurs. The basal portion of the endometrium,
or stratum basalis (indicated by letter A), remains to serve as the source for
regeneration of the stratum functionalis in the next uterine cycle.
Question:
Identify the structure labeled 'B' on the photomicroscopic image.

Answer Choices:
A Mature follicle
B Atretic follicle
C Secondary follicle
D Primary follicle
E Primordial follicle

Image(s) / Chart(s):
Click image to view full size. Click open image to close. Click and hold open image to move.

B Atretic follicle

The oocyte of the primordial follicle is surrounded by a single layer of squamous


follicular cells, bounded by a basal lamina. Frequently a large filamentous nucleus may
be identified filling most of the oocyte cytoplasm and containing a prominent nucleolus.
The primordial follicle represents the earliest stage of oocyte development and appears
in the ovaries during fetal development.
The primary follicle represents the first stage of a growing follicle and is characterized
by one or more layers of cuboidal cells surrounding the enlarging oocyte. These
follicular cells become stratified and are identified in later primary follicles as granulosa
cells. A secondary follicle as identified by 'A', possesses intercellular fluid (liquor
folliculi), which collects into an antrum within the granulosa cell layers. In the later
stages of development, the antrum enlarges and is lined with several layers of
granulosa cells. The oocyte moves to one side of the antrum and is surrounded by a
layer(s) of granulosa cells, the corona radiata, all of which are held on a thickened
mound of pedestal of cells, the cumulus oophorus that projects into the antrum. As the
follicle reaches a diameter of approximately 10mm or more, the follicle is known as a
mature or Graafian follicle.
The letter 'B' identifies a structure that has a 'glassy' appearance and represents the
appearance of an atretic follicle. A follicle may undergo atresia in any stage of
development, but in this example, the follicle was probably in an early stage of
development when it began the degenerative process.
Question:
Which one of the following statements best describes a histological difference between the esophagus
and vagina?

Answer Choices:
A Vagina possesses submucosal glands, while the esophagus does not
B Vagina possesses keratinized stratified squamous epithelium; esophagus
possesses non-keratinized stratified squamous epithelium
C Esophagus possesses submucosal glands, while the vagina does not
D Esophagus possesses more extensive vascular sinuses than the vagina
E Vagina possesses a muscularis mucosa, while the esophagus does not
C Esophagus possesses submucosal glands, while the vagina does not

While the histological appearances of the vagina and esophagus are similar, there are
some important differences. One primary difference, is that the vagina does not have a
muscularis mucosa, while the esophagus does. Therefore, the esophagus has a
submucosa, and within this submucosa are located mucus glands (esophageal glands
proper). The esophagus also possess glands in the lamina propria of the mucosa
(cardiac glands). There are no glands in the vagina. Secretions of the vagina come
primarily from glands of the cervix. Both the esophagus and the vagina possess non-
keratinized stratified squamous epithelial linings. However, the vagina appears to have
a much greater vascularity in the form of sinuses located in the underlying connective
tissue than does the esophagus.
Question:
The structure within the female external genitalia possessing paired corpora cavernosa is the:
Answer Choices:
A Mons pubis
B Labia majora
C Labia minora
D Clitoris
E Vestibule
D Clitoris

The mons pubis is the prominence of subcutaneous tissue covered by stratified


squamous epithelium overlying the female pubic symphysis. The labia majora are 2
large folds of skin that extend posteriorly from the mons pubis. These form the lateral
boundaries of the vaginal opening and are covered with hair on the exterior surface, but
are devoid of hair on the smooth inner surface. Sebaceous and sweat glands are
associated with each surface and the folds contain various amounts of subcutaneous
adipose tissue.
The labia majora are homologous to the skin of the scrotum in the male. The labia
minora are smaller, paired folds of skin that underlie the labia majora and boarder the
vaginal vestibule. These hairless folds of skin have a core of richly vascularized
connective tissue and possess large sebaceous glands.
The clitoris is homologous to the penis and consists of erectile tissue covered by
stratified squamous epithelium. Similar to the structure of the penis, the body of the
clitoris is composed of paired erectile bodies referred to as corpora cavernosa. The
clitoris ends with a small, round tubercle of tissue called the glans clitoris. The vestibule
represents the opening of the vagina and is lined with stratified squamous epithelium.
Vestibular glands are present in the underlying connective tissue and provide mucus
secretions near the vaginal opening.
Question:
A pedestal of granulosa cells is observed to support the oocyte in the mature follicle. This pedestal of
cells is referred to as the

Answer Choices:
A Zona pellucida
B Corona radiata
C Theca interna
D Cumulus oophorus
E Macula pellucida

D Cumulus oophorus

The large antrum of the mature follicle fills the central part of the cavity pushing several
layers of granulosa cells to the periphery of the follicle. In one region the cells will form a
pedestal supporting the oocyte. This pedestal of cells is referred to as the cumulus
oophorus and projects into the oocyte into the antrum. Granulosa cells immediately
surrounding the oocyte, external to the zona pellucida, are the cells of the corona
radiata, which remain with the oocyte following ovulation. The theca interna cells are
hormone secreting cells surrounding the follicle underlying the basement membrane of
the follicle. The macula pellucida or stigma, is a small area on the surface of the ovary
overlying the bulge caused by the mature follicle. This small, elevated area of germinal
epithelium will be the site of rupture at ovulation.

Question:
The muscularis of the ampulla of uterine tube is comprised of:

Answer Choices:
A An inner circular and an outer longitudinal layer
B An inner longitudinal and an outer circular layer
C A single thick circular layer
D A single thick longitudinal layer
E An inner longitudinal layer, a middle circular layer, and an outer longitudinal
layer
A An inner circular and an outer longitudinal layer

The muscularis of the uterine tube is organized into 2 layers, a relatively thick inner
circular and a thinner outer longitudinal layer. These layers persist throughout most of
the length of the uterine tube.

SECTION- MAMMARY GLAND


Question:

Each tubuloalveolar gland of the breast ends in which one of the following ducts?

Answer Choices:

A Lactiferous sinus
B Areola

C Lactiferous duct

D Intercalary duct

E Striated duct

C Lactiferous duct

The mammary glands are composed of compound tubuloalveolar glands, which lie in
the subcutaneous tissue of the pectoral area. Each of the tubuloalveolar glands ends in
a lactiferous duct that opens to the surface at the nipple. Proximal to the opening of
the lactiferous duct is a dilated portion of the duct, referred to as the lactiferous sinus.
The areola is a pigmented area surrounding the nipple of each breast containing
sebaceous glands and sweat glands. The intercalary duct and the striated duct are
excretory ducts associated with salivary glands.

Question:

The histology of the mammary gland at about the time of ovulation in a sexually mature woman would
best be characterized by

Answer Choices:

A Secretory cell hypertrophy and alveolar accumulation of secretory product

B Poorly developed ductal elements in which some contain fluids

C Increased height of the secretory cells and increased mitotic activity

D The presence of extensively branched ducts and alveoli


E Infiltration of plasma cells, lymphocytes, and eosinophils in the fibrous
component of the connective tissue

B Poorly developed ductal elements in which some contain fluids

The glandular component of the inactive mammary gland is best characterized by


possessing poorly developed ductal elements as the predominant feature. Being
responsive to estrogen, these ducts may vary somewhat during the ovarian cycle,
appearing as cords with little evidence of a lumen early in the cycle, but revealing a
lumen at about the time of ovulation, even with some small amounts of fluid within
the lumen being present.

With the advent of pregnancy, the mammary glands change by decreases in the
amount of connective tissue and adipose tissue, stimulation of the development of
glandular tissue by evidence of extensively branched ducts, and developing alveoli.
Another change that occurs is the infiltration of plasma cells, lymphocytes, and
eosinophils in the fibrous component of the connective tissue. Near the time of lactation,
the proliferation of the acinar cells decline, and subsequent enlargement of the breast
occurs due to secretory cell hypertrophy and the alveolar accumulation of secretory
product.

Question:

The histology of the "inactive" mammary gland would best be characterized by

Answer Choices:

A Secretory cell hypertrophy and alveolar accumulation of secretory product

B Ductal elements representing the primary feature of the glandular


component

C Increased height of the secretory cells and increased mitotic activity


D The presence of extensively branched ducts and alveoli

E Infiltration of plasma cells, lymphocytes, and eosinophils in the fibrous


component of the connective tissue

B Ductal elements representing the primary feature of the glandular component

The glandular component of the inactive mammary gland is best characterized by


possessing poorly developed ductal elements as the predominant feature. Being
responsive to estrogen, these ducts may vary somewhat during the ovarian cycle,
appearing as cords with little evidence of a lumen early in the cycle, but revealing a
lumen at about the time of ovulation, even with some small amounts of fluid within the
lumen being present.

With the advent of pregnancy, the mammary glands change by decreases in the
amount of connective tissue and adipose tissue, stimulation of the development of
glandular tissue by evidence of extensively branched ducts, and developing alveoli.
Another change that occurs is the infiltration of plasma cells, lymphocytes, and
eosinophils in the fibrous component of the connective tissue. Near the time of lactation,
the proliferation of the acinar cells decline, and subsequent enlargement of the breast
occurs due to secretory cell hypertrophy and the alveolar accumulation of secretory
product.

Question:

The lipid component of the milk from the mammary glands is secreted by which of the following
processes?

Answer Choices:

A Apocrine

B Holocrine
C Merocrine

D Eccrine

E Lacticrine

A Apocrine

Both merocrine and apocrine secretions are involved in the secretion of milk from the
mammary gland. The milk contains a proteinaceous component, as well as a lipid
component. The secretory cells within the acini are capable of synthesizing both
components, but they are each secreted in a different manner. Following synthesis of
the protein component by the rough ER, the proteins are packaged into secretory
granules and released from the cell by merocrine secretion (fusion of the secretory
granule with that of the cell plasma membrane).

In contrast, the lipid component of the milk is formed as free droplets in the cytoplasm,
which will coalesce to the apical region of the secretory cell. Release of the lipid
accumulation results in the loss of a minimal amount of cytoplasm into the lumen of the
acinus, hence the process of apocrine secretion. Holocrine secretion occurs in the
process of the loss of the whole cell with the secretory product, such as occurs with the
sebaceous gland.

Question:

The protein component of milk from the mammary glands is secreted by which of the following
processes?

Answer Choices:

A Apocrine

B Holocrine
C Merocrine

D Eccrine

E Lacticrine

C Merocrine

Both merocrine and apocrine secretions are involved in the secretion of milk from the
mammary gland. The milk contains a proteinaceous component as well as a lipid
component. The secretory cells within the acini are capable of synthesizing both
components, but they are each secreted in a different manner. Following synthesis of
the protein component by the rough ER, the proteins are packaged into secretory
granules and released from the cell by merocrine secretion (fusion of the secretory
granule with that of the cell plasma membrane).

In contrast, the lipid component of the milk is formed as free droplets in the cytoplasm,
which will coalesce to the apical region of the secretory cell. Release of the lipid
accumulation results in the loss of a minimal amount of cytoplasm into the lumen of the
acinus, hence the process of apocrine secretion. Holocrine secretion occurs in the
process of the loss of the whole cell with the secretory product, such as occurs with the
sebaceous gland.

Question:

A 35-year-old woman detects a painless mass in her breast. She sees her primary care doctor, and a
biopsy is done. The results come back and indicate a malignancy. When she is told that she has breast
cancer, she asks her doctor why this has happened to her. Which of the following put her at an
increased risk for the development of breast cancer?

Answer Choices:

A Menarche before age 12


B Multiparity

C Young age

D Multiple sexual partners

E History of cervical cancer

Image(s) / Chart(s):
Click image to view full size. Click open image to close. Click and hold open image to move.
A Menarche before age 12

Menarche before age 12 puts a woman at increased risk for breast cancer.
Nulliparous women, as opposed to multiparous women, have a higher incidence of
breast cancer. Most breast cancers are in women over 50 years old. Having multiple
sexual partners does not increase a woman's risk of breast cancer. A history of cervical
cancer does not increase a woman's risk of breast cancer.

Question:

All of the following endocrine glands play a role in the development and functioning of the mammary
gland, EXCEPT the

Answer Choices:

A Ovary

B Corpus luteum

C Adrenal medulla

D Adenohypophysis

E Neurohypophysis

C Adrenal medulla

The adrenal medulla synthesizes, stores, and releases the catecholamines (epinephrine
and norepinephrine). Their release, mediated by the preganglionic sympathetic fibers
that innervate chromaffin cells, occurs in response to intense emotional reactions.

Ovary and ovarian corpus luteum secret estrogen and progesterone. They initiate
growth and development of the glands. Adenohypophysis releases prolactin, which
stimulates milk secretion. Neurohypophysis secretes oxytocin that causes the ejection
of the milk by stimulating the myoepithelial cells.
SECTION- HUMAN PLACENTA
Question:

The placenta secretes a hormone that participates in stimulation of ductal growth in the mammary
gland during pregnancy. This hormone is

Answer Choices:

A Endothelial growth factor (EGF)

B Human chorionic gonadotropin (hCG)

C Human chorionic somatotropin (HCS)

D Relaxin

E Insulin-like growth factor (IGF)

C Human chorionic somatotropin (HCS)

All of these hormones are produced by the placenta, but hCG is the hormone that
maintains the corpus luteum of pregnancy. The synthesis and secretion of hCG begins
about day 6 of pregnancy, presumably from the trophoblast cells. Pregnancy tests
utilize the measurement of hCG for detection of the embryo. EGF is synthesized by the
cytotrophoblast cells in the early placenta, then is synthesized by the
syncytiotrophoblast cells later (6-8 week old placenta). EGF maintains the trophoblasts.
Relaxin is synthesized by decidual cells at the time of parturition and acts to "soften" the
cervix and pelvic ligaments. IGF act similarly to EGF by stimulating differentiation of the
cytotrophoblast cells. HCS is synthesized by syncytiotrophoblast cells and will promote
general growth. It is essential in the stimulation of mammary duct proliferation in
development of the breast during pregnancy.

Question:
Which of the following portions of the uterus participates in formation of the placental decidua?

Answer Choices:

A Stratum basalis

B Stratum functionalis

C Myometrium

D Stratum basalis and stratum functionalis

E Myometrium, stratum basalis, and stratum functionalis

B Stratum functionalis

Only the endometrium undergoes changes during placental formation, and the decidua,
which is formed upon implantation, normally includes only the stratum functionalis.
The stratum functionalis consists of the portion of the uterine lining which is normally
shed during the menses, leaving the basal portion. This also occurs with the formation
and shedding of the decidua at the end of pregnancy.

Question:

Which one of the following invades the uterine endometrium for the purpose of implantation?

Answer Choices:

A Ovum

B Zygote
C Conceptus

D Blastocyst

E Syncytiotrophoblast

E Syncytiotrophoblast

Syncytiotrophoblast invades the endometrium carrying along the blastocyst which


implants in the wall of the uterus. Ovum is the unfertilized egg released by the ovary.
When this ovum is fertilized, it becomes a zygote. Zygote undergoes cleavage and
transforms itself into morula. Conceptus is the preimplantation embryo and its
surrounding membranes. It develops into a blastocyst which attaches to the
endometrium and becomes surrounded by an inner cellular layer, the cytotrophoblast,
and an outer, multinucleated layer, the syncytiotrophoblast.

Question:

Which of the following layers of the placental barrier in a near term fetus is discontinuous?

Answer Choices:

A Syncytiotrophoblast

B Cytotrophoblast

C Basal lamina of trophoblast

D Basal lamina of endothelial cell

E Endothelium of the fetal placental capillary

B Cytotrophoblast
In the early placenta, the primary villi are surrounded by a continuous layer of outer
syncytiotrophoblast cells, and an inner layer of cytotrophoblast cells. The villi are
composed of a connective tissue core, within which the fetal capillaries develop. Later in
pregnancy, the tertiary villi lack the continuous inner layer of cytotrophoblast cells.
Therefore the placental barrier in the near term fetus is constituted by an outer
syncytiotrophoblast layer, a discontinuous inner cytotrophoblast layer, the basal
lamina of the trophoblast layer, the basal lamina of the endothelial cell and the fetal
placental capillary endothelium. Frequently, varying thicknesses of connective tissue
may be posed between the two adjacent basal laminae.

Case #7357:

A 29-year-old gravida 3, para 2 woman was followed during her third pregnancy. Her
medical history was unremarkable and the previous pregnancies were uneventful.
Single umbilical artery is suspected after ultrasound. Further evaluation to confirm this
finding is required, as a single umbilical artery may be associated with other
congenital anomalies or syndromes. Prenatal identification of 3-vessels in the
umbilical cord can be done by ultrasound visualization of the free-floating cord during
the second and third trimesters.

Question:

Which of the following is another method to confirm the number of vessels in the
umbilical cord when ultrasound findings are equivocal?

Answer Choices:

A CT showing presence of 2 umbilical veins and a large umbilical artery

B Doppler ultrasound showing Wharton jelly covering the umbilical vessels

C Post-delivery examination of the cord for the number of vessels

D Doppler ultrasound showing hanging noose sign of the umbilical cord


E Doppler ultrasound showing 2 umbilical arteries lateral to the fetal
bladder

E Doppler ultrasound showing 2 umbilical arteries lateral to the fetal bladder

Identification of 3-vessels in the umbilical cord in a free floating cord is sometimes not
possible on ultrasound examination; however, a color Doppler imaging of the
umbilical arteries at the level of the fetal bladder can confirm the vessel number.
Ultrasound (transverse view) is helpful in the visualization of the number of vessels in
the free-floating umbilical cord. When in doubt, color Doppler ultrasound is used for
visualizing the number of umbilical arteries at the level of the fetal bladder. The umbilical
cord has 2 arteries and 1 vein that twist around each other in a spiral pattern. Normally,
there are 2 umbilical arteries that course lateral to the fetal bladder. In single umbilical
artery, there would be only 1 artery present lateral to the fetal bladder. Ultrasound can
be used to follow the cord vessels as they leave the placenta and enter the fetal
abdomen. Another approach is to image the umbilical arteries running along the
lateral margins of the fetal bladder with color flow Doppler. This image rules out a
single umbilical artery and agenesis of the kidneys (Potter's syndrome). Prenatal
ultrasound examination helps in outlining the normal anatomy and also depicts various
congenital abnormalities of the umbilical cord, which includes single umbilical artery,
cysts and pseudocysts, hemangiomas, umbilical vein varix, persistent right umbilical
vein, umbilical aneurysm, velamentous insertion, and vasa previa.

Wharton jelly surrounds the cord vessels and provides protection to the vessels;
however, it cannot be relied on for the confirmation of the vessel number.

Doppler ultrasound showing hanging noose sign of the umbilical cord is indicative of
true knot and does not indicate single umbilical artery.

Post-delivery examination of the cord for the number of vessels can be done; however,
the purpose of early identification of umbilical cord abnormalities and fetal monitoring
would not be possible. If ultrasound findings are equivocal, prenatal confirmation of the
number of cord vessels should be done by Doppler ultrasound and not wait for delivery
for examining the cord.

Normally the umbilical cord has 2 arteries and 1 vein; therefore, CT showing presence
of 2 umbilical veins and a large umbilical artery is not the correct choice.
SECTION- MALE REPRODUCTIVE
SYSTEM

Question:

The following developmental gene is involved in both spermatogenesis and breast maturation.

Answer Choices:

A MYBA (avian myeloblastosis viral oncogene homologue A)

B MYBB (avian myeloblastosis viral oncogene homologue B)

C MYBC (avian myeloblastosis viral oncogene homologue C)

D MYC (avian myelocytomatosis viral oncogene homologue)

E TSPY (testis-specific protein, Y-linked)

A MYBA (avian myeloblastosis viral oncogene homologue A)

MYBA is involved in the progression of spermatogenesis beyond pachytene. It is also


involved in the maturation ductal branching and alveolar development, by regulating
progesterone-induced events. It appears not to be involved in the development of the
ovaries.

Question:

The most accurate classification of the type of gland for the prostate based upon the structure of the
gland is
Answer Choices:

A Simple coiled tubular

B Compound alveolar

C Branched tubular

D Compound tubuloalveolar

E Saccular

D Compound tubuloalveolar

The prostate gland is an exocrine gland having a compound tubuloalveolar structure.


The glandular epithelium is most frequently observed as pseudostratified columnar, but
is extremely variable and may be seen as simple columnar, cuboidal, or even
squamous. A collection of 30-50 tubuloalveolar glands is embedded within fibroelastic
stroma, which contains smooth muscle randomly dispersed throughout the stroma.
Prostatic concretions (corpora amylacea) are commonly found in the prostatic alveoli,
and increase in number and size with age. The primary secretory products of the
prostate gland are citric acid and acid phosphatase, which are released into the
prostatic urethra at the time of ejaculation.

Question:

The primary secretory product of the prostate gland is

Answer Choices:
A Melatonin

B Milk

C Citric acid and acid phosphatase

D Fructose and prostaglandins

E Fructose and citric acid

C Citric acid and acid phosphatase

The seminal vesicle is a tubular gland possessing a muscular and fibrous coat, but has
a highly folded mucosa. The epithelium is a pseudostratified columnar epithelium
without cilia. The major secretory product of the seminal vesicle is fructose and
prostaglandins released at the time of ejaculation. The pineal gland, a neuroendocrine
gland which secretes melatonin, does not possess exocrine glands, but as an endocrine
organ, has a heterogeneous array of pinealocytes interspersed by capillaries and
calcified structures called corpora arenacea (brain sand). The prostate gland is an
exocrine gland having a compound tubuloalveolar structure.

The glandular epithelium is most frequently observed as pseudostratified columnar, but


is extremely variable and may be seen as simple columnar, cuboidal or even
squamous. The collection of 30-50 tubuloalveolar glands is embedded within fibroelastic
stroma, which contains smooth muscle randomly dispersed throughout the stroma.
Prostatic concretions (corpora amylacea) are commonly found in the prostatic alveoli,
and increase in number and size with age. The primary secretory products of the
prostate gland are citric acid and acid phosphatase, which are released into the
prostatic urethra at the time of ejaculation.

The mammary glands also are compound tubuloalveolar glands, but show alveoli
embedded within the connective tissue of the breast, sometimes revealing the presence
of adipose tissue surrounding the connective tissue that divide the breast into lobules.
During lactation, the mammary gland secretes a protein and lipid mixture referred to as
milk.

Question:

The primary function of the organ as depicted in the attached digitized photomicroscopic image is best
described as
Answer Choices:

A Secretion of melatonin

B Secretion of milk

C Secretion of citric acid and acid phosphatase

D Secretion of fructose and prostaglandins

E Secretion of fructose and citric acid

Image(s) / Chart(s):
Click image to view full size. Click open image to close. Click and hold open image to move.

C Secretion of citric acid and acid phosphatase

The prostate gland is an exocrine gland having a compound tubuloalveolar structure.


The glandular epithelium is most frequently observed as pseudostratified columnar, but
is extremely variable and may be seen as simple columnar, cuboidal or even
squamous. The collection of 30-50 tubuloalveolar glands is embedded within fibroelastic
stroma, which contains smooth muscle randomly dispersed throughout the stroma.
Prostatic concretions (corpora amylacea) are commonly found in the prostatic alveoli,
and increase in number and size with age. The primary secretory products of the
prostate gland are citric acid and acid phosphatase, which are released into the
prostatic urethra at the time of ejaculation. The mammary glands also are compound
tubuloalveolar glands, but show alveoli embedded within the connective tissue of the
breast, sometimes revealing the presence of adipose tissue surrounding the connective
tissue that divide the breast into lobules. During lactation, the mammary gland secretes
a protein and lipid mixture referred to as milk.

The seminal vesicle is a tubular gland possessing a muscular and fibrous coat, but has
a highly folded mucosa. The epithelium is a pseudostratified columnar epithelium
without cilia. The major secretory product of the seminal vesicle is fructose and
prostaglandins released at the time of ejaculation. The pineal gland, a neuroendocrine
gland which secretes melatonin, does not possess exocrine glands, but as an endocrine
organ, has a heterogeneous array of pinealocytes interspersed by capillaries and
calcified structures called corpora arenacea (brain sand).

Question:

The organ as depicted in the attached digitized photomicroscopic image is best identified as the

Answer Choices:

A Pineal gland

B Efferent ductule

C Prostate gland

D Mammary gland

E Seminal vesicle

Image(s) / Chart(s):
Click image to view full size. Click open image to close. Click and hold open image to move.
C Prostate gland

The efferent ducts are lined with a pseudostratified epithelium with alternating areas of
cuboidal cells possessing microvilli, and areas of tall columnar cells possessing cilia.
This gives the epithelium a characteristic "sawtooth" appearance when observed in
cross-section. A circularly arranged smooth muscle layer is first observed underlying the
epithelial lining in the efferent ductule. The seminal vesicle is a tubular gland possessing
a muscular and fibrous coat, but has a highly folded mucosa. The epithelium is a
pseudostratified columnar epithelium without cilia. The pineal gland, a neuroendocrine
gland which secretes melatonin, does not possess exocrine glands, but as an endocrine
organ, has a heterogeneous array of pinealocytes interspersed by capillaries and
calcified structures called corpora arenacea (brain sand). The prostate gland is an
exocrine gland having a compound tubuloalveolar structure.

The glandular epithelium is most frequently observed as pseudostratified columnar, but


is extremely variable and may be seen as simple columnar, cuboidal or even
squamous. The collection of 30-50 tubuloalveolar glands is embedded within fibroelastic
stroma, which contains smooth muscle randomly dispersed throughout the stroma.
Prostatic concretions (corpora amylacea) are commonly found in the prostatic alveoli,
and increase in number and size with age. The primary secretory products of the
prostate gland are citric acid and acid phosphatase, which are released into the
prostatic urethra at the time of ejaculation. The mammary glands also are compound
tubuloalveolar glands, but show alveoli embedded within the connective tissue of the
breast, sometimes revealing the presence of adipose tissue surrounding the connective
tissue that divide the breast into lobules.

Question:

The function of the organ as depicted in the attached digitized photomicroscopic image is best
described as
Answer Choices:

A Storage and secretion of bile

B Production of male gametes

C Secretion of a fluid, containing fructose

D Secretion of a fluid, containing citric acid

E Secretion of milk

Image(s) / Chart(s):
Click image to view full size. Click open image to close. Click and hold open image to move.

C Secretion of a fluid, containing fructose

The attached photomicroscopic image depicts that of a seminal vesicle. The seminal
vesicle is a tubular gland possessing a muscular and fibrous coat, but has a highly
folded mucosa. The epithelium is a pseudostratified columnar epithelium without cilia.
The secretory product of the gland is a whitish-yellow material, which contains fructose
and prostaglandins. Fructose is the principal metabolic substrate for the sperm upon
ejaculation.

The gall bladder, which is similar in histological appearance to the seminal vesicle, is
the principal site for the storage and release of bile. Production of gametes in the male
occurs in the seminiferous tubules of the testes, and from the mature follicle of the ovary
in the female. The prostate gland produces a fluid containing citric acid, and acid
phosphatase. Milk is produced by the specialized compound tubuloalveolar cells of the
mammary gland.

Question:

The organ as depicted in the attached digitized photomicroscopic image is best identified as the

Answer Choices:

A Rete testes

B Efferent ductule

C Ductus epididymis

D Ductus deferens

E Seminal vesicle

Image(s) / Chart(s):
Click image to view full size. Click open image to close. Click and hold open image to move.
C Ductus epididymis

The rete testes are complex anastomosing channels within the mediastinum testes that
receive spermatozoa from the tubuli recti (straight tubules) of the seminiferous tubules.
The rete testes are lined with a simple cuboidal or columnar epithelium and possess a
single cilium and/or microvilli on the apical surface. The rete testes empty into the
efferent ductule, which consist of numerous channels that lead to the head of the ductus
epididymis. The efferent ducts are lined with a pseudostratified epithelium with
alternating areas of cuboidal cells possessing microvilli, and areas of tall columnar cells
possessing cilia. This gives the epithelium a characteristic "sawtooth" appearance when
observed in cross-section. A circularly arranged smooth muscle layer is first observed
underlying the epithelial lining in the efferent ductule.

The ductus epididymis is a highly coiled structure having a pseudostratified columnar


epithelium with prominent stereocilia present. The height of the epithelium is greater in
the proximal (head) portion, than in the distal (tail) portion. Spermatozoa undergo further
maturation in the epididymis. The circular smooth muscle layer gradually increases in
thickness, and two more layers of smooth muscle are added forming an inner
longitudinal, a thick middle circular, and outer longitudinal layer upon reaching the
ductus deferens. The ductus deferens leads from the epididymis to the ejaculatory duct
and is lined with a pseudostratified columnar epithelium similar to that of the epididymis.
The tall columnar cells possess long microvilli that extend into the lumen. The lumen of
the ductus deferens is frequently observed having deep longitudinal folds and the
smooth muscle forms thick inner longitudinal, middle circular, and outer longitudinal
layers. The seminal vesicle is a tubular gland possessing a muscular and fibrous coat,
but has a highly folded mucosa. The epithelium is a pseudostratified columnar
epithelium without cilia.
Question:

The primary secretory product of the cell identified at the tip of the arrow in the attached
photomicrographic image is

Answer Choices:

A LH

B FSH

C Androgen binding protein

D Testosterone

E Estrogen

Image(s) / Chart(s):
Click image to view full size. Click open image to close. Click and hold open image to move.
D Testosterone

The cell located at the tip of the arrow in the attached photomicrographic image best
represents that of a Leydig, or interstitial cell. These cells are large, polygonal cells,
located in the interstitial tissue between adjacent seminiferous tubules. These cells may
contain lipid droplets since they are involved in testosterone biosynthesis. The
ultrastructure of the Leydig cells show extensive cytoplasmic smooth endoplasmic
reticulum, common to steroid secreting cells. Testosterone synthesis and secretion from
these cells is stimulated by the gonadotrophin-luteinizing hormone (LH) from the
anterior pituitary gland. The Sertoli cells are the supportive cells of the germinal
epithelium and are located within the seminiferous tubules. These cells synthesize
androgen-binding protein. Androgen-binding protein is regulated by FSH from the
anterior pituitary gland. Estrogens are not normally formed and secreted by Leydig cells.

Question:

Which one of the labeled cells in the attached photomicroscopic image is regulated by FSH from the
pituitary gland?

Answer Choices:

A Arrow C

B Arrow D

C Arrow E

D Arrow F

E Unlabeled arrowhead

Image(s) / Chart(s):
Click image to view full size. Click open image to close. Click and hold open image to move.

B Arrow D

The attached digitized image demonstrates the high powered histological appearance of
a seminiferous tubule from a mature testes as cut in cross section. Note that in this
image, the cellular nature of the epithelium of the seminiferous tubule is evident,
revealing the nuclear profiles of the various cells composing the germinal epithelium.
The letter 'c' represents the nuclear profiles of late spermatids. These cells are
completing the spermiogenesis process and consist of the condensed and somewhat
elongated nuclear head. A flagellum is associated with each head. Also, cytoplasm is
shed from the cell, representing residual bodies. The letter 'd' represents the nuclear
profiles of Sertoli cells.

The Sertoli cells are the supportive cells of the germinal epithelium, which are regulated
by FSH and synthesize androgen-binding protein. The developing germ cells are
embedded between adjacent Sertoli cells, and tight junctions between adjacent Sertoli
cells form the blood:testis barrier providing the basal and luminal compartments. The
letter 'e' depicts the nuclear profiles of a group of early spermatids. These cells have
completed their second meiotic division and are undergoing the changes associated
with spermiogenesis to form sperm. The letter 'f' represents the nuclear profile of a
primary spermatocyte. The primary spermatocytes, characterized by the condensed
chromosomes, represent cells involved in meiosis I, and upon completion of meiosis I,
become similar cells (but much smaller) called secondary spermatocytes, which then
undergo meiosis II to complete the meiotic division. The unlabeled arrowhead is
pointing to a type B spermatogonium in the basal portion of the germinal epithelium.
The type B spermatogonia are characterized by the large nucleus with clumps of
condensed chromatin.
Question:

Which one of the labeled cells in the attached photomicroscopic image is undergoing meiosis?

Answer Choices:

A C

B D

C E

D F

E Unlabelled arrowhead

Image(s) / Chart(s):
Click image to view full size. Click open image to close. Click and hold open image to move.

D F
The attached digitized image demonstrates the high powered histological appearance of
a seminiferous tubule from a mature testes as cut in cross section. Note that in this
image, the cellular nature of the epithelium of the seminiferous tubule is evident,
revealing the nuclear profiles of the various cells composing the germinal epithelium.
The letter 'c' represents the nuclear profiles of late spermatids. These cells are
completing the spermiogenesis process and consist of the condensed and somewhat
elongated nuclear head. A flagellum is associated with each head. Also, cytoplasm is
shed from the cell, representing residual bodies. The letter 'd' represents the nuclear
profiles of Sertoli cells.

The Sertoli cells are the supportive cells of the germinal epithelium and synthesize
androgen-binding protein. The developing germ cells are embedded between adjacent
Sertoli cells, and tight junctions between adjacent Sertoli cells form the blood:testis
barrier providing the basal and luminal compartments. The letter 'e' depicts the nuclear
profiles of a group of early spermatids. These cells have completed their second meiotic
division and are undergoing the changes associated with spermiogenesis to form
sperm. The letter 'f' represents the nuclear profile of a primary spermatocyte.

The primary spermatocytes, characterized by the condensed chromosomes,


represent cells involved in meiosis I, and upon completion of meiosis I, become similar
cells (but much smaller) called secondary spermatocytes, which then undergo meiosis II
to complete the meiotic division. The unlabeled arrowhead is pointing to a type B
spermatogonium in the basal portion of the germinal epithelium. The type B
spermatogonia are characterized by the large nucleus with clumps of condensed
chromatin.

Question:

Which one of the labeled cells in the attached photomicroscopic image depicts the cell that will
phagocytose the residual body?

Answer Choices:

A C

B D
C E

D F

E Unlabelled arrowhead

Image(s) / Chart(s):
Click image to view full size. Click open image to close. Click and hold open image to move.

B D

The attached digitized image demonstrates the high powered histological appearance of
a seminiferous tubule from a mature testes as cut in cross section. Note that in this
image, the cellular nature of the epithelium of the seminiferous tubule is evident,
revealing the nuclear profiles of the various cells composing the germinal epithelium.
The letter 'c' represents the nuclear profiles of late spermatids. These cells are
completing the spermiogenesis process and consist of the condensed and somewhat
elongated nuclear head. A flagellum is associated with each head. Also, cytoplasm is
shed from the cell, representing residual bodies. The letter 'd' represents the nuclear
profiles of Sertoli cells.

The Sertoli cells are the supportive cells of the germinal epithelium and synthesize
androgen-binding protein. The developing germ cells are embedded between adjacent
Sertoli cells, and tight junctions between adjacent Sertoli cells form the blood:testis
barrier providing the basal and luminal compartments. Sertoli cells also function to
absorb the residual bodies from the spermiogenesis process. The letter 'e' depicts the
nuclear profiles of a group of early spermatids. These cells have completed their second
meiotic division and are undergoing the changes associated with spermiogenesis to
form sperm. The letter 'f' represents the nuclear profile of a primary spermatocyte.

The primary spermatocytes, characterized by the condensed chromosomes, represent


cells involved in meiosis I. The unlabeled arrowhead is pointing to a type B
spermatogonium in the basal portion of the germinal epithelium. The type B
spermatogonia are characterized by the large nucleus with clumps of condensed
chromatin.

Question:

Which one of the cells whose labeled nucleus is depicted in the attached digitized photomicrographic
image functions in the formation of the blood:testis barrier?

Answer Choices:

A C

B D

C E

D F

E Unlabeled arrowhead

Image(s) / Chart(s):
Click image to view full size. Click open image to close. Click and hold open image to move.
B D

The attached digitized image demonstrates the high powered histological appearance of
a seminiferous tubule from a mature testes as cut in cross section. Note that in this
image, the cellular nature of the epithelium of the seminiferous tubule is evident,
revealing the nuclear profiles of the various cells composing the germinal epithelium.
The letter 'c' represents the nuclear profiles of late spermatids. These cells are
completing the spermiogenesis process and consist of the condensed and somewhat
elongated nuclear head. A flagellum is associated with each head. Also, cytoplasm is
shed from the cell, representing residual bodies. The letter 'd' represents the nuclear
profiles of Sertoli cells.

The Sertoli cells are the supportive cells of the germinal epithelium and synthesize
androgen-binding protein. The developing germ cells are embedded between adjacent
Sertoli cells, and tight junctions between adjacent Sertoli cells form the blood:testis
barrier providing the basal and luminal compartments. The letter 'e' depicts the nuclear
profiles of a group of early spermatids. These cells have completed their second meiotic
division and are undergoing the changes associated with spermiogenesis to form
sperm. The letter 'f' represents the nuclear profile of a primary spermatocyte.

The primary spermatocytes, characterized by the condensed chromosomes, represent


cells involved in meiosis I, and upon completion of meiosis I, become similar cells (but
much smaller) called secondary spermatocytes, which then undergo meiosis II to
complete the meiotic division. The unlabeled arrowhead is pointing to a type B
spermatogonium in the basal portion of the germinal epithelium. The type B
spermatogonia are characterized by the large nucleus with clumps of condensed
chromatin.
Question:

The cell whose nucleus is depicted in the attached digitized photomicrographic image labeled with the
letter 'c' is best identified as a:

Answer Choices:

A Type B spermatogonia.

B Primary spermatocyte.

C Early spermatid.

D Late spermatid.

E Sertoli cell.

Image(s) / Chart(s):
Click image to view full size. Click open image to close. Click and hold open image to move.
D Late spermatid.

The attached digitized image demonstrates the high powered histological appearance of
a seminiferous tubule from a mature testes as cut in cross section. Note that in this
image, the cellular nature of the epithelium of the seminiferous tubule is evident,
revealing the nuclear profiles of the various cells composing the germinal epithelium.
The letter 'c' represents the nuclear profiles of late spermatids. These cells are
completing the spermiogenesis process and consist of the condensed and somewhat
elongated nuclear head. A flagellum is associated with each head. Also, cytoplasm is
shed from the cell, representing residual bodies. The letter 'd' represents the nuclear
profiles of Sertoli cells.

The Sertoli cells are the supportive cells of the germinal epithelium and synthesize
androgen-binding protein. The developing germ cells are embedded between adjacent
Sertoli cells, and tight junctions between adjacent Sertoli cells form the blood:testis
barrier providing the basal and luminal compartments. The letter 'e' depicts the nuclear
profiles of a group of early spermatids. These cells have completed their second meiotic
division and are undergoing the changes associated with spermiogenesis to form
sperm. The letter 'f' represents the nuclear profile of a primary spermatocyte.

The primary spermatocytes, characterized by the condensed chromosomes, represent


cells involved in meiosis I, and upon completion of meiosis I, become similar cells (but
much smaller) called secondary spermatocytes, which then undergo meiosis II to
complete the meiotic division. The unlabeled arrowhead is pointing to a type B
spermatogonium in the basal portion of the germinal epithelium. The type B
spermatogonia are characterized by the large nucleus with clumps of condensed
chromatin.
Question:

The cells whose nucleus is at the end of the arrowhead without a letter as depicted in the attached
digitized photomicrographic image is best identified as a

Answer Choices:

A Type B spermatogonia

B Primary spermatocyte

C Early spermatid

D Leydig cell

E Sertoli cell

Image(s) / Chart(s):
Click image to view full size. Click open image to close. Click and hold open image to move.
A Type B spermatogonia

The attached digitized image demonstrates the high powered histological appearance of
a seminiferous tubule from a mature testes as cut in cross section. Note that in this
image, the cellular nature of the epithelium of the seminiferous tubule is evident,
revealing the nuclear profiles of the various cells composing the germinal epithelium.
The letter 'c' represents the nuclear profiles of late spermatids. These cells are
completing the spermiogenesis process and consist of the condensed and somewhat
elongated nuclear head. A flagellum is associated with each head. Also, cytoplasm is
shed from the cell, representing residual bodies. The letter 'd' represents the nuclear
profiles of Sertoli cells.

The Sertoli cells are the supportive cells of the germinal epithelium and synthesize
androgen-binding protein. The developing germ cells are embedded between adjacent
Sertoli cells, and tight junctions between adjacent Sertoli cells form the blood:testis
barrier providing the basal and luminal compartments. The letter 'e' depicts the nuclear
profiles of a group of early spermatids. These cells have completed their second meiotic
division and are undergoing the changes associated with spermiogenesis to form
sperm. The letter 'f' represents the nuclear profile of a primary spermatocyte.

The primary spermatocytes, characterized by the condensed chromosomes, represent


cells involved in meiosis I, and upon completion of meiosis I, become similar cells (but
much smaller) called secondary spermatocytes, which then undergo meiosis II to
complete the meiotic division. The unlabeled arrowhead is pointing to a type B
spermatogonium in the basal portion of the germinal epithelium. The type B
spermatogonia are characterized by the large nucleus with clumps of condensed
chromatin.

Question:

The cell whose nucleus is depicted in the attached digitized photomicrographic image labeled with the
letter 'f' is best identified as a

Answer Choices:

A Type B spermatogonia

B Primary spermatocyte
C Early spermatid

D Leydig cell

E Sertoli cell

Image(s) / Chart(s):
Click image to view full size. Click open image to close. Click and hold open image to move.

B Primary spermatocyte

The attached digitized image demonstrates the high powered histological appearance of
a seminiferous tubule from a mature testes as cut in cross section. Note that in this
image, the cellular nature of the epithelium of the seminiferous tubule is evident,
revealing the nuclear profiles of the various cells composing the germinal epithelium.
The letter 'c' represents the nuclear profiles of late spermatids. These cells are
completing the spermiogenesis process and consist of the condensed and somewhat
elongated nuclear head. A flagellum is associated with each head. Also, cytoplasm is
shed from the cell, representing residual bodies. The letter 'd' represents the nuclear
profiles of Sertoli cells.

The Sertoli cells are the supportive cells of the germinal epithelium and synthesize
androgen-binding protein. The developing germ cells are embedded between adjacent
Sertoli cells, and tight junctions between adjacent Sertoli cells form the blood:testis
barrier providing the basal and luminal compartments. The letter 'e' depicts the nuclear
profiles of a group of early spermatids. These cells have completed their second meiotic
division and are undergoing the changes associated with spermiogenesis to form
sperm. The letter 'f' represents the nuclear profile of a primary spermatocyte.

The primary spermatocytes, characterized by the condensed chromosomes, represent


cells involved in meiosis I, and upon completion of meiosis I, become similar cells (but
much smaller) called secondary spermatocytes, which then undergo meiosis II to
complete the meiotic division. The unlabeled arrowhead is pointing to a type B
spermatogonium in the basal portion of the germinal epithelium. The type B
spermatogonia are characterized by the large nucleus with clumps of condensed
chromatin.

Question:

At what point along the excretory passages of the male reproductive system is a layer of circular
smooth muscle first described?

Answer Choices:

A Rete testes

B Efferent ductule

C Ductus epididymis

D Ductus deferens

E Penile urethra

B Efferent ductule

The rete testes are complex anastomosing channels within the mediastinum testes that
receive spermatozoa from the tubuli recti (straight tubules) of the seminiferous tubules.
The rete testes are lined with a simple cuboidal or columnar epithelium and possess a
single cilium and/or microvilli on the apical surface. The rete testes empty into the
efferent ductule, which consist of numerous channels that lead to the head of the ductus
epididymis.

The efferent ducts are lined with a pseudostratified epithelium with alternating areas of
cuboidal cells possessing microvilli, and areas of tall columnar cells possessing cilia.
This gives the epithelium a characteristic "sawtooth" appearance when observed in
cross-section. A circularly arranged smooth muscle layer is first observed underlying the
epithelial lining in the efferent ductule.

The ductus epididymis is a highly coiled structure having a pseudostratified columnar


epithelium with prominent stereocilia present. The height of the epithelium is greater in
the proximal (head) portion, than in the distal (tail) portion. Spermatozoa undergo further
maturation in the epididymis. The circular smooth muscle layer gradually increases in
thickness, and two more layers of smooth muscle are added forming an inner
longitudinal, a thick middle circular, and outer longitudinal layer upon reaching the
ductus deferens.

The ductus deferens leads from the epididymis to the ejaculatory duct and is lined with
a pseudostratified columnar epithelium similar to that of the epididymis. The tall
columnar cells possess long microvilli that extend into the lumen. The lumen of the
ductus deferens is frequently observed having deep longitudinal folds and the smooth
muscle forms thick inner longitudinal, middle circular, and outer longitudinal layers. The
penile urethra also is lined with a pseudostratified columnar epithelium, but lacks cilia or
microvilli on the surface. At the most distal end of the penile urethra, non-keratinized
stratified squamous epithelium may be observed.

Question:

Which one of the following excretory passages of the male reproductive system are found in the
mediastinum testes as complex anastomosing channels lined with simple cuboidal epithelium whose
cells may possess a single cilia?

Answer Choices:

A Rete testes

B Efferent ductule
C Ductus epididymis

D Ductus deferens

E Penile urethra

A Rete testes

The rete testes are complex anastomosing channels within the mediastinum testes that
receive spermatozoa from the tubuli recti (straight tubules) of the seminiferous tubules.
The rete testes are lined with a simple cuboidal or columnar epithelium and possess a
single cilium and/or microvilli on the apical surface. The rete testes empty into the
efferent ductule, which consist of numerous channels that lead to the head of the ductus
epididymis.

The efferent ducts are lined with a pseudostratified epithelium with alternating areas of
cuboidal cells possessing microvilli, and areas of tall columnar cells possessing cilia.
This gives the epithelium a characteristic "sawtooth" appearance when observed in
cross-section. A circularly arranged smooth muscle layer is first observed underlying the
epithelial lining in the efferent ductule.

The ductus epididymis is a highly coiled structure having a pseudostratified columnar


epithelium with prominent stereocilia present. The height of the epithelium is greater in
the proximal (head) portion, than in the distal (tail) portion. Spermatozoa undergo further
maturation in the epididymis. The circular smooth muscle layer gradually increases in
thickness, and two more layers of smooth muscle are added forming an inner
longitudinal, a thick middle circular, and outer longitudinal layer upon reaching the
ductus deferens.

The ductus deferens leads from the epididymis to the ejaculatory duct and is lined with
a pseudostratified columnar epithelium similar to that of the epididymis. The tall
columnar cells possess long microvilli that extend into the lumen. The lumen of the
ductus deferens is frequently observed having deep longitudinal folds and the smooth
muscle forms thick inner longitudinal, middle circular, and outer longitudinal layers. The
penile urethra also is lined with a pseudostratified columnar epithelium, but lacks cilia or
microvilli. At the most distal end of the penile urethra, non-keratinized stratified
squamous epithelium may be observed.

Question:
The arrangement of the smooth muscle layers in the ductus deferens is best described as

Answer Choices:

A A thick circular layer

B Thick inner circular layer and outer longitudinal layers

C Thick inner longitudinal and outer circular layers

D Thick inner longitudinal, middle circular, and outer longitudinal layers

E Thick inner circular, middle longitudinal, and outer circular layers

D Thick inner longitudinal, middle circular, and outer longitudinal layers

The rete testes are complex anastomosing channels within the mediastinum testes that
receive spermatozoa from the tubuli recti (straight tubules) of the seminiferous tubules.
The rete testes are lined with a simple cuboidal or columnar epithelium and possess a
single cilium and/or microvilli on the apical surface. The rete testes empty into the
efferent ductule, which consist of numerous channels that lead to the head of the ductus
epididymis.

The efferent ducts are lined with a pseudostratified epithelium with alternating areas of
cuboidal cells possessing microvilli, and areas of tall columnar cells possessing cilia.
This gives the epithelium a characteristic "sawtooth" appearance when observed in
cross-section. A circularly arranged smooth muscle layer is first observed underlying the
epithelial lining in the efferent ductule.

The ductus epididymis is a highly coiled structure having a pseudostratified columnar


epithelium with prominent stereocilia present. The height of the epithelium is greater in
the proximal (head) portion, than in the distal (tail) portion. Spermatozoa undergo further
maturation in the epididymis. The circular smooth muscle layer gradually increases in
thickness, and two more layers of smooth muscle are added forming an inner
longitudinal, a thick middle circular, and outer longitudinal layer upon reaching the
ductus deferens. The ductus deferens leads from the epididymis to the ejaculatory duct
and is lined with a pseudostratified columnar epithelium similar to that of the epididymis.
The tall columnar cells possess long microvilli that extend into the lumen. The lumen of
the ductus deferens is frequently observed having deep longitudinal folds and the
smooth muscle forms thick inner longitudinal, middle circular, and outer
longitudinal layers. The penile urethra also is lined with a pseudostratified columnar
epithelium, but lacks cilia or microvilli on the surface. At the most distal end of the penile
urethra, non-keratinized stratified squamous epithelium may be observed.

Question:

Which one of the following excretory passages of the male reproductive system exhibits
pseudostratified epithelium with stereocilia and functions as an area where sperm continue their
maturation process?

Answer Choices:

A Rete testes

B Efferent ductule

C Ductus epididymis

D Ductus deferens

E Penile urethra

C Ductus epididymis

The rete testes are complex anastomosing channels within the mediastinum testes that
receive spermatozoa from the tubuli recti (straight tubules) of the seminiferous tubules.
The rete testes are lined with a simple cuboidal or columnar epithelium and possess a
single cilium and/or microvilli on the apical surface. The rete testes empty into the
efferent ductule, which consist of numerous channels that lead to the head of the ductus
epididymis.

The efferent ducts are lined with a pseudostratified epithelium with alternating areas of
cuboidal cells possessing microvilli, and areas of tall columnar cells possessing cilia.
This gives the epithelium a characteristic "sawtooth" appearance when observed in
cross-section. A circularly arranged smooth muscle layer is first observed underlying the
epithelial lining in the efferent ductule.

The ductus epididymis is a highly coiled structure having a pseudostratified columnar


epithelium with prominent stereocilia present. The height of the epithelium is greater in
the proximal (head) portion, than in the distal (tail) portion. Spermatozoa undergo further
maturation in the epididymis. The circular smooth muscle layer gradually increases in
thickness, and two more layers of smooth muscle are added forming an inner
longitudinal, a thick middle circular, and outer longitudinal layer upon reaching the
ductus deferens. The ductus deferens leads from the epididymis to the ejaculatory duct
and is lined with a pseudostratified columnar epithelium similar to that of the epididymis.
The tall columnar cells possess long microvilli that extend into the lumen. The lumen of
the ductus deferens is frequently observed having deep longitudinal folds and the
smooth muscle forms thick inner longitudinal, middle circular, and outer longitudinal
layers. The penile urethra also is lined with a pseudostratified columnar epithelium, but
lacks cilia or microvilli. At the most distal end of the penile urethra, non-keratinized
stratified squamous epithelium may be observed.

Question:

Which one of the following excretory passages of the male reproductive system exhibits
pseudostratified epithelium with alternating areas of cuboidal cells possessing microvilli, and areas of
tall columnar cells possessing cilia

Answer Choices:

A Rete testes

B Efferent ductule

C Ductus epididymis

D Ductus deferens
E Penile urethra

B Efferent ductule

The rete testes are complex anastomosing channels within the mediastinum testes that
receive spermatozoa from the tubuli recti (straight tubules) of the seminiferous tubules.
The rete testes are lined with a simple cuboidal or columnar epithelium and possess a
single cilium and/or microvilli on the apical surface. The rete testes empty into the
efferent ductule, which consist of numerous channels that lead to the head of the ductus
epididymis.

The efferent ducts are lined with a pseudostratified epithelium with alternating areas of
cuboidal cells possessing microvilli, and areas of tall columnar cells possessing cilia,
giving the epithelium a characteristic "sawtooth" appearance when observed in cross-
section. A circularly arranged smooth muscle layer is first observed underlying the
epithelial lining in the efferent ductule.

The ductus epididymis is a highly coiled structure having a pseudostratified columnar


epithelium with prominent stereocilia present. The height of the epithelium is greater in
the proximal (head) portion, than in the distal (tail) portion. Spermatozoa undergo further
maturation in the epididymis. The circular smooth muscle layer gradually increases in
thickness, and two more layers of smooth muscle are added forming an inner
longitudinal, a thick middle circular, and outer longitudinal layer upon reaching the
ductus deferens. The ductus deferens leads from the epididymis to the ejaculatory duct
and is lined with a pseudostratified columnar epithelium similar to that of the epididymis.
The tall columnar cells possess long microvilli that extend into the lumen. The lumen of
the ductus deferens is frequently observed having deep longitudinal folds and the
smooth muscle forms thick inner longitudinal, middle circular, and outer longitudinal
layers. The penile urethra also is lined with a pseudostratified columnar epithelium, but
lacks cilia or microvilli. At the most distal end of the penile urethra, non-keratinized
stratified squamous epithelium may be observed.

Question:

The structural feature that best distinguishes the ductus deferens from the other genital ducts is its
Answer Choices:

A Smooth-bore lumen

B Thick muscular wall containing three muscle layers

C Linings of transitional epithelium

D Flattened mucosa

E Motile stereocilia

B Thick muscular wall containing three muscle layers

The ductus (vas) deferens possesses three layers of smooth muscle in its wall, whereas
the other genital ducts do not. As in the ductus epididymidis, the ductus deferens is
lined by a pseudostratified columnar epithelium. The principal cells of the epithelium
possess nonmotile stereocilia.

Question:

Spermatozoa are conveyed from the seminiferous tubules to the rete testis via the

Answer Choices:

A Ductus epididymidis

B Tubuli recti

C Ductuli efferentes
D Ductus deferens

B Tubuli recti

The seminiferous tubules are connected to the rete testis by the tubuli recti. Ductuli
efferentes are a collection of 10-20 tubules leading from the rete testis to the ductus
epididymidis, which together with the ductuli efferentes constitutes the epididymis.
Ductus (vas) deferens is the continuation of the epididymis. It is a straight tube with a
thick, muscular wall which continues towards the prostatic urethra and empties into it.

Question:

Type A spermatogonia are germ cells that

Answer Choices:

A Develop from secondary spermatocytes

B Undergo meiotic activity subsequent to sexual maturity

C Develop through meiotic divisions

D Give rise to primary spermatids

E May be dark or pale

E May be dark or pale

Type A spermatogonia, which may be pale or dark, are primitive germ cells. Pale type A
spermatogonia become mitotically active at puberty and give rise to type B
spermatogonia. Type B spermatogonia undergo mitoses to give rise to primary
spermatocytes.

Question:

Testosterone is produced by which of the following cells?

Answer Choices:

A Cells of Leydig

B Sertoli cells

C Spermatogonia

D Spermatids

E Spermatocytes

A Cells of Leydig

The hormone, testosterone, is produced by the interstitial cells of Leydig. Sertoli cells,
which support and protect spermatogenic cells, (i) secrete a fluid to transport
spermatozoa to genital ducts, (ii) synthesize androgen-binding protein (ABP) under the
influence of FSH (follicle stimulating hormone), and (iii) secrete inhibin, a hormone that
inhibits the synthesis and release of FSH (follicle stimulating hormone) by the anterior
pituitary. Spermatogonia, spermatids, and spermatocytes are male germ cells, and they
do not secrete any known substance.
Question:

The Wolffian body (or mesonephros), from which urinary and generative organs in both sexes develop,
persists in the male as

Answer Choices:

A Hydatid of Morgagni

B Epididymis

C Scrotum

D Penis

E Tunica vaginalis

B Epididymis

The epididymis, the vas deferens, and the ejaculatory duct are formed from the Wolffian
duct. Hydatid of Morgagni (visible between the testis and globus major) is the remains
of the duct of Muller. Scrotum is formed from the part of the skin from the groin. Tunica
vaginalis is the process of peritoneum surrounding the testis. Male penis develops from
genital eminence.

Question:

Which one of the following excretory passages of the male reproductive system exhibits
pseudostratified epithelium with stereocilia, and the wall of the duct has thick layers of smooth muscle?
Answer Choices:

A Rete testes

B Efferent ductule

C Ductus epididymis

D Ductus deferens

E Penile urethra

D Ductus deferens

The rete testes are complex anastomosing channels within the mediastinum testes that
receive spermatozoa from the tubuli recti (straight tubules) of the seminiferous tubules.
The rete testes are lined with a simple cuboidal or columnar epithelium and possess a
single cilium and/or microvilli on the apical surface. The rete testes empty into the
efferent ductule, which consist of numerous channels that lead to the head of the ductus
epididymis.

The efferent ducts are lined with a pseudostratified epithelium with alternating areas of
cuboidal cells possessing microvilli, and areas of tall columnar cells possessing cilia.
This gives the epithelium a characteristic "sawtooth" appearance when observed in
cross-section. A circularly arranged smooth muscle layer is first observed underlying the
epithelial lining in the efferent ductule. The ductus epididymis is a highly coiled structure
having a pseudostratified columnar epithelium with prominent stereocilia present. The
height of the epithelium is greater in the proximal (head) portion, than in the distal (tail)
portion. Spermatozoa undergo further maturation in the epididymis. The circular smooth
muscle layer gradually increases in thickness, and two more layers of smooth muscle
are added forming an inner longitudinal, a thick middle circular, and outer longitudinal
layer upon reaching the ductus deferens.

The ductus deferens leads from the epididymis to the ejaculatory duct and is lined with
a pseudostratified columnar epithelium similar to that of the epididymis. The tall
columnar cells possess long microvilli that extend into the lumen. The lumen of the
ductus deferens is frequently observed having deep longitudinal folds and the smooth
muscle forms thick inner longitudinal, middle circular, and outer longitudinal layers. The
penile urethra also is lined with a pseudostratified columnar epithelium, but lacks cilia or
microvilli. At the most distal end of the penile urethra, non-keratinized stratified
squamous epithelium may be observed.

Question:

Which one of the following excretory passages of the male reproductive system exhibits
pseudostratified epithelium without surface modifications such as stereocilia or cilia

Answer Choices:

A Rete testes

B Efferent ductule

C Ductus epididymis

D Ductus deferens

E Penile urethra

E Penile urethra

The rete testes are complex anastomosing channels within the mediastinum testes that
receive spermatozoa from the tubuli recti (straight tubules) of the seminiferous tubules.
The rete testes are lined with a simple cuboidal or columnar epithelium and possess a
single cilium and/or microvilli on the apical surface. The rete testes empty into the
efferent ductule, which consist of numerous channels that lead to the head of the ductus
epididymis.

The efferent ducts are lined with a pseudostratified epithelium with alternating areas of
cuboidal cells possessing microvilli, and areas of tall columnar cells possessing cilia.
This gives the epithelium a characteristic "sawtooth" appearance when observed in
cross-section. A circularly arranged smooth muscle layer is first observed underlying the
epithelial lining in the efferent ductule.
The ductus epididymis is a highly coiled structure having a pseudostratified columnar
epithelium with prominent stereocilia present. The height of the epithelium is greater in
the proximal (head) portion, than in the distal (tail) portion. Spermatozoa undergo further
maturation in the epididymis. The circular smooth muscle layer gradually increases in
thickness, and two more layers of smooth muscle are added forming an inner
longitudinal, a thick middle circular, and outer longitudinal layer upon reaching the
ductus deferens. The ductus deferens leads from the epididymis to the ejaculatory duct
and is lined with a pseudostratified columnar epithelium similar to that of the epididymis.
The tall columnar cells possess long microvilli that extend into the lumen. The lumen of
the ductus deferens is frequently observed having deep longitudinal folds and the
smooth muscle forms thick inner longitudinal, middle circular, and outer longitudinal
layers.

The penile urethra also is lined with a pseudostratified columnar epithelium, but lacks
cilia or microvilli. At the most distal end of the penile urethra, non-keratinized stratified
squamous epithelium may be observed.

Question:

The cells whose nucleus is depicted in the attached digitized photomicrographic image labeled with the
letter 'd' is best identified as a

Answer Choices:

A Type B spermatogonia

B Primary spermatocyte

C Early spermatid

D Leydig cell

E Sertoli cell

Image(s) / Chart(s):
Click image to view full size. Click open image to close. Click and hold open image to move.
E Sertoli cell

The attached digitized image demonstrates the high powered histological appearance of
a seminiferous tubule from a mature testes as cut in cross section. Note that in this
image, the cellular nature of the epithelium of the seminiferous tubule is evident,
revealing the nuclear profiles of the various cells composing the germinal epithelium.
The letter 'c' represents the nuclear profiles of late spermatids. These cells are
completing the spermiogenesis process and consist of the condensed and somewhat
elongated nuclear head. A flagellum is associated with each head. Also, cytoplasm is
shed from the cell, representing residual bodies. The letter 'd' represents the nuclear
profiles of Sertoli cells.

The Sertoli cells are the supportive cells of the germinal epithelium and synthesize
androgen-binding protein. The developing germ cells are embedded between adjacent
Sertoli cells, and tight junctions between adjacent Sertoli cells form the blood:testis
barrier providing the basal and luminal compartments. The letter 'e' depicts the nuclear
profiles of a group of early spermatids. These cells have completed their second meiotic
division and are undergoing the changes associated with spermiogenesis to form
sperm. The letter 'f' represents the nuclear profile of a primary spermatocyte.

The primary spermatocytes, characterized by the condensed chromosomes, represent


cells involved in meiosis I, and upon completion of meiosis I, become similar cells (but
much smaller) called secondary spermatocytes, which then undergo meiosis II to
complete the meiotic division. The unlabeled arrowhead is pointing to a type B
spermatogonium in the basal portion of the germinal epithelium. The type B
spermatogonia are characterized by the large nucleus with clumps of condensed
chromatin.
Question:

Which one of the labeled cells in the attached photomicroscopic image has a single flagellum?

Answer Choices:

A C

B D

C E

D F

E Unlabelled arrowhead

Image(s) / Chart(s):
Click image to view full size. Click open image to close. Click and hold open image to move.

A C
The attached digitized image demonstrates the high powered histological appearance of
a seminiferous tubule from a mature testes as cut in cross section. Note that in this
image, the cellular nature of the epithelium of the seminiferous tubule is evident,
revealing the nuclear profiles of the various cells composing the germinal epithelium.
The letter 'c' represents the nuclear profiles of late spermatids. These cells are
completing the spermiogenesis process and consist of the condensed and somewhat
elongated nuclear head. A flagellum is associated with each head. Also, cytoplasm is
shed from the cell, representing residual bodies. The letter'd' represents the nuclear
profiles of Sertoli cells.

The Sertoli cells are the supportive cells of the germinal epithelium and synthesize
androgen-binding protein. The developing germ cells are embedded between adjacent
Sertoli cells, and tight junctions between adjacent Sertoli cells form the blood:testis
barrier providing the basal and luminal compartments. The letter 'e' depicts the nuclear
profiles of a group of early spermatids. These cells have completed their second meiotic
division and are undergoing the changes associated with spermiogenesis to form
sperm. The letter 'f' represents the nuclear profile of a primary spermatocyte.

The primary spermatocytes, characterized by the condensed chromosomes, represent


cells involved in meiosis I, and upon completion of meiosis I, become similar cells (but
much smaller) called secondary spermatocytes, which then undergo meiosis II to
complete the meiotic division. The unlabeled arrowhead is pointing to a type B
spermatogonium in the basal portion of the germinal epithelium. The type B
spermatogonia are characterized by the large nucleus with clumps of condensed
chromatin.

Question:

The cell identified at the tip of the arrow in the attached photomicrographic image is best identified as a

Answer Choices:

A Leydig cell
B Sertoli cell

C Capillary endothelial cell

D Spermatogonium

E Peritubular contractile cell

Image(s) / Chart(s):
Click image to view full size. Click open image to close. Click and hold open image to move.

A Leydig cell

The cell located at the tip of the arrow in the attached photomicrographic image best
represents that of a Leydig, or interstitial cell. These cells are large, polygonal cells,
located in the interstitial tissue between adjacent seminiferous tubules. These cells may
contain lipid droplets since they are involved in testosterone biosynthesis. The
ultrastructure of the Leydig cells show extensive cytoplasmic smooth endoplasmic
reticulum, common to steroid secreting cells. The gonadotrophin-luteinizing hormone
(LH) stimulates testosterone synthesis and secretion from these cells from the anterior
pituitary gland.
The Sertoli cells are the supportive cells of the germinal epithelium and are located
within the seminiferous tubules. These cells synthesize androgen-binding protein. The
developing germ cells are embedded between adjacent Sertoli cells, and tight junctions
between adjacent Sertoli cells form the blood:testis barrier providing the basal and
luminal compartments. The interstitial area surrounding the seminiferous tubule shows
rich vascular innervation and an endothelial nucleus may be seen in cross sections of
capillaries. Also located in the interstitial area, and surrounding the seminiferous tubules
are specialized contractile cells referred to as myoid cells or peritubular contractile cells.
These cells resemble smooth muscle cells, ultrastructurally possessing actin filaments.
Contraction of these cells provides peristaltic waves that help move spermatozoa and
testicular fluid through the lumen of the seminiferous tubules towards the epididymis.
Spermatogonium is located in the basal portion of the germinal epithelium and is
characterized by a large nucleus with clumps of condensed chromatin. These are the
stem cells for formation of spermatozoa.

Question:

The organ as depicted in the digitized photomicroscopic image is best identified as the

Answer Choices:

A Rete testes

B Efferent ductule

C Ductus epididymis

D Ductus deferens

E Seminal vesicle

Image(s) / Chart(s):
Click image to view full size. Click open image to close. Click and hold open image to move.
D Ductus deferens

The rete testes are complex anastomosing channels within the mediastinum testes that
receive spermatozoa from the tubuli recti (straight tubules) of the seminiferous tubules.
The rete testes are lined with a simple cuboidal or columnar epithelium and possess a
single cilium and/or microvilli on the apical surface. The rete testes empty into the
efferent ductule, which consist of numerous channels that lead to the head of the ductus
epididymis. The efferent ducts are lined with a pseudostratified epithelium with
alternating areas of cuboidal cells possessing microvilli, and areas of tall columnar cells
possessing cilia. This gives the epithelium a characteristic "sawtooth" appearance when
observed in cross-section. A circularly arranged smooth muscle layer is first observed
underlying the epithelial lining in the efferent ductule.

The ductus epididymis is a highly coiled structure having a pseudostratified columnar


epithelium with prominent stereocilia present. The height of the epithelium is greater in
the proximal (head) portion, than in the distal (tail) portion. Spermatozoa undergo further
maturation in the epididymis. The circular smooth muscle layer gradually increases in
thickness, and two more layers of smooth muscle are added forming an inner
longitudinal, a thick middle circular, and outer longitudinal layer upon reaching the
ductus deferens. The ductus deferens leads from the epididymis to the ejaculatory duct
and is lined with a pseudostratified columnar epithelium similar to that of the epididymis.
The tall columnar cells possess long microvilli that extend into the lumen. The lumen of
the ductus deferens is frequently observed having deep longitudinal folds and the
smooth muscle forms thick inner longitudinal, middle circular, and outer longitudinal
layers. The seminal vesicle is a tubular gland possessing a muscular and fibrous coat,
but has a highly folded mucosa. The epithelium is a pseudostratified columnar
epithelium without cilia.

Question:

The best description of the epithelial lining of the tubuloalveolar glands within the prostate is
Answer Choices:

A Pseudostratified columnar

B Simple columnar

C Simple squamous

D Simple cuboidal

E Variable

E Variable

The prostate gland is an exocrine gland having a compound tubuloalveolar structure.


The glandular epithelium is most frequently observed as pseudostratified columnar, but
is extremely variable and may be seen as simple columnar, cuboidal, or even
squamous. A collection of 30-50 tubuloalveolar glands is embedded within fibroelastic
stroma, which contains smooth muscle randomly dispersed throughout the stroma.
Prostatic concretions (corpora amylacea) are commonly found in the prostatic alveoli,
and increase in number and size with age. The primary secretory products of the
prostate gland are citric acid and acid phosphatase, which are released into the
prostatic urethra at the time of ejaculation.

Question:

Which of the following statements concerning the cells of Leydig is true?

Answer Choices:
A They become functional at puberty

B They are located within the seminiferous tubules

C They are stimulated by FSH (follicle stimulating hormone)

D They secrete much of the fluid portion of semen

E They respond to inhibin

A They become functional at puberty


Image(s) / Chart(s):
Click image to view full size. Click open image to close. Click and hold open image to
move.

Leydig cells become functional at puberty because of the action of LH (luteinizing


hormone) produced by the adenohypophysis. These cells are located in between the
seminiferous tubules (see the attached Figure). They are endocrine cells and secrete
testosterone. Inhibin is secreted by the Sertoli cells. This hormone inhibits the synthesis
and release of FSH (follicle stimulating hormone) by the anterior pituitary.
Question:

A 40-year-old male desires to be sterilized but fears any invasive procedure to his penis or testicles.
His physician reassures the male by telling him that a bilateral vasectomy is an effective sterilization
procedure that is non-invasive to the penis or testicles. Instead, bilateral vasectomy accesses, (just
above the penis and testicles near the pubic hair), to excise a portion of which semen-conveying duct?

Answer Choices:

A Urethra

B Epididymis

C Seminal vesicle

D Vas deferens

E Ejaculatory duct

D Vas deferens

The vas deferens are a pair of muscular ducts each arising from a testicle. Mature
sperm is stored in the coiled tubules of the epididymis at the testicles, and upon
ejaculation, sperm pass from the epididymis to the vas deferens. The ducts of the vas
deferens run around the pubis bone and behind the urinary bladder to join with the
ejaculatory duct at the prostate. Bilateral vasectomy excises the ducts of the vas
deferens above the penis near the pubis bone, without invasive treatment to the penis
or testicles.

The seminal vesicle is a gland below and behind the urinary bladder that secretes
amino acid and fructose containing mucus into the semen at the ejaculatory duct and
provides an energy source for the sperm. The ejaculatory duct combines with the
urethra, the tube that drains both urine and semen. The urethra runs through the penis
and opens to the outside at the tip of the penis.

Keywords: Vas deferens, sterilized

Question:

The cell whose nucleus is depicted in the attached digitized photomicrographic image labeled with the
letter 'e' is best identified as a

Answer Choices:

A Type B spermatogonia

B Primary spermatocyte

C Early spermatid

D Leydig cell

E Sertoli cell

Image(s) / Chart(s):
Click image to view full size. Click open image to close. Click and hold open image to move.
C Early spermatid

The attached digitized image demonstrates the high powered histological appearance of
a seminiferous tubule from a mature testes as cut in cross section. Note that in this
image, the cellular nature of the epithelium of the seminiferous tubule is evident,
revealing the nuclear profiles of the various cells composing the germinal epithelium.
The letter 'c' represents the nuclear profiles of late spermatids. These cells are
completing the spermiogenesis process and consist of the condensed and somewhat
elongated nuclear head. A flagellum is associated with each head. Also, cytoplasm is
shed from the cell, representing residual bodies. The letter 'd' represents the nuclear
profiles of Sertoli cells.

The Sertoli cells are the supportive cells of the germinal epithelium and synthesize
androgen-binding protein. The developing germ cells are embedded between adjacent
Sertoli cells, and tight junctions between adjacent Sertoli cells form the blood:testis
barrier providing the basal and luminal compartments. The letter 'e' depicts the nuclear
profiles of a group of early spermatids. These cells have completed their second
meiotic division and are undergoing the changes associated with spermiogenesis to
form sperm. The letter 'f' represents the nuclear profile of a primary spermatocyte.

The primary spermatocytes, characterized by the condensed chromosomes, represent


cells involved in meiosis I, and upon completion of meiosis I, become similar cells (but
much smaller) called secondary spermatocytes, which then undergo meiosis II to
complete the meiotic division. The unlabeled arrowhead is pointing to a type B
spermatogonium in the basal portion of the germinal epithelium. The type B
spermatogonia are characterized by the large nucleus with clumps of condensed
chromatin.
Question:

The organ as depicted in the attached digitized photomicroscopic image is best identified as the

Answer Choices:

A Rete testes

B Efferent ductule

C Ductus epididymis

D Ductus deferens

E Seminal vesicle

Image(s) / Chart(s):
Click image to view full size. Click open image to close. Click and hold open image to move.
E Seminal vesicle

The rete testes are complex anastomosing channels within the mediastinum testes that
receive spermatozoa from the tubuli recti (straight tubules) of the seminiferous tubules.
The rete testes are lined with a simple cuboidal or columnar epithelium and possess a
single cilium and/or microvilli on the apical surface. The rete testes empty into the
efferent ductule, which consist of numerous channels that lead to the head of the ductus
epididymis. The efferent ducts are lined with a pseudostratified epithelium with
alternating areas of cuboidal cells possessing microvilli, and areas of tall columnar cells
possessing cilia. This gives the epithelium a characteristic "sawtooth" appearance when
observed in cross-section. A circularly arranged smooth muscle layer is first observed
underlying the epithelial lining in the efferent ductule.

The ductus epididymis is a highly coiled structure having a pseudostratified columnar


epithelium with prominent stereocilia present. The height of the epithelium is greater in
the proximal (head) portion, than in the distal (tail) portion. Spermatozoa undergo further
maturation in the epididymis. The circular smooth muscle layer gradually increases in
thickness, and two more layers of smooth muscle are added forming an inner
longitudinal, a thick middle circular, and outer longitudinal layer upon reaching the
ductus deferens. The ductus deferens leads from the epididymis to the ejaculatory duct
and is lined with a pseudostratified columnar epithelium similar to that of the epididymis.
The tall columnar cells possess long microvilli that extend into the lumen. The lumen of
the ductus deferens is frequently observed having deep longitudinal folds and the
smooth muscle forms thick inner longitudinal, middle circular, and outer longitudinal
layers. The seminal vesicle is a tubular gland possessing a muscular and fibrous coat,
but has a highly folded mucosa. The epithelium is a pseudostratified columnar
epithelium without cilia.

Question:

The attached photomicroscopic image depicts which one of the following organs?

Answer Choices:

A Seminal vesicle

B Mature testes
C Immature testes

D Ductus epididymis

E Kidney cortex

Image(s) / Chart(s):
Click image to view full size. Click open image to close. Click and hold open image to move.

B Mature testes

The attached digitized image demonstrates the low powered histological appearance of
mature testes as cut in cross section. Note the thick connective tissue capsule, the
tunica albuginea, on the surface of the organ. The seminiferous tubules, containing the
stratified epithelial germinal epithelium, are highly coiled and appear cut in cross-
section, longitudinal section, or obliquely. While the identification of individual cells are
not apparent in this image, the cellular nature of the epithelium, and the highly coiled
appearance of the tubules are evident. The histological appearance of immature testes
would show less developed seminiferous tubule epithelium, which consists primarily of
two cell types (sertoli cells and spermatogonia), with little or no lumen present.

The ductus epididymis is a highly coiled structure having a pseudostratified columnar


epithelium with prominent stereocilia present. The height of the epithelium is greater in
the proximal (head) portion, than in the distal (tail) portion. Spermatozoa undergo further
maturation in the epididymis. The ductus deferens has an inner longitudinal, a thick
middle circular, and outer longitudinal layer of smooth muscle. The low powered image
of the kidney cortex may have histological similarities, but the kidney capsule is not as
thick or predominant, and the parenchyma of the kidney cortex consists of tubules and
glomeruli.

Question:

The attached photomicroscopic image depicts which one of the following organs?

Answer Choices:

A Seminal vesicle

B Mature testes

C Immature testes

D Ductus epididymis

E Prostate gland

Image(s) / Chart(s):
Click image to view full size. Click open image to close. Click and hold open image to move.
B Mature testes

The attached digitized image demonstrates the high powered histological appearance of
a seminiferous tubule from a mature testes as cut in cross section. Note that in this
image, the cellular nature of the epithelium of the seminiferous tubule is evident,
revealing the nuclear profiles of the various cells composing the germinal epithelium.
The histological appearance of immature testes would show less developed
seminiferous tubule epithelium, which consists primarily of 2 cell types (sertoli cells and
spermatogonia), with little or no lumen present.

The ductus epididymis is a highly coiled structure having a pseudostratified columnar


epithelium with prominent stereocilia present. The height of the epithelium is greater in
the proximal (head) portion than in the distal (tail) portion. Spermatozoa undergo further
maturation in the epididymis. The ductus deferens has an inner longitudinal, a thick
middle circular, and outer longitudinal layer of smooth muscle. The epithelium also is
pseudostratified columnar with stereocilia. The prostate gland is a compound tubular-
alveolar gland with a pseudostratified columnar epithelium, although the epithelium may
be extremely variable. Furthermore, the gland is embedded in a connective tissue
stroma and the alveoli may possess prostatic concretions.

Question:

A 65-year-old man comes in to see you. He is complaining that he is having difficulty urinating, and
most often just a dribble of urine comes out. He does not experience any burning in the urethra, and
his urine specimen reveals no abnormal findings. You conduct a urine flow test on the patient, and find
a significantly decreased urine flow. In the image provided, identify the area that is the most likely
location for the cause of this patient's urinary difficulties
Answer Choices:

A A

B B

C C

D D

E E

Image(s) / Chart(s):
Click image to view full size. Click open image to close. Click and hold open image to move.

B B
Inability or reduced ability to urinate are common symptoms of benign prostatic
hyperplasia (BPH) or enlarged prostate. With irritated or swollen urethra, urinary tract
infection, and yeast infection ruled out by a negative urine sample, and the urine flow
test showing that the patient has a significantly decreased urine flow, the symptoms
point to BPH. A in the diagram points to the seminal vesicle, B points to the prostate
gland, C points to the Cowper's gland, D points to the vas deferens, and E points to the
penis.

Question:

A 30-year-old man consults a gynecologist along with his wife for infertility. On questioning, he informs
the doctor that he never suffered from any major illness in the past and also got married 5 years ago.
However, his physical examination reveals failure of testes to descend down into the scrotum. The
semen analysis shows semen quantity to be 2 ml and absence of spermatozoa. The blood lab report
shows plasma testosterone of 170 ng/dL (normal range of serum testosterone -280-980 ng/dL). The
factor responsible for the absence of spermatozoa in this patient is:

Answer Choices:

A Degeneration of the epithelium of seminiferous tubules

B Decreased secretion of gonadotrophins

C Decreased concentration of testosterone

D Hypofunctioning of the pituitary gland

E Increased level of inhibin

A Degeneration of the epithelium of seminiferous tubules

Primary infertility affects 15-20% of married couples. Apart from case history and
physical examination, endocrinal profile and semen analysis are essential to disclose
the underlying cause.
The testes have 2 distinct but related functions, both of which are under
adenohypophysial and hypothalamic control. The first one is the production and storage
of viable spermatozoa, and the second one is the synthesis and secretion of androgenic
hormones. Various factors that can induce sterility in men are trauma, infections like
mumps, and environmental factors such as excessive heat, medications, and/or drugs.

Because of the failure of testes to descend into the scrotum, seminiferous tubules
remain infantile in structure, and due to greater temperature of the inguinal canal, they
degenerate. This results in oligospermia or azoospermia and subsequent male infertility.
This spermatogenic impairment could be related to lack of timely surgical correction;
however, the secondary sexual characteristics are not affected.

Although gonadotrophins and testosterone play a role in descent of the testes, they are
not responsible directly for absence of spermatozoa. Hypofunctioning pituitary and high
levels of inhibin do not cause absence of spermatozoa.

Question:

As illustrated by the labeled nuclear profiles in the attached photomicroscopic image, the germ cells
are embedded or recessed within the cytoplasm of which one of the cells?

Answer Choices:

A C

B D

C E

D F

E Unlabeled arrowhead

Image(s) / Chart(s):
Click image to view full size. Click open image to close. Click and hold open image to move.
B D

The attached digitized image demonstrates the high powered histological appearance of
a seminiferous tubule from a mature testes as cut in cross section. Note that in this
image the cellular nature of the epithelium of the seminiferous tubule is evident,
revealing the nuclear profiles of the various cells that compose the germinal epithelium.
The letter 'c' represents the nuclear profiles of late spermatids. These cells are
completing the spermiogenesis process and consist of the condensed and somewhat
elongated nuclear head. A flagellum is associated with each head. Also, cytoplasm is
shed from the cell, representing residual bodies. The letter 'd' represents the nuclear
profiles of Sertoli cells.

The Sertoli cells are the supportive cells of the germinal epithelium and synthesize
androgen-binding protein. The developing germ cells are embedded within the
cytoplasm of the Sertoli cells to become mature spermatozoa with an elongated nuclei
and a flagellum. Sertoli cells form the blood: testis barrier providing the basal and
luminal compartments. The letter 'e' depicts the nuclear profiles of a group of early
spermatids. These cells have completed their second meiotic division and are
undergoing the changes associated with spermiogenesis to form sperm. The letter 'f'
represents the nuclear profile of a primary spermatocyte.

The primary spermatocytes, characterized by the condensed chromosomes, undergo


the first meiotic division and upon completion of meiosis I become secondary
spermatocytes, which then undergo second meiotic division (without synthesizing new
DNA) to complete the meiotic division and result in spermatids. The unlabeled
arrowhead is pointing to a type B spermatogonium in the basal portion of the germinal
epithelium. The type B spermatogonia are characterized by the large nucleus with
clumps of condensed chromatin
Question:

A twins pregnancy was terminated in the third month of gestation. One fetus is male, another is female.
The proof that one fetus is male would be presence of a derivate of what structure?

Answer Choices:

A Cloaca

B Urogenital sinus

C Wolffian duct

D Duct of Muller

E Gartner's duct

C Wolffian duct

Males become externally distinct between 8 and 12 weeks. In males, the Wolffian
duct develops the epididymus, the vas deferens, and the seminal vesicle. In the female,
in the absence of testosterone support, the Wolffian duct regresses.

Cloaca is present in both sexes. Human beings have only an embryonic cloaca, which
is split up into separate tracts during the development of urogenital system.

The urogenital sinus is present in both sexes during the development of the urinary and
reproductive organs.

Müllerian ducts (or paramesonephric ducts) are paired ducts of the embryo that will
develop to form the Fallopian tubes, uterus, cervix, and the upper portion of the vagina;
in the male, they are lost.

Gartner's duct is a potential embryological remnant in human female development of the


paired mesonephric ducts. In contrast, mesonephric ducts in the male go on to form the
paired epididymis, ductus deferens, ejaculatory duct, and seminal vesicle.

SECTION- ENDOCRINE
HYPOPHYSIS
Question:

The secretory product of the cell as identified at the tip of the arrow in the attached photomicrographic
image is regulated primarily by

Answer Choices:

A GnRH

B LH

C FSH

D Androgen binding protein

E Estrogen

Image(s) / Chart(s):
Click image to view full size. Click open image to close. Click and hold open image to move.
B LH

The cell located at the tip of the arrow in the attached photomicrographic image best
represents that of a Leydig, or interstitial cell. These cells are large, polygonal cells,
located in the interstitial tissue between adjacent seminiferous tubules. These cells may
contain lipid droplets since they are involved in testosterone biosynthesis. The
ultrastructure of the Leydig cells show extensive cytoplasmic smooth endoplasmic
reticulum, common to steroid secreting cells. Testosterone synthesis and secretion from
these cells is stimulated by the gonadotrophin-luteinizing hormone (LH) from the
anterior pituitary gland. The Sertoli cells are the supportive cells of the germinal
epithelium and are located within the seminiferous tubules. These cells synthesize
androgen-binding protein as regulated by FSH. Gonadotrophin hormone releasing
hormone (GnRH) is a neuroendocrine peptide secreted by the hypothalamus, which
stimulates the synthesis and secretion of the pituitary gonadotropins, LH and FSH.

Question:

The primary function of the postganglionic fibers of the autonomic nervous system that enter the
adenohypophysis is to
Answer Choices:

A Stimulate ACTH secretion

B Stimulate prolactin secretion

C Inhibit oxytocin secretion

D Modulate vascular flow

E Modulate neurohormone secretion

D Modulate vascular flow

The primary function of the postganglionic fibers of the autonomic nervous system that
enter the adenohypophysis is to modulate blood flow through the capillaries and
sinuses. The postganglionic fibers of the autonomic nervous system have no intrinsic
function in the modulation of any of the hormones secreted by the adenohypophysis or
neurohypophysis.

Question:

Blood supply to the pars nervosa is derived primarily from which one of the following vessels?

Answer Choices:

A Superior hypophyseal artery

B Primary capillary plexus of the hypophyseal portal veins


C Hypophyseal portal veins

D Secondary capillary plexus of the hypophyseal portal veins

E Inferior hypophyseal artery

E Inferior hypophyseal artery

The vascular supply to the pars nervosa is derived primarily from the inferior
hypophyseal arteries. The superior hypophyseal arteries give rise to a primary portal
plexus of capillaries of the median eminence which bathe the neurosecretory cells
which produce releasing factors from the hypothalamus. These capillaries coalesce into
portal veins called the hypophyseal portal veins that run inferiorly along the pars
tuberalis and infundibular stalk to give rise to another anastomotic plexus, the
secondary portal plexus. This plexus will allow releasing factors from the hypothalamus
to access the cells of the anterior pituitary gland and regulate the synthesis and
secretion of the trophic hormones. Similarly, the trophic hormones of the anterior
pituitary are secreted into the secondary portal plexus for transport to their respective
target organs.

Question:

The synthesis and secretion of growth hormone from the adenohypophyseal acidophil is regulated by
which hypothalamic hormone?

Answer Choices:

A GnRH

B CRH

C TRH

D Dopamine
E GHRH

E GHRH

The synthesis and secretion of ACTH from the adenohypophyseal basophil is regulated
by the hypothalamic releasing hormone, corticotropin releasing hormone (CRH). GnRH,
gonadotropin releasing hormone, acts to stimulate the synthesis and release of FSH
and/or LH from the gonadotrope. TRH, thyrotropin releasing hormone, acts to stimulate
the synthesis and release of TSH (thyroid stimulating hormone) from the thyrotrope.
Dopamine is the putative prolactin inhibiting factor. GHRH, growth hormone releasing
hormone, acts to stimulate the synthesis and secretion of GH from the somatotrope.

Question:

The synthesis and secretion of FSH from the adenohypophyseal basophil is regulated by which
hypothalamic hormone?

Answer Choices:

A GnRH

B CRH

C TRH

D Dopamine

E Vasopressin

A GnRH

The synthesis and secretion of ACTH from the adenohypophyseal basophil is regulated
by the hypothalamic releasing hormone, corticotropin releasing hormone (CRH). GnRH,
gonadotropin releasing hormone, acts to stimulate the synthesis and release of FSH
and/or LH from the gonadotrope. TRH, thyrotropin releasing hormone, acts to stimulate
the synthesis and release of TSH (thyroid stimulating hormone) from the thyrotrope.
Dopamine is the putative prolactin inhibiting factor. Vasopressin is synthesized in the
cell bodies lying in the supraoptic and paraventricular nuclei in the hypothalamus, and
secreted upon stimulation, from axons terminating in the pars nervosa.

Question:

A young man presented to the Emergency Department two days following the celebration of his 21st
birthday. He indicated that feeling inebriated, he walked out of the bar and tripped over a curb, falling
and sharply striking his forehead, which showed bruising, but no lacerations. Judging that he was not
critically hurt, he was transported home where he placed an ice pack on the site of the injury. He
indicated he did not loose consciousness, or experience dizziness or nausea, but he did report a mild
headache the day after, which was relieved by aspirin. He did, however, experience what he thought
was extreme thirst and reported extensive urine production. The sharp blow to his head may have
caused disruption of the hormonal secretion from which one of the portions of the pituitary gland?

Answer Choices:

A Pars tuberalis

B Infundibulum

C Pars distalis

D Pars intermedia

E Adenohypophysis

B Infundibulum

It is reasonable that the young man who presented two days following a fall, sharply
striking his head, may be suffering from a condition called diabetes insipidus. This
condition is characterized by the production of large volumes (15-20 liters) of dilute
urine per day. Extreme thirst also is associated with this condition. This condition may
occur when the neurosecretory axons extending through the infundibulum are
damaged or lesioned and vasopressin (ADH) secretion is reduced or absent. While
other portions of the pituitary may have been damaged in the fall, no other hormones in
the areas indicated would have a dramatic effect on urine production.

Question:

The synthesis and secretion of which one of the following anterior pituitary hormones is inhibited by its
hypothalamic regulating factor?

Answer Choices:

A TSH

B MSH

C Prolactin

D LH

E ACTH

C Prolactin

Hypothalamic regulating factors (hormones), which are released into the hypophyseal
portal system, will act upon the cells of the adenohypophysis to selectively stimulate
secretion of their respective pituitary hormones. An exception is the regulation of
prolactin, which is tonically inhibited by prolactin inhibiting factor, presumably
dopamine. All of the other listed hormones from the pituitary gland are regulated by
hypothalamic releasing factors.

The release of TSH secreted by the anterior pituitary is stimulated by TRH (Thyrotropin
releasing hormone) from the hypothalamus. The release of LH (lutenizing hormone) is
stimulated by GnRH (Gonadotropin releasing hormone) from the hypothalamus.
Production of ACTH (Adrenocorticotropic Hormone) is dependent on CRH (Corticotropin
releasing hormone) from the hypothalamus.
From the options,
Prolactin is the only hormone of the anterior pituitary that has an inhibitory control from
the hypothalamus via the PIH (Prolactin inhibitory hormone). MSH (Melanocyte
Stimulating Hormone) does not have a hypothalamic regulating factor.

Question:

The cells as indicated by the arrowhead labeled 'x' in the attached photomicroscopic image of the
pituitary gland will secrete which hormone(s)?

Answer Choices:

A LH

B FSH

C TSH

D Prolactin

E LH & FSH

Image(s) / Chart(s):
Click image to view full size. Click open image to close. Click and hold open image to move.
D Prolactin

The photomicroscopic image depicts a high power view of the anterior pituitary gland
(hypophysis). This portion of the endocrine gland is composed of glandular epithelial
tissue, the cells of which are arranged in random whirls or clumps. H&E staining reveals
three cell types. The acidophils, 'X'; the basophils, 'Y'; and chromophobes (not
indicated).

Using immunohistochemical stains, the acidophils have been show to synthesize and
secrete somatotropin (growth hormone or GH) and mammotropin (prolactin or
lactogenic hormone). Similarly, the basophils represent a population of cells that
secrete thyrotropin (thyroid stimulating hormone - TSH), gonadotropins (follicle
stimulating hormone - FSH and luteinizing hormone - LH), and adrenocorticotropic
hormone (ACTH). Another cell type, not depicted, is the chromophobe - so named
because it does not readily stain with H&E stains. The anterior pituitary gland is
extensively vascularized. A sinusoidal capillary of the secondary portal plexus is
represented by the asterisk in the image. Pituicytes are found within the posterior lobe
of the pituitary gland (neurohypophysis) and represent supportive cells, similar to glial
cells of the central nervous system. Herring bodies, also found within the
neurohypophysis, represent secretory granules that accumulate in the axons of the
neurosecretory cells found in the posterior pituitary gland.

Question:

The structure labeled with the asterisk in the photomicroscopic image best represents which structure
in the pituitary gland?

Answer Choices:

A Capillaries of the primary portal plexus

B Hypophyseal portal veins

C Capillaries of the secondary portal plexus

D Cystic cavities of the intermediate pituitary lobe

E The hypothalamohypophyseal tract

Image(s) / Chart(s):
Click image to view full size. Click open image to close. Click and hold open image to move.

C Capillaries of the secondary portal plexus


The photomicroscopic image depicts a high power view of the anterior pituitary gland
(hypophysis). This portion of the endocrine gland is composed of glandular epithelial
tissue, the cells of which are arranged in random whirls or clumps. H&E staining reveals
three cell types. The acidophils, 'X'; the basophils, 'Y'; and chromophobes (not
indicated).

Using immunohistochemical stains, the acidophils have been show to synthesize and
secrete somatotropin (growth hormone or GH) and mammotropin (prolactin or
lactogenic hormone). Similarly, the basophils represent a population of cells that secrete
thyrotropin (thyroid stimulating hormone - TSH), gonadotropins (follicle stimulating
hormone - FSH and luteinizing hormone - LH), and adrenocorticotropic hormone
(ACTH). Another cell type, not depicted, is the chromophobe - so named because it
does not readily stain with H&E stains. The vascular supply of the adenohypophysis is
intimately related to its function. Superior hypophyseal arteries give rise to a primary
portal plexus of capillaries of the median eminence which bathe the neurosecretory cells
which produce releasing factors from the hypothalamus. These capillaries coalesce into
portal veins called the hypophyseal portal veins that run inferiorly along the pars
tuberalis and infundibular stalk to give rise to another anastomotic plexus, the
secondary portal plexus (asterisk). This plexus will allow releasing factors from the
hypothalamus to access the cells of the anterior pituitary gland and regulate the
synthesis and secretion of the trophic hormones. Similarly, the trophic hormones of the
anterior pituitary are secreted into the secondary portal plexus for transport to their
respective target organs.

Question:

The asterisk in the attached photomicroscopic image best represent which structure in the pituitary
gland?

Answer Choices:

A Basophils

B Acidophils

C Pituicytes
D Chromophobes

E Capillary sinus

Image(s) / Chart(s):
Click image to view full size. Click open image to close. Click and hold open image to move.

E Capillary sinus

The photomicroscopic image depicts a high power view of the anterior pituitary gland
(hypophysis). This portion of the endocrine gland is composed of glandular epithelial
tissue, the cells of which are arranged in random whirls or clumps. H&E staining reveals
three cell types. The acidophils, 'X'; the basophils, 'Y'; and chromophobes (not
indicated).

Using immunohistochemical stains, the acidophils have been show to synthesize and
secrete somatotropin (growth hormone or GH) and mammotropin (prolactin or
lactogenic hormone). Similarly, the basophils represent a population of cells that secrete
thyrotropin (thyroid stimulating hormone - TSH), gonadotropins (follicle stimulating
hormone - FSH and luteinizing hormone - LH), and adrenocorticotropic hormone
(ACTH). Another cell type, not depicted, is the chromophobe - so named because it
does not readily stain with H&E stains. The anterior pituitary gland is extensively
vascularized. A sinusoidal capillary of the secondary portal plexus is represented by
the asterisk in the image. Pituicytes are found within the posterior lobe of the pituitary
gland (neurohypophysis) and represent supportive cells, similar to glial cells of the
central nervous system. Herring bodies, also found within the neurohypophysis,
represent secretory granules that accumulate in the axons of the neurosecretory cells
found in the posterior pituitary gland.

Question:

The cells as indicated by the arrowhead labeled 'y' in the attached photomicroscopic image best
represent which cells in the pituitary gland?

Answer Choices:

A Basophils

B Acidophils

C Pituicytes

D Chromophobes

E Herring bodies

Image(s) / Chart(s):
Click image to view full size. Click open image to close. Click and hold open image to move.
A Basophils

The photomicroscopic image depicts a high power view of the anterior pituitary gland
(hypophysis). This portion of the endocrine gland is composed of glandular epithelial
tissue, the cells of which are arranged in random whirls or clumps. H&E staining reveals
three cell types. The acidophils, 'X'; the basophils, 'Y'; and chromophobes (not
indicated).

Using immunohistochemical stains, the acidophils have been show to synthesize and
secrete somatotropin (growth hormone or GH) and mammotropin (prolactin or
lactogenic hormone). Similarly, the basophils represent a population of cells that secrete
thyrotropin (thyroid stimulating hormone - TSH), gonadotropins (follicle stimulating
hormone - FSH and luteinizing hormone - LH), and adrenocorticotropic hormone
(ACTH). Another cell type, not depicted, is the chromophobe - so named because it
does not readily stain with H&E stains. The anterior pituitary gland is extensively
vascularized. A sinusoidal capillary of the secondary portal plexus is represented by the
asterisk in the image. Pituicytes are found within the posterior lobe of the pituitary gland
(neurohypophysis) and represent supportive cells, similar to glial cells of the central
nervous system. Herring bodies, also found within the neurohypophysis, represent
secretory granules that accumulate in the axons of the neurosecretory cells found in the
posterior pituitary gland.
Question:

The cells as indicated by the arrowhead labeled 'x' in the attached photomicroscopic image best
represent which cells in the pituitary gland?

Answer Choices:

A Basophils

B Acidophils

C Pituicytes

D Chromophobes

E Herring bodies

Image(s) / Chart(s):
Click image to view full size. Click open image to close. Click and hold open image to move.
B Acidophils

The photomicroscopic image depicts a high power view of the anterior pituitary gland
(hypophysis). This portion of the endocrine gland is composed of glandular epithelial
tissue, the cells of which are arranged in random whirls or clumps. H&E staining reveals
three cell types. The acidophils, 'X'; the basophils, 'Y'; and chromophobes (not
indicated).

Using immunohistochemical stains, the acidophils have been show to synthesize and
secrete somatotropin (growth hormone or GH) and mammotropin (prolactin or
lactogenic hormone). Similarly, the basophils represent a population of cells that secrete
thyrotropin (thyroid stimulating hormone - TSH), gonadotropins (follicle stimulating
hormone - FSH and luteinizing hormone - LH), and adrenocorticotropic hormone
(ACTH). Another cell type, not depicted, is the chromophobe - so named because it
does not readily stain with H&E stains. The anterior pituitary gland is extensively
vascularized. A sinusoidal capillary of the secondary portal plexus is represented by the
asterisk in the image. Pituicytes are found within the posterior lobe of the pituitary gland
(neurohypophysis) and represent supportive cells, similar to glial cells of the central
nervous system. Herring bodies, also found within the neurohypophysis, represent
secretory granules that accumulate in the axons of the neurosecretory cells found in the
posterior pituitary gland.

Question:
Prolactin is synthesized and secreted by which of the following cells of the adenohypophysis?

Answer Choices:

A Acidophils in the pars distalis

B Basophils in the pars tuberalis

C Somatotrophs in the pars distalis

D Basophils in the pars intermedia

E Chromophobic of pars distalis

A Acidophils in the pars distalis

Prolactin is produced by mammotrophs, one of the two types of acidophils located in the
pars distalis of the pituitary gland. As their name implies, these cells produce a hormone
that regulates the development of the mammary gland during pregnancy and lactation.

Basophils produce and secrete gonadotropic (FSH and LH), thyrotropic (TSH), and
corticotropic (ACTH) hormones. Somatotrophs produce somatostatin (GHRIF), while
chromophobes are not known to secrete any hormone.

Question:
+
Which one of the following pituitary cell types is PAS ?
Answer Choices:

A Somatotrophic acidophils

B Mammotrophic acidophils

C Chromophobes

D Basophils

D Basophils

The granules of all basophils give a strong positive reaction to PAS (periodic acid-
Schiff) reagent since these cells synthesize and release peptide hormones that contain
bound carbohydrates (glycoproteins). Acidophils stain deeply with acidic dyes (e.g.,
eosin), while chromophobes have no affinity for either basic or acidic dyes.

Question:

Hypothalamus inhibits the release of both growth hormone and thyroid-stimulating hormone from the
pituitary through the release of

Answer Choices:

A Vasopressin

B TRH (thyrotropin-releasing hormone)

C GnRH (Gonadotropin-releasing hormone


D Dopamine

E Somatostatin

E Somatostatin

Somatostatin, secreted by the hypothalamus, inhibits the release of both growth


hormone (GH) and thyroid-stimulating hormone (TSH). Vasopressin is secreted and
released by the pars nervosa (to increase water permeability of renal collecting tubules
and to promote vasoconstriction). TRH (thyrotropin-releasing hormone), GnRH
(Gonadotropin-releasing hormone, and dopamine are released by hypothalamus. TRH
stimulates release of thyroid-stimulating hormone and prolactin, and GnRH stimulates
the release of both follicle-stimulating hormone (FSH) and luteinizing hormone (LH).
Dopamine can also act as a neurotransmitter.

Question:

The cells as indicated by the arrowhead labeled 'x' in the attached photomicroscopic image of the
pituitary gland will secrete which hormone(s)?

Answer Choices:

A LH

B FSH

C GH

D TSH

E Vasopressin
Image(s) / Chart(s):
Click image to view full size. Click open image to close. Click and hold open image to move.

C GH

The photomicroscopic image depicts a high power view of the anterior pituitary gland
(hypophysis). This portion of the endocrine gland is composed of glandular epithelial
tissue, the cells of which are arranged in random whirls or clumps. H&E staining reveals
three cell types. The acidophils, 'X'; the basophils, 'Y'; and chromophobes (not
indicated).

Using immunohistochemical stains, the acidophils have been show to synthesize and
secrete somatotropin (growth hormone or GH) and mammotropin (prolactin or
lactogenic hormone). Similarly, the basophils represent a population of cells that secrete
thyrotropin (thyroid stimulating hormone - TSH), gonadotropins (follicle stimulating
hormone - FSH and luteinizing hormone - LH), and adrenocorticotropic hormone
(ACTH). Another cell type, not depicted, is the chromophobe - so named because it
does not readily stain with H&E stains. The anterior pituitary gland is extensively
vascularized. A sinusoidal capillary of the secondary portal plexus is represented by the
asterisk in the image. Vasopressin (antidiuretic hormone - ADH) is a neurohormone
secreted by the neurohypophysis and acts to increase the permeability of the distal
convoluted tubule in the kidney.
Question:

The synthesis and secretion of ACTH from the adenohypophyseal basophil, is regulated by which
hypothalamic hormone?

Answer Choices:

A GnRH

B CRH

C TRH

D Dopamine

E Vasopressin

B CRH

The synthesis and secretion of ACTH from the adenohypophyseal basophil is regulated
by the hypothalamic releasing hormone, corticotropin releasing hormone (CRH).
GnRH, gonadotropin releasing hormone, acts to stimulate the synthesis and release of
FSH and/or LH from the gonadotrope. TRH, thyrotropin releasing hormone, acts to
stimulate the synthesis and release of TSH (thyroid stimulating hormone) from the
thyrotrope. Dopamine is the putative prolactin inhibiting factor. Vasopressin is
synthesized in the cell bodies lying in the supraoptic and paraventricular nuclei in the
hypothalamus, and secreted upon stimulation, from axons terminating in the pars
nervosa.

Question:

Which of the following cells is specific only to the pars nervosa?


Answer Choices:

A Chromophobe

B Acidophil

C Pituicyte

D Basophil

E Herring body

C Pituicyte

The cell specific only to the pars nervosa of the neurohypophysis is the pituicyte, an
irregular shaped cell, usually associated with the capillaries. These cells resemble glial
cells and may function similar to the astrocyte since they have processes that terminate
in perivascular spaces. The acidophils, basophils and chromophobes are secretory cells
associated with the adenohypophysis. Herring bodies, also found within the
neurohypophysis, represent secretory granules that accumulate in the axons of the
neurosecretory cells found in the posterior pituitary gland. However, Herring bodies are
NOT individual cells.

Question:

This attached photomicroscopic image depicts the appearance of an endocrine organ. What endocrine
organ does this image best represent?

Answer Choices:

A Anterior pituitary gland


B Thyroid gland

C Posterior pituitary gland

D Pineal gland

E Zona glomerulosa of the adrenal cortex

Image(s) / Chart(s):
Click image to view full size. Click open image to close. Click and hold open image to move.

A Anterior pituitary gland

The photomicroscopic image depicts a high power view of the anterior pituitary gland
(hypophysis). This portion of the endocrine gland is composed of glandular epithelial
tissue, the cells of which are arranged in random whirls or clumps. H&E staining reveals
three cell types. The acidophils, 'X'; the basophils, 'Y'; and chromophobes (not
indicated).

Using immunohistochemical stains, the acidophils have been show to synthesize and
secrete somatotropin (growth hormone or GH) and mammotropin (prolactin or
lactogenic hormone). Similarly, the basophils represent a population of cells that secrete
thyrotropin (thyroid stimulating hormone - TSH), gonadotropins (follicle stimulating
hormone - FSH and luteinizing hormone - LH), and adrenocorticotropic hormone
(ACTH). Another cell type, not depicted, is the chromophobe - so named because it
does not readily stain with H&E stains. The anterior pituitary gland is extensively
vascularized. A sinusoidal capillary of the secondary portal plexus is represented by the
asterisk in the image.

Question:

The pars nervosa is derived from which one of the following embryological structures?

Answer Choices:

A Roof of the diencephalon

B Infundibular stalk

C Posterior portion of Rathke's pouch

D Rostral portion of Rathke's pouch

E Anterior portion of Rathke's pouch

B Infundibular stalk

The pars nervosa is derived from an evagination of the floor of the diencephalon, the
infundibular stalk. During embryonic development of the pituitary gland, an
invagination of the oral ectoderm, Rathke's pouch, migrates rostral towards the
infundibular stalk. Rathke's pouch looses its connection with the roof of the mouth cavity
and invests itself around the finger-like infundibular projection. Most of the anterior
portion of Rathke's pouch forms the pars distalis of the adenohypophysis, the largest
portion of the anterior pituitary gland. A slender portion of the most rostral part of
Rathke's pouch covers over the infundibular stalk to form the pars tuberalis. The
posterior portion of Rathke's pouch, which contacts the base of the infundibulum,
becomes the pars intermedia.

PINEAL GLAND:
Question:

A 27-year old female presents with a complaint of being "depressed and down" for the past few weeks,
to the point that it is difficult for her to get out of bed in the mornings or go to work. She indicates she
has lost interest in her sewing hobby, which she used to get great pleasure from. She also reports
having lost about 8 pounds in the past month. Review of her systems is non-contributory. Further
evaluation reveals that this depression occurs at about the same time each year, in late October and
persists until sometime in January. You suspect she has "seasonal affective disorder" and prescribe
light treatment to extend the day length. The basis of this treatment is to:

Answer Choices:

A Depress melatonin levels from the pineal gland

B Depress thyroxine levels from the thyroid gland

C Enhance melatonin levels from the pineal gland

D Enhance melanin levels in the skin

E Reduce MSH levels in the pituitary gland

A Depress melatonin levels from the pineal gland

This patient may be suffering from seasonal affective disorder, which is expressed as
depression with the onset of short day lengths. The pineal gland receives neural input
from the eyes, via sympathetic nervous innervation. In doing so, melatonin synthesis
and secretion is reduced when the individual perceives light.

Melatonin secretion occurs during darkness, and with the advancement of the longer
dark periods with the onset of shorter day lengths, melatonin secretion may occur
earlier, somewhat out of phase to the rest of the body rhythms. In doing so, some
individuals may suffer from depression. Light treatment serves to inhibit melatonin
secretion from the pineal gland and depress systemic levels. Light treatment may also
enhance melanin levels in the skin, but this does not serve as the biomedical
explanation for reduction of depression in individuals suffering from seasonal affective
disorder.

Question:

The structures appearing black as depicted in the attached digitized photomicroscopic image are best
identified as

Answer Choices:

A Corpora arenacea of the pineal gland

B Corpora amylacea of the pineal gland

C Corpora amylacea of the prostate gland

D Corpora arenacea of the prostate gland

E Concretions within the intermediate lobe of the pituitary gland

Image(s) / Chart(s):
Click image to view full size. Click open image to close. Click and hold open image to move.
A Corpora arenacea of the pineal gland

The attached photomicroscopic image depicts the appearance of the pineal gland. The
pineal gland is a neuroendocrine organ whose primary secretory product is melatonin, a
tryptophan derivative. The endocrine gland consists of several cell types within the
stroma, the pinealocytes, the glial cells, fibroblasts, and endothelial cells associated with
numerous capillaries. The pineal gland regularly contains calcareous deposits called
corpora arenacea or brain sand. These are not contained within any tubules or alveoli,
as with the prostate gland concretions.

Melatonin has been shown to have an anti-gonadal effect, be associated with jet lag,
seasonal affective depression (SAD), and be a scavenger of free radicals. Calcareous
deposits are not normally observed in any part of the pituitary gland, although
occasionally concretions may be observed within follicles sometimes seen associated
with the intermediate lobe of the pituitary gland.

Question:

The organ as depicted in the attached digitized photomicroscopic image is best identified as the

Answer Choices:

A Pineal gland

B Anterior lobe of the pituitary gland

C Prostate gland

D Posterior lobe of the pituitary gland

E Intermediate lobe of the pituitary gland


Image(s) / Chart(s):
Click image to view full size. Click open image to close. Click and hold open image to move.

A Pineal gland

The attached photomicroscopic image depicts the appearance of the pineal gland. The
pineal gland is a neuroendocrine organ whose primary secretory product is melatonin, a
tryptophan derivative. The endocrine gland consists of several cell types within the
stroma, the pinealocytes, the glial cells, fibroblasts, and endothelial cells associated with
numerous capillaries. The pineal gland regularly contains calcareous deposits called
corpora arenacea or brain sand. These are not contained within any tubules or alveoli,
as with the prostate gland concretions.

Melatonin has been shown to have an anti-gonadal effect, be associated with jet lag,
seasonal affective depression (SAD), and be a scavenger of free radicals. Calcareous
deposits are not normally observed in any part of the pituitary gland. The anterior lobe
of the pituitary gland possesses an array of cells that stain differentially and can be
identified by their staining characteristics as acidophils, basophils and chromophobes.

Question:

Characteristics of pinealocytes include which of the following?

Answer Choices:

A They produce melatonin and serotonin


B They are astrocyte-like cells

C They contain calcified concretions of unknown function

D The are postganglionic sympathetic somata

E They comprise about 5% of the cells in the gland

A They produce melatonin and serotonin

Pinealocytes, about 95% of the total cell population of the pineal gland, produce
melatonin during the night and serotonin during the day. The pineal gland also has
interstitial (glial) cells, about 5% of the cells in the gland, that resemble astrocytes and
have calcified concretions (brain sand, Corpora arenacea) in their interstitium.

Question:

The primary secretory product of the organ as depicted in the attached digitized photomicroscopic
image is described as

Answer Choices:

A Acid phosphatase and citrate

B Melatonin

C Melanin

D Melanocyte stimulating hormone (MSH)

E Oxytocin
Image(s) / Chart(s):
Click image to view full size. Click open image to close. Click and hold open image to move.

B Melatonin

The attached photomicroscopic image depicts the appearance of the pineal gland. The
pineal gland is a neuroendocrine organ whose primary secretory product is melatonin, a
tryptophan derivative. The endocrine gland consists of several cell types within the
stroma, the pinealocytes, the glial cells, fibroblasts, and endothelial cells associated with
numerous capillaries. The pineal gland regularly contains calcareous deposits called
corpora arenacea or brain sand. These are not contained within any tubules or alveoli,
as with the prostate gland concretions.

Melatonin has been shown to have an anti-gonadal effect, be associated with jet lag,
seasonal affective depression (SAD), and be a scavenger of free radicals. Calcareous
deposits are not normally observed in any part of the pituitary gland. The anterior lobe
of the pituitary gland possesses an array of cells that stain differentially and can be
identified by their staining characteristics as acidophils, basophils and chromophobes.
These cells secrete gonadotrophins, which act upon target organs to regulate their
activity. MSH is one such hormone, which is secreted by the cells of the intermediate
lobe of the pituitary gland.

Oxytocin is released by the neurons terminating in the neurohypophysis, or posterior


lobe of the pituitary gland. Oxytocin is synthesized in the hypothalamus, transferred and
stored within the axons located in the posterior lobe. Melanin is secreted by
melanocytes within the basal layers of the epidermis to cause darkening of the skin.

Question:

A tumor that would interfere with pinealocyte function in the pineal gland of a 9-year old boy may be
expressed as

Answer Choices:

A Delayed puberty

B Type I diabetes

C Cryptorchism

D Prostatic atrophy

E Precocious puberty

E Precocious puberty

Tumors that destroy the pineal gland, and hence, interfere with its antigonadotropic
function are associated with individuals showing precocious puberty. Melatonin, or other
pineal hormones, serves to inhibit reproductive maturation. Destruction of the
pinealocytes may result in the clinical advancement of the onset of puberty.
Alternatively, enhanced secretion of pineal hormones may result in the clinical delay in
the onset of puberty. Since the prostate is influenced by the presence of testosterone,
early onset of puberty would result in the advancement of the size of the prostate, by
age comparison. Cryptorchism is the term for the failure for one or both testes to
descend into the scrotum during development.

THYROID GLAND:
Question:

Many different minerals are required in trace quantities in the diet. These minerals are incorporated into
many different types of molecules that serve a variety of cellular functions. You have isolated a cell line
in which the level of iodine found in thyroglobulin is greatly reduced compared to the control cells. If
such abnormal cells existed in a patient, this patient would be expected to
Answer Choices:

A Have normal levels of thyroxin

B Have hypo-iodinated triiodothyronine and thyroxin

C Completely lack triiodothyronine and thyroxin

D Have increased amounts of thyroglobulin

E Suffer from thyroglobulin toxicity

B Have hypo-iodinated triiodothyronine and thyroxin

Dietary iodine is absorbed and transported to the thyroid gland. It is stored there and
used for the synthesis of the thyroid hormones, triiodothyronine (T3) and thyroxin (T4).
Both of these hormones are derived from the large protein thyroglobulin. Several of the
tyrosines in thyroglobulin are iodinated at 1 or 2 sites. This is followed by the reaction of
2 iodinated species to form T3 or T4. T3 and T4 are released following protein hydrolysis.
These hormones play a role in regulating the basal metabolic rate in adults and growth
and development in children. Decreased amounts of iodine in the cell would result in a
decreased incorporation of iodine into thyroglobulin resulting in hypo-iodinated
triiodothyronine and thyroxin. The thyroid hormones bind to cell surface receptors
and the hormone-receptor complex is translocated to the nucleus where it binds to
specific DNA sequences called hormone responsive elements. This activates the
transcription of genes involved in energy related processes.

Question:

Many different minerals are required in trace quantities in the diet. These minerals are found in many
different types of molecules that serve a variety of functions. Which of the following minerals are
covalently incorporated into thyroxin?
Answer Choices:

A calcium

B iodine

C cobalt

D Iron

E zinc

B iodine

Dietary iodine is absorbed and transported to the thyroid gland. It is stored there and
used for the synthesis of the thyroid hormones, triiodothyronine (T3) and thyroxin (T4).
Both of these hormones are derived from the large protein thyroglobulin. Several of the
tyrosines in thyroglobulin are iodinated at 1 or 2 sites. This is followed by the reaction of
2 iodinated species to form T3 or T4. T3 and T4 are released following protein hydrolysis.
These hormones play a role in regulating the basal metabolic rate in adults and growth
and development in children. The thyroid hormones bind to cell surface receptors and
the hormone-receptor complex is translocated to the nucleus where it binds to specific
DNA sequences called hormone responsive elements. This activates the transcription of
genes involved in energy related processes.

Question:

Stimulation of the secretion of calcitonin is regulated by:

Answer Choices:
A TSH .

B High blood calcium levels.

C Low calcium levels.

D Parathormone.

E CRH.

B High blood calcium levels.

Calcitonin is regulated by the presence of high blood calcium levels and when
secreted, it acts to lower blood calcium levels predominantly by increasing the rate of
osteoid calcification and suppressing bone resorption. Secretion of calcitonin is
unaffected by the pituitary gland, so TSH (thyroid stimulating hormone) would have no
effect on calcitonin. CRH is corticotrophin releasing hormone from the hypothalamus,
which does not affect calcitonin secretion. Low calcium levels stimulate the release of
parathormone.

Question:

One known mechanism by which calcitonin acts to lower blood calcium levels is to:

Answer Choices:

A Increase the rate of osteoid calcification.

B Increase the activity of osteoclasts.


C Decrease the activity of osteocytes.

D Decrease calcium excretion by the kidney.

E Increase calcium absorption from the diet.

A Increase the rate of osteoid calcification.

Calcitonin is secreted in response to high blood calcium levels and will lower blood
calcium levels predominantly by increasing the rate of osteoid calcification and
suppressing bone resorption.

Question:

The hormone thyroxine is formed by combining

Answer Choices:

A Two iodinated tyrosine residues together, each having two iodinated


sites

B Four iodinated tyrosine residues together, each having one iodinated site

C Four iodinated sites on one tyrosine molecule

D Two iodinated tyrosine residues together, one having two iodinated sites and
the other having one iodinated site
E Two iodinated tyrosine residues together, each having an iodinated site

A Two iodinated tyrosine residues together, each having two iodinated sites

Thyroxine is formed by the thyroid gland by coupling together two iodinated tyrosine
residues, each residue having two iodinated sites. Each of the iodinated tyrosine
residues are referred to as diiodothyronine (DIT) and when combined will yield
thyroxine, or T4. T3 is formed by the coupling of two iodinated tyrosine residues, one
having two iodinated sites (diiodothyronine or DIT) and the other having one iodinated
site (monoiodothyronine or MIT).

Question:

When viewed with an electron microscope, the follicular cells of the thyroid gland may contain relatively
large membrane-limited vesicles in the apical (luminal) region, particularly when the gland is stimulated
by TSH (thyroid stimulating hormone). These membrane-bound vesicles are identified as

Answer Choices:

A Microvesicles containing thyroxine

B Microvesicles containing calcitonin

C Colloid resorption droplets

D Lysosomes

E Neurosecretory granules

C Colloid resorption droplets


Thyroglobulin is taken up by receptor-mediated endocytosis from the colloid by the
follicular cells of the thyroid gland. The thyroglobulin is contained within membrane-
bound vesicles, called colloidal resorption droplets, which can be visualized with the
electron microscope. These vesicles are larger than the secretory vesicles filled with the
thyroid hormones. Neurosecretory granules are present in neurons associated with
secretion of peptide hormones, such as found in the posterior lobe of the pituitary gland.

Question:

The gel-like material as seen contained in the follicles of the attached photomicroscopic image is called

Answer Choices:

A Corpora amylacea

B Thyroglobulin

C Semen

D Milk

E Zymogen granules

Image(s) / Chart(s):
Click image to view full size. Click open image to close. Click and hold open image to move.
B Thyroglobulin

The photomicroscopic image depicts the view of the thyroid gland. The thyroid gland, as
one of the endocrine glands, consists of a gel-like mass of colloid which fills the follicles,
each surrounded by a wall of simple squamous or cuboidal cells, the follicular cells. The
follicular cells secrete thyroxine (T4) and T3 (triiodothyronine). These hormones regulate
cellular metabolism. One other cell type present in the follicular wall is the parafollicular
cells (C-cells), which secrete calcitonin, a hormone that lowers blood calcium levels.

The colloid filled follicles are somewhat spherical in shape and separated from one
another by connective tissue where a rich vascular innervation may be found. The
colloidal material is a large iodinated glycoprotein called thyroglobulin, synthesized by
the follicular epithelial cells. Corpora amylacea are also known as prostatic concretions.

Semen may be found in the seminal vesicles, but is not found in follicles. The thyroid
may resemble the secretory acini of either the mammary gland (which produces milk),
or the pancreas, where zymogen granules may be found in the pancreatic acinar cells.

Question:

A hormone secreted by the organ as depicted in the attached photomicroscopic image is best identified
as

Answer Choices:
A Renin

B Thyroxine

C Parathormone

D Melanocyte stimulating hormone (MSH)

E An odiferous proteinaceous secretion

Image(s) / Chart(s):
Click image to view full size. Click open image to close. Click and hold open image to move.

B Thyroxine

The photomicroscopic image depicts the appearance of the thyroid gland. The thyroid
gland, as one of the endocrine glands, consists of a gel-like mass of colloid which fills
the follicles, each surrounded by a wall of simple squamous or cuboidal cells, the
follicular cells. The follicular cells secrete thyroxine (T4) and T3 (triiodothyronine). These
hormones regulate cellular metabolism. One other cell type present in the follicular wall
is the parafollicular cells (C-cells), which secrete calcitonin, a hormone that lowers blood
calcium levels. The colloid filled follicles are somewhat spherical in shape and
separated from one another by connective tissue where a rich vascular innervation may
be found.
On occasion, limited numbers of similarly appearing follicles or cysts may be observed
in the intermediate lobe of the pituitary gland. These differ, however, in that various cells
associated with the anterior lobe of the pituitary may be found in approximation to the
cysts located in the intermediate lobe. Melanocyte stimulating hormone is believed to be
one hormone secreted from this part of the pituitary gland.

Likewise, there may be confusion of the thyroid with the appearance of apocrine glands,
the modified sweat glands commonly associated with the axillary and groin regions.
Apocrine glands, however, do not commonly appear filled with the colloid substance,
and are more limited in their numbers in the connective tissue. Apocrine glands produce
an odiferous proteinaceous secretion. Parathormone is a product of the Principal cells
of the parathyroid glands and renin is a product of the cells of the JG apparatus in the
kidney glomerulus.

Question:

What type of capillary would be found in the thyroid gland?

Answer Choices:

A Continuous

B Fenestrated

C Discontinuous

D Sinusoidal

B Fenestrated

Based upon their ultrastructural characteristics, capillary endothelia may be described


as being continuous, fenestrated or discontinuous (sinusoidal). Fenestrated capillaries
possess small "windows" through the capillary wall of about 80-100 nm in diameter. The
fenestration has a central diaphragm or central thickening consistent with glycocalyx
which may have been from a pinocytotic vesicle originally associated with the formation
of the fenestra. Fenestrated capillaries are typically located in endocrine glands, but
may also be found in areas where active absorption occurs, such as the capillaries in
the intestinal tract.

Question:

A hormone secreted by the organ as depicted in the attached photomicroscopic image is best identified
as

Answer Choices:

A Renin

B T3 (triiodothyronine)

C Parathormone

D Melanocyte stimulating hormone (MSH)

E An odiferous proteinaceous secretion

Image(s) / Chart(s):
Click image to view full size. Click open image to close. Click and hold open image to move.
B T3 (triiodothyronine)

The photomicroscopic image depicts the appearance of the thyroid gland. The thyroid
gland, as one of the endocrine glands, consists of a gel-like mass of colloid which fills
the follicles, each surrounded by a wall of simple squamous or cuboidal cells, the
follicular cells. The follicular cells secrete thyroxine (T4) and T3 (triiodothyronine).
These hormones regulate cellular metabolism. One other cell type present in the
follicular wall is the parafollicular cells (C-cells), which secrete calcitonin, a hormone that
lowers blood calcium levels. The colloid filled follicles are somewhat spherical in shape
and separated from one another by connective tissue where a rich vascular innervation
may be found.

On occasion, limited numbers of similarly appearing follicles or cysts may be observed


in the intermediate lobe of the pituitary gland. These differ, however, in that various cells
associated with the anterior lobe of the pituitary may be found in approximation to the
cysts located in the intermediate lobe. Melanocyte stimulating hormone is believed to be
one hormone secreted from this part of the pituitary gland.

Likewise, there may be confusion of the thyroid with the appearance of apocrine glands,
the modified sweat glands commonly associated with the axillary and groin regions.
Apocrine glands, however, do not commonly appear filled with the colloid substance,
and are more limited in their numbers in the connective tissue. Apocrine glands produce
an odiferous proteinaceous secretion. Parathormone is a product of the Principal cells
of the parathyroid glands and renin is a product of the cells of the JG apparatus in the
kidney glomerulus.

Question:

Which one of the following hormones is secreted by the parafollicular or C cells?


Answer Choices:

A PTH

B TSH

C Calcium

D Calcitonin

E Thyroxine

D Calcitonin

Calcitonin is secreted by the parafollicular or C cells of the thyroid gland. High blood
levels of calcium stimulate the synthesis and secretion of calcitonin. PTH or
parathormone is a hormone from the Principal cells of the parathyroid glands, which will
increase blood calcium levels. TSH is thyroid stimulating hormone, which is secreted by
the pituitary gland to regulate synthesis and secretion of thyroxine and T 3 from the
thyroid gland. Thyroxine is secreted from the follicular cells of the thyroid gland.

Question:

The parafollicular cells associated with the thyroid gland arise from what embryological source?

Answer Choices:

A Endoderm
B Mesoderm

C Ectoderm

D Neural ectoderm

E Neural crest cells

E Neural crest cells

The thyroid follicular cells arise embryologically from endoderm that migrates downward
from the floor of the foregut, in the midline area near the base of the developing tongue.
However, the parafollicular cells are derived from the neural crest cells that migrate
into the thyroid gland during development.

Question:

The principal hormones secreted by the thyroid gland are thyroxine (T 4) and triiodothyronine (T3). Both
hormones are derivatives of the amino acid tyrosine. Iodine is a raw material essential for thyroid
hormone synthesis, as synthesis of thyroid hormones requires the iodination of the amino acid tyrosine.
How much iodide is required per day to maintain an euthyroid status?

Answer Choices:

A 500 microgram

B 50 microgram

C 150 microgram

D 2 microgram
E 5 microgram

C 150 microgram

About 150 microgram iodide per day is required to maintain an euthyroid status.
The normal plasma iodide level is about 0.3 microgram/dL, and iodide is distributed in a
"space" of approximately 25L (35% of body weight). About 120 microgram/day enter the
thyroid at normal rates of thyroid hormone synthesis and secretion. The thyroid secretes
80µg/day as iodine in T3 and T4. 40 micrograms of iodide per day diffuses into ECF.

The secreted T3 and T4 are metabolized in the liver and other tissues with the release of
60µg iodide per day into the ECF. Some thyroid hormone derivatives are excreted in the
bile and some of the iodine in them is reabsorbed (enterohepatic circulation), but there
is a net loss of iodide in the stool of approximately 20µg/day.

If the ingested amount of iodide is significantly less for a long time, then
hypothyroidism will develop. Generally, a sufficient amount of iodide is present in the
normal diet, however, in some areas of the world hypothyroidism due to insufficient
iodide intake does occur. The iodide content of the soil is low in these areas, resulting in
decreased amount of iodide in their food. To counter the occurrence of endemic goiter
due to the presence of insufficient amounts of iodine in the diet, common salt is
supplied, fortified with an iodine supplement, and is referred to as iodized salt.

If significantly larger doses (2 mg) are ingested per day, then the concentration of
organic and inorganic iodide in the thyroid gland will reach a level where the
biosynthetic pathways are inhibited and the hormone synthesis declines. This
autoregulatory phenomenon is called Wolff-Chaikoff effect.

PARATHYROID GLAND:
Case #200064:

A 45-year-old woman is bending to pick up some groceries while grocery shopping


and she falls. X-ray confirms a fracture in her leg. The radiologist comments that there
is generalized osteopenia. Her primary doctor is concerned about the radiologist's
comments, so he questions her further. She asserts that neither parent had any
skeletal problems. Her father died of a heart attack, and her mother is healthy. She
acknowledges vague gastrointestinal complaints. She complains of anorexia, nausea,
and abdominal discomfort. She also has complains of weakness. Her laboratory work
is as follows:
Test Results Normal
BUN 15 mg/dL 10-20 mg/dL
Potassium 4.2 mEq/L 3.5-5.0 mEq/L
Sodium 140 mEq/L 135-145 mEq/L
Glucose (fasting) 91 mg/dL 65-110 mg/dL
Calcium 12.5 mg/dL 8.4-10.2 mg/dL
Alkaline Phosphatase 93 U/L 20-70 U/L
PTH radioimmunoassay elevated
Question:

What is the disorder of bone metabolism that she is experiencing called?

Answer Choices:

A Osteitis fibrosa cystica

B Osteomalacia

C Osteitis deformans

D Osteoporosis

E Osteoarthritis

A Osteitis fibrosa cystica

Osteitis fibrosa cystica is a disorder of bone metabolism seen with excess


parathyroid hormone.

This patient has signs and symptoms of primary hyperparathyroidism. The most
common cause of primary hyperparathyroidism is a benign parathyroid adenoma.

A key feature is the presence of hypercalcemia. The cause of this patient's


hypercalcemia is primary hyperparathyroidism. Primary hyperparathyroidism is a
common cause of hypercalcemia. Symptoms of hypercalcemia include anorexia,
nausea, abdominal pain, and weakness.

Osteomalacia is softening of the bones due to poor mineralization.


Osteitis deformans is Paget's disease.

Osteoporosis is a disease in which the bone density is reduced.

Osteoarthritis is a degenerative bone disease. There would be normal laboratory


results.

Question:

The superior parathyroid glands are derived embryologically from what source?

Answer Choices:

A Second branchial pouch

B Second branchial arch

C Third branchial arch

D Third branchial pouch

E Fourth branchial pouch

E Fourth branchial pouch

The parathyroid glands are derived from the endoderm of the branchial pouches. The
inferior parathyroid glands, which come from the third branchial pouch, migrate inferior
to the superior parathyroid glands, which are derived from the fourth branchial pouch.

Question:

The inferior parathyroid glands are derived embryologically from what source?
Answer Choices:

A Second branchial pouch

B Second branchial arch

C Third branchial arch

D Third branchial pouch

E Fourth branchial pouch

D Third branchial pouch

The parathyroid glands are derived from the endoderm of the branchial pouches. The
inferior parathyroid glands, which come from the third branchial pouch, migrate
inferior to the superior parathyroid glands, which are derived from the fourth branchial
pouch.

Question:

Parathormone (PTH) responds to low blood calcium levels to increase the levels by which one of the
following actions?

Answer Choices:

A Stimulation of tubular resorption of calcium in the kidney


B Inhibition of osteoclastic activity in the bone marrow

C Stimulation of osteoblastic activity in the bone marrow

D Inhibition of glomerular filtration of calcium in the kidney

E Inhibition of Vitamin D secretion

A Stimulation of tubular resorption of calcium in the kidney

PTH functions at several sites to increase levels of blood calcium. A primary site is its
direct effect on osteoclasts in the bone marrow to induce their activity and induce
osteolysis. This action releases both calcium and phosphate from bone matrix into
extracellular fluid, which then raises blood calcium levels as well as blood phosphate
levels. A second action is upon the intestinal enterocytes, which will increase the
absorption of dietary calcium to raise blood calcium levels. A third action is upon the
kidney to reduce excretion of calcium by stimulating tubular resorption of filtered
calcium. PTH, however, will increase phosphate excretion by this same action on the
renal tubules.

Question:

Parathormone (PTH) responds to low blood calcium levels to increase the levels by which one of the
following actions?

Answer Choices:

A Stimulation of enterocyte absorption of dietary calcium

B Inhibition of osteoclastic activity in the bone marrow

C Stimulation of osteoblastic activity in the bone marrow


D Inhibition of enterocyte release of calcium from the gastrointestinal epithelium

E Inhibition of glomerular filtration of calcium in the kidney

A Stimulation of enterocyte absorption of dietary calcium

PTH functions at several sites to increase levels of blood calcium. A primary site is its
direct effect on osteoclasts in the bone marrow to induce their activity and induce
osteolysis. This action releases both calcium and phosphate from bone matrix into
extracellular fluid, which then raises blood calcium levels as well as blood phosphate
levels. A second action is upon the intestinal enterocytes, which will increase the
absorption of dietary calcium to raise blood calcium levels. A third action is upon the
kidney to reduce excretion of calcium by stimulating tubular resorption of filtered
calcium. PTH, however, will increase phosphate excretion by this same action on the
renal tubules.

Question:

Stimulation of parathormone (PTH) secretion occurs in response to what situation

Answer Choices:

A High blood calcium levels

B High blood calcitonin levels

C Low blood calcitonin levels

D Low blood calcium levels

E Secretion of parathormone stimulating hormone from the anterior pituitary


gland

D Low blood calcium levels


The parathyroid glands are composed of two primary types of cells, the Chief cells
(Principal cells), and the oxyphils. The Chief cells synthesize and secrete parathyroid
hormone (or parathormone), which acts to increase blood calcium levels in response
to low blood calcium levels. The oxyphils constitute a smaller number of parenchymal
cells of the parathyroid glands and are not known to have a secretory role.

These cells are usually found intermixed within the parenchymal tissue and appear
strongly acidophilic. They also may be found in clusters. Their numbers increase with
age of the individual. Parafollicular cells are cells found in the thyroid gland which
secrete calcitonin, the hormone that reduces blood calcium levels. The follicular cells,
also found in the thyroid gland, synthesize and secrete thyroid hormones. C-cells are
another name for the parafollicular cells.

Question:

Which one of the following conditions is caused by hypoparathyroidism?

Answer Choices:

A Graves' disease

B Addison's disease

C Diabetes insipidus

D Tetany

E Osteitis fibrosa cystica

D Tetany
Hypoparathyroidism causes an increase in the concentration of PO4- and a decrease in
the concentration of Ca+ in the blood. The bones become denser and more mineralized.
This condition causes spastic contractions of the skeletal muscles and generalized
convulsions called tetany.

Graves' disease (characterized by a diffuse enlargement of the thyroid gland and


protrusion of eyeballs) is associated with hyperthyroidism (or thyrotoxicosis).

Addison's disease is caused by inadequate amounts of adrenocortical hormones.

Diabetes insipidus results from inadequate amounts of vasopressin (antidiuretic


hormone).

Osteitis fibrosa cystica (characterized by an increased number of osteoclasts and


multiple bone cysts) is caused by hyperparathyroidism.

Question:

Which one of the following statements concerning the histological characteristics of the parathyroid
glands is true?

Answer Choices:

A They have a parenchyma composed of three types of cells: chromaffin cells,


epinephrine-producing cells, and norepinephrine producing cells

B The parathyroid hormone secreting cells are large, eosinophilic cells that are
present singly, or in clusters, within the parenchyma of the gland

C Low level blood calcium levels inhibit the secretion of parathyroid hormone

D The small, basophilic chief cells synthesize and secrete parathyroid


hormone (PTH)

E With increasing age, chief cells are replaced by chromaffin cells of an unknown
function

D The small, basophilic chief cells synthesize and secrete parathyroid hormone (PTH)
Image(s) / Chart(s):
Click image to view full size. Click open image to close. Click and hold open image to
move.

The small, polygonal, and pale-staining, slightly acidophilic chief cells synthesize and
release the parathyroid hormone (PTH). The parathyroid glands have a parenchyma
composed of two types of cells: (i) the small, polygonal, and pale-staining, slightly
acidophilic chief cells, and (ii) the large, eosinophilic oxyphil cells of an unknown
function. High blood calcium levels inhibit the secretion of parathyroid hormone. With
increasing age, chief cells are replaced by adipocytes. Adipose cells can constitute
more than 50% of the gland in older individuals.

See the attached Figure: Histology of parathyroid glands. (a) Chief cells; (b) Oxyphil
cells.

Question:

Which one of the following statements concerning the parathyroid gland is FALSE?
Answer Choices:

A They are small glands, situated behind the thyroid gland, usually in the
capsule that covers the lobes of the thyroid

B They are derived from pharyngeal pouches

C They sometimes may be found lying beside the thymus, which originates from
the same pharyngeal pouches

D Normally, they cover the back of the thyroid glands completely but,
rarely, they may cover only half of it

E They are four in number and are situated behind the thyroid gland, one at each
end of the upper and lower poles

D Normally, they cover the back of the thyroid glands completely but, rarely, they may
cover only half of it
Image(s) / Chart(s):
Click image to view full size. Click open image to close. Click and hold open image to
move.
Parathyroid glands are four small glands that lie, one at each end of the upper and
lower poles, on the posterior surface of the thyroid gland, embedded in its capsule.
Sometimes they are found embedded in the thyroid gland. The parathyroid glands are
derived from the pharyngeal pouches -- the superior glands from the fourth pouch and
the inferior glands from the third. They can be found in the mediastinum lying beside the
thymus, which originates from the same pharyngeal pouches.

See the attached Figure: The human parathyroid gland, viewed from the posterior of
thyroid glands and pharynx.

Question:

The major parenchymal cell types associated with the parathyroid glands include

Answer Choices:
A Oxyphils and follicular cells

B Parafollicular cells and chief cells

C Chief cells and oxyphils

D Follicular cells and parafollicular cells

E Chief cells and C-cells

C Chief cells and oxyphils

The parathyroid glands are composed of two primary types of cells, the Chief cells
(Principal cells), and the oxyphils. The Chief cells synthesize and secrete parathyroid
hormone (or parathormone), which acts to increase blood calcium levels. The oxyphils
constitute a smaller number of parenchymal cells of the parathyroid glands and are not
known to have a secretory role.

These cells are usually found intermixed within the parenchymal tissue and appear
strongly acidophilic. They also may be found in clusters. Their numbers increase with
age of the individual. Parafollicular cells are cells found in the thyroid gland which
secrete calcitonin, the hormone that reduces blood calcium levels. The follicular cells,
also found in the thyroid gland, synthesize and secrete thyroid hormones. C-cells are
another name for the parafollicular cells.

Question:

Which one of the following cells secretes parathormone (PTH)?

Answer Choices:

A C-cells

B Chief cells
C Follicular cells

D Parafollicular cells

E Oxyphils

B Chief cells

The parathyroid glands are composed of two primary types of cells, the Chief cells
(Principal cells), and the oxyphils. The Chief cells synthesize and secrete parathyroid
hormone (or parathormone), which acts to increase blood calcium levels. The oxyphils
constitute a smaller number of parenchymal cells of the parathyroid glands and are not
known to have a secretory role. These cells are usually found intermixed within the
parenchymal tissue and appear strongly acidophilic. They also may be found in clusters.
Their numbers increase with age of the individual. Parafollicular cells are cells found in
the thyroid gland which secrete calcitonin, the hormone that reduces blood calcium
levels. The follicular cells, also found in the thyroid gland, synthesize and secrete
thyroid hormones. C-cells are another name for the parafollicular cells.

Case #200010:

A 32-year-old woman presents with a solitary thyroid nodule. A biopsy confirms that
she has thyroid cancer. She subsequently has a thyroidectomy. However, during the
procedure, the surgeon accidentally removes most of her parathyroid glands as well.
As a consequence, she develops hypoparathyroidism. Her laboratory results are in
the chart.

TEST RESULTS REFERENCE RANGE


RBC 4.8 x 106/µl 4.5-5.7 x 106/µl (male)
3.9-5.0 x 106/µl (female)
Hematocrit 42 40-50 (male)
36-44 (female)
Hemoglobin 15 gm/dL 13.8-17.2 gm/dL (male)
12.1-15.1 gm/dL (female)
Platelets 252,000/mm3 130,000-400,000/mm3
Potassium 4.1 mEq/L 3.5-5.0 mEq/L
Sodium 144 mEq/L 135-145 mEq/L
Calcium 7.8 mg/dL 8.5-10.5 mg/dL
Question:
Which of the following symptoms would be consistent with her history and findings?

Answer Choices:

A Paresthesias

B Constipation

C Anorexia

D Vomiting

E Coma

A Paresthesias

This patient has hypocalcemia secondary to hypoparathyroidism. Paresthesias can be


seen with hypocalcemia due to the increase in neuromuscular irritability. Common sites
for the paresthesias are around the mouth and on the fingertips. Tetany can also be
seen.

Constipation, anorexia, vomiting, and coma can be seen with hypercalcemia.

Case #200064:

A 45-year-old woman is bending to pick up some groceries while grocery shopping


and she falls. X-ray confirms a fracture in her leg. The radiologist comments that there
is generalized osteopenia. Her primary doctor is concerned about the radiologist's
comments, so he questions her further. She asserts that neither parent had any
skeletal problems. Her father died of a heart attack, and her mother is healthy. She
acknowledges vague gastrointestinal complaints. She complains of anorexia, nausea,
and abdominal discomfort. She also has complains of weakness. Her laboratory work
is as follows:
Test Results Normal
BUN 15 mg/dL 10-20 mg/dL
Potassium 4.2 mEq/L 3.5-5.0 mEq/L
Sodium 140 mEq/L 135-145 mEq/L
Glucose (fasting) 91 mg/dL 65-110 mg/dL
Calcium 12.5 mg/dL 8.4-10.2 mg/dL
Alkaline Phosphatase 93 U/L 20-70 U/L
PTH radioimmunoassay elevated
Question:

Of the following choices, which of the following is most likely?

Answer Choices:

A Parathyroid adenoma

B Hyperthyroidism

C DiGeorge syndrome

D Hypoalbuminemia

E Benign familial hypercalcemia

A Parathyroid adenoma

This patient has signs and symptoms of primary hyperparathyroidism. The most
common cause of primary hyperparathyroidism is a benign parathyroid adenoma.

A key feature is the presence of hypercalcemia. The cause of this patient's


hypercalcemia is primary hyperparathyroidism. Primary hyperparathyroidism is a
common cause of hypercalcemia. Symptoms of hypercalcemia include anorexia,
nausea, abdominal pain, and weakness.

Osteopenia visualized on X-ray can be seen with primary hyperparathyroidism when


there is bony involvement. Elevated alkaline phosphatase is not always seen with
hyperparathyroidism; however, an elevated alkaline phosphatase is much more likely if
there is bone pathology, as is this case in this scenario.

This patient has no symptoms of hyperthyroidism. Although sometimes


hyperthyroidism can result in hypercalcemia, there would be concomitant symptoms of
hyperthyroidism present.
DiGeorge syndrome is a congenital condition in which the parathyroid glands are
rudimentary. This would lead to hypoparathyroidism, not hyperparathyroidism, as the
case here.

When hypoalbuminemia exists, there can be a decrease in the measurement of total


serum calcium, not an increase in the calcium measurement.

Benign familial hypercalcemia would result in elevated calcium, as the case here.
However, parathyroid radioimmunoassay would show normal to low levels of PTH. In
addition, benign familial hypercalcemia is inherited in an autosomal dominant manner,
so it would also be expected that her family history would be positive.

Reference:

Case #200010:

A 32-year-old woman presents with a solitary thyroid nodule. A biopsy confirms that
she has thyroid cancer. She subsequently has a thyroidectomy. However, during the
procedure, the surgeon accidentally removes most of her parathyroid glands as well.
As a consequence, she develops hypoparathyroidism. Her laboratory results are in
the chart.

TEST RESULTS REFERENCE RANGE


RBC 4.8 x 106/µl 4.5-5.7 x 106/µl (male)
3.9-5.0 x 106/µl (female)
Hematocrit 42 40-50 (male)
36-44 (female)
Hemoglobin 15 gm/dL 13.8-17.2 gm/dL (male)
12.1-15.1 gm/dL (female)
Platelets 252,000/mm3 130,000-400,000/mm3
Potassium 4.1 mEq/L 3.5-5.0 mEq/L
Sodium 144 mEq/L 135-145 mEq/L
Calcium 7.8 mg/dL 8.5-10.5 mg/dL
Question:

Which of the following is a sign of her secondary condition?

Answer Choices:

A Chvostek's sign
B Kernig's sign

C Babinski's sign

D Kussmaul's sign

E Quincke's sign

A Chvostek's sign

This patient has hypocalcemia secondary to hypoparathyroidism. Paresthesias can be


seen with hypocalcemia due to the increase in neuromuscular irritability. Common sites
for the paresthesias are around the mouth and on the fingertips. Tetany can also be
seen. Chvostek's signcan be seen with tetany. When tapping on the facial nerve
produces contraction on that side of the face, it is called Chvostek's sign.

Kernig's sign is a sign of meningeal irritation. The patient lies on his back and his leg
is raised and knee bent at a 90-degree angle. When the examiner straightens the knee,
if there is pain or resistance to further extension, Kernig's sign is said to be positive.

Babinski's sign is an abnormal response to stimulation on the sole of the foot; it results
in dorsiflexion of the big toe and fanning of the other toes. Babinski's sign is seen with
pyramidal disease.

Kussmaul's sign is an increase in venous pressure during inspiration; it can be seen


with cardiac tamponade.

Quincke's sign (Quincke's pulse) is an alteration seen in nails with each heartbeat.
Quincke's sign can be seen with aortic insufficiency.

Question:

A 44-year-old woman has parathyroid hyperplasia resulting in primary hyperparathyroidism. She sees
you for consultation. Several tests, including blood work, are performed. What symptom would be
consistent with her history and findings?

TEST RESULTS REFERENCE RANGE


4.5-5.7 X 106/µl (male)
RBC 4.8 X 106/µl
3.9-5.0 X 106/µl (female)
40-50 (male)
Hematocrit 42
36-44 (female)
13.8-17.2 gm/dL (male)
Hemoglobin 15 gm/dL
12.1-15.1 gm/dL (female)
Platelets 250,000/mm3 130,000 - 400,000 mm3
Potassium 4.1 mEq/L 3.5-5.0 mEq/L
Sodium 144 mEq/L 135-145 mEq/L
Calcium 12.8 mg/dL 8.5-10.5 mg/dL

Answer Choices:

A Coma

B Paresthesia

C Trousseau's sign

D Chvostek's sign

E Cataracts

A Coma

This patient has hypercalcemia secondary to primary hyperparathyroidism.


Hypercalcemia can result in central nervous system manifestations such as stupor,
somnolence, coma, and death. Constipation, anorexia, and vomiting can also be seen
with hypercalcemia.

Paresthesias can be seen with hypocalcemia. This is due to an increase in


neuromuscular irritability. Common sites for the paresthesias are around the mouth and
on the fingertips. Tetany can also be seen. Trousseau's sign, Chvostek's sign, and
cataracts can also be seen with hypocalcemia.

Case #200491:

A 44-year-old female has parathyroid hyperplasia resulting in primary


hyperparathyroidism. She sees you for consultation. You do several tests, including
blood work. Her laboratory results are as follows.

TEST RESULTS REFERENCE RANGE


6 4.5 -5.7 x 106/μl (male)
RBC 4.8 x 10 /μl
3.9 -5.0 x 106/μl (female)
40-50 (male)
Hematocrit 42
36-44 (female)
13.8-17.2 gm/dL (male)
Hemoglobin 15 gm/dL
12.1-15.1 gm/dL (female)
Platelets 250,000/mm3 130,000 - 400,000 mm3
potassium 4.1 mEq/L 3.5-5.0 mEq/L
sodium 144 mEq/L 135-145 mEq/L
calcium 12.8 mg/dL 8.5-10.5 mg/dL
Question:

Which of the following symptoms would be consistent with her history and findings?

Answer Choices:

A Constipation

B Paresthesia

C Trousseau's sign

D Chvostek's sign

E Cataracts

A Constipation

This patient has hypercalcemia secondary to primary hyperparathyroidism.


Hypercalcemia can result in central nervous system manifestations: stupor,
somnolence, coma, and death. Constipation, anorexia, and vomiting can also be seen
with hypercalcemia.

Paresthesias can be seen with hypocalcemia. This is due to an increase in


neuromuscular irritability. Common sites for the paresthesias are around the mouth and
on the fingertips. Tetany can also be seen. Trousseau's sign, Chvostek's sign, and
cataracts can also be seen with hypocalcemia.

Case #200492:

A 44-year-old female has parathyroid hyperplasia resulting in primary


hyperparathyroidism. She sees you for consultation. You do several tests, including
blood work. Her laboratory results are as follows.

TEST RESULTS REFERENCE RANGE


4.5 -5.7 x 106/μl (male)
6
RBC 4.8 x 10 /μl
3.9 -5.0 x 106/μl (female)
40-50 (male)
Hematocrit 42
36-44 (female)
13.8-17.2 gm/dL (male)
Hemoglobin 15 gm/dL
12.1-15.1 gm/dL (female)
Platelets 250,000/mm3 130,000 - 400,000 mm3
potassium 4.1 mEq/L 3.5-5.0 mEq/L
sodium 144 mEq/L 135-145 mEq/L
calcium 12.8 mg/dL 8.5-10.5 mg/dL
Question:

Which of the following symptoms would be consistent with her history and findings?

Answer Choices:

A Vomiting

B Paresthesia

C Trousseau's sign

D Chvostek's sign

E Cataracts

A Vomiting
This patient has hypercalcemia secondary to primary hyperparathyroidism.
Hypercalcemia can result in central nervous system manifestations: stupor,
somnolence, coma, and death. Constipation, anorexia, and vomiting can also be seen
with hypercalcemia.

Paresthesias can be seen with hypocalcemia. This is due to an increase in


neuromuscular irritability. Common sites for the paresthesias are around the mouth and
on the fingertips. Tetany can also be seen. Trousseau's sign, Chvostek's sign, and
cataracts can also be seen with hypocalcemia.

ADRENAL GLAND:
Question:

Innervation of the adrenal medulla comes from

Answer Choices:

A Preganglionic parasympathetic neurons

B Preganglionic sympathetic neurons

C Postganglionic parasympathetic neurons

D Postganglionic sympathetic neurons

B Preganglionic sympathetic neurons

The chromaffin cells of the adrenal medulla are innervated by fibers from preganglionic
sympathetic neurons. Stimulation of the cells of the medulla will result in the rapid
release of catecholamines. Because of this relationship to the preganglionic
sympathetic fibers, and the catecholamine product from the medullary cells, the
medullary cells are considered to be equivalent to postganglionic sympathetic neurons,
except they lack axons. These cells are similar to endocrine cells, both in appearance,
and by the fact that the secretory product is secreted into the medullary capillaries.

Question:

Cells forming the adrenal medulla are derived from

Answer Choices:

A Surface ectoderm

B Neural crest

C Endoderm

D Intraembryonic mesoderm

E Extraembryonic mesoderm

B Neural crest

The adrenal gland arises from cells of two different embryological sources. The adrenal
medulla originates from neural crest cells, whereas, the cortical cells originate from
intraembryonic mesodermal cells.

Question:

The capillaries that provide oxygenated blood to the adrenal capsule are referred to as
Answer Choices:

A Capsular capillaries

B Medullary arterioles

C Medullary capillary sinusoids

D Cortical sinusoidal capillaries

E Cortical portal veins

A Capsular capillaries

Blood supply to the adrenal gland comes from three primary arteries, the superior,
middle and inferior adrenal arteries. Branches of these arteries form small arteries that
penetrate the capsule and provide blood supply to the capsule (capsular capillaries),
as well as branches that supply the cortex and medulla. The adrenal cortex is supplied
by cortical sinusoidal capillaries, which then drain into the medulla as capillary
sinusoids. The medulla receives arterioles that traverse the cortex via the trabeculae
from the capsule and provide oxygenated blood to medullary capillaries. Blood from
these medullary arterioles combine with the venous blood from the cortical sinusoidal
capillaries and drain to small medullary veins.

Question:

The capillaries that supply oxygenated blood to the adrenal cortex, then drain into the medulla are
called

Answer Choices:
A Capsular capillaries

B Medullary arterioles

C Medullary capillary sinusoids

D Cortical sinusoidal capillaries

E Cortical portal veins

D Cortical sinusoidal capillaries

Blood supply to the adrenal gland comes from three primary arteries, the superior,
middle and inferior adrenal arteries. Branches of these arteries form small arteries that
penetrate the capsule and provide blood supply to the capsule (capsular capillaries), as
well as branches that supply the cortex and medulla. The adrenal cortex is supplied by
cortical sinusoidal capillaries, which then drain into the medulla as capillary
sinusoids. The medulla receives arterioles that traverse the cortex via the trabeculae
from the capsule and provide oxygenated blood to medullary capillaries. Blood from
these medullary arterioles combine with the venous blood from the cortical sinusoidal
capillaries and drain to small medullary veins.

Question:

Which cells in the adrenal gland synthesize and secrete epinephrine?

Answer Choices:

A Cells located in the zona glomerulosa

B Cells located in the zona reticularis


C Cells located in the zona fasciculata

D Cells located in the adrenal medulla

E Cells located in the zona pellucida

D Cells located in the adrenal medulla

The adrenal cortex is the steroid-secreting area of the gland, which comprises most all
of the gland (adrenal medulla represents 10-15% of the gland). The cortex is divided
histologically into three zones.

The first is the zona glomerulosa, a narrow outer zone underlying the capsule, so
named because of the arrangement of the parenchymal cells in whirls or ovoid clusters
appearing somewhat like glomeruli. These cells synthesize and secrete
mineralocorticoids, the principal being aldosterone. Aldosterone acts on the distal
tubules of the nephron to help regulate sodium and potassium homeostasis and water
balance.

The second region is the zona fasciculata, a thick middle region having cells arranged in
columns. These cells are large, light staining polygonal cells that synthesize and secrete
glucocorticoids, principally cortisol. Cortisol acts on a variety of cells and tissues to
increase metabolic availability of glucose and to depress the immune response.

The third innermost region is the zona reticularis. These small, more deeply staining
cells synthesize and secrete weak androgens, principally dehydroepiandrosterone.
Epinephrine (adrenalin) and norepinephrine (noradrenalin) are hormones secreted from
the adrenal medulla.

Question:

Which cells in the adrenal gland synthesize and secrete dehydroepiandrosterone?

Answer Choices:

A Cells located in the zona glomerulosa


B Cells located in the zona reticularis

C Cells located in the zona fasciculata

D Cells located in the adrenal medulla

E Distal tubule cells located in the renal cortex

B Cells located in the zona reticularis

The adrenal cortex is the steroid-secreting area of the gland, which comprises most all
of the gland (adrenal medulla represents 10-15% of the gland). The cortex is divided
histologically into three zones.

The first is the zona glomerulosa, a narrow outer zone underlying the capsule, so
named because of the arrangement of the parenchymal cells in whirls or ovoid clusters
appearing somewhat like glomeruli. These cells synthesize and secrete
mineralocorticoids, the principal being aldosterone. Aldosterone acts on the distal
tubules of the nephron to help regulate sodium and potassium homeostasis and water
balance.

The second region is the zona fasciculata, a thick middle region having cells arranged in
columns. These cells are large, light staining polygonal cells that synthesize and secrete
glucocorticoids, principally cortisol. Cortisol acts on a variety of cells and tissues to
increase metabolic availability of glucose and to depress the immune response.

The third innermost region is the zona reticularis. These small, more deeply staining
cells synthesize and secrete weak androgens, principally dehydroepiandrosterone.
Epinephrine (adrenalin) and norepinephrine (noradrenalin) are hormones secreted from
the adrenal medulla.

Question:

Which cells in the adrenal gland synthesize and secrete aldosterone?


Answer Choices:

A Cells located in the zona glomerulosa

B Cells located in the zona reticularis

C Cells located in the zona fasciculata

D Cells located in the adrenal medulla

E Distal tubule cells located in the renal cortex

A Cells located in the zona glomerulosa

The adrenal cortex is the steroid-secreting area of the gland, which comprises most all
of the gland (adrenal medulla represents 10-15% of the gland). The cortex is divided
histologically into three zones.

The first is the zona glomerulosa, a narrow outer zone underlying the capsule, so
named because of the arrangement of the parenchymal cells in whirls or ovoid clusters
appearing somewhat like glomeruli. These cells synthesize and secrete
mineralocorticoids, the principal being aldosterone. Aldosterone acts on the distal
tubules of the nephron to help regulate sodium and potassium homeostasis and water
balance.

The second region is the zona fasciculata, a thick middle region having cells arranged in
columns. These cells are large, light staining polygonal cells that synthesize and secrete
glucocorticoids, principally cortisol. Cortisol acts on a variety of cells and tissues to
increase metabolic availability of glucose and to depress the immune response.

The third innermost region is the zona reticularis. These small, more deeply staining
cells synthesize and secrete weak androgens, principally dehydroepiandrosterone.
Epinephrine (adrenalin) and norepinephrine (noradrenalin) are hormones secreted from
the adrenal medulla.

Question:
Which cells in the adrenal gland synthesize and secrete cortisol?

Answer Choices:

A Cells located in the zona glomerulosa

B Cells located in the zona reticularis

C Cells located in the zona fasciculata

D Cells located in the adrenal medulla

E Distal tubule cells located in the renal cortex

C Cells located in the zona fasciculata

The adrenal cortex is the steroid-secreting area of the gland, which comprises most all
of the gland (adrenal medulla represents 10-15% of the gland). The cortex is divided
histologically into three zones.

The first is the zona glomerulosa, a narrow outer zone underlying the capsule, so
named because of the arrangement of the parenchymal cells in whirls or ovoid clusters
appearing somewhat like glomeruli. These cells synthesize and secrete
mineralocorticoids, the principal being aldosterone. Aldosterone acts on the distal
tubules of the nephron to help regulate sodium and potassium homeostasis and water
balance.

The second region is the zona fasciculata, a thick middle region having cells arranged
in columns. These cells are large, light staining polygonal cells that synthesize and
secrete glucocorticoids, principally cortisol. Cortisol acts on a variety of cells and tissues
to increase metabolic availability of glucose and to depress the immune response.

The third innermost region is the zona reticularis. These small, more deeply staining
cells synthesize and secrete weak androgens, principally dehydroepiandrosterone.
Epinephrine (adrenalin) and norepinephrine (noradrenalin) are hormones secreted from
the adrenal medulla.
Question:

The principal secretion of the cells of the zona glomerulosa of the adrenal cortex is

Answer Choices:

A Dehydroepiandrosterone

B Cortisol

C Epinephrine

D Noradrenalin

E Aldosterone

E Aldosterone

The adrenal cortex is the steroid-secreting area of the gland, which comprises most all
of the gland (adrenal medulla represents 10-15% of the gland). The cortex is divided
histologically into three zones.

The first is the zona glomerulosa, a narrow outer zone underlying the capsule, so
named because of the arrangement of the parenchymal cells in whirls or ovoid clusters
appearing somewhat like glomeruli. These cells synthesize and secrete
mineralocorticoids, the principal being aldosterone. Aldosterone acts on the distal
tubules of the nephron to help regulate sodium and potassium homeostasis and water
balance.

The second region is the zona fasciculata, a thick middle region having cells arranged
in columns. These cells are large, light staining polygonal cells that synthesize and
secrete glucocorticoids, principally cortisol. Cortisol acts on a variety of cells and tissues
to increase metabolic availability of glucose and to depress the immune response.

The third innermost region is the zona reticularis. These small, more deeply staining
cells synthesize and secrete weak androgens, principally dehydroepiandrosterone.
Epinephrine (adrenalin) and norepinephrine (noradrenalin) are hormones secreted from
the adrenal medulla.

Question:

The organ as depicted in the attached photomicroscopic image is best described as

Answer Choices:

A Adrenal cortex

B Adrenal medulla

C Fundic stomach

D Cardiac stomach

E Renal cortex

Image(s) / Chart(s):
Click image to view full size. Click open image to close. Click and hold open image to move.
A Adrenal cortex

The photomicroscopic image illustrates the appearance of the adrenal cortex. This is
the steroid-secreting area of the gland, which comprises most all of the gland (adrenal
medulla represents 10-15% of the gland). The photomicrograph reveals the presence of
thick connective tissue capsule surrounding the gland with trabeculae that extend into
the glandular tissue. The cortex is divided histologically into three zones.

The first is the zona glomerulosa, a narrow outer zone underlying the capsule, so
named because of the arrangement of the parenchymal cells in whirls or ovoid clusters
appearing somewhat like glomeruli. These cells synthesize and secrete
mineralocorticoids, the principal being aldosterone. Aldosterone acts on the distal
tubules of the nephron to help regulate sodium and potassium homeostasis and water
balance.

The second region is the zona fasciculata, a thick middle region having cells arranged in
columns. These cells are large, light staining polygonal cells that synthesize and secrete
glucocorticoids, principally cortisol. Cortisol acts on a variety of cells and tissues to
increase metabolic availability of glucose and to depress the immune response.

The third innermost region is the zona reticularis. These small, more deeply staining
cells synthesize and secrete weak androgens, principally dehydroepiandrosterone.
Question:

The production of cortisol in adrenal cortex is stimulated by

Answer Choices:

A Angiotensin I

B Angiotensin II

C ACTH

D Renin

E LH

C ACTH

ACTH (adrenocorticotropic hormone) stimulates the secretion of glucocorticoid cortisol


in zona fasciculata of adrenal cortex. The renin-angiotensin system regulates the
production of mineralocorticoids in zona glomerulosa. The primary hormone of this
system, angiotensin II, is formed from a decapeptide angiotensin I by angiotensin-
converting enzyme, found in lung, endothelial cells, and plasma. Renin is a protease
secreted by juxtaglomerular cells of the renal arteriole; it cleaves an α2-globulin
angiotensinogen to produce angiotensin II. LH (luteinizing hormone) is produced in the
pituitary gland and regulates the synthesis of sex hormones.

Question:

Which of the following statements concerning adrenal parenchymal cells is true?


Answer Choices:

A Those of zona fasciculata produce androgens

B Those of the adrenal medulla produce epinephrine and norepinephrine

C Those of zona glomerulosa produce glucocorticoids

D Those of the cortex contain secretory granules

E Those of the adrenal medulla produce mineralocorticoids

B Those of the adrenal medulla produce epinephrine and norepinephrine

Chromaffin cells in the adrenal medulla synthesize and store epinephrine and
norepinephrine in secretory granules, which also contain ATP, chromogranins, and
enkephalins.

The cortical parenchymal cells of the zona fasciculata produce glucocorticoids, and
those of the zona glomerulosa produce mineralocorticoids. The cortical parenchymal
cells do not store their secretory products and thus do not contain secretory granules.

Question:

The principal secretion of the cells of the zona fasciculata of the adrenal cortex is

Answer Choices:

A Aldosterone
B Cortisol

C Dehydroepiandrosterone

D Calcitonin

E Adrenalin

B Cortisol

The adrenal cortex is the steroid-secreting area of the gland, which comprises most all
of the gland (adrenal medulla represents 10-15% of the gland). The cortex is divided
histologically into three zones.

The first is the zona glomerulosa, a narrow outer zone underlying the capsule, so
named because of the arrangement of the parenchymal cells in whirls or ovoid clusters
appearing somewhat like glomeruli. These cells synthesize and secrete
mineralocorticoids, the principal being aldosterone. Aldosterone acts on the distal
tubules of the nephron to help regulate sodium and potassium homeostasis and water
balance.

The second region is the zona fasciculata, a thick middle region having cells arranged in
columns. These cells are large, light staining polygonal cells that synthesize and secrete
glucocorticoids, principally cortisol. Cortisol acts on a variety of cells and tissues to
increase metabolic availability of glucose and to depress the immune response.

The third innermost region is the zona reticularis. These small, more deeply staining
cells synthesize and secrete weak androgens, principally dehydroepiandrosterone.
Epinephrine (adrenalin) and norepinephrine (noradrenalin) are hormones secreted from
the adrenal medulla. Calcitonin is a hormone secreted from the parafollicular cells of the
thyroid gland.

Question:

Which cells in the adrenal gland synthesize and secrete norepinephrine?


Answer Choices:

A Cells located in the zona glomerulosa

B Cells located in the zona reticularis

C Cells located in the zona fasciculata

D Cells located in the adrenal medulla

E Distal tubule cells located in the renal cortex

D Cells located in the adrenal medulla

The adrenal cortex is the steroid-secreting area of the gland, which comprises most all
of the gland (adrenal medulla represents 10-15% of the gland). The cortex is divided
histologically into three zones.

The first is the zona glomerulosa, a narrow outer zone underlying the capsule, so
named because of the arrangement of the parenchymal cells in whirls or ovoid clusters
appearing somewhat like glomeruli. These cells synthesize and secrete
mineralocorticoids, the principal being aldosterone. Aldosterone acts on the distal
tubules of the nephron to help regulate sodium and potassium homeostasis and water
balance.

The second region is the zona fasciculata, a thick middle region having cells arranged in
columns. These cells are large, light staining polygonal cells that synthesize and secrete
glucocorticoids, principally cortisol. Cortisol acts on a variety of cells and tissues to
increase metabolic availability of glucose and to depress the immune response.

The third innermost region is the zona reticularis. These small, more deeply staining
cells synthesize and secrete weak androgens, principally dehydroepiandrosterone.
Epinephrine (adrenalin) and norepinephrine (noradrenalin) are hormones secreted from
the adrenal medulla.
Question:

The cortical sinusoidal capillaries that supply oxygenated blood to the adrenal cortex, drain into the
medulla into the

Answer Choices:

A Capsular capillaries

B Medullary arterioles

C Medullary capillary sinusoids

D Cortical sinusoidal capillaries

E Cortical portal veins

C Medullary capillary sinusoids

Blood supply to the adrenal gland comes from three primary arteries, the superior,
middle and inferior adrenal arteries. Branches of these arteries form small arteries that
penetrate the capsule and provide blood supply to the capsule (capsular capillaries), as
well as branches that supply the cortex and medulla. The adrenal cortex is supplied by
cortical sinusoidal capillaries, which then drain into the medulla as capillary sinusoids.
The medulla receives arterioles that traverse the cortex via the trabeculae from the
capsule and provide oxygenated blood to medullary capillaries. Blood from these
medullary arterioles combine with the venous blood from the cortical sinusoidal
capillaries and drain to small medullary veins.

Question:

Conversion of norepinephrine to epinephrine in adrenal medullary cells is induced by which one of the
following substances?

Answer Choices:

A Cortisol, which induces methylation of norepinephrine to produce


epinephrine

B Cortisol, which induces hydroxylation of norepinephrine to produce


epinephrine

C Aldosterone, which induces methylation of norepinephrine to produce


epinephrine

D Aldosterone, which induces hydroxylation of norepinephrine to produce


epinephrine

E Dehydroepiandrosterone, which induces methylation of norepinephrine to


produce epinephrine

A Cortisol, which induces methylation of norepinephrine to produce epinephrine

Glucocorticoids, such as cortisol, produced in the adrenal cortical zona fasciculata, is


secreted into the cortical sinusoidal capillaries and drain into the medullary capillaries to
induce the enzyme that catalyzes the methylation of norepinephrine to epinephrine.

Question:

Zona reticularis of adrenal cortex secretes


Answer Choices:

A Dopamine

B Prednisolone

C Cortisol

D Dehydroepiandrosterone

E Aldosterone

D Dehydroepiandrosterone

Corticosterone and aldosterone, steroid hormones with predominantly mineralocorticoid


activity, are produced in zona glomerulosa of adrenal cortex; glucocorticoid cortisol is
made in zona fasciculata; the most potent precursor of androgen hormones
dehydroepiandrosterone (DHEA) is secreted in zona reticularis. Prednisolone is a
synthetic analog of cortisol (Δ1-cortisol). The adrenal medulla consists of chromaffin
cells which synthesize and store catecholamines (dopamine, norepinephrine, and
epinephrine).

Question:

Which of the following steroid hormones is an example of a glucocorticoid?

Answer Choices:

A Estrogen

B Testosterone
C Ecdysone

D Cortisol

E Aldosterone

D Cortisol

All steroid hormones are synthesized from cholesterol. Progesterone, estrogen, and
testosterone are the sex steroids, and are produced by the gonads. Cortisol and
aldosterone are corticosteroids, produced by the adrenal gland. Cortisol is a
glucocorticoid, and acts on many cell types, to stimulate glucose production.
Aldosterone is a mineralocorticoid and acts on the kidney to regulate salt and water
metabolism.

Ecdysone is an insect steroid hormone that plays a key role in development, by


triggering metamorphosis, from the larvae to the adult.

Question:

The principal secretion of the cells of the zona reticularis of the adrenal cortex is

Answer Choices:

A Adrenalin

B Aldosterone

C Cortisol

D Dehydroepiandrosterone
E Epinephrine

D Dehydroepiandrosterone

The adrenal cortex is the steroid-secreting area of the gland, which comprises most all
of the gland (adrenal medulla represents 10-15% of the gland). The cortex is divided
histologically into three zones.

The first is the zona glomerulosa, a narrow outer zone underlying the capsule, so
named because of the arrangement of the parenchymal cells in whirls or ovoid clusters
appearing somewhat like glomeruli. These cells synthesize and secrete
mineralocorticoids, the principal being aldosterone. Aldosterone acts on the distal
tubules of the nephron to help regulate sodium and potassium homeostasis and water
balance.

The second region is the zona fasciculata, a thick middle region having cells arranged in
columns. These cells are large, light staining polygonal cells that synthesize and secrete
glucocorticoids, principally cortisol. Cortisol acts on a variety of cells and tissues to
increase metabolic availability of glucose and to depress the immune response.

The third innermost region is the zona reticularis. These small, more deeply staining
cells synthesize and secrete weak androgens, principally dehydroepiandrosterone.
Epinephrine (adrenalin) and norepinephrine (noradrenalin) are hormones secreted from
the adrenal medulla.

Question:

Neural crest cells migrate into the developing adrenal gland to form which area?

Answer Choices:

A Cells of the medulla


B Cells of the zona glomerulosa

C Cells of the zona reticularis

D Cells of the zona fasciculata

E Neural crest cells do NOT contribute to formation of the adrenal gland

A Cells of the medulla

The adrenal gland arises from cells of two different embryological sources. The adrenal
medulla originates from neural crest cells, whereas the cortical cells originate from
intraembryonic mesodermal cells.

Question:

A 24-year-old college graduate presents with paroxysmal attacks of headaches, anxiety, and excessive
sweating for the past month. On examination, there is no pallor, icterus, edema, or lymphadenopathy.
His BP is 150/78 mmHg. On funduscopy, hypertensive changes are noted. Biochemical investigations
reveal an elevated blood sugar level and increased levels of urinary VMA (vanillyl mandelic acid). He
was diagnosed with pheochromocytoma, a diagnosis that is confirmed by a CT scan showing a mass
on the left. On being informed of the condition and the treatment options available to him, he consents
to surgery. During surgery, after location of the adrenal tumor, the vein that needs to be ligated before
excision is the left adrenal vein. The left adrenal vein drains into which of the following veins?

Answer Choices:

A Inferior vena cava

B Renal vein

C Gonadal vein
D Azygous vein

E Inferior mesenteric vein

B Renal vein

The venous drainage of the adrenal gland is through a single vein. The left adrenal
vein drains into the left renal vein, whereas the right adrenal vein drains into the
inferior vena directly. The patient here has an adrenal mass on the left. The
pheochromocytoma (PCC) tumors arise from chromaffin cells of the adrenal medulla.
The presentations of PCC vary with the production of active metabolites. The episodic
alpha-adrenergic hypersecretion results in the manifestations of intermittent malignant
hypertension. PCC commonly develops in young-to-middle-aged adults. The classic
triad is episodic headache, tachycardia, and diaphoresis. Hypertension is the most
common clinical sign of pheochromocytoma.

The adrenal gland is supplied by 3 arteries: the superior suprarenal arteries (a branch of
inferior phrenic artery), middle suprarenal arteries (directly from the aorta), and inferior
suprarenal arteries (from renal arteries).

The venous drainage of the adrenal gland must be ligated before handling the tumor to
prevent the entry of enormous amounts of adrenaline into the blood stream. Failure to
ensure proper ligation can lead to life threatening complications.

SECTION- THE EYE


Question:

Which one of the following structures of the eye is regarded as the anterior "window" of the eye?

Answer Choices:

A Lens

B Cornea
C Optic nerve

D Anterior chamber

E Vitreous space

B Cornea

The transparent components of the eye allow light rays to pass and become focussed
upon the photoreceptors of the retina. The first refractile component regarded as the
"window" of the eye is the cornea. The lens, also refractile, is a crystalline biconcave
structure suspended from the ciliary body. The lens separates the aqueous-filled
anterior and posterior chambers from the vitreous chamber.

Question:

The disease called Leber's Hereditary Optic Neuropathy (LHON), is unusual in its pattern of inheritance
because

Answer Choices:

A It is always transmitted by males to their offspring

B It shows a pattern of cytoplasmic rather than nuclear inheritance

C It always skips one generation

D It can be linked to either the X or Y chromosome

E It can only occur in one eye

B It shows a pattern of cytoplasmic rather than nuclear inheritance


The disease Leber's Hereditary Optic Neuropathy (LHON) is a rare inherited disorder
resulting in blindness due to optic nerve degeneration, usually between ages 15 and 35.
The majority of cases are males, which suggests an X-linked trait. However, males
never transmit the disease to their offspring. This defect is the result of a cytoplasmic
pattern of inheritance and is due to a defect in a mitochondrial gene. The fact that
mitochondria are inherited from the egg explains this inheritance pattern. The mutation
is found in the gene coding for one of the subunits of Complex I of the respiratory chain,
NADH dehydrogenase. Reduction in this gene product results in a decreased capacity
of the mitochondria to generate ATP by electron transport. Because the brain and nerve
tissue are the most dependent on oxidative metabolism, these tissues are greatly
affected.

Question:

Which one of the following components surrounds the fovea centralis?

Answer Choices:

A Outer plexiform layer

B Outer nuclear layer

C Müller's cells

D Macula lutea

E Layer of rods and cones

D Macula lutea
Image(s) / Chart(s):
Click image to view full size. Click open image to close. Click and hold open image to
move.
Macula lutea, a yellow pigmented zone, surrounds the fovea. The outer plexiform layer
(5th layer of retina) contains the processes of retinal rods and cones and processes of
the horizontal, amacrine, and bipolar cells that connect to them. The outer nuclear layer
(4th layer of retina) contains the cell bodies (nuclei) of retinal rods and cones. Müller's
cells are supporting cells of the retina. Layer of rods and cones form the outer and inner
segments of the photoreceptor cells.

See the attached Figure: A scan of the human retina. Rods and cones (Rc); Nuclei of
rods and cones (N); Outer plexiform layer (P); Nuclei of bipolar, horizontal, amacrine
and Müller's cells (M); Inner plexiform layer (Pi); and Ganglion cell layer (G).

Question:

Bruch's membrane

Answer Choices:

A Is a loose layer of connective tissue rich in melanocytes

B Is an oxytalan fiber that inserts into the capsule of the lens and anchors it in
place

C Separates the choriocapillary layer from retina

D Is an anterior expansion of the choroid at the level of the lens

E Is an area of transition from the transparent collagen bundles of the cornea to


the white opaque fibers of the sclera

C Separates the choriocapillary layer from retina

Bruch's membrane is a thin (3-4 μm) amorphous hyaline membrane that separates the
choriocapillary layer from the retina. It extends from the optic disk to the ora serrata.
Suprachoroidal lamina is a loose layer of connective tissue rich in melanocytes. Zonule
fiber is an oxytalan fiber that inserts into the capsule of the lens and anchors it in place.
The ciliary body is an anterior expansion of the choroid at the level of the lens. The
corneoscleral junction, or limbus, is an area of transition from the transparent collagen
bundles of the cornea to the white opaque fibers of the sclera.

Question:

Which one of the following conditions of the eye occurs regularly with age?

Answer Choices:

A Acute glaucoma

B Presbyopia

C Detachment of retina

D Hordeolum

E Chronic glaucoma
B Presbyopia

Advancing age reduces the elasticity of lens, making accommodation for near objects
difficult. This is a normal aging process (presbyopia) which can be corrected by wearing
glasses with convex lenses.

Glaucoma is caused by obstruction that prevents drainage of aqueous humor from the
anterior chamber of the eye. Acute glaucoma is relatively rare but leads to blindness
within a few days if not treated. Chronic glaucoma may be associated with few
symptoms except for gradual loss of peripheral vision.

Detachment of the retina occurs when the neural and pigmented retinae become
separated from each other (as they are not adhering to each other).

Hordeolum (or stye) is a localized, inflammatory infection of a sebaceous gland of the


eyelid frequently caused by staphylococcal bacteria. It can involve the meibomian
glands or the glands of Moll.

Question:

The letter "B" as seen in this photomicroscopic image of the retina refers to which one of the following
structures?

Answer Choices:

A Outer nuclear layer.

B Outer limiting membrane.

C Photoreceptor layer.

D Inner nuclear layer.

E Ganglion cell layer.


Image(s) / Chart(s):
Click image to view full size. Click open image to close. Click and hold open image to move.

E Ganglion cell layer.

The photomicroscopic image depicts the histological image through a cross- section of
the retina.

The letter 'A' indicates the space within the eye in which the vitreous humor is located,
referred to as the vitreous chamber.

The letter 'B' indicates the location of the ganglion cell layer. The axons from these
cells pass into the nerve fiber layer and thence into the optic nerve.

The dendrites of the ganglion cells ramify in the inner plexiform layer, indicated by the
letter 'C' in the photomicrographic image. The inner plexiform layer (C), is a layer in
which processes of the amacrine, bipolar and ganglion cells synapse.

The letter 'D' indicates the location of the inner nuclear layer which contains cell
bodies of the horizontal, amacrine, bipolar and Mueller's cells.
Three of the cells are regarded as neural conducting cells, the bipolar, horizontal and
amacrine cells. The bipolar cells are oriented so that their processes extend to both the
inner and outer plexiform layers to establish connections in both of these layers. The
horizontal cell processes synapse with the bipolar cell processes in the external
plexiform layer. The amacrine cells provide synaptic interaction among the axonal
terminals of the bipolar cells and the ganglion cell dendrites. Their axons synapse in the
inner plexiform layer.

The Mueller cells function to provide support for the retina. Their basal and apical ends
form the internal and external limiting membranes. The letter 'E' indicates the outer
plexiform layer where processes of the photoreceptor cells synapse with the processes
of the cells located in the inner nuclear layer. The nuclei of the photoreceptors (rods and
cones) lie within the outer nuclear layer, indicated by the letter 'F'. The outer segments
of the photoreceptors are located in area indicated by the letter 'G', whose apical
portions are loosely embedded within the retinal pigment epithelium (H).

Question:

Which one of the following components of the eye arises from the embryonic surface ectoderm?

Answer Choices:

A Epithelium of the retina

B Sclera

C Lens

D Iris

E Ciliary body

C Lens

The embryonic surface epithelium give rise to lens, epithelium of the cornea,
conjunctiva, and lacrimal gland and its drainage system. The iris and ciliary body arise
from neural ectoderm, while mesoderm give rise to sclera.

Question:

Zonules

Answer Choices:

A Are radially oriented fibers that insert on one side of the lens capsule
and on the other side on the ciliary body

B Form the homogeneous, refractile, carbohydrate-rich capsule coating the outer


surface of the epithelium of the lens

C Form the lens

D Form the iris

E Are part of the transparent gel contained inside the posterior chamber of the
eye

A Are radially oriented fibers that insert on one side of the lens capsule and on the
other side on the ciliary body

The lens is held in place by a radially oriented group of fibers, the zonule, that inserts on
one side of the lens capsule and on the other side on the ciliary body. Collagen type IV
and amorphous glycoprotein form the homogeneous, refractile, carbohydrate-rich
capsule coating the outer surface of the epithelium of the lens. Lens fibers form the
lens.

Iris is an extension of the choroid. It is formed by a discontinuous layer of pigment cells


and fibroblasts, a connective tissue layer with few fibers but many fibroblasts and
melanocytes, vascular layer, and the epithelium.
The vitreous body contains water (99%), collagen, and heavily hydrated
glycosaminoglycans.

Question:

Which one of the followings is located inside the anterior chamber of the eye?

Answer Choices:

A Ora serrata

B Fovea

C Canal of Schlemm

D Optic papilla

E Vitreous body

C Canal of Schlemm

The canal of Schlemm, formed by the union of the spaces of Fontana, is located inside
the anterior chamber of the eye. Ora serrata (marking the anterior limit of both the retina
and the choroid), fovea, optic papilla, and vitreous body are contained inside the
posterior chamber.

Question:

The external (or outer) limiting membrane of the retina is formed by tight junctions among the apical
boundaries of which one of the following cell types?
Answer Choices:

A Horizontal cells

B Bipolar cells

C Amacrine cells

D Photoreceptor cells

E Mueller cells

E Mueller cells

The axons of the ganglion cell layer pass into the nerve fiber layer and thence into the
optic nerve. The dendrites of the ganglion cells ramify in the inner plexiform layer.
Processes of the amacrine, bipolar and ganglion cells synapse in the inner plexiform
layer; their cell bodies are located in the inner nuclear layer, which also contains cell
bodies of the Mueller cells. The bipolar cells are oriented so that their processes extend
to both the inner and outer plexiform layers to establish connections in both of these
layers. The horizontal cell processes synapse with the bipolar cell processes in the
external plexiform layer. The amacrine cells provide synaptic interaction among the
axonal terminals of the bipolar cells and the ganglion cell dendrites. Their axons
synapse in the inner plexiform layer.

The Mueller cells function to provide support for the retina. Their basal and apical ends
form the internal and external limiting membranes. Processes of the photoreceptor cells
synapse in the outer plexiform layer with processes of cells located in the inner nuclear
layer. The nuclei of the photoreceptors (rods and cones) lie within the outer nuclear
layer. The outer segments of the photoreceptors are loosely embedded within the retinal
pigment epithelium.

Question:

The internal (or inner) limiting membrane of the retina, is formed by tight junctions among the basal
boundaries of which one of the following cell types?
Answer Choices:

A Horizontal cells

B Bipolar cells

C Amacrine cells

D Photoreceptor cells

E Mueller cells

E Mueller cells

The axons of the ganglion cell layer pass into the nerve fiber layer, and thence, into the
optic nerve. The dendrites of the ganglion cells ramify in the inner plexiform layer.
Processes of the amacrine, bipolar, and ganglion cells synapse in the inner plexiform
layer; their cell bodies are located in the inner nuclear layer, which also contains cell
bodies of the Mueller cells. The bipolar cells are oriented so that their processes extend
to both the inner and outer plexiform layers to establish connections in both of these
layers. The horizontal cell processes synapse with the bipolar cell processes in the
external plexiform layer. The amacrine cells provide synaptic interaction among the
axonal terminals of the bipolar cells and the ganglion cell dendrites. Their axons
synapse in the inner plexiform layer.

The Mueller cells function to provide support for the retina. Their basal and apical ends
form the internal and external limiting membranes. Processes of the photoreceptor cells
synapse in the outer plexiform layer with processes of cells located in the inner nuclear
layer. The nuclei of the photoreceptors (rods and cones) lie within the outer nuclear
layer. The outer segments of the photoreceptors are loosely embedded within the retinal
pigment epithelium.

Question:

Cell bodies whose axonal projections form the optic nerve are located in which layer of the retina?
Answer Choices:

A Photoreceptor cell layer

B Ganglion cell layer

C Outer nuclear layer

D Inner nuclear layer

E Outer plexiform layer

B Ganglion cell layer

The axons of the ganglion cell layer pass into the nerve fiber layer and thence into the
optic nerve. The dendrites of the ganglion cells ramify in the inner plexiform layer.
Processes of the amacrine, bipolar and ganglion cells synapse in the inner plexiform
layer; their cell bodies are located in the inner nuclear layer, which also contains cell
bodies of the Mueller cells. The bipolar cells are oriented so that their processes extend
to both the inner and outer plexiform layers to establish connections in both of these
layers. The horizontal cell processes synapse with the bipolar cell processes in the
external plexiform layer. The amacrine cells provide synaptic interaction among the
axonal terminals of the bipolar cells and the ganglion cell dendrites. Their axons
synapse in the inner plexiform layer.

The Mueller cells function to provide support for the retina. Their basal and apical ends
form the internal and external limiting membranes. Processes of the photoreceptor cells
synapse in the outer plexiform layer with processes of cells located in the inner nuclear
layer. The nuclei of the photoreceptors (rods and cones) lie within the outer nuclear
layer. The outer segments of the photoreceptors are loosely embedded within the retinal
pigment epithelium.

Question:

A 73-year-old African American man presents for routine follow-up at a primary eye care clinic. He
notes that he has been diagnosed with glaucoma and his ophthalmologist has started him on a topical
carbonic anhydrase inhibitor (dorzolamide) in order to lower eye pressure by decreasing aqueous
humor production. The site of action of this drug would most likely be:

Answer Choices:

A Trabecular meshwork

B Canal of Schlemm

C Corneal endothelium

D Ciliary body epithelium

E Retinal pigment epithelium

D Ciliary body epithelium

Aqueous humor is produced in the epithelium of the ciliary body, travels through the
pupil into the anterior chamber and drains through the trabecular meshwork into the
canal of Schlemm. A topical drug which decreases aqueous humor production would be
expected to act at the ciliary body epithelium.

The trabecular meshwork and canal of Schlemm are part of the outflow pathway of
aqueous humor and are not involved in aqueous humor production. The corneal
endothelium and retinal pigment epithelium are not involved in aqueous humor
production.

Case #20100:

A 70-year-old male is hospitalized following a ruptured aortic aneurysm. 10 days after


this rupture, physical exam shows a blood pressure of 142/80 mmHg, Hollenhorst
crystals, retinal artery narrowing, livedo on his extremities, and several bluish-purple
colored toes. His urine output in the past day was 600 ml.
Laboratory studies reveal the following
Blood urea nitrogen 40 mg/dl
Serum creatinine 3.5 mg/dl
Urine eosinophils positive
Serum complement 3 reduced
Urinalysis and sediment 1.020, ++ protein, few granular casts, 5 red blood cells/high powered field
Blood glucose 100 mg/dl

Question:

What diagnosis addresses his retinal and renal findings?

Answer Choices:

A Hypertensive retinopathy

B Diabetic retinopathy

C Malignant hypertension

D Cholesterol emboli syndrome

E Diabetic kidney disease

D Cholesterol emboli syndrome

This patient most likely has developed cholesterol emboli syndrome following his aortic
aneurysm rupture. In this syndrome, cholesterol emboli dislodge following a vascular
procedure (i.e. cardiac catheterization) or catastrophe (aortic aneurysm rupture) and
travel to distal arteries where they lodge. Findings may include:

 livedo, a lacy rash surrounding an emboli lodged in peripheral arteries


 Hollenhorst crystals, the retinal arterial plaques seen when atheromatous
particles lodge in retinal arteries
 blue and/or purple toes, due to ischemia distal to affected vessel
 renal failure due to cholesterol emboli to glomerular vessels

Renal failure may present 1-14 days after the vascular event. Retinal emboli are seen in
40-70% of cases of cholesterol emboli syndrome (Adam and Victor, chapter 13). 50% of
cases of cholesterol emboli syndrome cause some degree of renal failure (Brenner,
chapter 33). Elevated blood pressure, non-nephrotic proteinuria, microscopic hematuria,
hypocomplementemia, eosinophiluria, and granular/hyaline casturia may be noted.
Renal prognosis is not good following emboli, and many patients have progressive
declines in renal function. Therapies include dialysis, which is needed in some 40% of
patients who develop renal insufficiency and cholesterol lowering agents.

Hypertensive retinopathy is characterized by narrowing of the retinal arteries, retinal


arterio-venous nicking, retinal blot hemorrhages, microaneurysms, cotton wool spots,
and tortuous retinal vessels. It is typically seen in patients with longstanding
hypertension. Patients are generally asymptomatic. Treating hypertension with
antihypertensive agents is suggested. In this vignette, the diagnosis of hypertensive
retinopathy does not address the findings of retinal emboli (Hollenhorst crystals) and the
hypocomplementemia, hematuria, eosinophiluria, and elevated serum creatinine.

Diabetic retinopathy is a microvascular complication of hyperglycemia. Duration of


diabetes (after puberty), presence and control of hypertension, and presence of renal
disease are all predictors of diabetic retinopathy. Retinal microaneurysms are the
hallmark lesion; retinal hemorrhages, cotton-wool spots, macular edema,
neovascularization, and vitreous hemorrhages may also occur, depending on the
disease severity. Cataracts may also be noted.

If untreated, diabetic retinopathy is a serious risk for visual loss. Treatments include
pan-retinal photocoagulation (laser therapy), tight glycemic control, and vitrectomy for
non-clearing vitreal hemorrhages. This patient doesn't meet criteria for diabetes (such
as fasting sugars >126 mg/dl, abnormal sugars after glucose load, etc.). Diabetic
retinopathy may precede the diagnosis of diabetes in type 2 diabetes.

In malignant hypertension, blood pressure is markedly elevated and end organ effects,
such as renal failure, encephalopathy, visual changes, and angina, are noted. Visual
symptoms may include scotoma, diplopia, and dimmed vision; on retinal exam, retinal
spasm, retinal hemorrhages, cotton-wool spots, and optic disc edema may be noted.
With prolonged papilledema due to malignant hypertension, optic atrophy may occur,
leading to permanent visual changes. Retinal detachments may also occur. The
treatment of malignant hypertension involves carefully and gradually lowering the
patient's blood pressure. In this vignette, the patient's blood pressure was 142/80
mmHg. A blood pressure around 200/100 mmHg or higher would be typical of malignant
hypertension. Although two end-vascular events (retinal and renal) are noted in this
vignette, the retinal findings are more suggestive of cholesterol emboli (Hollenhorst
crystals) than malignant hypertension (papilledema and retinal vascular spasm).

Diabetic kidney disease is a microvascular complication of diabetes. Patients may be


asymptomatic, while microscopic proteinuria and changes in glomerular permeability
are occurring due to hyperglycemia. Diabetic kidney disease is an important cause of
progressive kidney disease and the leading cause of end stage renal disease in the
United States. Renal disease is an excellent predictor of diabetic retinopathy (Yanoff,
chapter 113). Neither diabetic kidney disease nor diabetic retinopathy will explain
the embolic findings (Hollenhorst crystals, ischemic toes) found in this vignette.
Case #20101:

A 49-year-old male with a 2-year history of diet-controlled type 2 diabetes comes to


the emergency room complaining of headache, chest pain, diplopia, and dimmed
vision. His blood pressure is 200/110 mmHg, and his heart rate is 102 beats per
minute. His retinal exam shows retinal artery spasm, retinal hemorrhages, and
papilledema. The remainder of his examination is normal. His diagnostic studies
include:
Electrocardiogram Sinus tachycardia, non-specific ST changes
Troponin Mildly elevated
Creatinine 1.5 mg/dl
Urinalysis 1.020, 7 red blood cells/high powered field, no casts, + protein, no glucose
Blood toxicology Positive for cocaine
Fasting Blood glucose 96 mg/dl
Hemoglobin A1C 5.6%

Question:

What diagnosis explains his renal and retinal findings?

Answer Choices:

A Chronic hypertension

B Diabetic retinopathy

C Malignant hypertension

D Hypertensive retinopathy

E Diabetic kidney disease

C Malignant hypertension

This patient most likely has malignant hypertension due to recent cocaine use. Cocaine
is a powerful stimulant of monoamines into the neural synaptic junction, and it inhibits its
reuptake (Belmaker). Epinephrine and norepinephrine are vasoconstrictors and are
used as pressor support for hypotensive intensive care patients; in this case, it can also
markedly elevate blood pressure after acute cocaine exposure.

Symptoms of malignant hypertension may include visual (scotoma, diplopia, dimmed


vision), cardiac (angina, palpitations), and neurological (headache, encephalopathy or
clouding). Many forms of end organ dysfunction secondary to impaired blood flow may
occur. This patient complained of visual symptoms and had the retinal artery spasm and
papilledema characteristic of malignant hypertension. Prolonged papilledema may lead
to optic atrophy and visual loss, but this is uncommon.

Also noted in this vignette are an elevation in creatinine, microscopic hematuria, and
mild proteinuria. Cocaine may lead to renal infarctions, tubular necrosis, and renal
vascular spasm. Impaired renal blood flow translates into impaired waste clearance and
elevated serum creatinines. Over time, repeated cocaine use may cause progressive
loss in renal function due to the above. Malignant hypertension is treated by gradual
and careful lowering of blood pressure to the normal range of <140/80 mmHg.

Diabetic retinopathy is a microvascular complication of hyperglycemia. Duration of


diabetes (after puberty), presence and control of hypertension, and presence of renal
disease are all predictors of diabetic retinopathy. Retinal microaneurysms are the
hallmark lesion; retinal hemorrhages, cotton-wool spots, macular edema,
neovascularization, and vitreous hemorrhages may also occur, depending on the
disease severity. Cataracts may also be noted.

If untreated, diabetic retinopathy is a serious risk for visual loss. Treatments include
pan-retinal photocoagulation (laser therapy), tight glycemic control, and vitrectomy for
non-clearing vitreal hemorrhages. Although he had retinal hemorhages, which may be
seen in diabetic retinopathy, he did not have the microaneurysms that usually
accompany them, and he did have the vascular spasm and hypertension suggestive of
acute cocaine use. Additionally, no hyperglycemia or glucosuria was noted, and his
hemoglobin A1C was indicative of excellent glycemic control (<7%).

Retinal findings in chronic hypertension may include narrowed and irregular retinal
arteries, arterio-venous nicking, blot hemorrhages, microaneurysms, and tortuous
vessels.

None of the above were noted in this vignette. Retinal exam has a 50% predictive value
for diagnosing systemic (chronic) hypertension. In one series, some 15% of
hypertensive patients had hypertensive retinopathy (Yanoff, chapter 113). Hypertensive
retinopathy is generally asymptomatic, unlike the vignette above, and is treated with
antihypertensive medications.

Diabetic kidney disease is a microvascular complication of diabetes. Patients may be


asymptomatic, while microscopic proteinuria and changes in glomerular permeability
are occurring due to hyperglycemia. Diabetic kidney disease is an important cause of
progressive kidney disease, and the leading cause of end stage renal disease in the
United States. Renal disease is an excellent predictor of diabetic retinopathy (Yanoff,
chapter 113). Neither diabetic kidney disease nor diabetic retinopathy will explain the
retinal artery spasm or markedly elevated blood pressure described in this vignette.

Case #20102:

On your first encounter with a 60-year-old male, you note retinal microaneurysms,
thinned retinal arteries, a blood pressure of 155/85 mmHg, an S4 sound on cardiac
exam, and diminished peripheral pulses. Fasting blood work from a local laboratory
shows:
Serum creatinine 1.3 mg/dl
1.020, pH 6, trace protein, no blood,
Urinalysis
No cells, no glucose, no casts
Blood glucose 100 mg/dl
Hemoglobin A1C 6%

Question:

What is the most likely diagnosis?

Answer Choices:

A Chronic hypertension

B Malignant hypertension

C Diabetic retinopathy

D Diabetic Kidney Disease

E Cholesterol emboli syndrome

A Chronic hypertension

Although we only have one blood pressure reading for him, this patient most likely has
chronic hypertension. In chronic hypertension, the patient's blood pressure will be
chronically elevated. Due to elevated pressure, vascular organs such as the heart,
kidneys, and eyes may become diseased. Left ventricular hypertrophy, the presence of
an S4 on cardiac exam, reduced pulses, and thinning of retinal vessels may be noted.
In the kidneys, reduced blood flow and higher glomerular pressures may lead to
reduced creatinine clearance, elevated serum creatinine, glomerular scarring and
proteinuria. In general, patients whose blood pressure is chronically in the 150-160
mmHg systolic range will have twice the rate of decline in renal function with aging as
do patients with systolic pressures in the 130-140 mmHg range (Brenner, chapter 47).
Chronic hypertension is generally asymptomatic and may be treated with a variety of
agents, including diet and exercise, diuretics, beta blockers, ACE inhibitors, calcium
channel blockers, etc, depending on its duration, severity, and presence of other chronic
diseases such as diabetes and heart disease.

Symptoms of malignant hypertension may include visual (scotoma, diplopia, dimmed


vision), cardiac (angina, palpitations), and neurological (headache, encephalopathy or
clouding). Many forms of end organ dysfunction secondary to impaired blood flow may
occur. This patient had no complaints of visual symptoms or the retinal artery spasm
and papilledema characteristic of malignant hypertension. Prolonged papilledema may
lead to optic atrophy and visual loss, but this is uncommon.

Malignant hypertension is treated by gradual and careful lowering of blood pressure to


the normal range of <140/80 mmHg.

Diabetic retinopathy is a microvascular complication of hyperglycemia. Duration of


diabetes (after puberty), presence and control of hypertension, and presence of renal
disease are all predictors of diabetic retinopathy. Retinal microaneurysms are the
hallmark lesion; retinal hemorrhages, cotton-wool spots, macular edema,
neovascularization, and vitreous hemorrhages may also occur, depending on the
disease severity. Cataracts may also be noted.

If untreated, diabetic retinopathy is a serious risk for visual loss. Treatments include
pan-retinal photocoagulation (laser therapy), tight glycemic control, and vitrectomy for
non-clearing vitreal hemorrhages. Microaneurysms may be noted in both diabetic and
hypertensive retinopathy. Hypertension is common in diabetics, so other findings may
be needed to distinguish the two diseases. The patient described above does not meet
criteria for the diagnosis of diabetes; his fasting sugar is < 126 mg/dl.

Diabetic kidney disease is a microvascular complication of diabetes. Patients may be


asymptomatic, while microscopic proteinuria and changes in glomerular permeability
are occurring due to hyperglycemia. Diabetic kidney disease is an important cause of
progressive kidney disease and the leading cause of end stage renal disease in the
United States. Renal disease is an excellent predictor of diabetic retinopathy (Yanoff,
chapter 113). Tight diabetic glycemic control and blood pressure control are important in
the prevention of diabetic kidney disease. Blockade of the renin-angiotensin system via
ACE inhibitors and other agents are also important in the treatment of diabetic kidney
disease.
In cholesterol emboli syndrome, emboli dislodge following a vascular procedure (i.e.
cardiac catheterization) or catastrophe (aortic aneurysm rupture) and travel to distal
arteries where they lodge. Findings may include:

 livedo, a lacy rash surrounding an emboli lodged in peripheral arteries


 Hollenhorst crystals, the retinal arterial plaques seen when atheromatous
particles lodge in retinal arteries
 blue and/or purple toes, due to ischemia distal to affected vessel
 renal failure due to cholesterol emboli to glomerular vessels

Renal failure may present 1-14 days after the vascular event. Retinal emboli are seen in
40-70% of cases of cholesterol emboli syndrome (Adam and Victor, chapter 13). 50% of
cases of cholesterol emboli syndrome cause some degree of renal failure (Brenner,
chapter 33). Elevated blood pressure, non-nephrotic proteinuria, microscopic hematuria,
hypocomplementemia, eosinophiluria, and granular/hyaline casturia may be noted.

Renal prognosis is not good following emboli, and many patients have progressive
declines in renal function. Therapies include dialysis, which is needed in some 40% of
patients who develop renal insufficiency, and cholesterol-lowering agents.

Question:

Multiple microaneurysms and hemorrhages are noted on the retinal exam of a 62-year-old female with
a 20-year history of diet-controlled type 2 diabetes. She has no other medical problems and no other
known diabetic complications. How should you target her preventive care, given her retinal findings?

Answer Choices:

A Evaluate blood pressure and blood sugar regularly

B Evaluate renal function and blood pressure regularly

C Prescribe atorvastatin and evaluate renal function regularly

D Prescribe aspirin and evaluate blood pressure

E Regular retinal specialist, renal function, hypertension, and blood sugar


checks
E Regular retinal specialist, renal function, hypertension, and blood sugar checks

This patient has evidence of diabetic retinopathy, a microvascular complication of


diabetes, and should have periodic evaluations of her blood pressure, blood sugar,
hemoglobin A1C, and renal function, in addition to an evaluation by a retinal specialist.

In elderly diabetics, there are significant associations between microaneurysms, retinal


exudates, and declining renal function (Edwards). The retinal microvasculature can act
as a non-invasive view of the renal microvasculature and therefore show microvascular
disease that may otherwise require a renal biopsy to see.

Findings in diabetic retinopathy may include microaneurysms (which may also be seen
in hypertensive retinopathy), retinal hemorrhages, cotton-wool spots, macular edema,
exudates (which are hardened areas where edema leaked from aneurysms and
persisted), neovascularization, and vitreal hemorrhages.

The post-pubertal duration of diabetes, the presence of hypertension, and the presence
of renal disease are all predictors of retinopathy (Klein).

If untreated, advanced retinopathy is a risk for visual loss. Treatments include


aggressive blood pressure and glycemic control, pan-retinal photocoagulation (laser
therapy). Vitrectomy is required if persistent vitreal hemorrhages exist (Yanoff).
Antiplatelet agents are not beneficial in the treatment of retinopathy (Yanoff).

In the above vignette, since vascular disease is already noted in the retina, it is prudent
to evaluate regularly for renal disease. This may be done by periodically checking urine
protein (or albumin) to creatinine ratios and calculating glomerular filtration rates from
serum creatinine. Since blood pressure may become elevated in renal insufficiency and
is a risk factor for both retinal disease and renal disease progression, evaluating it
regularly is prudent in the preventive care of diabetics. Atorvastatin is prescribed only if
lipid levels are high, which is not indicated in this vignette.

Case #20103:

A 23-year-old female complains of headache, blurred vision, nausea, and shortness


of breath. She has no known medical history and takes no prescribed or illicit drugs.
She recently had pharyngitis. Her blood pressure is 210/140 mmHg. She has bilateral
papilledema and retinal artery spasms, a normal cardiopulmonary exam, a normal
neurological exam, intact peripheral pulses, and a normal abdominal exam. Her
diagnostic studies include:
15 red cells/high powered field, no glucose, no white cells, 3 red cell casts/high powered field,
Urinalysis
++ protein
Urine pregnancy test negative
Toxicology Drug screen negative
CT head normal
Serum Creatinine 1.5 mg/dl
Erythrocyte sedimentation
normal
rate
Total white cell count 11x10.3 cells/cmm
Hemoglobin 11.5 g/dl
Lactate Dehyrogenase normal
Chest x-ray normal

Question:

How to prevent loss of renal and retinal function?

Answer Choices:

A Intravenous nicardipine, immediate and long-term pressure goal of 160/80


mmHg

B Intravenous nicardipine, immediate and long-term pressure goal of 130/80


mmHg

C Intravenous nicardipine, reduce pressure 10% now & gradually to 130/80


mmHg

D Intravenous enalapril to reach blood pressure goal of 130/80 mmHg in 1 hour

E Oral clonidine to reach blood pressure goal of 130/80 mmHg in 2 hours

C Intravenous nicardipine, reduce pressure 10% now & gradually to 130/80 mmHg

This patient with hematuria has a hypertensive emergency, characterized by severely


elevated systolic and diastolic blood pressures, evidence of end organ dysfunction, and
should be treated with intravenous nicardipine to reach blood pressure reduction
of 10% in the first hour and an additional 15% in the next 2-3 hours. Severely
elevated pressure without end organ damage is termed hypertensive urgency.

Reducing her pressure to 130/80 mmhg in 2 hours or less is too fast as described below
.
Many causes of hypertensive crisis, including hypertensive emergency and
hypertensive urgency, include:

 Head injury or stroke


 Renovascular disease
 Glomerulonephritis
 Discontinuation of antihypertensives
 Medications (cocaine, amphetamines, cold medications)
 Vasculitis
 Microangiopathic hemolytic anemia
 Pregnancy
 Aortic dissection

Except in cases of hypertensive emergencies related to aortic dissections, the goal of


blood pressure management in hypertensive emergencies is to lower blood pressure by
10% in the first hour and an additional 15% in the next 2-3 hours (Feldstein, Varon).
Abrupt lowering of blood pressure to the normal range may compromise cerebral blood
flow and in cases of stroke may be dangerous. Preventing end organ damage is an
important goal of treatment. Eventually, blood pressure should be brought to the normal
range, i.e. <140/80 mmHg for otherwise healthy patients, <130/80 mmHg for diabetic
patients, etc.

Since we know that chronically elevated blood pressures may lead to renal disease, a
long-term blood pressure goal of 160/80 mmHg is not advised.

Patients with sustained blood pressures at this level have twice the rate of decline in
renal function as do patients with normal blood pressures (Brenner, chapter 47).

Although patients without end organ dysfunction (hypertensive urgency) may be treated
with oral antihypertensives, this is not recommended in patients with hypertensive
emergencies. Clonidine may cause hypertensive rebound if discontinued.

This patient may benefit from intravenous treatment with nicardipine or fenoldopam, two
agents used for the treatment of hypertensive emergencies related to acute renal failure
(Varon). Scleroderma renal crisis is a well-recognized form of severe hypertension +
acute renal failure and is treated with angiotensin converting enzyme (ACE) inhibitors;
here, although glomerulonephritis is suspected given the red blood cell casts and
elevated serum creatinine, there is no dermatological or rhematological evidence of
scleroderma. ACE inhibitors may increase creatinine levels, and creatinine levels should
be closely monitored; this need to monitor does not prohibit their use. This patient may
have post-streptococcal glomerulonephritis accompanied by severe hypertension.

In malignant hypertension, neurophthalmologic symptoms may include headache,


blurred vision, and diplopia. Papilledema and retinal artery spasm may be noted. If left
untreated, optic atrophy may occur, hence blood pressure reduction is advised, as
described above (Yanoff).
SECTION- EAR

Question:

A 22-year-old female joined her boyfriend for a ride aboard a sailboat on an open lake. Within minutes,
she began to experience dizziness without nausea, but was unable to walk without help. No alcoholic
beverages were consumed by the patient, nor did she report using any medications or illicit drugs. After
returning to shore, and following several hours of lying down, she reported a complete recovery without
any residual dizziness, nor any hearing impairment. You conclude that the dizziness was probably due
to:

Answer Choices:

A Excessive stimulation of the Organ of Corti

B Excessive stimulation of the utricle

C Impaired stimulation of the Organ of Corti

D Impaired stimulation of the utricle

E Excessive endolymph in the endolymphatic sac

B Excessive stimulation of the utricle

Motion sickness as experienced in conjunction with the movement of a boat in water


may be experienced as dizziness in some individuals due to excessive stimulation of
the utricle. The stimulation of the Organ of Corti would produce the effect of sound
perception. Excessive endolymph in the endolymphatic sac may result in distension of
the membranous labyrinth and result in dizziness associated with ringing (tinnitus)
consistent with Méinère's disease.

Question:
The tunnel of Corti is formed by which of the following cells?

Answer Choices:

A Inner hair cells

B Outer hair cells

C Böttcher cells

D Inner/outer phalangeal cells

E Inner/outer pilar cells

E Inner/outer pilar cells

The stereocilia of the outer hair cells, and inner hair cells, which are inserted into the
tectorial membrane connect the basilar membrane to the tectorial membrane. The outer
and inner phalangeal cells serve as supportive cells to the outer and inner hair cells,
respectively, and prevents the hair cells from directly touching the basilar membrane.
The pilar cells are supportive cells forming the tunnel of Corti intermediate to the outer
and inner hair cells. Böttcher cells rest on the lateral aspect of the basilar membrane,
which serves to support the lateral aspect of the Organ of Corti.

Question:

Into which one of the following structures are the stereocilia of the hair cells of the semicircular canals
embedded?

Answer Choices:
A Cupula

B Otolithic membrane

C Organ of Corti

D Tectorial membrane

E Basilar membrane

A Cupula

The otolithic membrane is the gelatinous membrane overlying either the macula utricle
or macula saccule. The outer surface of this membrane contains particles of calcium
carbonate, called otolith or otoconia. The stereocilia of the hair cells are bent by the
action of gravity on the otoconia. The cupula also is a gelatinous structure attached to
hair cells of each crista of the semicircular canals. Movement of the endolymph
contained within the ducts sways the cupula which project into the endolymph and bend
the stereocilia to initiate a neural impulse.

The Organ of Corti is the structure within the cochlear duct specialized to transduce the
mechanical vibrations into neural impulses. The Organ of Corti rests upon the basilar
membrane and the stereocilia of the hair cells of the Organ of Corti are embedded
within the tectorial membrane.

Question:

Into which one of the following structures are the stereocilia of the Type I and Type II hair cells of the
saccule embedded?

Answer Choices:

A Cupula

B Otolithic membrane
C Organ of Corti

D Tectorial membrane

E Basilar membrane

B Otolithic membrane

The otolithic membrane is the gelatinous membrane overlying either the macula utricle
or macula saccule. The outer surface of this membrane contains particles of calcium
carbonate, called otolith or otoconia. The stereocilia of the hair cells are bent by the
action of gravity on the otoconia. The cupula also is a gelatinous structure attached to
hair cells of each crista of the semicircular canals. Movement of the endolymph
contained within the ducts sways the cupula which project into the endolymph and bend
the stereocilia to initiate a neural impulse. The Organ of Corti is the structure within the
cochlear duct specialized to transduce the mechanical vibrations into neural impulses.
The Organ of Corti rests upon the basilar membrane and the stereocilia of the hair cells
of the Organ of Corti are embedded within the tectorial membrane.

Question:

The specialized glands found in the external auditory meatus secrete what substance?

Answer Choices:

A Sebum

B Mucin

C Cerumen

D Endolymph

E Perilymph
C Cerumen

The ear canal or external auditory meatus is a tubular connection continuous with the
base of the auricle, leading from the surface of the head to the tympanic membrane, or
eardrum. Specialized sweat glands, called ceruminous glands line the outer lateral
portion of the canal. The apocrine secretion of the ceruminous glands combines with the
secretion of sebaceous glands in the external auditory meatus to form cerumen
(earwax). This functions to filter foreign particles from entry into the meatus, as well as
lubricate the meatal hairs. Excessive accumulation of cerumen may impede sound
waves and result in conductive hearing loss. Sebum is the oily substance secreted by
the sebaceous glands. Mucin is a lipid-rich product of goblet cells. Endolymph is found
in the membranous labyrinth and is ionically similar to intracellular fluid (high
potassium/low sodium). Perilymph, found in the bony labyrinth, is ionically similar to
extracellular fluid. Neither endolymph nor perilymph are produced in the external
auditory meatus.

Question:

The oval-shaped portion of the ear that projects from the lateral surface of the head that functions to
collect and funnel sound waves is called the

Answer Choices:

A Auricle

B External auditory meatus

C Tympanic membrane

D Oval window

E Round window
A Auricle

The oval-shaped portion of the ear that projects from the lateral surface of the head that
functions to collect and funnel sound waves is called the auricle, or pinna. It is
supported by an internal core of elastic cartilage and covered by thin skin. Sweat
glands, sebaceous glands and hair follicles cover the surface. The ear canal or external
auditory meatus is a tubular connection continuous with the base of the auricle, leading
from the surface of the head to the tympanic membrane, or eardrum. Specialized sweat
glands, called ceruminous glands line the outer lateral portion of the canal. Both the
oval and round windows are structures found in the middle ear where sound waves are
converted into mechanical vibrations.

Question:

Position of the head is sensed by which one of the following structures

Answer Choices:

A Organ of Corti

B Macula of the utricle

C Cristae ampullaris of the semicircular ducts

D Spiral ganglion

B Macula of the utricle

The macula of the saccule and utricle both function as sensors of gravity, and as
such, participate in the sense of head position. The macula utriculi is oriented in a
horizontal plane, whereas the macula sacculi is oriented in a vertical plane when a
person is in the standing position. Calcium carbonate and protein particles, the
otoconia, on the surface of the otolithic membrane within the macula, will move the
membrane to stimulate stereocilia of the hair cells embedded within the membrane.
Positional changes in the head result in the action of gravity on these "weighted"
membranes causing transduction of the movement (positional change) into neural
signals.

The Organ of Corti and spiral ganglion are involved in the transduction of mechanical
signals into neural signals associated with audition. The cristae ampullaris within each
of the semicircular ducts transduce angular movement (acceleration) into neural signals.

Question:

The cochlear duct is separated from the scala vestibuli by which one of the labeled structures in the
attached photomicroscopic image?

Answer Choices:

A A

B B

C C

D D

E E

Image(s) / Chart(s):
Click image to view full size. Click open image to close. Click and hold open image to move.
A A

The inner ear contains a conically shaped helix, the cochlea, within the bony labyrinth of
the temporal bone. The cochlear duct divides the cochlear canal into three
compartments. The endolymphatic fluid filled cochlear duct is the middle compartment,
called the scala media (#2), whereas the compartments of the cochlear canal filled with
perilymph include the scala vestibuli (#1), and the scala tympani (#3).

Reissner's membrane (a) separates the scala vestibuli from the scala media. A
specialized pseudostratified epithelium on the lateral wall of the scala media is the stria
vascularis (c), which may produce the endolymph. The specialized structure for auditory
sensation is a complex called the Organ of Corti, which rests on the floor of the scala
media called the basilar membrane (e).

The tectorial membrane (b) extends from the spiral limbus to lie over the hair cells of the
organ of corti, into which the kinocilia of the hair cells are imbedded. Movement of the
stereocilia in these hair cells initiates transduction of the mechanical motion into
membrane potentials conveyed to the nervous system by way of the vestibulocochlear
nerve.

The bipolar nerve cell bodies reside in the spiral ganglion (d).
Question:

Which one of the following structures is not considered a component of the membranous labyrinth?

Answer Choices:

A Semicircular ducts

B Cochlear duct

C Saccule

D Utricle

E Vestibule

E Vestibule

The inner ear consists of a bony labyrinth consisting of three connected spaces within
the temporal bone. The spaces are the semicircular canals, the vestibule and cochlea.
The vestibule is the central portion of the bony labyrinth containing the utricle and
saccule, parts of the membranous labyrinth. The semicircular ducts and cochlear duct
also are parts of the membranous labyrinth.

Question:

Kinocilia from the outer hair cells are embedded into which one of the labeled structures in the attached
photomicroscopic image?

Answer Choices:
A A

B B

C C

D D

E E

Image(s) / Chart(s):
Click image to view full size. Click open image to close. Click and hold open image to move.

B B

The inner ear contains a conically shaped helix, the cochlea, within the bony labyrinth of
the temporal bone. The cochlear duct divides the cochlear canal into three
compartments. The endolymphatic fluid filled cochlear duct is the middle compartment,
called the scala media (#2), whereas the compartments of the cochlear canal filled with
perilymph include the scala vestibuli (#1), and the scala tympani (#3).
Reissner's membrane (a) separates the scala vestibuli from the scala media. A
specialized pseudostratified epithelium on the lateral wall of the scala media is the stria
vascularis (c), which may produce the endolymph. The specialized structure for auditory
sensation is a complex called the Organ of Corti, which rests on the floor of the scala
media called the basilar membrane (e).

The tectorial membrane (b) extends from the spiral limbus to lie over the hair cells of
the organ of corti, into which the kinocilia of the hair cells are imbedded. Movement of
the stereocilia in these hair cells initiates transduction of the mechanical motion into
membrane potentials conveyed to the nervous system by way of the vestibulocochlear
nerve.

The bipolar nerve cell bodies reside in the spiral ganglion (d).

Question:

Perilymph is found within which one of the following labeled structures in the attached
photomicroscopic image

Answer Choices:

A C

B B

C D

D 2

E 3

Image(s) / Chart(s):
Click image to view full size. Click open image to close. Click and hold open image to move.
E 3

The inner ear contains a conically shaped helix, the cochlea, within the bony labyrinth of
the temporal bone. The cochlear duct divides the cochlear canal into three
compartments. The endolymphatic fluid filled cochlear duct is the middle compartment,
called the scala media, (2), whereas the compartments of the cochlear canal filled with
perilymph include the scala vestibuli, (1), and the scala tympani,(3) .

Reissner's membrane, (a), separates the scala vestibuli from the scala media. A
specialized pseudostratified epithelium on the lateral wall of the scala media, is the stria
vascularis (c), which may produce the endolymph. The specialized structure for auditory
sensation is a complex called the Organ of Corti, which rests on the floor of the scala
media called the basilar membrane, (e).

The tectorial membrane, (b), extends from the spiral limbus, to lie over the hair cells of
the organ of corti, into which the kinocilia of the hair cells are imbedded. Movement of
the stereocilia in these hair cells initiates transduction of the mechanical motion into
membrane potentials conveyed to the nervous system, by way of the vestibulocochlear
nerve.

The bipolar nerve cell bodies reside in the spiral ganglion, (d).

Question:

How is the structure as indicated by the letter 'e' in the attached photomicroscopic image is best
identified?
Answer Choices:

A Reissner's membrane.

B Basilar membrane.

C Tectorial membrane.

D Stria vascularis.

E Spiral ganglion.

Image(s) / Chart(s):
Click image to view full size. Click open image to close. Click and hold open image to move.

B Basilar membrane.
The inner ear contains a conically shaped helix, the cochlea, within the bony labyrinth of
the temporal bone. The cochlear duct divides the cochlear canal into 3 compartments.
The endolymphatic fluid filled cochlear duct is the middle compartment, called the scala
media (2), whereas the compartments of the cochlear canal filled with perilymph include
the scala vestibuli (1) and the scala tympani (3).

Reissner's membrane (a) separates the scala vestibuli from the scala media. A
specialized pseudostratified epithelium on the lateral wall of the scala media is the stria
vascularis (c), which may produce the endolymph. The specialized structure for auditory
sensation is a complex called the Organ of Corti, which rests on the floor of the scala
media called the basilar membrane (e).

The tectorial membrane (b) extends from the spiral limbus to lie over the hair cells of the
organ of corti into which the kinocilia of the hair cells are imbedded. Movement of the
stereocilia in these hair cells initiates transduction of the mechanical motion into
membrane potentials conveyed to the nervous system by way of the vestibulocochlear
nerve.

The bipolar nerve cell bodies reside in the spiral ganglion (d).

Question:

The structure as indicated by the letter 'd' in the attached photomicroscopic image is best identified as

Answer Choices:

A Reissner's membrane

B Basilar membrane

C Tectorial membrane

D Stria vascularis

E Spiral ganglion
Image(s) / Chart(s):
Click image to view full size. Click open image to close. Click and hold open image to move.

E Spiral ganglion

The inner ear contains a conically shaped helix, the cochlea, within the bony labyrinth of
the temporal bone. The cochlear duct divides the cochlear canal into three
compartments. The endolymphatic fluid filled cochlear duct is the middle compartment,
called the scala media (#2), whereas the compartments of the cochlear canal filled with
perilymph include the scala vestibuli (#1), and the scala tympani (#3).

Reissner's membrane (a) separates the scala vestibuli from the scala media. A
specialized pseudostratified epithelium on the lateral wall of the scala media is the stria
vascularis (c), which may produce the endolymph. The specialized structure for auditory
sensation is a complex called the Organ of Corti, which rests on the floor of the scala
media called the basilar membrane (e). The tectorial membrane (b) extends from the
spiral limbus to lie over the hair cells of the organ of corti, into which the kinocilia of the
hair cells are imbedded. Movement of the stereocilia in these hair cells initiates
transduction of the mechanical motion into membrane potentials conveyed to the
nervous system by way of the vestibulocochlear nerve.

The bipolar nerve cell bodies reside in the spiral ganglion (d).
Question:

The structure as indicated by the letter 'b' in the attached photomicroscopic image is best identified as

Answer Choices:

A Reissner's membrane

B Basilar membrane

C Tectorial membrane

D Stria vascularis

E Scala media

Image(s) / Chart(s):
Click image to view full size. Click open image to close. Click and hold open image to move.
C Tectorial membrane

The inner ear contains a conically shaped helix, the cochlea, within the bony labyrinth of
the temporal bone. The cochlear duct divides the cochlear canal into three
compartments. The endolymphatic fluid filled cochlear duct is the middle compartment,
called the scala media (#2), whereas the compartments of the cochlear canal filled with
perilymph include the scala vestibuli (#1), and the scala tympani (#3).

Reissner's membrane (a) separates the scala vestibuli from the scala media. A
specialized pseudostratified epithelium on the lateral wall of the scala media is the stria
vascularis (c), which may produce the endolymph. The specialized structure for auditory
sensation is a complex called the Organ of Corti, which rests on the floor of the scala
media called the basilar membrane (e).

The tectorial membrane (b) extends from the spiral limbus to lie over the hair cells of
the organ of corti, into which the kinocilia of the hair cells are imbedded. Movement of
the stereocilia in these hair cells initiates transduction of the mechanical motion into
membrane potentials conveyed to the nervous system by way of the vestibulocochlear
nerve.

The bipolar nerve cell bodies reside in the spiral ganglion (d).

Question:

The structure as indicated by the number '3' in the attached photomicroscopic image is best identified
as

Answer Choices:

A Scala vestibuli

B Basilar membrane

C Scala tympani

D Stria vascularis
E Scala media

Image(s) / Chart(s):
Click image to view full size. Click open image to close. Click and hold open image to move.

C Scala tympani

The inner ear contains a conically shaped helix, the cochlea, within the bony labyrinth of
the temporal bone. The cochlear duct divides the cochlear canal into three
compartments. The endolymphatic fluid filled cochlear duct is the middle compartment,
called the scala media (2), whereas the compartments of the cochlear canal filled with
perilymph include the scala vestibuli (1), and the scala tympani, (3) .

Reissner's membrane (a) separates the scala vestibuli from the scala media. A
specialized pseudostratified epithelium on the lateral wall of the scala media, is the stria
vascularis (c), which may produce the endolymph. The specialized structure for auditory
sensation is a complex called the Organ of Corti, which rests on the floor of the scala
media, called the basilar membrane (e).

The tectorial membrane (b), extends from the spiral limbus to lie over the hair cells of
the organ of corti, into which the kinocilia of the hair cells are imbedded. Movement of
the stereocilia in these hair cells initiates transduction of the mechanical motion into
membrane potentials conveyed to the nervous system by way of the vestibulocochlear
nerve.
The bipolar nerve cell bodies reside in the spiral ganglion (d).

Question:

The structure as indicated by the number '2' in the attached photomicroscopic image is best identified
as

Answer Choices:

A Scala vestibuli

B Basilar membrane

C Sala tympani

D Stria vascularis

E Scala media

Image(s) / Chart(s):
Click image to view full size. Click open image to close. Click and hold open image to move.
E Scala media

The inner ear contains a conically shaped helix, the cochlea, within the bony labyrinth of
the temporal bone. The cochlear duct divide the cochlear canal into three
compartments. The endolymphatic fluid filled cochlear duct is the middle compartment,
called the scala media (#2), whereas the compartments of the cochlear canal filled with
perilymph include the scala vestibuli (#1), and the scala tympani (#3).

Reissner's membrane (a) separates the scala vestibuli from the scala media. A
specialized pseudostratified epithelium on the lateral wall of the scala media is the stria
vascularis (c), which may produce the endolymph. The specialized structure for auditory
sensation is a complex called the Organ of Corti, which rests on the floor of the scala
media called the basilar membrane (e). The tectorial membrane (b) extends from the
spiral limbus to lie over the hair cells of the organ of corti, into which the kinocilia of the
hair cells are imbedded.

Movement of the stereocilia in these hair cells initiates transduction of the mechanical
motion into membrane potentials conveyed to the nervous system by way of the
vestibulocochlear nerve. The bipolar nerve cell bodies reside in the spiral ganglion (d).

Question:

Which one of the following components is lacking inside the endolymphatic sac?
Answer Choices:

A Endolymph

B Electron-dense columnar cells

C Electron-lucent columnar cells

D Macrophages

E Reissner's membrane

E Reissner's membrane

Reissner's (vestibular) membrane is contained inside the spiral organ of Corti. It helps
maintain the high ionic gradients between the perilymph in the scala vestibuli and the
endolymph in the cochlear duct. The endolymphatic sac has an epithelial lining
containing electron-dense columnar cells, which have an irregularly shaped nucleus,
and electron-lucent columnar cells, which possess long microvilli, many pinocytic
vesicles, and vacuoles. It also contains phagocytic cells (macrophages, neutrophils) in
its lumen. It may function in resorption of the endolymph.

Question:

In which one of the following ganglia are the location of the cell bodies of the neurons of the cochlear
nerve fibers

Answer Choices:

A Scarpa's ganglion
B Dorsal root ganglion

C Superior cervical ganglion

D Spiral ganglion

E Auerbach's ganglion

D Spiral ganglion

The bipolar cell bodies of the neurons of the cochlear nerve fibers are located in the
spiral ganglion of the modiolus. Scarpa's ganglion are bipolar neurons of the vestibular
division of cranial nerve VIII. The dorsal root ganglia are the location of the bipolar cell
bodies of afferent fibers from the somatic nervous system. The superior cervical
ganglion is the location of the cell bodies of sympathetic nerve fibers entering the
cranium and Auerbach's ganglion is the location of cell bodies associated with the
parasympathetic nervous system whose nerve fibers innervate muscle within the
muscularis externa of the GI system.

Question:

Which one of the following structures in the inner ear is supplied by the posterior vestibular artery?

Answer Choices:

A Cristae ampullaris of the superior semicircular duct

B Macula of the utricle

C Macula of the saccule

D Basal one-fourth of the cochlea


E Cristae ampullaris of the lateral semicircular duct

C Macula of the saccule

The posterior vestibular artery is a branch of the vestibulocochlear artery, which comes
from the common cochlear artery. The cristae ampullaris of the superior and lateral
semicircular ducts are supplied by blood from the anterior vestibular artery. The anterior
vestibular artery also supplies the macula of the utricle. The macula of the saccule is
supplied by the posterior vestibular artery, but also may derive a small amount from the
anterior vestibular artery. The basal one-fourth of the cochlea is supplied by the
cochlear branch of the cochlear artery.

Question:

Which one of the following structures connects the middle ear to the nasopharynx?

Answer Choices:

A External auditory meatus

B Auricle

C Cochlea

D Vestibule

E Eustachian tube

E Eustachian tube

The Eustachian (auditory) tube connects the middle ear cavity with the nasopharynx.
The walls of the pharyngeal portion of this somewhat tubular channel are normally
flattened together, and may be separated by yawning or swallowing. It is lined with
ciliated pseudostratified columnar epithelium, which is continuous with the nasopharynx.
The oval-shaped portion of the ear that projects from the lateral surface of the head that
functions to collect and funnel sound waves is called the auricle, or pinna. It is
supported by an internal core of elastic cartilage and covered by thin skin. Sweat
glands, sebaceous glands and hair follicles cover the surface.

The ear canal or external auditory meatus is a tubular connection continuous with the
base of the auricle, leading from the surface of the head to the tympanic membrane, or
eardrum. The vestibule is a space within the inner ear containing membranous labyrinth
structures, the utricle and saccule. The cochlea also is found within the inner ear,
connected to the vestibule and makes a 2 3/4 spiral turn towards its apex. The cochlear
duct subserves audition.

Question:

The structure as indicated by the letter 'c' in the attached photomicroscopic image is best identified as

Answer Choices:

A Reissner's membrane

B Basilar membrane

C Tectorial membrane

D Stria vascularis

E Spiral ganglion

Image(s) / Chart(s):
Click image to view full size. Click open image to close. Click and hold open image to move.
D Stria vascularis

The inner ear contains a conically shaped helix, the cochlea, within the bony labyrinth of
the temporal bone. The cochlear duct divides the cochlear canal into three
compartments. The endolymphatic fluid filled cochlear duct is the middle compartment,
called the scala media (#2), whereas the compartments of the cochlear canal filled with
perilymph include the scala vestibuli (#1), and the scala tympani (#3).

Reissner's membrane (a) separates the scala vestibuli from the scala media. A
specialized pseudostratified epithelium on the lateral wall of the scala media is the stria
vascularis (c), which may produce the endolymph. The specialized structure for
auditory sensation is a complex called the Organ of Corti, which rests on the floor of the
scala media called the basilar membrane (e).

The tectorial membrane (b) extends from the spiral limbus to lie over the hair cells of the
organ of corti, into which the kinocilia of the hair cells are imbedded. Movement of the
stereocilia in these hair cells initiates transduction of the mechanical motion into
membrane potentials conveyed to the nervous system by way of the vestibulocochlear
nerve.

The bipolar nerve cell bodies reside in the spiral ganglion (d).

Question:
The structure as indicated by the letter 'a' in the attached photomicroscopic image is best identified as

Answer Choices:

A Reissner's membrane

B Basilar membrane

C Scala tympani

D Stria vascularis

E Scala media

Image(s) / Chart(s):
Click image to view full size. Click open image to close. Click and hold open image to move.

A Reissner's membrane
The inner ear contains a conically shaped helix, the cochlea, within the bony labyrinth of
the temporal bone. The cochlear duct divides the cochlear canal into three
compartments. The endolymphatic fluid filled cochlear duct is the middle compartment,
called the scala media (#2), whereas the compartments of the cochlear canal filled with
perilymph include the scala vestibuli (#1), and the scala tympani (#3).

Reissner's membrane (a) separates the scala vestibuli from the scala media. A
specialized pseudostratified epithelium on the lateral wall of the scala media is the stria
vascularis (c), which may produce the endolymph. The specialized structure for auditory
sensation is a complex called the Organ of Corti, which rests on the floor of the scala
media called the basilar membrane (e). The tectorial membrane (b) extends from the
spiral limbus to lie over the hair cells of the organ of corti, into which the kinocilia of the
hair cells are imbedded. Movement of the stereocilia in these hair cells initiates
transduction of the mechanical motion into membrane potentials conveyed to the
nervous system by way of the vestibulocochlear nerve.

The bipolar nerve cell bodies reside in the spiral ganglion (d).

Question:

The bony ossicles of the middle ear cavity are arranged in a series bridging the tympanic cavity
beginning at the tympanic membrane and ending at the

Answer Choices:

A Round window

B Oval window

C Helicotrema

D Endolymphatic duct

E Cochlear window
B Oval window
Image(s) / Chart(s):
Click image to view full size. Click open image to close. Click and hold open image to
move.

The bony ossicles of the middle ear cavity articulate in a series from the tympanic
membrane to the oval (vestibular) window. The round (cochlear) window connects
middle ear to scala tympani. The helicotrema connects the scala vestibuli to the scala
tympani. The endolymphatic duct begins at utricle and saccule and ends in the
expanded endolymphatic sac.

See the attached Figure: Gross anatomy of the middle ear. Semicircular canals (a);
Subdural space (b); Perilymphatic duct (c); Saccule (d); Utricle (e); Mastoid cavities (f);
External Auditory meatus (g); auditory tube (h); Cochlear duct (k).

Question:

The structure as indicated by the number '1' in the attached photomicroscopic image is best identified
as

Answer Choices:

A Scala vestibuli

B Basilar membrane

C Scala tympani

D Stria vascularis

E Scala media

Image(s) / Chart(s):
Click image to view full size. Click open image to close. Click and hold open image to move.

A Scala vestibuli
The inner ear contains a conically shaped helix, the cochlea, within the bony labyrinth of
the temporal bone. The cochlear duct divides the cochlear canal into three
compartments. The endolymphatic fluid filled cochlear duct is the middle compartment,
called the scala media (#2), whereas the compartments of the cochlear canal filled with
perilymph include the scala vestibuli (#1), and the scala tympani (#3).

Reissner's membrane (a) separates the scala vestibuli from the scala media. A
specialized pseudostratified epithelium on the lateral wall of the scala media is the stria
vascularis (c), which may produce the endolymph. The specialized structure for auditory
sensation is a complex called the Organ of Corti, which rests on the floor of the scala
media called the basilar membrane (e). The tectorial membrane (b) extends from the
spiral limbus to lie over the hair cells of the organ of corti, into which the kinocilia of the
hair cells are imbedded.

Movement of the stereocilia in these hair cells initiates transduction of the mechanical
motion into membrane potentials conveyed to the nervous system by way of the
vestibulocochlear nerve. The bipolar nerve cell bodies reside in the spiral ganglion (d).

Question:

Auditory tubes communicate anteriorly with the pharynx, and posteriorly with the

Answer Choices:

A Round window

B Oval window

C Helicotrema

D Mastoid process of temporal

E Endolymphatic sac
D Mastoid process of temporal

The eustachian (auditory) tubes communicate anteriorly with pharynx and posteriorly
with the air-filled cavities of the mastoid process of the temporal bone. The round
(cochlear) window connects middle ear to scala tympani. The oval (vestibular) window
connects with the stapes. The helicotrema connects the scala vestibuli to the scala
tympani. The endolymphatic duct begins at utricle and saccule and ends in the
expanded endolymphatic sac.

Question:

Which of the following statements concerning the bony labyrinth is correct?

Answer Choices:

A It houses the membranous labyrinth

B It includes the saccule

C It is filled with endolymph

D It includes the utricle

E A part of it is lined by a simple squamous epithelium

A It houses the membranous labyrinth

The bony labyrinth houses the membranous labyrinth. It contains perilymph. The
membranous labyrinth includes saccule and utricle, which are lined by a simple
squamous epithelium. It is filled with endolymph.
Question:

Endolymph is found within which one of the labeled structures in the attached photomicroscopic image

Answer Choices:

A C

B B

C 1

D 2

E 3

Image(s) / Chart(s):
Click image to view full size. Click open image to close. Click and hold open image to move.
D 2

The inner ear contains a conically shaped helix, the cochlea, within the bony labyrinth of
the temporal bone. The cochlear duct divides the cochlear canal into three
compartments. The endolymphatic fluid filled cochlear duct is the middle compartment,
called the scala media (#2), whereas the compartments of the cochlear canal filled with
perilymph include the scala vestibuli (#1), and the scala tympani (#3).

Reissner's membrane (a) separates the scala vestibuli from the scala media. A
specialized pseudostratified epithelium on the lateral wall of the scala media is the stria
vascularis (c), which may produce the endolymph. The specialized structure for auditory
sensation is a complex called the Organ of Corti, which rests on the floor of the scala
media called the basilar membrane (e).

The tectorial membrane (b) extends from the spiral limbus to lie over the hair cells of the
organ of corti, into which the kinocilia of the hair cells are imbedded. Movement of the
stereocilia in these hair cells initiates transduction of the mechanical motion into
membrane potentials conveyed to the nervous system by way of the vestibulocochlear
nerve.

The bipolar nerve cell bodies reside in the spiral ganglion (d).

Question:
Formation of the endolymph would be the function of which one of the labeled structures in the
attached photomicroscopic image?

Answer Choices:

A A

B B

C C

D D

E E

Image(s) / Chart(s):
Click image to view full size. Click open image to close. Click and hold open image to move.

C C
The inner ear contains a conically shaped helix, the cochlea, within the bony labyrinth of
the temporal bone. The cochlear duct divides the cochlear canal into three
compartments. The endolymphatic fluid filled cochlear duct is the middle compartment,
called the scala media (#2), whereas the compartments of the cochlear canal filled with
perilymph include the scala vestibuli (#1), and the scala tympani (#3).

Reissner's membrane (a) separates the scala vestibuli from the scala media. A
specialized pseudostratified epithelium on the lateral wall of the scala media is the
striavascularis (c), which may produce the endolymph. The specialized structure for
auditory sensation is a complex called the Organ of Corti, which rests on the floor of the
scala media called the basilar membrane (e).

The tectorial membrane (b) extends from the spiral limbus to lie over the hair cells of the
organ of corti, into which the kinocilia of the hair cells are imbedded. Movement of the
stereocilia in these hair cells initiates transduction of the mechanical motion into
membrane potentials conveyed to the nervous system by way of the vestibulocochlear
nerve.

The bipolar nerve cell bodies reside in the spiral ganglion (d).

Question:

A 66-year-old male presents to the clinic with a complaint of not being able to hear the beeping of his
microwave oven when it signals completion of it's heating cycle. Knowing that the "beeping" is high
pitched and the age of the individual, you suspect hearing loss in this patient that is typically associated
with aging. This type of hearing loss is related to which one of the following alterations in the ear?

Answer Choices:

A Fibrosis of the tympanic membrane

B Hypersecretion of cerumen in the external auditory meatus

C Ankylosis of the stapes at the oval window

D Loss of cochlear hair cells


E Loss of otoconia in the otolithic membrane

D Loss of cochlear hair cells

Hearing loss may be the result of one of two basic problems. Auditory disorders may be
related to either conductive disorders, or sensorineural disorders. Conductive disorders
are those that result from the mechanical impedance of sound waves from reaching the
auditory sensory receptors. Sensorineural disorders are those that result from the loss
of the ability to transduce or convey the mechanical signal into the neural signal.
Fibrosis of the tympanic membrane, excessive secretion of cerumen in the external
auditory meatus or ankylosis (bone deposition) of the stapes at the oval window are all
examples of conductive disorders leading to hearing loss.Furthermore, conductive
disorders such as these would result in a clinical situation with the loss of sound at all
frequencies, rather than only a high frequency or selected frequency. Loss of the
cochlear hair cells, particularly at the beginning of the basal turn of the cochlea,
typically result in the loss of high frequency sounds. This is due to a sensorineural
disorder which results in the loss of a specific frequency due to inability to transduce or
convey the mechanical signal to a neural signal. This selective hearing loss of high
frequency sounds, such as that of a beeping microwave oven, can be associated with
hearing disorders during the process of aging.Loss of neurons from the spiral ganglion
would be another example of a sensorineural disorder. The loss of otoconia in the
otolithic membrane would probably have little effect on auditory responses.

Question:

The cochlear duct is separated from the scala tympani by which one of the labeled structures in the
attached photomicroscopic image?

Answer Choices:

A A

B B

C C
D D

E E

Image(s) / Chart(s):
Click image to view full size. Click open image to close. Click and hold open image to move.

E E

The inner ear contains a conically shaped helix, the cochlea, within the bony labyrinth of
the temporal bone. The cochlear duct divides the cochlear canal into three
compartments. The endolymphatic fluid filled cochlear duct is the middle compartment,
called the scala media (#2), whereas the compartments of the cochlear canal filled with
perilymph include the scala vestibuli (#1), and the scala tympani (#3).

Reissner's membrane (a) separates the scala vestibuli from the scala media. A
specialized pseudostratified epithelium on the lateral wall of the scala media is the stria
vascularis (c), which may produce the endolymph. The specialized structure for auditory
sensation is a complex called the Organ of Corti, which rests on the floor of the scala
media called the basilar membrane (e).

The tectorial membrane (b) extends from the spiral limbus to lie over the hair cells of the
organ of corti, into which the kinocilia of the hair cells are imbedded. Movement of the
stereocilia in these hair cells initiates transduction of the mechanical motion into
membrane potentials conveyed to the nervous system by way of the vestibulocochlear
nerve.

The bipolar nerve cell bodies reside in the spiral ganglion (d).

Question:

The spiral ganglion is best identified as which one of the labeled structures in the attached
photomicroscopic image?

Answer Choices:

A A

B B

C C

D D

E E

Image(s) / Chart(s):
Click image to view full size. Click open image to close. Click and hold open image to move.
D D

The inner ear contains a conically shaped helix, the cochlea, within the bony labyrinth of
the temporal bone. The cochlear duct divides the cochlear canal into three
compartments. The endolymphatic fluid filled cochlear duct is the middle compartment,
called the scala media (#2), whereas the compartments of the cochlear canal filled with
perilymph include the scala vestibuli (#1), and the scala tympani (#3).

Reissner's membrane (a) separates the scala vestibuli from the scala media. A
specialized pseudostratified epithelium on the lateral wall of the scala media is the stria
vascularis (c), which may produce the endolymph. The specialized structure for auditory
sensation is a complex called the Organ of Corti, which rests on the floor of the scala
media called the basilar membrane (e).

The tectorial membrane (b) extends from the spiral limbus to lie over the hair cells of the
organ of corti, into which the kinocilia of the hair cells are imbedded. Movement of the
stereocilia in these hair cells initiates transduction of the mechanical motion into
membrane potentials conveyed to the nervous system by way of the vestibulocochlear
nerve.

The bipolar nerve cell bodies reside in the spiral ganglion (d).

Question:

At what location in the ear would the incus be found


Answer Choices:

A Vestibule of the inner ear

B Cochlea of the inner ear

C Middle ear cavity

D Eustachian tube

E External auditory meatus

C Middle ear cavity

The auditory ossicles include the malleus, the incus and the stapes. These bones cross
the middle ear cavity to connect the oval window with the tympanic membrane. The
sound waves that impinge on the tympanic membrane are converted to mechanical
vibrations and conveyed through the fluid-filled chambers of the inner ear. The vestibule
is a space within the inner ear containing membranous labyrinth structures, the utricle,
and saccule. The cochlea also is found within the inner ear, connected to the vestibule
anteriorly, and makes a 2 3/4 spiral turn towards its apex. The Eustachian (auditory)
tube connects the middle ear cavity with the nasopharynx. The ear canal or external
auditory meatus is a tubular connection continuous with the base of the auricle, leading
from the surface of the head to the tympanic membrane, or eardrum.

Question:

What is the common route by which infections may spread from the pharynx to the middle ear?

Answer Choices:
A External auditory meatus

B Auricle

C Cochlea

D Vestibule

E Eustachian tube

E Eustachian tube

The Eustachian (auditory) tube connects the middle ear cavity with the nasopharynx.
The walls of the pharyngeal portion of this somewhat tubular channel are normally
flattened together, and may be separated by yawning or swallowing. It is lined with
ciliated pseudostratified columnar epithelium, which is continuous with the nasopharynx.
Infections commonly spread from the pharynx into the middle ear by way of the
auditory tube. The oval-shaped portion of the ear that projects from the lateral surface
of the head that functions to collect and funnel sound waves is called the auricle, or
pinna. It is supported by an internal core of elastic cartilage and covered by thin skin.
Sweat glands, sebaceous glands, and hair follicles cover the surface.

The ear canal or external auditory meatus is a tubular connection continuous with the
base of the auricle, leading from the surface of the head to the tympanic membrane, or
eardrum. The vestibule is a space within the inner ear containing membranous labyrinth
structures, the utricle and saccule. The cochlea is also found within the inner ear
connected to the vestibule and makes a 2 3/4 helical spiral towards its apex. The
cochlear duct subserves audition.

Question:

Which of the following cells in the inner ear are involved in detecting movements of the head?
Answer Choices:

A Hair cells in the maculae

B Outer pillar cells

C Inner pillar cells

D Cells of Hensen

E Hair cells in the organ of Corti

A Hair cells in the maculae

Neuroepithelial hair cells in the maculae of the saccule and the utricle detect linear
movement of the head. These cells are connected to the vestibular portion of the
acoustic nerve. The outer and inner pillar cells form the tunnel of Corti. They move
laterally in response to the movement of ossicles, in turn causing a lateral shearing of
the stereocilia of the sensory hair cells of the organ of Corti against the tectorial
membrane. The cells of Hensen delineate the inner and outer borders of the spiral
organ of Corti. They, along with hair cells of the organ of Corti, function in the reception
of sound and can respond to different sound frequencies.

Question:

What structures are found in the vestibule of the inner ear

Answer Choices:

A Utricle & saccule


B Utricle & semicircular canals

C Three semicircular canals

D Saccule & semicircular canals

E Semicircular canals & cochlear duct

A Utricle & saccule

The utricle and saccule are membranous labyrinth structures contained within the
vestibule. The cochlea also is found within the inner ear, connected to the vestibule
anteriorly, and makes a 2 3/4 spiral turn towards its apex. The cochlear duct subserves
audition. Likewise, the semicircular canals are found within the inner ear, but not as part
of the vestibule since the project posteriorly from the vestibule. The oval (vestibular)
window is located in the lateral wall of the vestibule. The footplate of the stapes inserts
into this window.

Question:

Into which one of the following structures are the stereocilia of the Type I and Type II hair cells of the
utricle embedded?

Answer Choices:

A Cupula

B Otolithic membrane

C Organ of Corti
D Tectorial membrane

E Basilar membrane

B Otolithic membrane

The otolithic membrane is the gelatinous membrane overlying either the macula utricle
or macula saccule. The outer surface of this membrane contains particles of calcium
carbonate, called otolith or otoconia. The stereocilia of the hair cells are bent by the
action of gravity on the otoconia. The cupula also is a gelatinous structure attached to
hair cells of each crista of the semicircular canals. Movement of the endolymph
contained within the ducts sways the cupula which project into the endolymph and bend
the stereocilia to initiate a neural impulse.

The Organ of Corti is the structure within the cochlear duct specialized to transduce the
mechanical vibrations into neural impulses. The Organ of Corti rests upon the basilar
membrane and the stereocilia of the hair cells of the Organ of Corti are embedded
within the tectorial membrane.

Question:

Which one of the following structures connect the basilar membrane to the tectorial membrane?

Answer Choices:

A Outer hair cells

B Outer phalangeal cells

C Outer pilar cells

D Böttcher cells
E Claudius cells

A Outer hair cells

nly the stereocilia of the outer and inner hair cells, which are inserted into the tectorial
membrane connect the basilar membrane to the tectorial membrane. The outer and
inner phalangeal cells serve as supportive cells to the outer and inner hair cells,
respectively, and prevents the hair cells from directly touching the basilar membrane.
The pilar cells also are supportive cells forming the tunnel of Corti intermediate to the
outer and inner hair cells. Böttcher cells rest on the lateral aspect of the basilar
membrane and have a second row of cells, the Claudius cells, superficial to the
Böttcher cells. This stratified layer of cells serve to support the lateral aspect of the
Organ of Corti.

Question:

Which of the following structures are derived from the ventral component of the otic vesicle?

Answer Choices:

A Utricle and semicircular canals

B Saccule and cochlear duct

C Utricle and saccule

D Semicircular canals and cochlear duct

E Saccule, utricle, and endolymphatic duct


B Saccule and cochlear duct

The otic vesicle divides into ventral and dorsal components. The saccule and cochlear
duct are derived from the ventral portion, whereas the utricle, semicircular canals and
endolymphatic duct are derived from the dorsal component of the otic vesicle.

Question:

Which one of the following structures is considered to be a component of the membranous labyrinth?

Answer Choices:

A Semicircular canals

B Cochlea

C Vestibule

D Utricle

D Utricle

The inner ear consists of a bony labyrinth consisting of three connected spaces within
the temporal bone. The spaces are the semicircular canals, the vestibule and cochlea.
The vestibule is the central portion of the bony labyrinth containing the utricle and
saccule, parts of the membranous labyrinth. The semicircular ducts and cochlear duct
also are parts of the membranous labyrinth.

Question:
Communication between the scala vestibuli and scala tympani occurs at the

Answer Choices:

A Cochlear window

B Vestibular window

C Helicotrema

D Endolymphatic sac

E Endolymphatic duct

C Helicotrema

The scala vestibuli and the scala tympani are, in reality, one perilymphatic space
separated by the cochlear duct (scala media). The scala vestibuli and tympani
communicate with each other at the helicotrema. The round (cochlear) window connects
middle ear to scala tympani. The oval (vestibular) window connects with the stapes. The
helicotrema connects the scala vestibuli to the scala tympani. The endolymphatic duct
begins at utricle and saccule and ends in the expanded endolymphatic sac.

Question:

Which one of the following vessels supplies the basal one-fourth of the cochlea?

Answer Choices:
A Posterior vestibular artery

B Cochlear branch of the vestibulocochlear artery

C Main cochlear artery

D Cochlear branch of the anterior vestibular artery

E Vestibulocochlear artery

B Cochlear branch of the vestibulocochlear artery

The vestibulocochlear artery branches into a posterior vestibular artery and a cochlear
branch which supplies the basal one-fourth of the cochlea. The main cochlear artery
supplies the remaining portion of the cochlea. The posterior vestibular artery supplies
the macula of the utricle and the crista ampullaris of the posterior semicircular duct. The
anterior vestibular artery supplies the remaining cristae of the semicircular ducts and the
macula of the utricle.

Question:

Which one of the following vessels supplies three-fourths of the cochlea?

Answer Choices:

A Posterior vestibular artery

B Cochlear branch of the vestibulocochlear artery

C Main cochlear artery


D Anterior vestibular artery

E Vestibulocochlear artery

C Main cochlear artery

The vestibulocochlear artery branches into a posterior vestibular artery and a cochlear
branch which supplies the basal one-fourth of the cochlea. The main cochlear artery
supplies the remaining portion of the cochlea. The posterior vestibular artery supplies
the macula of the utricle and the crista ampullaris of the posterior semicircular duct. The
anterior vestibular artery supplies the remaining cristae of the semicircular ducts and the
macula of the utricle.

Question:

The inner ear contains 2 labyrinths. Which of the following is part of the bony labyrinth?

Answer Choices:

A Utricle

B Saccule

C Endolymphatic sac

D Vestibule

E Membranous ampulla
D Vestibule

The vestibule is an oval-shaped structure that is one of 3 cavities comprising the bony
labyrinth. The vestibule contains part of the membranous labyrinth. The other 2 bony
cavities include the semicircular canals and the cochlea. The bony labyrinth is lined with
periosteum and also contains perilymph, a fluid similar in makeup to spinal fluid. This
fluid surrounds the membranous labyrinth.

The utricle is part of the membranous labyrinth. It is a fluid-filled sac which lies within
the bony vestibule and is linked to the semicircular ducts by the ampullae. The utricle is
also connected to the saccule.

The saccule is part of the membranous labyrinth. It is located in the bony vestibule of
the inner ear. The saccule is a crescent-shaped membranous sac which is connected to
the utricle via the utriculo-saccular duct. The other end of the saccule is connected to
the cochlear duct.

The endolymphatic sac is part of the membranous labyrinth. It connects to the utriculo-
saccular duct on one end and on the other, it dead-ends in a pouch lying in the dura of
the posterior fossa of the skull.

The membranous ampulla is situated at the base of each semicircular duct (in the
membranous labyrinth). Within each ampulla is a large projection called the ampullary
crest. This projection contains receptors that are responsible for maintaining equilibrium.

You might also like